Всі тести з Крок 1 - Стоматологія
1 / 4299
Досліджуються клітини червоного кісткового мозку людини, які відносяться до клітинного комплексу, що постійно поновлюється. Яким чином у нормі утворюються ці клітини?

Шизогонія

Бінарний поділ

Мітоз

Амітоз

Мейоз

2 / 4299
У здорового подружжя народилась дитина з розщілинами губи та піднебіння, аномаліями великих пальців кисті та мікроцефалією. Каріотип дитини: 47,18+. Який тип мутації спричинив цю спадкову хворобу?

Нулісомія

Моносомія за аутосомою

Моносомія за Х-хромосомою

Поліплоїдія

Трисомія за аутосомою

3 / 4299
У жінки, яка під час вагітності вживала алкогольні напої, народилася глуха дитина із розщілинами верхньої губи і піднебіння. Ці ознаки нагадують прояв деяких хромосомних аномалій. Який процес призвів до таких наслідків?

Канцерогенез

Філогенез

Мутагенез

Тератогенез

Онтогенез

4 / 4299
У чоловіка, його сина та дочки відсутні малі корінні зуби. Така аномалія спостерігалася також у дідуся по батьківській лінії. Який найбільш імовірний тип успадкування цієї аномалії?

Аутосомно-рецесивний

Рецесивний, зчеплений з Х-хромосомою

Домінантний, зчеплений з Х-хромосомою

Зчеплений з Y-хромосомою

Аутосомно-домінантний

5 / 4299
Під час мікроскопії харкотиння хворого на пневмонію випадково виявлені личинки. У крові - еозинофілія. Який гельмінтоз найбільш імовірно наявний у хворого?

Аскаридоз

Трихоцефальоз

Парагонімоз

Опісторхоз

Ентеробіоз

6 / 4299
До лікаря звернувся юнак 16 років зі скаргами на свербіння між пальцями рук і на животі, яке посилюється вночі. Під час огляду на шкірі виявлені тоненькі смужки сірого кольору і дрібнокрапковий висип. Який збудник найбільш імовірно спричинив це захворювання?

Ixodes ricinus

Ixodes persulcatus

Ornithidorus papillipes

Sarcoptes scabiei

Dermacentor pictus

7 / 4299
У 50-річної жінки на місці видаленого зуба регенерувала нова тканина. Які органели клітин, виходячи з їх функції, найбільш активні при відновленні тканини?

Рибосоми

Гладка ЕПС

Центросоми

Лізосоми

Постлізосоми

8 / 4299
У пацієнта спостерігаються лихоманка і поява везикул, розташованих на межі шкіри і слизових оболонок. За допомогою яких методів дослідження можна підтвердити наявність в організмі хворого вірусу простого герпесу?

Мікроскопічне і біологічне

Вірусологічне і біологічне

Вірусологічне і серологічне

Мікроскопічне і алергічне

Серологічне і біологічне

9 / 4299
Через певний час після огляду вимушено забитої корови у ветеринарного лікаря на щоці з’явився карбункул чорного кольору. Під час мікроскопічного дослідження його вмісту виявлені грампозитивні, великі, розташовані ланцюжками палички з обрубаними кінцями, що нагадують бамбукову палицю. Для якого збудника характерні вказані морфологічні та тинкторіальні властивості?

Francisella tularensis

Proteus vulgaris

Clostridium perfringens

Bacillus anthracis

Yersinia pestis

10 / 4299
При очікуванні екстракції зуба у пацієнта виникло сильне відчуття боязні. Який лікарський препарат йому слід ввести для усунення цих відчуттів?

Діазепам

Аміназин

Етимізол

Карбамазепін

Анальгін

11 / 4299
Після операції у хворого знижена чутливість шкіри передньої та латеральної поверхні шиї. Який нерв було пошкоджено під час операції?

N. auricularis magnus

N. transversus colli

N. phrenicus

N. occipitalis minos

Nn. supraclaviculares

12 / 4299
Хворий 56 років страждає на фіброзно-кавернозний туберкульоз легень. Протягом останніх 3 тижнів посилилися кашель, слабкість, збільшилася кількість гнійно-слизового харкотиння з прожилками крові. Яка причина вентиляційної недостатності, що виникла в даному випадку?

Порушення функції нервово-м’язового апарату

Зменшення кількості функціонуючих альвеол

Порушення прохідності повітроно-сних шляхів

Порушення функції дихального центру

Порушення рухливості грудної клітки

13 / 4299
У хворого зниження хвилинного викиду серця зумовлене зниженням іно-тропної функції міокарда. Яка з перелічених речовин підвищує цю функцію в результаті прямої дії на міокард?

Строфантин

Фуросемід

Анаприлін

Дроперидол

Еналаприл

14 / 4299
Який нейромедіатор у тканині мозку може бути синтезований з продукту переамінування альфа-кетоглутарової кислоти?

Серотонін

Триптамін

Норадреналін

Дофамін

ГАМК

15 / 4299
Серед студентів однієї групи присутні представники різних рас. Один з студентів має пряме чорне волосся та нависаючу шкірну складку верхньої повіки - епікантус. Представником якої раси найвірогідніше є цей студент?

Монголоїдна

Негроїдна

Австралоїдна

Ефіопська

Європеоїдна

16 / 4299
В результаті інтоксикації в епітеліальній клітині слизової оболонки порожнини рота не синтезуються ферменти, що забезпечують сплайсинг. Яка причина припинення біосинтезу білку у цьому випадку?

Не активуються амінокислоти

Не утворюється р-РНК

Не утворюється зріла і-РНК

Порушено транспорт амінокислот

Не синтезується АТФ

17 / 4299
У хворої 43 років, яка була прооперована з приводу 'гострого живота', сеча набула коричневого кольору, кількість індикану в крові різко підвищилася. Про що може свідчити цей показник?

Посилення гниття білків в кишечнику

Посилення дезамінування амінокислот

Зниження швидкості клубочкової фільтрації у нирках

Інгібування глюконеогенезу

Зниження інтенсивності орнітиново-го циклу

18 / 4299
Під час розтину трупа новонародженого хлопчика виявлено: полідактилія, мікроцефалія, незарощення верхньої губи та верхнього піднебіння, а також гіпертрофія паренхіматозних органів. Сукупність вказаних вад відповідає синдрому Патау. Яке порушення у генетичному апараті є причиною даної патології?

Трисомія 18-ої хромосоми

Нерозходження статевих хромосом

Трисомія 21-ої хромосоми

Трисомія 13-ої хромосоми

Часткова моносомія

19 / 4299
До гастроентерологічного відділення надійшов хворий із запаленням жовчних шляхів. У порціях жовчі виявлені рухомі найпростіші грушоподібної форми, двоядерні, з опорним стрижнем-аксостилем. Яке протозойне захворювання діагностується у хворо- го?

Трихомоноз

Балантидіаз кишковий

Лямбліоз

Амебна дизєнтєрія

Амебіаз кишковий

20 / 4299
У результаті травми у хворого був ушкоджений відвідний нерв. Який з пе-рєлічєних симптомів спостерігатиметься у такого хворого?

Розлад акомодації

Стійке розширення зіниці

Параліч латерального прямого м’яза очного яблука

Порушення сльозовиділення

Параліч медіального прямого м’яза очного яблука

21 / 4299
До травмпункту надійшов хворий, який не може розігнути руку в ліктьовому суглобі. Який м’яз ушкоджено?

Двоголовий м’яз плеча

Триголовий м’яз плеча

Плечо-променевий м’яз

Плечовий м’яз

Дзьобо-плечовий м’яз

22 / 4299
У хворого на хронічне запалення субмаксилярної слинної залози спостерігається гіпосалівація. Порушення інкреції якої речовини спостерігається при цьому?

Паротин

Глюкагон

Паратирін

Кальцитонін

Соматостатин

23 / 4299
Хворий 43 років звернувся до лікаря зі скаргою на пухлиноподібне утворення на корені язика. Хірург-стоматолог встановив, що це злоякісна пухлина. Плануючи операцію, він вирішив перев’язати артерію, яка проходить у трикутнику Пирогова. Що це за артерія?

А. lrnguahs

А. profunda lrnguae

А. palatina ascendens

А. palatina desсendens

А. sublrnguahs

24 / 4299
У медико-генетичній консультації 14-річній дівчинці встановлено діагноз: синдром Шерешевського - Тернера. Який каріотип дитини?

45, Х0

47, ХХУ

46, ХУ

46, ХХ

47, трисомія за 13-ою парою

25 / 4299
На практичному занятті студенти вивчали забарвлений мазок крові миші, у якому було виявлено фагоцито-вані лейкоцитами бактерії. Які органе-ли клітини завершують перетравлення цих бактерій?

Рибосоми

Гранулярна ендоплазматична сітка

Лізосоми

Мітохондрії

Апарат Гольджі

26 / 4299
До хірурга-стоматолога потрапив чоловік 28 років з вуличною травмою. Лікар виявив ушкодження луски скроневої кістки та субдуральну гематому, яка виникла внаслідок цього. Ушкодження якої артерії найбільш імовірно спричинило утворення гематоми?

A. ophthalmia

A. sphenopalatina

A. maxtilaris

A. carotis rnterna

A. menrngea media

27 / 4299
Хворий надійшов до відділення щелепно-лицьової хірургії з вивихом скроневонижньощелепного суглоба з пошкодженням основної його зв’язки. Назвіть цю зв’язку.

Крило-нижньощелепна

Нижньощелепна

Латеральна

Шило-нижньощелепна

Медіальна

28 / 4299
При артритах спостерігається порушення ковзання суглобових поверхонь. Яка тканина ушкоджується?

Колагеново-волокниста хрящова

Кісткова

Гіалінова хрящова

Пухка сполучна

Ретикулярна

29 / 4299
Під час рентгеноскопічного дослідження органів грудної клітки у хворого відзначена нерухомість діафрагми з правого боку. Який нерв пошкоджено з правого боку?

Блукаючий нерв

Діафрагмальний нерв

Додатковий нерв

Симпатичний стовбур

Шийна петля

30 / 4299
Гнійні вогнища на о6личчі вище ротової порожнини небезпечні можливістю поширення інфекції у порожнину черепа. Наявністю яких анастомозів вен обличчя це зумовлено?

З передніми вушними венами

З очними венами

З середньою і глибокими скроневими венами

З потиличною веною

З крилоподібним сплетенням

31 / 4299
Відомо, що накопичення аміаку є основною причиною мозкової коми при печінковій недостатності. Яка вільна амінокислота відіграє першочергову роль в утилізації аміаку в мозку?

Цистеїн

Глютамінова кислота

Аланін

Гістидин

Триптофан

32 / 4299
Хворій потрібно зробити пункцію прямокишково-маткової заглибини (Дугласового простору). Через яку частину склепіння піхви її здійснюють?

Бічна права

Будь-яка

Задня

Передня

Бічна ліва

33 / 4299
До хірурга-стоматолога привели хворого з вуличною травмою в ділянці піднижньощелепного трикутника. Обробляючи рану, лікар побачив, що травмована артерія, яка іде до м’якого піднебіння. Яку артерію пошкоджено?

A. sphenopalatina

A. pharingea ascendens

A. palatina descendens

A. fatiatis

A. palatina ascendens

34 / 4299
Хворий на застудне захворюванням страждає від безперервного кашлю з харкотинням, що погано відділяється. Який з перелічених препаратів поліпшить відділення харкотиння?

Фалімінт

Лібексин

Кодеїну фосфат

Настій трави термопсису

Ілауцину гідрохлорид

35 / 4299
У постраждалого у ДТП лівобічний перелом середньої третини нижньої щелепи. Які судини можуть бути травмовані?

Щелепно-під’язикові артерія і вена

Язикові артерія і вена

Верхньощелепна артерія і крилоподібне сплетення

Нижні альвеолярні артерія і вена

Верхні альвеолярні артерія і вена

36 / 4299
На розтині тіла померлого від ниркової недостатності, який протягом останніх 5 років хворів на бронхое-ктатичну хворобу, виявлені збільшені у розмірах нирки щільної консистенції з потовщеним кірковим шаром білого кольору та сальним блиском. Яке захворювання нирок у хворого?

Гломерулонефрит

Вторинний амілоїдоз

Некротичний нефроз

Хронічний пієлонефрит

37 / 4299
У новонародженої дитини під час смоктання молоко потрапляє до носової порожнини. Про яку ваду розвитку свідчить цей факт?

Заяча губа

Звуження стравоходу

Незарощення піднебіння

Атрезія стравоходу

Дивертикул стравоходу

38 / 4299
У 37-річної хворої в аналізі крові: Нв - 60 г/л, ер. - 3,0 * 1012/л, КП - 0,6. Лейкоцитарна формула без змін, тромбоцити - 200 * 109/л, ретикулоцити - 0,3 %, ШОЕ - 18 мм/год. Мікроцитоз, пой-кілоцитоз. Вкажіть найбільш імовірний тип анемії за механізмом розвитку:

В12-фолієводефіцитна

Залізодефіцитна

Гіпопластична

Гостра постгеморагічна

Гемолітична

39 / 4299
Під час обстеження пацієнта ЛОР-лікар діагностував запалення верхньощелепних пазух. У якому носовому ході під час риноскопії було виявлено гній?

Верхній

Середній

Найвищий

Нижній

Загальний

40 / 4299
До хірургічного відділення доставлено чоловіка 35 років з гнійною раною на шиї попереду трахеї (в ділянці передвісцерального простору). Куди може поширитися інфекція, якщо хворому терміново не зробити операцію?

У ретровісцеральний простір

У надгрудинний міжапоневротичний простір

У переднє середостіння

У заднє середостіння

У середнє середостіння

41 / 4299
Внаслідок інсульту (крововилив у головний мозок) у хворого відсутні активні рухи м’язів голови і шиї. Обстеження головного мозку за допомогою ЯМР показало, що гематома знаходиться в коліні внутрішньої капсули. Який провідний шлях пошкоджено у хворого?

Tr.cortico-spinalis.

Tr.thalamo-corticalis.

Tr.cortico-nuclearis.

Тr.cortico-thalamicus.

Tr.cortico-fronto-pontinus.

42 / 4299
До лікаря звернувся пацієнт зі скаргами на підвищену больову чутливість шкіри вушної раковини та зовнішнього слухового проходу. Пальпація позаду грудинно-ключичнососковидного м’язу болюча. Подразнення якого нерва може дати таку клінічну картину?

Nn. supraclaviculares

N. auricularis magnus

N. transversus colli

N. vagus

N. occipitalis minor

43 / 4299
Чоловік 43 років скаржиться на кровоточивість ясен, рухомість нижніх фронтальних зубів. Під час огляду виявлено: гіперемія і набряк слизової оболонки ясни нижньої щелепи. Різці та ікла нижньої щелепи рухомі, шийки цих зубів оголені. При натискуванні пінцетом з-під слизової ясни виділяється гній. Для якого захворювання найбільш характерні наведені симптоми?

Парадонтит

Остеомієліт

Періодонтит

Стоматит

Гінгівіт

44 / 4299
Позазародковий орган на ранніх етапах ембріогенезу має одношаровий плоский епітелій, який з третього місяця набуває призматичної і кубічної форми, бере участь у виробленні навколоплідних вод. Назвіть цей орган:

Жовтковий мішок

Плацента

Алантоїс

Амніон

Пуповина

45 / 4299
Хворому з порушенням функції зовнішнього дихання необхідно зробити трахеостомію. При цьому слід пам’ятати про розташування перешийку щитоподібної залози. На рівні яких хрящових кілець трахеї він знаходиться частіше за все?

IV-V

II- IV

I-II

V-VI

III-IV

46 / 4299
У здорової людини визначають абсолютні пороги смакової чутливості до різних речовин. До якої з наведених речовин поріг буде найменшим?

Лимонна кислота

Хінін

Глюкоза

Хлорид натрію

47 / 4299
Перед екстракцією зуба хворому місцево ввели знеболюючий препарат із групи заміщених амідів (похідних анілі-ду). Визначте цей препарат.

Новокаїн

Танін

Лідокаїн

Дикаїн

Анестезин

48 / 4299
Зуб, видалений у пацієнта, має два корені. Що це за зуб?

Верхній різець

!кло

Верхній великий кутній

Нижній малий кутній

Нижній великий кутній

49 / 4299
При обстеженні пацієнта 15 років виявлено, що після перенесеної щелепно-лицевої травми він не може опустити нижню щелепу. Пошкодження якого м’язу може бути причиною вказаного порушення?

Латеральний крилоподібний

Жувальний

Скроневий

Медіальний крилоподібний

Підборідно-під’язиковий

50 / 4299
У хворого запалення середнього вуха (отит). При цьому він скаржиться на розлад відчуття смаку в передній частині язика. Який з нервів ушкоджено?

N. vagus

N. vestibulo-cochlearis

N. fatiaUs

N. trigemmus

N. glossopharyngeus

51 / 4299
До лікаря звернулися декілька пацієнтів з подібними скаргами: слабкість, болі в животі, діарея. Після дослідження фекалій виявилось, що терміновій госпіталізації підлягає один з пацієнтів, у якого були виявлені цисти з чотирма ядрами. Для якого найпростішого характерні такі цисти?

Балантидія

Трихомонада

Дизентерійна амеба

Лямблія

Кишкова амеба

52 / 4299
Пролонгована дія ряду антибіотиків та сульфаніламідів зумовлена тим, що вони циркулюють у крові тривалий час у комплексі з:

Іаптоглобіном

Гемоглобіном

Іемопексином

Альбуміном

Трансферином

53 / 4299
Після вживання м’ясних консервів у хворого з’явилося двоіння в очах, сильний головний біль, порушення ковтання, ускладнення дихання, м’язова слабкість. Встановлено діагноз: ботулізм. З яким фактором патогенності пов’язані клінічні прояви цього захворювання?

Іемолізин

Екзотоксин

Ендотоксин

Фібринолізин

Плазмокоагулаза

54 / 4299
У хворого утруднене щільне змикання щелеп при жуванні. Відзначається часткова атрофія жувальних м’язів, що розташовані нижче скулової дуги. Гілки якого нерва інервують вказані м’язи?

N. infraorbitalis

N. maxUlaris

Nn. alveolares superiores

N. alveolaris inferior

N. mandtoularis

55 / 4299
У робітника, який працював улітку в щільному костюмі, різко підвищилася температура тіла, з’явилися задишка, тахікардія, нудота, судоми, непритомність. Що найбільш імовірно стало причиною розвитку згаданих симптомів?

Підвищення тепловіддачі

Зниження теплопродукції

Рівність тепловіддачі та теплопродукції

Підвищення теплопродукції

Зниження тепловіддачі

56 / 4299
У гістологічному препараті виявляються судини, що починаються сліпо, мають вигляд сплющених ендотеліаль-них трубок, не містять базальної мембрани і перицитів, ендотелій цих судин фіксований стропними філаментами до колагенових волокон сполучної тканини. Які це судини?

Артеріоли

Артеріо-венозні анастомози

Іемокапіляри

Лімфокапіляри

Венули

57 / 4299
При онкологічному захворюванні одного з органів грудної порожнини пухлина метастазує дуже швидко, оскільки лімфа від цього органу частіше за все відтікає одразу до грудної лімфатичної протоки. Який це орган?

Трахея

Серце

Вилочкова залоза

Стравохід

Легеня

58 / 4299
У хворої жінкі 47 років при обстеженні виявлено рефлюкс (зворотній закид сечі). У якій частині нирки розташовані гладенькі м’язи, порушення функції яких може бути причиною ре-флюкса?

Capsula glomeruli

Ductuli papillares

Pelvis renalis

Calyx renalis major

Calyx renalis minor

59 / 4299
У гістологічному препараті судини добре виражені внутрішня та зовнішня еластичні мембрани, у середній оболонці багато міоцитів. До якого типу відноситься ця судина?

Артерія еластичного типу

Артерія м’язового типу

Вена з сильним розвитком м’язів

Екстраорганна лімфатична судина

Артерія змішаного типу

60 / 4299
Внаслідок перенесеної хвороби у хворого порушено розширення голосової щілини. Який м’яз пошкоджено?

Голосовий

Задній персне-черпакуватий

Щито-черпакуватий

Персне-щитоподібний

Бічний персне-черпакуватий

61 / 4299
До лікарні надійшов хворий з пошкодженням щоки. Протока якої слинної залози може бути ушкоджена?

Кутня

Привушна

Під’язикова

Піднижньощелепна

Щічна

62 / 4299
У дітей, хворих на квашіоркор, поряд з іншими ознаками виявлені порушення процесу утворення зубів. В основі цього явища лежить недостатнє надходження до організму:

Вітаміну В1

Жирів

Вуглеводів

Вітаміну С

Білків

63 / 4299
Хворому проведена субтотальна субфасціальна резекція щитоподібної залози. У післяопераційному періоді тривалий час зберігається охриплість голосу. Який нерв ушкоджено в ході операції?

Верхній гортанний

Зворотний гортанний

Під’язиковий

Язиковий

Нижньощелепний

64 / 4299
У гістологічному препараті видно м’язове утворення ротової порожнини, яке складається з двох відділів: шкірного і слизового, у якому розрізняють верхню, проміжну і нижню зони. У проміжній зоні слизового відділу епітелій багатошаровий плоский зроговілий. Яке утворення представлено у препараті?

Ясна

Щока

Тверде піднебіння

Язик

Губа

65 / 4299
Тривалий вплив на організм токсичних речовин призвів до значного зниження синтезу білків у гепатоцитах. Які органели постраждали від інтоксикації найбільше?

Лізосоми

Мітохондрії

Гранулярна ендоплазматична сітка

Комплекс Гольджі

Мікротрубочки

66 / 4299
У хворого після психічної травми відзначаються підвищена збудливість, дратівливість, безсоння, тривога. Який лікарський засіб доцільно призначити?

Імізин

Бемегрид

Кофеїн

Феназепам

Пірацетам

67 / 4299
У хворого на цукровий діабет після ін’єкції інсуліну спостерігалися непритомність, судоми. Який результат біохімічного аналізу крові на вміст глюкози найбільш імовірний?

3,3 ммоль/л

2,5 ммоль/л

10 ммоль/л

5,5 ммоль/л

8,0 ммоль/л

68 / 4299
Лікар-бактеріолог виділив у хворої дитини збудника дизентерії Флекснера - тип 2, Зонне - тип І та етеропатогенну кишкову паличку - О55/В5. Як зветься тип інфекції у цієї дитини?

Змішана інфекція

Суперінфекція

Вторинна інфекція

Реінфекція

Носійство патогенних бактерій

69 / 4299
В результаті травми голови у чоловіка 32 роки ушкодженні ампули півко-лових каналів. Сприйняття яких подразнень буде порушено внаслідок цього?

Кутове прискорення

Гравітація

Вібрація та гравітація

Лінійне прискорення

Вібрація

70 / 4299
У хворого на хронічний ентероколіт виявлено порушення травлення та всмоктування білків у тонкій кишці внаслідок недостатньої кількості дипептидаз у кишковому соці. У яких клітинах порушено синтез цих ферментів?

Ендокриноцити

Стовпчасті з облямівкою

Стовпчасті без облямівки

Клітини Панета

Келихоподібні

71 / 4299
Під час розтину тіла жінки 38 років виявлені нирки розміром 18хі6х8 см, що нагадують виноградні грона, на розрізі складаються з численних порожнин від 0,5 см до 3 см у діаметрі, заповнених серозною рідиною або колоїдними масами. Паренхіма нирки між порожнинами різко стоншена до 0,1 см. На яке захворювання страждала померла?

Полікістоз нирок

Дисплазія нирок

Нефролітіаз

Гострий пієлонефрит

Хронічний пієлонефрит

72 / 4299
Після перенесеного хімічного опіку стравоходу відбулося локальне його звуження внаслідок утворення рубця. Які клітини пухкої сполучної тканини беруть участь в утворенні рубців?

Юні малоспеціалізовані фібробласти

Фіброкласти

Фіброцити

Міофібробласти

Зрілі спеціалізовані фібробласти

73 / 4299
У жінки має місце гіперемія яєчника, підвищення проникності гемато-фолікулярного бар’єру з послідовним розвитком набряку, інфільтрація стінки фолікула сегментоядерними лейкоци- тами. Об’єм фолікула великий, стінка його потоншена. Якому періоду статевого циклу відповідає описана картина?

Період відносного спокою

Постменструальний період

Предовуляторна стадія

Менструальний період

Овуляція

74 / 4299
З ектодермального епітелію вистил-ки верхньої частини ротової ямки зародка людини формується кишеня Ра-тке, яка направляється до основи майбутнього головного мозку. Що розвивається з даного ембріонального зачатка?

Медіальне підвищення

Нейрогіпофіз

Передній гіпоталамус

Аденогіпофіз

Гіпофізарна ніжка

75 / 4299
У приміщенні відзначається підвищений вміст вуглекислого газу. Як зміниться дихання (глибина і частота) у людини, яка увійшла до цього приміщення?

Зменшиться глибина

Збільшиться глибина і частота

Збільшиться частота

Збільшиться глибина

Зменшиться частота

76 / 4299
У хлопчика перелом плечової кістки. За рахунок якої структури відбуватиметься репаративна регенерація кістки?

Шар зовнішніх генеральних пластинок

Шар внутрішніх генеральних пластинок

Діафіз

Епіфіз

Окістя

77 / 4299
У новонародженої дитини вивих кришталика, довгі й тонкі кінцівки з дуже довгими і тонкими пальцями, аневризма аорти, виділення із сечею окремих амінокислот. Для якого захворювання характерні дані ознаки?

Галактоземія

Фенілкетонурія

Гіпофосфатемія

Фруктозурія

Синдром Марфана

78 / 4299
До лікарні надійшов пацієнт у непритомному стані. Шкіра холодна, зіниці звужену дихання за типом Чейн-Стокса, артеріальний тиск знижений, сечовий міхур переповнений. Попередній діагноз: отруєння наркотиками. Який препарат необхідно застосувати як антагоніст?

Налоксон

Бемегрид

Тіосульфат натрію

Цититон

Унітіол

79 / 4299
Під час мікроскопічного дослідження у препараті виявлений орган нервової системи, що складається з псевдоу-ніполярних нейронів, тіла яких вкриті гліальною і сполучнотканинною оболонками. Визначте цей орган.

Мозочок

Кора великих півкуль

Спинний мозок

Вегетативний ганглій

Спинномозковий вузол

80 / 4299
Під час гістологічного дослідження біоптату шкіри людини виявляється лише щільна неоформлена сполучна тканина. Який шар даного органу було представлено для вивчення?

М’язова пластинка

Сосочковий шар дерми

Гіподерма

Епідерміс

Сітчастий шар дерми

81 / 4299
В експерименті тварині нанесена травма рогівки. За рахунок чого буде відбуватися регенерація її багатошарового епітелію?

Клітини базального шару епітелію

Базальна мембрана

Плоскі клітини

Остистий шар епітелію рогівки

Власна речовина рогівки

82 / 4299
На електронній мікрофотографії представлені структури у вигляді відкритих міхурців, внутрішня поверхня яких вистелена одношаровим епітелієм, який утворений респіраторними та секреторними клітинами. Які структури представлені на мікрофотографії?

Альвеоли

Альвеолярні ходи

Термінальні бронхи

Бронхіоли

Ацинуси

83 / 4299
Під час емоційного збудження частота серцевих скорочень (ЧСС) у людини 30 років досягла 112 за хвилину. Зміна стану якої структури провідної системи серця є причиною збільшення ЧСС?

Атріовентрикулярний вузол

Волокна Пуркін’є

Пучок Гіса

Ніжки пучка Гіса

Синоатріальний вузол

84 / 4299
У новонародженої дитини недорозвинений тимус. Який вид гемопоезу буде порушений?

Лімфопоез

Мегакаріоцитопоез

Моноцитопоез

Гранулоцитопоез

Еритропоез

85 / 4299
В ембріогенезі ротової порожнини відбулося порушення розвитку емалі зубів. Яке джерело розвитку зубів було пошкоджено?

Зубний мішечок

Мезодерма

Епітелій

Зубний сосочок

Мезенхіма

86 / 4299
У мазку крові людини, що страждає на алергію, можна бачити велику кількість клітин округлої форми з сегмен-тованим ядром і великими яскраво - рожевими гранулами в цитоплазмі. Які це клітини?

Еозинофільні гранулоцити

Лімфоцити

Базофільні гранулоцити

Еритроцити

Нейтрофільні гранулоцити

87 / 4299
На електронній мікрофотографії фрагменту нирки представлена прино-сна артеріола, у якій під ендотелієм видно великі клітини, що містять секреторні гранули. Назвіть цей вид клітин.

Мезангіальні

Іладеньком’язеві

Юкставаскулярні

Юкстагломерулярні

Інтерстиціальні

88 / 4299
У хворого з алкогольним цирозом печінки скарги на загальну слабкість, задишку. Встановлено зниження артеріального тиску, асцит, розширення поверхневих вен передньої стінки живота, спленомегалія. Яке порушення гемоди-наміки спостерігається у хворого?

Синдром портальної гіпертензії

Тотальна серцева недостатність

Недостатність лівого шлуночка серця

Недостатність правого шлуночка серця

Колапс

89 / 4299
Під час кружіння на каруселі у жінки 25 років з’явилися нудота, блювання, посилення потовиділення. Активація яких рецепторів зумовила рефлекторний розвиток цих симптомів?

Кортієвого органу

Пропріорецептори скелетних м’язів

Зорові

Вестибулярні півколових каналів

Вестибулярні отолітові

90 / 4299
У людей похилого віку швидкість поширення пульсової хвилі вища, ніж у молодих. Причиною цього є зменшення з віком:

Лінійної швидкості кровотоку

Частоти серцевих скорочень

Об’ ємної швидкості кровотоку

Величини серцевого викиду

Еластичності судинної стінки

91 / 4299
У людини через 10 хвилин після початку інтенсивної фізичної роботи кількість еритроцитів у крові збільшилася з 4,0 * 1012/л до 4,5 * 1012/л. Що є основною причою цього?

Пригнічення руйнування еритроцитів

Втрата води організмом

Активація еритропоезу

Вихід еритроцитів з депо

Збільшення хвилинного об’єму крові

92 / 4299
У дитини спостерігається затримка росту, психічного розвитку і формування зубів, запізніла поява точок окості-ніння, зниження основного обміну. Гіпофункція якої ендокринної залози (залоз) є причиною такого стану?

Наднирники

Щитовидна

Нейрогіпофіз

Підшлункова

Статеві

93 / 4299
Піддослідній тварині через зонд у порожнину шлунку ввели 150 мл м’ясного бульйону. Вміст якої речовини швидко збільшиться у крові?

Соматостатин

Глюкагон

Гастрин

Нейротензин

Інсулін

94 / 4299
У хворого, який протягом тривалого часу приймав глюкокортикоїди, в результаті відміни препарату виникло загострення хвороби, зниження артеріального тиску, слабкість. З чим можна пов’язати ці явища?

Звикання до препарату

Кумуляція

Недостатність кори наднирників

Сенсибілізація

Гіперпродукція кортикотропного гормону

95 / 4299
У хворої з набряками у сечі велика кількість білку. Про порушення функції якого відділу нефрону це свідчить?

Нисхідна частина петлі Іенле

Проксимальний звитий каналець

Ниркове тільце

Дистальний звитий каналець

Висхідна частина петлі Іенле

96 / 4299
Під час обстеження пацієнта виникла необхідність визначити силу жувальних м’язів. Який метод дослідження треба використати?

Електроодонтодіагностика

Гнатодинамометрія

Мастікаціографія

Електроміографія

Міоартрографія

97 / 4299
Студент перед іспитом скаржився на гострий зубний біль, під час складання іспиту біль послабився. Розвитком якого виду гальмування у корі головного мозку зумовлене зменшення больових відчуттів?

Згасаюче

Запізніле

Зовнішнє

Позамежне

Диференціювальне

98 / 4299
При аналiзi електрокардіограми встановлено, що тривалість серцевого циклу у людини дорівнює 1 сек. Якою у неї є частота серцевих скорочень за хвилину?

50

60

100

80

70

99 / 4299
У людини внаслідок хронічного захворювання пєчінки суттєво порушена її білковосинтезуюча функція. До зменшення якого параметру гомеостазу це призведе?

Гематокрит

Щільність крові

Онкотичний тиск плазми крові

Осмотичний тиск

рН

100 / 4299
Експериментальній тварини перерізали передні корінці п’яти сегментів спинного мозку. Які зміни відбудуться у зоні інервації?

Втрата рухів

Втрата дотикової чутливості

Втрата температурної чутливості

Втрата пропріоцептивної чутливості

Гіперчутливість

101 / 4299
У пацієнта порушена координація рухів, їх амплітуда і спрямованість; рухи розмашисті, непропорційні; хода 'півняча', 'п’яна'. Який відділ мозку пошкоджено?

Спинний мозок

Гіпоталамус

Таламус

Мозочок

Довгастий мозок

102 / 4299
В експерименті на ізольованій збудливій клітині необхідно отримати збільшення мембранного потенціалу спокою (гіперполяризацію). Які іонні канали слід активувати для цього?

Калієві та натрієві

Калієві

Натрієві та кальцієві

Натрієві

Кальцієві

103 / 4299
Студентка 18 років має масу тіла 50 кг. Робочий (загальний) обмін її складає 11 000 кДж/д. Якою повинна бути калорійність харчового раціону студентки, якщо вона хоче збільшити масу тіла?

8 000 - 9 000 кДж/д

9 000 - 10 000 кДж/д

10 500 - 11 500 кДж/д

10 000 - 11 000 кДж/д

12 000 - 13 000 кДж/д

104 / 4299
У хворого крововилив у задню центральну звивину. До порушення якого виду чутливості з протилежного боку це призведе?

Слухова і зорова

Слухова

Зорова

Нюхова і смакова

Шкірна та пропріоцептивна

105 / 4299
В експерименті електричними імпульсами подразнюють нерв, що призводить до виділення малої кількості густої в’язкої слини підщелепною та під’язиковою залозами. Який нерв стимулюють?

N. sympaticus

N. glossopharyngeus

N. trigemrnus

N. facialis

N. vagus

106 / 4299
Внаслідок ураження патологічним процесом провідних шляхів спинного мозку у людини порушена больова чутливість шкіри та м’язів. Які шляхи уражені?

Спинно-таламічні

Медіальні спинно-кортикальні

Латеральні спинно-кортикальні

Задні спинно-мозочкові

Передні спинно-мозочкові

107 / 4299
У людини визначили величину енерговитрат. У якому стані знаходилася людина, якщо її енерговитрати виявилися меншими за основний обмін?

Сон

Спокій

Нервове напруження

Відпочинок

Легка робота

108 / 4299
На слизовій оболонці лівої щоки визначається безболісне, еластичної консистенції новоутворення, розташоване на вузькій, м’якій ніжці, розміром 1х1см, нерухоме, поверхня його нерівна, сосочкова. Слизова оболонка, що вкриває новоутворення, у кольорі не змінена. Для якого захворювання найбільше характерні такі зміни?

Аденома

Рак

Аденокистозна карцинома

Папілома

Поліп

109 / 4299
У хворого спостерігається збільшення проникності стінок кровоносних судин із розвитком підвищеної кровоточивості ясен, виникнення дрібнокра-пчастих крововиливів на шкірі, випадіння зубів. Яким порушенням вітамінного обміну пояснюються ці симптоми?

Гіповітаміноз А

Гіповітаміноз С

Гіпервітаміноз D

Гіповітаміноз D

Гіпервітаміноз С

110 / 4299
У хворого спостерігаються дерматит, діарея, деменція. В анамнезу відомо, що основним продуктом харчування хворого є кукурудза. З нестачею якого вітаміну пов’язані ці порушення?

РР

B1

В8

В9

B2

111 / 4299
Хвора 58 років надійшла у важкому стані: свідомість затьмарена, шкіра суха, очі запалі, ціаноз, запах гнилих яблук з рота. Глюкоза крові 15,1 ммоль/л, в сечі 3,5% глюкози. Що є причиною такого стану?

Гіпоглікемічна кома

Уремічна кома

Гіповолемічна кома

Гіперглікемічна кома

Анафілактичний шок

112 / 4299
У дитини грудного віку спостерігається потемніння склер, слизових обо- лонок, вушних раковин, виділена сеча темніє на повітрі. У крові та сечі виявлено гомогентизинову кислоту. Яке захворювання у дитини?

Цистинурія

Альбінізм

Порфірія

Алкаптонурія

Гемолітична анемія

113 / 4299
До лікарні надійшов 9-річний хлопчик з відставанням у розумовому і фізичному розвитку. Під час біохімічного аналізу крові виявлено підвищену кількість фенілаланіну. Блокування якого фермента може призвести до такого стану?

Оксидаза гомогентизинової кислоти

Аспартатамінотрансфераза

Глутаматдекарбоксилаза

Глутамінтрансаміназа

Фенілаланін-4-монооксигеназа

114 / 4299
Хворому на хронічний гепатит для оцінки знешкоджуючої функції печінки було проведено пробу з навантаженням бензоатом натрію. Виділення якої кислоти з сечею характеризуватиме знешкоджуючу функцію печінки?

Щавлева

Валеріанова

Фенілоцтова

Лимонна

Гіпурова

115 / 4299
У експериментальної тварини, що знаходиться на безбілковому раціоні, розвинулася жирова інфільтрація печінки внаслідок дефіциту метилюючих агентів. Утворення якого метаболіту порушено у піддослідної тварини?

Лінолева кислота

ДОФА

Холестерин

Ацетоацетат

Холін

116 / 4299
До відділення травматології надійшов хворий з розчавлюванням м’язової тканини. Який біохімічний показник сечі при цьому буде збільшений?

Сечова кислота

Креатинін

Загальні ліпіди

Мінеральні солі

Глюкоза

117 / 4299
При хронічному панкреатиті спо- стерігається зменшення синтезу і секреції трипсину. Розщеплення яких речовин буде порушене?

Нуклеїнові кислоти

Вуглеводи

Ліпіди

Білки

Білки і вуглеводи

118 / 4299
У хворого, що страждає на цингу, порушено процеси утворення сполучної тканини, що призводить до розхитування і випадіння зубів. Порушення активності якого ферменту зумовлює ці симптоми?

Глікозилтрансфераза

Лізилгідроксилаза

Проколагенпептидаза N-кінцевого пептиду

Еластаза

Проколагенпептидаза С-кінцевого пептиду

119 / 4299
При вживанні печива, цукерок у змішаній слині тимчасово зростає рівень лактату. Активація якого біохімічного процесу призводить до цього?

Тканинне дихання

Ілюконеогенез

Аеробний гліколіз

Мікросомальне окислення

Анаеробний гліколіз

120 / 4299
Підвищена ламкість судин, руйнування емалі і дентину у хворих на цингу здебільшого зумовлено порушенням дозрівання колагену. Який етап модифікації проколагену порушено при цьому авітамінозі?

Ілікозилювання гідроксилізинових залишків

Видалення з проколагену С-кінцевого пептиду

Відщеплення N-кінцевого пептиду

Утворення поліпептидних ланцюгів

Гідроксилювання проліну

121 / 4299
У хворого після переливання крові спостерігається жовтуватість шкіри та слизових оболонок, в крові підвищено рівень загального та непрямого білірубіну, у сечі підвищено рівень уробіліну, у калі - стеркобіліну. Який вид жовтяниці у хворого?

Жовтяниця новонароджених

Паренхіматозна

Спадкова

Обтураційна

Гемолітична

122 / 4299
У дитини на протязі перших трьох місяців після народження розвинулася важка форма гіпоксії, що проявлялася задухою та синюшністю шкіри. Причиною цього є порушеня заміни фетального гемоглобіну на:

Ілікозильований гемоглобін

Гемоглобін А

Гемоглобін М

Гемоглобін S

Метгемоглобін

123 / 4299
При дефіциті якого вітаміну спостерігається одночасне порушення репродуктивної функції і дистрофія скелетної мускулатури?

Вітамін Е

Вітамін Д

Вітамін К

Вітамін В1

Вітамін А

124 / 4299
У нормі рН слини складає 6,4 - 7,8. До яких змін емалі призводить зсув рН слини у кислий бік (менше 6,2)?

Флюороз

Підвищення стійкості

Кальцифікація

Демінералізація

Мінералізація

125 / 4299
При обстеженні хворого похилого віку був встановлений супутній діагноз “блукаюча нирка' Зміни в яких структурах нирки призвели до цього стану?

Corpus a^posum pararenale

Fascia renaUs

ffilus renaUs

Smus renaUs

Ureter

126 / 4299
Під час дослідження плазми крові пацієнта через 4 години після приймання ним жирної їжі встановлено, що вона є каламутною. Найбільш ймовірною причиною даного стану є підвищення концентрації в плазмі:

Хіломікронів

ЛПНГ

Фосфоліпідів

Холестерину

ЛПВГ

127 / 4299
У хворого 43 років з хронічним атрофічним гастритом і мегалобла-стною гіперхромною анемiєю підвищене виділення метилмалонової кислоти з сечею. Недостатністю якого вітаміну обумовлене виникнення зазначеного симптомокомплексу?

В 5

B з

В 6

В12

B 2

128 / 4299
Внаслідок тривалого голодування в органiзмi людини швидко зникають резерви вуглеводів. Який з процегів метаболізму поновлює вміст глюкози у крові?

Анаеробний гліколіз

Аеробний гліколіз

Глюконеогенез

Глікогеноліз

Пентозофосфатний шлях

129 / 4299
При гістологічному дослідженні щитовидної залози хворого, який помер від серцевої недостатності при явищах гіпотіреозу, виявлено дифузну інфільтрацію залози лімфоцитами і пла-змоцитами, атрофію паренхіми і розростання сполучної тканини. Який найбільш імовірний діагноз?

Аденома щитовидної залози

Гнійний тиреоїдит

Зоб тиреотоксичний

Аутоімунний тиреоїдит Хашимото

130 / 4299
У цитоплазмі міоцитів розчинена велика кількість метаболітів окислення глюкози. Назвіть один з них, що безпосередньо перетворюється на лактат.

Оксалоацетат

Фруктозо-6-фосфат

Гліцерофосфат

Піруват

Глюкозо-6-фосфат

131 / 4299
Вагітна жінка вживала алкоголь, що призвело до порушення закладки ектодерми ембріону. У яких похідних цього листка розвинуться вади?

Статеві залози

Нирки

Надниркові залози

Нервова трубка

Епітелій кишечника

132 / 4299
У хворого з опіками 40% поверхні тіла розвинувся опіковий шок. Який механізм розвитку шоку домінує на його початку?

Порушення мінерального обміну

Зневоднення

Аутоімунізація

Порушення білкового обміну

Больовий

133 / 4299
На ранній стадії цукрового діабету у хворих спостерігається поліурія. Чим вона зумовлена?

Гіперглікемія

Кетонемія

Гіпохолестеринемія

Гіперкаліемія

Гіперхолестеринемія

134 / 4299
При аналізі спірограми хворого встановлено зменшення частоти і глибини дихання. До зменшення якого показника це призведе?

Життєва ємність легень

Резервний об’єм видиху

Хвилинний об’єм дихання

Резервний об’єм вдиху

Залишковий об’єм

135 / 4299
Постраждалому у ДТП наклали кровозупинюючий джгут на верхню третину стегна на 3 години. Після зняття джгута у хворого виник різкий набряк тканин стегна, частий пульс, холодний піт, виражена гіпотонія. Який патологічний процес розвинувся у хворого?

Колапс

Кардіогенний шок

Токсемічний шок

Анафілактичний шок

Геморагічний шок

136 / 4299
У хворого після важкої травми грудної клітки розвинувся шок та з’явилися ознаки гострої ниркової недостатності [ГНН]. Що є провідним механізмом розвитку ГНН у даному випадку?

Підвищення тиску в ниркових артеріях

Підвищення тиску в капсулі клубочка

Порушення відтоку сечі

Падіння артеріального тиску

Зменшення онкотичного тиску крові

137 / 4299
У хворого на променеву хворобу з’явилися ознаки геморагічного синдрому. Який механізм має найбільше значення у патогенезі цього синдрому?

Тромбоцитопенія

Нейтропенія

Еритропенія

Еозинопенія

Лімфопєнія

138 / 4299
Хворий 68 років переніс інфаркт мiокарда. Під час ЕКГ-обстеження відзначається прогресуюче збільшення тривалості інтервалу PQ аж до випадіння комплексу QRS, після чого інтервал PQ відновлюється. З порушенням якої функції серця пов’язане таке порушення серцевого ритму?

Збудливість

Провідність

Автоматизм

Скоротливість

139 / 4299
У хворого 37 років після отруєння невідомим лікарським препаратом відзначаються швидкі самодовільні стереотипні скорочення м’язів обличчя, що імітують мигання і примружування. До якої форми розладів рухової функції нервової системи слід віднести ці пору-шенн?

Акінезія

Гіперкінез

Гіпокінез

Атаксія

140 / 4299
У хворої ушкодження задньої долі гіпофізу призвело до збільшення добового діурезу до 10-15 л. Що є головним механізмом у розвитку поліурії?

Дефіцит вазопресину

Надлишок альдостерону

Надлишок натрійуретичного фактора

Надлишок вазопресину

Дефіцит кортикотропіну

141 / 4299
Після вживання меду у підлітка з’явилася кропив’янка, що супроводжувалася лейкоцитозом. Який вид лейкоцитозу виник у даному випадку?

Лімфоцитоз

Нейтрофілія

Моноцитоз

Еозинофілія

Базофілія

142 / 4299
Пацієнт 16 років, що страждає на хворобу Іценка-Кушінга, звернувся з приводу надлишкової маси тіла. Під час опитування виявилося, що енергетична цінність їжї, що вживається, складає 1700-1900 ккал/добу. Яка провідна причина ожиріння у цьому випадку?

Надлишок інсуліну

Гіподинамія

Надлишок глюкокортикоїдів

Нестача глюкокортикоїдів

Нестача інсуліну

143 / 4299
У людини, яку покусали бджоли, розвинувся набряк верхніх кінцівок та обличчя. Який основний патогенетичний механізм розвитку цього набряку?

Підвищення проникності стінки судин

Зменшення гідростатичного тиску тканин

Збільшення онкотичного тиску тканин

Зменшення онкотичного тиску крові

Збільшення гідростатичного тиску в капілярах

144 / 4299
У хворого на пневмосклероз розвинулася легенева гіпертензія та пра-вошлуночкова серцева недостатність з асцитом та набряками. Який основний патогенетичний механізм розвитку набряків у цього хворого?

Зменшення осмотичного тиску крові

Збільшення онкотичного тиску міжклітинної рідини

Зменшення онкотичного тиску крові

Збільшення гідростатичного тиску крові у венах

Збільшення проникності стінок судин

145 / 4299
Через декілька годин після опіку в ділянці гіперемії та набряку шкіри у хворого з’явилось вогнище некрозу. Який головний механізм забезпечує посилення руйнівних явищ в осередку запалення?

Діапедез еритроцитів

Проліферація фібробластів

Первинна альтерація

Вторинна альтерація

Еміграція лімфоцитів

146 / 4299
У хворого на аденому клубочкової зони кори наднирників (хвороба Кон-на) спостерігаються артеріальна гіпертензія, напади судом, поліурія. Що є головною ланкою в патогенезі цих порушень?

Гіперсекреція альдостерону

Гіперсекреція катехоламінів

Гіпосекреція глюкокортикоїдів

Гіпосекреція альдостерону

Гіперсекреція глюкокортикоїдів

147 / 4299
У дитини 5 років через 2 тижні після перенесеної ангіни виник гострий дифузний гломерулонефрит, що характеризувався олігурією, протеїнурією, гематурією, гіперазотемією. Порушення якого процесу у нирках найбільш суттєве для виникнення цих змін?

Клубочкова фільтрація

Канальцева секреція

Сечовиведення

Канальцева реабсорбція

Секреція гормонів

148 / 4299
У хворого на хронічний мієлолейкоз виявлено ознаки анемії - зменшення кількості еритроцитів і вмісту гемоглобіну, оксифільні і поліхроматофіль-ні нормоцити, мікроцити. Який патогенетичний механізм є провідним у розвитку цієї анемії?

Внутрішньосудинний гемоліз еритроцитів

Дефіцит вітаміну В12

Хронічна кровотрата

Зменшення синтезу еритропоетину

Заміщення еритроцитарного ростка

149 / 4299
Під час плевральної пункції було отримано гнійний ексудат. При мікроскопічному дослідженні ексудату знайдено багато гнійних тілець. У результаті руйнування яких клітин крові вони утворюються?

Еозинофіли

Базофіли

Нейтрофіли

Лімфоцити

Еритроцити

150 / 4299
Дитина 3 років мала множинні порушення розвитку кісток лицевого відділу черепа. Загинула від сепсису, який розвинувся на фоні бронхопневмонії. У крові вміст імуноглобулінів в межах норми. Під час розтину встановлена відсутність тимуса. Яка головна причина страждань дитини?

Хронічна інтоксикація

Гострий лімфолейкоз

Вторинний імунодефіцит

Недостатність клітинного імунітету

Комбінований імунодефіцит

151 / 4299
Під час розтину тіла померлого від набряку легень у міокарді знайдено велике вогнище жовто-сірого кольору, а в коронарній артерії - свіжий тромб. Який найбільш імовірний діагноз?

Міокардит

Амілоїдоз

Кардіоміопатія

!нфаркт міокарда

Кардіосклероз

152 / 4299
Хвора 45 років скаржиться на біль і кровотечу з каріозної порожнини 36 під час їжі. Раніше спостерігався непро-вокований біль. Під час огляду 36 - на жувальній поверхні глибока каріозна порожнина, виповнена тканиною червоного кольору, під час зондування відмічається біль і кровотеча. Яка тканина розрослася у каріозну порожнину?

Грануляційна

М’язова

Жирова

Фіброзна

Епітеліальна

153 / 4299
Під час розтину тіла померлого від серцевої недостатності виявлено серозний менінгіт, некроз і крововиливи в наднирниках, пролежні. Мікроскопічно: деструктивнопроліферативний ендотромбоваскуліт (гранульоми Попова) у довгастому мозку, інтерстиці-альний міокардит. Яке захворювання було у померлого?

Кір

Грип

Скарлатина

Черевний тиф

Висипний тиф

154 / 4299
Жінка 38 років захворіла гостро з підвищенням температури тіла до 40оС. На другому тижні хвороби на шкірі живота з’явилися розеоли. На 18-й день хвороби виявлені ознаки “гострого живота' діагностовано перитоніт, від якого хвора померла. На аутопсії у клубовій кишці виявлені глибокі виразки у ділянці групових фолікулів, одна з яких перфорувала, у черевній порожнині -фібринозно-гнійний ексудат. Який найбільш імовірний діагноз?

Кампілобактерійний ентероколіт

Черевний тиф

Дизентерія

Амебіаз

Лямбліоз

155 / 4299
У хворого на хронічний періодон-тит рентгенологічно виявлено кісту кореня зуба. Мікроскопово в операційному матерiалi виявлено, що порожнина кісти вистелена багатошаровим плоским єпітєлієм, стінка кісти утворена грануляційною тканиною різного ступеня зрілості з дифузним запальним інфільтратом. Який найбільш імовірний діагноз?

Радикулярна кіста

Амелобластома

Примордіальна кіста

Фолікулярна кіста

156 / 4299
Під час розтину тіла виявлено, що вся права легеня збільшена, щільна, на плеврі нашарування фібрину, на розрізі тканина сірого кольору, з якої стікає каламутна рідина. Для якого захворювання легенів характерна така картина?

Гангрена легені

Вогнищева пневмонія

!нтерстиціальна пневмонія

Крупозна пневмонія

Фіброзуючий альвеоліт

157 / 4299
Під час аутопсії у потиличній долі головного мозку виявлена порожнина 2,5х1,5 см, заповнена прозорою рідиною, стінка її гладенька, буроватого кольору. Який процес розвинувся у головному мозку?

Киста на місці крововиливу

Абсцес мозку

Вада розвитку мозку

Кіста на місці сірого розм’якшення мозку

Сіре розм’якшення мозку

158 / 4299
На слизовій оболонці мигдаликів та м’якого піднебіння виявляються білувато-сірого кольору плівки, які щільно з’єднані з підлеглою тканиною, при спробі зняти плівку на її місці виникає глибокий дефект тканини. Який вид запалення виник на слизовій оболонці мигдаликів та м’якого піднебіння?

Серозне

Змішане

Гнійне

Крупозне

Дифтеритичне

159 / 4299
Під час дослідження коронарних артерій виявлені атеросклеротичні бляшки з кальцинозом, що закривають просвіт на 1/3. У м’язі дрібні множинні білуваті прошарки сполучної тканини. Який процес виявлено у міокарді?

Інфаркт міокарда

Дифузний кардіосклероз

Міокардит

Післяінфарктний кардіосклероз

Тигрове серце

160 / 4299
У чоловіка 43 років, який тривалий час хворів на туберкульоз, розвинулася легенева кровотеча, що призвело до смерті. Під час розтину тіла у легенях виявлено декілька порожнин овальної або округлої форми, стінка яких утворена некротичними масами та тканиною легень. Для якої форми туберкульозу легень характерні такі зміни?

Фіброзно-кавернозний

Туберкулома

Казеозна пневмонія

Гострий вогнищевий

Гострий кавернозний

161 / 4299
Хворий помер від серцевої недостатності, в анамнезі - пульмонектомія з приводу кісти правої легені. Під час розтину виявлено збільшену у розмірах ліву легеню. Назвіть патологічний процес у лівій легені.

Нейротична атрофія

Вікарна гіпертрофія

Дисциркуляторна атрофія

Нейрогуморальна гіпертрофія

Дисфункціональна атрофія

162 / 4299
Хворий захворів гостро, скаржився на озноб, підвищення температури тіла до 40оС, головний біль, кашель, задишку. На 5-й день захворювання помер. Під час розтину: легені збільшені в об’ємі, пістрявого вигляду - 'велика пістрява легеня'. Для якого захворювання характерна така патологоанато-мічна картина?

Респіраторно-синцитіальна інфекція

Бронхоектатична хвороба

Крупозна пневмонія

Аденовірусна інфекція

Грип

163 / 4299
Під час розтину тіла жінки 45 років, що померла від хронічної ниркової недостатності, знайдено: склероз та гі-аліноз дерми, великовогнищеві некрози кортикального шару нирок та нефросклероз, великовогнищевий кардіосклероз та базальний пневмосклероз. Який найбільш імовірний діагноз?

Системна склеродермія

Дерматоміозит

Вузликовий периартеріїт

Системний червоний вовчак

Ревматизм

164 / 4299
Під час гістологічного дослідження легень померлого від серцевої недостатності виявлені вогнища запалення з заповненням альвеол рідиною, забарвленою у блідо-рожевий колір, місцями з наявністю тонких рожевих ниток, які утворюють дрібнопетлисту сітку з невеликою кількістю лімфоцитів. Який характер ексудата у легенях?

Геморагічний

Фібринозний

Серозний

Серозно-фібринозний

Гнійний

165 / 4299
Під час розтину тіла померлого через тиждень від початку профузної діареї, виявлено різко виражений ексікоз, кров густа. Під час бактеріологічного дослідження вмісту тонкої кишки, що нагадує рисовий відвар, знайдені вібріони. Яке захворювання призвело до смерті хворого?

Дизентерія

Сальмонельоз

Холера

Черевний тиф

Харчова токсикоінфекція

166 / 4299
Експериментальній тварині після попередньої сенсибілізації підшкірно введено дозу антигену. У місці ін’єкції розвинулося фібринозне запалення з альтерацією стінок судин, основної речовини та волокнистих структур сполучної тканини у вигляді мукоїдного на фібриноїдного набухання, фібрино-їдного некрозу. Яка імунологічна реакція розвинулася у тварини?

Гіперчутливість сповільненого типу

Гранульоматоз

Гіперчутливість негайного типу

Реакція трансплантаційного імунітету

Нормергічна реакція

167 / 4299
Вміст везикул зі слизової оболонки хворого на натуральну віспу направлений до вірусологічної лабораторії. Що з наведеного буде виявлено при мікроскопії мазків?

Тільця Пашена

Тільця Бабеша-Негрі

Тільця Іуарнієрі

Синцитій

Тільця Бабеша-Ернста

168 / 4299
Хвора 27 років скаржиться на сухість губ, появу лусочок, котрі вона зкушує. Об’єктивно: губи сухі, на межі червоної кайми та слизової оболонки губ наявність сірих прозорих лусочок, краї яких дещо виступають над поверхнею слизової. Після зняття лусочок ерозій немає, відмічається гіперемія. Який найбільш імовірний діагноз?

Червоний вовчак

Лейкоплакія

Метеорологічний хейліт

Кандидозний хейліт

Ексфоліативний хейліт

169 / 4299
У хворого хлопчика 5 років на п’ятий день після початку захворювання з слизової носоглотки і мигдаликів виділений збудник поліомієліту. Яку реакцію слід використати для встановлення серотипу збудника?

Реакція непрямої гемаглютинації

Реакція зв’язування комплементу

Реакція вірусної нейтралізації цито-патичної дії

Реакція гальмування гемаглютинації

Реакція гальмування гемадсорбції

170 / 4299
При гістологічному дослідженні слизової оболонки матки знайдені звивисті залози, пилко- та штопороподібні, подовжені розростанням строми з проліферацією її клітин. Який найбільш імовірний діагноз?

Пухирний занос

Лейоміома

Гострий ендометрит

Плацентарний поліп

Залозиста гіперплазія ендометрія

171 / 4299
Лікарю-пародонтологу необхідно оцінити у пацієнтки фактори неспецифічної резистентності слини і видільного слизової оболонки ротової порожнини. Який фактор неспецифічної резистентності слід вивчити у досліджуваному матеріалі у першу чергу?

Пропердин

Лізоцим

Секреторний IgA

Інтерферон

Комплемент

172 / 4299
До відділення хірургічної стоматології надійшов пацієнт з відкритим переломом нижньої щелепи. Який препарат слід застосувати з метою активної імунізації проти правцю?

Протиправцева сироватка

Правцевий анатоксин

Коклюшно-дифтерійна-правцева вакцина

Іаммаглобулін з крові донорів, імунізованих проти правцю

Протиправцевий імуноглобулін

173 / 4299
У відділенні щелепно-лицьової хірургії у хворого виникло гнійне ускладнення. Під час бактеріологічного дослідження матеріалу із рани виділено культуру, що утворювала пігмент синьо-зеленого кольору. Який мікроорганізм найімовірніше є збудником інфекції?

Klebsiella pneumornae

Batillus subtihs

Staphylococcus epidermidis

Pseudomonas aerugmosa

Proteus vulgaris

174 / 4299
У пацієнта після тривалого вживання антибіотиків розвинувся дизба-ктеріоз кишечника. Які препарати слід призначити для відновлення нормальної мікрофлори?

Протигрибкові препарати

Сульфаніламіди

Інтерферон

Еубіотики (пробіотики)

Нітрофурани

175 / 4299
У пацієнта в результаті активації власних мікроорганізмів, що входять до складу мікрофлори слизової оболонки рота, виник гнійно-запальний процес тканин пародонту. До якої форми інфекції належить захворювання?

Рецидив

Суперінфекція

Екзогенна інфекція

Аутоінфекція

Реінфекція

176 / 4299
У хворого виділено чисту культуру коринебактерій дифтерії. Яку імунологічну реакцію слід використати для виявлення токсигенності бактерій?

Преципітації в агарі

Зв’язування комплементу

Аглютинації

Іальмування гемаглютинації

Непрямої гемаглютинації

177 / 4299
Дитина 1 року часто хворіє на вірусно-бактеріальні інфекції, які погано піддаються терапії. Під час проведення дослідження імунологічного статусу виявлено відсутність у крові лімфоцитів, що забезпечують клітинний імунітет. Який імунодефіцит і за якою системою виявлений у цієї дитини?

Вторинний за Т-системою

Первинний за Т-системою

Первинний за мікрофагами

Первинний за макрофагальною системою

Первинний за В-системою

178 / 4299
Пацієнт одужав після перенесеної дизентерії Зонне і повторно заразився цим же збудником. Як називається така форма інфекції?

Реінфекція

Рецидив

Персистуюча інфекція

Хронічна інфекція

Суперінфекція

179 / 4299
Хворому на інсулінзалежний цукровий діабет було введено інсулін. Через деякий час у хворого з’явилися слабкість, дратівливість, посилення потовиділення. Який основний патогенетичний механізм розвитку гіпоглікемічної коми, що виникла?

Вуглеводне голодування головного мозку

Зменшення глюконеогенезу

Посилення глікогенолізу

Посилення ліпогенезу

Посилення кетогенезу

180 / 4299
Людині внутрішньовенно ввели розчин, що спричинило збільшення частоти та сили серцевих скорочень. Які складові розчину зумовили ці зміни?

Ілюкоза

Іони хлору

Іони кальцію

Іони натрію

Іони калію

181 / 4299
Для лікування стоматиту дитині 10 місяців призначено антисептичний препарат в розчині для змазування слизової оболонки рота. Що з наведеного призначено дитині?

Водний розчин метиленового синього

Спиртовий розчин дiамантового зеленого

Спиртовий розчин етакридину лактату

Водний розчин фенолу

Водний розчин саліцилової кислоти

182 / 4299
Хворий із серцевою недостатністю приймає дигоксин. Який ефект диго-ксину зумовлює доцільність його застосування при цій патології?

Збільшення частоти серцевих скорочень

Зменшення постнавантаження на серце

Зменшення переднавантаження на серце

Зменшення потреби міокарду в кисні

Збільшення сили серцевих скорочень

183 / 4299
У клітинах людини під дією ультрафіолетового випромінювання відбулося пошкодження молекули ДНК. Реалізувалася система відновлення пошкодженої ділянки молекули ДНК по непошкодженому ланцюгу за допомогою специфічного ферменту. Як називається це явище?

Репарація

Реплікація

Дуплікація

Термінація

Ініціація

184 / 4299
Для лікування ксеростомії хворому було призначено пілокарпіну гідрохло-рид в краплях всередину. Хворий порушував режим лікування: у нього виникли брадикардія, бронхоспазм, болісне посилення перистальтики, різке потовиділення, слинотеча, звуження зініць і спазм акомодації. Для усунення симптоматики був призначений атропіну сульфат. Який вид взаємодії існує між зазначеними препаратами?

Кумуляція

Синергізм

Звикання

Антагонізм

Тахіфілаксія

185 / 4299
Для лікування пародонтиту в комплекс препаратів був включений лікарський засіб з групи водорозчинних вітамінів, похідне біофлавоноїдів, який призначають разом з кислотою аскор- біновою. Препарат має антиоксидантні властивості, зменшує кровоточивість ясен. Який це препарат?

Кислота фолієва

Кальцію пантотенат

Ціанокобаламін

Кальцію пангамат

Рутин

186 / 4299
Хірург використав 70% розчин спирту етилового для обробки рук перед оперативним втручанням. Який основний механізм антисептичної дії препарату на мікроорганізми?

Взаємодія з гідроксильними групами ферментів

Блокада сульфгідрильних груп ферментних систем

Дегідратація білків протоплазми

Взаємодія з аміногрупами білків протоплазми

Окислення органічних компонентів протоплазми

187 / 4299
У наркологічному відділенні психоневрологічної лікарні хворому призначено тетурам. Яке захворювання може бути показанням для його застосування?

Алкогольний психоз

Гостре отруєння морфіном

Наркоманія

Гостре отруєння спиртом етиловим

Алкоголізм

188 / 4299
Хворий з феохромоцитомою страждає від підвищеного артеріального тиску, який переростає в гіпертонічні кризи. Препарати якої групи допоможуть хворому?

Гангліоблокатори

Симпатолітики

о-адреноблокатори

^-адреноблокатори

Блокатори кальцієвих каналів

189 / 4299
У хворого, що тривало хворіє на туберкульоз, виявлене внутрішньоклітинне розташування мікобактерій. Який препарат обов’язково має бути включений до комплексної терапії туберкульозу?

Етамбутол

Етіонамід

Ізоніазид

Рифампіцин

Натрію парааміносаліцилат

190 / 4299
Хворому було призначено препарат. Через декілька діб дія препарату значно знизилась i для отримання початкового ефекту стало необхідним збільшити дозу. За яким типом змінилася дія лiкарської речовини?

Кумуляція

Звикання

Ідіосинкразія

Лікарська залежність

Тахіфілаксія

191 / 4299
Назвіть групу препаратів, що зменшують потребу міокарду в кисні, зменшують силу серцевих скорочень та гальмують ліполіз:

Селективні бета-адреноміметики

Альфа-адреноблокатори

Симпатолітики

Альфа-адреноміметики

Бета-адреноблокатори

192 / 4299
Чоловік під час епідемії грипу звернувся до лікаря з приводу профілактики та раннього лікування цього захворювання. Який препарат можна призначити?

Бонафтон

!нтерферон

Метисазон

Адамантан

Ацикловір

193 / 4299
Хворий 25 років звернувся до лікаря із скаргами на нежить та головний біль протягом 4 днів. Після обстеження йому встановлено діагноз фронтит. Через який носовий хід інфекція потрапила до лобної пазухи?

Середній

Носоглотковий

Загальний

Верхній

Нижній

194 / 4299
У пацієнта після переохолодження у ділянці крил носа та верхньої губи з’явились герпетичні висипання. Для лікування була застосована мазь. Який противірусний засіб містить ця мазь?

Інтерферон

Ацикловір

Індометацин

Дексаметазон

Азидотимідин

195 / 4299
У хворого перелом кістки повільно загоюється. Який засіб можна вико- ристати для пришвидшення загоєння її сполучнотканинної матриці?

Циклофосфан

Циклоспорин

Метилурацил

Метотрексат

Преднізолон

196 / 4299
Хворому 50 років з ішемічною хворобою серця лікар призначив лікарський засіб, який має антиагрегантну дію. Хворий почав приймати засіб у дозах більших, ніж було призначено, внаслідок чого виникли нудота, блювання, біль натщесерце у животі. Який засіб було призначено хворому?

Тиклід

Ацетилсаліцилова кислота

Пармідин

Діпіридамол

Пентоксифілін

197 / 4299
Хворому 60 років було встановлено діагноз цироз печінки, причиною якого було токсичне ураження алкоголем. Який препарат краще за все призначити хворому?

Аскорбінова кислота

Кислота глютамінова

Вітамін Е

Ессенціале

Рибоксин

198 / 4299
У хворого на гострий міокардит з’явилися клінічні ознаки кардіогенно-го шоку. Який патогенетичний механізм є провідним у розвитку шоку?

Зниження діастолічного притоку крові до серця

Порушення насосної функції серця

Зниження судинного тонусу

Депонування крові у венах

Збільшення судинного тонусу

199 / 4299
Який препарат, що має протигли-стну дію, застосовують для стимуляції імунної системи організму при хронічному генералізованому парадонтиті?

Насіння гарбуза

Пірантел

Піперидину адипінат

Хлоксил

Левамізол

200 / 4299
Хвора 50 років страждає на безсоння. Протягом 3 місяців вона приймала різні снодійні засоби: етамінал натрію, фенобарбітал, барбаміл. Після відміни препаратів хвора стала дратівливою, відновилось безсоння, з‘явилися агресивність, втрата апетиту, тремор кінцівок. Яке ускладнення фармакотерапії виникло у хворої?

Звикання до препаратів

Функціональна кумуляція

Сенсибілізація

Фізична і психічна залежність

Тахіфілаксія

201 / 4299
Хвора на ішемічну хворобу серця приймала антиангінальний препарат, який має наступні властивості розширює вінцєві артерії, периферичні судини (артерiальнi та венозні), зменшує потребу міокарда в кисні, покращує ен-докардіальний кровообіг. Вкажіть цей засіб:

Папаверин

Еуфілін

Дібазол

Нітрогліцерин

Валідол

202 / 4299
Під час обстеження хлопчика 2-х місяців педіатр звернув увагу на те, що плач дитини схожий на котяче нявкання; мають місце мікроцефалія та вада серця. За допомогою цитогенетичного методу було встановлено каріотип - 46 XY, 5р-. На якій стадії мітозу дослідили каріотип хворого?

Прометафаза

Метафаза

Профаза

Анафаза

Телофаза

203 / 4299
У 50-річної жінки на місці видаленого зуба регенерувала нова тканина. Які з наведених кліткових органел найбільш активні при відновленні тканини?

Гладка ЕПС

Лізосоми

Рибосоми

Постлізосоми

Центросоми

204 / 4299
У хворого встановлено попередній діагноз - токсоплазмоз. Який матеріал було використано для діагностики цієї хвороби?

Дуоденальний вміст

Харкотиння

Сеча

Фекалії

Кров

205 / 4299
У червоподібному відростку виявлено гельмінта білого кольору, завдовжки 40 мм з тонким ниткоподібним переднім кінцем. У фекаліях знайдені яйця овальної форми з пробками на полюсах. Визначте вид гельмінта:

Вугриця кишкова

Кривоголовка

Гострик

Аскарида

Волосоголовець

206 / 4299
Після операції у хворого знижена чутливість шкіри передньої та латеральної поверхні шиї. Який нерв ушкоджено?

Nn.supraclaviculares

N.phrenicus

N.transversus colli

N.occipitalis minos

N.auricularis magnus

207 / 4299
Надмірна волосатість вушних раковин (гіпертрихоз) визначається геном, локалізованим у Y-хромосомі. Цю ознаку має батько. Яка вірогідність народження хлопчика з такою аномалією?

0%

75%

25%

100%

35%

208 / 4299
Дитина скаржиться на свербіж потиличної та скроневих ділянок голови. Під час огляду голови мати виявила поверхневі виразки внаслідок розчухів та гниди білого кольору на волоссі. Вкажіть збудника цього патологічного стану:

Воша головна

Муха вольфартова

Воша платяна

Воша лобкова

Блоха людська

209 / 4299
Студенти під час вивчення особливостей генетичного коду з’ясували, що є амінокислоти, яким відповідають по 6 кодонів, 5 амінокислот - 4 різні кодони. Інші амінокислоти кодуються трьома або двома кодонами і тільки дві амінокислоти - одним кодоном. Вкажіть, яку властивість генетичного коду виявили студенти?

Триплетність

Однонаправленість

Універсальність

Надлишковість

Колінеарність

210 / 4299
Дівчина 16 років звернулася до стоматолога з приводу темної емалі зубів. При вивченні родоводу встановлено, що вказана патологія передається від батька всім дівчаткам, а від матері -50% хлопчиків. Для якого типу успадкування характерні ці особливості?

Домінантний, зчеплений з Х- хромосомою

Рецесивний, зчеплений з Х- хромосомою

Аутосомно-рецесивний

Рецесивний, зчеплений з Y- хромосомою

Аутосомно-домінантний

211 / 4299
До клініки надійшла дитина віком 1,5 роки. Під час обстеження було відзначено недоумкуватість, розлади регуляції рухових функцій, слабка пігментація шкіри, у крові високий вміст фенілаланіну. Який найбільш вірогідний діагноз?

Фенілкетонурія

Іалактоземія

Тирозиноз

Муковісцидоз

Синдром Дауна

212 / 4299
У батька та матері широка щілина між різцями є домінантною ознакою. Обидва гомозиготні. Яка генетична закономірність проявиться у їх дітей?

Розщеплення гібридів за фенотипом

Незалежне успадкування ознаки

Одноманітність гібридів першого покоління

Зчеплене успадкування

Незчеплене успадкування

213 / 4299
На ринку батько купив свинину. Якою хворобою можуть заразитися члени сім’ї, якщо це м’ясо не пройшло ветеринарний контроль?

Теніоз

Ехінококоз

Гіменолепідоз

Фасціольоз

Теніаринхоз

214 / 4299
У хворого на гепатоцеребральну дегенерацію під час обстеження виявлено дефект синтезу білка - церулоплазмі-ну. З якими органелами пов’язаний цей дефект?

Комплекс Іольджі

Мітохондрії

Гранулярна ендоплазматична сітка

Лізосоми

Агранулярна ендоплазматична сітка

215 / 4299
Під час постсинтетичного періоду мітотичного циклу було порушено синтез білків - тубулінів, які беруть участь у побудові веретена поділу. Це може призвести до порушення:

Цитокінезу

Деспіралізації хромосом

Спіралізації хромосом

Тривалості мітозу

Розходження хромосом

216 / 4299
У немовляти виявлено мікроцефалію. Лікарі вважають, що це пов’язано із застосуванням жінкою під час вагітності актиноміцину Д. На який зародковий листок подіяв цей тератоген?

Мезодерма

Усі листки

Ектодерма

Ентодерма

Ентодерма та мезодерма

217 / 4299
Інтенсивність пігментації шкіри в людини контролюють кілька незалежних домінантних генів. Встановлено, що при збільшенні кількості цих генів пігментація стає інтенсивнішою. Який тип взаємодії між цими генами?

Комплементарність

Епістаз

Плейотропія

Полімерія

Кодомінування

218 / 4299
Хворий надійшов до відділення щелепно-лицьової хірургії з вивихом скроневонижньощелепного суглоба та пошкодженням основної його зв’язки. Назвіть цю зв’язку:

Крило-нижньощелепна

Шило-нижньощелепна

Нижньощелепна

Медіальна

Латеральна

219 / 4299
В ендокринологічному відділенні знаходиться хлопчик 9 років, у котрого вже декілька разів були переломи кінцівок, пов’язані з крихкістю кісток. Функція яких ендокринних залоз (залози) порушена?

Паращитоподібні

Наднирковозалозні

Щитоподібна

Вилочкова

Епіфіз

220 / 4299
Лікар-стоматолог під час огляду ро- тової порожнини пацієнта відмітив незначне перекриття зубів нижньої щелепи різцями верхньої щелепи. До якого прикусу відноситься таке розміщення зубів?

Біпрогнатія

Прямий прикус

Закритий прикус

Ортогнатія

Прогенія

221 / 4299
У хворого скарги на порушення сечовипускання. Діагностовано гіпертрофію передміхурової залози. Пошкодження якої частини залози призводить до порушення сечовипускання?

Права частка

Верхівка

Ліва частка

Основа

Середня частка

222 / 4299
Хворому для виготовлення функціонального повноцінного протеза необхідно видалити ліве верхнє ікло. Після інфраорбітальної анестезії у хворого з’явилася гематома в передній ділянці обличчя, яка швидко збільшувалася. Встановлено, що травмовано артерію, яка є гілкою:

A.alveolaris inferior

A.ophthalmica

A.labialis superior

A .temporalis superf icialis

A.maxillaris

223 / 4299
До хірурга-стоматолога звернувся чоловік з травмою в ділянці піднижньо-щелепного трикутника. Обробляючи рану, лікар побачив, що травмована артерія, яка іде до м’якого піднебіння. Яку артерію пошкоджено?

A.palatina descendens

A.palatina ascendens

A.pharingea ascendens

A.sphenopalatina

A.facialis

224 / 4299
Під час видалення у хворого ліпоми біля крила носа, хірург-стоматолог пошкодив судину, що призвело до утворення підшкірної гематоми. Яку судину пошкоджено?

A .maxillaris

A.infraorbitalis

A .facialis

A.supraorbitalis

A.angularis

225 / 4299
У хворого поранення у праву бічну ділянку живота. Яка частина товстої кишки найвірогідніше може бути пошкоджена?

Висхідна ободова

Низхідна ободова

Поперечна ободова

Пряма

Сигмоподібна ободова

226 / 4299
У хворого 28 років діагностовано гострий запальний процес слизової оболонки нососльозової протоки. З анамнезу відомо, що після перенесеного грипу протягом 10-ти днів були виділення із носа. З якого відділу носової порожнини інфекція могла надійти у нососльозову протоку?

Нижній носовий хід

Лобова пазуха

Середній носовий хід

Верхній носовий хід

Присінок носа

227 / 4299
Дитина 3 років надійшла до лікарні зі стороннім тілом у бронхах. У який бронх найвірогідніше потрапило стороннє тіло?

Лівий сегментарний

Часточковий

Правий сегментарний

Лівий головний

Правий головний

228 / 4299
До лікаря звернулася мати з дитиною віком 1 рік, у якої прорізалося шість зубів. Яку кількість зубів повинна мати дитина даного віку?

10

12

7

6

8

229 / 4299
Хронічна нежить ускладнилася запаленням лобової пазухи. Через який носовий хід потрапила інфекція до цього синуса?

Носоглотковий

Нижній

Середній

Загальний

Верхній

230 / 4299
У постраждалого в ДТП з правого боку відзначається відрив ділянки кута нижньої щелепи, зміщення відламка назад та вверх. Яка зв’язка сприяє цьо- му зміщенню?

Крилоподiбно-нижньощелепна

Клиноподiбно-нижньощелепна

Внутрішньосуглобова

Латеральна

Шило-нижньощелепна

231 / 4299
Хірург планує взяти у хворого лімфу з грудної протоки в місці її впадіння до венозного русла. Куди слід провести катетер з цією метою?

Місце утворення нижньої порожнистої вени

Лівий венозний кут

Місце утворення ворітної вени

Правий венозний кут

Місце утворення верхньої порожнистої вени

232 / 4299
Рентгенологічно встановлено скупчення гною у верхньощелепному синусі. У який носовий хід виділяється гній?

Середній носовий

Нижній носовий

Верхній носовий

Загальний носовий

Носоглотковий

233 / 4299
У хворого спостерігається асиметрія обличчя, особливо при спробах виконати активне скорочення його м’язів. Пошкодження якого нерва можна запідозрити?

Під’язиковий

Трійчастий, II гілка

Трійчастий, I гілка

Лицевий (рухова частина)

Трійчастий, III гілка

234 / 4299
Дитина 5 років надійшла до ЛОР-відділення з діагнозом гнійне запалення середнього вуха (тимпаніт). Захворювання розпочалося із запалення носоглотки. Через який канал скроневої кістки інфекція потрапила до барабанної порожнини?

Барабанний каналець

Каналець барабанної струни

Сонно-барабанні канальці

Сонний канал

М’язово-трубний канал

235 / 4299
Хворий втратив здатність впізнавати предмети за характерними для них звуками (годинник, дзвін, музика). Яка частина мозку, найбільш вірогідно, ушкоджена?

Lobus parietalis

Lobus frontalis

Lobus occipitalis

Insula

Lobus temporalis

236 / 4299
У хворого запалення середнього вуха (отит). При цьому він скаржиться на розлад відчуття смаку в передній частині язика. Який з нервів ушкоджено?

N.facialis

N.trigeminus

N.glossopharyngeus

N.vestibulo — cochlearis

N.vagus

237 / 4299
Після вживання м’ясних консервів у хворого з’явилося двоїння в очах, сильний головний біль, порушення ковтання, ускладнене дихання, м’язова слабкість. Встановлено діагноз ботулізм. З яким фактором патогенності пов’язані клінічні прояви цього захворювання?

Фібринолізин

Гемолізин

Плазмокоагулаза

Екзотоксин

Ендотоксин

238 / 4299
Хворий скаржиться на біль в яснах та зубах верхньої щелепи. Про запалення якого нерва йде мова?

Додатковий

Під’язиковий

I гілка V пари

II гілка V пари

III гілка V пари

239 / 4299
У боксера, що отримав удар у ділянку скронево-нижньощелепного суглоба, відбувся травматичний вивих нижньої щелепи. Зміщення яких суглобових поверхонь вийде за межі фізіологічної норми?

Вінцевий відросток та крилоподібна ямка

Вінцевий відросток та піднижньоще-лепна ямка

Шийка нижньої щелепи та підни-жньощелепна ямка

Голівка нижньої щелепи та підни-жньощелепна ямка

Голівка нижньої щелепи та нижньощелепна ямка

240 / 4299
У хворого сечокам’яна хвороба ускладнилася виходом конкремента з нирки. На якому рівні сечоводу, найві- рогідніше, він зупиниться?

На 2 см вище впадіння в сечовий міхур

На 5 см вище тазової частини

У середній черевній частиш

У лоханці

На межі черевної та тазової частин

241 / 4299
Під час огляду порожнини рота стоматолог виявив каріозну порожнину на поверхні коронки другого малого кутнього зуба, зверненого до першого моляра. Назвіть уражену поверхню коронки:

Facies vestibularis

Facies lingualis

Facies distalis

Facies mesialis

Facies occlusalis

242 / 4299
До лікарні госпіталізовано жінку в важкому стані з діагнозом геморагічний інсульт у ділянці лобної частки правої півкулі головного мозку. Ушкодження якої артерії, найбільш вірогідно, призвело до цього стану?

A .communicans posterior

A.cerebri media

A.communicans anterior

A.cerebri posterior

A.cerebri anterior

243 / 4299
У хворого на рак спинки язика виникла сильна кровотеча внаслідок ураження пухлиною дорсальної артерії язика. Яку судину слід перев’язати лікарю для зупинки кровотечі?

Лицева артерія

Висхідна артерія глотки

Глибока артерія язика

Дорсальна артерія язика

Язикова артерія

244 / 4299
У хворої виявлено ексудативний плеврит. На якому рівні необхідно провести плевральну пункцію по задній пахвовій лінії?

IX міжреберний простір

VII міжреберний простір

XI міжреберний простір

VI міжреберний простір

VIII міжреберний простір

245 / 4299
Студентка 17 років видавила гній-ничок у медіальному куті ока. Через 2 дні вона надійшла до інституту нейрохірургії з діагнозом тромбоз печеристого синуса. Через яку вену інфекція потра- пила до цього синуса?

V.transversa faciei

V.profunda faciei

V.maxillaris

V.angularis

V.diploicae frontalis

246 / 4299
Перед прорізуванням зубів на їх корені формується тверда тканина, що має характер грубоволокнистої кістки. Як вона називається?

Пухка волокниста сполучна

Щільна волокниста сполучна

Дентин

Цемент

Емаль

247 / 4299
У хворого внаслідок травми ушкоджені передні корінці спинного мозку. Вкажіть, які структури при цьому постраждали:

Центральні відростки нейронів спинномозкових вузлів

Аксони мотонейронів

Периферійні відростки нейронів спинномозкових вузлів

Аксони мотонейронів та нейронів бокових рогів

Аксони нейронів бокових рогів

248 / 4299
При травмі скелетних м’язів процес їх регенерації відбувається повільно. Які елементи скелетного м’язового волокна беруть участь у цьому процесі?

Міофібробласти

Міосателітоцити

Міоепітеліальні клітини

Міобласти

Гладкі міоцити

249 / 4299
Хворий 46 років потрапив до гематологічного відділення лікарні. У нього виявлено порушення процесів грануло-цитопоезу та тромбоцитогенезу. В якому з наведених органів має місце патологічний процес?

Червоний кістковий мозок

Вилочкова залоза

Лімфатичний вузол

Селезінка

Піднебінний мигдалик

250 / 4299
У хворого з гострим ринітом виявлена гіперемія та підвищене утворення слизу в носовій порожнині. Активність яких клітин епітелію слизової оболонки підвищена?

Базальні

Війчасті

Мікроворсинчасті

Ендокринні

Келихоподібні

251 / 4299
В експерименті, у тварини шляхом звуження ниркової артерії, отримано стійке підвищення кров’яного тиску. Підвищення функції яких клітин нирок зумовлює цей ефект?

Ендотеліоцити

Інтерстиційні клітини

Подоцити

Клітини щільної плями

ЮГА-клітини

252 / 4299
У гістологічному препараті трубчастої кістки, на місці зламу виявлені ознаки регенераторного процесу (мозоля). Яка тканина формує цю структуру?

Пухка сполучна

Ретикулярна

Епітеліальна

Пластинчаста кісткова

Грубоволокниста кісткова

253 / 4299
Після хімічного опіку стравоходу відбулося локальне його звуження внаслідок утворення рубця. Які клітини пухкої сполучної тканини беруть участь в утворенні рубців?

Фіброкласти

Юні малоспеціалізовані фібробласти

Фіброцити

Міофібробласти

Зрілі спеціалізовані фібробласти

254 / 4299
У жінки має місце гіперемія яєчника, підвищення проникності гемато-фолікулярного бар’єру з розвитком набряку, інфільтрація стінки фолікула се-гментоядерними лейкоцитами. Об’єм фолікула великий, стінка його потоншена. Якому періоду статевого циклу відповідає описана картина?

Овуляція

Період відносного спокою

Постменструальний період

Предовуляторна стадія

Менструальний період

255 / 4299
У гістологічному препараті визначаються кінцеві відділи залоз, утворені клітинами з центрально розташованим круглим ядром та базофільною цитоплазмою. Визначте вид кінцевих відділів:

Серомукозні

Змішані

Сальні

Серозні

Слизові

256 / 4299
Під час дослідження мікропрепара-ту шкіри пальця дитини встановлено, що епідерміс має ознаки недостатнього розвитку. Який ембріональний листок був пошкоджений у процесі розвитку?

Ектодерма

Ектомезенхіма

Ентодерма

Мезенхіма

Мезодерма

257 / 4299
У препараті, забарвленому за методом імпрегнації сріблом, виявляються клітини грушоподібної форми з добре вираженими 2-3 дендритами. Назвіть структуру, що досліджується:

Спіральний орган внутрішнього вуха

Кора мозочка

Спинномозковий вузол

Кора головного мозку

Сітківка ока

258 / 4299
Під час експериментального дослідження хондрогістогенезу було ушкоджено склеротом. До порушення диференціювання яких клітин це призведе?

Гладенькі міоцити

Міобласти

Фібробласти

Хондробласти

Епідермоцити

259 / 4299
При нестачі вітаміну А у людини відбувається порушення сутінкового зору. Яким клітинам притаманна фо-торецепторна функція?

Колбочкові нейросенсорні клітини

Паличкові нейросенсорні клітини

Горизонтальні нейроцити

Біполярні нейрони

Гангліонарні нервові клітини

260 / 4299
В ембріогенезі ротової порожнини відбулося порушення розвитку емалі зубів. Яке джерело розвитку зубів було пошкоджено?

Епітелій

Зубний сосочок

Зубний мішечок

Мезодерма

Мезенхіма

261 / 4299
Під час розвитку зуба у складі емалевого органу з’являються клітини призматичної форми, які мають шести-кутовий перетин; ядро розташоване у центральні частиш клітини. Що це за клітини?

Клітини пульпи емалевого органа

Камбіальні клітини

Зовнішні енамелобласти

Преенамелобласти

Преодонтобласти

262 / 4299
В ембріогенезі відбулася затримка зростання верхньо- та нижньощелепних відростків. Які аномалії розвитку слід очікувати у цьому випадку?

Макростомія

Мікростомія

Вовча паща

Готичне піднебіння

Розщілина верхньої губи

263 / 4299
Одужання організму від інфекційної хвороби супроводжується нейтралізацією антигенів специфічними антитілами. Якими клітинами вони продукуються?

Плазмоцити

Тканинні базофіли

Фібробласти

Еозинофіли

Т-лімфоцити

264 / 4299
У гістологічному препараті кори на-днирника спостерігаються дрібні полігональні клітини, які утворюють округлі скупчення та містять невелику кількість ліпідних включень. Яку частину наднирника представлено у гістологічному препараті?

Пучкова зона

Проміжна зона

Клубочкова зона

Сітчаста зона

265 / 4299
Хворий 20 років скаржиться на сильну спрагу та збільшене сечовиділення (до 10 л на добу). Рівень глюкози в крові нормальний, у сечі глюкоза відсутня. Дефіцит якого гормону може викликати такі зміни?

Трийодтиронін

Окситоцин

Кортизол

Вазопресин

Інсулін

266 / 4299
Людина довільно затримала дихання впродовж 60 c. Після цього хвилинний об’єм дихання (ХОД) збільшився до 12 л. Яка зміна у крові є головною причиною збільшення ХОД?

Підвищення р СС>2

Зниження р О2

Підвищення р О 2

Підвищення pH

Зниження р С02

267 / 4299
У хворого із зниженою видільною функцією нирок відзначається неприємний запах з рота. Збільшення екскреції слинними залозами якої речовини є причиною цього?

Сечовина

Лізоцим

Муцин

Альфа-амілаза

Фосфатаза

268 / 4299
У людини з захворюванням нирок збільшений артеріальний тиск, особливо діастолічний. Посилена секреція якої біологічно активної речовини спричиняє підвищення тиску у хворого?

Адреналін

Ренін

Катехоламіни

Норадреналін

Вазопресин

269 / 4299
У гострому експерименті у тварини здійснювали електричне подразнення chorda tympani, внаслідок чого з протоки привушної слинної залози виділялося:

Мало в’язкої слини

Багато в’язкої слини

Мало рідкої слини

Багато рідкої слини

Слина не виділялася

270 / 4299
Людина постійно живе високо в горах. Яку зміну показників крові можна виявити у неї?

Зниження показників вмісту гемоглобіну

Поява в крові еритробластів

Зменшення колірного показника

Збільшення кількості еритроцитів

Зниження кількості ретикулоцитів

271 / 4299
У хворого, який протягом тривалого часу приймав глюкокортикоїди, в результаті відміни препарату виникло загострення хвороби, зниження артеріального тиску, слабкість. З чим можна пов’язати ці явища?

Звикання до препарату

Кумуляція

Гіперпродукція кортикотропного гормону

Сенсибілізація

Виникнення недостатності надниркових залоз

272 / 4299
Обстежено спортсмена після інтенсивного фізичного навантаження. Виявлено порушення координації рухів при збереженні сили скорочення м’язів. Причиною цього може бути зменшення швидкості проведення збудження:

Еферентними нервами

Аферентними нервами

Через центральні синапси

Через нервово-м’язові синапси

Провідними шляхами

273 / 4299
У хворого поперечний розрив спинного мозку нижче VI грудного сегмента. Як внаслідок цього зміниться дихання?

Стане більш частим

Стане більш рідким

Стане більш глибоким

Припиниться

Суттєво не зміниться

274 / 4299
Людина вживає суху їжу. Які слинні залози при цьому секретують найбільше?

Піднебінні

Підщелепні

Щічні

Під’язикові

Привушні

275 / 4299
Під час підготовки пацієнта до операції на серці, проведено вимірювання артеріального тиску в камерах серця. В одній з них тиск протягом серцевого циклу змінювався від 0 до 120 мм рт.ст. Назвіть цю камеру серця:

Праве передсердя

Лівий шлуночок

Ліве передсердя

Правий шлуночок

276 / 4299
У хворого різко знизився вміст Са2+ у крові. До збільшення секреції якого гормону це призведе?

Тирокальцитонін

Паратгормон

Альдостерон

Вазопресин

Соматотропний

277 / 4299
У пацієнта після крововиливу в стовбур мозку відсутній рефлекс звуження зіниці у відповідь на збільшення освітлення. Ураження якої структури є причиною цього?

Медіальні ретикулярні ядра

Вегетативні ядра окорухових нервів

Червоні ядра

Чорна речовина

Латеральні ретикулярні ядра

278 / 4299
Під час реєстрації фонокардіограми встановлено, що тривалість першого тону серця вдвічі перевищує норму. Найбільш правильним є висновок про те, що у досліджуваного уражені:

Півмісяцеві клапани

Кардіоміоцити шлуночків

Атріовентрикулярні клапани

Кардіоміоцити передсердь

Мітральні клапани

279 / 4299
В експерименті електричними імпульсами подразнюють нерв, що призводить до виділення малої кількості густої в’язкої слини підщелепною та під’язиковою залозами. Який нерв стимулюють?

N.glossopharyngeus

N.vagus

N.facialis

N.sympathicus

N.trigeminus

280 / 4299
Охолодження тіла людини у воді виникає значно швидше, ніж на повітрі. За рахунок якого шляху тепловіддачі це відбувається?

Тепловипромінювання

Теплопроведення

Конвекція

Випаровування поту

281 / 4299
У людини порушено всмоктування продуктів гідролізу жирів. Дефіцит яких компонентів у порожнині тонкої кишки може бути причиною цього?

Жиророзчинні вітаміни

Жовчні кислоти

Іони натрію

Ліполітичні ферменти

Жовчні пігменти

282 / 4299
Людина вийшла з кондиційоновано-го приміщення на вулицю, де температура повітря дорівнює +40оС, вологість повітря - 60%. За рахунок якого механізму переважно здійснюватиметься віддача тепла за цих умов?

Проведення

Радіація

Конвекція

Випаровування поту

283 / 4299
У тварини під час експерименту перерізали задні корінці спинного мозку. Які зміни відбуватимуться в зоні іннервації?

Втрата рухових функцій

Підвищення тонусу м’язів

Зниження тонусу м’язів

Втрата чутливості

Втрата чутливості та рухових функцій

284 / 4299
Припинення кровотечі після пологів пов’язано з дією окситоцину на стінку матки. Яка оболонка органа реагує на дію цього гормону?

Підслизова

Ендометрій

Периметрій

Параметрій

Міометрій

285 / 4299
У хворого на хронічний гломерулонефрит порушується інкреторна функція нирок. До дефіциту яких формених елементів крові це призведе?

Лейкоцити

Еритроцити

Лейкоцити та тромбоцити

Еритроцити та лейкоцити

Тромбоцити

286 / 4299
У хворого з порушенням мозкового кровотоку порушений акт ковтання. Який відділ мозку постраждав?

Шийний відділ спинного мозку

Стовбур мозку

Середній мозок

Проміжний мозок

Передній мозок

287 / 4299
У хворого із запаленням легень ШЗЕ дорівнює 48 мм/год. Що призвело до подібних змін?

Гіперальбумінемія

Еритроцитоз

Гіпопротеїнемія

Гіпергаммаглобулінемія

Гіпогаммаглобулінемія

288 / 4299
Під час огляду дитини, що не отримувала протягом зими свіжих овочів та фруктів, виявлені множинні підшкірні крововиливи, запалення ясен, каріозні порожнини у зубах. Комбінацію яких вітамінів слід призначити дитині?

Тіамін та піридоксин

Рибофлавін та нікотинамід

Кальциферол та аскорбінова кислота

Фолієва кислота та кобаламін

Аскорбінова кислота та рутин

289 / 4299
До лікаря звернувся пацієнт з приводу сильного свербежу шкіри, особливо між пальцями рук, у пахвових западинах, нижній частині живота. Під час огляду шкіри хворого помічені звивисті ходи брудно-білуватого кольору з кра-пинками на кінцях. Для якого захворювання характерні такі клінічні ознаки?

Дерматотропний лейшманіоз

Скабієс

Міаз

Педикульоз

Демодекоз

290 / 4299
У дитини першого року життя спостерігається збільшення розмірів голови та живота, запізніле прорізування зубів, порушення структури емалі. Наслідком якого гіповітамінозу є ці зміни?

Гіповітаміноз С

Гіповітаміноз А

Гіповітаміноз D

Гіповітаміноз В2

Гіповітаміноз В1

291 / 4299
Якій сумарній кількості молекул АТФ еквівалентне повне окиснення глюкози та спряження його з фосфо-рилюванням?

8

58

12

52

38

292 / 4299
У пацієнтки з постійною гіпоглікемією після введення адреналіну аналіз крові суттєво не змінився. Лікар запідо- зрив патологію печінки. Про зміну якої функції пєчінки може йти мова?

Екскреторна

Гліколітична

Холестеринутворююча

Глікогендепонуюча

Кетогенна

293 / 4299
Хворому на хронічний гепатит для оцінки знешкоджуючої функції печінки було проведено пробу з навантаженням бензоатом натрію. Виділення якої кислоти з сечею характеризуватиме знешкоджуючу функцію печінки?

Щавлева

Гіпурова

Фенілоцтова

Лимонна

Валеріанова

294 / 4299
У хворого 57 років, який страждає на цукровий діабет, розвинувся кето-ацидоз. Біохімічною основою цього стану є зменшення ступеня утилізації ацетил-КоА. Нестачею якої сполуки в клітинах це обумовлено?

2-оксоглутарат

Ілутамат

Аспартат

Оксалоацетат

Сукцинат

295 / 4299
У хворого виявлена болісність по ходу крупних нервових стовбурів та підвищений вміст пірувату в крові. Нестача якого вітаміну може викликати такі зміни?

B1

B2

РР

Пантотенова кислота

Біотин

296 / 4299
Після бігу на коротку дистанцію у нетренованих людей спостерігається м’язова крепатура внаслідок накопичення лактату. З посиленням якого біохімічного процесу в організмі це може бути пов’язано?

Гліколіз

Пентозофосфатний цикл

Ілюконеогенез

Глікогенез

Ліпогенез

297 / 4299
Зниження співвідношення аденіло-вих нуклеотидів АТФ/АДФ призводить до посилення гліколізу в тканинах па- родонту в умовах гіпоксії. Яка реакція при цьому активується?

Енолазна

Альдолазна

Тріозофосфатізомеразна

Фосфофруктокіназна

Лактатдегідрогеназна

298 / 4299
Для підвищення результатів спортсмену рекомендували застосовувати препарат, що містить карнітин. Який процес у найбільшому ступені активізується карнітином?

Синтез стероїдних гормонів

Транспорт жирних кислот у мітохон- дрії

Тканинне дихання

Синтез ліпідів

Синтез кетонових тіл

299 / 4299
У пацієнта цироз печінки. Дослідження якої з перелічених речовин, що екскретуються з сечею, може характеризувати стан антитоксичної функції печінки?

Амінокислоти

Сечова кислота

Амонійні солі

Ппурова кислота

Креатинін

300 / 4299
При дефіциті якого вітаміну спостерігається одночасне порушення репродуктивної функції та дистрофія скелетної мускулатури?

Вітамін D

Вітамін К

Вітамін В1

Вітамін А

Вітамін Е

301 / 4299
Хворий госпіталізований до клініки з попереднім діагнозом прогресуюча м’язова дистрофія. Збільшення вмісту якої речовини у сечі може підтвердити цей діагноз?

Тропонін

Карнозин

Пдроксипролін

Креатин

Піруват

302 / 4299
У хворого на системну склеродермію посилений розпад колагену. Посилення екскреції з сечею якої амінокислоти буде віддзеркалювати процеси деструкції колагену?

Фенілаланін

Оксипролін

Триптофан

Серин

Аланін

303 / 4299
Яка речовина надає слині в’язкий, слизовий характер, виконує захисну роль, у тому числі від механічного пошкодження слизової рота?

Глюкоза

Муцин

Калікреїн

Амілаза

Лізоцим

304 / 4299
У вагітної 28 років, досліджували ферменти в клітинах амніотичної рідини. При цьому виявилася недостатня активність ^-глюкуронідази. Який патологічний процес спостерігається?

Аглікогеноз

Колагеноз

Глікогеноз

Ліпідоз

Мукополісахародоз

305 / 4299
Молодий чоловік після імплантації серцевого клапана систематично отримує непрямі антикоагулянти. Його стан ускладнився кровотечею. Із зменшенням у крові якої речовини це пов’язане?

Протромбін

Гаптоглобін

Креатин

Гепарин

Церулоплазмін

306 / 4299
Під час гістологічного дослідження щитоподібної залози хворого, який помер від серцевої недостатності при явищах гіпотиреозу, виявлено дифузну інфільтрацію залози лімфоцитами та плазмоцитами, атрофію паренхіми та розростання сполучної тканини. Визначте діагноз:

Зоб тиреотоксичний

Аутоімунний тиреоїдит Хашимото

Гнійний тиреоїдит

Аденома щитоподібної залози

307 / 4299
У цитоплазмі міоцитів розчинена велика кількість метаболітів окиснен-ня глюкози. Назвіть один з них, що безпосередньо перетворюється на лактат:

Піруват

Гліцерофосфат

Оксалоацетат

Глюкозо-6-фосфат

Фруктозо-6-фосфат

308 / 4299
При хронічному передозуванні глюкокортикоїдів у хворого розвивається гіперглікемія. Який процес вуглеводного обміну зумовлює це?

Глікогеноліз

Аеробний гліколіз

Пентозофосфатний цикл

Глікогенез

Глюконеогенез

309 / 4299
Хворі на пігментну ксеродерму характеризуються аномально високою чутливістю до ультрафіолетових променів, результатом чого є рак шкіри внаслідок нездатності ферментних систем відновлювати ушкодження спадкового апарату клітин. З порушенням якого процесу пов’язана ця патологія?

Генна комплементація

Генна конверсія

Рекомбінація ДНК

Репарація ДНК

Редуплікація ДНК

310 / 4299
За клінічними ознаками хвороби людині призначено піридоксальфо-сфат. Для корекції яких процесів рекомендовано цей препарат?

Синтез пуринових та піримідинових основ

Трансамінування та декарбоксилю-вання амінокислот

Синтез білка

Окисне декарбоксилювання кетокислот

Дезамінування пуринових нуклеоти-дів

311 / 4299
У чоловіка 35 років через 15 хвилин після автомобільної аварії виявлена масивна травма нижніх кінцівок без значної зовнішньої крововтрати. Постраждалий перебуває у збудженому стані. Який компонент патогенезу травматичного шоку є провідним та потребує негайної корекції?

Порушення функції серця

Внутрішня плазмовтрата

Гостра ниркова недостатність

Інтоксикація

Біль

312 / 4299
У хворого виявлено порушення прохідності дихальних шляхів на рівні дрібних та середніх бронхів. Які зміни кислотно-основної рівноваги (КОР) можуть розвинутися у нього?

Респіраторний алкалоз

КОР не зміниться

Метаболічний ацидоз

Респіраторний ацидоз

Метаболічний алкалоз

313 / 4299
У хворого на інфекційне захворювання температура тіла через добу підвищується до 39,5 — 40,50С та тримається на цьому рівні близько години, а потім повертається до вихідної. Який тип лихоманки має місце у даному випадку?

Виснажуюча

Послаблююча

Атипова

Постійна

Переміжна

314 / 4299
Хворий 23 років надійшов до лікарні у важкому стані з черепно-мозковою травмою. Дихання характеризується судомним тривалим вдихом, який переривається коротким видихом. Для якого типу дихання це характерно?

Чейн-Стокса

Куссмауля

Біота

Апнейстичне

Гаспінг

315 / 4299
У хворого на 2 добу після розвитку інфаркту міокарда різко знизився систолічний АТ до 60 мм рт.ст., виникли тахікардія до 140/хв, задишка; хворий знепритомнів. Який механізм є провідним у патогенезі шоку, що розвинувся?

Інтоксикація

Зниження об’єму циркулюючої крові

Зменшення ударного об’єму крові

Пароксизмальна тахікардія

Анафілактична реакція

316 / 4299
У пацієнта 48 років після психоемоційного навантаження раптово з’явився гострий біль у ділянці серця з іррадіацією у ліву руку. Нітрогліцерин зняв напад болю через 10 хвилин. Який патогенетичний механізм є провідним у розвитку болю?

Розширення периферійних судин

Підвищення потреби міокарда у кисні

Стиснення коронарних судин

Закупорка коронарних судин

Спазм коронарних судин

317 / 4299
Чоловік 40 років, що брав участь у ліквідації наслідків аварії на АЕС, захворів на пародонтит. Який етіологічний фактор є найбільш важливим у розвитку цієї патології?

Дефіцит заліза

Стрептококи

Підвищене навантаження зубощеле-пного апарата

Емоційний стрес

Неповноцінне харчування

318 / 4299
У жінки після пологів зменшилася маса тіла на 20 кг, випадають зуби та волосся, спостерігається атрофія м’язів (гіпофізарна кахексія). З порушенням синтезу якого гормону гіпофіза це пов’язано?

Кортикотропний

Гонадотропний

Тиреотропний

Соматотропний

Пролактин

319 / 4299
Після вживання меду у підлітка з’явилася кропив’янка, що супроводжується лейкоцитозом. Який вид лейкоцитозу виник у даному випадку?

Лімфоцитоз

Нейтрофілія

Еозинофілія

Базофілія

Моноцитоз

320 / 4299
Під час підйому в гори, на висоті 5000 метрів, у альпіністів з’явилися скарги на задишку, прискорення серцебиття, головний біль, запаморочення, дзвін у вухах. Який патогенетичний фактор визначає розвиток вказаних явищ?

Гіпернатріємія

Зниження кисневої ємності крові

Лактацидемія

Гіпокаліємія

Гіпоксемія

321 / 4299
У хворого на пневмосклероз розвинулася легенева гіпертензія та пра-вошлуночкова серцева недостатність з асцитом та набряками. Який основний патогенетичний механізм розвитку набряків у цього хворого?

Збільшення проникності стінок судин

Збільшення онкотичного тиску міжклітинної рідини

Зменшення осмотичного тиску крові

Збільшення гідростатичного тиску крові у венах

Зменшення онкотичного тиску крові

322 / 4299
У результаті пошкодження одного з реакторів АЕС відбулося витікання радіоактивних продуктів. Люди, які знаходилися в зоні підвищеної радіації, орієнтовно отримали по 250-300 Р Їх негайно госпіталізовано. Які зміни у крові будуть характерні у цей період?

Лейкопенія

Тромбоцитопенія

Нейтропенія

Лімфопенія

Анемія

323 / 4299
Дистрофічні зміни серцевого м’яза супроводжуються розширенням порожнин серця, зниженням сили серцевих скорочень, збільшенням об’єму крові, що залишається під час систоли в порожнині серця; переповнюються вени. Якому стану відповідає описана картина?

Тампонада серця

Тоногенна дилатація

Стадія кардіосклерозу

Міогенна дилатація

Аварійна стадія гіпертрофії міокарда

324 / 4299
У хворого 37 років після отруєння невідомим лікарським препаратом відзначаються стереотипні скорочення м’язів обличчя, що імітують кліпання та примружування. До якої форми розладів рухової функції нервової системи слід віднести описане порушення?

Гіпокінезія

Атаксія

Гіперкінезія

Акінезія

325 / 4299
У хворого на аденому клубочкової зони кори наднирників (хвороба Кон-на) спостерігаються артеріальна гіпертензія, напади судом, поліурія. Що є головною ланкою в патогенезі цих порушень?

Гіперсекреція альдостерону

Гіперсекреція глюкокортикоїдів

Гіпосекреція альдостерону

Гіперсекреція катехоламінів

Гіпосекреція глюкокортикоїдів

326 / 4299
У ліквідатора наслідків аварії на АЕС, під час гострої променевої хвороби, виник геморагічний синдром. Що має найбільше значення в патогенезі цього синдрому?

Підвищення активності протизсі-дальної системи крові

Підвищення активності факторів фібринолізу

Порушення структури стінки судин

Зменшення активності зсідальної системи крові

Тромбоцитопенія

327 / 4299
Після тривалої і тяжкої хвороби у хворого знизився АТ до 60/40 мм рт.ст.; спостерігається тахікардія, задишка, затьмарення свідомості. Як можна визначити цей стан?

Клінічна смерть

Шок

Преагонія

Агонія

328 / 4299
Чоловік 56 років надійшов до клініки зі скаргами на загальну слабкість, біль та печіння в області язика, відчуття оніміння в кінцівках. У минулому переніс резекцію кардіального відділу шлунка. У крові: Hb- 80 г/л; ер.-2,0 * 1012/л; колір. показн.- 1,2; лейк.-3,5 * 109/л. Який вид анемії в даного хворого?

В12-фолієво-дефіцитна

Гемолітична

Апластична

Залізодефіцитна

Постгеморагічна

329 / 4299
Хворий після перенесеного епідемічного паротиту почав худнути, постійно відчував спрагу, пив багато води, відмічалося часте сечовиділення, підвищений апетит. В даний час скаржиться на шкірний свербіж, слабкість, фурункульоз. У крові: глюкоза- 16 ммоль/л, кетонові тіла - 100 мкмоль/л; глюкозурія. Яке захворювання розвинулося у пацієнта?

Стероїдний діабет

Нецукровий діабет

Інсулінонезалежний цукровий діабет

Цукровий діабет недостатнього харчування

Інсулінозалежний цукровий діабет

330 / 4299
Хворому поставлено діагноз ниркової артеріальної гіпертензії. Назвіть ініціальний патогенетичний фактор розвитку артеріальної гіпертензії в даному випадку?

Збільшення синтезу реніну

Гіпернатріємія

Гіперальдостеронізм

!шемія нирок

Збільшення синтезу ангіотензину

331 / 4299
Дитина 7 років захворіла гостро. Під час огляду педіатр відзначив, що слизова оболонка зіву гіперемована, набрякла, вкрита великою кількістю слизу. На слизовій оболонці щік білуваті плями. На наступну добу у дитини з’явилася великоплямиста висипка на шкірі обличчя, шиї, тулуба. Про яке захворювання можна думати?

Кір

Скарлатина

Дифтерія

Алергічний дерматит

Менінгококемія

332 / 4299
Дитина 10 років мешкає в місцевості, де вміст фтору в воді перевищує припустиму норму. Під час обстеження лікарем-стоматологом виявлено пошкодження зубів у вигляді крейдяно-подібних, а також пігментних плям та смуг. Який діагноз є найбільш вірогідним?

Ерозія зубів

Середній карієс

Флюороз

Кислотний некроз твердих тканин зубів

Клиноподібні дефекти

333 / 4299
До лікаря звернувся хворий 53 років зі скаргами на наявність білої бля-тттки на слизовій оболонці язика. Бляшка здіймається над слизовою оболонкою, її поверхня вкрита тріщинами. Мікроскопічно визначається потовщення багатошарового епітелію, паракератоз та акантоз. Який найбільш вірогідний діагноз?

Папілома

Ромбоподібний глосит

Плоскоклітинний рак

Лейкоплакія

Географічний язик

334 / 4299
Під час мікроскопічного дослідження бронхобіопсії виявлена пухлина, яка побудована з гніздних скупчень атипових клітин багатошарового плоского епітелію, місцями із характерними “перлинами' Який найбільш вірогідний діагноз?

Слизовий рак

Плоскоклітинний рак із ороговінням

Плоскоклітинний рак без ороговіння

Солідний рак

Скір

335 / 4299
Гістологічно в усіх шарах апендикса знайдені поліморфноядерні лейкоцити в значній кількості; повнокров’я, стази. Якому захворюванню притаманна така картина?

Флегмонозний апендицит

Хронічний апендицит

Гангренозний апендицит

Простий апендицит

Поверхневий апендицит

336 / 4299
При мікроскопічному дослідженні в препараті виявлено орган нервової системи, який складається з псевдоу-ніполярних нейронів, тіла яких вкриті гліальною та сполучнотканинною оболонками. Визначте даний орган:

Спинний мозок

Мозочок

Вегетативний ганглій

Кора великих півкуль

Спинномозковий вузол

337 / 4299
Під час макроскопічного дослідження судин основи головного мозку хворого, що помер від ішемічного інсульту, виявлено, що інтима судин головного мозку нерівна з помірною кількістю жовтих плям та жовтувато-білуватих бляшок, що звужують просвіт. Який найбільш вірогідний діагноз?

Ревматизм

Вузликовий періартеріїт

Атеросклероз

Цукровий діабет

Гіпертонічна хвороба

338 / 4299
При розтині тіла дитини 5 років, м’які мозкові оболонки різко повнокровні, каламутні, у вигляді жовтувато- зеленуватого «чепчика». Мікроскопічно: м’яка мозкова оболонка різко потовщена, повнокровна, просякнута гнійним ексудатом з наявністю фібрину. Про яке захворювання слід думати?

Грип

Туберкульоз

Кір

Менінгококова інфекція

Сибірка

339 / 4299
У біоптаті дужок м’якого піднебіння, взятого з приводу підозри на пухлину (макроскопічно визначалася виразка з щільним дном), виявлений некроз слизової оболонки з інфільтрацією підслизового шару лімфоцитами, епіте-ліоїдними клітинами, плазматичними клітинами, поодинокими нейтрофілами. Звертає увагу наявність вираженого ендо- та периваскуліту. Для якого захворювання властиві зазначені зміни?

Виразковий стоматит

Виразково-некротичний стоматит Венсана

Первинний сифіліс

Дифтерія зіва

Афтозний стоматит

340 / 4299
Під час морфологічного дослідження дна каріозної порожнини зуба чітко диференціюються три зони: розм’якшеного дентину, прозорого дентину, замісного дентину. Для якої стадії карієсу властиві ці зміни?

Глибокий карієс

Стадія плями

Поверхневий карієс

Середній карієс

Хронічний карієс

341 / 4299
Хвора 65 років раптово померла. Страждала на тромбофлебіт глибоких вен гомілки. На автопсії: у загальному стовбурі та біфуркації легеневої артерії вільно лежать червоні пухкі маси з тьмяною гофрованою поверхнею. Який патологічний процес у легеневій артерії виявив патологоанатом?

Тканинна емболія

Емболія сторонніми тілами

Тромбоемболія

Тромбоз

Жирова емболія

342 / 4299
При мікроскопічному дослідженні біоптату шкіри хворого виявляються гранульоми, які складаються з епітелі-оїдних клітин, оточених у більшості Т- лімфоцитами. Серед епітеліоїдних клітин розташовані поодинокі гігантські багатоядерні клітини типу Пирогова-Лангханса. В центрі деяких гранульом виявляються ділянки казеозного некрозу. Кровоносні судини відсутні. Для якого захворювання характерні описані гранульоми?

Лепра

Сап

Туберкульоз

Риносклерома

Сифіліс

343 / 4299
Під час розтину тіла жінки, яка померла внаслідок пухлинної дисемінації муцинозної цистаденокарциноми та тривалий час мала вимушене положення в ліжку, були знайдені великі ділянки некрозу шкіри та підлеглих м’яких тканин крижової ділянки. Яка форма некрозу має місце у даному випадку?

Сирнистий некроз

Воскоподібний (ценкеровський) некроз

Секвестр

Пролежень

Інфаркт

344 / 4299
Під час електронно-мікроскопічного дослідження слинної залози виявлені фрагменти клітини, які оточені мембраною, містять конденсовані часточки ядерної речовини та окремі орга-нели; запальна реакція, безпосередньо навколо цих клітин, відсутня. Про який процес іде мова?

Каріопікноз

Коагуляційний некроз

Каріолізис

Апоптоз

Каріорексис

345 / 4299
На розтині тіла померлого від черевного тифу, виявлені виразки розташовані вздовж клубової кишки з рівними краями, чистим дном, утвореним м’язовим шаром або навіть серозною оболонкою кишки. Якій стадії хвороби відповідає зазначена картина?

“Чистих” виразок

Некрозу

“Брудних” виразок

Загоєння виразок

Мозкоподібного набухання

346 / 4299
Хворий захворів гостро, скаржився на озноб, підвищення температури тіла до 40оС, головний біль, кашель, за- дишку. На 5-й день захворювання помер. Під час розтину: легені збільшені в об’єму пістрявого вигляду - 'велика пістрява легеня'. Для якого захворювання характерна така патологоанато-мічна картина?

Респіраторно-синцитіальна інфекція

Аденовірусна інфекція

Бронхоектатична хвороба

Крупозна пневмонія

Грип

347 / 4299
У жінки 22 років виявлені збільшені лімфатичні вузли. Гістологічне дослідження вузла виявило лімфоцити, гі-стіоцити, ретикулярні клітини, малі та великі клітини Ходжкіна, багатоядерні клітини Березовського-Штернберга, поодинокі осередки казеозного некрозу. Для якого захворювання характерні такі зміни?

Метастаз рака

Лімфосаркома

Гострий лейкоз

Лімфогранулематоз

Хронічний лейкоз

348 / 4299
На автопсії померлого від отруєння етиленгліколем, нирки дещо збільшені у розмірах, набряклі; їх капсула знімається дуже легко. Кіркова речовина -широка, блідо-сіра. Мозкова речовина - темно-червона. Яка патологія нирок розвинулася у хворого?

Гострий тубуло-інтерстиціальний нефрит

Гострий гломерулонефрит

Некротичний нефроз

Гострий пієлонефрит

Ліпоїдний нефроз

349 / 4299
Під час гістологічного дослідження легень померлого від серцевої недостатності, виявлені вогнища запалення з заповненням альвеол рідиною, забарвленою у блідорожевий колір, місцями з наявністю тонких рожевих ниток, які утворюють дрібнопетлисту сітку з невеликою кількістю лімфоцитів. Який характер ексудату в легенях?

Геморагічний

Серозний

Серозно-фібринозний

Фібринозний

Гнійний

350 / 4299
Дитина 7 років надійшла до інфекційного відділення зі скаргами на різкий біль у горлі при ковтанні, підви- щення температури тіла до 390С, набряк шиї. Об’єктивно: мигдалики збільшені, їх слизова оболонка повнокровна, вкрита великою кількістю плівок білуватожовтого кольору, які щільно прилягають до слизової оболонки. При спробі зняти плівку залишається глибокий дефект, що кровоточить. Який вид запалення має місце?

Крупозне

Дифтеритичне

Серозне

Гнійне

Геморагічне

351 / 4299
Дитині 4-х років зроблено реакцію Манту. Через 60 годин після введення у шкіру туберкуліну з’явилося вогнищеве затвердіння та почервоніння шкіри діаметром 15 мм, що було розцінено як позитивний тест. Який вид реакції гі-перчутливості лежить в основі цього тесту?

Імунокомплекс-опосередкована гіперчутливість

Гіперчутливість негайного типу

Комплімент-опосередкована цитотоксична гіперчутливість

Гіперчутливість сповільненого типу

352 / 4299
На розтині тіла хлопчика 8 років, що хворів на дифтерію зіва та мигдаликів і помер на другий тиждень від початку захворювання, виявлено зміни в міокарді у вигляді дрібновогнищевих некрозів міокардиоцитів, набряку строми з незначною лімфоцитарною інфільтрацією. Діагностуйте вид міокардиту:

Інтерстиціальний

Септичний

Вогнищево-проміжний, ексудативний

Гранулематозний

Альтеративний

353 / 4299
Вміст везикул зі слизової оболонки хворого на натуральну віспу направлений до вірусологічної лабораторії. Що з наведеного буде виявлено під час мікроскопії мазків?

Синцитій

Тільця Гуарнієрі

Тільця Пашена

Тільця Бабеша-Негрі

Тільця Бабеша-Ернста

354 / 4299
Під час гістологічного дослідження слизової оболонки матки знайдені звивисті залози, пилко- та штопоропо- дібні, подовжені розростанням строми з проліферацією її клітин. Який діагноз?

Пухирний занос

Плацентарний поліп

Залозиста гіперплазія ендометрію

Лейоміома

Гострий ендометрит

355 / 4299
До лабораторії надійшов матеріал (витяжка тваринницької сировини) з району, де відзначаються випадки сибірки серед тварин. Яку серологічну реакцію необхідно застосувати для виявлення антигенів збудника в досліджуваному матеріалі?

Непрямої гемаглютинації

Зв’язування комплементу

Преципітації в агарі

Радіоімунний аналіз

Термопреципітації

356 / 4299
У бактеріологічній лабораторії необхідно простерилізувати поживні середовища, що містять речовини, які змінюються при температурі вище 100оС (сечовина, вуглеводи). Який спосіб стерилізації має обрати лаборант?

Парою під тиском у автоклаві

Тиндалізація

Пастеризація

Текучою парою, дрібно

Кип’ятіння

357 / 4299
Хворий був доставлений до лікарні зі скаргами на головний біль, підвищену температуру, часті випорожнення, біль у животі з тенезмами. Лікар встановив клінічний діагноз дизентерія та направив досліджуваний матеріал (випорожнення) до баклабораторії. Яким методом діагностики лікар-лаборант має підтвердити або відхилити клінічний діагноз?

Алергічний

Біологічний

Бактеріоскопічний

Серологічний

Бактеріологічний

358 / 4299
У пацієнта з бронхіальною астмою за допомогою шкірних алергічних проб встановлено сенсибілізацію алергеном тополиного пуху. Який фактор імунної системи відіграє вирішальну роль у розвитку цього імунопатологічного стану?

IgE

IgM

Сенсибілізовані Т-лімфоцити

IgD

359 / 4299
У баклабораторії, під час мікроскопії мазків з харкотиння хворого на хронічне легеневе захворювання, забарвлених за Цілем-Нільсоном, виявлені червоні палички. Яка властивість мікобактерії туберкульозу виявлена при цьому?

Спиртостійкість

Спороутворення

Капсулоутворення

Лугостійкість

Кислотостійкість

360 / 4299
Хворий на сифіліс пройшов курс антибіотикотерапії та повністю вилікувався. Через деякий час він знову був інфікований Treponema pallidum. Як називається така форма інфекції?

Ускладнення

Вторинна інфекція

Реінфекція

Суперінфекція

Рецидив

361 / 4299
До інфекційної клініки надійшла дівчинка 7 років з високою температурою, скаргами на біль у горлі, загальну слабкість. Лікар запідозрив дифтерію. Що із перерахованого є вирішальним для підтвердження діагнозу після виділення чистої культури збудника?

Проба на цистиназу

Проба на токсигенність

Виявлення у збудника волютинових зерен

Гемолітична здатність збудника

Фаголізабельність

362 / 4299
Збудниками багатьох гнійно-запальних процесів ротової порожнини є анаероби. Яке з перерахованих поживних середовищ можна використати для контролю контамінації перев’язувального матеріалу анаеробами?

Ендо

Ру

Плоскірєва

Кітта-Тароцці

Сабуро

363 / 4299
Під час мікроскопічного дослідження біоптату з ураженої ділянки слизової оболонки ротової порожнини виявлені палички, розташовані у вигляді скупчень, що нагадують пачку цигарок. За Цілем-Нільсеном фарбуються в червоний колір. Який вид збудника, найбільш вірогідно, виявлений у біо-птаті?

M.avium

M.leprae

A.bovis

A.israilii

M.tuberculosis

364 / 4299
З гнійного ексудату хворого з одонтогенною флегмоною виділена чиста культура грампозитивних мікробів кулястої форми, що має лецитина-зну активність, коагулює плазму кроля, розщеплює маніт в анаеробних умовах. Який з вказаних мікроорганізмів міг сприяти виникненню гнійного ускладнення?

S.epidermidis

S.pyogenes

S.mutans

S.aureus

S.viridans

365 / 4299
У хворого виділено чисту культуру коринебактерій дифтерії. Яку імунологічну реакцію слід використати для виявлення токсигенності бактерій?

Преципітації в агарі

Зв’язування комплементу

Гальмування гемаглютинації

Аглютинації

Непрямої гемаглютинації

366 / 4299
До інфекційної лікарні надійшов ветеринарний лікар з підозрою на бруцельоз. За допомогою якого серологічного тесту можна підтвердити діагноз?

Реакція аглютинації Райта

Реакція зв’язування комплементу Васермана

Реакція аглютинації Відаля

Реакція аглютинації Вейгля

Реакція преципітації Асколі

367 / 4299
Хворому на інсулінзалежний цукровий діабет було введено інсулін. Через деякий час у нього з’явилися слабкість, дратівливість, посилення потовиділення. Яка основна причина розвитку цих розладів?

Зменшення глюконеогенезу

Посилення ліпогенезу

Вуглеводне голодування головного мозку

Посилення глікогенолізу

Посилення кетогенезу

368 / 4299
З віком знижується активність нав-коловушних слинних залоз. Активність якого ферменту буде зменшуватися в слині?

Гексокіназа

Амілаза

Фосфатаза

Мальтаза

Лізоцим

369 / 4299
У збудливій клітині заблокували іонні канали. Це не змінило суттєво величину потенціалу спокою, але клітина втратила здатність до генерації ПД. Які канали заблокували?

Кальцієві

Калієві

Натрієві та калієві

Натрієві

Хлорні

370 / 4299
У хворого 25 років у стоматологічному кабінеті раптово розвинувся напад бронхіальної астми. Лікар дав хворому сальбутамол у вигляді інгаляції. Який механізм дії цього препарату?

Стимулює а-адренорецептори

Стимулює ^2-адренорецептори

Блокує Н !-гістамінорецептори

Блокує фосфодіестеразу

Блокує М-холінорецептори

371 / 4299
У хворого 30 років, який потрапив до клініки з діагнозом гострий гломерулонефрит, спостерігається протеїнурія. Яке порушення спричинило це явище?

Затримка виведення продуктів азотистого обміну

Підвищення гідростатичного тиску крові у капілярах

Підвищення проникності ниркового фільтра

Зниження онкотичного тиску плазми крові

Зменшення кількості функціонуючих нефронів

372 / 4299
У пацієнта в стоматологічному кабінеті розвинувся напад бронхіальної астми, який вдалося усунути сальбута-молом. До якої групи лікарських засобів належить цей препарат?

вгв2-адреноміметики

а-в-адреноміметики

в2-адреноміметики

а-адреноміметики

Симпатолітики

373 / 4299
У пацієнта, після застосування новокаїну для проведення провідникової анестезії, розвинувся анафілактичний шок. Який із наведених препаратів є препаратом вибору для його усунення?

Норадреналіну гідротартрат

Атропіну сульфат

Дімедрол

Кордіамін

Адреналіну гідрохлорид

374 / 4299
Хворому на стоматит призначили препарат з групи сульфаніламідів. Який механізм його антибактеріальної дії?

Зменшення проникності мембран

Коагуляція білка

Конкурентний антагонізм з ПАБК

Порушення синтезу білків клітинної стінки

Пригнічення сульфгідрильних груп тіолових ферментів

375 / 4299
Під час огляду ротової порожнини пацієнта лікар-стоматолог визначив сухість слизової оболонки, численні ерозії. Недостатність якого вітаміну спричинила ці явища?

Вітамін А

Вітамін Н

Вітамін К

Вітамін Р

Вітамін РР

376 / 4299
Хворому з гінгівітом після використання аплікацій призначили полоскання з препаратом, антисептичні властивості якого забезпечуються атомарним киснем, що відщеплюється в присутності органічних речовин. Виявляє також дезодоруючу, в’яжучу (протизапальну), а в більших концентраціях - припікаючу дію. Водні розчини використовуються для промивання ран, полоскання рота, а в більш високих концентраціях - для лікування опіків. Застосовується для промивання шлунка при отруєннях. Визначте препарат:

Перекис водню

Натрію гідрокарбонат

Хлоргексидину біглюконат

Калію перманганат

Спирт етиловий

377 / 4299
Для лікування пародонтиту в комплекс препаратів був включений лікарський засіб з групи водорозчинних вітамінів, похідне біофлавоноїдів, який призначають разом з кислотою аскорбіновою. Препарат має антиоксидантні властивості, зменшує кровоточивість ясен. Який це препарат?

Ціанокобаламін

Кальцію пангамат

Рутин

Кислота фолієва

Кальцію пантотенат

378 / 4299
У стоматологічному кабінеті в хворого розвинувся напад бронхіальної астми. Лікар застосував препарат з групи в-адреноміметиків. Який препарат був застосований?

Ефедрину гідрохлорид

Сальбутамол

Адреналіну гідрохлорид

Атропіну сульфат

Еуфілін

379 / 4299
Хворий з діагнозом вогнищевий туберкульоз верхньої частки правої легені в складі комбінованої терапії отримує ізоніазид. Через деякий час пацієнт почав скаржитися на м’язову слабкість, зниження шкірної чутливості, порушення зору, координації рухів. Який вітамінний препарат доцільно використати для усунення даних явищ?

Вітамін А

Вітамін В12

Вітамін D

Вітамін C

Вітамін В6

380 / 4299
У хворого після довготривалого застосування препарату, призначеного для лікування гострого респіраторного захворювання, з’явилися головний біль, запаморочення, шум у вухах, нудота, біль в епігастральній області. Який препарат міг викликати подібну клінічну картину?

Нафтизин

Бромгексин

Вітамін С

Мідантан

Ацетилсаліцилова кислота

381 / 4299
У клітині відбулася мутація першого екзону структурного гена. В ньому зменшилася кількість пар нуклеотидів - замість 290 пар стало 250. Визначте тип мутації:

Транслокація

Нонсенс-мутація

Дуплікація

Інверсія

Делеція

382 / 4299
Хворий тривало хворіє на туберкульоз. Який препарат обов’язково має бути включений до комплексної терапії туберкульозу?

Етіонамід

Натрію парааміносаліцилат

Етамбутол

Ізоніазид

Рифампіцин

383 / 4299
У пацієнтки 46 років на фоні миготливої аритмії почався набряк легень. Який сечогінний препарат необхідно ввести в першу чергу для виведення хворої з важкого стану?

Еуфілін

Тріамтерен

Амілорид

Фуросемід

Верошпірон

384 / 4299
Хворому з неоперабельним раком шлунка призначили промедол для зняття вираженого больового синдрому. З часом хворий відзначив зменшення знеболюючого ефекту та тривалості дії препарату, різке посилення болю у всьому тілі. Яким є механізм зниження ефективності промедола?

Звикання

Тахіфілаксія

Кумуляція

Психічна залежність

Зменшення реабсорбції промедолу в канальцях нирок

385 / 4299
У хворого після екстракції зуба виникла кровотеча. Який із запропонованих засобів доцільно використати у цьому випадку?

Вікасол орально

Неодикумарин орально

Адреналін місцево

Фібриноген ін’єкційно

Тромбін ін’єкційно

386 / 4299
Хворому, який лікується тетрацикліном, лікар порадив не вживати молочні продукти. Чому лікар дав таку рекомендацію?

Збільшується ризик дисбактеріозу

Зростає токсичність антибіотика

Порушується процес перетравлення їжі

Сповільнюється всмоктування антибіотика

Не засвоюються молочні продукти

387 / 4299
Хворому 50 років з ішемічною хворобою серця лікар призначив лікарський засіб, який має антиагрегантну дію. Хворий почав приймати засіб у дозах, що перевищують призначені. У нього розвилися нудота, блювання, біль у животі натщесерце. Який засіб було призначено хворому?

Пентоксифілін

Тиклід

Ацетилсаліцилова кислота

Дипіридамол

Пармідин

388 / 4299
Собаці в експерименті подразнювали на шиї периферичний відрізок перерізаного блукаючого нерва. При цьому спостерігали таку зміну серцевої діяльності:

Збільшення частоти та сили скорочень

Зменшення частоти скорочень

Збільшення швидкості проведення збудження по міокарду

Збільшення сили скорочень

Збільшення збудливості міокарда

389 / 4299
Відомо, що у людей з генетично обумовленою недостатністю глюкозо-6- фосфатдегідрогенази еритроцитів, у відповідь на призначення деяких протималярійних препаратів може розвиватися гемоліз еритроцитів. Як називається цей тип реакції на лікарські засоби?

Ідіосинкразія

Толерантність

Алергія

Тахіфілаксія

Сенсибілізація

390 / 4299
Під час огляду ротової порожнини стоматолог виявив появу у дитини перших великих нижніх кутніх зу6ів. Який вік дитини?

4-5 років

12-13 років

10-11 років

8-9 років

6-7 років

391 / 4299
У хворого після оперативного втручання розвинувся парез кишечнику. Який засіб із групи антихолінестера-зних слід йому призначити?

Прозерин

Карбахолін

Пілокарпін

Ацетилхолін

Ацеклідин

392 / 4299
При гострому тромбозі показана антикоагулянтна терапія. Який антикоагулянт прямої дії застосовується при загрозі тромбозу?

Гепарин

Фенілін

Дипіридамол

Пентоксифілін

Натрію цитрат

393 / 4299
Під час огляду учнів першого класу стоматолог виявив, що у однієї дитини зуби жовтокоричневого кольору, два зуба надщерблені. До цього хлопчик лікувався від пневмонії, назву таблеток згадати не може. Який препарат міг так негативно вплинути на зуби?

Бісептол

Ампіцилін

Доксициклін

Еритроміцин

Оксацилін

394 / 4299
У хворого повільно загоюється перелом кістки. Який засіб можна використати для прискорення утворення її сполучнотканинної матриці?

Циклофосфан

Метилурацил

Циклоспорин

Преднізолон

Метотрексат

395 / 4299
У хворого на гострий міокардит з’явилися клінічні ознаки кардіогенно-го шоку. Який із вказаних нижче патогенетичних механізмів є провідним у розвитку шоку?

Порушення насосної функції серця

Збільшення судинного тонусу

Зниження діастолічного притоку крові до серця

Депонування крові у венах

Зниження судинного тонусу

396 / 4299
Каретою швидкої допомоги до лікарні доставлено хворого, який у стані важкої депресії намагався покінчити життя самогубством. Які фармакологічні засоби необхідно призначити хворому?

Транквілізатори

Антидепресанти

Седативні

Солі літію

Нейролептики

397 / 4299
У хворого на гіпертонічну хворобу розвинулася гостра серцева недостатність. Який з наведених засобів найбільш доцільно використати в даному випадку?

Дігоксин

Кофеїн

Корглікон

Кордіамін

Кардіовален

398 / 4299
До стоматолога звернулася мати дитини 2-х років, яка під час вагітності, у зв’язку з інфекційним захворюванням, безсистемно приймала антибіотики. Під час огляду у дитини відзначається руйнування різців, емаль жовтого кольору, у шийки зубів коричнева облямівка. Який з препаратів приймала мати під час вагітності?

Ампіокс

Доксициклін

Октадин

Фуросемід

Ксантинола нікотинат

399 / 4299
У хворого відзначається різка болючість шкіри в ділянці обличчя. Який нерв уражено?

Язикоглотковий

Трійчастий

Блукаючий

Окоруховий

Лицевий

400 / 4299
У хворого на запалення трійчастого нерва, останніми роками прогресує пародонтит. Який фактор є головним у розвитку пародонтиту?

Нейродистрофічні порушення

Пвдвищення тонусу блукаючого нерва

Зменшення активності лейкоцитарної еластази

Зниження активносгі калiкреїн-кінінової системи

Послаблення утворення імуногло6у-лінів

401 / 4299
Під час мікроскопії клітин серцевого м’яза людини знайдеш органели овальної форми, оболонка яких утворена двома мембранами: зовнішня - гладенька, а внутрішня утворює кристи. Біохімічно встановлена наявність ферменту АТФ-синтетази. Які органели досліджувалися?

Мітохондрії

Ендоплазматичний ретикулюм

Лізосоми

Рибосоми

Центросоми

402 / 4299
При медичному огляді юнаків у деяких під пахвами були виявлені комахи розміром 1,0-1,5 мм сірого кольору, з коротким широким тілом, що вкрито волосками. Які комахи були виявлені?

Головна воша

Коростяний кліщ

Блоха

Постільна блошиця

Лобкова воша

403 / 4299
Відомі трисомна, транслокаційна та мозаїчна форми синдрому Дауна. За допомогою якого методу генетики людини можна диференціювати названі форми синдрому Дауна?

Генеалогічний

Цитогенетичний

Близнюковий

Популяційно-статистичний

Біохімічний

404 / 4299
Дуже крупні зуби - ознака, зчеплена з Y-хромосомою. У матері зуби нормальної величини, а у її сина - дуже крупні. Вірогідність наявності дуже крупних зубів у батька складає:

50%

75%

12,5%

100%

25%

405 / 4299
У дитини 8-ми місяців виявлено неза-рощення піднебіння, цілий ряд дефектів з боку очей, мікроцефалія, порушення серцево-судинної системи. Цитогенети-чні дослідження виявили 47 хромосом з наявністю додаткової 13-ої хромосоми. Який діагноз можна встановити на підставі клінічних спостережень і цито-генетичних досліджень?

Синдром Клайнфельтера

Синдром Едвардса

Синдром Дауна

Синдром 'котячого крику'

Синдром Патау

406 / 4299
До хірургічного відділення лікарні надійшов хворий з підозрою на абсцес печінки. Хворий тривалий час знаходився у відрядженні в одній з африканських країн і неодноразово хворів на гострі шлунково-кишкові розлади. Яке протозойне захворювання може бути в хворого?

Амебіаз

Лейшманіоз

Токсоплазмоз

Трипаносомоз

Малярія

407 / 4299
Під час обстеження дитини 7-ми років виявлено наступні ознаки: низький зріст, широке округле обличчя, близько розміщені очі із вузькими очними щілинами, напіввідкритий рот. Діагностовано також ваду серця. Ці клінічні ознаки найбільш характерні для хвороби Дауна. Вкажіть причину даної патології:

Трисомія 21-ої хромосоми

Трисомія 13-ої хромосоми

Нерозходження статевих хромосом

Часткова моносомія

Трисомія за Х-хромосомою

408 / 4299
Серед студентів однієї групи присутні представники різних рас. Один з студентів має пряме чорне волосся та нависаючу шкірну складку верхньої повіки - епікантус. Представником якої раси, найвірогідніше, є цей студент?

Монголоїдна

Ефіопська

Австралоїдна

Європеоїдна

Негроїдна

409 / 4299
У вагітної жінки, яка вживала алкоголь, порушилася закладка ектодерми в ембріональний період. В яких похідних цього листка розвинулися вади?

Печінка

Нервова трубка

Статеві залози

Епітелій кишечнику

Нирки

410 / 4299
У генетичній лабораторії під час роботи з молекулами ДНК білих щурів лінії Вістар замінили один нуклеотид на інший. При цьому отримали заміну лише однієї амінокислоти у пептиді. Такий результат буде наслідком наступної мутації:

Дуплікація

Трансверсія

Зміщення рамки зчитування

Транслокація

Делеція

411 / 4299
На ринку батько купив свинину. Якою хворобою можуть заразитися члени сім’ї, якщо це м’ясо не пройшло ветеринарний контроль?

Теніоз

Гіменолепідоз

Фасціольоз

Теніаринхоз

Ехінококоз

412 / 4299
У немовляти виявлено мікроцефалію. Лікарі вважають, що це пов’язано із застосуванням жінкою під час вагітності актиноміцину Д. На який зародковий листок подіяв цей тератоген?

Ектодерма

Ентодерма та мезодерма

Ентодерма

Усі листки

Мезодерма

413 / 4299
При обстеженні лікарями санітарно-епідеміологічної станції працівників сфери громадського харчування нерідко виявляється безсимптомне паразитоносійство, коли клінічно здорова людина є джерелом цист, які за-ражують інших людей. Для паразитування якого збудника неможливе таке явище?

Дерматотропні лейшманії

Вісцеротропні лейшманії

Кишкова трихомонада

Малярійний плазмодій

Дизентерійна амеба

414 / 4299
Відомо, що інформацію про послідовність амінокислот у молекулі білка записано у вигляді послідовності чотирьох видів нуклеотидів у молекулі ДНК, причому різні амінокислоти кодуються різною кількістю триплетів -від одного до шести. Як називається така особливість генетичного коду?

Неперекривність

Триплетність

Виродженість

Специфічність

Універсальність

415 / 4299
У клітинах усіх організмів присутні безмембранні органели, що складаються з двох неоднакових за розміром частинок. Вони мають мікроскопічний розмір та беруть участь у синтезі білків. Як називаються ці органели?

Клітинний центр

Лізосоми

Мітохондрії

Комплекс Гольджі

Рибосоми

416 / 4299
57-ми річна хвора на гіпертонію звернулася до лікаря з скаргою на біль великого пальця руки. При огляді лікар констатував зниження пульсації, блідість шкіри та зниження температури у цій ділянці. Яка з артерій верхньої кінцівки найвірогідніше ушкоджена?

A.perforans

A.digitalis communis

A.ulnaris

A.digitalis propria

A.princeps pollicis

417 / 4299
Під час виконання вагосимпати-чної блокади за Вишневським вводять розчин новокаїну по задньому краю груднино-ключично-соскоподібного м’яза вище місця перетинання її з зовнішньою яремною веною. У межах якого трикутника шиї виконують блокаду?

Лопатково-трапецієподібний

Сонний

Ключично-лопатковий

Піднижньощелепний

Трикутник Пирогова

418 / 4299
Під час підходу до щитоподібної залози з поперечного (коміроподібно-го) доступу розтинається клітковинний надгруднинний простір. Пошкодження якого анатомічного утворення, що знаходиться в даному просторі, є небезпечним?

Лімфатичні вузли

Сонна артерія

Підключична артерія

Внутрішня яремна вена

Венозна яремна дуга

419 / 4299
Під час огляду ротової порожнини у пацієнта спостерігається невелика щі- лина між верхніми та нижніми різцями. Контакту між передніми зубами немає. Якому прикусу притаманне зазначене розміщення зубів?

Прогенія

Ортогнатія

Закритий прикус

Прямий прикус

Відкритий прикус

420 / 4299
Хворий 24-х років звернувся до лікаря зі скаргою на біль під нижньою щелепою з правого боку. Хірург-стоматолог виявив камінь у піднижньо-щелепній залозі. Видаляючи його, він запобігав кровотечі з артерії:

A.lingualis

A.labialis inferior

A.submentalis

A.alveolaris inferior

A.facialis

421 / 4299
Чоловік 28-ми років з різаною раною шкіри лобової ділянки був доставлений до лікарні. Для зупинки кровотечі перев’язана судина, яка кровопоста-чає цю ділянку. Яка судина була перев’язана?

A.temporalis superf icialis

A.supraorbitalis

A.angularis

A.dorsalis nasi

A.infraorbitalis

422 / 4299
Хворий скаржиться на головний біль, утруднене дихання. Рентген підтвердив діагноз - фронтит. В якому носовому ході при огляді порожнини носа можуть спостерігатися гнійні виділення?

Середній

Над верхньою носовою раковиною

Загальний

Верхній

Нижній

423 / 4299
У хворого 28-ми років діагностовано гострий запальний процес слизової оболонки нососльозової протоки. З анамнезу відомо, що після перенесеного грипу протягом 10-ти днів були виділення із носа. З якого відділу носової порожнини інфекція могла потрапити до нососльозової протоки?

Лобова пазуха

Середній носовий хід

Присінок носа

Верхній носовий хід

Нижній носовий хід

424 / 4299
Після травми ока виникло нагноєння м’яких тканин орбіти. Через який анатомічний утвір гнійний процес може поширитися у середню черепну ямку?

Через задній решітчастий отвір

Через верхню очноямкову щілину

Через вилично-очноямковий отвір

Через передній решітчастий отвір

Через нижню очноямкову щілину

425 / 4299
У хворого під час висування язика спостерігається відхилення його кінчика ліворуч. Рухова іннервація якого черепного нерва порушена в цьому випадку?

N.glossopharyngeus dexter

N.trigeminus sinister

N.hypoglossus dexter

N.facialis sinister

N.vagus dexter

426 / 4299
При травмі потиличної ділянки визначається тріщина в ділянці поперечного синуса. Яка частина потиличної кістки пошкоджена?

Луска

Права бокова

Ліва бокова

Виросток

Основна

427 / 4299
Хірург планує взяти у хворого лімфу з грудної протоки в місці її впадіння до венозного русла. Куди слід провести катетер з цією метою?

Місце утворення верхньої порожнистої вени

Місце утворення нижньої порожнистої вени

Місце утворення ворітної вени

Правий венозний кут

Лівий венозний кут

428 / 4299
У хворого 69-ти років при гнійній інфекції носової порожнини виник абсцес лобової частки мозку. Через яке анатомічне утворення поширилася інфекція?

Foramen ovale

Foraminae cribrosae

Foramen rotundum

Foramen sphenopalatinum

Foramen ethmoidalae posterior

429 / 4299
У наслідок травми в ділянці вінцевого відростка нижньої щелепи в хворого спостерігається порушення зміщення нижньої щелепи назад. Пошкодження якого м’яза найбільш вірогідно?

M.temporalis

M.masseter

M.levator anguli oris

M.pterygoideus medialis

M.pterygoideus lateralis

430 / 4299
У хворого після простудного захворювання виникло порушення виділення сльози. Який вегетативний вузол найбільше при цьому постраждав?

Піднижньощелепний

Вушний

Під’язиковий

Війчастий

Крилопіднебінний

431 / 4299
Жінці 58-ми років проведено повне видалення матки з придатками, після чого виділення сечі припинилося. При цистоскопії: міхур сечі не містить, з устій сечоводів сеча не надходить. Який відділ сечовидільної системи було ушкоджено під час операції?

Uretra

Pelvis renalis

Ren

Ureter

V esica urinaria

432 / 4299
При огляді хворого 32-х років відзначається диспропорційний ріст скелету, збільшення надбрівних дуг, носа, губ, язика, щелепних кісток, стоп. Функція якої залози порушена?

Гіпофіз

Надниркова

Щитоподібна

Епіфіз

Підшлункова

433 / 4299
У хворого запалення середнього вуха (отит). При цьому він скаржиться на розлад відчуття смаку в передній частині язика. Який з нервів ушкоджено?

N.facialis

N.glossopharyngeus

N.vestibulo — cochlearis

N.vagus

N.trigeminus

434 / 4299
Під час оперативного втручання у черевну порожнину хірургу необхідно проникнути до чепцевої сумки. Як може проникнути хірург до цієї частини порожнини очеревини, не порушуючи цілісність малого чіпця?

Через правий брижовий синус

Через праву біляободову борозну

Через лівий брижовий синус

Через ліву біляободову борозну

Через чепцевий отвір

435 / 4299
Дитина 5-ти років страждає на деформацію шиї. При клінічному обстеженні виявлено такі симптоми: виражений нахил голови вліво, обличчя повернене праворуч, пасивні рухи голови в правий бік обмежені. Порушення розвитку якого м’яза мало місце при цій патології?

Ремінний м’яз голови

Груднино-ключично-соскоподібний

Груднино-під’язиковий

Довгий м’яз голови

Трапецієподібний

436 / 4299
У хворого утруднене щільне змикання щелеп під час жування. Відзначається часткова атрофія жувальних м’язів, що розташовані нижче виличної дуги. Гілки якого нерва іннервують вказані м’язи?

N.maxillaris

N.alveolaris inferior

N.mandibularis

Nn.alveolares superiores

N.infraorbitalis

437 / 4299
У боксера, що отримав удар у ділянку скронево-нижньощелепного суглоба, відбувся травматичний вивих нижньої щелепи. Зміщення яких суглобових поверхонь вийде за межі фізіологічної норми?

Голівка нижньої щелепи та нижньощелепна ямка

Вінцєвий відросток та піднижньоще-лепна ямка

Голівка нижньої щелепи та підни-жньощелепна ямка

Шийка нижньої щелепи та підни-жньощелепна ямка

Вінцевий відросток та крилоподібна ямка

438 / 4299
У хворого запалення середнього вуха ускладнилося мастоїдитом. Виникла загроза гнійного тромбозу найближчої венозної пазухи. Якої саме?

Верхня сагітальна

Сигмоподібна

Поперечна

Нижня кам’яниста

Пряма

439 / 4299
У постраждалого в дорожній пригоді лікар виявив перелом лівої ключиці та порушення кровообігу в кінцівці (немає пульсації променевої артерії). Яка з причин порушення кровообігу в кінцівці найбільш вірогідна?

Стиснення пахвової артерії

Стиснення підключичної вени

Стиснення підключичної артерії

Стиснення хребтової артерії

Стиснення пахвової вени

440 / 4299
Хворому 60-ти років важко формувати та рухати харчову грудку, це порушує процес їжі. При цьому язик нерухомий, мова стала неможливою. Який нерв при цьому пошкоджено?

V

VII

XI

IX

XII

441 / 4299
До лікарні госпіталізовано жінку в важкому стані з діагнозом геморагічний інсульт у ділянці лобної частки правої півкулі головного мозку. Утттко-дження якої артерії, найбільш вірогідно, призвело до цього стану?

A.cerebri anterior

A.cerebri posterior

A .communicans posterior

A.communicans anterior

A.cerebri media

442 / 4299
Хворий 35-ти років звернувся до приймального покою зі скаргами на біль та набряк у ділянці дна ротової порожнини. Після огляду діагностовано запальний процес у ділянці вивідної протоки піднижньощелепної слинної залози. Куди відкривається ця протока?

Caruncula sublingualis

Vestibulum oris

Plica fimbriata

Foramen caecum linguae

Recesus gingivalis

443 / 4299
При обстеженні хворого, який зазнав дії іонізуючого випромінювання, виявлено пошкодження білої пульпи селезінки. Які клітини білої пульпи зазнають патологічних змін?

Нейтрофільні лейкоцити

Лімфоцити

Тканинні базофіли

Базофільні лейкоцити

Моноцити

444 / 4299
У хворого внаслідок травми ушкоджені передні корінці спинного мозку. Вкажіть, які структури при цьому постраждали:

Периферійні відростки нейронів спинномозкових вузлів

Аксони нейронів бокових рогів

Аксони мотонейронів

Аксони мотонейронів та нейронів бокових рогів

Центральні відростки нейронів спинномозкових вузлів

445 / 4299
В результаті травми голови у чоловіка 32-х років ушкодженні ампули півколових каналів. Сприйняття яких подразнень буде порушено внаслідок цього?

Вібрація та гравітація

Гравітація

Вібрація

Лінійне прискорення

Кутове прискорення

446 / 4299
Під час морфологічного дослідження в гістопрепараті біопсійного матеріалу визначається судина неправильної форми, середня оболонка якої утворена пучками гладеньких міоцитів та прошарками сполучної тканини. Вкажіть вид цієї судини:

Лімфатична судина

Артеріола

Венула

Вена м’язового типу

Артерія м’язового типу

447 / 4299
На електронній мікрофотографії представлена клітина, в якій відсутні ядерця та ядерна оболонка. Хромосоми вільно розміщені, центріоли мігрують до полюсів. У якій фазі клітинного циклу знаходиться клітина?

Профаза

!нтерфаза

Анафаза

Метафаза

Телофаза

448 / 4299
У гістологічному препараті трубчастої кістки, на місці зламу виявлені ознаки регенераторного процесу (мозоля). Яка тканина формує цю структуру?

Епітеліальна

Ретикулярна

Грубоволокниста кісткова

Пластинчаста кісткова

Пухка сполучна

449 / 4299
Після перенесеного хімічного опіку стравоходу відбулося локальне його звуження внаслідок утворення рубця. Які клітини пухкої сполучної тканини беруть участь в утворенні рубців?

Фіброкласти

Міофібробласти

Юні малоспеціалізовані фібробласти

Зрілі спеціалізовані фібробласти

Фіброцити

450 / 4299
У жінки має місце гіперемія яєчника, підвищення проникності гематофо-лікулярного бар’єру з послідовним розвитком набряку, інфільтрація стінки фолікула сегментоядерними лейкоцитами. Об’єм фолікула великий, стінка його потоншена. Якому періоду статевого циклу відповідає зазначена картина?

Постменструальний період

Менструальний період

Період відносного спокою

Предовуляторна стадія

Овуляція

451 / 4299
У недоношених дітей розвивається синдром дихальної недостатності. Недостатність якого компоненту аероге-матичного бар’єру лежить в основі цієї патології?

Альвеолоцити

Базальна мембрана альвеолоцитів

Ендотелій капілярів

Сурфактант

Базальна мембрана ендотелію

452 / 4299
В стінці фолікулів та у міжфоліку-лярних прошарках сполучної тканини щитоподібної залози розміщуються великі ендокриноцити, секреторні гранули яких осміо- і аргірофільні. Назвіть ці клітини:

Пітуїцити

Паратироцити

Пінеалоцити

Кальцитоніноцити

Тироцити

453 / 4299
Під час гістологічного дослідження в ділянці шийки власної залози шлунка виявляються дрібні клітини, що мають високе ядерно-плазматичне відношення та базофільну цитоплазму. Вкажіть функцію даних клітин:

Регенерація залозистого епітелію

Секреція пепсиногену

Захисна

Секреція іонів хлору

Ендокринна

454 / 4299
При гістологічному дослідженні поперечного шліфу емалі виявлена лінійна посмугованість у вигляді концентричних кругів, яка направлена під кутом до дентиноемалевого з’єднання. Назвіть ці структури:

Смуги Іунтера-Шрегера

Лінії Ретціуса

Емалеві пучки

Емалеві пластини

Емалеві веретена

455 / 4299
Для вирішення питання ретроспективної діагностики перенесеної бактеріальної дизентерії було призначене серологічне дослідження сироватки крові з метою встановлення титру антитіл до шигел. Яку з перерахованих реакцій доцільно використати для цього?

Пасивної гемаглютинації

Преципітації

Бактеріолізу

Гемолізу

Зв’язування комплементу

456 / 4299
На мікропрепараті шкіри пальця дитини спостерігаємо, що епідерміс має ознаки недостатнього розвитку. Який ембріональний листок був пошкодже- ний у процесі розвитку?

Мезодерма

Ектомезенхiма

Мезенхiма

Ектодерма

Ентодерма

457 / 4299
Досліджується гістологічний препарат слинних залоз, в якому окрім білкових та змішаних кінцевих відділів визначаються слизові. Яка слинна залоза досліджується?

Підщелепна

Привушна

Під’язикова

Щічна

Іубна

458 / 4299
При дії на організм несприятливих факторів у тимусі має місце перебудова органу, що супроводжується масовою загибеллю тимоцитів, виселенням їх у периферійні органи, проліферація епітеліоретикулоцитів. Як зветься таке явище?

Вікова інволюція тимуса

Гіпотрофія тимуса

Дистрофія тимуса

Атрофія тимуса

Акцидентальна інволюція тимуса

459 / 4299
В гістологічному препараті органу ротової порожнини видно, що передня поверхня вистелена багатошаровим плоским незроговілим епітелієм, а задня поверхня - багаторядним війчастим епітелієм. Що це за орган?

М’яке піднебіння

Щока

Тверде піднебіння

Ясна

Іуба

460 / 4299
У пацієнта 42-х років, що страждає на парадонтоз, у коронковій частині пульпи виявлені округлі звапновані утворення діаметром 2-3 мм. Назвіть ці структури:

Дентиклі

Склерозований (прозорий) дентин

Мертвий дентин

Інтертубулярний дентин

Інтерглобулярні простори

461 / 4299
На електронній мікрофотографії фрагменту нирки представлена прино-сна артеріола, в якій під ендотелієм видно великі клітини, що містять секре- торні гранули. Назвіть цей вид клітин:

Мезангіальні

Юкставаскулярні

Юкстагломерулярні

Інтерстиціальні

Іладеньком’язові

462 / 4299
Дитина пошкодила бокову поверхню язика. Які сосочки язика при цьому пошкоджені найвірогідніше?

Конічні

Грибоподібні

Жолобуваті

Ниткоподібні

Листоподібні

463 / 4299
Продукуючи ряд гормонів, плацента відіграє роль тимчасової ендокринної залози. Якій гормон може бути визначений у крові жінки вже на третю-четверту добу після початку імплантації, що використовується в медичній практиці для раннього діагностування вагітності?

Окситоцин

Прогестерон

Хоріонічний гонадотропін

Соматостатин

Вазопресин

464 / 4299
Процес імплантації відбувається в два етапи: адгезія та інвазія. Морфологічним проявом процесу адгезії бластоцисти є:

Прикріплення бластоцисти до ендо-метрію

Руйнування судин ендометрію

Руйнування сполучної тканини ендо-метрію

Формування лакун

Руйнування епітелію ендометрію

465 / 4299
У жінки 24-х років під час очікування на видалення зубу збільшився тонус симпатичного відділу автономної нервової системи. Що з наведеного буде спостерігатися у пацієнтки?

Підсилення перистальтики

Збільшення частоти серцевих скорочень

Звуження зіниць

Підсилення секреції травних соків

Звуження бронхів

466 / 4299
У чоловіка 47-ми років за медичними показаннями була видалена одна із слинних залоз, після чого різко зменшилася активність амілази у слині. Яка залоза була видалена?

Ясенева

Привушна

Щічна

Підщелепна

Під’язикова.

467 / 4299
У альпініста при тривалому перебуванні в горах відбувається збільшення кількості еритроцитів з 5,0 • 1012/л до 6,0 • 1012/ л. Завдяки чому відбулася стимуляція еритропоезу?

Збільшення р О2 у венозній крові

Збільшення р О2 в артеріальній крові

Зменшення р О2 в артеріальній крові

Збільшення р О2 у клітинах

Зменшення р О2 у венозній крові

468 / 4299
Експериментатор хоче виробити у собаки слиновидільний умовний рефлекс. Що з наведеного доцільно використати як умовний подразник?

Сухарі

Звук помірної гучності

М’ясо

Дуже гучний звук

Електричний струм

469 / 4299
У людини травматичне пошкодження великого грудного м’яза. До зменшення величини якого показника це призведе?

Резервний об’єм вдиху

Залишковий об’єм

Функціональна залишкова ємність легенів

Резервний об’єм видиху

Дихальний об’єм

470 / 4299
У чоловіка 60-ти років крововилив у головний мозок спричинив тривалий сон. Пошкодження якої структури найвірогідніше призвело до цього стану?

Чорна субстанція

Гіпокамп

Кора великих півкуль

Ретикулярна формація

Чотиригорбикова структура

471 / 4299
У кроля через місяць після хірургічного звуження ниркової артерії зареєстровано суттєве підвищення системного артеріального тиску. Який з наведених механізмів регуляції спричинив зміну тиску у тварини?

Норадреналін

Серотонін

Ангіотензин-II

Адреналін

Вазопресин

472 / 4299
На ізольованому серці вивчалася швидкість проведення збудження в різних його ділянках. Де була виявлена найменша швидкість?

У волокнах Пуркін’є

У атріовентрикулярному вузлі

У пучку Гіса

У міокарді передсердь

У міокарді шлуночків

473 / 4299
У студента під час складання іспиту сохне в роті. Причиною цього є реалізація таких рефлексів:

Умовних симпатичних

Безумовних парасимпатичних

Умовних та безумовних симпатичних

Безумовних симпатичних та парасимпатичних

Умовних парасимпатичних

474 / 4299
Обстежено спортсмена після інтенсивного фізичного навантаження. Виявлено порушення координації рухів при збереженні сили скорочення м’язів. Причиною цього може бути зменшення швидкості проведення збудження:

Провідними шляхами

Еферентними нервами

Аферентними нервами

Через центральні синапси

Через нервово-м’язові синапси

475 / 4299
Після крововиливу в мозок у пацієнта виникло значне погіршення смакової чутливості. Яка структура мозку найбільш вірогідно пошкоджена?

гіпоталамус

гіпокамп

постцентральна звивина

чорна речовина

мигдалик

476 / 4299
Пацієнт скаржиться на часті кровотечі з ясен. При аналізі крові виявлено дефіцит II фактора зсідання крові (протромбіну). Яка фаза зсідання крові порушена у людини, перш за все?

Ретракція згустку

Фібриноліз

Утворення тромбіну

Утворення фібрину

Утворення протромбінази

477 / 4299
Людина знепритомніла у салоні автомобіля, де тривалий час очікувала приятеля при ввімкнєному двигуні. Яку сполуку гемоглобіну можна виявити у крові постраждалого?

Дезоксигемоглобін

Метгемоглобін

Карбоксигемоглобін

Оксигемоглобін

Карбгемоглобін

478 / 4299
У хворого різко знизився вміст Ca2+ у крові. До збільшення секреції якого гормону це призведе?

Паратгормон

Соматотропний

Тирокальцитонін

Вазопресин

Альдостерон

479 / 4299
В експерименті на ізольованій збудливій клітині необхідно отримати збільшення мембранного потенціалу спокою (гіперполяризацію). Які іонні канали слід активувати для цього?

Калієві

Калієві та натрієві

Натрієві та кальцієві

Кальцієві

Натрієві

480 / 4299
У людини суттєво порушено перетравлення білків, жирів та вуглеводів. Знижена секреція якого травного соку, найвірогідніше, є причиною цього?

Шлунковий

Слина

Кишковий

Підшлунковий

Жовч

481 / 4299
У хворого виявлено гіперкаліємію та гіпонатріємію. Знижена секреція якого гормону може спричинити такі зміни?

Паратгормон

Кортизол

Вазопресин

Альдостерон

Натрійуретичний

482 / 4299
У людини вимірюють енерговитра-ти натщесерце, лежачи, в умовах фізичного та психічного спокою, при температурі комфорту. В який час енерго-витрати будуть найменшими?

10-12 годин дня

14 -16 годин дня

7-8 годин ранку

17 - 18 годин вечора

3-4 години ранку

483 / 4299
Пацієнт скаржиться на швидку втомлюваність. Об’єктивно: у положенні стоячи з заплющеними очима похитується, втрачає рівновагу. Тонус скелетних м’язів знижений. Яка структура мозку найбільш вірогідно вражена у цієї людини?

Прецентральна звивина кори великих півкуль

Мозочок

Гіпоталамус

Таламус

Базальні ганглії

484 / 4299
У хворого на хронічний гломерулонефрит порушується інкреторна функція нирок. До дефіциту яких формених елементів крові це призведе?

Лейкоцити

Еритроцити

Еритроцити та лейкоцити

Тромбоцити

Лейкоцити та тромбоцити

485 / 4299
У тварини в експерименті подразнювали на шиї периферичний відрізок блукаючого нерву, при цьому спостерігали такі зміни серцевої діяльності:

Збільшення збудливості міокарда

Збільшення сили скорочень

Збільшення проведення збудження по міокарду

Збільшення частоти та сили скорочень

Зменшення частоти скорочень

486 / 4299
У хворого камінь загальної жовчної протоки перекрив надходження жовчі до кишечнику. Порушення якого з процесів, перш за все, при цьому спостерігається?

Всмоктування білків

Перетравлення жирів

Перетравлення вуглеводів

Всмоктування вуглеводів

Перетравлення білків

487 / 4299
У пацієнта виявлено гіпокальціє-мію. Дефіцит якого гормону може бути причиною цього?

Тирокальцитонін

Паратгормон

Кортиколі6єрин

Кортикотропін

Альдостерон

488 / 4299
Під час огляду дитини, що не отримувала протягом зими свіжих овочів та фруктів, виявлені множинні підшкірні крововиливи, запалення ясен, каріозні порожнини в зубах. Комбінацію яких вітамінів слід призначити дитині?

Тіамін та піридоксин

Кальциферол та аскорбінова кислота

Аскорбінова кислота та рутин

Фолієва кислота та кобаламін

Рибофлавін та нікотинамід

489 / 4299
У хворого спостерігається збільшення проникності стінок кровоносних судин із розвитком підвищеної кровоточивості ясен, виникнення дрібнокра-пчастих крововиливів на шкірі, випадіння зубів. Яким порушенням вітамінного обміну пояснюються ці симптоми?

Гіповітаміноз C

Гіпервітаміноз D

Гіпервітаміноз C

Гіповітаміноз A

Гіповітаміноз D

490 / 4299
У хворого на цукровий діабет після ін’єкції інсуліну настала втрата свідомості, судоми. Який результат дав біохімічний аналіз крові на вміст глюкози?

5,5 ммоль/л

8,0 ммоль/л

2,5 ммоль/л

3,3 ммоль/л

10 ммоль/л

491 / 4299
У жінки 46-ти років, що страждає на жовчнокам’яну хворобу, розвинулася жовтяниця. При цьому сеча стала темно-жовтого кольору, а кал - знебарвлений. Вкажіть, концентрація якої речовини у сироватці крові зросте в найбільшій мірі?

Вільний білірубін

Мезобілірубін

Уробіліноген

Білівердин

Кон’югований білірубін

492 / 4299
У хворого 27-ми років виявлено патологічні зміни печінки та головного мозку. У плазмі крові виявлено різке зниження, а в сечі підвищення вмісту міді. Поставлено діагноз - хвороба Вільсона. Активність якого ферменту в сироватці крові необхідно дослідити для підтвердження діагнозу?

Алкогольдегідрогеназа

Ксантиноксидаза

Лейцинамінопептидаза

Карбоангідраза

Церулоплазмін

493 / 4299
У немовляти на 6-й день життя в сечі виявлено надлишок фенілпірувату та фенілацетату. Обмін якої амінокислоти порушено в організмі дитини?

Фенілаланін

Аргінін

Триптофан

Гістидин

Метіонін

494 / 4299
У хворого, що страждає на цингу, порушено процеси утворення сполучної тканини, що призводить до розхитування та випадіння зубів. Порушення активності якого ферменту викликає ці симптоми?

Проколагенпептидаза N-кінцевого пептиду

Ілікозилтрансфераза

Еластаза

Лізилгідроксилаза

Проколагенпептидаза С-кінцевого пептиду

495 / 4299
У пацієнта, що страждає на хронічну ниркову недостатність, розвинувся остеопороз. Порушення синтезу в нирках якого регулятора мінерального обміну є основною причиною остеопо-розу?

Гідроксилювання проліну

Утворення 1, 25(OH)2D3

Гідроксилювання кортизолу

Гідроксилювання лізину

Карбоксилювання глутамату

496 / 4299
При отруєнні ціанідами настає миттєва смерть. В чому полягає механізм дії ціанідів на молекулярному рівні?

Блокують сукцинатдегідрогеназу

Інгібують цитохром В

Інгібують цитохромоксидазу

Інактивують кисень

Зв’язують субстрати ЦТК

497 / 4299
У хворого відмічені такі зміни: порушення зору в сутінках, підсихання кон’юнктиви та рогової оболонки. Такі порушення можуть бути при нестачі віта- міну:

Вітамін B

Вітамін D

Вітамін C

Вітамін B12

Вітамін A

498 / 4299
При дефіциті якого вітаміну спостерігається одночасне порушення репродуктивної функції та дистрофія скелетної мускулатури?

Вітамін K

Вітамін D

Вітамін A

Вітамін E

Вітамін B1

499 / 4299
Пацієнтці з високим ступенем ожиріння в якості харчової добавки рекомендований карнітин для покращення 'спалювання'жиру. Яку безпосередню участь бере карнітин у процесі оки-снення жирів?

Транспорт ВЖК з жирових депо до тканин

Активація внутрішньоклітинного ліполізу

Бере участь у одній з реакцій бета-окиснення ВЖК

Активація ВЖК

Транспорт ВЖК з цитозолю до мітохондрій

500 / 4299
У дитини має місце порушення формування емалі та дентину через знижений вміст іонів кальцію в крові. Дефіцит якого гормону може викликати такі зміни?

Паратгормон

Соматотропний гормон

Тироксин

Трийодтиронін

Тиреокальцитонін

501 / 4299
У дитини встановлена гостра ниркова недостатність. Якими біохімічними показниками слини це можна підтвердити?

Зменшення рівня фосфату

Збільшення альфа-амілази

Зниження лужної фосфатази

Збільшення імуноглобуліну А

Підвищення рівня залишкового азоту

502 / 4299
У дитини спостерігається порушення процесів окостеніння та 'крапчастість емалі'. Обмін якого мікроелементу при цьому порушений?

Мідь

Цинк

Хром

Фтор

Залізо

503 / 4299
Пародонтит супроводжується активацією протеолізу в тканинах па-родонту. Підвищення якого компоненту ротової рідини свідчить про активацію протеолізу?

Глюкоза

Органічні кислоти

Біогенні аміни

Холестерол

Амінокислоти

504 / 4299
У хворої дитини в крові встановлено гіперліпопротеїнемію, що передалася у спадок. Генетичний дефект синтезу якого ферменту обумовлює це явище?

Протеїназа

Фенілаланінгідроксилаза

Гемсинтетаза

Глікозидаза

Ліпопротеїнліпаза

505 / 4299
В ендокринологічному відділенні з діагнозом цукровий діабет лікується жінка 42-х років зі скаргами на спрагу, підвищений апетит. Які патологічні компоненти виявлені при лабораторному дослідженні сечі пацієнтки?

Кров

Білок, креатин

Білок, амінокислоти

Глюкоза, кетонові тіла

Білірубін, уробілін

506 / 4299
Під час огляду дитини у віці 11-ти місяців педіатр виявив викривлення кісток нижніх кінцівок та затримку мінералізації кісток черепа. Нестача якого вітаміну призводить до цієї патології?

Рибофлавін

Біофлавоноїди

Холекальциферол

Пантотенова кислота

Тіамін

507 / 4299
У хворого в крові підвищений вміст сечової кислоти, що клінічно проявляється больовим синдромом внаслідок відкладення уратів у суглобах. У результаті якого процесу утворюється ця кислота?

Розпад піримідинових нуклеотидів

Розпад пуринових нуклеотидів

Розщеплення білків

Катаболізм гему

Реутилізація пуринових основ

508 / 4299
У хворого в результаті неповноцінного харчування з’явилися діарея, деменція та дерматит. Нестачею якого вітаміну викликаний цей стан?

Вітамін Б12

Вітамін Б2

Вітамін C

Вітамін PP

Вітамін Б1

509 / 4299
Під час бігу на короткі дистанції у нетренованої людини виникає м’язова гіпоксія. До накопичення якого метаболіту в м’язах це призводить?

Оксалоацетат

Лактат

Ілюкозо-6-фосфат

Кетонові тіла

510 / 4299
У цитоплазмі міоцитів розчинена велика кількість метаболітів окиснен-ня глюкози. Назвіть один з них, що безпосередньо перетворюється на лактат:

Оксалоацетат

Гліцерофосфат

Ілюкозо-6-фосфат

Піруват

Фруктозо-6-фосфат

511 / 4299
У хворого діагностована алкапто-нурія. Вкажіть фермент, дефект якого є причиною цієї патології:

Фенілаланінгідроксилаза

Піруватдегідрогеназа

дОФА-декарбоксилаза

Оксидаза гомогентизинової кислоти

Ілутаматдегідрогеназа

512 / 4299
У дитини 2-х років кишечний дисбактеріоз, на фоні якого виник геморагічний синдром. Найбільш вірогідною причиною геморагій у цієї дитини є:

Нестача вітаміну К

Активація тромбопластину тканин

Гіповітаміноз РР

Гіпокальціємія

Дефіцит фібриногену

513 / 4299
У щура відтворено токсичний набряк легень за допомогою розчину хлориду амонію. Який провідний патогенетичний фактор цього набряку?

Посилення лімфовідтоку

Зниження колоїдно-осмотичного тиску

Підвищення венозного тиску

Розлади нервової та гуморальної регуляції

Підвищення проникності капілярів

514 / 4299
До неврологічного відділення з приводу мозкового крововиливу надійшов хворий 62-х років у важкому стані. Об’єктивно: спостерігається наростання глибини та частоти дихання, а потім його зменшення до апное, після чого цикл дихальних рухів відновлюється. Який тип дихання виник у хворого?

Кусмауля

Чейн-Стокса

Іаспінг-дихання

Апнейстичне

Біота

515 / 4299
У тварини штучно спровокували карієс. Який компенсаторний механізм є найважливішим при розвитку цього захворювання?

Утворення одонтобластами вторинного дентину

Пригнічення фагоцитозу

Гіперфункція паращитоподібних залоз

Новоутворення емалі

Гіпотрофія слинних залоз

516 / 4299
У чоловіка 35-ти років через 30 хвилин після автомобільної аварії виявлена масивна травма нижніх кінцівок без значної зовнішньої крововтрати. Постраждалий знаходиться в збудженому стані. Який компонент патогенезу травматичного шоку є в пацієнта провідним та потребує негайного коригування?

Інтоксикація

Внутрішня крововтрата

Внутрішня плазмовтрата

Біль

Порушення функції органів

517 / 4299
У жінки після пологів зменшилася маса тіла на 20 кг, випадають зуби та волосся, спостерігається атрофія м’язів (гіпофізарна кахексія). З порушенням синтезу якого гормону гіпофіза це пов’язано?

Гонадотропний

Пролактин

Кортикотропний

Соматотропний

Тиреотропний

518 / 4299
У хворого внаслідок отруєння сулемою розвинулася гостра ниркова недостатність, пєрє6іг якої включав 4 стадії: перша - початкова, друга - олігоану-рії, четверта - одужання. Як називається третя стадiя гострої ниркової недостатності?

Ішемічна

Поліурична

Патохiмiчна

Метаболічна

Гемодинамічна

519 / 4299
У хворого на пневмосклероз розвинулася легенева гіпєртєнзія та пра-вошлуночкова серцева недостатність з асцитом та набряками. Який основний патогенетичний механізм розвитку набряків у цього хворого?

Збільшення онкотичного тиску міжклітинної рідини

Збільшення гідростатичного тиску крові у венах

Зменшення осмотичного тиску крові

Збільшення проникності стінок судин

Зменшення онкотичного тиску крові

520 / 4299
Через рік після субтотальної резекції шлунка з приводу виразки малої кривини виявлені зміни в крові - анемія, лейко- та тромбоцитопенія, КП-1,3, наявність мегалобластів та мегало-цитів. Дефіцит якого фактору обумовив розвиток цієї патології?

Хлороводнева кислота

Фактор Касла

Пепсин

Гастрин

Муцин

521 / 4299
Постраждалий 45-ти років доставлений бригадою швидкої допомоги з тяжкою травмою черепа в стані шоку. Об’єктивно: непритомний, шкіра бліда, t° тіла - 35,00С, м’язовий тонус знижений, рефлекси відсутні, пульс частий та слабкий, АТ- 50/30 мм рт.ст. У якій клінічній стадії шоку знаходиться людина?

Гальмування

Торпідна

Термінальна

Еректильна

Збудження

522 / 4299
Після травматичного видалення зуба хворий скаржиться на тупий сильний біль у ясні без чіткої локалізації, підвищення температури тіла до 37,50С. Діагностований альвеоліт. Який вид болю в даного хворого?

Відображений

Фантомний

Вісцеральний

Протопатичний

Епікритичний

523 / 4299
Хлопчик 12-ти років повернувся зі школи та почав скаржитися на головний біль, нудоту, озноб, періодичний біль у м’язах, втрату апетиту, кволість. Для якого періоду хвороби характерні такі симптоми?

Інкубаційний

Продромальний

Розпал захворювання

Закінчення хвороби

Латентний

524 / 4299
Кролів годували їжею з додаванням холестерину. Через 5 місяців виявлені атеросклеротичні зміни в аорті. Назвіть головну причину атерогенезу в даному випадку:

Ендогенна гіперхолестеринемія

Гіподинамія

Переїдання

Екзогенна гіперхолестеринемія

525 / 4299
У хворого на первинний нефроти-чний синдром встановлений вміст загального білку крові 40 г/л. Яка причина обумовила гіпопротеїнемію?

Зниження синтезу білку в печінці

Вихід білку з судин у тканини

Протеїнурія

Порушення всмоктування білку в кишечнику

Підвищений протеоліз

526 / 4299
У експериментальних щурів, що тривалий час отримували лише вуглеводну їжу, спостерігалося накопичення води в тканинах. Який патогенетичний механізм є головним у розвитку набряку в даному випадку?

Дисрегуляторний

Гіперосмолярний

Гіпоонкотичний

Мембраногенний

Лімфогенний

527 / 4299
У хворого на аденому клубочкової зони кори наднирників (хвороба Кон-на) спостерiгаються артерiальна гіпер-тєнзія, напади судом, поліурія. Що є головною ланкою в патогенезі цих порушень?

Гіпосекреція глюкокортикоїдів

Гіперсекреція глюкокортикоїдів

Гіпосекреція альдостерону

Гіперсекреція катехоламінів

Гіперсекреція альдостерону

528 / 4299
Чоловік 30-ти років отримав опромінювання дозою біля 3 Гр. Яка зміна в крові буде через 8 годин після опромінювання?

Лімфопенія

Гранулоцитопенія

Тромбоцитопенія

Анемія

Лейкопенія

529 / 4299
Хворий 43-х років чотири місяці тому переніс травматичну ампутацію лівої нижньої кінцівки. Зараз він скаржиться на відчуття наявності ампутованої кінцівки та постійний сильний, іноді нестерпний біль у ній. Який вид болю в хворого?

Каузалгія

Рефлекторний

Фантомний

Невралгія

Таламічний

530 / 4299
У дитини 14-ти років, хворого на дифтерію, у період кризи, під час різкого падіння температури на фоні тахікардії, АТ- 70/50 мм рт.ст. До якої форми порушення судинного тонусу відноситься дане явище?

Вегето-судинна дистонія

Гостра гіпотензія

Гіпотонічна хвороба

Хронічна гіпотензія

531 / 4299
У хворого на жовтяницю у крові підвищений вміст прямого білірубіну, жовчних кислот, у сечі відсутній стер-кобіліноген. При якому виді жовтяниці можлива наявність цих ознак?

Печінкова

Надпечінкова

Механічна

Паренхіматозна

Гемолітична

532 / 4299
У новонародженої дитини з піло-ростенозом спостерігається блювання, що часто повторюється, супроводжується апатією, слабкістю, підвищенням тонусу м’язів, іноді судомами. Яка форма порушення кислотно-лужного стану розвинулася в хворого?

Видільний ацидоз

Газовий ацидоз

Негазовий алкалоз

Газовий алкалоз

Метаболічний ацидоз

533 / 4299
У хворого 19-ти років виявлена хронічна набута гемолітична анемія. Що є провідним патогенетичним механізмом розвитку цієї патології?

Аутоімунний гемоліз

Внутрішньоклітинний гемоліз

Гіпоосмолярність плазми

Осмотичний гемоліз

Токсичний гемоліз

534 / 4299
Хворий взимку впав у ополонку, замерз на вітрі, захворів. Температура тіла піднялася до 39, 70С та коливалася від 39,00С до 39,80С. Назвіть тип температурної кривої в хворого:

Febris intermittens

Febris remittens

Febris hectica

Febris continua

Febris recurrens

535 / 4299
У хворого на хронічний мієлолейкоз виявлено ознаки анемії - зменшення кількості еритроцитів і вмісту гемоглобіну, оксифільні та поліхромато-фільні нормоцити, мікроцити. Який патогенетичний механізм є провідним у розвитку цієї анемії?

Зменшення синтезу еритропоетину

Хронічна крововтрата

Внутрішньосудинний гемоліз еритроцитів

Заміщення еритроцитарного ростка

Дефіцит вітаміну В12

536 / 4299
У водолаза, що тривалий час перебував на глибині 40 м, при декомпресії розвинулася кесонна хвороба. Основною патогенетичною ланкою стала емболія:

Тканинна

Жирова

Повітряна

Газова

Парадоксальна

537 / 4299
У померлої з атеросклерозом судин головного мозку на автопсії в лівій півкулі головного мозку виявлено вогнище, яке представлене в’ялою, безструктурною сірувато-жовтуватою тканиною з нечіткими краями. Про наявність якого патологічного процесу йдеться?

Мультифокальний ріст пухлини з кістозним перетворенням

Стареча енцефалопатія

Вогнищевий енцефаліт

Ішемічний інсульт

Безліч вогнищ свіжих та старих крововиливів у мозок

538 / 4299
Гістологічно в усіх шарах апендикса знайдені поліморфноядерні лейкоцити в значній кількості, повнокров’я, стази. Якому захворюванню притаманна така картина?

Простий апендицит

Хронічний апендицит

Поверхневий апендицит

Гангренозний апендицит

Флегмонозний апендицит

539 / 4299
Чоловік віком 55-ти років тривалий час хворів на хронічний гломерулонефрит. Помер при явищах хронічної ниркової недостатності. На поверхні епікарда та перикарда виявляються сірувато-білуваті ворсинчасті нашарування. Який патологічний процес мав місце в перикарді?

Геморагічне запалення

Артеріальне повнокров’я

Фібринозне запалення

Проліферативне запалення

Організація

540 / 4299
На розтині в хворого в черевній порожнині виявлено близько 2,0 л. гнійної рідини. Очеревина тьмяна, з сіруватим відтінком, на серозній оболонці кишок сіруватого кольору нашарування, що легко знімаються. Найвірогідніше це:

Фібринозно-гнійний перитоніт

Туберкульозний перитоніт

Серозний перитоніт

Геморагічний перитоніт

541 / 4299
У біоптаті дужок м’якого піднебіння, взятого з приводу підозри на пухлину (макроскопічно визначалася виразка з щільним дном), виявлений некроз слизової оболонки з інфільтрацією підслизового шару лімфоцитами, епіте-ліоїдними клітинами, плазматичними клітинами, поодинокими нейтрофілами. Звертає увагу наявність вираженого ендо- та периваскуліту. Для якого захворювання властиві зазначені зміни?

Виразковий стоматит

Афтозний стоматит

Дифтерія зіва

Виразково-некротичний стоматит Венсана

Первинний сифіліс

542 / 4299
Під час гістологічного дослідження щитоподібної залози померлого від серцевої недостатності при явищах гіпотиреозу виявлено дифузну інфільтрацію залози лімфоцитами та плазмо-цитами, з утворенням лімфоїдних фолікулів, атрофію паренхіми і розростання сполучної тканини. Який найбільш вірогідний діагноз?

Зоб тиреотоксичний

Гнійний тиреоїдит

Аутоімунний тиреоїдит Хашімото

Аденома щитоподібної залози

543 / 4299
Під час патологоанатомічного дослідження тіла хлопчика 5-ти років, померлого від гострої легенево-серцевої недостатності, було знайдено: серозно-геморагічний трахеобронхіт з ділянками некрозу слизової оболонки, у легенях - множинні вогнища геморагічної пневмонії. Про яке захворювання йдеться?

Кір

Дифтерія

Грип

Скарлатина

Крупозна пневмонія

544 / 4299
Під час макроскопічного дослідження тканини легені виявлені ділянки підвищеної повітряності з наявністю дрібних пухирів, а гістологічно - стоншення та розрив альвеолярних перетинок з утворенням великих порожнин різної форми. Яке захворювання виявлене в легені?

Емфізема легень

Кавернозний туберкульоз

Бронхоектатична хвороба

Хронічний бронхіт

Фі6розуючий альвеоліт

545 / 4299
Під час дослідження коронарних артерій виявлені атеросклеротичні бляшки з кальцинозом, що закривають просвіт на 1/3. У м’язі дрібні множинні білуваті прошарки сполучної тканини. Який процес виявлено в міокарді?

Тигрове серце

Інфаркт міокарда

Дифузний кардіосклероз

Післяінфарктний кардіосклероз

Міокардит

546 / 4299
У дитини після перенесеного кору, при огляді в м’яких тканинах щік та промежини виявлено нечітко відмежовані, набряклі, червоно-чорного кольору ділянки, які злегка флуктують. Яке ускладнення розвинулося у дитини?

Пролежень

Трофічна виразка

Суха гангрена

Волога гангрена

Газова гангрена

547 / 4299
У хворого, що тривалий час страждає на ревматизм, виявлений стеноз мітрального отвору, смерть настала від серцево-легеневої недостатності. Під час розтину виявлена бура індурація легень. При якому виді порушення кровообігу виникають подібні зміни в легенях?

Гостра лівошлуночкова недостатність

Портальна гіпертензія

Хронічна правошлуночкова недостатність

Гостра правошлуночкова недостатність

Хронічна лівошлуночкова недостатність

548 / 4299
У дитини 12-ти років, що лікувалася в інфекційному відділенні з приводу грипу, на 5-ту добу захворювання з’явилися сильний головний біль, нудота, запаморочення, менінгеальні знаки. Смерть настала через добу від наростаючого набряку мозку. Під час розтину черепу м’які мозкові оболонки набряклі, повнокровні, дифузно просякнуті рідиною яскраво-червоного кольору. Звивини та борозни згладжені. Про яке ускладнення грипу слід думати?

Крововилив у мозок

Геморагічний менінгіт

Венозна гіперемія оболонок мозку

Гнійний лептоменінгіт

Серозний менінгіт

549 / 4299
Чоловік 42-х років, що протягом 8-ми років страждав на хронічний гра-нулематозний періодонтит, хронічний гнійний остеомієліт нижньої щелепи, помер при явищах хронічної ниркової недостатності. Яке ускладнення гнійного остеомієліту розвинулося в нирках?

Жирова дистрофія

Амілоїдоз

Гіаліноз

Некроз епітелію звивистих канальців

Атрофія

550 / 4299
Під час морфологічного дослідження дна каріозної порожнини зуба чітко диференціюються три зони: розм’якшеного дентину, прозорого дентину, замісного дентину. Вкажіть, для якої стадії карієсу характерні ці зміни?

Хронічний карієс

Середній карієс

Поверхневий карієс

Стадія плями

Глибокий карієс

551 / 4299
На слизовій оболонці правого піднебінного мигдалика спостерігається безболісна виразка з гладеньким лакованим дном та рівними хрящоподібної консистенції краями. Мікроскопічно: запальний інфільтрат, що складається з лімфоцитів, плазмоцитів, невеликої кількості нейтрофілів та епітеліоїдних клітин та наявність ендо- та периваскуліту. Про яке захворювання йдеться?

Сифіліс

Дифтерія зіву

Виразково-некротична ангіна Венса-на

Туберкульоз

Актиномікоз

552 / 4299
У хворої 57-ми років з’явилися маткові кровотечі, що виникають періодично. З діагностичною метою проведено вишкрібання матки. В отриманому матеріалі серед елементів крові спостерігаються залозисті комплекси різних розмірів та форми, утворені атиповими клітинами з гіперхромними ядрами з численними мітозами (у тому числі патологічними). Який найбільш вірогідний діагноз?

Хоріонепітеліома

Фіброміома матки

Аденокарцинома

Залозиста гіперплазія ендометрію

Ендометрит

553 / 4299
Під час розтину померлого, що страждав на гіпертонічну хворобу, у речовині головного мозку виявлена порожнина, стінки якої мають іржавий колір. Що передувало виникненню даних змін?

Немічний інфаркт

Абсцес

Гематома

Плазморагії

Діапедезні крововиливи

554 / 4299
Під час гістологічного дослідження легень померлого від серцевої недостатності, виявлені вогнища запалення з заповненням альвеол рідиною, забарвленою в блідорожевий колір, місцями з наявністю тонких рожевих ниток, які утворюють дрібнопетлисту сітку з невеликою кількістю лімфоцитів. Який характер ексудату в легенях?

Геморагічний

Фібринозний

Серозний

Гнійний

Серозно-фібринозний

555 / 4299
Хвора 38-ми років померла під час нападу бронхіальної астми, який не вдавалося купірувати. Під час гістологічного дослідження у просвіті бронхів виявлені накопичення слизу, у стінці бронхів багато тучних клітин (лабро-цитів), багато з них у стані дегрануляції, а також багато еозинофілів. Який патогенез цих змін у бронхах?

Гранулематоз

Клітинно обумовлений цитоліз

!мунокомплексний механізм

Цитотоксична, цитолітична дія антитіл

Атопія, анафілаксія

556 / 4299
Під час розтину тіла дитини 9-ти років у слизовій оболонці прямої кишки виявлені численні неправильної форми дефекти різної глибини з нерівними краями, а також сіробілі плівки, що щільно спаяні з підлеглими тканинами. Про яке захворювання слід думати?

Дизентерія

Сальмонельоз

Амебіаз

Холера

Черевний тиф

557 / 4299
Під час ендоскопії шлунка взято бі- оптат слизової оболонки. Його гістологічне дослідження виявило: слизова оболонка збережена, стовщена, набрякла, гіперемована, з дрібнокрапельни-ми крововиливами, щільно вкрита слизом. Визначити форму гострого гастриту:

Гнійний

Катаральний

Фібринозний

Ерозивний

Некротичний

558 / 4299
Хворий був доставлений до лікарні зі скаргами на головний біль, підвищену температуру, часті випорожнення, біль у животі з тенезмами. Лікар встановив клінічний діагноз дизентерія та направив досліджуваний матеріал (випорожнення) до баклабораторії. Яким методом діагностики лікар-лаборант має підтвердити або відхилити клінічний діагноз?

Біологічний

Алергічний

Бактеріологічний

Бактеріоскопічний

Серологічний

559 / 4299
Бактеріологічне дослідження гнійних виділень з уретри з’ясувало присутність бактерій, які за Грамом фарбувалися негативно, нагадували кавові зернини, розкладали глюкозу та мальтозу до кислоти. Розташовувалися в лейкоцитах. До збудників якої хвороби їх віднести?

М ’який шанкр

Гонорея

Сифіліс

Меліоїдоз

Венеричний лімфогранулематоз

560 / 4299
У відділенні щелепно-лицевої хірургії в хворого виникло гнійне ускладнення. Під час бактеріологічного дослідження матеріалу із рани виділено культуру, що утворювала пігмент синьо-зеленого кольору. Який мікроорганізм найвірогідніше є збудником інфекції?

Klebsiella pneumoniae

Proteus vulgaris

Staphylococcus epidermidis

Pseudomonas aeruginosa

Bacillus subtilis

561 / 4299
До інфекційної клініки надійшла дівчинка 7-ми років з високою температурою, скаргами на біль у горлі, загальну слабкість. Лікар запідозрив дифтерію. Що із перерахованого є вирішальним для підтвердження діагнозу після виділення чистої культури збудника?

Виявлення у збудника волютинових зерен

Гемолітична здатність збудника

Проба на цистиназу

Проба на токсигенність

Фаголізабельність

562 / 4299
В сироватці крові під час постановки імуноферментної реакції був визначений HBsантиген. При якому захворюванні зустрічається даний антиген?

Вірусний гепатит В

Туберкульоз

СНЩ

Вірусний гепатит А

Сифіліс

563 / 4299
Для серологічної діагностики поліомієліту досліджують парні сироватки хворого. Що слід використати як антиген в реакції нейтралізації цитопато-генної дії?

Живі віруси трьох типів

Антигени, інактивовані формаліном

Комплементзв’язуючі антигени вірусу

Антигени з капсидних білків вірусу

Антигени-гемаглютиніни

564 / 4299
Спеціалісти-стоматологи є групою ризику щодо професійного зараження гепатитом В. Вкажіть ефективний засіб для специфічної активної профілактики цієї хвороби серед лікарів-стоматологів:

Вакцинація рекомбінантною вакциною

Введення інтерфероногенів

Введення специфічного імуноглобу-ліну

Виконання робіт в гумових рукавичках

Надійна стерилізація медичного інструментарію

565 / 4299
У пацієнта із виразки, яка роз- ташована на слизовій оболонці ротової порожнини, при забарвленні за Романовським-Гімза, виявлені тонкі спіралеподібної форми мікроорганізми блідо-рожевого кольору із 12-14 завитками та загостреними кінцями. Збуднику якого захворювання властиві такі ознаки?

Сифіліс

Кампілобактеріоз

Лептоспіроз

Поворотний тиф

Содоку

566 / 4299
У хворого з клінічними ознаками первинного імунодефіциту виявлено порушення функції антигенпрезентації імунокомпетентним клітинам. Дефект структур яких клітин є можливим?

0-лімфоцити

Макрофаги, моноцити

В-лімфоцити

Т-лімфоцити

Фібробласти

567 / 4299
Після обстеження хворого на реци-дивуючий афтозний стоматит з супутнім кандидозом лікар вирішив виключити ВШ-інфекцію. Яке дослідження допоможе прояснити ситуацію та встановити попередній діагноз?

!муноферментний аналіз

Реакція гемаглютинації

Реакція преципітації у гелі

Реакція гальмування гемаглютинації

Фазово-контрастна мікроскопія

568 / 4299
На території певної місцевості була зареєстрована масова загибель гризунів. Виникло припущення, що причиною може бути збудник чуми. Яку серологічну реакцію слід використати для швидкого встановлення збудника цієї епізоотії?

Аглютинації

Преципітації

Зв’язування комплементу

Пасивної гемаглютинації

Нейтралізації

569 / 4299
У дитини 10-ти років поставлено пробу Манту (з туберкуліном). Через 48 годин на місці введення туберкуліну з’явилася папула розміром до 8 мм у діаметрі. Який тип реакції гіперчутливості виник після введення туберкуліну?

Атопічна реакщя

Реакщя гіперчутливості II типа

Реакція типа сироваткової хвороби

Реакція типа феномен Артюса

Реакція гіперчутливості IV типа

570 / 4299
При багатьох інфекційних хворобах у крові хворого можна виявити антигени збудника. Яку реакцію слід застосувати, враховуючи, що рівень ан-тигенемії низький?

Твердофазний !ФА

Реакція латекс-аглютинації

Реакція аглютинації

Імуноелектрофорез

Реакція непрямої гемаглютинації

571 / 4299
Хвору 58-ми років готували до операції холецистектомії. В комплекс засобів премедикації наркозу було введено бензогексоній. Яка роль цього фарма-копрепарату в наркозі?

Функціональна блокада вісцеральних рефлексів

Розслаблення гладенької мускулатури

Підсилення ретроградної амнезії

Редукція фази збудження

Розслаблення скелетної мускулатури

572 / 4299
У хворого 30-ти років, який потрапив до клініки з діагнозом гострий гломерулонефрит, спостерігається проте-їнурія. Яке порушення спричинило це явище?

Підвищення проникності ниркового фільтра

Підвищення гідростатичного тиску крові у капілярах

Затримка виведення продуктів азотистого обміну

Зменшення кількості функціонуючих нефронів

Зниження онкотичного тиску плазми крові

573 / 4299
Під час ефірного наркозу в хворого виникла виражена брадикардія з погрозою зупинки серця. Який із перелічених препаратів треба застосувати для прискорення серцевих скорочень в умовах наркозу, що повинен продовжуватися?

Адреналін

Ізадрін

Атропін

Камфора

Кофеїн

574 / 4299
Хворий з простудним захворюван- ням страждає від нестерпного кашлю з харкотинням, що погано виділяється. Який з перелічених препаратів поліпшує відходження харкотиння?

Лібексін

Кодеїну фосфат

Ілауцину гідрохлорид

Фалімінт

Настій трави термопсису

575 / 4299
Хворому на стоматит призначили препарат з групи сульфаніламідів. Який механізм його антибактеріальної дії?

Зменшення проникності мембран

Конкурентний антагонізм з ПАБК

Коагуляція білка

Пригнічення сульфгідрильних груп тіолових ферментів

Порушення синтезу білків клітинної стінки

576 / 4299
Хворому з інфарктом міокарда була проведена нейролептаналгезія. Який препарат з групи наркотичних анальгетиків найчастіше застосовується спільно з дроперидолом?

Омнопон

Пентазоцин

Морфін

Промедол

Фентаніл

577 / 4299
До приймального відділення надійшов хворий у непритомному стані. Об’єктивно: шкіра холодна, зіниці звужені, дихання з утрудненням, відзначається періодичність за типом Чейн-Стокса, артеріальний тиск знижений, сечовий міхур переповнений. Чим відбулося отруєння?

М-холіноблокатори

Ненаркотичні аналгетики

Транквілізатори

Наркотичні аналгетики

578 / 4299
Хворий 30-ти років звернувся до лікаря зі скаргами на пронос та біль у животі протягом 5 днів, підвищення температури тіла до 37,50С з ознобами. Напередодні хворий був у лісі, де випив води з відкритого водоймища. Встановлено бактеріологічно підтверджений діагноз: амебна дизентерія. Вкажіть препарат вибору для лікування цього захворювання:

Левоміцетин

Фуразолідон

Метронідазол

Еметина гідрохлорид

Фталазол

579 / 4299
Хірург використав 70% розчин спирту етилового для обробки рук перед оперативним втручанням. Який основний механізм антисептичної дії препарату на мікроорганізми?

Дегідратація білків протоплазми

Взаємодія з аміногрупами білків протоплазми

Блокада сульфгідрильних груп ферментних систем

Окиснення органічних компонентів протоплазми

Взаємодія з гідроксильними групами ферментів

580 / 4299
Для дезінфекції неметалічного інструментарію у хірургічному відділенні використали розчин формальдегіду. До якої групи за хімічною будовою відноситься даний антисептичний препарат?

Іалогенвмісні сполуки

Засоби ароматичного ряду

Спирти

Засоби аліфатичного ряду

Детергенти

581 / 4299
Хворій 39-ти років для зняття гіпертонічної кризи ввели розчин пента-міну. Який механізм дії препарату?

Пригнічення Н-холінорецепторів

Стимуляція Н-холінорецепторів

Пригнічення М-холінорецепторів

Стимуляція M-холінорецепторів

Стимуляція M-, Н-холінорецепторів

582 / 4299
Багато видів патології (запалення, набряк легень, шок різного походження) супроводжуються підвищенням проникності судин. Яку з перелічених речовин можна використовувати для протидії цьому при будь-якому з названих видів патології?

Преднізолон

Беклометазон

Індометацин

Димедрол

Ацетилсаліцилова кислота

583 / 4299
До стаціонару надійшов хворий з діагнозом виразкова хвороба 12-палої кишки у фазі загострення. Аналіз шлункового соку показав підвищення секреторної та кислотоутворю- ючої функції шлунка. Оберіть препарат, який знизить секреторну функцію залоз шлунка за рахунок блокади H2-рецепторів:

Атропін

Метацин

Ранітидин

Платифілін

Екстракт красавки сухий

584 / 4299
При огляді школярів першого класу стоматолог виявив, що у одної дитини зуби жовтокоричньового кольору, два з них надщерблені. До цього школяра лікували з приводу пневмонії 'якимись таблетками'. Який препарат міг так негативно вплинути на зуби?

Бісептол

Оксацилін

Доксициклін

Еритроміцин

Ампіцилін

585 / 4299
Собаці в експерименті подразнювали на шиї периферичний відрізок перерізаного блукаючого нерва. При цьому спостерігали таку зміну серцевої діяльності:

Збільшення частоти та сили скорочень

Збільшення сили скорочень

Збільшення швидкості проведення збудження по міокарду

Зменшення частоти скорочень

Збільшення збудливості міокарда

586 / 4299
Назвіть групу препаратів, що зменшують потребу міокарду в кисні, зменшують силу серцевих скорочень та гальмують ліполіз:

Симпатолітики

а-адреноблокатори

Селективні ^-адреноміметики

а-адреноміметики

^-адреноблокатори

587 / 4299
Хворому на тиреотоксикоз було призначено препарат, який пригнічує ферментні системи, що беруть участь у синтезі гормонів щитоподібної залози. Вкажіть цей препарат:

Тиреоїдин

Радіоактивний йод

Мерказоліл

Калію йодид

Дийодтирозин

588 / 4299
Хворому на туберкульоз в компле- ксній терапії призначено препарат - похідне гідразиду ізонікотинової кислоти. Визначить цей препарат:

Стрептоміцину сульфат

Рифампіцин

Цефалоридін

Канаміцин

Ізоніазид

589 / 4299
Вагітна жінка страждає від печії. Який засіб є оптимальним в даному випадку?

Альмагель

Омепразол

Ранітидин

Вісмуту субнітрат

Натрію гідрокарбонат

590 / 4299
Хворому при вилученні стороннього тіла з ока необхідно провести місцеву анестезію лідокаїном. Який механізм дії цього препарату?

Знижує активність дегідрогеназ

Пригнічує активність цитохромокси-даз

Блокує проходження оксиду азоту

Знижує вихід нейромедіаторів

Порушує проходження Na+ крізь мембрану

591 / 4299
Для купірування больового синдрому пацієнту з інфарктом міокарда лікар призначив аналгетичний засіб. Стан хворого поліпшився, але згодом, у зв’язку з передозуванням з’явилася сонливість, міоз, лікар відзначив пригнічення дихання. Який лікарський засіб було призначено?

Морфін

Баралгін

Парацетамол

Седалгін

Ібупрофен

592 / 4299
Хворому при різкому підвищенні артеріального тиску призначений кло-фелін парентерально. Який його механізм дії?

Стимулює центральні імідазолові1-рецептори

Блокує Н-холінорецептори гангліїв

Стимулює центральні а2-адренорецептори

Блокує а1-адренорецептори

Блокує аі та а2-адренорецептори

593 / 4299
Який препарат, що має протиглисну дію, застосовують для стимуляції імунної системи організму при хронічному генералізованому пародонтиті?

Пірантел

Левамізол

Насіння гарбуза

Піперидину адипінат

Хлоксил

594 / 4299
У хворого на гіпертонічну хворобу розвинулася гостра серцева недостатність. Який з наведених засобів найбільш доцільно використати в даному випадку?

Кардіовален

Кофеїн

Корглікон

Кордіамін

Дігоксин

595 / 4299
Хворому була проведена операція з приводу поранення в живіт із застосуванням тубокурарину. В кінці операції, коли дихання відновилося, хворому ввели гентаміцин. Несподівано настали зупинка дихання та довгочасне розслаблення скелетних м’язів. Який ефект лежить в основі цього явища?

Антагонізм

Потенціювання

Звикання

Сенсибілізація

Кумуляція

596 / 4299
У хворого відзначається різка болючість шкіри в ділянці обличчя. Який нерв уражено?

Лицевий

Трійчастий

Блукаючий

Язикоглотковий

Окоруховий

597 / 4299
Дитина 3-х років надійшла до клініки з діагнозом отит. Є вірогідність поширення гною із барабанної порожнини. Куди може потрапити гній?

У внутрішнє вухо

В задню черепну ямку

В зовнішній слуховий прохід

В соскоподібну печеру

В слухову трубу

598 / 4299
У хворого видалений зуб, у якого язикова поверхня менша за щічну. Жувальна поверхня у формі овалу. Глибока поперечна борозна розділяє щічний та язиковий горбики. Корінь сильно стиснений у мезіо-дистальному напрямку з поздовжніми борознами на апроксимальних поверхнях та розщеплений. Визначте, який зуб видалений:

Перший верхній малий кутній зуб

Перший нижній малий кутній зуб

Нижнє ікло

Другий верхній малий кутній зуб

Верхнє ікло

599 / 4299
Під час огляду хлопчика 16-ти років були знайдені збільшені підщелепні та шийні лімфатичні вузли. Була проведена біопсія. Мікроскопічно в лімфатичних вузлах було знайдено: типова будова стерта, клітинна популяція гетерогенна, присутні великі клітини з багатолопасним ядром, множинні одно-ядерні клітини великого розміру, еозинофільні та нейтрофільні лейкоцити, лімфоцити, окрім того знайдені ділян- ки некрозу та вогнища склерозу. Який найбільш вірогідний діагноз?

Неходжкінська лімфома

Гнійний лімфаденіт

Гранулематозний лімфаденіт

Лімфогранулематоз

Гіперплазія лімфатичного вузла

600 / 4299
Хворому із стрептококовою інфекцією ясен було призначено препарат, що містить у своїй структурі бета-лактамне кільце. Який препарат відноситься до цієї групи?

Рифампіцин

Бензилпеніцилін

Стрептоміцину сульфат

Еритроміцин

Левоміцетин

601 / 4299
При дегельмінтизації з фекаліями виділився гельмінт довжиною до 2 м. Тіло сегментоване, з маленькою голівкою, на якій є чотири присоски і гачки. Визначте вид гельмінта:

Стьожак широкий

Озброєний ціп’як

Карликовий ціп’як

Ехінокок

Неозброєний ціп’як

602 / 4299
У чоловіка виявлено протозойне захворювання, при якому вражений головний мозок і спостерігається втрата зору. При аналізі крові знайдені одноклітинні організми півмісяцевої форми з загостреним кінцем. Збудником цього захворювання є:

Лейшманія

Трихомонада

Лямблія

Токсоплазма

Амеба

603 / 4299
Серед студентів однієї групи присутні представники різних рас. Один з студентів має пряме чорне волосся та нависаючу шкірну складку верхньої повіки - епікантус. Представником якої раси, найвірогідніше, є цей студент?

Монголоїдна

Негроїдна

Ефіопська

Європеоїдна

Австралоїдна

604 / 4299
Вживання тетрациклінів в першій половині вагітності призводить до виникнення аномалій органів і систем плода, в тому числі до гіпоплазії зубів, зміни їх кольору. До якого виду мінливості належить захворювання дитини?

Мутаційна

Спадкова

Рекомбінантна

Комбінативна

Модифікаційна

605 / 4299
Дівчина 16-ти років звернулася до стоматолога з приводу темної емалі зубів. При вивченні родоводу встановлено, що вказана патологія передається від батька всім дівчаткам, а від матері -50% хлопчиків. Для якого типу успадкування характерні ці особливості?

Рецесивний, зчеплений з Х- хромосомою

Аутосомно-домінантний

Домінантний, зчеплений з Х- хромосомою

Аутосомно-рецесивний

Рецесивний, зчеплений з Y- хромосомою

606 / 4299
При мікроскопіюванні виділень з ясен хворого, який хворіє на парадонтоз, виявлені найпростіші грушоподібної форми, з довжиною тіла 6-13 мкм. У паразита одне ядро, на передньому кінці розташовані 4 джгутики, є унду-лююча мембрана. Яких найпростіших виявили в хворого?

Амеба

Лейшманія

Трихомонада

Лямблія

Балантидій

607 / 4299
При обстеженні лікарями санітарно-епідеміологічної станції працівників сфери громадського харчування нерідко виявляється безсимптомне парази-тоносійство, коли клінічно здорова людина є джерелом цист, які заражують інших людей. Для паразитування якого збудника неможливе таке явище?

Малярійний плазмодій

Кишкова трихомонада

Дизентерійна амеба

Вісцеротропні лейшманії

Дерматотропні лейшманії

608 / 4299
У пацієнта після обширного опіку залишився дефект шкіри. Для закриття дефекту на це місце хірурги перемістили шкірний клапоть з іншої частини тіла цього ж хворого. Який вид трансплантації було здійснено?

Алотрансплантація

Ксенотрансплантація

Аутотрансплантація

Гомотрансплантація

Експлантація

609 / 4299
Відомо, що інформацію про послідовність амінокислот у молекулі білку записано у вигляді послідовності чотирьох видів нуклеотидів у молекулі ДНК, причому різні амінокислоти кодуються різною кількістю триплетів -від одного до шести. Як називається така особливість генетичного коду?

Виродженість

Універсальність

Специфічність

Неперекриваність

Триплетність

610 / 4299
При дослідженні гельмінтологічними методами фекалій хворого виявлено яйця овальної форми, коричневі, з горбкуватою зовнішньою оболонкою. Встановіть вид гельмінта:

Стьожак широкий

Гострик

Ціп ’як карликовий

Аскарида

Волосоголовець

611 / 4299
В ендокринологічному відділенні знаходиться хлопчик 9-ти років, у якого вже декілька разів були переломи кінцівок, пов’язані з крихкістю кісток. Функція яких ендокринних залоз (залози) порушена?

Вилочкова

Щитоподібна

Наднирковозалозні

Паращитоподібні

Епіфіз

612 / 4299
Під час виконання вагосимпати-чної блокади за Вишневським вводять розчин новокаїну по задньому краю груднино-ключично-соскоподібного м’яза вище місця перетинання її з зовнішньою яремною веною. У межах якого трикутника шиї виконують блокаду?

Трикутник Пирогова

Лопатково-трапецієподібний

Піднижньощелепний

Ключично-лопатковий

Сонний

613 / 4299
Під час підходу до щитоподібної залози з поперечного (коміроподібно-го) доступу розтинається клітковинний надгруднинний простір. Пошкодження якого анатомічного утворення, що знаходиться в даному просторі, є небезпечним?

Венозна яремна дуга

Внутрішня яремна вена

Підключична артерія

Сонна артерія

Лімфатичні вузли

614 / 4299
У хворого 28-ми років діагностовано гострий запальний процес слизової оболонки нососльозової протоки. З анамнезу відомо, що після перенесено- го грипу протягом 10-ти днів були виділення із носа. З якого відділу носової порожнини інфекція могла потрапити до нососльозової протоки?

Верхній носовий хід

Присінок носа

Лобова пазуха

Нижній носовий хід

Середній носовий хід

615 / 4299
У потерпілого відзначається правобічний перелом і крововилив у ділянці передньої третини нижньої щелепи, втрата шкірної чутливості в ділянці підборіддя. Який нерв травмований?

Нижній альвеолярний нерв

Верхні альвеолярні нерви

Щелепно-під’язикові нерви

Підборідний нерв

Щічний нерв

616 / 4299
При розкритті глибокого абсцесу щоки був виконаний вертикальний розріз, після чого спостерігається парез (порушення функції) м’язів на боці операції. Гілки якого нерва були перерізані?

Нижньощелепний

Під’язиковий

Верхньощелепний

Лицевий

Блукаючий

617 / 4299
У постраждалого травма верхньої щелепи з пошкодженням підочноям-кового отвору. Яка поверхня щелепи ушкоджена?

Підскронева

Носова

Передня

Очна

618 / 4299
Хірург планує взяти у хворого лімфу з грудної протоки в місці її впадіння до венозного русла. Куди слід провести катетер з цією метою?

Місце утворення ворітної вени

Місце утворення верхньої порожнистої вени

Місце утворення нижньої порожнистої вени

Правий венозний кут

Лівий венозний кут

619 / 4299
У хворого після простудного захворювання виникло порушення виділення сльози. Який вегетативний вузол най- більше при цьому постраждав?

Війчастий

Піднижньощелепний

Під’язиковий

Вушний

Крилопіднебінний

620 / 4299
У хворого порушилася робота жувальних м’язів. Який нерв пошкоджений?

Язиковий

Вушно-скроневий

Щічний

Верхньощелепний

Нижньощелепний

621 / 4299
Під час ДТП водій отримав численні ушкодження бічної поверхні голови, серед яких був перелом виличної дуги. Функція якого з м’язів буде страждати?

M.buccinator

M.masseter

M.orbicularis oris

M.procerus

M.risorius

622 / 4299
Внаслідок травми у постраждалого відбувся перелом у зовнішній нижній третині правої гомілки. У якій кістці відбувся перелом?

П’яткова

Стегнова

Таранна

Малогомілкова

Великогомілкова

623 / 4299
Під час оперативного втручання у жінки виникла необхідність перев’язати маткову артерію. Яке з утворень може бути випадково перев’язаним разом з нею?

Сечовід

Сечівник

Маткова труба

Внутрішня клубова вена

Кругла зв’язка матки

624 / 4299
В результаті точкового крововиливу в сітківку ока хворий втратив здатність бачити предмети в центрі поля зору. В якому місці сітківки стався крововилив?

Жовта пляма

Циліарна частина сітківки

Райдужна частина сітківки

Сліпа пляма

Судинна оболонка

625 / 4299
Хворій 35-ти років з діагнозом безпліддя в гінекологічному відділенні зроблено діагностичну біопсію ендоме-трію. При мікроскопічному дослідженні з’ясувалося, що слизова оболонка з явищами набряку, маткові залози звивисті, заповнені густим секретом. Надлишок якого гормону обумовлюють такі зміни в ендометрії?

Естрогени

Прогестерон

Тестостерон

АКТГ

Соматотропін

626 / 4299
У гістологічному препараті трубчастої кістки, на місці зламу виявлені ознаки регенераторного процесу (мозоля). Яка тканина формує цю структуру?

Ретикулярна

Пластинчаста кісткова

Епітеліальна

Пухка сполучна

Грубоволокниста кісткова

627 / 4299
Мозкова речовина часточки кровотворного органа на гістологічному препараті має світліше забарвлення і містить епітеліальні тільця. Якому органу належать дані морфологічні ознаки?

Печінка

Тимус

Селезінка

Нирка

Лімфатичний вузол

628 / 4299
Під час гістологічного дослідження в ділянці шийки власної залози шлунка виявляються дрібні клітини, що мають високе ядерно-плазматичне відношення та базофільну цитоплазму. Вкажіть функцію даних клітин:

Ендокринна

Регенерація залозистого епітелію

Секреція пепсиногену

Секреція іонів хлору

Захисна

629 / 4299
У мазку крові хворого після перенесеного грипу виявлено 10% округлих клітин розмірами 4,5-7 мкм, які мають велике кулясте ядро, базофільно забарвлену цитоплазму у вигляді вузької облямівки навколо ядра. Який стан крові вони характеризують?

Тромбопенія

Лімфоцитоз

Лімфоцитопенія

Лейкопенія

Моноцитопенія

630 / 4299
У пацієнта 42-х років, що страждає на парадонтоз, у коронковій частині пульпи виявлені округлі звапновані утворення діаметром 2-3 мм. Назвіть ці структури:

!нтерглобулярні простори

Склерозований (прозорий) дентин

!нтертубулярний дентин

Дентиклі

Мертвий дентин

631 / 4299
Одужання організму від інфекційної хвороби супроводжується нейтралізацією антигенів специфічними антитілами. Якими клітинами вони продукуються?

Фібробласти

Плазмоцити

Тканинні базофіли

Еозинофіли

Т-лімфоцити

632 / 4299
В процесі ембріогенезу із трофо-бласта формується зачаток органа, який має ендокринну функцію. Вкажіть, який це зачаток:

Пуповина

Жовтковий мішок

Ворсинчастий хоріон

Амніон

Алантоїс

633 / 4299
Людина стоїть у кімнаті в легкому одязі; температура повітря +140Е. Вікна і двері зачинені. Яким шляхом вона віддає найбільше тепла?

Теплорадіація

Випаровування

Перспірація

Конвекція

Теплопроведення

634 / 4299
На ізольованому серці вивчалася швидкість проведення збудження в різних його ділянках. Де була виявлена найменша швидкість?

У пучку Гіса

У міокарді передсердь

У волокнах Пуркін’є

У атріовентрикулярному вузлі

У міокарді шлуночків

635 / 4299
Хворий 20-ти років скаржиться на виснажливу спрагу і підвищене сечовиділення (до 10 літрів на добу). Концентрація глюкози крові в межах норми, в сечі глюкоза відсутня. Нестача якого гормону може викликати таку клінічну картину?

Трийодтиронін

Вазопресин

!нсулін

Кортизол

Окситоцин

636 / 4299
У студента під час складання іспиту сохне в роті. Причиною цього є реалізація таких рефлексів:

Умовних та безумовних симпатичних

Умовних симпатичних

Умовних парасимпатичних

Безумовних симпатичних та парасимпатичних

Безумовних парасимпатичних

637 / 4299
У стоматологічній практиці широко використовується місцеве знеболення, коли до розчину анестетика додають адреналін. Яка мета такого методу?

Місцеве звуження судин

Покращення мікроциркуляції

Місцеве розширення судин

Місцеве зниження опору судин

Зниження артеріального тиску

638 / 4299
Хворий після травми черепа втратив зір. Яка ділянка кори великих півкуль мозку ушкоджена?

Потилична

Скронева

Фронтальна

Тім’яна

Тім’яна та скронева

639 / 4299
При захворюваннях печінки, що супроводжуються недостатнім надходженням жовчі в кишечник, спостерігається погіршення гемокоагуляції. Чим можна пояснити це явище?

Лейкопенія

Дефіцит заліза

Тромбоцитопенія

Дефіцит вітаміну К

Еритропенія

640 / 4299
У хворого відзначаються збільшення окремих частин тіла (щелепи, носа, вух, язика, стоп, кистей) при збереженні пропорцій тіла. Зі збільшеною секрецією якого гормону це може бути пов’язане?

Соматостатин

Соматотропін

Кортизол

Тетрайодтиронін

Трийодтиронін

641 / 4299
У людини суттєво порушено перетравлення 6ілків, жирів та вуглеводнів. Знижена секреція якого травного соку, найвірогідніше, є причиною цього?

Шлунковий

Кишковий

Жовч

Слина

Підшлунковий

642 / 4299
В експерименті електричними імпульсами подразнюють нерв, що призводить до виділення малої кількості густої в’язкої слини підщелепною та під’язиковою залозами. Який нерв стимулюють?

N.glossopharyngeus

N.facialis

N.vagus

N.trigeminus

N.sympathicus

643 / 4299
У хворого в результаті травми порушене ковтання. Ураження якого з перерахованих відділів ЦНС є найбільш вірогідною причиною цього порушення?

Гіпоталамус

Довгастий мозок

Спинний мозок, Th II-IV

Середній мозок

Спинний мозок, C V-VI

644 / 4299
При визначенні енерговитрат організму людини методом непрямої калориметрії встановлено, що за одну хвилину споживається 1000 мл кисню і виділяється 800 мл вуглекислого газу. Яким є дихальний коефіцієнт у досліджуваної людини?

1,0

0,8

1,25

0,9

0,84

645 / 4299
У хворого на хронічний гломерулонефрит порушується інкреторна функція нирок. До дефіциту яких формених елементів крові це призведе?

Еритроцити

Тромбоцити

Лейкоцити

Лейкоцити та тромбоцити

Еритроцити та лейкоцити

646 / 4299
Після бігу на коротку дистанцію у нетренованих людей спостерігається м’язова крепатура внаслідок накопичення лактату. З посиленням якого біохімічного процесу в організмі це може бути пов’язано?

Глікогенез

Пентозофосфатний цикл

Гліколіз

Ліпогенез

Глюконеогенез

647 / 4299
У пацієнта, що страждає на хронічну ниркову недостатність, розвинувся остеопороз. Порушення синтезу в нирках якого регулятора мінерального обміну є основною причиною остеопо-розу?

Утворення 1,25(OH)2D3

Карбоксилювання глутамату

Гідроксилювання лізину

Гідроксилювання кортизолу

Гідроксилювання проліну

648 / 4299
У крові хворих на цукровий діабет спостерігається підвищення вмісту вільних жирних кислот (ВЖК). Причиною цього може бути:

Накопичення в цитозолі пальмітоїл-КоА

Активація утилізації кетонових тіл

Зниження активності фосф атидилхолін-холестеїн-ацилтрансферази плазми крові

Підвищення активності тригліцери-дліпази адипоцитів

Активація синтезу аполіпопротеїнів А-1, А-2, А-4

649 / 4299
Причиною захворювання на пелагру може бути переважне харчування кукурудзою та зниження у раціоні продуктів тваринного походження. Відсутність у раціоні якої амінокислоти призводить до даної патології?

Ізолейцин

Гістидин

Фенілаланін

Метіонін

Триптофан

650 / 4299
У хворого на системну склеродермію посилений розпад колагену. По- силення екскреції з сечею якої амінокислоти буде віддзеркалювати процеси деструкції колагену?

Серин

Оксипролін

Триптофан

Фенілаланін

Аланін

651 / 4299
Яка речовина надає слині в’язкий, слизовий характер, виконує захисну роль, у тому числі від механічного пошкодження слизової рота?

Муцин

Лізоцим

Амілаза

Глюкоза

Калікреїн

652 / 4299
Пародонтит супроводжується активацією протеолізу в тканинах пародон-ту. Підвищення якого компоненту ротової рідини свідчить про активацію протеолізу?

Органічні кислоти

Амінокислоти

Глюкоза

Холестерол

Біогенні аміни

653 / 4299
До неврологічного відділення з приводу мозкового крововиливу надійшов хворий 62-х років у важкому стані. Об’єктивно: спостерігається наростання глибини та частоти дихання, а потім його зменшення до апное, після чого цикл дихальних рухів відновлюється. Який тип дихання виник у хворого?

Чейн-Стокса

Апнейстичне

Гаспінг-дихання

Біота

Кусмауля

654 / 4299
Чоловік 52-х років. 3 роки тому назад переніс операцію видалення шлунка. В крові: 2,0 • 1012/л, Hb- 85 г/л, КП-1,27 Порушення засвоєння якого вітаміну викликало такі зміни?

C

B12

P

A

B6

655 / 4299
Під час роботи з радіоактивними речовинами працівник внаслідок аварії отримав дозу загального опромінення 4 Гр. Скаржиться на головний біль, нудоту, запаморочення. Які зміни в складі крові можна очікувати в хворого через 10 годин після опромінення?

Лейкопенія

Агранулоцитоз

Нейтропенія

Нейтрофільний лейкоцитоз

Лімфоцитоз

656 / 4299
Після введення місцевого анестетика у пацієнта розвинувся анафілактичний шок. Який механізм порушення кровообігу є провідним при цьому?

Зниження скоротливої функції серця

Зменшення тонусу судин

Активація симпато-адреналової системи

Гіперволемія

Біль

657 / 4299
У хворого на аденому клубочкової зони кори наднирників (хвороба Кон-на) спостерігаються артеріальна гіпертензія, напади судом, поліурія. Що є головною ланкою в патогенезі цих порушень?

Гіперсекреція катехоламінів

Гіпосекреція глюкокортикоїдів

Гіперсекреція глюкокортикоїдів

Гіпосекреція альдостерону

Гіперсекреція альдостерону

658 / 4299
Чоловік 30-ти років отримав опромінювання дозою близько 3 Гр. Яка зміна у крові буде через 8 годин після опромінювання?

Лімфопенія

Гранулоцитопенія

Анемія

Лейкопенія

Тромбоцитопенія

659 / 4299
Хворий взимку впав у ополонку, замерз на вітрі, захворів. Температура тіла піднялася до 39, 70C та коливалася від 39,00C до 39,80C. Назвіть тип температурної кривої в хворого:

Febris continua

Febris recurrens

Febris hectica

Febris intermittens

Febris remittens

660 / 4299
На розтині померлого 34-х років від ревматизму т-поверхня епікарду ворсиста, вкрита плівками сірого кольору, що легко відділяються. Після їх відокремлення визначається набрякла повнокровна поверхня. Який найбільш вірогідний діагноз?

Гнійний перикардит

Катаральний перикардит

Проліферативний перикардит

Фі6ринозний перикардит

Геморагічний перикардит

661 / 4299
У дівчинки 5-ти років спостерігаються висока температура і біль у горлі. Об’єктивно: набряк м’якого піднебіння, на мигдаликах сірі плівки, які важко відокремлюються, залишаючи глибокі кровоточиві дефекти тканини. Яке захворювання найбільш вірогідне?

Некротична ангіна

Ангіна Симановського-Венсана

Лакунарна ангіна

Інфекційний мононуклеоз

Дифтерія зіву

662 / 4299
У хворої людини деформовані кістки щелеп. Гістологічно виявлено розростання на місці кісток клітинно-волокнистої пухлиноподібної тканини з примітивним остеогенезом без чітких меж. Для якого захворювання характерна така картина?

Фіброзна дисплазія

Еозинофільна гранульома

Амелобластома

Паратиреоїдна остеодистрофія

Остеосаркома

663 / 4299
У біоптаті дужок м’якого піднебіння, взятого з приводу підозри на пухлину (макроскопічно визначалася виразка з щільним дном), виявлений некроз слизової оболонки з інфільтрацією підслизового шару лімфоцитами, епіте-ліоїдними клітинами, плазматичними клітинами, поодинокими нейтрофілами. Звертає увагу наявність вираженого ендо- та периваскуліту. Для якого захворювання властиві зазначені зміни?

Афтозний стоматит

Виразковий стоматит

Виразково-некротичний стоматит Венсана

Дифтерія зіву

Первинний сифіліс

664 / 4299
Під час морфологічного дослідження дна каріозної порожнини зуба чітко диференціюються три зони: розм’якшеного дентину, прозорого дентину, замісного дентину. Для якої стадії карієсу властиві ці зміни?

Стадія плями

Хронічний карієс

Глибокий карієс

Поверхневий карієс

Середній карієс

665 / 4299
Під час макроскопічного дослідження тканини легені виявлені ділянки підвищеної повітряності з наявністю дрібних пухирів, а гістологічно - стоншення та розрив альвеолярних перетинок з утворенням великих порожнин різної форми. Яке захворювання виявлене в легені?

Фіброзуючий альвеоліт

Кавернозний туберкульоз

Емфізема легень

Бронхоектатична хвороба

Хронічний бронхіт

666 / 4299
Чоловік 42-х років, що протягом 8-ми років хворів на хронічний грануле-матозний періодонтит, хронічний гнійний остеомієліт нижньої щелепи, помер при явищах хронічної ниркової недостатності. Яке ускладнення гнійного остеомієліту розвинулося в нирках?

Атрофія

Жирова дистрофія

Гіаліноз

Амілоїдоз

Некроз епітелію звивистих канальців

667 / 4299
У дитини 5-ти років підвищилася температура до 400C, виник різкий головний біль, блювання, неспокій, озноб. Через 4 дні з’явився геморагічний висип на шкірі, олігурія та надниркова недостатність, що і стало причиною смерті. Під час бактеріологічного дослідження мазків з глотки виявлений менінгокок. Яка форма менінгококової інфекції була в хворого?

Менінгококовий менінгіт

Менінгококовий назофарингіт

Менінгоенцефаліт

Менінгококцемія

668 / 4299
Після проведення туберкулінової проби (проба Манту) у дитини через 48 годин на місці введення туберкуліну утворилася папула розміром до 10 мм у діаметрі. Який механізм гіперчутливо-сті полягає в основі зазначених змін?

Антитілозалежна цитотоксичність

Гранулематоз

Імунокомплексна цитотоксичність

Клітинна цитотоксичність

Анафілаксія

669 / 4299
У хворого на слизовій оболонці порожнини рота з’явилося вогнище сірувато-білого кольору, щільне, яке виступає над слизовою оболонкою. Гістологічно в цій ділянці виявлений гіперкератоз, паракератоз та акантоз епітелію. Який патологічний процес розвинувся в слизовій оболонці?

Осередковий іхтіоз

Лейкоплакія

Гіаліноз

Локальний пухлинний амілоїдоз

Лейкодерма

670 / 4299
До інфекційної клініки надійшла дівчинка 7-ми років з високою температурою, скаргами на біль у горлі, загальну слабкість. Лікар запідозрив дифтерію. Що із перерахованого є вирішальним для підтвердження діагнозу після виділення чистої культури збудника?

Фаголізабельність

Виявлення у збудника волютинових зерен

Гемолітична здатність збудника

Проба на цистиназу

Проба на токсигенність

671 / 4299
В сироватці крові під час постановки імуноферментної реакції був визначений HBsантиген. При якому захворюванні зустрічається даний антиген?

Вірусний гепатит А

Сифіліс

Туберкульоз

СНІД

Вірусний гепатит В

672 / 4299
В першому класі було проведене медичне обстеження учнів з метою відбору дітей для ревакцинації проти туберкульозу. Яку з наведених нижче проб при цьому використали?

Проба Бюрне

Проба Манту

Проба Шика

Проба з антраксином

Нашкірна проба з тулярином

673 / 4299
Для вакцинації використовують токсин, знешкоджений формальдегідом (0,4%) при 37 — 400C протягом чотирьох тижнів. Вперше такий препарат застосував для профілактики дифтерії Рамон. Що це за препарат?

Антитоксична сироватка

Вбита вакцина

Ад’ювант

Імуноглобулін

Анатоксин

674 / 4299
При багатьох інфекційних хворобах у крові хворого можна виявити антигени збудника. Яку реакцію слід застосувати, враховуючи, що рівень антигене-мії низький?

Реакція латекс-аглютинації

Реакція непрямої гемаглютинації

Імуноелектрофорез

Реакція аглютинації

Твердофазний ІФА

675 / 4299
У хворого 30-ти років, який потрапив до клініки з діагнозом гострий гломерулонефрит, спостерігається проте-їнурія. Яке порушення спричинило це явище?

Зменшення кількості функціонуючих нефронів

Зниження онкотичного тиску плазми крові

Затримка виведення продуктів азотистого обміну

Підвищення гідростатичного тиску крові у капілярах

Підвищення проникності ниркового фільтра

676 / 4299
Для дезінфекції неметалічного інструментарію в хірургічному відділенні використали розчин формальдегіду. До якої групи за хімічною будовою відноситься даний антисептичний препарат?

Засоби ароматичного ряду

Детергенти

Спирти

Галогеновмісні сполуки

Засоби аліфатичного ряду

677 / 4299
Усі нестероїдні протизапальні засоби можуть пошкоджувати слизову шлунка. Для пошуку речовин, що не викликають цього ускладнення, необхідно знати, з чим воно пов’язане. Вплив на який молекулярний субстрат слід зменшити, щоб послабити вираженість цього ускладнення?

Калікреїн

Циклооксигеназа 2

Циклооксигеназа 1

Лізосомальні ферменти

Аденілатциклаза

678 / 4299
До стаціонару надійшов хворий з діагнозом виразкова хвороба 12-ти па-лої кишки у фазi загострення. Аналіз шлункового соку показав підвищення секреторної та кислотоутворюю-чої функції шлунка. Оберіть препарат, який знизить секреторну функцію залоз шлунка за рахунок блокади H2-рецепторів:

Екстракт красавки сухий

Метацин

Платифілін

Ранітидин

Атропін

679 / 4299
У хворого після оперативного втручання розвинувся парез кишечнику. Який засіб із групи антихолінестера-зних слід йому призначити?

Ацетилхолін

Пілокарпін

Карбахолін

Прозерин

Ацеклідин

680 / 4299
При лікуванні шкірних захворювань з вираженим запальним компонентом у щелепнолицевій ділянці в стоматологічній практиці використовують глю-кокортикоїди місцевого застосування. Які з нижче перерахованих препаратів мають найменшу резорбтивну дію?

Дексаметазон

Гідрокортизон

Флуметазона півалат

Триамцинолон

Преднізолон

681 / 4299
Каретою швидкої допомоги до лікарні доставлено хворого, який у стані важкої депресії намагався покінчити життя самогубством. Які фармакологічні засоби необхідно призначити хворому?

Нейролептики

Антидепресанти

Транквілізатори

Седативні

Солі літію

682 / 4299
У хворого на гіпертонічну хворобу розвинулася гостра серцева недостатність. Який з наведених засобів най- більш доцільно використати в даному випадку?

Кордіамін

Дігоксин

Кардіовален

Кофеїн

Корглікон

683 / 4299
Хворому призначено препарат з вираженими ліпофільними властивостями. Яким буде головний механізм його всмоктування?

Пасивна дифузія

Зв’язування з транспортними білками

Фільтрація

Активний транспорт

Піноцитоз

684 / 4299
Пародонтоз уражує у всьому світі до 50% населення віком після 30-ти років. Вирішальну роль в патогенезі даного захворювання відіграє:

Утворення зубного каменю мікрофлорою

Пошкодження пародонту активними факторами лейкоцитів

Пошкодження калікреїном тканин пародонту

Нервово-дистрофічний фактор

Імунні пошкодження тканин

685 / 4299
У 40-ка річного чоловіка із стенозу-ючим (без метастазів) раком стравоходу виявляються наступні зміни: атрофія скелетних м’язів та жирової клітковини. Шкіра землистокоричневого забарвлення, епідерміс потоншений, серце зменшене за розмірами. Міокард та печінка бурого кольору. Який найбільш вірогідний діагноз?

Ракова кахексія

Аліментарна кахексія

Бура атрофія

Хвороба Аддісона

Міастенія

686 / 4299
Хворий надійшов до клініки з пораненням в ділянці шиї. При обстеженні виявлено пошкоджений нерв, розташований попереду переднього драбинчастого м’яза. Який нерв ушкоджено?

Язикоглотковий

Діафрагмальний

Блукаючий

Шийний відділ симпатичного стовбуру

Під’язиковий

687 / 4299
До лікарні надійшов 9-ти річний хлопчик, розумово і фізично відсталий. При біохімічному аналізі крові виявлено підвищений вміст фенілаланіну. Блокування якого ферменту може призвести до такого стану?

Ілутаматдекарбоксилаза

Ілутамінтрансаміназа

Оксидаза гомогентизинової кислоти

Фенілаланін-4-монооксигеназа

Аспартатамінотрансфераза

688 / 4299
Хворому з закритим переломом плечової кістки накладена гіпсова пов’язка. Наступного дня з’явилися припухлість, синюшність і похолодання кисті травмованої руки. Про який розлад периферичного кровообігу свідчать ці ознаки?

Артеріальна гіперемія

Ішемія

Венозна гіперемія

Тромбоз

Емболія

689 / 4299
У дитини 2-х років з катаральними явищами та висипкою на шкірі лікар запідозрив скарлатину. Внутрішньо-шкірно дитині було введено невелику кількість сироватки до еритрогенного токсину стрептокока, на місці ін’єкції висип зник. Що означають результати реакції?

Всю дозу сироватки можна вводити внутрішньовенно

У дитини підвищена чутливість до еритрогенного токсину

Клінічний діагноз підтвердився

Імунна система дитини дуже ослаблена

Захворювання викликав негемоліти-чний стрептокок

690 / 4299
До лікаря звернулася вагітна жінка зі скаргами, характерними для токсоплазмозу. Для підтвердження клінічного діагнозу в неї взято кров. Яку серологічну реакцію необхідно поставити в цьому випадку?

Реакція зв’язування комплементу

Реакція преципітації

Реакція нейтралізації

Реакція Васермана

Реакція Відаля

691 / 4299
На електронній мікрофотографії представлена клітина, в якій відсутні ядерця та ядерна оболонка. Хромосоми вільно розміщені, центріолі мігру- ють до полюсів. В якій фазі клітинного циклу знаходиться клітина?

Метафаза

Телофаза

Інтерфаза

Профаза

Анафаза

692 / 4299
До лікаря-генетика звернувся юнак 18-ти років астенічної тілобудови: вузькі плечі, широкий таз, високий зріст, нерясна рослинність на обличчі. Виражена розумова відсталість. Було встановлено попередній діагноз: синдром Клайнфельтера. Який метод медичної генетики дозволить підтвердити даний діагноз?

Іенеалогічний

Популяційно-статистичний

Дерматогліфіка

Близнюковий

Цитогенетичний

693 / 4299
На електронній мікрофотографії представлена клітина нейрального походження, що знаходиться у складі епітелію слизової оболонки. Дистальна частина периферичного відростка клітини має булавоподібне потовщення, від якого відходять 10-12 війок. Що це за клітина?

Колбочкова зорова клітина

Біполярний нейрон спинномозкового вузла

Сенсорні епітеліоцити органа смаку

Паличкова зорова клітина

Нюхова клітина

694 / 4299
До лікаря звернувся пацієнт із скаргами на підвищену больову чутливість шкіри позаду вушної раковини та зовнішнього слухового проходу. Об’єктивно: пальпація позаду груднинно-ключично-соскоподібного м’язу болісна. Подразнення якого з нервів може дати таку клінічну картину?

N.auricularis magnus

N.vagus

Nn.supraclaviculares

N.occipitalis minor

N.transversus colli

695 / 4299
У чоловіка 50-ти років раптово виникло сильне серцебиття, біль у серці, різка слабкість, підвищення артеріального тиску; пульс неправильний з дефіцитом. На ЕКГ: відсутність зубця P і різні інтервали R — R. Яке порушення серцевого ритму в хворого?

Миготлива аритмія

Синусова екстрасистолiя

Поперечна блокада серця

Пароксизмальна тахікардія

Дихальна аритмія

696 / 4299
При електронно-мікроскопічному вивченні клітини виявлені кулясті пухирці, які обмежені мембраною i містять безліч різноманітних гідролітичних ферментів. Відомо, що ці органе-ли забезпечують внутрішньоклітинне травлення, захисні реакції клітини. Ці елементи:

Лізосоми

Рибосоми

Мітохондрії

Центросоми

Ендоплазматична сітка

697 / 4299
У хворої дитини з підозрою на дифтерію було взято на дослідження виділення ураженої слизової оболонки зіву. Приготовлений і забарвлений мазок. Під час мікроскопії виявлені жовті палички з темно-синіми потовщеннями на кінцях. Який структурний елемент мікробної клітини визначається у виявлених мікроорганізмів?

Плазміди

Джгутики

Спори

Зерна волютину

Капсула

698 / 4299
У хворого на 2-гу добу після розвитку інфаркту міокарда відбулося різке падіння систолічного артеріального тиску до 60 мм рт.ст. з тахікардією до 140/хв, задишкою, непритомністю. Який механізм є провідним у патогенезі шоку, що розвинувся?

Анафілактична реакція на міокарді-альні білки

Зменшення ударного об’єму крові

Інтоксикація продуктами некротичного розпаду

Пароксизмальна тахікардія

Зниження об’єму циркулюючої крові

699 / 4299
У хворого 42-х років скарги на біль в епігастральній ділянці, блювання; блювотні маси кольору 'кавової гущі', мелена. В анамнезі виразкова хвороба шлунка. У крові: ер.- 2,8 • 1012/л, лейк.-8 • 109/л, Hb- 90 г/л. Вкажіть найбільш вірогідне ускладнення, яке виникло у хворого:

Пенетрація

Пререродження в рак

Кровотеча

Перфорація

Пілоростеноз

700 / 4299
До лікарні звернулася хвора 38-ми років із скаргами на те, що після перенесеного гострого вірусного респіраторного захворювання вона втратила відчуття дотику їжі до передніх 2/3 язика, а також відчуття болю та температури (попекла язик гарячим чаєм). Вкажіть, яка з гілок якого нерва при цьому була ушкоджена?

Язикові гілки язикоглоткового нерва

Верхній гортанний нерв блукаючого нерва

Барабанна струна лицевого нерва

Язиковий нерв нижньощелепної гілки трійчастого нерва

Язикові гілки під’язикового нерва

701 / 4299
Хворий страждає брадиаритмією на фоні гіпертонічної хвороби. Який з лікарських засобів йому доцільно призначити?

Папаверину гідрохлорид

Метилдофа

Платифіліну гідротартрат

Клофелін

Резерпін

702 / 4299
У хворого спостерігається дерматит, діарея, деменція. При зборі анамнезу виявлено, що основним продуктом харчування хворого є кукурудза. З нестачею якого вітаміну пов’язані дані порушення?

Вітамін PP

Вітамін B1

Вітамін В2

Вітамін В8

Вітамін В9

703 / 4299
Зареєстровано спалах харчового отруєння, пов’язаний з використанням кондитерських виробів, що зберігались при кімнатній температурі та при виготовлені яких використовували качині яйця. Який мікроорганізм міг зумовити це захворювання?

Холерний вібріон

Легіонела

Стафілокок

Кишкова паличка

Сальмонела

704 / 4299
Хворий 20-ти років скаржиться на загальну слабкість, запаморочення, швидку втомлюваність. При обстеженні виявлено: гемоглобін крові 80 г/л, мікроскопічно - еритроцити зміненої форми. Причиною цього може бути:

Обтураційна жовтяниця

Гостра переміжна порфірія

Серпоподібноклітинна анемія

Хвороба Аддісона

Паренхіматозна жовтяниця

705 / 4299
Лікар записав в історії хвороби, що у хворого дихання поверхневе (знижена глибина дихання). Це означає, що зменшеним є такий показник зовнішнього дихання:

Функціональна залишкова ємність

Хвилинний об’єм дихання

Життєва ємність легень

Дихальний об’єм

Ємність вдиху

706 / 4299
На електронній мікрофотографії фрагменту кіркової речовини нирки виявляються клітини щільної плями та юкстагломерулярні клітини з великими секреторними гранулами. Яка структура нирки представлена на мікрофотографії?

Простагландиновий апарат

Судинний клубочок

Фільтраційний бар’єр

Ниркове тільце

Юкстагломерулярний апарат

707 / 4299
У людини визначили частоту серцевих скорочень за пульсом. Вона дорівнює 120 за хвилину. Яка при цьому тривалість серцевого циклу?

0,5 с

0,8 с

1,0 с

0,7 с

0,9 с

708 / 4299
Пацієнта на дачі вжалила бджола. Під час огляду: кисть лівої руки гаряча, рожева, набрякла, у місці укусу бджоли великий червоний пухир. Який з механізмів є провідним у розвитку набряку?

Зниження онкотичного тиску тканини

Пошкодження судин під час вжален-ня

Підвищення проникливості судин

Зниження кровонаповнення судин

Зниження осмотичного тиску тканини

709 / 4299
До травмпункту доставлено пацієнта із кровотечею з рваної рани в ділянці кута рота. Цілісність якої артерії порушена?

Лицева

Язикова

Верхньощелепна

Підочноямкова

Передня верхньокоміркова

710 / 4299
Під час проведення хірургічних маніпуляцій було використано новокаїн з метою знеболення. Через 10 хвилин у хворого з’явилися блідість шкірних покривів, задишка, гіпотензія. Алергічна реакція якого типу розвинулася в хворого?

Імунокомплексна

Анафілактична

Цитотоксична

Клітинно-опосередкована

Стимулююча

711 / 4299
Після операційного втручання експериментальна тварина загинула від сильних судом. Які ендокринні залози було видалено?

Щитоподібна

Прищитоподібні

Надниркові

Яєчники

Яєчка

712 / 4299
До лікаря звернувся пацієнт зі скаргами на запаморочення, погіршення пам’яті, періодичні судоми. Встановлено, що причиною таких змін є продукт декарбоксилювання глутамінової кислоти. Назвіть його:

ГАМК

ПАЛФ

ТДФ

АТФ

ТГФК

713 / 4299
До лікарні надійшов хворий зі скаргами на здуття живота, діарею, метеоризм після вживання білкової їжі, що свідчить про порушення травлення білків та їх посиленого гниття. Яка з пере- рахованих речовин є продуктом цього процесу в кишечнику?

!ндол

Агматин

Кадаверин

Бїлїрубїн

Путресцин

714 / 4299
До лікаря звернувся студент з проханням призначити препарат для лікування алергічного риніту, який виник у нього під час цвітіння липи. Який засіб можна застосувати?

Лоратадин

Лозартан

Норадреналіну гідротартрат

Амброксол

Анаприлін

715 / 4299
Спортсмену необхідно підвищити спортивні результати. Для цього йому рекомендовано вживати препарат, який містить карнітин. Який процес в найбільшому ступені активується цією сполукою?

Транспорт жирних кислот

Транспорт амінокислот

Транспорт глюкози

Транспорт вітаміну К

Транспорт іонів кальцію

716 / 4299
У зародка порушено процес сегментації дорзальної мезодерми й утворення сомітів. В якій частині шкіри можливі порушення розвитку?

Дерма

Потові залози

Сальні залози

Епідерміс

Волосся

717 / 4299
Жінка під час пологів втратила близько 800 мл крові. Відзначаються тахікардія, АТ100/70 мм рт.ст., тахі-пное до 28/хв. Який тип гіпоксії первинно розвивається при такій клінічній ситуації?

Серцево-судинна

Кров’яна

Тканинна

Дихальна

Змішана

718 / 4299
При лабораторному обстеженні дитини виявлено підвищений вміст у крові та сечі лейцину, валіну, ізолейцину та їх кетопохідних. Сеча мала характерний запах кленового сиропу. Недо- статність якого ферменту зумовила це захворювання?

Фосфофруктокіназа

Фосфофруктомутаза

Дегідрогеназа розгалужених амінокислот

Ілюкозо-6-фосфатаза

Амінотрансфераза

719 / 4299
Хворий звернувся з приводу карбункула на обличчі. Об’єктивно: нещільний, без болю набряк підшкірної клітковини, у центрі карбункулу чорний струп, по периферії везикулярні висипання навколо карбункула. Бактеріологічне дослідження з’ясувало наявність нерухомих стрептобацил, які здатні утворювати капсули. Які мікроорганізми є збудниками даної хвороби?

Bacillus megaterium

Bacillus subtilis

Bacillus anthracoides

Staphylococcus aureus

Bacillus antracis

720 / 4299
Хворий звернувся до лікарні із скаргами на шум та больові відчуття в вусі. Об’єктивно: у хворого гостре респіраторне захворювання, риніт. Крізь який отвір глотки інфекція потрапила до барабанної порожнини та викликала її запалення?

Ілотковий отвір слухової труби

Барабанний отвір слухової труби

Вхід до гортані

Хоани

Зів

721 / 4299
У тримісячного немовляти з’явився білий наліт на слизовій оболонці рота, язика та губ. Лікар запідозрив канди-доз. На яке поживне середовище необхідно зробити посів досліджуваного матеріалу для підтвердження діагнозу?

Ендо

Сабуро

Клауберга

Ру

Левенштейна-Йєнсена

722 / 4299
У процесі розвитку зуба в пері-одонті зберігаються залишки ембріональної кореневої піхви Іертвіга, що носять назву епітеліальних острівців Малассе. Вони можуть бути джерелом розвитку кіст або пухлин у ділянці кореня зуба. Якими клітинами утворена піхва Іертвіга?

Одонтобласти

Клітини емалевого органу

Мезенхімні клітини

Пульпоцити

Цементобласти

723 / 4299
На об’єктах зовнішнього середовища i харчових продуктах антигени ттти-гел Зонне виявляють у реакції з використанням діагностичної тест-системи, в набір якої входить полістероловий планшет з адсорбованими специфічними антитілами. Назвіть цю реакцію:

РЗПГА

РІФ

ІФА

РПГА

Реакція імуноелектрофорезу

724 / 4299
Інозитолтрифосфати в тканинах організму утворюються в результаті гідролізу фосфатидилінозитолдифосфа-тів і відіграють роль вторинних посередників (месенджерів) в механізмі дії гормонів. На що направлена їх дія у клітині?

Гальмування протеїнкінази С

Гальмування фосфодіестерази

Вивільнення іонів кальцію з клітинних депо

Активація протеїнкінази А

Активація аденілатциклази

725 / 4299
Синильна кислота та ціаніди належать до найсильніших отрут. Залежно від дози смерть наступає через декілька секунд чи хвилин. Пригнічення активності якого ферменту є причиною смерті?

Метгемоглобінредуктаза

Ацетилхолінестераза

АТФ-синтетаза

Каталаза

Цитохромоксидаза

726 / 4299
На препараті печінки внутрішньо-часточкові капіляри мають широкий просвіт, нерівномірний по всій довжині. Базальна мембрана у більшій частині капіляра відсутня. До якого типу відносяться такі капіляри?

Посткапіляри

Вісцерального типу

Соматичного типу

Прекапіляри

Синусоїдного типу

727 / 4299
У хворого з порушенням згортання кровї виявлений тромбоз однієї з гілок нижньої брижової артерії. Який відділ кишечнику вражений?

Colon ascendens

Ileum

Colon sigmoideum

Colon transversum

Caecum

728 / 4299
При ферментативних жовтяницях має місце порушення активності ферменту УДФглюкуронілтрасферази. Яка сполука накопичується в сироватці крові при цих патологіях?

Білівердин

Прямий білірубін

Непрямий білірубін

Мезобілірубін

Вердоглобін

729 / 4299
Хворий госпіталізований з приводу небезпеки поширення запального процесу з потиличної ділянки в порожнину черепа. Крізь яке анатомічне утворення можливе це поширення?

Круглий отвір

Тім ’яний отвір

Овальний отвір

Виростковий канал

Остистий отвір

730 / 4299
У хворого спостерігається алергічна реакція, яка супроводжується свербежем, набряками та почервонінням шкіри. Концентрація якого біогенного аміну підвищилась у тканинах?

Гамааміномасляна кислота

Гістамін

Триптамін

Серотонін

Дофамін

731 / 4299
Клінічне обстеження хворого дозволило встановити попередній діагноз: рак шлунка. В шлунковому соці виявлено молочну кислоту. Який тип катаболізму глюкози має місце в ракових клітинах?

Глюкозо-аланіловий цикл

Анаеробний гліколіз

Аеробний гліколіз

Пентозофосфатний цикл

Глюконеогенез

732 / 4299
Характерними ознаками холери є втрата організмом великої кількості води та іонів натрію. Який механізм лежить в основі виникнення діареї при цьому?

Активація аденілатциклази ентеро-цитів

Гальмування синтезу вазопресину в гіпоталамусі

Окислення альдостерону в корі на-днирників

Посилення синтезу кортикотропіну

Посилення секреції реніну клітинами ниркових артеріол

733 / 4299
У хворого на цукровий діабет після оперативного втручання (розтин абсцесу в задній ділянці шиї) загоєння рани відбувалося протягом 1,5 місяців, з рани постійно виділявся гнійний вміст. Після загоєння рани на її місці утворився грубий рубець. У який спосіб загоювалася рана?

Змішаний

Вторинний

Первинний

Епітелізація

Під струпом

734 / 4299
У хворого з черепно-мозковою травмою спостерігається зупинка дихання. Пошкодження якого відділу мозку є найбільш вірогідним?

Середній мозок

Кінцевий мозок

Мозочок

Проміжний мозок

Довгастий мозок

735 / 4299
В експерименті на тварині зруйнували середню частину завитки. Це призвело до порушення сприйняття звукових коливань такої частоти:

Низької та середньої

Високої

Середньої

Низької

Високої та середньої

736 / 4299
У людини збільшена частота серцевих скорочень, розширені зіниці, сухість у роті. Це є наслідком активації в організмі такої системи регуляції функцій:

Метасимпатична

Симпатична

Ваго-інсулярна

Гипоталамо-гіпофізарно-надниркова

Парасимпатична

737 / 4299
Пацієнт, що хворіє на хронічний бронхіт, приймає синтетичний муколі-тичний препарат, який сприяє розрі- джуванню харкотиння. Назвіть цей препарат:

Ацетилцистеїн

Гепарин

Діазепам

Фуросемід

Еналаприл

738 / 4299
У хворого, що знаходиться у опіковому відділенні, виникло гнійне ускладнення. Гній, що виділяється, має синьо-зелений відтінок, що вказує на інфекцію, зумовлену Pseudomonas aeruginosa. Яка ознака характерна для цього збудника?

Наявність спор

Розташування клітин парами

Кокова форма

Утворення міцелію

Негативне забарвлення за Грамом

739 / 4299
У дитини 10-ти років на альвеолярному відростку нижньої щелепи виявлене нерухливе пухлиноподібне утворення 1,5 см у діаметрі, що закриває коронку премолярів з вестибулярного боку. Слизова оболонка на його поверхні червоно-бура, кровоточить при незначній механічній дії. Дані біопсії: утворення складається з судин дрібного калібру, розділених тонкими прошарками сполучної тканини, інфільтрованими плазмоцитами, слизова оболонка на деяких ділянках звиразкована. Який найбільш вірогідний діагноз?

Гіпертрофічний гінгівіт

Гігантоклітинна форма епулісу

Фіброзна форма епулісу

Ангіоматозна форма епулісу

Фіброматоз ясен

740 / 4299
Після закривання рота і стискання зубів відбувається його рефлекторне відкривання. З яких рецепторів починається зазначений рефлекс?

Пропріорецептори м’язів, що опускають нижню щелепу

Механорецептори слизової ротової порожнини

Смакові рецептори

Пропріорецептори м’язів, що піднімають нижню щелепу

Рецептори періодонту

741 / 4299
До медико-генетичної консультації звернулось подружжя з питанням про вірогідність народження у них дітей з X-зчепленою формою рахіту (домінантна ознака). Батько здоровий, мати ге- терозиготна і страждає на це захворювання. На вітаміностійкий рахіт можуть захворіти:

Половина дочок i синів

Всі діти

Тільки дочки

Тільки сини

Всі діти будуть здорові

742 / 4299
У хворого на ентерит, що супроводжувався значною діареєю, спостерігається зменшення кількості води в позаклітинному просторі, збільшення її всередині клітин та зниження осмолярності крові. Як називають таке порушення водно-електролітного обміну?

Гіперосмолярна гіпергідратація

Гіпоосмолярна гіпогідратація

Гіпоосмолярна гіпергідратація

Гіперосмолярна гіпогідратація

Осмолярна гіпогідратація

743 / 4299
У хворого для обробки опікової поверхні шкіри був використаний препарат, антисептичні властивості якого забезпечуються вільним киснем, що виділяється в присутності органічних речовин. Оберіть правильну відповідь:

Хлоргексидин

Натрію гідрокарбонат

Калію перманганат

Фурацилін

Розчин йоду спиртовий

744 / 4299
Хворому 30-ти років бактеріологічно підтверджений діагноз амебної дизентерії. Вкажіть препарат вибору для лікування цього захворювання:

Фурацилін

Метронідазол

!траконазол

Ацикловір

Мебендазол

745 / 4299
Хворому чоловіку 75-ти років, в якого частота серцевих скорочень була 40/хв, імплантували серцевий електростимулятор. Після цього ЧСС зросла до 70/хв. Функцію якого відділу серця взяв на себе електростимулятор?

Волокон пучка Гіса

Ніжки Гіса

Синоатрільного вузла

Волокон Пуркін’є

Атріовентрикулярного вузла

746 / 4299
Людина, що тривалий час знаходилася у задушливому приміщенні, зне- притомніла. Свідомість відновилася після вдихання парів нашатирного спирту. З прямим впливом на які структури пов’язана дія цієї речовини?

Ємкісні судини

Дихальний центр

Рецептори верхніх дихальних шляхів

Судиноруховий центр

Резистивні судини

747 / 4299
У хворого 40-а років в результаті щелепно-лицьової травми порушилася функція під’язикової і підщелепної залоз зліва - залози почали секретувати невелику кількість густої слини. Функція якого нерва порушена?

Трійчастий

Язикоглотковий

Лицьовий

Під’язиковий

Блукаючий

748 / 4299
При огляді порожнини рота дитини педіатр знайшла 8 різців. Розвиток дитини відповідає віковій нормі. Визначте вік дитини:

7-8 місяців

12-15 місяців

16-20 місяців

10-12 місяців

6-7 місяців

749 / 4299
У хворого на жовтяницю у крові виявлено збільшення загального білірубіну за рахунок непрямої його фракції. Сеча і кал інтенсивно забарвлені. Який найбільш вірогідний механізм вказаних порушень?

Пошкодження паренхіми печінки

Порушення утворення прямого білірубіну

Утруднення відтоку жовчі з печінки

Порушення перетворення уробіліно-гену в печінці

Підвищений гемоліз еритроцитів

750 / 4299
У потерпілого артеріальна кровотеча з різаної рани в ділянці переднього краю жувального м’яза. Яку судину потрібно перев’язати?

A.lingualis

A.mentalis

Aa.labiales inferiores

A.maxillaris

A.facialis

751 / 4299
В стоматологічному кабінеті необхідно терміново провести дезинфе- кцію обладнання. Оберіть препарат з відсутністю неприємного запаху і фарбувальних властивостей:

Хлоргексидину біглюконат

Розчин карболової кислоти

Хлорне вапно

Етакридину лактат

Формалін

752 / 4299
Хворий проходив чергове обстеження, в результаті якого у нього виявлено гіперглікемію, кетонурію, поліурію, глюкозурію. Яка форма КОС має місце за наявності цих явищ?

Метаболічний ацидоз

Газовий ацидоз

Газовий алкалоз

Негазовий ацидоз

Метаболічний алкалоз

753 / 4299
Після перенесеної стафілококової інфекції у хворої з’явився набряковий синдром (анасарка). У сечі виявлена масивна протеїнурія. У крові: гіпопро-теїнемія, гіперліпідемія. Яку патологію можна припустити?

Сечокам’яна хвороба

Пієлонефрит

Нефротичний синдром

Гломерулонефрит

ХНН

754 / 4299
На прийомі у лікаря-стоматолога в хворого стався напад стенокардії. Який лікарський засіб, що відноситься до групи нітратів, доцільно використати в даному випадку?

Ментол

Талінолол

Валідол

Нітрогліцерин

Ериніт

755 / 4299
Хворий звернувся до поліклініки зі скаргами на утруднення під час сечовиділення. Після огляду виявлена гіпертрофія органу, який охоплює проксимальний відділ уретри. Що це за орган?

Сім’яні міхурці

Бульбо-уретральна залоза

Цибулина статевого члена

Придаток яєчка

Передміхурова залоза

756 / 4299
При обстеженні дитини виявлено: розростання глоткового мигдалика, що утруднює вихід повітря із носової порожнини. Які отвори носової порожнини перекриваються при цьому захворюванні?

Хоани

Ніздрі

Крило-піднебінний

Грушоподібний

Гайморова щілина

757 / 4299
У новонародженого виявляється гиперемія, набряк на слизовій рота, невеликі ерозії з в’язкими слизово-гнійними виділеннями. У мазках з виділень виявляється велика кількість лейкоцитів, що містять грамнегативні диплококи. Такі ж мікроорганізми розташовуються і поза лейкоцитами. Який діагноз можна припустити?

Вроджений сифіліс

Стафілококовий стоматит

Гонококовий стоматит

Бленорея

Токсоплазмоз

758 / 4299
Під час розтину трупа чоловіка 70-ти років, який тривало хворів на атеросклероз і помер при явищах серцево-судинної недостатності, в черевному відділі аорти при макроскопічному дослідженні спостерігаються щільні, овальної форми фіброзні бляшки з відкладенням вапна у вигляді щільних, ламких пластин. Вкажіть стадію морфогенезу атеросклерозу:

Атерокальциноз

Атероматоз

Ліпосклероз

Звиразкування

Ліпоїдоз

759 / 4299
У хворого діагностовано кореневу кісту, яка проросла у носову порожнину. Який зуб найбільш вірогідно уражений?

Верхній медіальний різець

Верхній латеральний різець

Верхнє ікло

Верхній перший моляр

Верхній перший премоляр

760 / 4299
Хворий, який протягом багатьох років страждає на захворювання нирок, помер при явищах уремії. На розтині: нирки різко зменшені за розміром, щільні, дрібнозернисті, світло-сірого кольору. Як називаються нирки з такими змінами?

Строкаті

Сальні

Вторинно зморщені

Первинно зморщені

761 / 4299
З метою профілактики запалення ясен та покращення регенерації епітеліальних клітин пародонту до зубних паст додають один з наступних вітамінів:

Тіамін

Біотин

Філохінон

Ретинол

Кальциферол

762 / 4299
Після закладки молочних зубів на початку 5-го місяця ембріогенезу певними чинниками пошкоджено ростову властивість зубної пластинки позаду вказаних закладок. Який можливий важкий наслідок?

Порушиться утворення піхви Іертвіга

Не відбудеться закладка постійних зубів

Постраждає процес утворення при-сінку ротової порожнини

Не відбудеться утворення дентину молочних зубів

Не руйнуватиметься шийка емалевого органа

763 / 4299
При дослідженні трубчастого органу встановлено, що його середня оболонка складається із гіалінових суцільних кілець. Який епітелій вистилає слизову оболонку цього органа?

Одношаровий кубічний

Багатошаровий плоский незрогові-лий

Одношаровий призматичний залозистий

Багаторядний призматичний війчастий

Одношаровий призматичний з облямівкою

764 / 4299
Новонароджена дитина погано набирає вагу, в сечі виявлено підвищений вміст оротової кислоти, що свідчить про порушення синтезу піримідинових нуклеотидів. Який метаболіт необхідно використати для нормалізації метаболізму?

Тимідин

Аденозин

Уридин

Іуанозин

Гістидин

765 / 4299
При гістологічному дослідженні періапікальної тканини, видаленої у хворого, що тривалий час хворів на хронічний періодонтит, було виявлено грануляційну тканину, пронизану тяжами плоского епітелію, оточену фіброзною капсулою. Який найбільш вірогідний діагноз?

Проста гранульома

Абсцедуючий періодонтит

Складна гранульома

Гранулюючий періодонтит

Кістогранульома

766 / 4299
На мікропрепараті, зробленому з невідомого органа, виявлені ацинуси, які містять 10-15 клітин конічної форми з базофільною цитоплазмою, круглим ядром і добре розвиненою гранулярною ендоплазматичною сіткою. Аци-нус оточений базальною мембраною, в розщепленні якої локалізуються мі-оепітеліальні клітини. З якого органа зроблено зріз?

Підшлункова залоза

Під’язикова залоза

Привушна слинна залоза

Печінка

Легені

767 / 4299
Під час операції з приводу гранульоми в ділянці правого верхнього різця виникла кровотеча, яку вдалося зупинити тільки через 3 години. В анамнезі хворого хронічний лімфолейкоз. Що найбільш вірогідно стало причиною кровотечі?

Лейкопенія

Тромбоцитопенія

Тромбоцитопатія

Еозинофілія

Лімфоцитоз

768 / 4299
У дитини 5-ти років спостерігається покреслені жовтувато-коричневими смугами емаль і дентин, оголення дентину, множинний карієс. Відомо, що під час вагітності мати приймала антибіотики. Який препарат міг спричинити таку побічну дію?

Ністатин

Ампіцилін

Стрептоцид

Лінкоміцин

Тетрациклін

769 / 4299
При вивченні шліфу зуба чоловіка 40-а років на дентино-емалевій межі визначені оптично щільні лінійні структури довжиною до 1/3 товщі емалі веретеноподібної форми. Що за структури визначив дослідник?

Каріозне ушкодження

Емалеві пучки

Емалеві веретена

Дентиклі

'Мертві'шляхи

770 / 4299
Емаль характеризується високою стійкістю до дії різних механічних та хімічних факторів. Синтез якого компоненту забезпечує таку резистентність?

Гідроксиапатит

Колаген

Карбонатний апатит

Фторапатит

Хлорапатит

771 / 4299
Лікар-стоматолог знайшов неглибоку порожнину з ураженням емалі між двома верхніми центральними різцями. Діагноз: карієс. На якій поверхні коронки лікар пломбує зуб?

Facies occlusialis

Facies contactus mesialis

Facies contactus distalis

Facies lingualis

Facies labialis

772 / 4299
Зроблено мікропрепарат м’якого піднебіння, на якому визначаються ротова і носова поверхня. На ротовій поверхні виявлено пошкодження епітелію. Який епітелій пошкоджено?

Багатошаровий призматичний незро-говілий

Багатошаровий кубічний незрогові-лий

Багатошаровий плоский незрогові-лий

Багаторядний миготливий

Багатошаровий плоский зроговілий

773 / 4299
При регенерації епітелію слизової оболонки порожнини рота (розмноження клітин) відбулася реплікація (авторепродукція) ДНК за напівконсерва-тивним механізмом. При цьому нукле-отиди нової нитки ДНК є комплемен- тарними до:

Змістовних кодонів

Інтронних ділянок гену

Ферменту РНК-полімерази

Материнської нитки

Ферменту ДНК-полімерази

774 / 4299
У лабораторії група дослідників експериментально отримала без’ядер-цеві мутантні клітини. Синтез яких сполук буде в них порушений у першу чергу?

Полісахариди

Рибосомна РНК

Транспортна РНК

Моносахариди

Ліпіди

775 / 4299
У хворого з синдромом Іценко-Кушінга спостерігаються стійка гіперглікемія та глюкозурія, гіпертензія, остеопороз, ожиріння. Синтез та секреція якого гормону збільшується в даному випадку?

Кортизол

Глюкагон

Адреналін

Альдостерон

Тироксин

776 / 4299
У хворого, що тривалий час лікувався в клініці нервових хвороб седативним засобом, виникло ускладнення: кашель, нежить, сльозотеча. Який препарат призвів до таких порушень?

Діазепам

Феназепам

Натрію бромід

Валеріана

Резерпін

777 / 4299
У хворого інфаркт міокарда задньої стінки правого шлуночка. Гілки якої артерії тромбовані?

Ліва та права коронарна артерія

Права підключична артерія

Ліва підключична артерія

Права коронарна артерія

Ліва коронарна артерія

778 / 4299
При тривалому перебуванні у погано вентильованому приміщені у студента збільшилась частота дихання. Які рецептори найшвидше відреагували на збільшення концентрації вуглекислого газу в повітрі?

Іритантні рецептори

Нюхові

Юкстагломерулярні

Хеморецептори судин

Центральні хеморецептори

779 / 4299
Проводять дослідження на ізольованому м’язовому волокні. Встановлено, що поріг сили подразнення клітини суттєво зменшився. Що з вказаного може бути причиною цього?

Активація калієвих каналів мембрани

Активація натрієвих каналів мембрани

Блокада енергоутворення у клітині

Інактивація натрієвих каналів мембрани

Інактивація кальцієвих каналів мембрани

780 / 4299
При диспансерному обстеженні хлопчику 7-ми років встановлено діагноз - синдром ЛешаНайхана (хворіють тільки хлопчики). Батьки здорові, але у дідуся за материнською лінією таке ж захворювання. Який тип успадкування захворювання?

Автосомно-рецесивний

Автосомно-домінантний

Домінантний, зчеплений із статтю

Рецесивний, зчеплений із статтю

Неповне домінування

781 / 4299
В кардіологічному відділенні в хворого виникла аритмія. Лікар призначив аміодарон. Який основний механізм протиаритмічної дії аміодарону?

Пригнічує холінорецептори

Змінює чутливість міокарду до ацетилхоліну

Переважно блокує калієві канали

Стимулює гістамінові рецептори

Активує серотонінові рецептори

782 / 4299
При переливанні крові у пацієнта почав розвиватися внутрішньосудин-ний гемоліз еритроцитів. Гіперчутли-вість якого типу розвинулась у пацієнта?

Гіперчутливість І типу (анафілактична)

Гіперчутливість II типу (антитілоза-лежна)

Гіперчутливість IV типу (клітинна цитотоксичність)

Гіперчутливість III типу (імуноком-плексна)

Гіперчутливість V типу (гранулема-тоз)

783 / 4299
В результаті експресії окремих компонентів геному клітини зародка набувають характерних для них морфологічних, біохімічних та функціональних особливостей. Яку назву має цей процес?

Рецепція

Диференціювання

Капацитація

Детермінація

Адукція

784 / 4299
До приймального відділення лікарні доставлено людину з гострим отруєнням морфіном. Який специфічний антагоніст наркотичних аналгетиків необхідно застосувати в цьому випадку?

Унітіол

Дігоксин

Налоксон

Парацетамол

Метацин

785 / 4299
Хворому з набряками призначено К+-зберігаючий діуретик - антагоніст альдостерону. Визначте цей препарат:

Алопуринол

Дігоксин

Новокаїнамід

Спіронолактон

Клофелін

786 / 4299
Для зниження температури тіла при лихоманці і зменшення зубного болю хворому призначили парацетамол. Який механізм дії цього препарату?

Блокування ліпооксигенази

Блокування фосфодіестерази

Блокування моноамінооксидази

Блокування циклооксигенази

Блокування холінестерази

787 / 4299
При втручанні з метою лікування вивиху нижньої щелепи, лікар повинен пам’ятати про дію м’яза, задні пучки якого тягнуть назад висунуту нижню щелепу. Про який з м’язів йдеться?

M.pterygoideus lateralis

M.masseter

M.mylohyoideus

M.temporalis

M.pterygoideus medialis

788 / 4299
У хворого при обстеженні було виявлено зміну секреторної функції привушної слинної залози, що пов’язане з порушенням її вегетативної іннервації. Який з вузлів вегетативної нервової системи віддає післявузлові парасимпатичні волокна для неї?

Ganglion submandibulare

Ganglion oticum

Ganglion sublinguale

Ganglion ciliare

Ganglion pterygopalatinum

789 / 4299
Під час огляду зуба у його коронці виявлена велика порожнина, дном якої є вузький шар розм’якшеного дентину, що відділяє цю порожнину від пульпи. Який найбільш вірогідний діагноз?

Глибокий карієс

Періодонтит

Пульпіт

Поверхневий карієс

Середній карієс

790 / 4299
Стоматолог, оглядаючи хворого, звернув увагу на надмірне слиновиділення. Лікар застосував препарат, що спричиняє сухість в ротовій порожнині. Вкажіть цей препарат:

Атропіну сульфат

Фентоламін

Пілокарпіну гідрохлорид

Галантамін

Прозерин

791 / 4299
При мікроскопічному дослідженні інфільтрату, котрий був видалений із піднижньощелепної ділянки шкіри хворого 30-ти років, виявлено: осередки гнійного розплавлення, які оточені дозріваючими грануляціями та зрілою сполучною тканиною, в гної - друзи, які складаються з численних коротких паличкоподібних елементів, прикріплених одним кінцем до гомогенного центру. Визначте, яке захворювання в хворого?

Актиномікоз

Сифіліс

Кандидоз

Туберкульоз

792 / 4299
При мікроскопічному дослідженні операційного матеріалу (частина губи з виразкою), біля країв і під дном виразкового дефекту в сполучній тканині слизової оболонки, виявлено епітеліальні комплекси з атипового багатошарового епітелію, з фігурами патологічного мітозу. В центрі комплексів накопичення яскраво-рожевих концентричних утворів. Яка патологія розвинулася?

Базальноклітинний рак

Плоскоклітинний рак з ороговінням

Перехідноклітинний рак

Плоскоклітинний рак без ороговіння

Папілома

793 / 4299
Хвора на рак нижньої щелепи 52-х років пройшла курс променевої терапії. Розмір пухлини зменшився. Який з наведених механізмів ушкодження клітини найбільш обумовлює ефективність променевої терапії?

Лізис NK-клітинами

Тромбоз судин

Утворення вільних радикалів

Гіпертермія

Мутагенез

794 / 4299
Лікарю необхідно знеболити передній відділ слизової оболонки твердого піднебіння. Блокаду яких нервів йому потрібно зробити?

Виличні нерви

Підочноямкові нерви

Носо-піднебінні нерви

Нижні коміркові нерви

Глоткові нерви

795 / 4299
Під час рентгенологічного дослідження у хворого виявлена кіста у ділянці премоляра, у порожнині якої міститься зуб. Мікроскопічно: стінка кісти представлена сполучною тканиною і вистелена багатошаровим плоским епітелієм. Який найбільш вірогідний діагноз?

Фолікулярна кіста

Еозинофільна гранульома

Радикулярна кіста

Епуліс

Примордіальна кіста

796 / 4299
При обстеженні хворого встановлено, що причиною гіпоплазії зубів є гіповітаміноз А та D. Ці вітаміни призначили перорально, проте лікувального ефекту не досягли. Яка можлива причина порушення засвоєння вітамі- нів?

Гіпохлоргідрія

Ахілія

Нестача жовчних кислот

Гіпєрхлоргідрія

Ахлоргідрія

797 / 4299
Під час розтину жінки 35-ти років поряд із збільшенням багатьох лімфатичних вузлів була виявлена збільшена селезінка вагою 600,0; на розрізі неоднорідна, темно-червоного кольору, щільна, з ділянками некрозу сірувато-жовтуватого кольору, в діаметрі до 1 см (порфірова селезінка). Про яке захворювання можна думати?

Лімфогранулематоз

Хронічний мієлоїдний лейкоз

Метастази раку

Лімфосаркома

Хронічний лімфоїдний лейкоз

798 / 4299
У дитини 9-ти років після тривалого прийому антибіотиків з приводу бронхопневмонії з’явилися біль та печіння в ділянці слизової оболонки губ та язика. Об’єктивно: на слизовій оболонці губ, язика сирнисті нальоти та сірі бляшки, які легко знімаються шпателем, залишаючи на своєму місці осередки гіперемії. При мікроскопічному досліджені нальоту виявлено міцелій. Який найбільш вірогідний діагноз?

Контактний алергічний хейліт

Ексфоліативний хейліт

Лейкоплакія

Кандидозний хейліт

Хейліт Манганотті

799 / 4299
У пацієнта герпетичний стоматит на фоні імунодепресії. Який препарат, що застосовують внутрішньовенно, всередину і місцево, забезпечить противірусну та імуностимулюю-чу дію?

Левамізол

Ремантадин

Ацикловір

Метисазон

Амоксицилін

800 / 4299
При огляді порожнини рота чоловіка 60-ти років виявлені наступні зміни: 26 і 27 зуби вкриті металічними коронками, які глибоко заходять під ясна. Між ними пародонтальна кишеня глибиною 0,7 см з незначною кількістю гною. Ясенні сосочки цих зубів гіперемовані, набряклі, з ціанотичним відтінком, при дотику зондом кровоточать. На рентгенограмі - резорбція міжзубних перегородок на 1/2 довжини кореня. Який найбільш вірогідний діагноз?

Гіпертрофічний гінгівіт

Локальний пародонтит

Генералізований пародонтит

Хронічний катаральний гінгівіт

801 / 4299
У чоловіка виявлено протозойне захворювання, при якому вражений головний мозок i спостерiгається втрата зору. При аналiзi крові знайдеш одноклітинні організми півмісяцєвої форми з загостреним кінцем. Збудником цього захворювання є:

Лямблія

Токсоплазма

Амеба

Трихомонада

Лейшманія

802 / 4299
У медико-генетичну консультацію звернулося подружжя у зв’язку з народженням дитини з багатьма вадами розвитку (мікроцефалія, ідіотія, тощо). Жінка під час вагітності хворіла, але мутагенів та тератогенів не вживала. Каріотип батьків і дитини нормальний. Під час збору анамнезу з’ясовано, що сім’я у квартирі тримає кота. Яке захворювання матері під час вагітності, найбільш вірогідно стало причиною вад розвитку у дитини?

Балантидіаз

Трихомоноз

Лейшманіоз

Дизентерія

Токсоплазмоз

803 / 4299
У хворого має місце розумова відсталість, низький зріст, короткопалі руки та ноги, монголоїдний розріз очей. Вивчення каріотипу показало наявність трисомії за 21-ою парою хромосом. Як називається ця хромосомна аномалія?

Хвороба Дауна

Синдром Тернера

Синдром Клайнфельтера

Трисомія за X-хромосомою

Специфічна фетопатія

804 / 4299
У хворої симптоми запального процесу сечостатевих шляхів. У мазку із слизової оболонки піхви виявлено великі одноклітинні організми грушоподібної форми з загостреним шипом на задньому кінці тіла, великим ядром та ундулюючою мембраною. Які найпростіші знайдені в мазку?

Trichomonas hominis

Lamblia intestinalis

Trichomonas vaginalis

Trypanosoma gambiense

Trichomonas buccalis

805 / 4299
Під час дослідження електронограми у клітині виявлено деструкцію мітохондрій. Які клітинні процеси можуть бути порушені внаслідок цього?

Кросинговер

Поділ ядра

Дроблення

Окиснення органічних речовин

806 / 4299
Експериментально (дією мутагенних факторів) у клітині порушено формування субодиниць рибосом. На якому метаболічному процесі це позначиться?

Біосинтез білка

Біосинтез вуглеводів

Фотосинтез

Біологічне окиснення

Синтез АТФ

807 / 4299
При обстеженні лікарями СЕС працівників сфери громадського харчування нерідко виявляється безсимптомне паразитоносійство, коли клінічно здорова людина є джерелом цист, які за-ражують інших людей. Для паразитування якого збудника неможливе таке явище?

Дерматотропні лейшманії

Дизентерійна амеба

Кишкова трихомонада

Малярійний плазмодій

Вісцеротропні лейшманії

808 / 4299
Відомо, що інформацію про послідовність амінокислот у молекулі білка записано у вигляді послідовності чотирьох видів нуклеотидів у молекулі ДНК, причому різні амінокислоти кодуються різною кількістю триплетів -від одного до шести. Як називається така особливість генетичного коду?

Неперекривність

Універсальність

Виродженість

Триплетність

Специфічність

809 / 4299
У хворого з карієсом нижнього лівого премоляра з’явилася припухлість шиї вище під’язикової кістки. Підвищилася температура, з’явилася слинотеча, скорочення жувальних м’язів; хворий з утрудненням відкриває рот. Встановлено діагноз - флегмона дна ротової порожнини. Які м’язи будуть залучені у процес?

Під’язиково-щічний і шилоязиковий

Щелепно-під’язиковий i підборідно-під’язиковий

Підшкірний і шилопід’язиковий

Двочеревцевий і під’язиково-шилоподібний

Щитопід’язиковий і груднино-під’язиковий

810 / 4299
Під час виконання вагосимпати-чної блокади за Вишневським вводять розчин новокаїну по задньому краю груднино-ключично-соскоподібного м’яза вище місця перетинання його з зовнішньою яремною веною. У межах якого трикутника шиї виконують блокаду?

Трикутник Пирогова

Сонний

Лопатково-трапецієподібний

Піднижньощелепний

Ключично-лопатковий

811 / 4299
Під час підходу до щитоподібної залози з поперечного (коміроподібно-го) доступу розтинається клітковинний надгруднинний простір. Пошкодження якого анатомічного утворення, що знаходиться в даному просторі, є небезпечним?

Підключична артерія

Лімфатичні вузли

Венозна яремна дуга

Сонна артерія

Внутрішня яремна вена

812 / 4299
Хвора з діагнозом пієлонефрит надійшла до урологічного відділення. Після проведення обстеження відмічено приєднання інфекції, що супроводжувалося пієловенозним рефлюксом. Ураження якої структури призвело до виникнення ускладнення?

Ниркові канальці

Екскреторні шляхи нирки

Форнікальний апарат нирки

Прямі трубочки

Ниркове тільце

813 / 4299
Хворий 27-ми років звернувся до лікаря зі скаргою на наявність твердої пухлини попереду від козелка вушної раковини. Хірург-стоматолог, видаляючи пухлину, виявив вену. Яка вена проходить у даній ділянці?

V retromandibularis

V auricularis posterior

V jugularis interna

V jugularis externa

V facialis

814 / 4299
Хворий 24-х років звернувся до лікаря зі скаргою на біль під нижньою щелепою з правого боку. Хірург-стоматолог виявив камінь у піднижньо-щелепній залозі. Видаляючи його, він запобігав кровотечі з артерії:

A. facialis

A. submentalis

A. labialis inferior

A. lingualis

A. alveolaris inferior

815 / 4299
При обстеженні міміки хворого виявлено, що він не може свистіти, скласти губи трубочкою; під час сміху кутки рота не піднімаються догори, ротова щілина розтягується в боки (поперечна посмішка). На атрофію якого м’яза вказують дані симптоми?

Великий виличний м’яз

Шийний м’яз

Жувальний м’яз

М’яз сміху

Коловий м’яз рота

816 / 4299
При огляді травмованого у дорожній пригоді лікар встановив ураження зовнішньої стінки очної ямки. Потерпілий втратив можливість відведення очного яблука на травмованому боці. Який нерв міг бути ураженим у даному випадку?

N. ophthalmicus

N. infraorbitalis

N. oculomotorius

N. trochlearis

N. abducens

817 / 4299
Після видалення другого великого кутнього зуба верхньої щелепи у хворого відмічається кровотеча з альвеолярної комірки. З системи якої артерії спостерігається кровотеча?

Висхідна глоткова

Лицева

Верхньощелепна

Щелепно-під’язикова

Нижня альвеолярна

818 / 4299
Спортсмен-баскетболіст скаржиться на біль вище п’яти, що підсилюється під час ходіння. Сухожилок якого м’язу ушкоджено?

M. triceps surae

M. fibularis longus

M. tibialis posterior

M. fibularis brevis

M. flexor digitorum longus

819 / 4299
Випадково вдарившись ліктем об край стола, студент відчув печіння і поколювання на внутрішній поверхні пе-рєдпліччя. Який нерв був травмований в цьому випадку?

N. axillaris

N. medianus

N. radialis

N. musculocutaneus

N. ulnaris

820 / 4299
У цитоплазмі клітин підшлункової залози в процесі секреторного циклу в апікальній частині з’являються і зникають гранули секрету. До яких структурних елементів можна віднести ці гранули?

Включення

Гранулярна ендоплазматична сітка

Лізосоми

Екзоцитозні вакуолі

Мікрофіламенти

821 / 4299
Хворій 35-ти років з діагнозом безпліддя в гінекологічному відділенні зроблено діагностичну біопсію ендоме-трію. При мікроскопічному дослідженні з’ясувалося, що слизова оболонка з явищами набряку, маткові залози звивисті, заповнені густим секретом. Надлишок якого гормону обумовлюють такі зміни в ендометрії?

Соматотропін

АКТГ

Тестостерон

Естрогени

Прогестерон

822 / 4299
При дії на організм несприятливих факторів у тимусі має місце перебудова органу, що супроводжується масовою загибеллю тимоцитів, виселенням їх у периферійні органи, проліферацією епітеліоретикулоцитів. Як зветься таке явище?

Гіпотрофія тимуса

Дистрофія тимуса

Атрофія тимуса

Акцидентальна інволюція тимуса

Вікова інволюція тимуса

823 / 4299
При обстеженні пацієнта було вияв- лено аномалію розвитку емалі. З пошкодженням яких структурних компонентів зубного зачатка це пов’язано?

Шийка зубного емалевого органа

Проміжний шар емалевого органа

Зовнішній емалевий епітелій емалевого органа

Пульпа емалевого органа

Внутрішній емалевий епітелій емалевого органа

824 / 4299
В гістологічному препараті органу ротової порожнини видно, що передня поверхня вистелена багатошаровим плоским незроговілим епітелієм, а задня поверхня - багаторядним війчастим епітелієм. Що це за орган?

Щока

Тверде піднебіння

М’яке піднебіння

Іуба

Ясна

825 / 4299
У порожнині матки було виявлене плідне яйце, не прикріплене до ендо-метрію. Якій стадії розвитку відповідає зародок?

Гаструла

Бластоциста

Нейрула

Морула

Зигота

826 / 4299
У альпініста при тривалому перебуванні в горах відбувається збільшення кількості еритроцитів з 5,0 • 1012/л до 6, 0 • 1012 /л. Завдяки чому відбулася стимуляція еритропоезу?

Збільшення pO2 у венозній крові

Збільшення pO2 в артеріальній крові

Зменшення pO2 в артеріальній крові

Зменшення pO2 у венозній крові

Збільшення pO2 у клітинах

827 / 4299
Що з наведеного може бути причиною збільшення енерговитрат організму людини на 100%?

Споживання вуглеводної їжі

Споживання білкової їжі

Споживання жирної їжі

Зниження зовнішньої температури

Підвищення зовнішньої температури

828 / 4299
Європеєць 40-ка років працює в одній з країн Південно-Східної Азії. Скаржиться, що йому важко переносити високу температуру при високій відносній вологості повітря. Причиною цього є утруднення віддачі організмом тепла шляхом:

Теплопроведення

Конвекції

Конвекції та теплопроведення

Випаровування

Випромінювання

829 / 4299
У хворого після захворювання печінки виявлено зниження вмісту протромбіну в крові. Це призведе, перш за все, до порушення:

Фібринолізу

Антикоагулянтних властивостей крові

Судинно-тромбоцитарного гемостазу

Другої фази коагуляційного гемостазу

Першої фази коагуляційного гемостазу

830 / 4299
На ізольованому серці вивчалася швидкість проведення збудження в різних його ділянках. Де була виявлена найменша швидкість?

У пучку Гіса

У волокнах Пуркін’є

У атріовентрикулярному вузлі

У міокарді передсердь

У міокарді шлуночків

831 / 4299
Людина, що приймає блокатор мембранних циторецепторів синапсів еферентних провідників автономної нервової системи, скаржиться на сухість у роті. Які з рецепторів у неї заблоковані?

а-адренорецептори

Н2-рецептори

в-адренорецептори

M-холінорецептори

Н-холінорецептори

832 / 4299
У людини опік кінчика язика. Сприйняття яких смакових подразників у неї буде порушено найбільше?

Кислих та солоних

Гірких

Солоних

Солодких

Кислих

833 / 4299
У студента під час складання іспиту сохне в роті. Причиною цього є реалізація таких рефлексів:

Безумовних парасимпатичних

Умовних та безумовних симпатичних

Умовних парасимпатичних

Безумовних симпатичних та парасимпатичних

Умовних симпатичних

834 / 4299
У дитини наявне порушення формування емалі та дентину зубів через знижений вміст іонів кальцію в крові. Дефіцит якого гормону може спричинити такі порушення?

Соматотропний гормон

Тироксин

Паратгормон

Тирокальцитонін

Трийодтиронін

835 / 4299
У хворого на ЕКГ виявлено, що інтервал RR дорівнює 1,5 с, частота серцевих скорочень - 40/хв. Що є водієм ритму серця?

Пучок Гіса

Синусовий вузол

Атріовентрикулярний вузол

Ліва ніжка пучка Гіса

Права ніжка пучка Гіса

836 / 4299
У хворого виявлено гіперкаліємію та гіпонатріємію. Знижена секреція якого гормону може спричинити такі зміни?

Вазопресин

Паратгормон

Натрійуретичний

Альдостерон

Кортизол

837 / 4299
На тканину діють електричним імпульсом катодного напрямку, амплітуда якого дорівнює 70% порогу. Які зміни мембранного потенціалу це викличе?

Змін не буде

Гіперполяризація

Потенціал дії

Часткова деполяризація

838 / 4299
У слині пацієнта виявлено підвищений вміст лактату. Активація якого процесу є основною причиною підвищення лактату?

Анаеробний розпад глюкози

Гідроліз вуглеводів

Розпад глікогену

Аеробний розпад глюкози

Глюкозо-лактатний цикл

839 / 4299
У хворого з діагнозом злоякісного карціноїду різко підвищений вміст серотоніну в крові. Оберіть амінокислоту, з якої може утворюватися даний біогенний амін:

Треонін

Триптофан

Метіонін

Лейцин

Аланін

840 / 4299
У немовляти внаслідок неправильного годування виникла виражена діарея. Одним з основних наслідків діареї є екскреція великої кількості бікарбонату натрію. Яка форма порушення кислотно-лужного балансу має місце у цьому випадку?

Видільний ацидоз

Респіраторний ацидоз

Метаболічний алкалоз

Респіраторний алкалоз

Не буде порушень кислотно-лужного балансу

841 / 4299
У пацієнта, що страждає на хронічну ниркову недостатність, розвинувся остеопороз. Порушення синтезу в нирках якого регулятора мінерального обміну є основною причиною остеопо-розу?

Гідроксилювання проліну

Утворення 1,25(ОН)2 D3

Гідроксилювання лізину

Гідроксилювання кортизолу

Карбоксилювання глутамату

842 / 4299
В процесі лікування парадонтозу застосовують антиоксиданти. Вкажіть, яка з наведених природних сполук використовується в якості антиоксидантного засобу:

Піридоксин

Тіамін

Холін

Глюконат кальцію

Токоферол

843 / 4299
У хворого відмічається підвищена чутливість шкіри до сонячного світла. При відстоюванні сеча набуває темно-червоного кольору. Яка найбільш імовірна причина такого стану?

Порфірія

Пелагра

Гемолітична жовтяниця

Алкаптонурія

Альбінізм

844 / 4299
Пародонтит супроводжується активацією протеолізу в тканинах пародон-ту. Підвищення якого компоненту ротової рідини свідчить про активацію протеолізу?

Амінокислоти

Біогенні аміни

Глюкоза

Органічні кислоти

Холестерол

845 / 4299
У хворого діагностована алкаптонурія. Вкажіть фермент, дефект якого є причиною цієї патології:

Фенілаланінгідроксилаза

Піруватдегідрогеназа

Ілутаматдегідрогеназа

Оксидаза гомогентизинової кислоти

дОФА-декарбоксилаза

846 / 4299
Піддослідному щуру внутрішньоо-черевинно введено 10 мл 40% розчину глюкози. Через 60 хвилин у щура розвинувся коматозний стан внаслідок дегідратації. Який механізм розвитку цього стану?

Збільшення осмотичного тиску позаклітинної рідини

Збільшення онкотичного тиску позаклітинної рідини

Втрата води та солей

Зменшення секреції вазопресину

Порушення кислотно-основного стану

847 / 4299
До неврологічного відділення з приводу мозкового крововиливу надійшов хворий 62-х років у важкому стані. Об’єктивно: спостерігається наростання глибини та частоти дихання, а потім його зменшення до апное, після чого цикл дихальних рухів відновлюється. Який тип дихання виник у хворого?

Біота

Чейн-Стокса

Іаспінг-дихання

Апнейстичне

Кусмауля

848 / 4299
У хворих на поворотний тиф виникає лихоманка, яка характеризується кількаденними періодами високої гарячки, що чергується з періодами нормальної температури. Така температурна крива називається:

Febris hectica

Febris recurrens

Febris continua

Febris intermittens

Febris atypica

849 / 4299
У хворого внаслідок отруєння сулемою розвинулася гостра ниркова недостатність, пєрє6іг якої включав 4 стадії: перша - початкова, друга - олігоанурії, четверта - одужання. Як називається третя стадія гострої ниркової недостатності?

Метаболічна

Ішемічна

Іемодинамічна

Поліурична

Патохімічна

850 / 4299
Чоловік 48-ми років надійшов до лікарні із загостренням хронічного гломерулонефриту. Обстеження виявило наявність у хворого хронічної недостатності нирок. Чим пояснюється гіпера-зотемія при хронічній нирковій недостатності?

Зниження канальцевої реабсорбції

Порушення водно-електролітного обміну

Зниження канальцевої екскреції

Зниження клубочкової фільтрації

Порушення білкового обміну

851 / 4299
У експериментальних щурів, що тривалий час отримували лише вуглеводну їжу, спостерігалося накопичення води в тканинах. Який патогенетичний механізм є головним у розвитку набряку в даному випадку?

Гіпоонкотичний

Гіперосмолярний

Лімфогенний

Дисрегуляторний

Мембраногенний

852 / 4299
У хворого через добу після апен-дектомії в крові виявили нейтрофіль-ний лейкоцитоз з регенеративним зсувом. Який найбільш вірогідний механізм розвитку абсолютного лейкоцитозу в периферичній крові хворого?

Перерозподіл лейкоцитів в організмі

Посилення лейкопоезу

Активація імунітету

Зменшення руйнування лейкоцитів

Уповільнення еміграції лейкоцитів у тканини

853 / 4299
У хворого на хронічний мієлолей- коз виявлено ознаки анемії - зменшення кількості еритроцитів і вмісту гемоглобіну, оксифільні та поліхромато-фільні нормоцити, мікроцити. Який патогенетичний механізм є провідним у розвитку цієї анемії?

Хронічна крововтрата

Дефіцит вітаміну B12

Зменшення синтезу еритропоетину

Заміщення еритроцитарного ростка

Внутрішньосудинний гемоліз еритроцитів

854 / 4299
Під час розтину померлого м’які мозкові оболонки тьмяні, на їх поверхні наявні нашарування зеленувато-жовтого кольору, що захоплюють практично усю конвекситальну поверхню півкуль головного мозку. При гістологічному дослідженні спостерігається різке повнокров’я оболонок мозку з дифузною лейкоцитарною інфільтрацією. Яке з нижче перерахованих захворювань найбільш вірогідне?

Грип

Туберкульоз

Менінгококова інфекція

Кір

Сибірка

855 / 4299
У пункційному біоптаті встановлена дистрофія гепатоцитів, їх некроз, склероз з порушенням балочної і часточкової будови з утворенням несправжніх часточок, регенераторних вузлів. Який найбільш імовірний діагноз?

Гострий гепатит

Прогресуючий масивний некроз печінки

Хронічний гепатоз

Хронічний гепатит

Цироз печінки

856 / 4299
У дитини 6-ти років, госпіталізованої в зв’язку з коревою пневмонією, стоматолог виявив на слизовій оболонці щоки ділянку брудно-сірого кольору розмірами 2х2,5 см без чітких меж. М’які тканини набряклі, тьмяні, з неприємним запахом. Який найбільш вірогідний діагноз встановлений стоматологом?

Пустульозний стоматит

Гангренозний стоматит

Нома

Виразковий стоматит

Флегмонозний стоматит

857 / 4299
У хворої людини деформовані кістки щелеп. Гістологічно виявлено розростання на місці кісток клітинно-волокнистої пухлиноподібної тканини з примітивним остеогенезом без чітких меж. Для якого захворювання характерна така картина?

Еозинофільна гранульома

Фіброзна дисплазія

Паратиреоїдна остеодистрофія

Остеосаркома

Амелобластома

858 / 4299
Чоловік 55-ти років хворів на хронічний гломерулонефрит. Смерть настала при явищах хронічної ниркової недостатності. Макроскопічно: на поверхні епікарда і перикарда сірувато-білуваті ворсинчасті нашарування, після зняття яких добре видно різко розширені і переповнені кров’ю судини. Який процес мав місце в перикарді?

Артеріальне повнокров’я

Проліферативне запалення

Геморагічне запалення

Організація

Фібринозне запалення

859 / 4299
У померлого від хронічної серцевої недостатності у віці 68-ми років на розтині виявлені деформовані, потовщені стулки мітрального клапану, що зрослися між собою, по краю змикання клапанів визначаються дрібні (1-2 мм) тромби. Який ендокардит був причиною розвитку хронічної серцевої недостатності?

Зворотньо-бородавчастий

Дифузний

Гострий бородавчастий

Поліпозно-виразковий

Фібропластичний

860 / 4299
Чоловік 42-х років, що протягом 8-ми років страждав на хронічний гра-нулематозний періодонтит, хронічний гнійний остеомієліт нижньої щелепи, помер при явищах хронічної ниркової недостатності. Яке ускладнення гнійного остеомієліту розвинулося в нирках?

Атрофія

Гіаліноз

Амілоїдоз

Жирова дистрофія

Некроз епітелію звивистих канальців

861 / 4299
У дитини 5-ти років підвищилася температура до 40oC, виникли різкий головний біль, блювання, неспокій, озноб. Через 4 дні з’явився геморагічний висип на шкірі, олігурія та наднир- кова недостатність, що і стало причиною смерті. Під час бактеріологічного дослідження мазків з глотки виявлений менінгокок. Яка форма менінгококової інфекції була в хворого?

Менінгококовий менінгіт

Менінгококовий назофарингіт

Менінгококемія

Менінгоенцефаліт

862 / 4299
Хворий 75-ти років звернувся до хірурга зі скаргами на виразку гомілки буруватого кольору, що довго не загоюється. При біопсійному дослідженні: дифузне розростання поліморфних атипових клітин, в цитоплазмі яких знаходиться бурий пігмент. Реакція Перлса негативна. Багато патологічних міто-зів та осередків некрозу тканини. Який найбільш імовірний діагноз?

Трофічна виразка

Місцевий гемосидероз

Рак шкіри

Внутрішньодермальний невус

Меланома

863 / 4299
Під час розтину померлого, що страждав на гіпертонічну хворобу, у речовині головного мозку виявлена порожнина, стінки якої мають іржавий колір. Що передувало виникненню даних змін?

Абсцес

Діапедезні крововиливи

Плазморагії

Ішемічний інфаркт

Гематома

864 / 4299
Під час медичного обстеження учнів 1 класу поставлена проба Манту. З 35 учнів у 15 проба Манту була негативною. Що необхідно зробити дітям з негативною пробою Манту?

Ввести вакцину антирабічну

Ввести вакцину БЦЖ

Зробити повторну пробу

Дослідити сироватку крові

Ввести антитоксичну сироватку

865 / 4299
На території певної місцевості була зареєстрована масова загибель гризунів. Виникло припущення, що причиною може бути збудник чуми. Яку серологічну реакцію слід використати для швидкого встановлення збудника цієї епізоотії?

Зв’язування комплементу

Пасивної гемаглютинації

Нейтралізації

Аглютинації

Преципітації

866 / 4299
У дитини 10-ти років поставлено пробу Манту (з туберкуліном). Через 48 годин на місці введення туберкуліну з’явилаcя папула розміром до 8 мм у діаметрі. Який тип реакції гіперчутливості виник після введення туберкуліну?

Реакція типу сироваткової хвороби

Атопічна реакція

Реакція типу феномен Артюса

Реакція гіперчутливості IV типу

Реакція гіперчутливості II типу

867 / 4299
Хвору 58-ми років готували до операції холецистектомії. В комплекс засобів премедикації наркозу було введено бензогексоній. Яка роль цього фарма-копрепарату в наркозі?

Розслаблення скелетної мускулатури

Підсилення ретроградної амнезії

Редукція фази збудження

Функціональна блокада вісцеральних рефлексів

Розслаблення гладенької мускулатури

868 / 4299
У пацієнта в стоматологічному кабінеті розвинувся напад бронхіальної астми, який вдалося усунути сальбута-молом. До якої групи лікувальних засобів належить цей препарат?

-адреноміметики

Симпатолітики

а-адреноміметики

ві-в2-адреноміметики

а-в-адреноміметики

869 / 4299
До стаціонару надійшов хворий з діагнозом виразкова хвороба 12-ти па-лої кишки у фазі загострення. Аналіз шлункового соку показав підвищення секреторної та кислотоутворюю-чої функції шлунка. Оберіть препарат, який знизить секреторну функцію залоз шлунка за рахунок блокади H2-рецепторів:

Ранітидин

Метацин

Екстракт беладони сухий

Платифілін

Атропін

870 / 4299
Хворому при різкому підвищенні артеріального тиску призначений клофе- лін парентерально. Який його механізм дії?

Блокує аі та а2-адренорецептори

Стимулює центральні а2- адренорецептори

Стимулює центральні імідазоловіі-рецептори

Блокує H-холінорецептори гангліїв

Блокує а1 -адренорецептори

871 / 4299
Хворому призначено препарат з вираженими ліпофільними властивостями. Яким буде головний механізм його всмоктування?

Пасивна дифузія

Фільтрація

Активний транспорт

Піноцитоз

Зв’язування з транспортними білками

872 / 4299
Хворий, відчувши передвісники нападу бронхіальної астми, прийняв без контролю лікаря кілька таблеток всередину через короткі проміжки часу. Проте нетривале поліпшення стану відмітив тільки після вживання перших двох таблеток. Наступні прийоми препарату не покращили його стану. Яким явищем обумовлене зниження ефекту препарату?

Звикання

Залежність

Кумуляція

Ідіосинкразія

Тахіфілаксія

873 / 4299
Хворому із стрептококовою інфекцією ясен було призначено препарат, що містить у своїй структурі бета-лактамне кільце. Який препарат відноситься до цієї групи?

Бензилпеніциліну натрієва сіль

Еритроміцин

Стрептоміцину сульфат

Рифампіцин

Левоміцетин

874 / 4299
У здорової людини визначають абсолютні пороги смакової чутливості до різних речових. До якої з наведених речовин поріг буде найменшим?

Хінін

Лимонна кислота

Сахароза

Хлорид натрію

Глюкоза

875 / 4299
Хворий надійшов до клініки з пора- ненням в ділянці шиї. При обстеженні виявлено пошкоджений нерв, розташований попереду переднього драбинчастого м’яза. Який нерв ушкоджено?

Шийний відділ симпатичного стовбуру

Язикоглотковий

Під’язиковий

Блукаючий

Діафрагмальний

876 / 4299
У хворої 38-ми років після прийому аспірину і сульфаніламідів спостерігається посилений гемоліз еритроцитів, який викликаний недостатністю глюкозо-6-фосфатдегідрогенази. Порушенням утворення якого коферменту зумовлена ця патологія?

ФМН - Н2

ФАД - Н2

Убіхінон

НАДФ - Н

Піридокеальфоефат

877 / 4299
Після ДТП у водія відзначається деформація у середній третині лівої гомілки, сильний біль, особливо при спробі рухати лівою гомілкою. З рани виступають кінці кістки тригранного січення, велика крововтрата. Яка кістка може бути пошкоджена?

Великогомілкова

Надп’яткова

Малогомілкова

Стегнова

Надколінок

878 / 4299
В гистопрепараті представлена тканина, що містить клітини, які позбавлені відростків, і мають кожна декілька десятків ядер, а одна з поверхонь клітки має гофровану зону, через яку відбувається секреція гідролітичних ферментів. Яка тканина представлена в препараті?

Кісткова

Хрящова

Епітеліальна

М’язова

Нервова

879 / 4299
Ціанід калію, що є отрутою, потрапив до організму пацієнта і викликав смерть через декілька хвилин. Найбільш вірогідною причиною його токсичної дії було порушення активності:

Синтезу гемоглобіну

Цитохромоксидази

АТФ-синтетази

НАДФ-Н-дегідрогенази

Каталази

880 / 4299
Внаслідок порушення розходження хромосом при мейозі утворилися: яйцеклітина тільки з 22 аутосомами і полярне тільце з 24 хромосомами. Який синдром можливий у дитини при заплідненні такої яйцеклітини нормальним сперматозооном (22 + X)?

Синдром Едвардса

Синдром Клайнфельтера

Трисомія Х

Синдром Дауна

Синдром Шерешевського-Тернера

881 / 4299
У хворої проведена операція кесарського розтину, при цьому була розрізана на значній ділянці стінка матки і вилучений плід. Яким чином відбудеться загоєння у ділянці ушитого ендо-метрію?

Утворення гладенької м’язової тканини

Формування поперечносмугастих м’язових волокон

Проліферація міосателітоцитів

Гіпертрофія гладеньких міоцитів

Формування сполучнотканинного рубця

882 / 4299
У студента 18-ти років під час фізичного навантаження реографічно зареєстровано перерозподіл кровотоку між органами. У судинах яких органів кро-вотік підвищився найбільшою мірою?

Шлунково-кишковий тракт

Скелетні м’язи

Головний мозок

Нирки

Печінка

883 / 4299
Під час мікроскопії мазку ексудату, отриманого від щура з асептичним перитонітом, з додаванням в ексудат пташиних еритроцитів, виявлені макрофаги, оточені чужорідними еритроцитами. Якій стадії фагоцитозу відповідає описана картина?

Занурення

Незавершеного фагоцитозу

Прилипання

Наближення

Внутрішньоклітинного перетравлювання

884 / 4299
У хворого на 2-гу добу після розвитку інфаркту міокарда відбулося різке падiння систолічного артеріального тиску до 60 мм рт.ст. з тахiкардiєю до 140/хв, задишкою, нєпритомністю. Який механізм є провідним у патогенезі шоку, що розвинувся?

Зниження об’єму циркулюючої крові

Зменшення ударного об’єму крові

Пароксизмальна тахікардія

Інтоксикація продуктами некротичного розпаду

Анафілактична реакція на міокарді-альні білки

885 / 4299
Хворий 35-ти років висловлює скарги на постійну спрагу, знижений апетит. За добу випиває 9 л води. Добовий діурез збільшений, сеча знебарвлена, відносна густина - 1,005. Найбільш вірогідною причиною розвитку даної патології в хворого є пошкодження:

Аденогіпофізу

Базальної мембрани капілярів клубочків

Епітелію ниркових канальців

Епіфізу

Гіпоталамічних ядер

886 / 4299
Хворому з порушенням функції зовнішнього дихання необхідно зробити трахеостомію. На рівні яких хрящових кілець трахеї частіше за все може знаходитися перешийок щитоподібної залози?

I-II

III-IV

V-VI

IV-V

II-IV

887 / 4299
У людини під час активної фізичної праці підвищується концентрація вуглекислоти в крові. Це призводить до поглиблення та прискорення дихання, унаслідок чого в крові зменшується концентрація вуглекислоти та іонів водню. Завдяки цьому підтримується:

ортабіоз

імунітет

анабіоз

гомеостаз

онтогенез

888 / 4299
При гнійному запаленні середнього вуха в патологічний процес втягнена артерія, яка розташована на передній стінці барабанної порожнини. Що це за артерія?

A. carotis externa

A. carotis interna

A. temporalis superficialis

A. meningea media

A. auricularis posterior

889 / 4299
Хворий 34-х років звернувся з приводу карбункулу на обличчі. Об’єктивно: нещільний, безболісний набряк підшкірної клітковини, у центрі карбункулу чорний струп, по периферії везикулярні висипання навколо карбункулу. Мікробіологічне дослідження з’ясувало наявність нерухомих стрептобацил, які здатні утворювати капсули. Які мікроорганізми є збудником даної хвороби?

Bacillus megaterium

Bacillus anthracoides

Staphylococcus aureus

Bacillus subtilis

Bacillus antracis

890 / 4299
До лікарні звернулася хвора 38-ми років із скаргами на те, що після перенесеного гострого вірусного респіраторного захворювання вона втратила відчуття дотику їжі до передніх 2/3 язика, а також відчуття болю та температури, внаслідок чого, попекла язик гарячим чаєм. Вкажіть, яка з гілок якого нерва при цьому була ушкоджена?

Язикові гілки язикоглоткового нерва

Барабанна струна лицевого нерва

Язиковий нерв нижньощелепної гілки трійчастого нерва

Язикові гілки під’язикового нерва

Верхній гортанний нерв блукаючого нерва

891 / 4299
Під час мікроскопічного дослідження коронарної артерії у померлого 53-х років виявлено звуження просвіту судини за рахунок фіброзної бляшки з до-мішком ліпідів. Найвірогідніша форма атеросклерозу:

Доліпідна

Атероматоз

Ліпоїдоз

Ліпосклероз

Виразкування

892 / 4299
У вагітної жінки визначили групу крові. Реакція аглютинації еритроцитів відбулася зі стандартними сироватками груп 0а, в(I), Ba(III), та не відбулася з сироваткою Ав(II). Досліджувана кров належить до групи:

AB (IV)

Ba(III)

Ав(II)

0а,в(І)

893 / 4299
Хворій на гострий бронхіт, яка скаржиться на утруднене дихання та кашель з густим в’язким харкотинням, призначено муколітик, стимулятор синтезу сурфактанту. Який муколітик призначено?

Морфіну гідрохлорид

Теофілін

Натрію гідрокарбонат

Ілауцин

Амброксол

894 / 4299
При копрологічному дослідженні у фекаліях хворої знайдено яйця дрібних розмірів із кришечкою. З анамнезу відомо, що жінка часто вживає рибні страви. Який сисун паразитує в організмі?

Легеневий

Кров’яний

Печінковий

Ланцетоподібний

Котячий

895 / 4299
Під час клінічного обстеження пацієнтки виявлено зниження основного обміну на 40%, збільшення маси тіла, зниження температури тіла, одутлість обличчя, порушення статевих функцій, млявість і апатія, зниження інтелекту. Яке порушення функції і якої залози внутрішньої секреції призводить до появи даних симптомів?

Гіперфункція гіпофізу

Гіпофункція щитоподібної залози

Гіперфункція щитоподібної залози

Гіпофункція паращитоподібних залоз

Гіпофункція епіфізу

896 / 4299
У людини під час нейрохірургічної операції подразнюють потиличні частки кори великих півкуль головного мозку. Які відчуття це викликатиме у хворого?

Дотикові

Смакові

Зорові

Нюхові

Слухові

897 / 4299
При мікроскопічному дослідженні ендокринної залози з’ясувалося, що її паренхіма складається з фолікулярних структур, стінка яких утворена одношаровим кубічним епітелієм, а порожнина заповнена оксифільною речовиною. Який гормон секретує ця залоза?

тироксин

альдостерон

кортизол

паратирин

окситоцин

898 / 4299
Під час голодування нормальний рівень глюкози підтримується за рахунок активації глюконеогенезу. Назвіть речовину яка може використовуватись як субстрат для цього процесу:

Амоніак

Аланін

Аденін

Сечовина

Гуанін

899 / 4299
Які зміни з боку ізольованого серця можна очікувати після введення до пер-фузійного розчину адреналіну?

Зменшення сили скорочень

Збільшення частоти і сили скорочень

Зупинка серця в діастолі

Збільшення частоти скорочень

Збільшення сили скорочень

900 / 4299
Спортсмену необхідно підвищити спортивні результати. Для цього йому рекомендовано вживати препарат, який містить карнітин. Який процес в найбільшому ступені активується цією сполукою?

Транспорт амінокислот

Транспорт жирних кислот

Транспорт глюкози

Транспорт іонів кальцію

Транспорт вітаміну K

901 / 4299
Пацієнт звернувся зі скаргами на гострий біль у правому підребер’ї. При огляді лікар звернув увагу на те, що склери хворого пожовтішали. Лабораторні аналізи показали підвищену активність АЛТ та негативну реакцію на стеркобілін у калі. Для якого захворювання характерні такі симптоми?

Хронічний коліт

Гемолітична жовтяниця

Хронічний гастрит

Гепатит

Жовчнокам’яна хвороба

902 / 4299
Хворому, який надійшов до лікарні з отруєнням неякісною їжею, було про- мито шлунок розчином калію перманганату. Який механізм дії цього засобу?

Порушення синтезу ферментів дихального ланцюга

Вившьнення йоду

Вивільнення хлору

Руйнування мембран бактерій

Вивільнення атомарного кисню

903 / 4299
Хворий звернувся до лікарні із скаргами на шум та больові відчуття в вусі Об’єктивно: у хворого гостре респіраторне захворювання, риніт. Крізь який отвір глотки інфекція потрапила до барабанної порожнини та викликала її запалення?

Глотковий отвір слухової труби

Барабанний отвір слухової труби

Хоани

Зів

Вхід до гортані

904 / 4299
У тримісячного немовляти з’явився білий наліт на слизовій оболонці рота, язика та губ. Лікар запідозрив канди-доз. На яке поживне середовище необхідно зробити посів досліджуваного матеріалу для підтвердження діагнозу?

Клауберга

Левенштейна-Йєнсена

Ру

Сабуро

Ендо

905 / 4299
З метою специфічної профілактики використана вакцина, яка містить в своєму складі мікроорганізми і знешкоджений формаліном екзотоксин. До яких вакцин вона належить?

Жива

Анатоксин

Хімічна

Генно-інженерна

Асоційована

906 / 4299
У хворого 70-ти років із серцевою недостатністю внаслідок безконтрольного прийому дигоксину виникли екстрасистолія, блювання, погіршення зору, тривожність, порушення сну, зменшення діурезу. Застосування препаратів якої групи є патогенетично виправданим у даному випадку?

Блокатори рецепторів ангіотензину II

Донатори оксиду азоту

Стимулятори ^і-адренорецепторів

Донатори сульфгідрильних груп

Блокатори калієвих каналів

907 / 4299
У хворого на гепатоцеребральну дистрофію в сироватці крові знижений вміст церулоплазміну. Накопичення якого елемента в печінці, мозку та нирках спостерігається у хворого?

Калій

Мідь

Залізо

Кальцій

Натрій

908 / 4299
При електронній мікроскопії в кірковій речовині нирки визначаються структури, що вистелені призматичним епітелієм, для якого характерна щіто-чкова облямівка та глибокі складки плазмолеми у базальній частині. Між складками розташована велика кількість мітохондрій. До якого відділу не-фрона належать описані структури?

Звивистий дистальний каналець

Проксимальний каналець

Ниркове тільце

Прямий дистальний каналець

Петля Генле

909 / 4299
На препараті печінки внутрішньо-часточкові капіляри мають широкий просвіт, нерівномірний по всій довжині. Базальна мембрана у більшій частині капіляра відсутня. До якого типу відносяться такі капіляри?

Посткапіляри

Соматичного типу

Вісцерального типу

Синусоїдного типу

Прекапіляри

910 / 4299
Для прискорення загоєння опікової рани необхідно призначити засіб, який сприяє епітелізації шкіри та слизових оболонок. Визначте цей препарат:

Аскорбінова кислота

Ретинолу ацетат

Токоферолу ацетат

Нікотинова кислота

Ергокальциферол

911 / 4299
До кардіологічного відділення надійшов хворий з шлуночковою аритмією. Який препарат доцільно призначити?

Прозерин

Дротаверин

Аміодарон

Аміназин

Амлодипін

912 / 4299
Після відновлення кровообігу в ушкодженій тканині припиняється накопичення лактату та зменшується швидкість споживання глюкози. Активацією якого процесу зумовлені ці метаболічні зміни?

Ілюконеогенез

Анаеробний гліколіз

Біосинтез глікогену

Ліполіз

Аеробний гліколіз

913 / 4299
Дослідженнями останніх десятиріч встановлено, що безпосередніми 'виконавцями'апоптозу в клітині є особливі ферменти - каспази. В утворенні одного з них бере участь цитохром С. Вкажіть його функцію в нормальній клітині:

Фермент ЦТК

Фермент бета-окиснення жирних кислот

Компонент піруватдегідрогеназної системи

Компонент H+-АТФазной системи

Фермент дихального ланцюга переносу електронів

914 / 4299
До кардіологічного відділення надійшов хворий з гіпертонічною хворобою. Для зниження артеріального тиску лікар призначив лікарський засіб, що блокує ві та /32- адренорецептори. Назвіть цей лікарський засіб:

Прозерин

Анаприлін

Целекоксиб

Преднізолон

Індометацин

915 / 4299
До відділення реанімації був доставлений хворий з інфарктом міокарда. Який препарат необхідно ввести хворому для профілактики больового шоку?

Целекоксиб

Парацетамол

Анальгін

Налоксон

Промедол

916 / 4299
До відділення реанімації надійшов хворий з інфарктом міокарда. Який препарат необхідно ввести хворому для профілактики тромбоутворення?

Димедрол

Бісептол-480

Тироксин

Хінгамін

Іепарин

917 / 4299
До гінеколога звернулася жінка 28-ми років з приводу безпліддя. При обстеженні знайдено: недорозвинені яєчники і матка, нерегулярний менструальний цикл. При дослідженні статевого хроматину у більшості соматичних клітин виявлено 2 тільця Бара. Яка хромосомна хвороба найбільш вірогідна у жінки?

Синдром Патау

Синдром Шерешевського-Тернера

Синдром трипло-Х

Синдром Едвардса

Синдром Клайнфельтера

918 / 4299
При лікуванні багатьох захворювань використовується фармацевтичний препарат кокарбоксилаза (тіамін-пірофосфат) для забезпечення клітин енергією. Вкажіть, який метаболічний процес при цьому активується?

Окисне декарбоксилювання пірувату

Детоксикація шкідливих речовин у печінці

Декарбоксилювання біогенних амінів

Декарбоксилювання амінокислот

Дезамінування глутамату

919 / 4299
До клініки надійшов хворий з яскраво вираженою жовтяничністю шкіри, склер, слизових оболонок. Сеча кольору темного пива, аналіз показав наявність у ній прямого білірубіну. Вміст жовчних пігментів в калі знижений. Який тип жовтяниці спостерігається в хворого?

Кон’югаційна

Іемолітична

Паренхіматозна

Абсорбційна

Обтураційна

920 / 4299
У хворого хронічна лівошлуночко-ва недостатність. Який з перерахованих препаратів доцільно призначити хворому?

Пірацетам

Дигоксин

Етимізол

Бемегрид

Вінпоцетин

921 / 4299
У дитяче відділєння швидкою допомогою доставлена дівчинка 2-х років. Об’єктивно: дитина квола, апатична. Печінка збільшена, у її біоптаті -значний надлишок глікогену. Концентрація глюкози в крові нижча від норми. Найбільш вірогідною причиною гіпоглікемії є:

Знижена активність глюкозо-6-фосфатази

Знижена активність глюкозо-1-фосфатуридинтрансферази

Знижена активність глікогенфосфо-рилази

Знижена активність глікогенсинтази

Підвищена активність глюкокінази

922 / 4299
У 22-річної жінки внаслідок довготривалого вживання сульфаніламідних препаратів з’явилися ознаки гемолітичної анемії, що зумовлюється спадковим порушенням синтезу ферменту пентозофосфатного циклу глюкозо-6-фосфатдегідрогенази, який забезпечує утворення в організмі:

НАД

НАДФ - Н2

ФМН

АТФ

ФАД

923 / 4299
Хворій, що хворіє на інсулінонеза-лежний цукровий діабет, призначили всередину глібенкламід. Вкажіть механізм гіпоглікемічної дії цього препарату:

Пригнічує а-глюкозидазу і розпад полісахаридів

Стимулює виділення ендогенного інсуліну в-клітинами

Посилює утилізацію глюкози периферичними тканинами

Пригнічує всмоктування глюкози у кишечнику

Пригнічує глюконеогенез у печінці

924 / 4299
Під час профілактичного огляду чоловіка, який працює шахтарем, лікар встановив зміни функціонального стану серця, що свідчать про серцеву недостатність в стадії компенсації. Що з нижче наведеного є головним підтвердженням компенсації діяльності серця?

Тахікардія

Збільшення артеріального тиску

Ціаноз

Задишка

Гіпертрофія міокарда

925 / 4299
Характерними ознаками холери є втрата організмом великої кількості води та іонів натрію. Який механізм лежить в основі виникнення діареї при цьому?

Окиснення альдостерону в корі на-днирників

Активація аденілатциклази ентеро-цитів

Гальмування синтезу вазопресину в гіпоталамусі

Посилення синтезу кортикотропіну

Посилення секреції реніну клітинами ниркових артеріол

926 / 4299
У новонародженого спостерігається диспепсія після годування молоком. При заміні молока розчином глюкози симптоми диспепсії зникають. Про недостатню активність якого ферменту в новонародженого можна думати у цьому випадку?

Амілаза

Ізомальтаза

Сахараза

Лактаза

Мальтаза

927 / 4299
Внаслідок травми у чоловіка 35-ти років настав повний розрив спинного мозку на рівні першого шийного сегменту. Як зміниться при цьому зовнішнє дихання?

Стане діафрагмальним

Зупиниться

Не зміниться

Стане рідким і глибоким

Стане поверхневим і частим

928 / 4299
В експерименті на тварині зруйнували середню частину завитки. Це призвело до порушення сприйняття звукових коливань такої частоти:

Низької та середньої

Середньої

Низької

Високої

Високої та середньої

929 / 4299
Людина отруїлася грибами. Вони містять мускарин, який стимулює M-холінорецептори. За яким симптомом можна запідозрити отруєння неїстівни- ми грибами?

Збільшення частоти серцевих скорочень

Розширення зіниць

Підвищення артеріального тиску

Звуження зіниць

Розширення 6ронхів

930 / 4299
Хворому на сифіліс призначили лікарський засіб, в основі механізму дії якого лежить порушення утворення муреїну, що призводить до загибелі збудника. Визначить цей препарат:

Азітроміцин

Бензилпеніциліну натрієва сіль

Ципрофлоксацин

Бійохінол

Доксацикліну гідрохлорид

931 / 4299
Пацієнт, що хворіє на хронічний бронхіт, приймає синтетичний муколітичний препарат, який сприяє розріджуванню харкотиння. Назвіть цей препарат:

Еналаприл

Ацетилцистеїн

Фуросемід

Діазепам

Гепарин

932 / 4299
На мікропрепараті, зробленому з привушної слинної залози, визначається кінцеві секреторні відділи з се-роцитами, що синтезують переважно ферменти. До яких залоз згідно класифікації за хімічним складом секрету вона належить?

Ферментативна

Білково-слизова

Змішана

Білкова

Слизова

933 / 4299
Після тривалого лікування антибіотиками у пацієнта розвинулися симптоми стоматиту. У препаратах, приготованих зі слизової рота, виявлені овальні поліморфні Грам-позитивні мікроорганізми, розташовані скупченнями. Який з перелічених нижче мікроорганізмів може бути причиною таких проявів?

S.pyogenes

C.perfringens

C.albicans

C.pylori

S.aureus

934 / 4299
Після закривання рота і стискання зубів відбувається його рефлекторне відкривання. З яких рецепторів починається зазначений рефлекс?

Пропріорецептори м’язів, що піднімають нижню щелепу

Механорецептори слизової ротової порожнини

Рецептори періодонту

Смакові рецептори

Пропріорецептори м’язів, що опускають нижню щелепу

935 / 4299
При обстеженні у жінки встановлена недостатність активності ліпопроте-їнліпази, котра гідролізує тригліцериди хіломікронів на поверхні ендотелію капілярів жирової тканини. Які біохімічні порушення слід очікувати?

Гіперліпопротеїнемія IV типу

Гіперліпопротеїнемія II Б типу

Гіперліпопротеїнемія I типу

Гіперліпопротеїнемія II А типу

Гіперліпопротеїнемія III типу

936 / 4299
У дитини спостерігається затримка фізичного та розумового розвитку, глибокі порушення з боку сполучної тканини внутрішніх органів, у сечі виявлено кератансульфати. Обмін яких речовин порушений?

Фібронектин

Глікозаміноглікани

Колаген

Гіалуронова кислота

Еластин

937 / 4299
У хворої 40-ка років збільшена щитоподібна залоза. При пальпації залоза щільна, поверхня її дрібногорби-ста. При гістологічному дослідженні біоптату щитоподібної залози виявлено дифузну інфільтрацію тканини лімфоцитами, плазматичними клітинами, утворення лімфоїдних фолікулів. Яке захворювання у хворої?

Спорадичний зоб

Аутоімунний тиреоїдит

Тиреоїдит Ріделя

Ендемічний зоб

Дифузний токсичний зоб

938 / 4299
У жінки 30-ти років при тривалому використанні губної помади з флю-оресцуючою речовиною на облямівці губ розвинулись обмежена еритема, незначне лущення; пізніше виникли поперечні дрібні борозни та тріщини. При мікроскопічному дослідженні цієї зони ураження в сполучній тканині виявлені сенсибілізовані лімфоцити і макрофаги, явища цитолізу. Який тип імунологічної гіперчутливості розвинувся на губі?

III тип (імунокомплексна цитотокси-чність)

IV тип (клітинна цитотоксичність)

Гранулематоз

I тип (реагінового типу)

II тип (антитільна цитотоксичність)

939 / 4299
У хворого на есенційну артеріальну гіпертензію розвинувся гіпертонічний криз, що призвело до нападу серцевої астми. Який механізм серцевої недостатності є провідним в даному випадку?

Перевантаження серця підвищеним опором

Абсолютна коронарна недостатність

Перевантаження серця збільшеним об’ємом крові

Пошкодження міокарда

Порушення надходження крові до серця

940 / 4299
Людина, що тривало приймала ліки, не може припинити їх використання, так як при цьому виникають порушення психічних і соматичних функцій. Як називається синдром, що виникає при відмові від прийому лікарської речовини?

Сенсибілізація

Ідіосинкразія

Тахіфілаксія

Кумуляція

Абстиненція

941 / 4299
Жінка 43-х років госпіталізована зі скаргами на біль у правому підребер’ї, свербіж шкіри. При тривалому обстеженні встановлено: зниження больової чутливості та швидкості згортання крові, жовтяничність шкірних покривів, брадикардія, гіпотонія. Що є найбільш вірогідною причиною даного симпто-мокомплекса?

Підвищення гемолізу еритроцитів

Паренхіматозна жовтяниця

Холемія

Цукровий діабет

Печінкова жовтяниця

942 / 4299
За медичним показанням пацієнту було проведено видалення частини однієї із структур ЦНС. В результаті видалення у пацієнта розвинулися ато- нія, астазія, інтенційний тремор, атаксія, адіадохокінез. Частина якої структури ЦНС була вилучена?

Мозочок

Гіпокамп

Рухова кора

Мигдалеподібний комплекс

Базальні ганглії

943 / 4299
У хворого на жовтяницю у крові виявлено збільшення загального білірубіну за рахунок непрямої його фракції. Сеча і кал інтенсивно забарвлені. Який найбільш вірогідний механізм вказаних порушень?

Підвищений гемоліз еритроцитів

Пошкодження паренхіми печінки

Порушення перетворення уробіліно-гену в печінці

Утруднення відтоку жовчі з печінки

Порушення утворення прямого білірубіну

944 / 4299
Яким буде скорочення м’язів верхньої кінцівки при утриманні (але не переміщенні) вантажу в певному положенні?

Концентричне

Ексцентричне

Ауксотонічне

!зометричне

Ьотонічне

945 / 4299
У хворого круглясті виразки на обличчі, запалення та збільшення лімфатичних вузлів. Ці симптоми з’явилися після укусів москітів. Під час лабораторного дослідження виділень із виразок на обличчі виявлено одноклітинні безджгутикові організми. Який діагноз найбільш вірогідний?

Токсоплазмоз

Трипаносомоз

Короста

Міаз

Дерматотропний лейшманіоз

946 / 4299
Хворий протягом 5-ти років страждає на цукровий діабет. В результаті порушення дієти у нього розвинувся коматозний стан. Лікар швидкої допомоги ввів глюкозу. Стан хворого покращився. Яка кома найбільш вірогідно була в хворого?

Гіпоглікемічна

Печінкова

Гіперглікемічна

Ацидотична

Гіпотиреоїдна

947 / 4299
Хворий надійшов до інфекційного відділення: шкіра суха, тургор шкіри знижений, випорожнення у вигляді рисового відвару. Поставлений діагноз: холера. Яке порушення водно-електролітного балансу, найбільш ймовірно, виникає при цьому захворюванні?

Гіпоосмотична гіпергідратація

Гіпоосмотична гіпогідратація

Гіперосмотична гіпогідратація

Ізоосмотична гіпогідратація

Гіперосмотична гіпергідратація

948 / 4299
В препараті сполучної тканини дерми шкіри, забарвленому суданом-III і гематоксиліном, виявляються скупчення великих багатокутних клітин, які зафарбовуються в помаранчевий колір. Ядра мають сплощену форму, зміщені на периферію. Яка це тканина?

Бура жирова

Гіалінова хрящова

Біла жирова

Пластинчаста кісткова

Ретикулярна сполучна

949 / 4299
Хворому з метою знеболення ввели розчин місцевого анестетику. Через декілька хвилин у хворого виникли задишка, тахікардія, втрата свідомості. Який шок розвинувся в хворого?

Опіковий

Кардіогенний

Травматичний

Геморагічний

Анафілактичний

950 / 4299
У новонародженого виявляється гіперемія, набряк на слизовій рота, невеликі ерозії з в’язкими слизово-гнійними виділеннями. У мазках з виділень виявляється велика кількість лейкоцитів, що містять грамнегативні диплококи. Такі ж мікроорганізми розташовуються і поза лейкоцитами. Який діагноз можна припустити?

Бленорея

Стафілококовий стоматит

Токсоплазмоз

Вроджений сифіліс

Гонококовий стоматит

951 / 4299
При дослідженні біоптату збільшених шийних лімфатичних вузлів молодої жінки виявлена проліферація лімфоцитів, ретикулярних клітин, гістіоцитів, великих і малих клітин Ходжкіна, багатоядерних клітин Березовського-Штернберга, численні еозинофіли, поодинокі вогнища казе-озного некрозу тканини лімфатичного вузла. Який найбільш імовірний діагноз?

Лімфогранулематоз

Туберкульоз

Гострий лейкоз

Метастаз раку легень

Лімфосаркома

952 / 4299
При мікроскопічному дослідженні виділень з уретри хворого гострим уретритом виявлені бобоподібної форми мікроорганізми діаметром до 1 мкм, розміщені попарно та розташовані всередині лейкоцитів. Які це мікроорганізми?

Гонококи

Менінгококи

Тетракоки

Стрептококи

Стафілококи

953 / 4299
У хворого з переломом стегнової кістки в ділянці хірургічної шийки з’явились ознаки гострої правошлуночко-вої недостатності внаслідок емболії легеневої артерії. Який вид емболії має місце?

Метастатична

Газова

Жирова

Повітряна

Тканинна

954 / 4299
Внаслідок вивиху нижньої щелепи у пацієнта порушена смакова чутливість передніх 2/3 язика та слиновиділення. Травмою якого нерва це викликано?

Глибокий кам’янистий нерв

Барабанна струна

Великий кам’янистий нерв

Під’язиковий нерв

Малий кам’янистий нерв

955 / 4299
Хворий 40-ка років помер від набряку головного мозку. В анамнезі карбункул обличчя. На аутопсії відмічено повнокров’я та набряк тканини головного мозку. У білій речовині лівої півкулі виявлено дві порожнини розміра- ми 6х5,5 та 5х4,5 см, що заповнені вершковоподібною рідиною жовтувато-зеленого кольору. Стінками порожнин є нервова тканина з нєрівними краями. Яке ускладнення карбункула розвинулось у хворого?

Гострі абсцеси

Кісти

Емпієма

Колікваційні некрози

Хронічні абсцеси

956 / 4299
У дитини 9-ти років під час огляду порожнини рота стоматолог виявив на щічній поверхні ясни у ділянці нижнього ікла утворення у вигляді вузлика діаметром 1 см, червоного кольору, м’якої консистенції, що легко кровоточить при дотику. Під час мікроскопічного дослідження утворення представлене великою кількістю дрібних судин типу венул і капілярів, розділених тонкими прошарками сполучної тканини, з вогнищевою інфільтрацією лімфоїдними і плазматичними клітинами. Такі зміни характерні для:

Фіброзного епуліда

Радикулярної гранульоми

Капілярної гемангіоми

Ангіоматозного епуліда

Папіломи

957 / 4299
У хірургічному відділенні стоматологічної клініки мали місце випадки госпітальної стафілококової інфекції, викликаної штамами, які характеризувались множинною лікарською стійкістю. Така ознака визначається наявністю:

Екзотоксигенів

Вірулентних бактеріофагів

R-плазмід

F-плазмід

Помірних бактеріофагів

958 / 4299
Під час рентгенівського дослідження була виявлена деформація нижньої стінки правої очниці. Який параназаль-ний синус найімовірніше уражений?

Лобна пазуха

Лівий решітчастий лабіринт

Правий решітчастий лабіринт

Права верхньощелепна пазуха

Клиноподібна пазуха

959 / 4299
Під час огляду лікар виявив при-шийковий карієс нижніх правих різців. При цьому відзначено збільшення у розмірах групи лімфатичних вузлів. Які лімфовузли збільшені?

Потиличні

Лицеві

Глибокі шийні

Підпідборідні

Поверхневі шийні

960 / 4299
При дослідженні трубчастого органу встановлено, що його середня оболонка складається із гіалінових суцільних кілець. Який епітелій вистилає слизову оболонку цього органа?

Багаторядний призматичний війчастий

Одношаровий призматичний з облямівкою

Багатошаровий плоский незрогові-лий

Одношаровий призматичний залозистий

Одношаровий кубічний

961 / 4299
Який метод стерилізації доцільно використати для стоматологічних інструментів, які не псуються від дії температури і вологості для гарантованого знищення вірусів, вегетативних та спорових форм мікроорганізмів?

Прожарювання над полум’ям газового пальника

Автоклавування

Тиндалізація

Кип’ятіння

Пастеризація

962 / 4299
Хвора госпіталізована з діагнозом плеврит. В якому місці плевральної порожнини міститься найбільша кількість ексудату?

Під куполом плеври

Під коренем легенів

Реберно-медіастинальний синус

Реберно-діафрагмальний синус

Діафрагмально-медіастинальний синус

963 / 4299
У дитини 10-ти місяців спостерігається висока збудливість, порушення сну, знижений тонус м’язів, запізніле прорізування зубів з недостатньо звапнованою емаллю. Дефіцит в організмі якого вітаміну зумовлює такі зміни?

Ретинол

Холекальциферол

Тіамін

Рибофлавін

Нікотинамід

964 / 4299
При огляді порожнини носа виявлено викривлення задньої частини носової перегородки. Яка кістка викривлена?

Перпендикулярна пластинка решітчастої кістки

Латеральна пластинка крилоподі6но-го відростка

Леміш

Медiальна пластинка крилоподі6ного відростка

Вертикальна пластинка піднє6інної кістки

965 / 4299
В процесі ембріогенезу епітеліальний тяж, який має назву вестибулярної пластинки, дає початок розвитку при-сінку порожнини рота. Який біологічний механізм запрограмованої загибелі клітин забезпечує утворення з епітеліальної пластинки щічно-губної борозни?

Апоптоз

Амітоз

Некроз

Паранекроз

Мейоз

966 / 4299
У 27-річного чоловіка при рентгенологічному дослідженні нижньої щелепи виявлено вогнище деструкції кісткової тканини, гістологічно визначається пухлина, що складається із тяжів одонтогенного епітелію, незрілої сполучної тканини і острівців диспласти-чного дентину. Діагностуйте пухлину:

Амелобластична фіброодонтома

Дентинома

Одонтогенна фіброма

Одонтоамелобластома

Складна одонтома

967 / 4299
У чоловіка під час операції з приводу пахвинної грижі хірург пошкодив вміст пахвинного каналу. Яку структуру пошкодив хірург?

Urarchus

Lig. inguinale

Funiculus spermaticus

Lig. teres uteri

968 / 4299
До відділення черепно-мозкової травми надійшов хворий з пошкодженням великого крила клиноподібної кістки. Лінія перелому пройшла через остистий отвір. Яка судина постраждала?

Передня глибока скронева артерія

Задня глибока скронева артерія

Латеральна крилоподібна артерія

Середня оболонкова артерія

Поверхнева скронева артерія

969 / 4299
У хворого алергічний нежить з рясним виділенням слизу, свербінням, частим чханням. Який з перелічених засобів слід обрати у цьому випадку, якщо відомо, що він вибірково блокує гіста-мінові рецептори?

Адреналіну гідрохлорид

Преднізолон

Мезатон

Лоратадин

Нафтизин

970 / 4299
До кардіологічного відділення надійшов хворий з інфарктом міокарда. Для усунення болю було вирішено потенціювати дію фентанілу нейролептиком. Який з перелічених нейролептиків найбільш придатний для нейролепта-налгезії?

Сульпірид

Трифтазин

Аміназин

Іалоперидол

Дроперідол

971 / 4299
При огляді порожнини рота дитини педіатр виявила наявність нижніх медіальних різців. Дитина розвивається нормально. Визначте її вік:

13-14 місяців

10-12 місяців

6-7 місяців

8-9 місяців

972 / 4299
У чоловіка 23-х років виникла перфорація твердого піднебіння, в ділянці якого знайдено щільне утворення з чіткими межами. Після операції при мікроскопічному дослідженні цього утворення було виявлено: значний осередок казеозного некрозу, який оточений грануляційною тканиною з ендоваскулі-том, клітинним інфільтратом, що складається з лімфоцитів, епітеліоїдних клітин, з перевагою плазмоцитів. Яке найбільш вірогідне захворювання у хворого?

Саркома

Сифіліс

Лепра

Туберкульоз

Склерома

973 / 4299
У молодої жінки, яка зайшла до виробничого цеху з різким запахом лакофарбової продукції, виник бронхоспа-зм. Подразнення яких рецепторів викликало виникнення даного рефлексу?

Периферичні хеморецептори

!ритантні

Юкстакапілярні

Центральні хеморецептори

Рецептори плеври

974 / 4299
На гістологічному препараті визначається орган, який має кіркову та мозкову речовину. Кіркова речовина складається з зовнішньої зони, яка містить лімфатичні вузлики, та паракортикаль-ної зони. У мозковій речовині розташовані мозкові тяжі, синуси і трабекули. Який орган має дані морфологічні ознаки?

Тимус

Лімфатичний вузол

Нирка

Селезінка

Наднирники

975 / 4299
В результаті травми порушено цілісність переднього корінця спинного мозку. Визначте, які відростки яких нейронів при цьому пошкоджено?

Аксони рухових нейронів

Дендрити рухових нейронів

Дендрити чутливих нейронів

Дендрити вставних нейронів

Аксони чутливих нейронів

976 / 4299
У дитини 7-ми років, хворої на ангіну, був взятий мазок з поверхні мигдаликів і засіяний на кров’яний агар. Через добу виросли колонії стрептококів, навколо яких середовище стало прозорим. Наявність якого фактора патогенності у збудника виявило це дослідження?

Лейкоцидин

Бета -лактамаза

Нейрамінідаза

Ендотоксин

Гемолізин

977 / 4299
Хвороба Хартнепа зумовлена точковою мутацією лише одного гена, наслідком чого є порушення всмокту- вання амінокислоти триптофану в кишечнику та реабсорбції її в ниркових канальцях. Це призводить до одночасних розладів у травній і сечовидільній системах. Яке генетичне явище спостерігається в цьому випадку?

Неповне домінування

Полімерія

Плейотропія

Кодомінування

Комплементарна взаємодія

978 / 4299
У хворого з синдромом Іценко-Кушінга спостерігаються стійка гіперглікемія та глюкозурія, гіпертензія, остеопороз, ожиріння. Синтез та секреція якого гормону збільшується в даному випадку?

Альдостерон

Кортизол

Адреналін

Глюкагон

Тироксин

979 / 4299
Хворий надійшов до приймального відділення лікарні з отруєнням інсектицидом антихолінестера-зної дії. Який препарат, що блокує M-холінорецептори, потрібно призначити в даному випадку?

Дитилін

Мезатон

Бензогексоній

Атропіну сульфат

Пілокарпіну гідрохлорид

980 / 4299
Дівчина 15-ти років була доставлена в лікарню з запаленням червоподібного відростка. У крові - ознаки анемії. У фекаліях виявлено яйця гельмінта, які мають лимоноподібну форму (50х30 мкм), з 'пробочками'на полюсах. Який вид гельмінта паразитує у дівчинки?

Ехінокок

Карликовий ціп’як

Анкілостома

Волосоголовець

Гострик

981 / 4299
При диспансерному обстеженні хлопчику 7-ми років встановлено діагноз - синдром ЛешаНайхана (хворіють тільки хлопчики). Батьки здорові, але у дідуся за материнською лінією таке ж захворювання. Який тип успадкування захворювання?

Автосомно-домінантний

Домінантний, зчеплений із статтю

Автосомно-рецесивний

Рецесивний, зчеплений із статтю

Неповне домінування

982 / 4299
При ендоскопічному обстеженні 12-ти палої кишки дiагностовано пухлину великого сосочка. У якому відділі кишки локалізується патологічне утворення?

Висхідна частина

Верхній вигин

Низхідна частина

Горизонтальна частина

Верхня частина

983 / 4299
На розтині верхня частка правої легені збільшена, сірого кольору, безповітряна, з поверхні розрізу стікає каламутна рідина, на плеврі багато фібринозних плівок. Мікроскопічно в альвеолах виявляється ексудат з присутністю нейтрофілів, десквамативних альвео-лоцитів і ниток фібрину. Стінка бронха інтактна. Який найбільш імовірний діагноз?

!нтерстиційна пневмонія

Грипозна пневмонія

Крупозна пневмонія

Вогнищева пневмонія

Абсцес легень

984 / 4299
У альпіністів, які тривалий час знаходились у високогірному районі, спостерігалось збільшення кількості еритроцитів - більше 6 • 1012/л та гемоглобіну - більше 170 г/л у крові. Який механізм спричинив виникнення цього явища?

Посилення процесів безкисневого енергоутворення

Послаблення гемолізу еритроцитів у кровоносному руслі

Послаблення внутрішньоклітинного гемолізу еритроцитів

Посилення продукції еритропоетинів нирками

Посилення здатності тканин утилізувати кисень

985 / 4299
Хворий 60-ти років скаржиться на печіння язика, підвищене слиновиділення, явище глосалгії, котрі з’явились після 5 днів користування металевим мостоподібним протезом. Об’єктивно: слизова оболонка ротової порожнини набрякла, гіперемована. Який стоматит за характером запалення розвинувся у хворого?

Катаральний

Гнійний

Фібринозний

Виразковий

Гангренозний

986 / 4299
У хворого видалений зуб, у якого коронка овальної форми, на жувальній поверхні два горбки. Корінь сильно здавлений у мезіо-дистальному напрямку, його верхівка роздвоєна. Який зуб видалений?

Ікло

Другий верхній премоляр

Перший верхній премоляр

Другий нижній премоляр

Перший нижній премоляр

987 / 4299
У сечі лікаря-стоматолога, зібраній наприкінці робочого дня, виявлений вміст білку 0,7 г/л. У ранковій сечі таких змін не виявлено. Як називається це явище?

Гематурія

Позаниркова протеїнурія

Органічна протеїнурія

Неселективна протеїнурія

Функціональна протеїнурія

988 / 4299
Внаслідок травми у хворого спостерігається порушення функції слізної залози. Який нерв забезпечує її секрецію?

N. petrosus major

N. petrosus minor

N. occipitalis minor

N. auricularis magnus

Chorda tympany

989 / 4299
Посів патологічного матеріалу від хворого на пульпіт було здійснено на середовище КіттаТароцці. Які мікроорганізми планується виділити?

Гемолітичні

Аеробні

Ацидофільні

Анаеробні

Кислотостійкі

990 / 4299
У хворого відсутній 16 зуб. На рентгенограмі в товщі альвеолярного відростку визначається розрідження кістки з утворенням порожнини з чіткими межами, що містить недорозвинену коронку зуба. Мікроскопічно: стінка порожнини вистелена багатошаровим плоским епітелієм і оточена фіброзною капсулою. Встановіть діагноз:

Фолікулярна кіста щелепи

Кіста різцевого каналу

Радикулярна кіста щелепи

Примордіальна кіста щелепи

Кістозна амелобластома щелепи

991 / 4299
Хвора на рак нижньої щелепи 52-х років пройшла курс променевої терапії. Розмір пухлини зменшився. Який з наведених механізмів ушкодження клітини найбільш обумовлює ефективність променевої терапії?

Лізис NK-клітинами

Утворення вільних радикалів

Тромбоз судин

Мутагенез

Гіпертермія

992 / 4299
На гістологічному препараті сформованого зуба виявляється оболонка, яка стійка до дії кислот, але зберігається лише на бічних поверхнях коронки зуба. Назвіть цю оболонку:

Кутикула

Цемент

Дентин

Емаль

Пелікула емалі

993 / 4299
Чутливий нервовий ганглій складається з нейроцитів кулястої форми з одним відростком, який на певній відстані від перикаріону поділяється на аксон і дендрит. Як називаються такі клітини?

мультиполярні

псевдоуніполярні

біполярні

уніполярні

аполярні

994 / 4299
Під час рентгенологічного дослідження у хворого виявлена кіста у ділянці премоляра, у порожнині якої міститься зуб. Мікроскопічно: стінка кісти представлена сполучною тканиною і вистелена багатошаровим плоским епітелієм. Який найбільш вірогідний діагноз?

Еозинофільна гранульома

Радикулярна кіста

Епуліс

Примордіальна кіста

Фолікулярна кіста

995 / 4299
До стоматолога звернувся хворий зі скаргою на свербіж і печіння у порожнині рота, підвищену температуру. Був встановлений діагноз: трихомонадний гінгівостоматит. Який засіб слід обрати для лікування?

Ністатин

Ампіцилін

Метронідазол

Гентаміцину сульфат

Доксацикліну гідрохлорид

996 / 4299
У хворого з пухлиною в ділянці верхніх горбків чотиригорбкової пластинки сталося випадіння зіничного рефлексу. Функція якого ядра черепних нервів найімовірніше порушена?

Рухового ядра відвідного нерва

Рухового ядра окорухового нерва

Рухового ядра додаткового нерва

Додаткового ядра окорухового нерва

Рухового ядра блокового нерва

997 / 4299
Які профілактичні препарати слід ввести хворому з відкритою щелепно-лицевою травмою, якщо він раніше ніколи не одержував профілактичних щеплень?

Протиправцевий імуноглобулін і анатоксин

Протиправцеву сироватку та антибіотики

Правцевий анатоксин і антибіотики

Протисудомні препарати та анатоксин

Вакцину АКДП та антибіотики

998 / 4299
При мікроскопічному дослідженні періодонта відзначено: повнокровні судини, набряк сполучної тканини з інфільтрацією поодинокими нейтрофілами. Який різновид ексудативного запалення в періодонті?

Фібринозне

Гнилісне

Катаральне

Гнійне

Серозне

999 / 4299
При гістологічному дослідженні екстирпованої пульпи зуба в її периферичному шарі визначені клітини циліндричної форми. Яку назву мають ці клітини?

Фібробласти

Амелобласти

Моноцити

Міофібробласти

Одонтобласти

1000 / 4299
На прийомі стоматолог виявив у пацієнта каріозні порожнини на передніх зубах, які не мають додаткових антагоністів. Назвіть ці зуби:

Нижні медіальні різці

Верхні латеральні різці

Верхні ікла

Нижні латеральні різці

Верхні медіальні різці

1001 / 4299
У гетерозиготних батьків з A(II) і B(III) групами крові за системою АВ0 народилась дитина. Яка імовірність наявності у неї 0(I) групи крові?

100%

0%

75%

50%

25%

1002 / 4299
У мешканців Закарпаття внаслідок дефіциту йоду в харчових продуктах часто зустрічається ендемічний зоб. Який вид мінливості спричиняє це захворювання?

Модифікаційна

Онтогенетична

Мутаційна

Співвідносна

Комбінативна

1003 / 4299
У медико-генетичну консультацію звернулася хвора дівчинка з попереднім діагнозом 'синдром Шерешевського-Тернера'. Яким генетичним методом можна уточнити діагноз?

Гібридологічний

Біохімічний

Визначення статевого хроматину

Генеалогічний

Дерматогліфіка

1004 / 4299
Провідником наукової експедиції по Індії був місцевий житель, який ніколи не розлучався зі своїм улюбленим собакою. Якими інвазійними захворюваннями можуть бути заражені члени експедиції при контакті з цим собакою, якщо він є джерелом інвазії?

Парагонімоз

Теніоз

Ехінококоз

Дикроцеліоз

Фасціольоз

1005 / 4299
У клітині людини в гранулярну ендоплазматичну сітку до рибосом доставлена і-РНК, що містить як екзонні, так і інтронні ділянки. Який процес НЕ ВІДБУВАЄТЬСЯ?

Транскрипція

Трансляція

Процесінг

Реплікація

Пролонгація

1006 / 4299
Через два тижні після переливання крові у реципієнта виникла лихоманка. Про яке протозойне захворювання повинен думати лікар?

Малярія

Амебіаз

Трипаносомоз

Лейшманіоз

Токсоплазмоз

1007 / 4299
Під час огляду ротової порожнини встановлено, що у пацієнта має місце дотикання ріжучих країв верхніх та нижніх різців. Для якого прикусу характерне таке розміщення зубів?

Прямий прикус

Біпрогнатія

Ортогнатія

Закритий прикус

Прогенія

1008 / 4299
У хворого інфаркт міокарда в ділянці передньої стінки лівого шлуночка. В басейні якої судини виникло порушення кровообігу?

Передсердно-шлуночкова гілка лівої вінцевої артерії

Передня шлуночкова гілка правої вінцевої артерії

Огинаюча гілка лівої вінцевої артерії

Ліва крайова гілка лівої вінцевої артерії

Передня міжшлуночкова гілка лівої вінцевої артерії

1009 / 4299
Після травми хворий не може розігнути руку у ліктьовому суглобі. Порушення функції якого основного м’язу може це спричинити?

Musculus infraspinatus

Musculus levator scapule

Musculus triceps brachii

Musculus teres major

Musculus subscapularis

1010 / 4299
Яка артерія може бути пошкоджена при виконанні провідникової анестезії в ділянці отвору нижньої щелепи?

Язикова артерія

Крилоподібні гілки

Нижня альвеолярна артерія

Щічна артерія

Середня оболонкова артерія

1011 / 4299
Чоловік 65-ти років звернувся до стоматолога зі скаргою, що в нього нижня щелепа не рухається назад. Встановлено, що в нього після падіння пошкоджений м’яз:

Бічний крилоподібний

Двочеревцевий

Скроневий

Присередній крилоподібний

Жувальний

1012 / 4299
У хворого після застудного захворювання з’явилося порушення больової і температурної чутливості передніх 2/3 язика? Який із нервів при цьому постраждав?

Додатковий

Блукаючий

Під’язиковий

Трійчастий

Язикоглотковий

1013 / 4299
У хворого кривошия. Який м’яз шиї уражений?

M.mylohyoideus

M.omohyoideus

M.sternohyoideus

M.platisma

M.sternocleidomastoideus

1014 / 4299
У хворого 25-ти років, який скаржився на утруднене ковтання, виявлене пухлиноподібне підвищення на корені язика 1-2 см в діаметрі, в ділянці сліпого отвору. Розростання залишків якої залози встановив лікар?

Вилочкова

Під’язикова

Аденогіпофіз

Прищитоподібна

Щитоподібна

1015 / 4299
Хворий скаржиться на біль під час жування, особливо під час висунення нижньої щелепи і зміщення її вбік. Функція яких м’язів порушена?

Скроневі

Латеральні крилоподібні

Щелепно-під’язикові

Медіальні крилоподібні

Жувальні

1016 / 4299
Під час виконання операції з приводу пухлини черевної частини сечоводу у хворого, лікар повинен пам’ятати, що попереду його розташована важлива артеріальна судина. Яка саме?

a. iliaca interna

a. iliaca communis

a. ileocolica

a. renalis

a. testicularis

1017 / 4299
Після падіння з дерева у хлопчика утруднене відведення руки до горизонтального положення. Який із м’язів імовірно пошкоджений?

M.coracobrachialis

M.deltoideus

M.anconeus

M.supinator

M.triceps brachii

1018 / 4299
Під час оперативного втручання у жінки виникла необхідність перев’язати маткову артерію. Яке з утворень може бути випадково перев’язаним разом з нею?

Кругла зв’язка матки

Сечівник

Сечовід

Внутрішня клубова вена

Маткова труба

1019 / 4299
При аускультації серця був визначений діастолічний шум в II міжреберному проміжку по правій пригруднинній лінії. Про патологію якого клапана це свідчить?

Клапан легеневого стовбура

Клапан аорти

Тристулковий

Двостулковий

1020 / 4299
У хворої з пухлиною підшлункової залози розвинулася механічна жовтяниця внаслідок стиснення жовчовивідної протоки. Яка протока піддається стисненню?

Ductus cysticus

Ductus hepaticus dexter

Ductus hepaticus communis

Ductus hepaticus sinister

Ductus choledochus

1021 / 4299
У гістологічному препараті шліфа зуба у міжклітинній речовині визначаються колагенові волокна, що йдуть тангенційно до дентино-емалевої межі і перпендикулярно до дентинних трубочок (волокна Ебнера). Назвіть даний шар дентину:

Плащовий дентин

Навколопульпарний дентин

Вторинний дентин

Інтерглобулярний дентин

Зернистий шар

1022 / 4299
Хворій 35-ти років з діагнозом безпліддя в гінекологічному відділенні зроблено діагностичну біопсію ендометрію. При мікроскопічному дослідженні з’ясувалося, що слизова оболонка з явищами набряку, маткові залози звивисті, заповнені густим секретом. Надлишок якого гормону обумовлює такі зміни в ендометрії?

Естроген

Тестостерон

Прогестерон

Соматотропін

АКТГ

1023 / 4299
У крові дівчини 16-ти років, котра страждає на аутоімунне запалення щитоподібної залози, виявлено численні плазматичні клітини. З проліферацією та диференціюванням яких клітин крові пов’язано збільшення кількості плазмоцитів?

Т-супресорів

В-лімфоцитів

Т-кілерів

Т-хелперів

Тканинних базофілів

1024 / 4299
У хворого при гастроскопії виявлено недостатню кількість слизу, що вкриває слизову оболонку. З порушенням функції яких клітин стінки шлунка це пов’язано?

Ендокриноцити

Клітини призматичного залозистого епітелію

Шийкові клітини залоз шлунка

Парієтальні клітини залоз шлунка

Головні екзокриноцити залоз шлунка

1025 / 4299
В червоному кістковому мозку в постембріональному гемопоезі в клітинах одного з диферонів поступово знижується базофілія цитоплазми і підвищується оксифілія, ядро виштовхується. Для якого виду гемопоезу характерні дані морфологічні зміни?

Базофілоцитопоез

Нейтрофілоцитопоез

Лімфопоез

Еозинофілоцитопоез

Еритропоез

1026 / 4299
Недорозвиненість яких відділів лицьового черепу в ембріональний період призводить до появи такої вади розвитку, як 'вовча паща'?

Лобні та верхньощелепні відростки

Піднебінні відростки

Нижньощелепні та піднебінні відростки

Нижньощелепні відростки

Лобні відростки

1027 / 4299
Під час розвитку облітеруючого атеросклерозу у хворих виникають зміни в судинах нижніх кінцівок. На гістологічному препараті такої судини добре виражені внутрішня та зовнішня еластичні мембрани, у середній оболонці багато міоцитів. Яка судина ушкоджується при цьому захворюванні?

Вена з сильним розвитком м’язів

Артерія змішаного типу

Лімфатична судина

Артерія м’язового типу

Артерія еластичного типу

1028 / 4299
У порожнині матки було виявлене плідне яйце, не прикріплене до ендометрію. Якій стадії розвитку відповідає зародок?

Морула

Бластоциста

Гаструла

Нейрула

Зигота

1029 / 4299
На гістологічному препараті очного яблука видно структуру, що має вигляд двоопуклого утвору, сполученого з циліарним тілом за допомогою волокон війкового пояска, зверху вкритий прозорою капсулою. Назвіть цю структуру:

Склера

Рогівка

Кришталик

Війкове тіло

Скловидне тіло

1030 / 4299
У жінки 30-ти років зменшений вміст ферментів у підшлунковому соці. Недостатня секреція якого гастроінтестинального гормону може бути причиною цього?

Шлунково-інгібуючий пептид

Секретин

Вазо-інтестинальний пептид

Соматостатин

Холецистокінін-панкреозимін

1031 / 4299
Відомо, що у людей, які постійно мешкають в умовах високогір’я, збільшується вміст еритроцитів в одиниці об’єму крові. Це сприяє оптимальному виконанню кров’ю такої функції:

Транспортування амінокислот

Транспортування газів

Участь у гемостазi

Підтримка кислотно-лужної рiвнова-ги

Пвдтримка іонної рiвноваги

1032 / 4299
У ході тренування на велоергометрі спортсмен підбирав навантаження для досягнення максимальної величини роботи, що виконується його м’язами. Якою у даному випадку має бути величина навантаження на м’язи спортсмена?

Максимальна

Тривала мінімальна

Середня

Чергування мінімального і максимального

Мінімальна

1033 / 4299
Людина стоїть у кімнаті в легкому одязі; температура повітря +14o C. Вікна і двері зачинені. Яким шляхом вона віддає найбільше тепла?

Конвекція

Перспірація

Випаровування

Теплорадіація

Теплопроведення

1034 / 4299
При аналізі ЕКГ людини з’ясовано, що у другому стандартному відведенні від кінцівок зубці T позитивні, їх амплітуда та тривалість у нормі. Вірним є висновок, що у шлуночках серця нормально відбувається процес:

Деполяризації

Розслаблення

Реполяризації

Скорочення

Збудження

1035 / 4299
У хворого 70-ти років діагностовано крововилив у стовбур мозку. Обстеження виявило підвищення тонусу м’язів-згиначів на тлі зниження тонусу м’язів-розгиначів. Подразненням яких структур мозку можна пояснити зміни тонусу м’язів?

Вестибулярні ядра

Ретикулярна формація

Чотиригорбикова структура

Червоні ядра

Чорна речовина

1036 / 4299
Людина, що приймає блокатор мембранних циторецепторів синапсів еферентних провідників автономної нервової системи, скаржиться на сухість у роті. Які з рецепторів у неї заблоковані?

α-адренорецептори

β-адренорецептори

M-холінорецептори

Н-холінорецептори

Н2-рецептори

1037 / 4299
У дитини наявне порушення формування емалі та дентину зубів через знижений вміст іонів кальцію в крові. Дефіцит якого гормону може спричинити такі порушення?

Тирокальцитонін

Паратгормон

Трийодтиронін

Тироксин

Соматотропний гормон

1038 / 4299
У хворого на ЕКГ виявлено, що інтервал RR дорівнює 1,5 с, частота серцевих скорочень - 40/хв. Що є водієм ритму серця?

Права ніжка пучка Гіса

Атріовентрикулярний вузол

Синусовий вузол

Пучок Гіса

Ліва ніжка пучка Гіса

1039 / 4299
При обстежені пацієнта встановили що він має сильний, врівноважений, інертний тип вищої нервової діяльності за Павловим. Який темперамент за Гіппократом має цей пацієнт?

Холеричний

Флегматичний

Сангвінічний

Меланхолічний

1040 / 4299
У людини вміст глюкози в крові 15 ммоль/л (поріг реабсорбції - 10 ммоль/л). Наслідком цього буде:

Глюкозурія

Зменшення секреції альдостерону

Зменшення діурезу

Зменшення реабсорбції глюкози

Зменшення секреції вазопресину

1041 / 4299
Після руйнування структури ЦНС тварина втратила орієнтувальні рефлекси. Що саме зруйновано?

Чорна речовина

Червоні ядра

Латеральні вестибулярні ядра

Медiальнi ретикулярні ядра

Чотиригорбикова структура

1042 / 4299
При визначенні основного обміну з’ясовано, що його величина у досліджуваного перевищує належну величину на 8%. Це означає, що інтенсивність процесів енергетичного метаболізму у досліджуваного:

Суттєво підвищена

Нормальна

Суттєво пригнічена

Помірно підвищена

Помірно пригнічена

1043 / 4299
На судово-медичну експертизу надійшла кров дитини та передбачуваного батька для встановлення батьківства. Ідентифікацію яких хімічних компонентів необхідно здійснити в досліджуваній крові?

ДНК

р-РНК

м-РНК

т-РНК

мя-РНК

1044 / 4299
Хворий 13-ти років скаржиться на загальну слабкість, запаморочення, втомлюваність. Спостерігається відставання у розумовому розвитку. При обстеженні виявлено високу концентрацію валіну, ізолейцину, лейцину в крові та сечі. Сеча специфічного запаху. Що може бути причиною такого стану?

Тирозиноз

Базедова хвороба

Гістидинемія

Хвороба Аддісона

Хвороба кленового сиропу

1045 / 4299
Пацієнт звернувся зі скаргами на напади утрудненого дихання, запаморочення. Працює на хімічному підприємстві з виробництва синильної кислоти. З порушенням функції якого ферменту можуть бути пов’язані вказані симптоми?

Цитохромоксидаза

Лактатдегідрогеназа

Сукцинатдегідрогеназа

Піруватдегідрогеназа

Каталаза

1046 / 4299
У хворого в крові збільшена концентрація пірувату. Значна кількість його екскретується з сечею. Який авітаміноз спостерігається у хворого?

B1

B3

B2

E

B6

1047 / 4299
Людина в стані спокою штучно примушує себе дихати часто і глибоко впродовж 3-4 хвилин. Як це відбивається на кислотно-лужній рівновазі організму?

Кислотно-лужна рівновага не змінюється

Виникає дихальний ацидоз

Виникає дихальний алкалоз

Виникає метаболічний алкалоз

Виникає метаболічний ацидоз

1048 / 4299
Пацієнтка з високим ступенем ожиріння у якості харчової добавки рекомендований карнітин для поліпшення 'спалювання' жиру. Яку безпосередню участь бере карнітин у процесі окислення жирів?

Транспорт жирних кислот з цитозоля до мітохондрій

Транспорт жирних кислот з жирових депо до тканин

Активація внутрішньоклітинного ліполізу

Бере участь в одній з реакцій бета-окислення жирних кислот

Активація жирних кислот

1049 / 4299
В сечі новонародженого визначається цитрулін та високий рівень аміаку. Утворення якої речовини найімовірніше порушене у цього малюка?

Аміак

Креатин

Сечова кислота

Сечовина

Креатинін

1050 / 4299
Причиною захворювання на пелагру може бути переважне харчування кукурудзою та зниження у раціоні продуктів тваринного походження. Відсутність у раціоні якої амінокислоти призводить до даної патології?

Гістидин

Фенілаланін

Ізолейцин

Метіонін

Триптофан

1051 / 4299
При лікуванні пародонтиту використовують препарати кальцію та гормон, що має здатність стимулювати мiнералiзацiю зу6ів та гальмувати резорбцію кісткової тканини, а саме:

Кальцитонін

Альдостерон

Тироксин

Паратгормон

Адреналін

1052 / 4299
У пілота на висоті 14000 м трапилася аварійна розгерметизація кабіни. Який із видів емболії у нього розвинувся?

Повітряна

Емболія стороннім тілом

Тромбоемболія

Газова

Жирова

1053 / 4299
Після введення місцевого анестетика у пацієнта розвинувся анафілактичний шок. Який механізм порушення кровообігу є провідним при цьому?

Біль

Активація симпато-адреналової системи

Гіперволемія

Зменшення тонусу судин

Зниження скоротливої функції серця

1054 / 4299
У підлітка було видалено зуб із застосуванням новокаїну. Через 10 хвилин у нього з’явилися блідість шкірних покривів, задишка, гіпотензія. При розвитку цієї реакції алерген на тканинних базофілах реагує з:

IgD

Т-лімфоцитами

IgA

IgM

IgE

1055 / 4299
Чоловік 50-ти років хворіє на хронічний бронхіт, скаржиться на задишку при фізичному навантаженні, постійний кашель з відходженням харкотиння. При обстеженні діагностовано ускладнення - емфізема легень. Чим вона обумовлена?

Зменшенням альвеолярної вентиляції

Зменшенням перфузії легень

Зменшенням розтяжності легень

Порушенням вентиляційно-перфузійного співвідношення в легенях

Зниженням еластичних властивостей легень

1056 / 4299
Хвора надійшла до інфекційного відділення зі скаргами на нестримне блювання. Які порушення водно-сольового обміну будуть у хворої?

Гіперосмолярна гіпергідратація

Ізоосмолярна дегідратація

Гіпоосмолярна дегідратація

Гіперосмолярна дегідратація

Гіпоосмолярна гіпергідратація

1057 / 4299
Робітник хімічної промисловості звернувся до лікаря зі скаргою на стирання емалі. Об’єктивно: розповсюджене руйнування коронок зубів з утворенням замісного дентину. Який діагноз найбільш вірогідний?

Клиноподібні дефекти

Середній карієс

Ерозія зубів

Флюороз

Некроз твердих тканин зубів

1058 / 4299
У чоловіка 30-ти років на нижній щелепі зліва в ділянці моляра визначається пухлиноподібне утворення щільної консистенції, дрібногорбисте, що значно деформує щелепу. Утворення місцями нечітко відшароване від кісткової тканини. Мікроскопічно: у стромі біоптату виявляються тяжі, фолікули, на периферії яких - клітини одонтогенного циліндричного епітелію, у центрі - зірчасті клітини, які нагадують пульпу емалевого органа. Який найбільш імовірний діагноз?

Аденоматоїдна пухлина

Аденокарцинома

Остеобластокластома

Первинний внутрішньокістковий рак

Амелобластома

1059 / 4299
У хворої, що страждала на вторинний сифіліс, з’явилися вогнища депігментації шкіри у верхніх відділах спини. Назвіть патологічний процес у шкірі:

Лейкодерма

Паракератоз

Дисплазія

Метаплазія

Лейкоплакія

1060 / 4299
У хворого 14-ти років діагностована тріада Гетчинсона: зуби діжкоподібної форми, паренхіматозний кератит та глухота. Для якої хвороби характерні виявлені зміни?

Проказа

Туберкульоз

Опісторхоз

Сифіліс

Токсоплазмоз

1061 / 4299
Під час розтину тіла чоловіка 70-ти років, який помер від серцевої недостатності, виявлені деформовані, звужені коронарні артерії. На розрізі внутрішня поверхня артерій горбиста, стінка білувата, ламка, кам’янистої щільності. Про яку стадію атеросклерозу йдеться?

Виразкування

Ліпоїдоз

Ліпосклероз

Атероматоз

Атерокальциноз

1062 / 4299
На розтині тіла чоловіка похилого віку, який протягом останніх 2-х тижнів страждав від гострого розладу кишечнику, виявлені зміни у прямій та сигмоподібній кишках: на поверхні слизової оболонки відмічається коричнево-зелена плівка. Стінка кишки потовщена, порожнина різко звужена. Мікроскопічно виявляється проникаючий на різну глибину некроз слизової оболонки, некротичні маси пронизані нитками фібрину, з лейкоцитарною інфільтрацією. Який найбільш вірогідний діагноз?

Фібринозний коліт

Фолікулярний коліт

Виразковий коліт

Катаральний коліт

1063 / 4299
На розтині тіла чоловіка, який помер на 5-у добу захворювання черевним тифом, виявлені наступні зміни: групові фолікули клубової кишки збільшені, повнокровні і виступають над слизовою оболонкою, на їх поверхні видно борозни та звивини. Гістологічно: повнокров’я і набряк тканини, наявність гранульом, які складаються з великих клітин зі світлою цитоплазмою і містять черевнотифозні палички. Про який період місцевих змін при черевному тифі можна думати?

Стадія загоєння виразок

Стадія утворення виразок

Стадія чистих виразок

Стадія некрозу

Стадія мозкоподібного набухання

1064 / 4299
У померлого від хронічної серцевої недостатності у віці 68-ми років на розтині виявлені деформовані, потовщені стулки мітрального клапану, що зрослися між собою, по краю змикання клапанів визначаються дрібні (1-2 мм) тромби. Який ендокардит був причиною розвитку хронічної серцевої недостатності?

Зворотньо-бородавчастий

Дифузний

Гострий бородавчастий

Поліпозно-виразковий

Фібропластичний

1065 / 4299
У дитини 5-ти років підвищилася температура до 40o C, виник різкий головний біль, блювання, неспокій, озноб. Через 4 дні з’явився геморагічний висип на шкірі, олігурія та надниркова недостатність, що і стало причиною смерті. Під час бактеріологічного дослідження мазків з глотки виявлений менінгокок. Яка форма менінгококової інфекції була в хворого?

Менінгококцемія

Менінгококовий менінгіт

Менінгококовий назофарингіт

Менінгоенцефаліт

1066 / 4299
На розтині тіла чоловіка, який помер від гострої постгеморагічної анемії внаслідок легеневої кровотечі, було знайдено: макроскопічно - верхівки легень деформовані, на розрізі містять множинні білувато-сірі осередки діаметром 10-15 мм та множинні патологічні порожнини діаметром до 15 мм зі щільними стінками; мікроскопічно - в стінках порожнин розростання сполучної тканини з наявністю інфільтрату, який складається з епітеліоїдних клітин, багатоядерних гігантів та лімфоцитів. Який найбільш імовірний діагноз?

Гематогенний міліарний туберкульоз легень

Вторинний фіброзно-кавернозний туберкульоз

Гематогенно-дисемінований туберкульоз легень

Прогресуючий первинний туберкульозний комплекс

Первинний туберкульоз без ознак прогресування

1067 / 4299
Для вакцинації використовують токсин, знешкоджений формальдегідом (0,4% розчин) при 37 — 40oC протягом чотирьох тижнів. Вперше такий препарат застосував для профілактики ди- фтерії Рамон. Що це за препарат?

Вбита вакцина

Імуноглобулін

Антитоксична сироватка

Ад’ювант

Анатоксин

1068 / 4299
Пацієнтка 65-ти років з хронічною серцевою недостатністю тривалий час лікується дигітоксином, самостійно визначаючи дозу. Надійшла до стаціонару із скаргами на погіршення загального стану, появу аритмії, нудоту, зменшення виділення сечі, безсоння. З чого слід розпочати лікування хворої?

Відмінити дигітоксин

Ввести розчин кальцію глюконату внутрішньовенно

Призначити дигоксин

Зменшити дозу дигітоксину

Призначити строфантин внутрішньовенно

1069 / 4299
В хірургічному відділенні стоматологічної поліклініки готують хворого до екстракції зуба. Який препарат слід додати до розчину місцевого анестетика для подовження його дії?

Сальбутамол

Октадин

Норадреналіну гідротартрат

Ізадрин

Адреналіну гідрохлорид

1070 / 4299
Хворий надійшов до хірургічного відділення з діагнозом гострий панкреатит. Розпочато консервативне лікування. Призначення якого препарату є патогенетично обґрунтованим?

Контрикал

Хімотрипсин

Фібринолізин

Трипсин

Панкреатин

1071 / 4299
Хвора скаржиться на біль у голені, який підсилюється під час ходи. Об’єктивно: відмічається набряк та почервоніння по ходу вени. Лікар призначив антикоагулянт прямої дії для місцевого вживання. Який препарат можна використати з цією метою?

Мазь саліцилова

Мазь бутадіонова

Мазь гепаринова

Тромбін

Мазь троксевазинова

1072 / 4299
До поліклініки звернувся хворий зі скаргами на біль за грудниною, зади-тттку і серцебиття. Після обстеження лікар діагностував у хворого ІХС і призначив верапаміл. Який механізм дії даного препарату?

Блокує калієві канали

Блокує в-адренорецептори

Блокує а-адренорецептори

Блокує кальцієві канали

Блокує натрієві канали

1073 / 4299
До стоматолога звернувся пацієнт з артритом скронево-нижньощелепного суглоба. Лікарстоматолог призначив мазь із диклофенак-натрієм. Який механізм дії цього лікарського засобу?

Активація циклооксигенази

Пригнічення циклооксигенази

Блокада опіатних рецепторів

Активація опіатних рецепторів

Пригнічення фосфоліпази

1074 / 4299
Новонароджений не зробив перший вдих. При патологоанатомічному розтині тіла встановлено, що при вільних дихальних шляхах легені не розправилися. Що з наведеного могло бути причиною цього?

Розрив бронхів

Збільшення розміру альвеол

Звуження бронхів

Потовщення плеври

Відсутність сурфактанту

1075 / 4299
У хворого спостерігається збільшення проникності стінок кровоносних судин із розвитком підвищеної кровоточивості ясен, виникнення дрібнокра-плинних крововиливів на шкірі, випадіння зубів. Яка патологія має місце у хворого?

Гіповітаміноз A

Гіповітаміноз C

Гіпервітаміноз C

Гіповітаміноз D

Гіпервітаміноз D

1076 / 4299
У хворого внаслідок травми ушкоджені передні корінці спинного мозку. Які структури постраждають при цьому?

Периферійні відростки чутливих нейронів спинномозкових вузлів

Аксони нейронів бокових рогів

Дендрити нейронів спинномозкових вузлів

Аксони мотонейронів і аксони нейронів бокових рогів

Центральні відростки чутливих нейронів спинномозкових вузлів

1077 / 4299
В стоматологічному кабінеті у хворого розвинувся напад бронхіальної астми. Лікар застосував препарат з групи ^-адреноміметиків у вигляді інгаляцій. Який препарат був застосований?

Еуфілін

Адреналіну гідрохлорид

Атропіну сульфат

Сальбутамол

Ефедрину гідрохлорид

1078 / 4299
Хворий скаржиться на головний біль, утруднене дихання. Рентген-дослідження підтвердило діагноз -фронтит (запалення лобової пазухи). В якому носовому ході при огляді порожнини носа можуть спостерігатись гнійні виділення?

Загальний

Нижній

Верхній

Середній

Над верхньою носовою раковиною

1079 / 4299
Під час дослідження мазку крові, взятого у хворого і забарвленого за Ро-мановським, лікар виявив найпростіші і встановив діагноз - хвороба Круза-Чагаса. Яке найпростіше викликало захворювання у даного хворого?

Trypanosoma cruzi

Toxoplasma gondii

Leishmania tropica

Trypanosoma brucei

Leishmania donovani

1080 / 4299
У повітрі приміщення збільшений вміст вуглекислого газу. Як зміниться дихання (глибина і частота) у людини, що увійшла в це приміщення?

Зменшиться частота

Зменшиться глибина

Збільшиться частота

Збільшиться глибина

Збільшаться глибина і частота

1081 / 4299
У хворої 38-ми років після прийому аспірину і сульфаніламідів спостерігається посилений гемоліз еритро- цитів, який викликаний недостатністю глюкозо-6-фосфатдегідрогенази. Порушенням утворення якого коферменту зумовлена ця патологія?

фад-н2

ФМН-Н2

Шридоксальфосфат

НАДФ-Н

У6іхінон

1082 / 4299
При підозрі на туберкульоз хворій дитині зробили пробу Манту Через 24 години у місці введення алергену з’явилися припухлість, гіперемія і болісність. Які основні компоненти визначають розвиток цієї реакції організму?

Макрофаги, В-лімфоцити і моноцити

Мононуклеари, Т-лімфоцити і лім-фокіни

Плазматичні клітини, Т-лімфоцити і лімфокіни

Гранулоцити, Т-лімфоцити і IgG

В-лімфоцити, IgM

1083 / 4299
У дітей часто можна спостерігати утруднене носове дихання, яке пов’язане з надмірним розвитком лімфої-дної тканини слизової оболонки глотки. Розростання яких мигдаликів може спричинити це явище?

Tonsilla tubaria

Tonsilla lingualis

Усіх названих мигдаликів

Tonsilla palatina

Tonsilla pharyngea

1084 / 4299
У крові хворого загальна кількість лейкоцитів 90 • 109/л. В лейкоцитарній формулі: е.- 0%, б.- 0%, ю.- 0%, п.- 2%, с.- 20%, лімфобласти - 1%, пролімфо-цити - 2%, лімфоцити - 70%, м.- 5%, наявні клітини Боткіна-Іумпрехта. Клінічно: збільшені шийні, підщелепні лімфатичні вузли. Для якої патології характерна така клінічна картина?

Гострий лімфолейкоз

Хронічний мієлолейкоз

Хронічний лімфолейкоз

Лімфогранульоматоз

Інфекційний мононуклеоз

1085 / 4299
У деяких клінічно здорових людей в умовах високогір’я виявляються ознаки анемії. У крові в них виявляють серпоподібні еритроцити. Іенотип цих людей:

Аа

ХсХс

ХСХс

АА

аа

1086 / 4299
Хвора на хронічний гепатит скаржиться на підвищення чутливості до барбiтyратiв, які раніше переносила без симптомів інтоксикації. З порушенням якої функції пєчінки це пов’язане у найбільшій мірі?

Іемодинамічна

Утворення жовчі

Фагоцитарна

Метаболічна

Іемопоетична

1087 / 4299
Бактеріологічне дослідження гнійних виділень з уретри з’ясувало присутність бактерій, які за Грамом фарбувалися негативно, нагадували кавові зернини, розкладали глюкозу і мальтозу до кислоти. Розташовувалися в лейкоцитах. До збудників якої хвороби їх віднести?

М ’який шанкр

Меліоїдоз

Іонорея

Сифіліс

Венеричний лімфогранулематоз

1088 / 4299
При обстеженні хворого 35-ти років проведено гістологічне дослідження пунктату червоного кісткового мозку і виявлено значне зменшення кількості мегакаріоцитів. До яких змін периферичної крові це призведе?

Лейкопенія

Тромбоцитопенія

Агранулоцитоз

Тромбоцитоз

Лейкоцитоз

1089 / 4299
Згідно правила сталості числа хромосом, кожний вид більшості тварин має певне і стале число хромосом. Механізмом, що підтримує цю сталість при статевому розмноженні організмів, є:

Амітоз

Мейоз

Шизогонія

Регенерація

Брунькування

1090 / 4299
Під час барбітурового наркозу у хворого 65-ти років почало прогресувати пригнічення дихання. Анестезіолог зробив внутрішньовенну ін’єкцію 10 мл 0,5% розчину бемегріду. Стан хворого покращився, об’єм легеневої вентиляції збільшився. Яке явище полягає в основі взаємодії цих препаратів?

Антагонізм прямий

Антагонізм однобічний

Антагонізм непрямий

Синергізм непрямий

Синергізм прямий

1091 / 4299
Під час проведення трахеотомії у хворого 45-ти років, який потрапив до реанімаційного відділення лікарні з набряком гортані, було випадково перерізано яремну венозну дугу, яка лежить у:

Spatium interscalenum

Spatium retropharyngeale

Spatium interaponeuroticum suprasternale

Spatium antescalenum

Spatium pretracheale

1092 / 4299
Під час виконання фізичного навантаження людина менш чутлива до болю. Причиною цього є активація:

Функції наднирників

Функції щитоподібної залози

Антиноцицептивної системи

Ноцицептивної системи

Симпато-адреналової системи

1093 / 4299
У чоловіка 35-ти років через 30 хвилин після автомобільної аварії виявлена масивна травма нижніх кінцівок без значної зовнішньої кровотечі. Постра-ждалий знаходиться у збудженому стані. Який компонент патогенезу травматичного шоку є у пацієнта провідним і вимагає негайного корегування?

Інтоксикація

Внутрішня плазмовтрата

Порушення функції органів

Біль

Внутрішня кровотеча

1094 / 4299
У хворого діагностовано себорейний дерматит, пов’язаний із дефіцитом вітаміну H (біотину). Порушення активності якого з перелічених ферментів спостерігається у хворого?

Ацетил-КоА-карбоксилаза

Амінотрансферази

Карбамоїлфосфатсинтетаза

Піруватдекарбоксилаза

Алкогольдегідрогеназа

1095 / 4299
Хворий 60-ти років звернувся до лікаря зі скаргами на загруднинний 6іль після фізичного навантаження. Лікар призначив нітрогліцерин. Після застосування препарату загруднинний біль зник, проте з’явився сильний біль голови. Який можливий механізм цієї побічної дії?

Блокада а-адренорецепторів

Гальмування утворення медіаторів у мозку

Зменшення накопичення іонів кальцію

Блокада фосфодиестерази

Підвищення внутрішньочерепного тиску

1096 / 4299
При копрологічному дослідженні у фекаліях хворої знайдено яйця дрібних розмірів із кришечкою. З анамнезу відомо, що жінка часто вживає рибні страви. Який сисун паразитує в організмі?

Кров’яний

Печінковий

Котячий

Легеневий

Ланцетоподібний

1097 / 4299
Відпочиваючи на дачі, хлопчик знайшов павука з такими морфологічними особливостями: довжина - 2 см, округле черевце чорного кольору, на спинному боці якого видно червоні плямочки у два ряди, чотири пари членистих кінцівок вкриті дрібними чорними волосками. Визначте дане членистоноге:

Тарантул

Фаланги

Кліщ

Каракурт

Скорпіон

1098 / 4299
Під час клінічного обстеження пацієнтки виявлено зниження основного обміну на 40%, збільшення маси тіла, зниження температури тіла, одутлість обличчя, порушення статевих функцій, млявість і апатія, зниження інтелекту. Яке порушення функції і якої залози внутрішньої секреції призводить до появи даних симптомів?

Гіперфункція гіпофізу

Гіперфункція щитоподібної залози

Гіпофункція епіфізу

Гіпофункція щитоподібної залози

Гіпофункція паращитоподібних залоз

1099 / 4299
У пацієнта з пошкодженням промі- жного мозку виявлено порушення слуху. Які структури при цьому ушкоджені?

Медіальні колінчасті тіла таламуса

Латеральні колінчасті тіла таламуса

Передні ядра гіпоталамуса

Інтраламінарні ядра гіпоталамуса

Медіальні ядра гіпоталамуса

1100 / 4299
У хворого внаслідок травми розвинувся травматичний шок. Хворий метушливий, багатослівний, блідий. АТ-140/90 мм рт.ст., Ps- 120/хв. Якій стадії шоку відповідає цей стан?

Термінальна

Еректильна

Торпідна

Кінцева

Латентний період

1101 / 4299
Хворому з токсичним паралічем дихального центру для його стимуляції вводили декілька разів кордіамін. Який побічний ефект може виникнути при цьому?

Аритмія

Бронхоспазм

Колапс

Клонічні судоми

Тонічні судоми

1102 / 4299
У хворого, який знаходиться на лікуванні з приводу вірусного гепатиту В, з’явилися ознаки печінкової недостатності. Які зміни крові, що свідчать про порушення білкового обміну, найбільш імовірно будуть спостерігатися у даному випадку?

Абсолютна гіпоальбумінемія

Абсолютна гіперальбумінемія

Білковий склад крові не змінений

Абсолютна гіперфібриногенемія

Абсолютна гіперглобулінемія

1103 / 4299
У чоловіка 30-ти років перед операцією визначили групу крові. Кров резус-позитивна. Реакція аглютинації еритроцитів не відбулася зі стандартними сироватками груп 0(І), А(ІІ), В(ІІІ). Досліджувана кров за системою АВ0 належить до групи:

0(І)

В(ІІІ)

А(ІІ)

АВ(ІУ)

1104 / 4299
Під час травми грудної клітки по- шкоджена хрящова частина ребра. За рахунок якого шару охрястя відбувається регенерація хряща?

Фіброзний

Колагеновий

Еластичний

Волокна Шарпея

Хондрогенний

1105 / 4299
Після парентерального введення препарату в пацієнта розвинувся коматозний стан, спостерiгається дихання типу Чейн-Стокса, зіниці різко звужену колінний рефлекс збережений. Який препарат міг спричинити отруєння?

Аміназин

Морфіну гідрохлорид

Сибазон

Анальгін

Фенобарбітал

1106 / 4299
На гістологічному препараті підщелепної залози видно вивідну протоку. Слизова оболонка протоки вистелена низьким кубічним епітелієм, клітини якого мають слабо розвинуті органели. Що це за вивідна протока?

Міжчасточкова

Вставна

Загальна вивідна

Посмугована

1107 / 4299
Під час голодування нормальний рівень глюкози підтримується за рахунок активації глюконеогенезу. Назвіть речовину яка може використовуватись як субстрат для цього процесу:

Аланін

Сечовина

Аденін

Гуанін

Амоніак

1108 / 4299
У хворого після застосування крапель, що містять атропін, виникло стійке розширення зіниці. Роботу якого м’яза блоковано?

Прямий

Косий

Війковий

Розширювач зіниці

Звужувач зіниці

1109 / 4299
Жінка під час пологів втратила близько 800 мл крові. Відзначаються тахікардія, АТ100/70 мм рт.ст., тахі-пное до 28/хв. Який тип гіпоксії пер- винно розвивається при такій клінічній ситуації?

Дихальна

Змішана

Тканинна

Серцево-судинна

Кров’яна

1110 / 4299
Хворому для лікування ІХС лікар призначив антиангінальний засіб, який активує гуанілатциклазу і накопичує цГМФ у клітинах міокарду. Який це засіб?

Панангін

Валідол

Ізосорбіду мононітрат

Верапаміл

Дипіридамол

1111 / 4299
У хворого, що надійшов до інфекційного відділення зі скаргами на судомне скорочення м’язів обличчя, з садна правої нижньої кінцівки були виділені бактерії з термінальним розташуванням спор, що надає їм вигляд 'барабанних паличок'. Яким бактеріям притаманні дані властивості?

Clostridium tetani

Bacillus anthracis

Clostridium botulinum

Bacillus cereus

Clostridium perfringens

1112 / 4299
Хворий звернувся з приводу карбункула на обличчі. Об’єктивно: нещільний, без болю набряк підшкірної клітковини, у центрі карбункулу чорний струп, по периферії навколо карбункула везикулярні висипання. Бактеріологічне дослідження довело наявність нерухомих стрептобацил, які здатні утворювати капсули. Які мікроорганізми є збудниками даної хвороби?

Bacillus subtilis

Bacillus antracis

Staphylococcus aureus

Bacillus anthracoides

Bacillus megaterium

1113 / 4299
На схемі представлена екзокринна залоза, яка має нерозгалужену вивідну протоку, в яку відкривається один кінцевий відділ у вигляді одного мішечка. Як буде називатися така залоза відповідно до морфологічної класифікації екзокринних залоз?

Складна розгалужена альвеолярна

Складна нерозгалужена альвеолярно-трубчаста

Проста нерозгалужена альвеолярна

Проста розгалужена трубчаста

Складна нерозгалужена альвеолярна

1114 / 4299
У хворого з порушенням згортання кровї виявлений тромбоз однієї з гілок нижньої брижової артерії. Який відділ кишечнику вражений?

Colon ascendens

Ileum

Colon sigmoideum

Colon transversum

Caecum

1115 / 4299
Дослідженнями останніх дєсятиріч встановлено, що безпосередніми 'виконавцями'апоптозу в клітині є особливі ферменти - каспази. В утворенні одного з них бере участь цитохром С. Вкажіть його функцію в нормальній клітині:

Фермент ЦТК

Компонент піруватдегідрогеназної системи

Фермент дихального ланцюга переносу електронів

Фермент бета-окислення жирних кислот

Компонент H+-АТФазної системи

1116 / 4299
До хірургічного відділення госпіталізовано хворого для оперативного втручання. Йому необхідно провести нейролептанальгезію. Який з препаратів доцільно застосувати в комбінації з фентанілом для нейролептанальгезії?

Пілокарпін

Холосас

Дроперидол

Сальбутамол

Фраксипарин

1117 / 4299
Хворому на ревматоїдний артрит для попередження можливого негативного впливу на слизову шлунка призначили препарат із групи нестероїдних протизапальних засобів - селективний інгібітор ЦОГ-2. Вкажіть препарат:

Ацетилсаліцилова кислота

Анальгін

Целекоксиб

Бутадіон

Ібупрофен

1118 / 4299
При лікуванні багатьох захворювань використовується фармацевти- чний препарат кокарбоксилаза (тіамін-пірофосфат) для забезпечення клітин енергією. Який метаболічний процес при цьому активується?

Декарбоксилювання біогенних амінів

Декарбоксилювання амінокислот

Дезамінування глутамату

Детоксикація шкідливих речовин у печінці

Окисне декарбоксилювання пірувату

1119 / 4299
Хворій, що хворіє на інсуліноне-залежний цукровий діабет, призначено всередину глібенкламід. Вкажіть механізм гіпоглікемічної дії цього препарату:

Посилює утилізацію глюкози периферичними тканинами

Пригнічує а-глюкозидазу і розпад полісахаридів

Пригнічує глюконеогенез у печінці

Стимулює виділення ендогенного інсуліну в-клітинами

Пригнічує всмоктування глюкози у кишечнику

1120 / 4299
При дегельмінтизації у хворого виявлені довгі фрагменти гельмінта, що має членисту будову. Зрілі членики прямокутної форми 30х12 мм, матка закритого типу у вигляді стовбура, від якого відходять 17-35 бічних відгалужень. Визначте вид гельмінта:

Альвеокок

Ціп ’як озброєний

Ціп’як неозброєний

Ціп ’як карликовий

Ехінокок

1121 / 4299
У хворого з виразковою хворобою після проведеного лікування нормалізувалися травлення, зник біль, поліпшився настрій. Однак через кілька тижнів знову з’явився біль у епігастрії, печія, відрижка кислим. Як охарактеризувати такий перебіг хвороби?

Латентний період

Рецидив

Продромальний період

Період ремісії

Ускладнення

1122 / 4299
Під час гемотрасфузій рекомендується переливати кров лише відповідної групи. Належність до даної (за системою АВ0) групи зумовлюють:

Вуглеводні детермінанти мембран лейкоцитів

Білкові детермінанти мембран еритроцитів

Вуглеводні детермінанти мембран еритроцитів

Білки сироватки крові

Білково-полісахаридні компоненти лейкоцитів

1123 / 4299
Внаслідок травми у чоловіка 35-ти років настав повний розрив спинного мозку на рівні першого шийного сегменту. Як зміниться при цьому зовнішнє дихання?

Стане діафрагмальним

Зупиниться

Стане поверхневим і частим

Не зміниться

Стане рідким і глибоким

1124 / 4299
При обстеженні хворого на атрофічний гастрит виявлено мегалобла-стну анемію. Дефіцит якої речовини є причиною виникнення анемії у цього хворого?

Вітамін Ві

Залізо

Еритропоетини

Гастромукопротеїд

Вітамін В6

1125 / 4299
Хворому на сифіліс призначили лікарський засіб, в основі механізму дії якого лежить порушення утворення муреїну, що призводить до загибелі збудника. Визначить цей препарат:

Ципрофлоксацин

Доксацикліну гідрохлорид

Бензилпеніциліну натрієва сіль

Бійохінол

Азитроміцин

1126 / 4299
У хворого, що знаходиться у опіковому відділенні, виникло гнійне ускладнення. Гній, що виділяється, має синьо-зелений відтінок, що вказує на інфекцію, зумовлену Pseudomonas aeruginosa. Яка ознака характерна для цього збудника?

Негативне забарвлення за Грамом

Наявність спор

Кокова форма

Утворення міцелію

Розташування клітин парами

1127 / 4299
При забарвленні препарату з харкотиння хворого були використані наступні барвники та реактиви: розчин фуксину Циля, розчин метиленового синього, 5% розчин сірчаної кислоти. Який спосіб забарвлення було застосовано?

Гінса-Буррі

Нейссера

Циля-Нільсона

Грама

Пєшкова

1128 / 4299
У людини цистинурія проявляється у вигляді наявності цистинових камінців у нирках (гомозиготи) або підвищеним рівнем цистину в сечі (гетерозиготи). Захворювання цистинурією є моногенним. Визначити тип взаємодії генів цистинурії і нормального вмісту цистину в сечі:

Комплементарність

Кодомінування

Епістаз

Неповне домінування

Повне домінування

1129 / 4299
При обстеженні в клініці у чоловіка діагностували гостру променеву хворобу. Лабораторно встановлено різке зниження серотоніну в тромбоцитах. Порушення метаболізму якої речовини є можливою причиною зниження тром-боцитарного серотоніну?

Фенілаланін

Тирозин

Серин

5-окситриптофан

Гістидин

1130 / 4299
У хворої 40-ка років збільшена щитоподібна залоза. Під час пальпації залоза щільна, поверхня її дрібногорби-ста. При гістологічному дослідженні біоптату щитоподібної залози виявлено дифузну інфільтрацію тканини лімфоцитами, плазматичними клітинами, утворення лімфоїдних фолікулів. Яке захворювання у жінки?

Тиреоїдит Ріделя

Спорадичний зоб

Дифузний токсичний зоб

Аутоімунний тиреоїдит

Ендемічний зоб

1131 / 4299
Хворий потрапив до лікарні після іонізуючого опромінення зі скаргами на блювання, анорексію, біль у різних ділянках живота, наявність крові у калі, підвищення температури тіла, кволість. Для якої форми гострої променевої хвороби характерна клінічна карти- на?

Токсемічна

Змішана

Кістковомозкова

Кишкова

Церебральна

1132 / 4299
Хворому чоловіку 75-ти років, в якого частота серцевих скорочень була 40/хв, імплантували серцевий електростимулятор. Після цього ЧСС зросла до 70/хв. Функцію якого відділу серця взяв на себе електростимулятор?

Ніжки Гіса

Атріовентрикулярного вузла

Волокон Пуркін’є

Волокон пучка Гіса

Синоатрільного вузла

1133 / 4299
У чоловіка 40-ка років на шиї виявлено пухлиноподібне утворення розміром 8х7 см, яке хірург видалив неповністю через інтимний зв’язок з великими судинами. Мікроскопічно в ньому виражений тканинний і клітинний атипізм, клітини типу ліпобластів різного ступеня зрілості, з поліморфізмом, гіперхромією ядер, патологічними мітозами, осередками некрозу. Визначте гістологічну форму пухлини:

Фібросаркома

Фіброма

Ліпома

Гібернома

Ліпосаркома

1134 / 4299
Відомо, що при цукровому діабеті у хворих частіше зустрічаються запальні процеси, знижена регенерація, уповільнюється загоєння ран. Причиною цього є:

Посилення катаболізму

Підвищення ліполізу

Зниження ліполізу

Прискорення глюконеогенезу

Зниження протеосинтезу

1135 / 4299
У хворої 60-ти років знижена активність основного травного ферменту слини. В цьому випадку порушується первинний гідроліз:

Жирів

Білків

Вуглеводів

Молочного цукру

Клітковини

1136 / 4299
Людина, що тривало приймала лі- ки, не може припинити їх використання, тому що при цьому у неї виникають порушення психічних і соматичних функцій. Як називається такий синдром, що виникає при відмові від прийому лікарської речовини?

Кумуляція

Сенсибілізація

Ідіосинкразія

Абстиненція

Тахіфілаксія

1137 / 4299
У жінки 49-ти років внаслідок тривалого стояння з’явився набряк ніг. Яка можлива причина появи набряків?

Збільшення гідростатичного тиску крові у венах

Зменшення гідростатичного тиску крові у венах

Збільшення системного артеріального тиску

Зменшення гідростатичного тиску крові в артеріях

Збільшення онкотичного тиску плазми крові

1138 / 4299
Хворому після апендектомії з метою профілактики інфекції призначений антибіотик групи цефалоспоринів. Порушення якого процесу полягає в основі протимікробної активності антибіотиків цієї групи?

Рибосомальний синтез білку

Утворення мікробної стінки

Енергетичний обмін

Блокада холінестерази

Синтез нуклеїнових кислот

1139 / 4299
Людина, що тривалий час знаходилася у задушливому приміщенні, знепритомніла. Свідомість відновилася після вдихання парів нашатирного спирту. З прямим впливом на які структури пов’язана дія цієї речовини?

Судиноруховий центр

Рецептори верхніх дихальних шляхів

Дихальний центр

Ємкісні судини

Резистивні судини

1140 / 4299
У хворого, що страждає на хронічну ниркову недостатність, виявлено у крові підвищення рівня залишкового азоту до 35 ммоль/л, більше половини якого складає сечовина. Виявлена гіпе-разотемія є:

Ретенційною

Резидуальною

Продукційною

Печінковою

Змішаною

1141 / 4299
При огляді порожнини рота дитини педiатр знайшла 8 різців. Розвиток дитини відповідає віковій нормі. Визначте вік дитини:

7-8 місяців

16-20 місяців

6-7 місяців

12-15 місяців

10-12 місяців

1142 / 4299
У хворого на жовтяницю у крові виявлено збільшення загального білірубіну за рахунок непрямої його фракції. Сеча і кал інтенсивно забарвлені. Який найбільш вірогідний механізм вказаних порушень?

Підвищений гемоліз еритроцитів

Порушення перетворення уробіліно-гену в печінці

Пошкодження паренхіми печінки

Порушення утворення прямого білірубіну

Утруднення відтоку жовчі з печінки

1143 / 4299
При мікроскопічному дослідженні виявляється паренхіматозний орган, в якому епітеліальні тяжі формують клу-бочкову, пучкову та сітчасту зони. Центральна частина органу представлена скупченнями хромафінних клітин. Визначте орган:

Гіпофіз

Епіфіз

Щитоподібна залоза

Надниркова залоза

Печінка

1144 / 4299
У хворого круглясті виразки на обличчі, запалення та збільшення лімфатичних вузлів. Ці симптоми з’явилися після укусів москітів. Під час лабораторного дослідження виділень із виразок на обличчі виявлено одноклітинні безджгутикові організми. Який діагноз найбільш вірогідний?

Короста

Трипаносомоз

Міаз

Дерматотропний лейшманіоз

Токсоплазмоз

1145 / 4299
У чоловіка має місце стеноз мі-трального отвору. Який механізм сер- цевої недостатності є провідним?

Перевантаження напругою

Перевантаження об’ємом

Пошкодження міокарда

Перевантаження серця припливом крові

Перевантаження опором

1146 / 4299
Хворий проходив чергове обстеження, в результаті якого у нього виявлено гіперглікемію, кетонурію, поліурію, глюкозурію. Яка форма порушення КОС має місце за наявності цих явищ?

Метаболічний алкалоз

Негазовий ацидоз

Газовий ацидоз

Газовий алкалоз

Метаболічний ацидоз

1147 / 4299
В препараті сполучної тканини дерми шкіри, забарвленому суданом-III і гематоксиліном, виявляються скупчення великих багатокутних клітин, які фарбуються в помаранчевий колір. Ядра мають сплощену форму, зміщені на периферію. Яка це тканина?

Бура жирова

Ретикулярна сполучна

Біла жирова

Гіалінова хрящова

Пластинчаста кісткова

1148 / 4299
Введення знеболюючого пацієнту перед екстракцією зуба призвело до розвитку анафілактичного шоку, який супроводжувався розвитком олігурії. Який патогенетичний механізм зумовив зменшення діурезу в даній клінічній ситуації?

Пошкодження клубочкового фільтру

Підвищення гідростатичного тиску в капсулі Шумлянського-Боумена

Збільшення онкотичного тиску плазми крові

Збільшення секреції вазопресину

Зниження гідростатичного тиску в капілярах ниркових тілець

1149 / 4299
Хворого госпіталізовано з проникаючою травмою дна порожнини рота. Який м’яз травмований?

Щито -під’язиковий

Груднинно-під’язиковий

Лопатково-під’язиковий

Шило-під’язиковий

Щелепно-під’язиковий

1150 / 4299
У хворого з переломом стегнової кістки в ділянці хірургічної шийки з’явились ознаки гострої правошлуночко-вої недостатності внаслідок емболії легеневої артерії. Який вид емболії має місце?

Повітряна

Жирова

Тканинна

Метастатична

Газова

1151 / 4299
Внаслідок вивиху нижньої щелепи у пацієнта порушена смакова чутливість передніх 2/3 язика та слиновиділення. Травмою якого нерва це викликано?

Глибокий кам’янистий нерв

Під’язиковий нерв

Великий кам’янистий нерв

Барабанна струна

Малий кам’янистий нерв

1152 / 4299
Пацієнт скаржиться на біль у верхній щелепі та зубах. Об’єктивно: відмічається біль при натискуванні в ділянці підочноямкового отвору. Який нерв уражено?

Лицевий нерв

Друга гілка трійчастого нерва

Перша гілка трійчастого нерва

Третя гілка трійчастого нерва

Блоковий нерв

1153 / 4299
Хворий 40-ка років помер від набряку головного мозку. В анамнезі карбункул обличчя. На аутопсії відмічено повнокров’я та набряк тканини головного мозку. У білій речовині лівої півкулі виявлено дві порожнини розмірами 6х5,5 см та 5х4,5 см, що заповнені вершковоподібною рідиною жовтувато-зеленого кольору. Стінками порожнин є нервова тканина з нерівними краями. Яке ускладнення карбункула розвинулось у хворого?

Хронічні абсцеси

Емпієма

Гострі абсцеси

Колікваційні некрози

Кісти

1154 / 4299
При дослідженні трубчастого органу встановлено, що його середня оболонка складається із гіалінових суцільних кілець. Який епітелій вистилає слизову оболонку цього органа?

Одношаровий призматичний з облямівкою

Одношаровий кубічний

Одношаровий призматичний залозистий

Багаторядний призматичний війчастий

Багатошаровий плоский незрогові-лий

1155 / 4299
Хвора госпіталізована з діагнозом плеврит. В якому місці плевральної порожнини міститься найбільша кількість ексудату?

Діафрагмально-медіастинальний синус

Реберно-медіастинальний синус

Під куполом плеври

Під коренем легенів

Реберно-діафрагмальний синус

1156 / 4299
При дослідженні бронхобіоптату встановлено: атрофія слизової оболонки, кістозне перетворення залоз, осередкова метаплазія покривного призматичного епітелію в багатошаровий плоский, збільшення числа келихоподібних клітин; місцями - у стінці бронха та особливо у слизовій оболонці різко виражена клітинна запальна інфільтрація і розростання грануляційної тканини, яка вибухає у просвіт бронха у вигляді поліпа. Який найбільш імовірний діагноз?

Гострий бронхіт

Інтерстиційна пневмонія

Часткова пневмонія

Хронічний бронхіт

Бронхопневмонія

1157 / 4299
У місцевому імунітеті порожнини рота важливу роль відіграють різноманітні клітини слизової оболонки і антимікробні речовини, які синтезуються ними. Які з перерахованих факторів мають вирішальну роль у забезпеченні локального імунітету?

В-лімфоцити

IgG

Секреторний IgA

Макрофаги

Еозинофіли

1158 / 4299
На прийомі лікар-стоматолог виявив у жінки 36-ти років на щічній поверхні ясни у ділянці 2-го моляра утворення у вигляді вузлика діаметром 0,8 см темно-бурого кольору, м’якої консистенції, на широкій основі. Гістологічно утворення багате на судини си- нусоїдного типу з великою кількістю округлих одноядерних і великих багатоядерних клітин, місцями зустрічається скупчення гранул гемосидерину. Який найбільш імовірний діагноз?

Остеобластокластома щелепи

Амелобластома

Ангіоматозний епулід

Прикоренева гранульома

Гігантоклітинний епулід

1159 / 4299
Для знечулення різців верхньої щелепи анестетик вводять в ділянку різцевого отвору. Який нерв залягає в цьому місці?

Rr.nasales posteriores inferiores

N.palatinus major

Nn.palatini minores

N.nasopalatinus

N.pharyngeus

1160 / 4299
В процесі гістогенезу тканин зуба з певних причин вчасно не утворився дентин. Який процес подальшого гістогенезу не відбудеться або буде віддалений у часі?

Утворення безклітинного цементу

Утворення клітинного цементу

Утворення пульпи

Утворення предентинового простору

Утворення емалі

1161 / 4299
При огляді порожнини носа виявлено викривлення задньої частини носової перегородки. Яка кістка викривлена?

Перпендикулярна пластинка решітчастої кістки

Леміш

Латеральна пластинка крилоподібного відростка

Вертикальна пластинка піднебінної кістки

Медіальна пластинка крилоподібного відростка

1162 / 4299
У чоловіка 70-ти років розвинувся протезний стоматит. Крім того, спостерігалось виражене ураження куточків рота. Під час мікроскопії виявлені великі овоїдні грампозитивні клітини. Які мікроорганізми найбільш імовірно стали провідним етіологічним фактором такого ураження?

Стафілококи

Нейсерії

Коринебактерії

Стрептококи

Гриби роду Candida

1163 / 4299
При остеолатеризмі зменшується міцність колагену, зумовлена помітним зменшенням утворення поперечних зшивок у колагенових фібрилах. Причиною цього є зниження активності:

Колагенази

Пролілгідроксилази

Лізилгідроксилази

Лізилоксидази

Моноаміноксидази

1164 / 4299
У біоптаті нирки виявлено: проміжна тканина інфільтрована лейкоцитами, міліарні абсцеси, канальці в стані дистрофії, заповнені десквамованим епітелієм та лейкоцитами. Про яке захворювання можна думати?

Пієліт

Некротичний нефроз

Гломерулонефрит

Нефролітіаз

Пієлонефрит

1165 / 4299
У дитини з нирковою недостатністю виявлена затримка прорізування зубів. Порушення утворення в нирках якої речовини найбільш імовірно є причиною цього?

Ілікоціамін

а-кетоглутарат

1,25 (OH)2D3

Глутамат

Гідроксильований лізин

1166 / 4299
У чоловіка під час операції з приводу пахвинної грижі хірург пошкодив вміст пахвинного каналу. Яку структуру пошкодив хірург?

Funiculus spermaticus

Urarchus

Lig. teres uteri

Lig. inguinale

1167 / 4299
Чоловік 26-ти років внаслідок автомобільної аварії перебуває в торпідній стадії шоку. У крові: лейк.- 3, 2 • 109/л. Який головний механізм в розвитку лейкопенії?

Пригнічення лейкопоезу

Порушення виходу зрілих лейкоцитів з кісткового мозку в кров

Підвищення виділення лейкоцитів з організму

Руйнування лейкоцитів у кровотворних органах

Перерозподіл лейкоцитів у судинному руслі

1168 / 4299
Під час дослідження чоловіка 24-х років виявлені наступні зміни сечі: добовий діурез - 10 літрів, відносна густина сечі -1,001, якісні зміни відсутні. Пацієнт скаржиться на сильну спрагу, часте сечовиділення. Що є найбільш імовірною причиною даного захворювання?

Гіперсекреція вазопресину

Гіперсекреція альдостерону

Гіперсекреція глюкокортикоїдів

Гіпосекреція вазопресину

Відносна інсулінова недостатність

1169 / 4299
При регенерації епітелію слизової оболонки порожнини рота (розмноження клітин) відбулася реплікація (авторепродукція) ДНК за напівконсерва-тивним механізмом. При цьому нукле-отиди нової нитки ДНК є комплементарними до:

Ферменту ДНК-полімерази

Материнської нитки

!нтронних ділянок гену

Змістовних кодонів

Ферменту РНК-полімерази

1170 / 4299
У дитини 7-ми років, хворої на ангіну, був взятий мазок з поверхні мигдаликів і засіяний на кров’яний агар. Через добу виросли колонії стрептококів, навколо яких середовище стало прозорим. Наявність якого фактора патогенності у збудника виявило це дослідження?

Нейрамінідаза

Гемолізин

Ендотоксин

Бета -лактамаза

Лейкоцидин

1171 / 4299
Хлопчик 5-ти місяців госпіталізований з приводу тонічних судом. Хворіє з народження. При огляді волосся жорстке, нігті витончені та ламкі, шкірні покриви бліді та сухі. У крові: кальцій -1,5 ммоль/л, фосфор -1,9 ммоль/л. З чим пов’язані ці зміни?

Гіпоальдостеронізм

Гіперпаратиреоз

Гіпотиреоз

Гіперальдостеронізм

Гіпопаратиреоз

1172 / 4299
У чоловіка 60-ти років спостерігається послаблення перистальтики кишечнику. Який з перерахованих харчових продуктів буде стимулювати перистальтику в найбільшій мірі?

Чай

М’ясо

Чорний хліб

Сало

Білий хліб

1173 / 4299
При пункційній біопсії печінки хворого з клінікою печінково-клітинної недостатності виявлено вакуольну, балонну дистрофію гепатоцитів, некроз окремих клітин, тільця Каунсильмена, інфільтрацію портальної та часточкової строми переважно лімфоцитами, макрофагами з незначною кількістю поліморфноядерних лейкоцитів. Який найбільш імовірний діагноз?

Гострий вірусний гепатит

Аутоімунний гепатит

Алкогольний гепатит

Хронічний активний гепатит

Хронічний персистуючий гепатит

1174 / 4299
При посіві матеріалу із зіва хворого ангіною на кров’яно-телуритовий агар виросли колонії діаметром 4-5 мм, сірого кольору, радіально покреслені (у вигляді розеток). Під мікроскопом грам-позитивні палички із булавоподібними потовщеннями на кінцях, розміщені у вигляді розчепірених пальців. Які це мікроорганізми?

Стрептобацили

Коринебактерії дифтерії

Клостридії ботулізму

Дифтероїди

Стрептококи

1175 / 4299
Під час цитогенетичного обстеження пацієнта з порушеною репродуктивною функцією виявлено в деяких клітинах нормальний каріотип 46,ХУ але у більшості клітин каріотип синдрому Клайнфельтера - 47,ХХУ. Яку назву носить таке явище неоднорідності клітин?

Мозаїцизм

Інверсія

Мономорфізм

Дуплікація

Транспозиція

1176 / 4299
При диспансерному обстеженні хлопчику 7-ми років встановлено діагноз - синдром ЛешаНайхана (хворіють тільки хлопчики). Батьки здорові, але у дідуся за материнською лінією таке ж захворювання. Який тип успадкування захворювання?

Домінантний, зчеплений із статтю

Рецесивний, зчеплений із статтю

Автосомно-рецесивний

Автосомно-домінантний

Неповне домінування

1177 / 4299
В експерименті необхідно оцінити рівень збудливості клітини. Для цього доцільно визначити:

Тривалість ПД

Критичний рівень деполяризації

Потенціал спокою

Поріг деполяризації

Амплітуду ПД

1178 / 4299
В кардіологічному відділенні в хворого виникла аритмія. Лікар призначив аміодарон. Який основний механізм протиаритмічної дії аміодарону?

Переважно блокує калієві канали

Активує серотонінові рецептори

Стимулює гістамінові рецептори

Пригнічує холінорецептори

Змінює чутливість міокарду до ацетилхоліну

1179 / 4299
При знеболенні слизової оболонки порожнини рота у пацієнта виник анафілактичний шок (генералізована ва-зодилятація, збільшення судинної проникності із виходом рідини із судин у тканини). Гіперчутливість якого типу розвинулася у пацієнта?

IV типу (клітинна цитотоксичність)

ІІ типу (антитілозалежна)

ІІІ типу (імунокомплексна)

І типу (анафілактична)

V типу (гранульоматоз)

1180 / 4299
В експерименті у тварини перерізали таламокортикальні шляхи. Який вид сенсорних відчуттів у піддослідної тварини зберігся?

Екстероцептивні

Зорові

Ноцицептивні

Нюхові

Слухові

1181 / 4299
Хворому з набряками призначено K+-зберігаючий діуретик - антагоніст альдостерону. Визначте цей препарат:

Спіронолактон

Новокаїнамід

Дигоксин

Клофелін

Алопуринол

1182 / 4299
У хворих на еритропоетичну пор-фірію (хвороба Гюнтера) зуби флюо-ресцують в ультрафіолеті яскраво червоним кольором, шкіра чутлива до світла, сеча забарвлена в червоний колір. З недостатністю якого ферменту пов’язана ця хвороба?

Дельта-амінолевулінатсинтаза

Уропорфіриноген-декарбоксилаза

Феррохелатаза

Уропорфіриноген-І-синтаза

Уропорфіриноген-ІІІ-косинтаза

1183 / 4299
Хворому 25-ти років з клінічною картиною нефротичного синдрому проведено пункційну біопсію нирки. Під час мікроскопічного дослідження клітини епітелію проксимальних канальців нефрону збільшені в об’ємі, у цитоплазмі вакуолі з прозорою рідиною, ядро зміщене до периферії. Яка дистрофія виявлена в епітелії каналь-ців?

Зерниста

Гіаліново-крапельна

Гідропічна

Жирова

Рогова

1184 / 4299
До стоматолога звернувся хворий з ураженням слизової оболонки ротової порожнини. Було встановлено діагноз: герпетичний стоматит. Який з перелічених засобів забезпечить дію на етіо-тропний фактор?

Димедрол

Левамізол

Ацикловір

Парацетамол

Фурацилін

1185 / 4299
У пацієнта 42-х років, що страждає на парадонтоз, у коронковій частині пульпи виявлені округлі звапно- вані утворення діаметром 2-3 мм. Назвіть ці структури:

Склерозований (прозорий) дентин

Дєнтиклі

Мертвий дентин

!нтерглобулярний дентин

!нтерглобулярні простори

1186 / 4299
У хворого виявлено порушення секреторної функції піднижньощелепної слинної залози. Який нерв забезпечує її вегетативну іннервацію?

N.mandibularis

N.petrosus major

Chorda tympani

N.auriculotemporalis

N.petrosus minor

1187 / 4299
При гістологічному дослідженні мі-кропрепарату злоякісної пухлини легень виявлено, що вона складається із лімфоцитоподібних клітин, що не утворюють будь-яких структур. Строма виражена мало, спостерігається багато мітозів та некрозів. Яка це пухлина?

Аденокарцинома

Плоскоклітинний зроговілий рак

Фіброма

Дрібноклітинний рак

Плоскоклітинний незроговілий рак

1188 / 4299
До стоматолога звернувся чоловік 35-ти років зі скаргами на зменшення щільності зубної тканини, підвищену крихкість зубів при вживанні твердої їжі. Лабораторно визначили співвідношення Ca/P в емалі при зіскоблюванні. Яке значення цього показника свідчить про посилення демінералізації?

2,5

1,5

1,85

0,9

1,67

1189 / 4299
У пацієнта вилучений верхній медіальний різець. Гілки якої артерії його кровопостачають?

A.sphenopalatina

A.alveolaris inferior

A.buccalis

A.palatina descendens

A.infraorbitalis

1190 / 4299
У хворої 35-ти років з хронічним періодонтитом біля кореня 15 зуба видалено кісту діаметром 3 см. При гістологічному досліджені встановлено, що стінка її тонка, утворена зрілою сполучною тканиною, яка інфільтрована лімфоцитами і плазматичними клітинами, внутрішня поверхня вистелена багатошаровим плоским епітелієм без ознак кератинізації, в порожнині серозний ексудат. Який найбільш імовірний діагноз?

Радикулярна кіста

Фолікулярна кіста

Херувізм

Примордіальна кіста

Фолікулярна амелобластома

1191 / 4299
При гістологічному дослідженні міокарда у хворого 47-ми років з ревматичною вадою серця (секційний матеріал) у кардіоміоцитах виявлені великі оптично порожні вакуолі. При забарвленні осмієвою кислотою вони чорного кольору, при забарвленні суданом III - жовто-червоні. Назвіть вид патологічного процесу:

Гідропічна дистрофія

Гіаліново-крапельна дистрофія

Вуглеводна дистрофія

Жирова дистрофія

Диспротеїноз

1192 / 4299
Чутливий нервовий ганглій складається з нейроцитів кулястої форми з одним відростком, який на певній відстані від перикаріону поділяється на аксон і дендрит. Як називаються такі клітини?

псевдоуніполярні

біполярні

аполярні

уніполярні

мультиполярні

1193 / 4299
У пацієнта навесні з’явилися пете-хіальні крововиливи, розхитування зубів, він відмічає високу чутливість до простудних хвороб. Лікар припустив гіповітаміноз C. Чим пояснюються зміни з боку зубів?

Зміна структури глікозаміногліканів

Порушення окисно-відновних процесів у навколозубних тканинах

Порушення структури колагену періодонтальних зв’язок

Підвищення проникності мембран навколозубних тканин

Механічне ушкодження зубів

1194 / 4299
До лабораторії особливо небезпечних інфекцій доставлено матеріал хворого з підозрою на холеру. Який ме- тод експрес діагностики може підтвердити цей діагноз?

PЗК

РГА

РА

РІФ

РП

1195 / 4299
При обстеженні хворого встановлено, що причиною гіпоплазії зу6ів є гіповітаміноз A та D. Ці вітаміни призначили перорально, проте лікувального ефекту не досягли. Яка можлива причина порушення засвоєння вітамінів?

Нестача жовчних кислот

Ахлоргідрія

Гіперхлоргідрія

Гіпохлоргідрія

Ахілія

1196 / 4299
У хворого на туберкульоз застосовували рифампіцин, що призвело до розвитку стійкості до препарату мікобактерій туберкульозу. З яким препаратом необхідно поєднувати рифампіцин для зменшення розвитку стійкості мікобактерій?

Амоксицилін

Ацикловір

Метронідазол

!нтраконазол

Ьоніазид

1197 / 4299
До стоматолога звернувся хворий з скаргами на біль, почервоніння, припухлість ясен. Запідозрено герпетичний гінгівостоматит. Який вірус міг викликати це захворювання?

Вірус простого герпеса тип 2

Вірус оперізуючого лишаю

Вірус цитомегалії

Вірус Епштейн-Бара

Вірус простого герпеса тип 1

1198 / 4299
Під час розтину тіла жінки 35-ти років поряд із збільшенням багатьох лімфатичних вузлів була виявлена збільшена селезінка вагою 600,0 г; на розрізі неоднорідна, темночервоного кольору, щільна, з ділянками некрозу сірувато-жовтуватого кольору, в діаметрі до 1 см (порфірова селезінка). Про яке захворювання можна думати?

Хронічний мієлоїдний лейкоз

Лімфосаркома

Лімфогранулематоз

Метастази раку

Хронічний лімфоїдний лейкоз

1199 / 4299
Хворому на стоматит лікар призначив полоскання ротової порожнини. Який антисептик з групи окислювачів найбільш придатний для цього?

Спирт етиловий

Калію перманганат

Борна кислота

Розчин йоду спиртовий

Хлорамін

1200 / 4299
При огляді порожнини рота чоловіка 60-ти років виявлені наступні зміни: 26 та 27 вкриті металічними коронками, які глибоко заходять під ясна. Між ними пародонтальна кишеня глибиною 0,7 см з незначною кількістю гною. Ясенні сосочки цих зубів гіпере-мовані, набряклі, з ціанотичним відтінком, при дотику зондом кровоточать. На рентгенограмі - резорбція міжзубних перегородок на 1/2 довжини кореня. Який найбільш вірогідний діагноз?

Хронічний катаральний гінгівіт

Генералізований пародонтит

Гіпертрофічний гінгівіт

Локальний пародонтит

1201 / 4299
У хворої дитини періодично з’являються рідкі випорожнення, іноді біль у ділянці живота, нудота, блювання. За розповіддю матері, одного разу у дитини з блювотною масою виділився гельмінт веретеноподібної форми, розміром 25 см. Яке захворювання можна запідозрити?

Трихоцефальоз

Аскаридоз

Трихінельоз

Ентеробіоз

Дракункульоз

1202 / 4299
У новонародженого хлопчика спостерігається деформація мозкового та лицьового черепа, мікрофтальмія, деформація вушної раковини, вовча паща. Каріотип дитини виявився 47, XY,13+. Про яку хворобу йдеться?

Синдром Патау

Синдром Дауна

Синдром Едвардса

Синдром Шерешевського-Тернера

Синдром Клайнфельтера

1203 / 4299
У 19-річної дівчини клінічно виявлено наступну групу ознак: низький зріст, статевий інфантилізм, відставання у інтелектуальному та статевому розвитку, вада серця. Які найбільш імовірні причини даної патології?

Моносомія за Х-хромосомою

Трисомія по 20-й хромосомі

Трисомія по 13-й хромосомі

Часткова моносомія

Трисомія по 18-й хромосомі

1204 / 4299
У клітині людини в гранулярну ендоплазматичну сітку до рибосом доставлена і-РНК, що містить як екзонні, так і інтронні ділянки. Який процес НЕ ВІДБУВАЄТЬСЯ?

Реплікація

Транскрипція

Пролонгація

Процесінг

Трансляція

1205 / 4299
У клітині людини відбувається транскрипція. Фермент РНК-полімераза, пересуваючись вздовж молекули ДНК, досяг певної послідовності нуклеотидів. Після цього транскрипція припинилась. Ця ділянка дНк має назву:

Термінатор

Репресор

Регулятор

Оператор

Промотор

1206 / 4299
Через два тижні після переливання крові у реципієнта виникла лихоманка. Про яке протозойне захворювання повинен думати лікар?

Трипаносомоз

Лейшманіоз

Амебіаз

Малярія

Токсоплазмоз

1207 / 4299
У хворого інфаркт міокарда в ділянці передньої стінки лівого шлуночка. В басейні якої судини виникло порушення кровообігу?

Передсердно-шлуночкова гілка лівої вінцевої артерії

Огинаюча гілка лівої вінцевої артерії

Ліва крайова гілка лівої вінцевої артерії

Передня міжшлуночкова гілка лівої вінцевої артерії

Передня шлуночкова гілка правої вінцевої артерії

1208 / 4299
У хворого після застудного захворювання з’явилося порушення больової і температурної чутливості передніх 2/3 язика. Який із нервів при цьому постраждав?

Додатковий

Трійчастий

Язикоглотковий

Блукаючий

Під’язиковий

1209 / 4299
Лікар прогнозує можливість запалення клітковини орбіти при наявності гнійного запалення в ділянці підочно-ямкового простору. Через яке з утворень можливе поширення інфекції?

Canalis pterygoideus

Canalis nasolacrimalis

Fissura orbitalis inferior

Canalis infraorbitalis

Canalis incisivus

1210 / 4299
У потерпілого лівобічний уламковий перелом виличної кістки з втратою шкірної чутливості над нею. Який нерв пошкоджений?

Щічний

Мала гусяча лапка

Нижньоочноямковий

Вилично-лицевий

Лицевий

1211 / 4299
Під час дослідження обличчя хворого відзначається, що правий кут рота зміщений у лівий бік, права щока і губи притиснеш до зу6ів і ясен. Порушенням функції якого м’яза це викликане?

Щічний м’яз

М’яз, що опускає кут рота

Коловий м’яз рота

М’яз сміху

Виличні м’язи

1212 / 4299
До лікаря звернувся хворий з запаленням комірок решітчастої кістки (етмоїдит). При обстеженні виявлено порушення кровопостачання кістки. Гілками якої артерії кровопостачаю-ться решітчасті комірки в нормі?

A. infraorbitalis

A. facialis

A. ophthalmica

A. cerebri anterior

A. transversa faciei

1213 / 4299
Хворому чоловіку 60-ти років після операції потрібно вивести сечу катетером. В якому місці сечівника можна зробити прорив при недостатньо вмілій маніпуляції (найбільш звужена у нормі частина)?

Pars membranacea

Fossa navicularis

Ostium uretrae externum

Pars spongiosa

Pars prostatica

1214 / 4299
У чоловіка 52-х років при піднятті надмірної ваги виникло грижове ви-п’ячування в здухвинній ділянці. Через яке анатомічне утворення найімовірніше виникло грижове вип’ячування?

Canalis inguinalis

Anulus femoralis

Linea alba

Anulus umbilicalis

Lig. inguinalis

1215 / 4299
Після падіння з дерева у хлопчика утруднене відведення руки до горизонтального положення. Який із м’язів імовірно пошкоджений?

M.anconeus

M.triceps brachii

M.coracobrachialis

M.deltoideus

M.supinator

1216 / 4299
Жінка звернулась до лікаря зі скаргами на набряклість та болючість нижньої кінцівки, припухлість вен та вузлів на медіальній поверхні стегна. Яка з вен уражена?

Велика підшкірна

Підколінна

Мала підшкірна

Стегнова

Великогомілкова

1217 / 4299
У хворої з пухлиною підшлункової залози розвинулася механічна жовтяниця внаслідок стиснення жовчовивідної протоки. Яка протока зазнала стиснення?

Ductus choledochus

Ductus hepaticus communis

Ductus hepaticus sinister

Ductus hepaticus dexter

Ductus cysticus

1218 / 4299
На обмеженій ділянці епідермісу внаслідок травми відсутні шари аж до росткового. Назвіть основне джерело регенерації епідермісу:

Шари зернистих клітин

Шар базальних клітин

Шари шипуватих і зернистих клітин незруйнованої ділянки

Клітини блискучого шару незруйно-ваної ділянки

Шари шипуватих клітин

1219 / 4299
В червоному кістковому мозку в постембріональному гемопоезі в клітинах одного з диферонів поступово знижується базофілія цитоплазми і підвищується оксифілія, ядро виштовхується. Для якого виду гемопоезу характерні дані морфологічні зміни?

Базофілоцитопоез

Еритропоез

Нейтрофілоцитопоез

Лімфопоез

Еозинофілоцитопоез

1220 / 4299
В препараті діагностується тканина, в якій клітини розміщуються поодинці та ізогрупами, а в міжклітинній речовині не видно волокнистих структур. Яка тканина присутня в препараті?

Волокниста хрящова

Кісткова

Епітеліальна

Гіалінова хрящова

Гладенька м’язова

1221 / 4299
У хворого з тяжкою травмою верхньої кінцівки спостерiгається порушення процесів регенерації хрящової тканини внаслідок пошкодження мало диференційованих клітин хрящового ди-ферону. Які клітини зазнали ушкодження?

Клітини у складі ізогенних груп

Клітини зони молодого хряща

Клітини, що надходять з кровоносних судин

Клітини внутрішнього шару охрястя

Клітини зовнішнього шару охрястя

1222 / 4299
На гістологічному препараті сагітального розрізу закладки нижньої щелепи 3,5-місячного плоду людини спостерігається епітеліальний емалевий орган, оточений компактно розташованими мезенхімними клітинами. Як називається це мезенхімне утворення?

Зубний сосочок

Внутрішні емалеві клітини

Зовнішні емалеві клітини

Зубний мішечок

Пульпа емалевого органа

1223 / 4299
Під час розвитку облітеруючого атеросклерозу у хворих виникають зміни в судинах нижніх кінцівок. На гістологічному препараті такої судини добре виражені внутрішня та зовнішня еластичні мембрани, у середній оболонці багато міоцитів. Яка судина ушкоджується при цьому захворюванні?

Артерія еластичного типу

Артерія м’язового типу

Артерія змішаного типу

Лімфатична судина

Вена з сильним розвитком м’язів

1224 / 4299
У першому критичному періоді в матковій трубі з невідомої причини в зародку відбулося розчинення оболонки запліднення. Яке ускладнення вагітності можливе в цьому випадку?

Повернення бластоцисти назад в ампулярну зону труби

Імплантація зародка в стінці труби

Утворення двох бластоцист

Інвагінація стінки бластоцисти

Загибель зародка

1225 / 4299
У жінки 30-ти років зменшений вміст ферментів у підшлунковому соці. Недостатня секреція якого гастроінте-стинального гормону може бути причиною цього?

Холецистокінін-панкреозимін

Шлунково-інгібуючий пептид

Секретин

Вазо-інтестинальний пептид

Соматостатин

1226 / 4299
Відомо, що у людей, які постійно мешкають в умовах високогір’я, збільшується вміст еритроцитів в одиниці об’єму крові. Це сприяє оптимальному виконанню кров’ю такої функції:

Підтримка іонної рівноваги

Підтримка кислотно-лужної рівноваги

Транспортування газів

Транспортування амінокислот

Участь у гемостазі

1227 / 4299
Пацієнт скаржиться на постійні кровотечі з ясен, які спостерігаються з дитинства. Аналіз крові виявив дефіцит VIII плазменного фактору зсідання крові. Це означає, що у пацієнта, перш за все, порушено:

Адгезія тромбоцитів

Утворення фібрину

Утворення протромбінази

Утворення тромбіну

Агрегація тромбоцитів

1228 / 4299
У стоматологічній практиці широко використовується місцеве знеболювання, коли до розчину новокаїну додають 0,1% розчин адреналіну. Адреналін, що додається, викликає:

Місцеве розширення судин

Зниження опору судин

Зниження артеріального тиску

Місцеве звуження судин

Підвищення артеріального тиску

1229 / 4299
В гострому досліді тварині в порожнину 12-ти палої кишки ввели слабкий розчин хлористоводневої кислоти. До збільшення секреції якого гастроін-тестинального гормону це призведе?

Гістамін

Секретин

Нейротензин

Мотилін

Гастрин

1230 / 4299
У хворого у результаті розладу моз- кового кровообігу порушений акт ковтання, він може поперхнутися під час прийому рідкої їжі. Вкажіть, який відділ мозку уражений?

Довгастий мозок

Шийний відділ спинного мозку

Проміжний мозок

Середній мозок

Мозочок

1231 / 4299
У хворого 70-ти років діагностовано крововилив у стовбур мозку. Обстеження виявило підвищення тонусу м’язів-згиначів на тлі зниження тонусу м’язів-розгиначів. Подразненням яких структур мозку можна пояснити зміни тонусу м’язів?

Чорна речовина

Чотиригорбикова структура

Вестибулярні ядра

Ретикулярна формація

Червоні ядра

1232 / 4299
У дитини наявне порушення формування емалі та дентину зубів через знижений вміст іонів кальцію в крові. Дефіцит якого гормону може спричинити такі порушення?

Соматотропний гормон

Тироксин

Паратгормон

Трийодтиронін

Тирокальцитонін

1233 / 4299
Людина вживає суху їжу. Які слинні залози при цьому секретують найбільше слини?

Піднебінні

Щічні

Під’язикові

Підщелепні

Привушні

1234 / 4299
У чоловіка 30-ти років методом непрямої калориметрії встановлено зменшення основного обміну на 30%. Зниження концентрації яких гормонів у плазмі крові може бути причиною цього?

Соматоліберин, соматостатин

Тиреокальцитонін, паратгормон

Катехоламіни

Трийодтиронін, тетрайодтиронін

Ілюкокортикоїди

1235 / 4299
При визначенні енерговитрат організму людини встановлено, що дихальний коефіцієнт дорівнює 1,0. Це озна- чає, що у клітинах досліджуваного переважно окислюються:

Білки

Білки та вуглеводи

Вуглеводи та жири

Жири

Вуглеводи

1236 / 4299
У жінки 32-х років запалення ясен (гінгівіт) супроводжується їх гіпоксією. Утворення якого метаболіту вуглеводного обміну значно збільшується при цьому в тканинах пародонта?

НАДФ-Н

Рибозо-5-фосфат

Ілікоген

Лактат

Ілюкозо-6-фосфат

1237 / 4299
Людина в стані спокою штучно примушує себе дихати часто і глибоко впродовж 3-4 хвилин. Як це відбивається на кислотно-лужній рівновазі організму?

Кислотно-лужна рівновага не змінюється

Виникає дихальний ацидоз

Виникає дихальний алкалоз

Виникає метаболічний алкалоз

Виникає метаболічний ацидоз

1238 / 4299
У 8-місячної дитини спостерігаються блювання та діарея після прийому фруктових соків. Навантаження фруктозою призвело до гіпоглікемії. Спадкова недостатність якого ферменту є причиною цих порушень?

Фруктокіназа

Фруктозо-1-фосфатальдолаза

Фруктозо-1,6-дифосфатаза

Іексокіназа

Фосфофруктокіназа

1239 / 4299
Використанню глюкози клітинами передує її транспорт з екстрацелюляр-ного простору через плазматичну мембрану всередину клітини. Яким гормоном стимулюється цей процес?

Інсулін

Тироксин

Адреналін

Альдостерон

Глюкагон

1240 / 4299
При лікуванні пародонтиту використовують препарати кальцію та гормон, що має здатність стимулювати мінералізацію зубів та гальмувати ре- зорбцію кісткової тканини, а саме:

Паратгормон

Тироксин

Альдостерон

Кальцитонін

Адреналін

1241 / 4299
У жінки 35-ти років із хронічним захворюванням нирок розвинувся осте-опороз. Дефіцит якої з перерахованих нижче речовин є основною причиною цього ускладнення?

Холестерин

25-гідрокси-Д3

1,25-дигідрокси-Дз

D2

D3

1242 / 4299
У хлопчика 4-х років після перенесеного важкого вірусного гепатиту спостерігаються блювання, епізоди непритомності, судоми. У крові - гіперамо-ніємія. Порушення якого біохімічного процесу в печінці викликало такий стан хворого?

Активація декарбоксилювання амінокислот

Порушення знешкодження біогенних амінів

Порушення знешкодження аміаку

Пригнічення синтезу білків

Пригнічення ферментів трансаміну-вання

1243 / 4299
У хворого на хронічний гепатит видалили зуб. Кровотеча, що виникла після цього, не припинялася впродовж 2 годин. Проведене дослідження системи гемостазу встановило зменшення вмісту в плазмі крові декількох факторів зсідання. Порушення якої функції печінки призвело до порушення гемостазу у цього хворого?

Антитоксична

Білоксинтезуюча

Травна

Захисна

Гормональна

1244 / 4299
У хворого, прооперованого з приводу ускладненого апендициту, в крові: ер.- 4,0 • 1012/л, НЬ- 120 г/л, кП -0,9, лейк.- 18 • 109/л, б.- 0, е.- 0, ю.- 0, п/я.- 20%, с/я.- 53%, л.- 21%, м.- 6%. Як називається такий зсув лейкоцитарної формули?

Регенеративний зсув вліво

Регенеративно-дегенеративний

Гіперрегенеративний

Дегенеративний зсув вліво

Зсув вправо

1245 / 4299
Хворого доставили до клініки у коматозному стані. В анамнезі цукровий діабет II типу впродовж 5-ти років. Об’єктивно: дихання шумне, глибоке, у видихуваному повітрі чути запах ацетону. Вміст глюкози у крові 15,2 ммоль/л, кетонових тіл - 100 мкмоль/л. Для якого ускладнення цукрового діабету характерні такі клінічні прояви?

Кетоацидотична кома

Гіперосмолярна кома

Гіпоглікемічна кома

Печінкова кома

Гіперглікемічна кома

1246 / 4299
В осередку запалення у судинах мікроциркуляторного русла відзначається підвищення їх проникливості, збільшення гідродинамічного тиску. У міжтканинній рідині має місце підвищення осмотичної концентрації і дисперсності білкових структур. Який вид набряку буде спостерігатися в даному випадку?

Лімфогенний

Мембраногенний

Колоїдно-осмотичний

Змішаний

Гідродинамічний

1247 / 4299
У хворого діагностовано хронічний гломерулонефрит. Внаслідок значних склеротичних змін маса функціонуючих нефронів зменшилася до 10%. Яке з перерахованих нижче порушень лежить в основі розвитку в хворого уремічного синдрому?

Порушення водного гомеостазу

Порушення осмотичного гомеостазу

Ниркова остеодистрофія

Азотемія

Артеріальна гіпертензія

1248 / 4299
Після травматичного видалення зуба хворий скаржиться на сильний, тупий, без чіткої локалізації біль у ясні, підвищення температури тіла до 37,5oC. Діагностований альвеоліт. Який вид болю в цього хворого?

Протопатичний

Відбитий

Епікритичний

Фантомний

Вісцеральний

1249 / 4299
У підлітка було видалено зуб із застосуванням новокаїну. Через 10 хвилин у нього з’явилися 6лідість шкірних покривів, задишка, гіпотензія. При розвитку цієї реакції алерген на тканинних базофілах реагує з:

IgE

IgM

IgA

IgD

Т-лімфоцитами

1250 / 4299
Хворий 50-ти років страждає на гіпертонічну хворобу. Під час фізичного навантаження у нього з’явились відчуття м’язової слабкості, нестачі повітря, синюшність слизової оболонки губ, шкіри, обличчя. Дихання супроводжувалось відчутними на відстані вологими хрипами. Який механізм лежить в основі виникнення такого синдрому?

Тампонада серця

Хронічна правошлуночкова недостатність

Гостра лівошлуночкова недостатність

Колапс

Хронічна лівошлуночкова недостатність

1251 / 4299
Чоловік 50-ти років хворіє на хронічний бронхіт, скаржиться на задишку при фізичному навантаженні, постійний кашель з відходженням харкотиння. При обстеженні діагностовано ускладнення - емфізема легень. Чим вона обумовлена?

Зменшенням альвеолярної вентиляції

Зниженням еластичних властивостей легень

Порушенням вентиляційно-перфузійного співвідношення в легенях

Зменшенням перфузії легень

Зменшенням розтяжності легень

1252 / 4299
У хворого хронічне генералізова-не ушкодження зубів з формуванням зубоясневих кишень. За результатами рентгенологічного обстеження щелепних кісток виявлено, що краї зубних ямок знаходяться на рівні 2/3 довжини кореня зуба. Який з перелічених діагнозів найбільш імовірний?

Пародонтит 4 ступеня резорбції

Пародонтит 1 ступеня резорбції

Пародонтит 2 ступеня резорбції

Пародонтит 3 ступеня резорбції

Пародонтоз

1253 / 4299
При якій інфекції у перший період її розвитку відзначається різке повнокров’я судин зіву, мигдаликів, слизової оболонки порожнини рота, язика ('малиновий язик')?

Скарлатина

Коклюш

Менінгококова інфекція

Кір

Дифтерія

1254 / 4299
У чоловіка 30-ти років на нижній щелепі зліва в ділянці моляра визначається пухлиноподібне утворення щільної консистенції, дрібногорбисте, що значно деформує щелепу. Утворення місцями нечітко відшароване від кісткової тканини. Мікроскопічно: у стромі біоптату виявляються тяжі, фолікули, на периферії яких - клітини одонтоген-ного циліндричного епітелію, у центрі -зірчасті клітини, які нагадують пульпу емалевого органа. Який найбільш імовірний діагноз?

Аденокарцинома

Остеобластокластома

Аденоматоїдна пухлина

Амелобластома

Первинний внутрішньокістковий рак

1255 / 4299
У хворого 50-ти років протягом багатьох років була ревматична вада серця. Під час загострення захворювання розвинулась геміплегія і настала смерть. Гістологічно в мітральному клапані виявлено виражений склероз, вогнищеві клітинні інфільтрати, дрібні тромботичні нашарування. Для якої форми ендокардиту характерні виявлені зміни?

Гострий бородавчастий

Виразково-поліпозний

Фібропластичний

Поворотно-бородавчастий

Дифузний

1256 / 4299
При розтині тіла чоловіка, померлого від внутрішньокишкової кровотечі, в клубовій кишці спостерігається некроз групових і солітарних фолікулів, імбібіція мертвих тканин жовчю і кров’ю; в нижньому відрізку кишки -явища секвестрації та відторгнення некротичних мас з утворенням дефектів. Який з перелічених діагнозів найбільш вірогідний?

Черевнотифозна форма сальмонельозу

Хвороба Крона

Черевний тиф, стадія 'бру- дних'виразок

Черевний тиф, стадія некрозу

Черевний тиф, стадія 'чи- стих'виразок

1257 / 4299
На розтині тіла чоловіка, який помер на 5-у добу захворювання черевним тифом, виявлені наступні зміни: групові фолікули клубової кишки збільшені, повнокровні і виступають над слизовою оболонкою, на їх поверхні видно борозни та звивини. Гістологічно: повнокров’я і набряк тканини, наявність гранульом, які складаються з великих клітин зі світлою цитоплазмою і містять черевнотифозні палички. Про який період місцевих змін при черевному тифі можна думати?

Стадія загоєння виразок

Стадія мозкоподібного набухання

Стадія утворення виразок

Стадія некрозу

Стадія 'чистих'виразок

1258 / 4299
Бригадою швидкої медичної допомоги до клініки доставлена хвора 46-ти років зі скаргами на луночкову кровотечу впродовж 6-ти годин після видалення зуба, загальну слабкість, запаморочення. В анамнезі - гіпертонічна хвороба. Об’єктивно: блідість шкіри та видимих слизових оболонок. У крові: Hb-80 г/л, Ht- 30%, показники часу кровотечі та зсідання крові знаходяться в межах норми. Яке ускладнення розвинулося в хворої в зв’язку з кровотечею?

Хронічна постгеморагічна анемія

Фолієводефіцитна анемія

Гостра постгеморагічна анемія

Гемолітична анемія

Залізодефіцитна анемія

1259 / 4299
У чоловіка 46-ти років на шкірі визначалась пляма темного кольору, яка вибухала та не спричиняла турбот. З часом пляма почала збільшуватись, з’явився біль, колір став чорнокоричневим; почав пальпуватися вузлик. На гістологічному дослідженні видаленої тканини визначалися веретеноподібні та поліморфні клітини, з численними мітозами, цитоплазма яких вміщувала пігмент бурого кольору. Про яку пухлину йдеться?

Гемангіома

Меланома

Базаліома

Невус

1260 / 4299
Чоловік, який багато років страждав на бронхіальну астму, помер від ядухи. При гістологічному дослідженні легень виявлено: в просвітах бронхіол та дрібних бронхів багато слизу з домішкою еозинофілів, склероз міжальвео-лярних перегородок, розширення просвітів альвеол. Який з механізмів розвитку реакції гіперчутливості має місце?

Імунокомплексна реакція

Гранулематоз

Цитотоксична реакція

Цитоліз, обумовлений лімфоцитами

Реагінова реакція

1261 / 4299
Під час оформлення дитини до школи для вирішення питання про необхідність ревакцинації поставлена проба Манту, яка виявилася негативною. Про що свідчить даний результат проби?

Наявність антитіл до туберкульозних бактерій

Наявність клітинного імунітету до туберкульозу

Відсутність антитоксичного імунітету до туберкульозу

Відсутність клітинного імунітету до туберкульозу

Відсутність антитіл до туберкульозних бактерій

1262 / 4299
Вірус імунодефіциту людини, маючи на своїй поверхні антигени gp41 та gp120, взаємодіє з клітинами-мішенями організму. Виберіть серед перерахованих антигени лімфоцитів людини, з якими комплементарно зв’язується gp120 вірусу:

CD 3

CD 8

CD 28

CD 19

CD 4

1263 / 4299
Пацієнтка 65-ти років з хронічною серцевою недостатністю тривалий час лікується дигітоксином, самостійно визначаючи дозу. Надійшла до стаціонару із скаргами на погіршення загального стану, появу аритмії, нудоту, зменшення виділення сечі, безсоння. З чого слід розпочати лікування хворої?

Зменшити дозу дигітоксину

Відмінити дигітоксин

Призначити дигоксин

Ввести розчин кальцію глюконату внутрішньовенно

Призначити строфантин внутрішньовенно

1264 / 4299
Хворий надійшов до хірургічного відділення з діагнозом гострий панкреатит. Розпочато консервативне лікування. Призначення якого препарату є патогенетично обґрунтованим?

Трипсин

Контрикал

Хімотрипсин

Фібринолізин

Панкреатин

1265 / 4299
Під час огляду школярів першого класу стоматолог виявив, що у дитини зуби жовтокоричневого кольору, два з них надщерблені. Зі слів дитини, до цього його лікували від пневмонії 'якимись таблетками'. Який препарат міг так негативно вплинути на зуби?

Ампіцилін

Бісептол

Еритроміцин

Оксацилін

Доксициклін

1266 / 4299
До стоматолога звернувся пацієнт з артритом скронево-нижньощелепного суглоба. Лікарстоматолог призначив мазь із диклофенак-натрієм. Який механізм дії цього лікарського засобу?

Активація опіатних рецепторів

Пригнічення фосфоліпази

Активація циклооксигенази

Блокада опіатних рецепторів

Пригнічення циклооксигенази

1267 / 4299
Під час дослідження мазку крові, взятого у хворого і забарвленого за Ро-мановським, лікар виявив найпростіші і встановив діагноз - хвороба Круза-Чагаса. Яке найпростіше викликало захворювання у даного хворого?

Leishmania donovani

Trypanosoma brucei

Trypanosoma cruzi

Leishmania tropica

Toxoplasma gondii

1268 / 4299
До приймального відділення надійшов хворий у непритомному стані. Об’єктивно: шкіра холодна, зіниці звужені, дихання утруднене, відзначається періодичність дихання за типом Чейн-Стокса, артеріальний тиск знижений, сечовий міхур переповнений. Чим відбулося отруєння?

Наркотичними анальгетиками

Транквілізаторами

М-холіноблокаторами

Ненаркотичними анальгетиками

1269 / 4299
На електронній мікрофотографії видно клітину відростчастої форми, що містить у глибоких інвагінаціях пла-змолеми лімфоцити, що диференціюються. Для якого органа характерна така ультраструктура?

Селезінка

Печінка

Тимус

Червоний кістковий мозок

Мигдалик

1270 / 4299
При підозрі на туберкульоз хворій дитині зробили пробу Манту. Через 24 години у місці введення алергену з’явилися припухлість, гіперемія і болісність. Які основні компоненти визначають розвиток цієї реакції організму?

Мононуклеари, Т-лімфоцити і лім-фокіни

Гранулоцити, Т-лімфоцити і IgG

В-лімфоцити, IgM

Плазматичні клітини, Т-лімфоцити і лімфокіни

Макрофаги, В-лімфоцити і моноцити

1271 / 4299
У собаки через 1-2 доби після видалення прищитоподібних залоз спостерігались: млявість, спрага, різке підвищення нервово-м’язової збудливості з розвитком тетанії. Яке порушення обміну електролітів має місце при цьому?

Гіпомагніємія

Гіперкальціємія

Гіпокальціємія

Гіпермагніємія

Гіпонатріємія

1272 / 4299
В селищі у декількох господарствах було виявлено масову загибель щурів. Виникла підозра, що причиною може бути чума. Які постмортальні дослідження тварин слід провести з метою екстреного встановлення збудника інфекції?

Реакція аглютинації

Реакція нейтралізації

Реакція пасивної аглютинації

Реакція кільцепреципітації

Реакція зв’язування комплементу

1273 / 4299
Жінці 58-ми років проведено повне видалення матки з придатками, після чого виділення сечі припинилося. При цистоскопії: міхур сечі не містить, з устій сечоводів сеча не поступає. Яку структуру сечовидільної системи було ушкоджено під час операції?

Vesica urinaria

Uretra

Ureter

Pelvis renalis

Ren

1274 / 4299
У дітей часто можна спостерігати утруднене носове дихання, яке пов’язане з надмірним розвитком лімфої-дної тканини слизової оболонки глотки. Розростання яких мигдаликів може спричинити це явище?

Tonsilla palatina

Tonsilla tubaria

Tonsilla lingualis

Усіх названих мигдаликів

Tonsilla pharyngea

1275 / 4299
При рентгенографії виявлено руйнування і збільшення ямки турецького сідла внаслідок пухлини гіпофіза. Яка кісткова порожнина при цьому вражена?

Зоровий канал

Лицевий канал

Барабанна порожнина

Сонний канал

Пазуха клиноподібної кістки

1276 / 4299
У хворого 26-ти років виявлений великий фурункул м’яких тканин обличчя біля кореня носа та нижньої повіки. Грізним ускладненням цього захворювання може бути розповсюдження інфекції венозними сполученнями цього регіону до пазух твердої мозкової оболонки. Яка з пазух найбільш імовірно може бути уражена?

Потилична

Верхня сагітальна

Верхня кам’яниста

Печериста

Сигмоподібна

1277 / 4299
Хворій 65-ти років, що страждає на інсулінонезалежний цукровий діа- бет, призначили всередину глібенкла-мід. Вкажіть механізм гіпоглікемічної дії цього препарату:

Пригнічує всмоктування глюкози у кишківнику

Пригнічує глюконеогенез у печінці

Посилює утилізацію глюкози периферичними тканинами

Стимулює виділення ендогенного інсуліну в-клітинами

Пригнічує а-глюкозидазу і розпад полісахаридів

1278 / 4299
На 1% лужній пептонній воді після посіву в неї досліджуваного матеріалу (випорожнень) і 8-годинної інкубації у термостаті при температурі 37°C виявлений ріст у вигляді ніжної блакитної плівки. Для збудника якого захворювання характерні такі культуральні властивості?

Паратиф А

Черевний тиф

Холера

Чума

Дизентерія

1279 / 4299
У пацієнта з хронічною серцевою недостатністю виявлене збільшення в’язкості крові. При капіляроскопії виявлено пошкодження стінок судин мі-кроциркуляторного русла. Яке з перелічених порушень можливе у даному випадку?

Емболія

Сладж-феномен

Артеріальна гіперемія

Венозна гіперемія

Тромбоз

1280 / 4299
У хворого на цукровий діабет розвинулася діабетична кома внаслідок порушення кислотно-основного стану. Який вид порушення кислотно-основного стану виник при цьому?

Метаболічний алкалоз

Газовий алкалоз

Респіраторний ацидоз

Негазовий алкалоз

Метаболічний ацидоз

1281 / 4299
Під час барбітурового наркозу у хворого 65-ти років почало розвиватися пригнічення дихання. Анестезіолог зробив внутрішньовенну ін’єкцію 10 мл 0,5% розчину бемегріду. Стан хворого покращився, об’єм легеневої вентиляції збільшився. Яке явище полягає в основі взаємодії цих препаратів?

Синергізм непрямий

Антагонізм однобічний

Антагонізм прямий

Антагонізм непрямий

Синергізм прямий

1282 / 4299
У лабораторії при експертизі шкур тварин була використана реакція преципітації за Асколі. При обліку результатів через декілька хвилин після поєднання імунної сироватки та екстракту шкури було відмічено утворення білуватого кільця. Про що свідчить даний результат?

Наявність антигенів сибірки

Наявність токсину анаеробної інфекції

Наявність антигену вірулентного сальмонел

Наявність поверхневого антигену ешеріхій

Наявність збудника бруцельозу

1283 / 4299
Клітину обробили речовиною, яка блокує фосфорилювання нуклеотидів у мітохондріях. Який процес життєдіяльності клітини буде порушений в першу чергу?

Окислювальне фосфорилювання

!нтеграція функціональних білкових молекул

Ресинтез АТФ

Фрагментація великих мітохондрій на менші

Синтез мітохондріальних білків

1284 / 4299
Хворому 60-ти років важко формувати і рухати харчову грудку, що порушує процес їжі. При цьому язик нерухомий, мова стає неможливою. Причиною цього може бутити порушення функції такого нерва:

Трійчастий

Додатковий

Язикоглотковий

Лицевий

Під’язиковий

1285 / 4299
В лабораторію направлено матеріал білуватого нашарування із слизової оболонки ротової порожнини. Висів матеріалу зроблено на середовище Са-буро, відмічено ріст сметаноподібних колоній, бактеріоскопія виявила короткі бруньковані нитки. До збудників якої інфекції відносять ізольовані мікроорганізми?

Рікетсіоз

Хламідіоз

Мікоплазмоз

Мікоз

Спірохетоз

1286 / 4299
У дитини відзначається затримка прорізування зубів, неправильне їх розташування. Скарги також на сухість у роті, появу тріщин в кутах рота з наступним нагноєнням. З нестачею якого вітаміну може бути пов’язаний цей стан?

A

K

E

C

D

1287 / 4299
У хворого з гломерулонефритом виявлено: анасарка, АТ- 185/105 мм рт.ст., анемія, лейкоцитоз, гіперазотемія, гі-попротеїнемія. Який показник свідчить про ускладнення гломерулонефриту нефротичним синдромом?

Гіперазотемія

Анемія

Гіпопротеїнемія

Лейкоцитоз

Артеріальна гіпертензія

1288 / 4299
На гістологічному препараті представлений орган нервової системи зірчастої форми, зовні вкритий сполучнотканинною капсулою, містить мульти-полярні нейрони, різноманітні за функцією. Який це орган?

Довгастий мозок

Мозочок

Вегетативний ганглій

Спинний мозок

Спинномозковий ганглій

1289 / 4299
Депресії, емоційні розлади є наслідком нестачі у головному мозку нора-дреналіну, серотоніну та інших біогенних амінів. Збільшення їх вмісту у си-напсах можна досягти за рахунок антидепресантів, які гальмують активність такого ферменту:

Моноамінооксидаза

Оксидаза D-амінокислот

Оксидаза L-амінокислот

Диамінооксидаза

Фенілаланін-4-монооксигеназа

1290 / 4299
Хворий 60-ти років звернувся до лікаря зі скаргами на загруднинний біль після фізичного навантаження. Лікар призначив нітрогліцерин. Після засто- сування препарату загруднинний біль зник, проте з’явився сильний біль голови. Який можливий механізм цієї побічної дії?

Зменшення накопичення іонів кальцію

Блокада а-адренорецепторів

Блокада фосфодиестерази

Підвищення внутрішньочерепного тиску

Гальмування утворення медіаторів у мозку

1291 / 4299
Розпад глікогену в печінці стимулюється адреналіном. Який вторинний ме-сенджер (посередник) утворюється при цьому в гепатоцитах?

ц-АМФ

ц-ГМФ

NO

СО

Триацилгліцерол

1292 / 4299
Хворий ходить хитаючись, широко розставляючи ноги. У нього знижений тонус м’язів рук і ніг, скандована мова. У якій структурі головного мозку локалізується ураження?

Червоне ядро

Хвостате ядро

Мозочок

Шкаралупа

Моторна кора

1293 / 4299
У хворого 65-ти років синдром паркінсонізму. Який засіб слід йому призначити?

Скополамін

Амізил

Леводопа

Атропіну сульфат

1294 / 4299
На підставі фенотипового аналізу у жінки встановлено попередній діагноз: полісомія Ххромосом. Для уточнення діагнозу використовується цитогенети-чний метод. Діагноз буде підтверджено, якщо каріотип буде:

48(XXYY)

48(XXXY)

47(ХХХ)

46(XX)

47(XXY)

1295 / 4299
У хворого, який знаходиться на лікуванні з приводу вірусного гепатиту В, з’явилися ознаки печінкової недо- статності. Які зміни крові, що свідчать про порушення білкового обміну, найбільш імовірно будуть спостерігатися у даному випадку?

Абсолютна гіперфібриногенемія

Абсолютна гіпоальбумінемія

Абсолютна гіперглобулінемія

Білковий склад крові не змінений

Абсолютна гіперальбумінемія

1296 / 4299
Під час травми грудної клітки пошкоджена хрящова частина ребра. За рахунок якого шару охрястя відбудеться регенерація хряща?

Фіброзний

Волокна Шарпея

Колагеновий

Хондрогенний

Еластичний

1297 / 4299
Для людини існує суворе обмеження в часі перебування на висоті понад 8000 метрів над рівнем моря без кисневих балонів. Що є лімітуючим фактором для життя в даному випадку?

Сила земного тяжіння

Рівень ультрафіолетового опромінення

Парціальний тиск кисню в повітрі

Рівень вологості

Температура

1298 / 4299
Після прийому блокатору мембранних циторецепторів хворий скаржиться на відчуття сухості в роті. Блокатор яких циторецепторів приймав пацієнт?

М-холінорецептори

а- та в-адренорецептори

Н-холінорецептори

в-адренорецептори

а-адренорецептори

1299 / 4299
При дослідженні культури тканин злоякісної пухлини виявили поділ клітин, який відбувався без ахроматиново-го апарату шляхом утворення перетяжки ядра, при якому зберігались ядерна оболонка та ядерце. Який тип поділу клітин відбувався у злоякісній пухлині, що вивчалась?

Ендомітоз

Мітоз

Мейоз

Екзомітоз

Амітоз

1300 / 4299
У людини під впливом іонізуючого випромінювання в крові зменшилася кількість гранулоцитів. Який механізм лежить в основі цих змін?

Розвиток аутоімунного процесу

Підвищене руйнування лейкоцитів

Порушення виходу зрілих лейкоцитів з кісткового мозку

Збільшений перехід гранулоцитів в тканини

Пригнічення лейкопоезу

1301 / 4299
У 19-ти місячної дитини із затримкою розвитку та проявами самоагре-сії вміст сечової кислоти в крові - 1,96 ммоль/л. При якому метаболічному порушенні це спостерігається?

Подагра

Хвороба Гірке

Хвороба Іценко-Кушінга

Синдром набутого імунодефіциту

Синдром Леша-Ніхана

1302 / 4299
Курареподібні речовини (дитилін) роблять неможливим скорочення скелетних м’язів, оскільки вони блокують:

Проведення збудження мембраною

Центральні синапси

Пропріорецептори

Нервово-м’язові синапси

Гангліонарні синапси

1303 / 4299
Під час гістологічного дослідження нирок жінки 25-ти років, що померла під час пологів, в епітелії канальців головних відділів нефрону виявлені конденсація хроматину ядер, розпад їх на глибки та лізис, а також плазморексис та цитоліз. Який патологічний процес виявлений в епітелії канальців нирок?

Гіаліноз

Жирова дистрофія

Некроз

Гідропічна дистрофія

Амілоїдоз

1304 / 4299
Під час бійки у чоловіка виникла рефлекторна зупинка серця внаслідок сильного удару у верхню ділянку передньої черевної стінки. На яку структуру серця впливають еферентні нерви, що зумовили його зупинку?

Провідна система шлуночків серця

Робочий міокард передсердь

Робочий міокард шлуночків

Синоатріальний вузол

Атріовентрикулярний вузол

1305 / 4299
Хворому на ішемічну хворобу серця з передсердною екстрасистолією призначено препарат, який переважно блокує калієві канали, знижує адренергічні впливи на серце, суттєво подовжує потенціал дії, розширює коронарні судини. Який препарат призначено?

Корглікон

Аміодарон

Нітрогліцерин

Добутамін

Лізиноприл

1306 / 4299
Клітина піддалась впливу іонізуючого випромінювання при дефіциті вітаміну E. Це сприяло посиленому виходу гідролітичних ферментів у цитоплазму і призвело до повного руйнування внутрішньоклітинних структур - ауто-лізу. Які органели клітини призвели до цього явища?

Комплекс Гольджі

Лізосоми

Мітохондрії

Мікротільця

Ендоплазматична сітка

1307 / 4299
Батьки дитини 3-х років звернули увагу на те, що при відстоюванні сеча дитини темнішає. Об’єктивно: температура тіла дитини в нормі, шкірні покриви рожеві, чисті, печінка не збільшена. Назвіть найбільш імовірну причину даного явища:

Гемоліз

Фенілкетонурія

Синдром Іценко-Кушінга

Подагра

Алкаптонурія

1308 / 4299
У хворого, що харчувався винятково полірованим рисом, недостатність тіаміну стала причиною поліневриту. Екскреція якої сполуки з сечею може бути індикатором цього авітамінозу?

Малат

Сечова кислота

Метил-малонова кислота

Фенілпіруват

Піровиноградна кислота

1309 / 4299
Під час голодування м’язові білки розпадаються до вільних амінокислот. В якому процесі найбільш імовірно будуть використовуватися амінокислоти за таких умовах?

Декарбоксилювання

Глюконеогенез у печінці

Глікогеноліз

Синтез вищих жирних кислот

Глюконеогенез у м’язах

1310 / 4299
До наркологічного відділєння го-спiталiзовано хворого з діагнозом -морфінізм. Лікар відзначив зниження фармакологічної активності морфіну. Як називається явище, коли зменшується ефективність препарату при повторному введенні?

Антагонізм

Звикання

Кумуляція

Тахіфілаксія

Алергія

1311 / 4299
40-річному чоловікові, який хворіє на туберкульоз легень призначено ізоніазид. Нестача якого вітаміну може розвинутися внаслідок тривалого вживання даного препарату?

Біотин

Піридоксин

Тіамін

Кобаламін

Фолієва кислота

1312 / 4299
У хворої дитини виявили у сечі підвищений рівень фенілпірувату (в нормі практично відсутній). Вміст фенілаланіну в крові становить 350 мг/л (норма близько 15 мг/л). Для якого захворювання характерні перелічені симптоми?

Подагра

Альбінізм

Тирозиноз

Фенілкетонурія

Алкаптонурія

1313 / 4299
Хворий на стенокардію приймає ацетилсаліцилову кислоту в дозі 100 мг на добу. З якою метою застосовується ацетилсаліцилова кислота у даного хворого?

Для пригнічення зсідання крові

Для пригнічення агрегації тромбоцитів

Для зниження вмісту холестерину

Для розширення коронарних судин

Для зниження рівня протромбіну

1314 / 4299
У 2-річної дитини з нирковою недостатністю виявили гіпероксалурію, оксалатний уролітіаз, що призвело до відкладання оксалату кальцію в нирках. Порушення обміну якої аміноки- слоти призвело до такого стану?

Аргінін

Гістидин

Гліцин

Лізин

Метіонін

1315 / 4299
Потенціал спокою клітини дорівнює -80 мВ. Під час якої фази ПД величина мембранного потенціалу складатиме +30 мВ?

Слідова гіперполяризація

Реверсполяризація

Слідова деполяризація

Деполяризація

1316 / 4299
Після тривалого фізичного навантаження під час заняття з фізичної культури у студентів розвинулась м’язова крепатура. Причиною її виникнення стало накопичення у скелетних м’язах молочної кислоти. Вона утворилась після активації в організмі студентів:

Ілікогенезу

Гліколізу

Пентозофосфатного циклу

Ілюконеогенезу

Ліполізу

1317 / 4299
При дегельмінтизації у хворого виявлені довгі фрагменти гельмінта, що має членисту будову. Зрілі членики прямокутної форми 30х12 мм, матка закритого типу у вигляді стовбура, від якого відходять 17-35 бічних відгалужень. Визначте вид гельмінта:

Ціп ’як озброєний

Альвеокок

Ціп ’як карликовий

Ціп’як неозброєний

Ехінокок

1318 / 4299
У пацієнта виник спазм гладеньких м’язів бронхів. Застосування антагоністів яких рецепторів буде фізіологічно доцільним для надання невідкладної допомоги?

в-адренорецептори

Аденозинові рецептори

Н-холінорецептори

а-адренорецептори

М-холінорецептори

1319 / 4299
У 42-річного чоловіка діагностований інфаркт міокарда. Неефективність попередніх заходів дала підставу лікарю провести нейролептаналгезію. Який нейролептик необхідно застосувати?

Іалоперидол

Аміназин

Прометазин

Дроперидол

Клозепін

1320 / 4299
Хворий 37-ми років за останні три місяці схуд на 5 кг, скаржиться на тремор рук, підвищене потовиділення, екзофтальм, тахікардію. Збільшення секреції якого гормону може бути причиною цього?

Тиреокальцитонін

Глюкагон

Кортизол

Тироксин

Інсулін

1321 / 4299
За проханням лікаря хворий після звичайного вдиху зробив максимально глибокий видих. Які м’язи з наведених нижче беруть участь у такому видоху?

Грудні

Трапецієподібні

Живота

Діафрагма

Зовнішні міжреберні

1322 / 4299
У людини має місце травматичне пошкодження скроневої зони кори великих півкуль. Це призведе до порушення формування таких відчуттів:

Температурні

Зорові

Смакові

Дотикові

Слухові

1323 / 4299
Пацієнт скаржиться на свербіж шкіри, особливо між пальцями рук, у пахвинних западинах, на нижній частині живота. При огляді в цих ділянках шкіри виявлено маленькі пухирці. Під час лабораторної діагностики встановлено, що причиною цього стану є представник членистоногих. Вкажіть назву хвороби, спричинену цим членистоногим:

Міаз

Короста

Дерматотропний лейшманіоз

Демодекоз

Педикульоз

1324 / 4299
Сімейний лікар діагностував ішемічну хворобу серця і призначив хворому нітрат тривалої дії, який повністю абсорбується в травному каналі, застосовується для профілактики нападів стенокардії. Визначте цей препарат:

Метопролол

Амлодипін

Ніфедипін

Ізосорбіду мононітрат

Валідол

1325 / 4299
У людини цистинурія проявляється у вигляді наявності цистинових камінців у нирках (гомозиготи) або підвищеним рівнем цистину в сечі (гетерозиготи). Захворювання цистинурією є моногенним. Визначити тип взаємодії генів цистинурії і нормального вмісту цистину в сечі:

Повне домінування

Неповне домінування

Кодомінування

Комплементарність

Епістаз

1326 / 4299
При обстеженні в клініці у чоловіка діагностували гостру променеву хворобу. Лабораторно встановлено різке зниження серотоніну в тромбоцитах. Порушення метаболізму якої речовини є можливою причиною зниження тром-боцитарного серотоніну?

5-окситриптофан

Гістидин

Тирозин

Фенілаланін

Серин

1327 / 4299
Хворий тривалий час вживав антибіотики широкого спектру дії, що викликали зниження апетиту та пронос з гнилісним запахом. Про яку побічну дію йдеться?

Нефротоксична дія

Кардіотоксична дія

Ототоксична дія

Дисбактеріоз

Алергічна реакція

1328 / 4299
Хвора 65-ти років страждає на жовчнокам’яну хворобу. Останнім часом з’явилися ознаки ахолічного синдрому внаслідок обтурації жовчних шляхів. Засвоєння яких компонентів їжі буде порушене найбільше?

Нуклеїнові кислоти

Білки

Електроліти

Вуглеводи

Жири

1329 / 4299
У хворого на 2-3 день після операції резекції шлунка не відновилась перистальтика кишок. Що потрібно призначити хворому для стимуляції рухової функції шлунковокишкового тракту?

Празозин

Прозерин

Норадреналіну гідротартрат

Атропіну сульфат

Циклодол

1330 / 4299
Жінці 57-ми років проведено операцію екстирпації матки із придатками. Макроскопічно: в товщі міометрію вузол щільної консистенції, округлої форми з чіткими контурами, розміром 5х6 см, на розрізі волокнистої будови. Гістологічно він представлений хаотично розташованими пучками гладень-ком’язевих клітин та надмірно розвиненою стромою. Який найбільш імовірний діагноз?

Фіброміома

Рабдоміома

Рабдоміосаркома

Фіброма

Лейоміосаркома

1331 / 4299
Хворий потрапив до лікарні після іонізуючого опромінення зі скаргами на блювання, анорексію, біль у різних ділянках живота, наявність крові у калі, підвищення температури тіла, кволість. Для якої форми гострої променевої хвороби характерна така клінічна картина?

Кістковомозкова

Змішана

Кишкова

Токсемічна

Церебральна

1332 / 4299
У чоловіка 56-ти років системний артеріальний тиск становить 135/100 мм рт.ст. Збільшення якого з наведених чинників є найімовірнішою причиною збільшення діастолічного тиску у людини?

Загальний периферичний опір

Частота серцевих скорочень

Еластичність артеріальних судин

Насосна функція серця

Об’єм циркулюючої крові

1333 / 4299
У чоловіка 40-ка років на шиї виявлено пухлиноподібне утворення розміром 8х7 см, яке хірург видалив неповністю через інтимний зв’язок з великими судинами. Мікроскопічно в ньому виражений тканинний і клітинний атипізм, клітини типу ліпобластів різного ступеня зрілості, з поліморфізмом, гіперхромією ядер, патологічними мітозами, осередками некрозу. Визначте гістологічну форму пухлини:

Ліпосаркома

Ліпома

Фіброма

Фібросаркома

Гібернома

1334 / 4299
Відомо, що при цукровому діабеті у хворих частіше зустрічаються запальні процеси, знижена регенерація, уповільнюється загоєння ран. Причиною цього є:

Прискорення глюконеогенезу

Зниження протеосинтезу

Зниження ліполізу

Підвищення ліполізу

Посилення катаболізму

1335 / 4299
У хворого внаслідок інсульту утворилася гематома в коліні внутрішньої капсули, що призвело до паралічу мімічних та жувальних м’язів. Який провідний шлях ушкоджено?

Tr. cortico-spinalis

Tr. cortico-fronto-pontinus

Tr. cortico-temporo-parieto-occipito-pontinus

Tr. cortico-nuclearis

Tr. cortico-thalamicus

1336 / 4299
У жінки 49-ти років внаслідок тривалого стояння з’явився набряк ніг. Що є причиною появи набряків?

Зменшення гідростатичного тиску крові у венах

Зменшення гідростатичного тиску крові в артерiях

Збільшення системного артеріального тиску

Збільшення гідростатичного тиску крові у венах

Збільшення онкотичного тиску плазми крові

1337 / 4299
Хворому після апендектомії з метою профілактики інфекції призначений антибіотик групи цефалоспоринів. Порушення якого процесу полягає в основі протимікробної активності антибіотиків цієї групи?

Утворення мікробної стінки

Блокада холінестерази

Синтез нуклеїнових кислот

Рибосомальний синтез білку

Енергетичний обмін

1338 / 4299
При токсичному ушкодженні гепа-тоцитів з порушенням їх білковосинте-зуючої функції у пацієнта різко знизився вміст альбумінів у плазмі крові та онкотичний тиск плазми. Що буде наслідком цих змін?

Зменшення ШОЕ

Поява набряків

Збільшення густини крові

Зменшення діурезу

Збільшення об’єму циркулюючої крові

1339 / 4299
При мікроскопічному дослідженні виявляється паренхіматозний орган, в якому епітеліальні тяжі формують клу-бочкову, пучкову та сітчасту зони. Центральна частина органу представлена скупченнями хромафінних клітин. Визначте орган:

Печінка

Епіфіз

Гіпофіз

Надниркова залоза

Щитоподібна залоза

1340 / 4299
У хворого діагностовано грип. Після прийому антипіретиків стан його різко погіршився: свідомість потьмарена, АТ- 80/50 мм рт.ст., Ps- 140/хв, температура тіла різко знизилась до 35,8oC. Яке ускладнення виникло у даного хворого?

Колапс

Алкалоз

Гіпертермія

Ацидоз

Гіповолемія

1341 / 4299
В ендокринологічному відділенні обласної лікарні знаходиться хворий із скаргами на швидку втомлюваність, поганий апетит. З анамнезу відомо: тривалий час лікувався кортикостероїдами з приводу бронхіальної астми. Об’єктивно: АТ- 90/60 мм рт.ст., гіперпігментація шкіри. Яке захворювання можна запідозрити у хворого?

Хвороба Іценка-Кушінга

Хвороба Грейвса

Хвороба Кона

Хвороба Сімондса

Хвороба Аддісона

1342 / 4299
Хворий надійшов до інфекційного відділення: шкіра суха, тургор шкіри знижений, випорожнення у вигляді 'відвару рису'. Встановлено діагноз: холера. Яке порушення водно-електролітного балансу найбільш імовірно виникає при даному захворюванні?

Гіпоосмотична гіпергідратація

Гіперосмотична гіпогідратація

Гіпоосмотична гіпогідратація

Гіперосмотична гіпергідратація

Ізоосмотична гіпогідратація

1343 / 4299
Слизова оболонка трубчастого органу вкрита перехідним епітелієм, утворює поздовжні складки. М’язова оболонка складається з двох шарів у верхній половині і трьох у нижній. Який це орган?

Пряма кишка

Сечовий міхур

Стравохід

Сечовід

Маткова труба

1344 / 4299
При обстеженні хворого на атрофічний гастрит виявлено: ер.- 2,0 • 1012/л, Hb- 87 г/л, КП1,3, лейк.- 4,0 • 109/л, тромб.- 180 • 109/л. Дефіцит якої речовини може бути причиною анемії в хворого?

Цинк

Вітамін A

Вітамін K

Вітамін B12

Залізо

1345 / 4299
У жінки 46-ти років видалена пухлина правої привушної ділянки, яка поступово збільшувалася протягом 5 років. Макроскопічно: інкапсульова-ний вузол діаметром 60 мм еластичної консистенції, тканина бiлувато-dра з множинними дрі6ними кiстами, що містять слиз. Мікроскопічно: дуктальні структури або осередки солідної будови, що складаються з мономорфних полігональних та кубічних клітин, між ними розміщуються ділянки міксоїдної і хондроїдної речовини. Діагностуйте пухлину:

Мукоепідермальна пухлина

Аденолімфома

Мономорфна аденома

Аденокістозна карцинома

Плеоморфна аденома

1346 / 4299
У зв’язку з підозрою на внутрі-шньолікарняну інфекцію проведено обстеження у відділенні хірургічної стоматології. У кількох пацієнтів, а також на деяких предметах догляду виявлено золотистий стафілокок. Які властивості виділених культур дають можливість встановити їх походження з одного джерела?

Пігментоутворення

Чутливість до антибіотиків

Антигенна структура

Біохімічні властивості

Фаготипування

1347 / 4299
При втручанні з метою лікування вивиху нижньої щелепи лікар повинен пам’ятати про м’яз, який при скороченні відтягує назовні капсулу і суглобовий диск скроневонижньощелепного суглоба. Який це м’яз?

M. mylohyoideus

M. masseter

M. pterygoideus lateralis

M. pterygoideus medialis

M. temporalis

1348 / 4299
У чоловіка 71-го року впродовж 10 днів спостерігався пронос з домішками у калі слизу та крові. Хворого було госпіталізовано у тяжкому стані, помер через 2 доби. При розтині тіла померлого виявлено: дифтеритичний коліт з множинними виразками неправильної форми різної глибини в сигмоподібній і прямій кишках. При бактеріологічному дослідженні висіяно шигели. Яке основне захворювання у хворого?

Неспецифічний виразковий коліт

Сальмонельоз

Черевний тиф

Ієрсиніоз

Дизентерія

1349 / 4299
У хворого 75-ти років, який страждав на розповсюджений атеросклероз та помер від хронічної серцевої недостатності, на розтині виявлені звужені деформовані коронарні артерії, горбиста інтима на розрізі білого кольору, кам’яниста. Назвіть стадію морфогенезу атеросклерозу:

Стадія атероматоза

Ліпосклероз

Доліпідна стадія

Атерокальциноз

Ліпоїдоз

1350 / 4299
При мікроскопічному дослідженні біоптата з товстої кишки виявлена пухлина з призматичного епітелію, що формує атипові залозисті структури різної форми і величини. Базальна мембрана залоз зруйнована. Клітки пухлини поліморфні, ядра гіперхромні, відзначається велика кількість патологічних мітозів. Який діагноз найбільш вірогідний?

Слизовий рак

Солідний рак

Недиференційований рак

Аденокарцинома

Базальноклітинний рак

1351 / 4299
У пацієнта на слизовій оболонці ротової порожнини, носа та губах з’явились везикулярні пухирці. Стоматолог запідозрив везикулярний стоматит. Яке дослідження надасть можливість підтвердити діагноз?

Постановка алергічної проби

Мікроскопія везикулярної рідини

Виділення вірусу з везикулярної рідини

Виділення бактерій з везикулярної рідини

Зараження тварин везикулярною рідиною

1352 / 4299
При дослідженні бронхобіоптату встановлено: атрофія слизової оболонки, кістозне перетворення залоз, осередкова метаплазія покривного призматичного епітелію в багатошаровий плоский, збільшення числа келихоподібних клітин; місцями - у стінці бронха та особливо у слизовій оболонці різко виражена клітинна запальна інфільтра- ція і розростання грануляційної тканини, яка вибухає у просвіт бронха у вигляді поліпа. Який найбільш імовірний діагноз?

Гострий бронхіт

!нтерстиційна пневмонія

Хронічний бронхіт

Часткова пневмонія

Бронхопневмонія

1353 / 4299
При мікроскопії мазка з плівки, яка з’явилась на пептонній воді через 6 годин після посіву фекалій і культивування в термостаті, виявлені зігнуті у вигляді коми рухливі бактерії, грам-негативні, спор і капсул не утворюють. Які це мікроорганізми?

Вібріони

Спірохети

Клостридії

Спірили

Коринебактерії

1354 / 4299
Під час операції на щелепно-лицьовій ділянці для зниження секреції слинних залоз хворому ввели атропіну сульфат. Виникли тахікардія, сухість та гіперемія шкіри, параліч акомодації та підвищення внутрішньоочного тиску. Який антагоніст доцільно використати в даному випадку?

Скополаміну гідробромід

Дипіридамол

Платифіліну гідротартрат

Атенолол

Прозерин

1355 / 4299
Внаслідок вірусного процесу в підщелепних слинних залозах відбувся значний склероз їх паренхіми і зменшилася продукція біологічно активних гормональних речовин. Через те погіршилася регенерація слизової оболонки ротової порожнини. Причиною цього є недостатній вміст у слині:

Фактора росту епітелію

Лізоциму

Інсуліноподібного фактора

Паротину

Тимоцит-трансформуючого фактора

1356 / 4299
Для знечулення різців верхньої щелепи анестетик вводять у ділянку різцевого отвору. Який нерв залягає в цьому місці?

N.palatinus major

N.nasopalatinus

Rr.nasales posteriores inferiores

Nn.palatini minores

N.pharyngeus

1357 / 4299
В процесі гістогенезу тканин зуба з певних причин вчасно не утворився дентин. Який процес подальшого гістогенезу не відбудеться або буде віддалений у часі?

Утворення клітинного цементу

Утворення пульпи

Утворення безклітинного цементу

Утворення емалі

Утворення предентинового простору

1358 / 4299
Під час гістологічного дослідження операційного матеріалу з ділянки ікла верхньої щелепи 20-річного чоловіка виявлено пухлину, яка побудована з численних дрібних примітивних залозистих та тубулярних структур, із незміненого одонтогенного епітелію, розташованих серед гіалінізованої сполучнотканинної строми. Діагностуйте пухлину:

Амелобластома

Амелобластична фіброма

Остеобластокластома

Аденоматоїдна пухлина

Первинна внутрішньокісткова карцинома

1359 / 4299
У чоловіка 70-ти років розвинувся протезний стоматит. Крім того, спостерігається виражене ураження куточків рота. Під час мікроскопії виявлені великі овоїдні грампозитивні клітини. Які мікроорганізми найбільш імовірно стали провідним етіологічним фактором такого ураження?

Стафілококи

Стрептококи

Гриби роду Candida

Коринебактерії

Нейсерії

1360 / 4299
Лікарю-стоматологу для знеболен-ня необхідно ввести анестетик у крило-піднебінну ямку із ротової порожнини. В ділянці якої анатомічної структури потрібно робити ін’єкцію?

Різцевий канал

Піднижньоочноямковий отвір

Нижньощелепний отвір

Великий піднебінний отвір

Підборідний отвір

1361 / 4299
Під час мікроскопічного дослідження в залозі дитини 10-ти років були визначені тільки серозні кінцеві відділи, міжчасточкові протоки вистелені двошаровим чи багатошаровим епітелієм. Визначте залозу:

Підщелепна

Підшлункова

Під’язикова

Привушна

Печінка

1362 / 4299
При обстеженні групи осіб, що мешкають на одній території, лікар-стоматолог звернув увагу на однакові симптоми захворювання - темно-жовті плями емалі зубів. Надлишок якого мікроелементу у продуктах або питній воді може бути причиною цього стану?

Мідь

Фтор

Кальцій

Нікель

Йод

1363 / 4299
У біоптаті нирки виявлено: проміжна тканина інфільтрована лейкоцитами, міліарні абсцеси, канальці в стані дистрофії, заповнені десквамованим епітелієм та лейкоцитами. Про яке захворювання можна думати?

Некротичний нефроз

Пієліт

Нефролітіаз

Пієлонефрит

Гломерулонефрит

1364 / 4299
Лікар оглянув зів хворого і встановив діагноз: тонзиліт (запалення піднебінних мигдаликів). При цьому було відзначено збільшення розмірів лімфатичних вузлів. Які це вузли?

Соскоподібні

Піднижньощелепні

Поверхневі шийні

Підпідборідні

Потиличні

1365 / 4299
У пульпі зуба є зона, у якій судини і нерви оточені колагеновими та ретикулярними волокнами, фібробластами, макрофагами, мало диференційованими клітинами - перицитами, адвенти-ційними клітинами, тканинними базофілами. Як називається ця зона пульпи?

Коренева

Предентинна

Центральна

Коронкова

Проміжна

1366 / 4299
У хлопчика із синдромом Дауна спостерігаються аномалії лицевої частини черепа, включаючи гіпоплазію верхньої щелепи, високе піднебіння, неправильний ріст зубів. Який із каріотипів притаманний цій дитині?

48, XXYY

47, XY+21

47XY+18

48, XXXY

47, XXY

1367 / 4299
Для морфологічного дослідження представлена ендокринна залоза, паренхіма якої складається з епітелію та нервової тканини. В епітеліальних тра-бекулах виявляється 2 типи клітин: хро-мофільні та хромофобні. Визначте даний орган:

Прищитоподібна залоза

Гіпоталамус

Щитоподібна залоза

Надниркова залоза

Гіпофіз

1368 / 4299
У дитини 12-ти років непереноси-мість ряду харчових продуктів. Їх вживання викликає алергічну реакцію у вигляді висипань на шкірі, що сверблять. Який антигістамінний засіб слід призначити, щоб не заважати шкільним заняттям дитини (не викликати сонливості)?

Мезатон

Диклофенак-натрій

Димедрол

Еуфілін

Лоратадин

1369 / 4299
До приймального відділення надійшов хворий з ознаками гострої серцевої недостатності: блідість, акроціаноз, часте, поверхневе дихання. Який з перелічених засобів показаний у цьому випадку?

Дигітоксин

Адреналіну гідрохлорид

Нітрогліцерин

Корглікон

Кордіамін

1370 / 4299
При огляді порожнини рота дитини педіатр виявив, що прорізалися ікла. Дитина розвивається нормально. Визначте вік дитини:

6-8 міс.

10-12 міс.

14-20 міс.

8-10 міс.

1371 / 4299
У хворого на вірусний гепатит на другому тижні захворювання виникли: розлад сну, головний біль, агресивність, нестерпний свербіж шкіри. Об’єктивно: відзначається зниження артеріального тиску, швидкості згортання крові, рефлекторної діяльності, брадикардія. Чим зумовлені ці зміни?

Гіперліпемія

Стеркобілінемія

Уробілінемія

Гіперхолестеринемія

Холемія

1372 / 4299
У чоловіка 25-ти років після тривалого перебування на сонці при високій вологості повітря підвищилась температура тіла до 39oC. Який патологічний процес спостерігається у цьому випадку?

Гіпотермія

Опікова хвороба

Гіпертермія

Неінфекційна гарячка

Інфекційна гарячка

1373 / 4299
Чоловік 26-ти років внаслідок автомобільної аварії перебуває в торпі-дній стадії шоку. У крові: лейк.- 3, 2 • 109/л. Який головний механізм в розвитку лейкопенії?

Руйнування лейкоцитів у кровотворних органах

Порушення виходу зрілих лейкоцитів з кісткового мозку в кров

Підвищення виділення лейкоцитів з організму

Пригнічення лейкопоезу

Перерозподіл лейкоцитів у судинному руслі

1374 / 4299
У пацієнта з’явились герпетичні висипання. Який засіб необхідно призначити?

Клотримазол

Бісептол

Бензилпеніцилін

Ацикловір

Гентаміцин

1375 / 4299
Хлопчик 5-ти місяців госпіталізований з приводу тонічних судом. Хворіє з народження. При огляді волосся жорстке, нігті витончені та ламкі, шкірні покриви бліді та сухі. У крові знижений вміст іонів кальцію. З чим пов’язані ці зміни?

Гіперпаратиреоз

Гіпопаратиреоз

Гіпоальдостеронізм

Гіперальдостеронізм

Гіпотиреоз

1376 / 4299
У хворого на алкоголізм почався алкогольний психоз із вираженим психомоторним збудженням. Який препарат з групи нейролептиків слід призначити з метою надання швидкої допомоги?

Діазепам

Резерпін

Аміназин

Фторотан

Натрію бромід

1377 / 4299
При посіві матеріалу із зіва хворого ангіною на кров’яно-телуритовий агар виросли колонії діаметром 4-5 мм, сірого кольору, радіально покреслені (у вигляді розеток). Під мікроскопом грам-позитивні палички із булавоподібними потовщеннями на кінцях, розміщені у вигляді розчепірених пальців. Які це мікроорганізми?

Дифтероїди

Коринебактерії дифтерії

Стрептококи

Клостридії ботулізму

Стрептобацили

1378 / 4299
У хворого, який страждає на вугри та запальні зміни шкіри обличчя, при мікроскопії матеріалу з осередків ураження виявлені живі членистоногі розміром 0,2-0,5 мм. Вони мають витягнуту червоподібну форму, чотири пари коротких кінцівок, розміщених у середній частині тіла. Виявлені членистоногі викликають:

Демодекоз

Коросту

Шкірний міаз

Педикульоз

Фтиріоз

1379 / 4299
У препараті кори головного мозку, імпрегнованому азотнокислим сріблом, ідентифікуються велетенські нейрони пірамідної форми. Який шар кори утворюють ці клітини?

Внутрішній зернистий

Зовнішній зернистий

Пірамідний

Молекулярний

Гангліонарний

1380 / 4299
В експерименті у тварини перерізали таламокортикальні шляхи. Який вид сенсорних відчуттів у піддослідної тварини зберігся?

Нюхові

Екстероцептивні

Слухові

Зорові

Ноцицептивні

1381 / 4299
У хворого внаслідок травми порушена чутливість верхньої губи та крила носа. Який нерв, найвірогідніше, ушкоджений?

N. nasociliaris

N. mandibularis

N. infraorbitalis

N. buccalis

N. frontalis

1382 / 4299
На розрізі легенева тканина має великокомірчастий вигляд у зв’язку з мішкоподібними і циліндричними розширеннями бронхів; мікроскопічно у стінці таких бронхів відзначається лейкоцитарна інфільтрація з переважанням нейтрофілів; еластичні, м’язові волокна, а також хрящові пластинки частково зруйновані і заміщені сполучною тканиною. Прилегла легенева тканина з фокусами запалення, полями фіброзу, склерозом судин, ознаками емфіземи. Гіпертрофований правий тттлу-ночок серця. Який найбільш імовірний діагноз?

Емфізема легень

Хронічний бронхіт

Інтерстиційна пневмонія

Пневмофіброз

Бронхоектатична хвороба

1383 / 4299
У ході експерименту у тварини перерізали спинний мозок нижче 5-го шийного сегменту. Внаслідок цього зовнішнє дихання у тварини:

Стане більш частим

Стане діафрагмальним

Стане глибоким

Припиниться

Стане поверхневим

1384 / 4299
У чоловіка 25-ти років на спинці язика зліва з’явилась блюдцеподібна виразка 0,8 см в діаметрі. Дно і краї виразки щільні, поверхня їх гладенька, блискуча, при пальпації безболісна. При мікроскопічному дослідженні на межі виразки та в області її дна - інфільтрат з лімфоїдних, плазматичних, епітеліоїдних клітин при наявності значної кількості судин з ендоваскулітом. Який найбільш вірогідний діагноз?

Ракова виразка

Первинний сифіліс

Декубітальна виразка

Туберкульоз

Афта Сеттона

1385 / 4299
Хворий потрапив до щелепно-лицьового відділення з переломом виличної дуги. У хворого утруднене закривання рота. Порушення функції якого м’язу призвело до цього?

Двочеревцевий

Бічний крилоподібний

Жувальний

Присередній крилоподібний

Виличний

1386 / 4299
Дитина під час гри отримала удар у ділянку рукоятки груднини. В результаті травми постраждав орган, що знаходиться за рукояткою груднини. Назвіть цей орган:

Вилочкова залоза

Гортань

Щитоподібна залоза

Перикард

Серце

1387 / 4299
До стоматолога звернувся чоловік 35-ти років зі скаргами на зменшення щільності зубної тканини, підвищену крихкість зубів при вживанні твердої їжі. Лабораторно визначили співвідношення Ca/P в емалі при зіскоблюванні. Яке значення цього показника свідчить про посилення демінералізації?

1,5

1,67

2,5

0,9

1,85

1388 / 4299
У пацієнта вилучений верхній медіальний різець. Гілки якої артерії його кровопостачають?

A.alveolaris inferior

A.palatina descendens

A.sphenopalatina

A.buccalis

A.infraorbitalis

1389 / 4299
У хворого зі скаргами на часті рідкі випорожнення з кров’ю ('малинове желе') при мікроскопічному дослідженні були виявлені великі клітини з одним ядром та поглиненими еритроцитами. Для якого з найпростіших характерна така морфологічна форма?

Campylobacter jejuni

Toxoplasma gondii

Giardia lamblia

Entamoeba histolytica

Balantidium coli

1390 / 4299
Під час гістологічного вивчення мі-кропрепаратів зуба людини похилого віку виявлені великі вакуолі в одонто-бластах. Тканина пульпи нагадує сітку з гіалінізованих тонких волокон, частина одонтобластів дуже низькі, у вигляді 'ниточки'. Для якого захворювання характерні такі зміни?

Вроджене недорозвинення пульпи

Сітчаста атрофія пульпи

Гострий пульпіт

Дефекти пульпи зуба

1391 / 4299
При рентгенологічному дослідженні встановлено проростання кореня зуба в гайморову порожнину. Визначте, який зуб міг прорости?

Перший верхній премоляр

Другий верхній премоляр

Перший верхній моляр

Верхнє ікло

Другий верхній моляр

1392 / 4299
У хворої 35-ти років з хронічним періодонтитом біля кореня 15 зуба видалено кісту діаметром 3 см. При гістологічному досліджені встановлено, що стінка її тонка, утворена зрілою сполучною тканиною, яка інфільтрована лімфоцитами і плазматичними клітинами, внутрішня поверхня вистелена багатошаровим плоским епітелієм без ознак кератинізації, в порожнині серозний ексудат. Який найбільш імовірний діагноз?

Примордіальна кіста

Фолікулярна кіста

Херувізм

Радикулярна кіста

Фолікулярна амелобластома

1393 / 4299
Хворого після автомобільної аварії госпіталізовано з масивною кровотечею з м’яких тканин обличчя. Яку артерію треба притиснути, щоб зупинити кровотечу?

Лицева

Язикова

Підключична

Верхньощелепна

Внутрішня сонна

1394 / 4299
Хворий 43-х років тривалий час не отримував з їжею достатньої кількості вітаміну C, що призвело до порушень в утворенні колагенових волокон. Яка структура підтримуючого апарата зуба буде страждати у першу чергу?

Ясеневі кишені

Дентин

Епітелій ясен

Епітелій борозни

Періодонт

1395 / 4299
У хворого є зміни секреторної функції сльозової залози, що пов’язано з порушенням її вегетативної іннервації. Який з вузлів вегетативної нервової системи віддає післявузлові парасимпатичні волокна для неї?

Ganglion oticum

Ganglion ciliare

Ganglion submandibulare

Ganglion pterygopalatinum

1396 / 4299
У чоловіка, який повернувся з весняної дослідницької експедиції, спостерігаються слабкість, нудота, порушення сну, підвищення температури тіла, ознаки паралічу м’язів шиї та плечового поясу. З анамнезу відомо, що його кусали кліщі. При лабораторній діагностиці встановлено діагноз - весняно-літній енцефаліт. Який шлях зараження хворого?

Статевий

Трансмісивний

Перкутанний

Контактно-побутовий

Пероральний

1397 / 4299
У пацієнта навесні з’явилися пете-хіальні крововиливи, розхитування зу- бів, він відмічає високу чутливість до простудних хвороб. Лікар припустив гіповітаміноз C. Чим пояснюються зміни з боку зубів?

Підвищення проникності мембран навколозубних тканин

Порушення окисно-відновних процесів у навколозубних тканинах

Зміна структури глікозаміногліканів

Порушення структури колагену періодонтальних зв’язок

Механічне ушкодження зубів

1398 / 4299
Хворому на остеомієліт нижньої щелепи був призначений антибіотик тетрациклінової групи. Вкажіть цей препарат:

Рифампіцин

Оксацилін

Амікацин

Стрептоміцин

Доксицикліну гідрохлорид

1399 / 4299
У хворої 45-ти років відзначається деформація обличчя через пухлиноподібне розростання без утворення капсули, яке росте у кістці, мікроскопічно представлене клітинно-волокнистою тканиною і остеоїдними балочками. Яка патологія найбільш імовірна?

Остеобластокластома

Херувізм

Еозинофільна гранульома

Кіста щелепних кісток

Фіброзна дисплазія щелепних кісток

1400 / 4299
При огляді порожнини рота пацієнта стоматолог виявив каріозну порожнину на поверхні першого верхнього моляра, яка звернена до другого моляра. Яка поверхня зуба уражена?

Вестибулярна

Жувальна

Мезіальна

Язична

Дистальна

1401 / 4299
Гіпоплазія емалі зумовлена домінантним геном, локалізованим в Х-хромосомі. Мати має нормальну емаль зубів, а у батька спостерігається гіпоплазія емалі. У кого з дітей буде виявлятися ця аномалія?

У всіх дітей

Тільки у синів

Тільки у дочок

У половини дочок

У половини синів

1402 / 4299
У районах Південної Африки у людей розповсюджена серпоподібно-клітинна анемія, при якій еритроцити мають форму серпа внаслідок зміни в молекулі гемоглобіну амінокислоти глутаміну на валін. Причиною цієї хвороби є:

Трансдукція

Генна мутація

Геномні мутації

Порушення механізмів реалізації генетичної інформації

Кросинговер

1403 / 4299
При обстеженні дівчини 18-ти років знайдені наступні ознаки: недорозвинення яєчників, широкі плечі, вузький таз, вкорочення нижніх кінцівок, 'шия сфінкса', розумовий розвиток не порушено. Встановлено діагноз: синдром Шерешевського-Тернера. Яке хромосомне порушення у хворої?

Трисомія 13

Трисомія Х

Трисомія 18

Моносомія Х

Нульсомія Х

1404 / 4299
У 5-річної дівчинки на періанальних складках мати знайшла білих 'черв’ячків' , які викликають у дитини свербіж і неспокій, і доставила їх до лабораторії. При дослідженні лікар побачив білих гельмінтів 0,5-1 см довжиною, ниткоподібної форми з загостреними кінцями, у деяких вони заокруглені. Який діагноз можна встановити?

Аскаридоз

Теніоз

Дифілоботріоз

Опісторхоз

Ентеробіоз

1405 / 4299
У дитини періодично з’являються рідкі випорожнення, іноді біль у ділянці живота, нудота, блювання. Зі слів матері, одного разу у дитини з блювотною масою виділився гельмінт веретеноподібної форми, розміром 20 см. Причиною такого стану може бути:

Трихоцефальоз

Анкілостомоз

Трихінельоз

Аскаридоз

Дракункульоз

1406 / 4299
У 60-річної жінки з тяжкою формою парадонтозу при мікроскопічному дослідженні зіскобу з ясен були виявлені одноядерні найпростіші розміром 360 мкм з широкими псевдоподіями. Які найпростіші були виявлені у хворої?

Toxoplasma gondii

Balantidium coli

Entamoeba histolytica

Entamoeba gingivalis

Trichomonas tenax

1407 / 4299
У хворого внаслідок неадекватного лікування запалення середнього вуха (мезотимпаніт) виникло гнійне запалення комірок соскоподібного відростка скроневої кістки (мастоїдит). Гнійник прорвався у ложе грудино-ключично-соскоподібного м’яза. У межах якої фасції шиї локалізується патологічний процес?

Внутрішня фасція

Передхребетна фасція

Поверхнева фасція

Поверхневий листок власної фасції

Глибокий листок власної фасції

1408 / 4299
Після видалення II малого кутнього зуба верхньої щелепи у хворого відзначається коміркова кровотеча. Пошкодженням якої артерії воно викликане?

Середня верхня альвеолярна

Нижня альвеолярна

Передня верхня альвеолярна

Задня верхня альвеолярна

Піднебінна

1409 / 4299
Чоловік 30-ти років звернувся до стоматолога зі скаргою на розлади жування, виникнення болю при відтягуванні щелепи назад. Лікар встановив запалення одного з жувальних м’язів. Якого саме?

Жувальний

Крилоподібний медіальний

Скроневий (задні волокна)

Скроневий (передні волокна)

Крилоподібний латеральний

1410 / 4299
Юнака 18-ти років доставлено в лікарню з ознаками внутрішньої кровотечі. Під час гри в футбол він отримав удар в ділянку лівого підребер’я. Ушкодження якого органу спричинило сильну кровотечу?

Flexura coli sinistra

Lien

Ren sinistra

Fundus ventriculi

Cauda pancreatis

1411 / 4299
У дітей часто можна спостерігати утруднене носове дихання, яке пов’язане з надмірним розвитком лімфої-дної тканини слизової оболонки глотки. Розростання яких мигдаликів може спричинити це явище?

Усіх названих мигдаликів

Tonsilla pharyngea

Tonsilla lingualis

Tonsilla tubaria

Tonsilla palatina

1412 / 4299
У хворого 26-ти років виявлений великий фурункул м’яких тканин обличчя біля кореню носа та нижньої повіки. Грізним ускладненням цього захворювання може бути розповсюдження інфекції по венозним сполученням цього регіону до пазух твердої мозкової оболонки. Яка з пазух найбільш імовірно може бути уражена?

Верхня кам’яниста

Печериста

Потилична

Верхня сагітальна

Сигмоподібна

1413 / 4299
У хворого з флегмоною щоки розвинувся менінгіт. Поширення інфекції, найбільш імовірно, відбулося по анастомозу між:

Лицевою і позадущелепною венами

Скроневою і надочноямковою артеріями

Лицевою веною та крилоподібним сплетінням

Лицевою і очною артеріями

Лицевою веною і венами очної ямки

1414 / 4299
У потерпілого лівобічний уламковий перелом виличної кістки з втратою шкірної чутливості над нею. Який нерв пошкоджений?

Вилично-лицевий

Нижньоочноямковий

Щічний

Мала гусяча лапка

Лицевий

1415 / 4299
Хворий з запаленням слизової язика скаржиться на розлад загальної чутливості передніх двох третин язика. Ураженням якого нерва це викликано?

Язикоглотковий

Барабанна струна

Блукаючий

Язиковий

Барабанний

1416 / 4299
У хворого 25-ти років, який скаржився на утруднене ковтання, виявлене пухлиноподібне підвищення на корені язика 1-2 см в діаметрі, в ділянці сліпого отвору. Розростання залишків якої залози встановив лікар?

Прищитоподібна

Під’язикова

Аденогіпофіз

Вилочкова

Щитоподібна

1417 / 4299
У хворого внаслідок пошкодження шкіри в середній ділянці грудинно-ключичнососкоподібного м’яза виникла повітряна емболія. Яка вена шиї була травмована?

Передня яремна

Внутрішня яремна

Зовнішня яремна

Поперечна вена шиї

Задня вушна

1418 / 4299
Під час дослідження обличчя хворого відзначається, що правий кут рота зміщений у лівий бік, права щока і губи притиснені до зубів і ясен. Порушенням функції якого м’яза це викликане?

Виличні м’язи

М’яз, що опускає кут рота

Коловий м’яз рота

Щічний м’яз

М’яз сміху

1419 / 4299
До лікаря звернувся хворий з запаленням комірок решітчастої кістки (етмоїдит). При обстеженні виявлено порушення кровопостачання кістки. Гілками якої артерії кровопостачаю-ться решітчасті комірки в нормі?

A. ophthalmica

A. facialis

A. cerebri anterior

A. infraorbitalis

A. transversa faciei

1420 / 4299
У пораненого кровотеча із гілок сонної артерії. Для тимчасової зупинки кровотечі сонну артерію треба притиснути до горбика поперечного відростка шийного хребця. Якого саме?

II

IV

V

III

VI

1421 / 4299
У чоловіка 40-ка років запалення яєчка ускладнилось його водянкою. Необхідне оперативне втручання. Яку з оболонок яєчка останньою розтинає хірург під час операції?

Парієтальний листок піхвової оболонки яєчка

М’яз-підіймач яєчка

М ’ясиста оболонка

Внутрішня сім’яна фасція

Зовнішня сім’яна фасція

1422 / 4299
Жінка 52-х років після травми. При огляді констатовано зниження пульсації, блідість шкіри і зниження температури в ділянці великого пальця правої руки. Ушкодження якої з артерій верхньої кінцівки, найімовірніше, приводить до таких наслідків?

A.ulnaris

A.princeps pollicis

A.digitalis propria

A.radialis

A.digitalis communis

1423 / 4299
У хворої з пухлиною підшлункової залози розвинулася механічна жовтяниця внаслідок стиснення жовчовивідної протоки. Яка протока піддається стисненню?

Ductus hepaticus communis

Ductus hepaticus sinister

Ductus choledochus

Ductus cysticus

Ductus hepaticus dexter

1424 / 4299
В гістопрепараті представлена частина органу, що містить багатошаровий плоский незроговілий епітелій та нижче розташовані сполучнотканинні пластинки, які не містять судин. Який це орган?

Кришталик

Рогівка

Сітківка

Слизова оболонка стравоходу

Слизова оболонка ротової порожнини

1425 / 4299
У гістологічному препараті шліфа зуба у міжклітинній речовині визначаються колагенові волокна, що йдуть тангенційно до дентино-емалевої межі і перпендикулярно до дентинних трубочок (волокна Ебнера). Назвіть даний шар дентину:

Інтерглобулярний дентин

Зернистий шар

Вторинний дентин

Навколопульпарний дентин

Плащовий дентин

1426 / 4299
У культурі тканин ядерним опроміненням пошкоджені ядерця ядер. Відновлення яких органел у цитоплазмі клітин стає проблематичним?

Ендоплазматична сітка

Рибосоми

Лізосоми

Мікротрубочки

Комплекс Іольджі

1427 / 4299
Під час гаструляції у зародку недостатньо сформувався первинний Іензе-новський вузлик. Розвиток якого осьового органу загальмується?

Мантійний шар нервової трубки

Нервові гребінці

Хорда

Нервовий жолобок

Нервова трубка

1428 / 4299
У дитини на шкірі навколо подряпини виникли ознаки запалення (біль, почервоніння та набряк), як ознаки негайної гіперчутливості. Які клітини крові обумовлюють ці зміни?

Базофіли

Нейтрофіли

Еозинофіли

Лімфоцити

Моноцити

1429 / 4299
В гістологічному препараті відділ нервової системи, в якому визначається пошарове розташування нейроци-тів, серед яких є клітини таких форм: зірчасті, веретеноподібні, горизонтальні, пірамідні. Який це відділ нервової системи?

Спинний мозок

Кора мозочка

Кора великих півкуль головного мозку

Спинномозковий вузол

Вегетативний вузол

1430 / 4299
У крові дівчини 16-ти років, котра страждає на аутоiмунне запалення щи-топоді6ної залози, виявлено числєнні плазматичні клітини. З проліферацією та диференціюванням яких клітин крові пов’язано збільшення кількості пла-змоцитів?

В-лімфоцитів

Т-кілерів

Тканинних базофілів

Т-хелперів

Т-супресорів

1431 / 4299
В препараті діагностується тканина, в якій клітини розміщуються поодинці та ізогрупами, а в міжклітинній речовині не видно волокнистих структур. Яка тканина присутня в препараті?

Волокниста хрящова

Гіалінова хрящова

Епітеліальна

Гладенька м’язова

Кісткова

1432 / 4299
При дослідженні оболонок головного мозку медичний експерт знайшов зяючі венозні судини, що зрощені з навколишніми тканинами. Назвіть, до яких вен відносяться ці судини:

Венули

Вени із сильним розвитком м’язових елементів

Вени із слабким розвитком м’язових елементів

Вени безм’язового типу

Вени із середнім розвитком м’язових елементів

1433 / 4299
В гострому досліді тварині в порожнину 12-ти палої кишки ввели слабкий розчин хлористоводневої кислоти. До збільшення секреції якого гастроін-тестинального гормону це призведе?

Гістамін

Нейротензин

Гастрин

Секретин

Мотилін

1434 / 4299
При визначенні групової належності крові за системою АВ0 аглютина- цію еритроцитів досліджуваної крові викликали стандартні сироватки першої та другої груп і не викликала -третьої групи. Якої група досліджувана кров?

B (III) а

0 (I) а,в

АВ (IV)

a(ii) в

1435 / 4299
У хворого з пересадженим серцем при фізичному навантаженні збільшився хвилинний об’єм крові. Який механізм регуляції забезпечує ці зміни?

Парасимпатичні умовні рефлекси

Симпатичні безумовні рефлекси

Симпатичні умовні рефлекси

Парасимпатичні безумовні рефлекси

Катехоламіни

1436 / 4299
При аналізі ЕКГ людини з’ясовано, що у другому стандартному відведенні від кінцівок зубці T позитивні, їх амплітуда та тривалість у нормі. Вірним є висновок, що у шлуночках серця нормально відбувається процес:

Деполяризації

Розслаблення

Реполяризації

Збудження

Скорочення

1437 / 4299
У хворого виявлено різке зниження активності сурфактанту легень. Що буде наслідком цього?

Схильність альвеол до спадання

Зменшення опору дихальних шляхів

Зменшення роботи дихальних м’язів

Збільшення вентиляції легень

Гіпероксемія

1438 / 4299
В результаті черепно-мозкової травми у хворого були виявлені наступні ознаки: інтенційний тремор, дисме-трія, адіадохокінез, дизартрія. Яка структура головного мозку пошкоджена?

Стріатум

Рухова кора

Бліда куля

Чорна речовина

Мозочок

1439 / 4299
У хворого 70-ти років діагностовано крововилив у стовбур мозку. Обстеження виявило підвищення тонусу м’язів-згиначів на тлі зниження тонусу м’язів-розгиначів. Подразненням яких структур мозку можна пояснити зміни у тонусі м’язів?

Чотиригорбикова структура

Вестибулярні ядра

Ретикулярна формація

Червоні ядра

Чорна речовина

1440 / 4299
У пацієнта після повторного протезування зу6ів виникли сухість і металічний присмак у роті, спотворення смаку, запалення слизової оболонки язика і ясен. Найбільш імовірною причиною вказаних проявів є:

Пошкодження чутливих нервових волокон

Занесення інфекції

Пошкодження смакових рецепторів

Застосування неякісної пластмаси

Явище гальванізму

1441 / 4299
При обстежені пацієнта встановили що він має сильний, врівноважений, інертний тип вищої нервової діяльності за Павловим. Який темперамент за Гіппократом має цей пацієнт?

Меланхолічний

Флегматичний

Сангвінічний

Холеричний

1442 / 4299
У жінки 32-х років запалення ясен супроводжується їх гіпоксією. Утворення якого метаболіту вуглеводного обміну значно збільшується при цьому в тканинах пародонта?

Ілюкозо-6-фосфат

Рибозо-5-фосфат

Глікоген

НАДФ-Н

Лактат

1443 / 4299
У хворого з нирковою недостатністю розвинулась остеодистрофія, що супроводжується інтенсивною деміне-ралізацією кісток. Порушення утворення активної форми якого вітаміну є причиною даного ускладнення?

Тіамін

Рибофлавін

Нафтохінон

Кальциферол

Ретинол

1444 / 4299
При обробці перекисом водню слизової оболонки ротової порожнини хворого, кров пофарбувалась у кори- чневий колір замість піноутворення. При зниженні концентрації якого з перелічених ферментів це можливо?

Псевдохолінестераза

Метгемоглобінредуктаза

Каталаза

Ілюкозо-6-фосфатдегідрогеназа

Ацетилтрансфераза

1445 / 4299
При цукровому діабеті і голодуванні в крові збільшується вміст ацетонових тіл, що використовуються в якості енергетичного матеріалу. Назвіть речовину, з якої вони синтезуються:

Ацетіл-КоА

Кетоглутарат

Малат

Сукциніл-КоА

Цитрат

1446 / 4299
При обстеженні хворого виявлені дерматит, діарея, деменція. Відсутність якого вітаміну є причиною цього стану?

Аскорбінова кислота

Біотин

Фолієва кислота

Рутин

Нікотинамід

1447 / 4299
Електрофоретичне дослідження сироватки крові хворого на пневмонію показало збільшення одної з білкових фракцій. Вкажіть її:

а2-глобуліни

Альбуміни

в-глобуліни

y-глобуліни

аі-глобуліни

1448 / 4299
Мати помітила темну сечу у її 5-річної дитини. Жовчних пігментів у сечі не виявлено. Встановлено діагноз: ал-каптонурія. Дефіцит якого ферменту має місце?

Декарбоксилаза фенілпірувату

Тирозиназа

Оксидаза оксифенілпірувату

Оксидаза гомогентизинової кислоти

Фенілаланінгідроксилаза

1449 / 4299
У хворого в крові збільшена концентрація пірувату. Значна кількість його екскретується з сечею. Який авітаміноз спостерігається у хворого?

E

B2

B3

B6

B1

1450 / 4299
У чоловіка 42-х років, який страждає на подагру, в крові підвищена концентрація сечової кислоти. Для зниження рівня сечової кислоті йому призначено аллопуринол. Конкурентним інгібітором якого ферменту є аллопу-ринол?

Аденозиндезаміназа

Аденінфосфорибозилтрансфераза

Ксантиноксидаза

Гуаніндезаміназа

Гіпоксантинф осф орибозилтрансфе-раза

1451 / 4299
У 8-місячної дитини спостерігаються блювання та діарея після прийому фруктових соків. Навантаження фруктозою призвело до гіпоглікемії. Спадкова недостатність якого ферменту є причиною стану дитини?

Гексокіназа

Фруктокіназа

Фруктозо-1,6-дифосфатаза

Фосфофруктокіназа

Фруктозо-1-фосфатальдолаза

1452 / 4299
При лікуванні пародонтиту використовують препарати кальцію та гормон, що має здатність стимулювати мінералізацію зубів та гальмувати резорбцію кісткової тканини, а саме:

Альдостерон

Кальцитонін

Паратгормон

Тироксин

Адреналін

1453 / 4299
Катіонні глікопротеїни є основними компонентами слини привушних залоз. Які амінокислоти обумовлюють їх позитивний заряд?

Цистеїн, гліцин, пролін

Лізин, аргінін, гістидин

Аспартат, глутамат, гліцин

Глутамат, валін, лейцин

Аспартат, аргінін, глутамат

1454 / 4299
З віком знижується секреторна активність привушних слинних залоз. Активність якого ферменту слини буде різко зменшуватись?

Лізоцим

Фосфатаза

Гексокіназа

Амілаза

Мальтаза

1455 / 4299
Для визначення антитоксичної функції печінки хворому призначено бен-зоат натрію, який в печінці перетворюється в гіпурову кислоту. Яка сполука використовується для цього процесу?

Гліцин

Цистеїн

ФАФС

УДФ-глюкуронова кислота

Метіонін

1456 / 4299
При спадковій оратацидурії виділення оротової кислоти в багато разів перевищує норму. Синтез яких речовин буде порушений при цій патології?

Піримідинові нуклеотиди

Сечова кислота

Сечовина

Пуринові нуклеотиди

Біогенні аміни

1457 / 4299
Чоловік на вулиці підняв двома руками електричний дріт, що був під високою напругою. Наступила миттєва смерть від:

Опіків

Зупинки дихання

Фібриляції серця

Крововтрати

Крововиливу у головний мозок

1458 / 4299
У ліквідатора наслідків аварії на АЕС, що отримав дозу опромінення 5 Гр, через тиждень у крові визначений агранулоцитоз. Який патогенетичний механізм є провідним у його виникненні?

Збільшений перехід гранулоцитів у тканини

Пригнічення лейкопоезу

Розвиток аутоімунного процесу

Збільшення руйнування лейкоцитів

Порушення виходу зрілих лейкоцитів з кісткового мозку

1459 / 4299
У хворого, якому було екстирповано зуб з приводу гострого гнійного періоститу, спостерігалася тривала кровотеча з комірки, яку не можна було спинити звичайними методами. У крові: ер.- 2,9 ■ 1012/л, НЬ- 90 г/л; КП- 0,9; тромб.- 60 ■ 109/л; лейк.- 52 ■ 109/л.; базофіли - 0, еозинофіли - 1%, моноцити - 0, нейтрофіли: юні - 0, паличкоядерні -2%, сегментоядерні -18%, лімфоцити -8%, моноцити -1%, мiєлобласти - 70%. Яке захворювання крові наявне у цього хворого?

Хронічний мієлоїдний лейкоз

Еритромієлоз

Промієлоцитарний лейкоз

Недиференційований лейкоз

Гострий мієлоїдний лейкоз

1460 / 4299
Жінка із токсикозом вагітності страждає на гіперсалівацію, що призводить до втрати 3-4 літрів слини щоденно. Яке порушення водно-сольового обміну виникає при цьому?

Гіпонатріємія

Гіпогідратація ізоосмолярна

Гіпокаліємія

Гіпогідратація гіпоосмолярна

Гіпогідратація гіперосмолярна

1461 / 4299
Хвора 23-х років скаржиться на виражену слабкість, сонливість, потемніння в очах, запаморочення, спотворення смаку. В анамнезі менорагії. Об’єктивно: блідість шкірних покривів, тріщини в кутах рота, розшаровані нігті, збільшення ЧД і ЧСС. У крові: ер.-2,8 • 1012/л, Hb- 70 г/л, КП- 0,75. Яка гіпоксія, найбільш імовірно, призвела до розвитку виявлених симптомів у хворої?

Гемічна

Субстратна

Респіраторна

Тканинна

Циркуляторна

1462 / 4299
У вагітної жінки розвинувся токсикоз з важкими повторними блюваннями упродовж доби. До кінця доби почали проявлятися тетанічні судоми і зневоднення організму. Який зсув кислотно-лужної рівноваги викликав зазначені зміни?

Газовий ацидоз

Негазовий видільний алкалоз

Негазовий метаболічний ацидоз

Негазовий видільний ацидоз

Газовий алкалоз

1463 / 4299
В аналізі крові хворого: ер.- 3,0 • 1012/л; Hb- 90 г/л; ретикулоцити - 0,5%. В мазку: пойкілоцити, гіпохромні еритроцити. Залізо сироватки крові - 80 мкмоль/л. Для якої патології це характерно?

B12-дефіцитна анемія

Серпоподібноклітинна анемія

Залізорефрактерна анемія

Залізодефіцитна анемія

Хвороба Мінковського-Шофара

1464 / 4299
Хворого доставили до клініки у коматозному стані. В анамнезі цукровий діабет II типу впродовж 5-ти років. Об’єктивно: дихання шумне, глибоке, у видихуваному повітрі чути запах ацетону. Вміст глюкози у крові 15,2 ммоль/л, кетонових тіл - 100 мкмоль/л. Для якого ускладнення цукрового діабету характерні такі клінічні прояви?

Гіперосмолярна кома

Гіперглікемічна кома

Кетоацидотична кома

Гіпоглікемічна кома

Печінкова кома

1465 / 4299
При мікроскопічному дослідження пунктату з осередку запалення у хворого із абсцесом шкіри знайдено велику кількість різних клітин крові. Які з цих клітин першими надходять із судин до тканин при запаленні?

Нейтрофіли

Моноцити

Базофіли

Еозинофіли

Лімфоцити

1466 / 4299
Хвора 18-ти років скаржиться на загальну слабкість, швидку втомлюваність, пригнічений настрій. Має астенічний тип конституції. Ps- 68/хв., АТ-90/60 мм рт.ст. Встановлена первинна нейроциркуляторна артеріальна гіпотензія. Що є провідним фактором зниження артеріального тиску у хворої?

Депонування крові в венах великого кола кровообігу

Зменшення хвилинного об’єму крові

Зменшення серцевого викиду

Гіповолемія

Зниження тонусу резистивних судин

1467 / 4299
Обстежуючи ротову порожнину хворого, стоматолог звернув увагу на наявність запальнодистрофічного процесу в слизовій оболонці (гунтеров-ський глосит, атрофічний стоматит). Аналіз крові виявив гіперхромну анемію. Який фактор є причиною цього захворювання?

Гіповітаміноз B12

Гіповітаміноз A

Підвищення кислотності шлункового соку

Гіповітаміноз B1

Гіповітаміноз B6

1468 / 4299
У хворого діагностовано хронічний гломерулонефрит. Внаслідок значних склеротичних змін маса функціонуючих нефронів зменшилася до 10%. Яке з перерахованих нижче порушень лежить в основі розвитку в хворого уремічного синдрому?

Порушення осмотичного гомеостазу

Порушення водного гомеостазу

Азотемія

Ниркова остеодистрофія

Артеріальна гіпертензія

1469 / 4299
У хворої жінки 29-ти років місяце-подібне обличчя, ожиріння верхньої частини тулуба, стриї на передній черевній стінці, гірсутизм, у сечі підвищено рівень 17- оксикетостероїдів. Вищезазначені прояви характерні для захворювання:

Вторинний гіперальдостеронізм

Первинний гіпоальдостеронізм

Синдром Іценко-Кушінга

Синдром Кона

Феохромоцитома

1470 / 4299
На розтині трупа жінки 52-х років, яка тривалий час хворіла на хронічний гломерулонефрит, виявлено: значно зменшені, щільні, дрібнозернисті нирки, фібринозне запалення серозних і слизових оболонок, дистрофічні зміни паренхіматозних органів, набряк головного мозку. Яким ускладненням зумовлені описані зміни серозних оболонок і внутрішніх органів?

Сепсис

ДВЗ -синдром

Тромбоцитопенія

Анемія

Уремія

1471 / 4299
Робітник хімічної промисловості звернувся до лікаря зі скаргою на стирання емалі. Об’єктивно: розповсюджене руйнування коронок зубів з утворенням замісного дентину. Який діагноз найбільш вірогідний?

Ерозія зубів

Некроз твердих тканин зубів

Флюороз

Середній карієс

Клиноподібні дефекти

1472 / 4299
Жінка 35-ти років звернулася до лікаря зі скаргами на наявність пухлинного утворення на слизовій оболонці ясни у ділянці верхніх різців. Під час огляду лікар на слизовій оболонці ясни виявив вузлик 1 см у діаметрі, м’якої консистенції, рожевого кольору, що кровить під час дотику. Про який процес йдеться у даному випадку?

Хронічний гінгівіт

Гігантоклітинний епулід

Ангіоматозний епулід

Пухлина

Фіброзний епулід

1473 / 4299
При гістологічному дослідженні стінки бронха і прилеглих ділянок легені виявлені пласти і тяжі атипового плоского епітелію. У клітинах - помірно виражені ознаки атипізму: поліморфізм, гіперхромія ядер, мітози. У центрі комплексів концентричні утворення рожевого кольору. Який найбільш імовірний діагноз?

Недиференційований рак

Аденокарцинома

Плоскоклітинний рак без ороговіння

Скірозний рак

Плоскоклітинний рак з ороговінням

1474 / 4299
При гістологічному дослідженні стінки кісти, що локалізується у ділянці верхньої щелепи, встановлено, що стінка кісти зсередини вистелена багатошаровим плоским епітелієм з підлеглою грануляційною тканиною з лім-фолейкоцитарною інфільтрацією. Зовнішній шар представлений пухкою волокнистою сполучною тканиною, оточеною рубцевою фіброзною тканиною. Ці дані є підставою для встановлення такого діагнозу:

Кератокіста

Амелобластома

Кістогранульома

Епітеліальна гранульома

Проста гранульома

1475 / 4299
При обстеженні у хворого на твердому піднебінні виявлена пухлина у вигляді невеликого щільного вузла сірого кольору без чітких меж. Після видалення пухлина досліджена гістологічно. Побудована з дрібних, кубічної форми клітин з гіперхромним ядром, які формують альвеоли, трабекули, солідні та крі6розні структури. Ріст пухлини - інвазивний. Назвіть пухлину:

Аденокістозна карцинома

Злоякісна плеоморфна аденома

Аденолімфома

Мономорфна аденома

Мукоепідермоїдний рак

1476 / 4299
У хворої, що страждала на вторинний сифіліс, з’явилися вогнища депігментації шкіри у верхніх відділах спини. Назвіть патологічний процес у ттткі-рі:

Лейкодерма

Лейкоплакія

Дисплазія

Метаплазія

Паракератоз

1477 / 4299
При гістологічному дослідженні збільшеного шийного лімфатичного вузла відмічено, що загальний малюнок вузла нечіткий за рахунок розростання атипових гістіоцитарних клітин, з наявністю гігантських клітин Б ерезовського-Штернберга, маються ділянки некрозу, склерозу, гіалінозу. Для якого захворювання характерні виявлені морфологічні зміни в лімфатичному вузлі?

Хронічний мієлолейкоз

Гострий мієлолейкоз

Грибоподібний мікоз

Туберкульоз

Лімфогранулематоз

1478 / 4299
На розтині тіла померлого чоловіка 70-ти років серце зменшене у розмірах, жирова клітковина під епікардом відсутня, міокард щільний, бурого кольору. При мікроскопії - кардіоміоцити зменшені у розмірах, в саркоплазмі багато гранул бурого пігменту ліпофусцину. Встановіть характер патологічного процесу в міокарді:

Жирова дистрофія

Бура атрофія

Гемохроматоз

Амілоїдоз

Гіпертрофія

1479 / 4299
У чоловіка 70-ти років, який помер від серцевої недостатності, під час розтину тіла виявлено деформовані, звужені коронарні артерії. На розрізі внутрішня поверхня артерій горбиста, стінка білувата, ламка, кам’янистої щільності. Про яку стадію атеросклерозу йдеться?

Атерокальциноз

Атероматоз

Ліпосклероз

Виразкування

Ліпоїдоз

1480 / 4299
В сироватці крові новонародженого знайдено антитіла до вірусу кору. Про наявність якого імунітету це може свідчити?

Природний пасивний

Спадковий

Природний активний

Штучний активний

Штучний пасивний

1481 / 4299
До стоматолога звернулась пацієнтка (доярка) із висипанням на слизовій оболонці порожнини рота у вигляді афт. Лікар виявив висипання на шкірі довкола нігтьових пластинок на руках. Який збудник спричинив дану хворобу?

Цитомегаловірус

Вірус ящуру

Герпесвірус

Вірус Коксакі В

Вірус везикулярного стоматиту

1482 / 4299
У чоловіка, мешканця сільської місцевості, в шийно-щелепній ділянці виявлено твердий флегмоноподі-бний інфільтрат, шкіра навколо синьо-багряного кольору. В центрі інфільтрат некротизований, з виразки виділяється гній з неприємним запахом. Для підтвердження діагнозу актиномікоз шийно-щелепної ділянки здійснено мікроскопічне дослідження гною. Що повинен виявити бактеріолог для підтвердження діагнозу?

Грамнегативні диплобактерії

Грамнегативні диплококи

Друзи

Грампозитивні стрептококи

Кислотостійкі палички

1483 / 4299
Бактеріолог при дослідженні крові та слизу із носоглотки дотримувався певних заходів щодо збереження збудників у матеріалі. При бактеріоскопі-чному дослідженні встановлено наявність грамнегативних коків, які нагадують кавові зерна і розташовані парами, або тетрадами. Назвіть збудника, який був ізольований бактеріологом:

Acinetobacter calcoaceticus

Moraxella lacunata

Staphilococcus aureus

Neisseria meningitidis

Neisseria gonorrhoeae

1484 / 4299
Хворому з підозрою на черевний тиф лікар-інфекціоніст призначив бактеріологічне дослідження крові. Доцільність цього призначення пояснюється тим, що на першому тижні захворювання тифо-паратифами спостерігається:

Септикопіємія

Вірусемія

Бактеріємія

Токсинемія

Септицемія

1485 / 4299
Від хворого з підозрою на черевний тиф виділено чисту культуру збудника, яку ідентифіковано за морфологічними, культуральними та біохімічними властивостями як сальмонела тифу. Яке дослідження слід застосувати для остаточної ідентифікації збудника?

Серодіагностика

Сероідентифікація

Фаготипування

Алергодіагностика

Антибіотикограма

1486 / 4299
Хворому з афтами слизової оболонки призначили препарат, діючим агентом котрого є галоген, а також поверхнево активна речовина, що має дезін-фікуючу, дезодоруючу дію. Застосовується для дезінфекції неметалевого інструментарію, рук, предметів догляду за інфекційними хворими. Як антисептик використовується для лікування інфікованих ран, слизової оболонки ротової порожнини, патологічних зубоя-сеневих кишень, дезінфекції кореневих каналів. Визначте препарат:

Перекис водню

Кислота борна

Хлоргексидину біглюконат

Калію перманганат

Діамантовий зелений

1487 / 4299
При введенні хворому з метою проведення провідникової анестезії засобу, який використовується в хірургічній стоматології, виникли симптоми отруєння: збудження ЦНС з наступним паралічем, гостра серцево-судинна недостатність (колапс), в патогенезі якого має значення сенсибілізація до даного препарату. Виникли також алергі- чні реакції (свербіж, набряклість, еритема). Визначте препарат:

Тубокурарину хлорид

Тіопентал-натрію

Ардуан

Лідокаїн

Дитилін

1488 / 4299
В комплексному лікуванні гінгівіту хворому призначили препарат, який за хімічною будовою відноситься до похідних піримідину, стимулює лейкопоез, прискорює загоєння ран, підсилює ріст та розмноження клітин (процеси проліферації), виявляє протизапальну дію. Застосовується при лейкопеніях різного генезу, в стоматологічній практиці при запальних захворюваннях слизової оболонки ротової порожнини. Визначте препарат:

Коамід

Ціанокобаламін

Меркаптопурин

Метотрексат

Метилурацил

1489 / 4299
У хворої з артеріальною гіпертензією розвинувся напад бронхіальної астми. Який з нижче перерахованих бронхолітичних засобів може спровокувати гіпертонічний криз?

Беротек

Еуфілін

Ефедрину гідрохлорид

Сальбутамол

Ьадрин

1490 / 4299
Для корекції артеріального тиску при колаптоїдному стані хворому було введено мезатон. Який механізм дії даного препарату?

Блокує в-адренорецептори

Стимулює а- в-адренорецептори

Блокує а-адренорецептори

Стимулює а-адренорецептори

Стимулює в-адренорецептори

1491 / 4299
Після повторного оперативного втручання з використанням того ж самого загального анестетика у хворого виникло гостре ураження печінки (гепатит). Який загальний анестетик найбільш імовірно може викликати дану патологію?

Пропанідид

Азоту закис

Ефір

Фторотан

Тіопентал натрію

1492 / 4299
На прийомі у стоматолога у хворого виник гострий біль за грудниною, який іррадіює в ліву руку і плече; відчуття нестачі повітря, страх. Яка з запропонованих речовин найбільш адекватна в цьому випадку?

Діазепам

Нітрогліцерин

Парацетамол

Нітросорбід

Анальгін

1493 / 4299
Під час оперативного втручання анестезіолог для керованої гіпотонії застосував гангліоблокуючий засіб. Який препарат було призначено хворому в даному випадку?

Бензогексоній

Пахікарпін

Пентамін

Гігроній

Пірілен

1494 / 4299
До поліклініки звернувся хворий зі скаргами на біль за грудниною, зади-тттку і серцебиття. Після обстеження лікар діагностував у хворого ІХС і призначив верапаміл. Який механізм дії даного препарату?

Блокує а-адренорецептори

Блокує в-адренорецептори

Блокує калієві канали

Блокує кальцієві канали

Блокує натрієві канали

1495 / 4299
Хворій потрібно зробити пункцію прямокишково-маткової заглибини (Дугласового простору). Через яку частину склепіння піхви її здійснюють?

Бічна права

Задня

Бічна ліва

Будь-яка

Передня

1496 / 4299
Для попередження сезонного підйому захворюваності на грип у лікувальних закладах міста санепідстанція зобов’язала провести імунізацію медпрацівників. Яким із перелічених препаратів слід проводити імунізацію?

Гамма-глобулін

Інтерферон

Ремантадин

Амантадин

Субодинична вакцина

1497 / 4299
У хворого на шкірі живота, грудної клітки з’явився висип у вигляді розеол і петехій, дрібноточковий кон’юн-ктивальний висип. Смерть настала при явищах ураження мозку. При мікроскопічному дослідженні секційного матеріалу у ЦНС (довгастий мозок, міст), у шкірі і нирках, міокарді виявлений деструктивно-проліферативний ендо-тромбоваскуліт. Про яке захворювання слід думати?

Висипний тиф

Сепсис

Бруцельоз

Вузликовий періартеріїт

Системний червоний вівчак

1498 / 4299
В хірургічне відділення доставлено чоловіка 35-ти років з гнійною раною на шиї попереду трахеї (в ділянці пере-двісцерального простору). Куди може розповсюдитись інфекція, якщо хворому терміново не зробити операцію?

В надгрудинний міжапоневротичний простір

В грудну порожнину - в середнє середостіння

В ретровісцеральний простір

В грудну порожнину - переднє середостіння

В грудну порожнину - в заднє середостіння

1499 / 4299
У деяких клінічно здорових людей в умовах високогір’я виявляються ознаки анемії. У крові в них виявляють серпоподібні еритроцити. Генотип цих людей:

Аа

аа

АА

ХСХс

ХсХс

1500 / 4299
У пацієнта з хронічною серцевою недостатністю виявлене збільшення в’язкості крові. При капіляроскопії виявлено пошкодження стінок судин мікроциркуляторного русла. Яке з перелічених порушень можливе у даному випадку?

Тромбоз

Артеріальна гіперемія

Емболія

Венозна гіпєрємія

Сладж-феномен

1501 / 4299
При обстеженні хворого 35-ти років проведено гістологічне дослідження пунктату червоного кісткового мозку і виявлено значне зменшення кількості мегакаріоцитів. До яких змін периферичної крові це призведе?

Тромбоцитоз

Агранулоцитоз

Лейкопенія

Лейкоцитоз

Тромбоцитопенія

1502 / 4299
Під час проведення трахеотомії у хворого 45-ти років, який потрапив до реанімаційного відділення лікарні з набряком гортані, було випадково перерізано яремну венозну дугу, яка лежить у:

Spatium interscalenum

Spatium retropharyngeale

Spatium antescalenum

Spatium interaponeuroticum suprasternale

Spatium pretracheale

1503 / 4299
Під час виконання фізичного навантаження людина менш чутлива до болю. Причиною цього є активація в організмі:

Функції щитоподібної залози

Ноцицептивної системи

Симпато-адреналової системи

Функції наднирників

Антиноцицептивної системи

1504 / 4299
У жінки з резус-негативною кров’ю А (II) групи народилася дитина з АВ (IV) групою, у якої діагностували гемолітичну хворобу внаслідок резус-конфлікту. Яка група крові можлива у батька дитини?

III (В), резус-негативна

III (В), резус-позитивна

IV (АВ), резус-негативна

I (0), резус-позитивна

II (А), резус-позитивна

1505 / 4299
До реанімаційного відділення госпіталізовано хворого з клінікою отруєння нейротропною речовиною. До комплексу лікування включено дихальний аналептик з ноотропною дією. Який препарат призначено?

Амітриптилін

Камфора

Теофілін

Сульфокамфокаїн

Етимізол

1506 / 4299
У хворого з’явилися жовтушність шкіри, склер та слизових оболонок. У плазмі крові підвищений рівень загального білірубіну, в калі - стеркобіліну, в сечі - уробіліну. Який вид жовтяниці у хворого?

Хвороба Жільбера

Холестатична

Гемолітична

Паренхіматозна

Обтураційна

1507 / 4299
У хворого внаслідок черепно-мозкової травми з’явилась значна поліурія, що не супроводжувалася глюкозурією. Пошкодження якої структури може призвести до подібних змін?

Мозкова речовина наднирників

Передня частка гіпофізу

Підшлункова залоза

Кора наднирників

Задня частка гіпофізу

1508 / 4299
У дитини 12-ти років низький зріст при непропорційній будові тіла і розумовій відсталості. Недостатня секреція якого гормону може бути причиною таких порушень?

Інсулін

Глюкагон

Соматотропін

Тироксин

Кортизол

1509 / 4299
При дослідженні культури тканин злоякісної пухлини виявили поділ клітин, який відбувався без ахроматиново-го апарату шляхом утворення перетяжки ядра, при якому зберігались ядерна оболонка та ядерце. Який тип поділу клітин відбувався у злоякісній пухлині, що вивчалась?

Екзомітоз

Мітоз

Ендомітоз

Амітоз

Мейоз

1510 / 4299
Хворому проведено лобектомію правої середньої частки легені. Які сегменти легені були уражені?

Верхівковий, передній

Бічний та присередній

Верхівково-задній та передній

Задній і бічний основні

Присередній і передній основні

1511 / 4299
У чоловіка 60-ти років, який страждає на хронічну кишкову непрохідність, посилюється гниття білків у товстому кишечнику. Підтвердженням цього процесу є:

Глюкозурія

Гіперурікурія

!ндиканурія

Білірубінурія

Креатинурія

1512 / 4299
У 19-ти місячної дитини із затримкою розвитку та проявами самоагре-сії вміст сечової кислоти в крові - 1,96 ммоль/л. При якому метаболічному порушенні це спостерігається?

Синдром Леша-Ніхана

Синдром набутого імунодефіциту

Подагра

Хвороба Гірке

Хвороба Іценко-Кушінга

1513 / 4299
Хворий у непритомному стані доставлений швидкою допомогою до лікарні. Об’єктивно: рефлекси відсутні, періодично з’являються судоми, дихання нерівномірне. Після лабораторного обстеження було діагностовано печінкову кому. Нагромадження якого метаболіту є суттєвим для появи розладів центральної нервової системи?

Гістамін

Білірубін

Амоніак

Глутамін

Сечовина

1514 / 4299
Під час гістологічного дослідження нирок жінки 25-ти років, що померла під час пологів, в епітелії канальців головних відділів нефрону виявлені конденсація хроматину ядер, розпад їх на глибки та лізис, а також плазморексис та цитоліз. Який патологічний процес виявлений в епітелії канальців нирок?

Гіаліноз

Гідропічна дистрофія

Жирова дистрофія

Амілоїдоз

Некроз

1515 / 4299
У хворого на діарею небактеріаль-ного генезу скарги на часту дефекацію. Який з перелічених засобів, механізм протидіарейної дії якого реалізується через опіатні рецептори кишок, доцільно використати?

Но-шпа

Лінекс

Атропіну сульфат

Де-нол

Імодіум (лоперамід)

1516 / 4299
У хворого, що надійшов до інфекційного відділення зі скаргами на судомне скорочення м’язів обличчя, з садна правої нижньої кінцівки були виділені бактерії з термінальним розташуванням спор, що надає їм вигляд 'барабанних паличок'. Яким бактеріям притаманні дані властивості?

Bacillus cereus

Bacillus anthracis

Clostridium botulinum

Clostridium tetani

Clostridium perfringens

1517 / 4299
Встановлено, що швидкість проведення збудження нервовими волокнами становить 120 м/сек. Зазначені волокна є:

Постгангліонарними парасимпатичними

Прегангліонарними парасимпатичними

Прегангліонарними симпатичними

Аксонами мотонейронів

Постгангліонарними симпатичними

1518 / 4299
У дитини фарингіт (запалення слизової оболонки глотки) ускладнився запаленням середнього вуха. Через яке анатомічне утворення сталося розповсюдження запального процесу на середнє вухо?

Каналець барабанної струни

М ’язово-трубний канал

Півканал м’яза-натягувача барабанної перетинки

Півканал слухової труби

Глотковий отвір слухової труби

1519 / 4299
Під час бійки у чоловіка виникла рефлекторна зупинка серця внаслідок сильного удару у верхню ділянку передньої черевної стінки. На яку структуру серця впливають еферентні нерви, що зумовили його зупинку?

Робочий міокард передсердь

Провідна система шлуночків серця

Синоатріальний вузол

Атріовентрикулярний вузол

Робочий мюкард шлуночків

1520 / 4299
У хворого спостерігаються геморагії, в крові знижена концентрація протромбіну. Недостатність якого вітаміну призвела до порушення синтезу цього фактору згортання?

C

D

E

A

K

1521 / 4299
У хворого, що харчувався винятково полірованим рисом, недостатність тіаміну стала причиною поліневриту. Екскреція якої сполуки з сечею може бути індикатором цього авітамінозу?

Сечова кислота

Піровиноградна кислота

Метил-малонова кислота

Малат

Фенілпіруват

1522 / 4299
Хворому на ревматоїдний артрит для попередження можливого негативного впливу на слизову шлунка призначили препарат із групи нестероїдних протизапальних засобів - селективний інгібітор ЦОГ-2. Вкажіть препарат:

Бутадіон

Анальгін

Целекоксиб

Ацетилсаліцилова кислота

Ібупрофен

1523 / 4299
Хворий на стенокардію приймає ацетилсаліцилову кислоту в дозі 100 мг на добу. З якою метою застосовується ацетилсаліцилова кислота у даного хворого?

Для зниження вмісту холестерину

Для пригнічення агрегації тромбоцитів

Для пригнічення зсідання крові

Для зниження рівня протромбіну

Для розширення коронарних судин

1524 / 4299
Пацієнт скаржиться на свербіж шкіри, особливо між пальцями рук, у пахвинних западинах, на нижній частині живота. При огляді в цих ділянках шкіри виявлено маленькі пухирці. Під час лабораторної діагностики встановлено, що причиною цього стану є представник членистоногих. Вкажіть назву хвороби, спричинену цим членистоногим:

Дерматотропний лейшманіоз

Педикульоз

Демодекоз

Короста

Міаз

1525 / 4299
У людини внаслідок втрати 1,5 л крові різко зменшився діурез. Посилена секреція якого гормону, перш за все, спричинила зміни діурезу?

Окситоцин

Вазопресин

Паратгормон

Кортизон

Натрійуретичний

1526 / 4299
При обстеженні хворого на атрофічний гастрит виявлено мегалобла-стну анемію. Дефіцит якої речовини є причиною виникнення анемії у цього хворого?

Гастромукопротеїд

Вітамін В6

Залізо

Вітамін Ві

Еритропоетини

1527 / 4299
Після відкривання рота відбувається його рефлекторне закривання. З яких рецепторів починається зазначений рефлекс?

Механорецептори слизової ротової порожнини

Пропріорецептори м’язів, що опускають нижню щелепу

Рецептори періодонту

Смакові рецептори

Пропріорецептори м’язів, що піднімають нижню щелепу

1528 / 4299
У дитини 2-х років діагностовано хворобу Гірке, яка проявляється важкою гіпоглікемією. Причиною такого стану є відсутність ферменту глюкозо-6-фосфатази. З порушенням якого процесу пов’язана ця патологія?

Синтез глікогену

Кетогенез

Гліколіз

Ілюконеогенез

Мобілізація глікогену

1529 / 4299
Хворому на хронічний гастрит зроблена внутрішньошлункова рН-метрія, за допомогою якої встановлено змен- шення кислотності шлункового соку. Функція яких клітин знижена?

Парiєтальнi екзокриноцити

Ендокриноцити

Головні екзокриноцити

Шийкові клітини

Додаткові клітини

1530 / 4299
Бригада швидкої допомоги була викликана до хворого 48-ми років із приводу інфаркту міокарда. У комплексну терапію було включено анальге-тик, що застосовують для нейролепта-налгезії. Вкажіть найбільш доцільний швидкодіючий препарат:

Промедол

Аспірин

Фентаніл

Морфіну гідрохлорид

!ндометацин

1531 / 4299
Відомо, що при цукровому діабеті у хворих частіше зустрічаються запальні процеси, знижена регенерація, уповільнюється загоєння ран. Причиною цього є:

Підвищення ліполізу

Зниження ліполізу

Зниження протеосинтезу

Прискорення глюконеогенезу

Посилення катаболізму

1532 / 4299
Жінка звернулася до лікаря зі скаргами на утруднення рухів язика. Обстеження головного мозку за допомогою ЯМР показало, що у хворої є крововилив у нижньому відділі довгастого мозку. Про пошкодження якого ядра довгастого мозку можна думати?

Нижнього слиновидільного ядра

Подвійного ядра

Ядра додаткового нерва

Ядра під’язикового нерва

Поодинокого ядра

1533 / 4299
При токсичному ушкодженні гепа-тоцитів з порушенням їх білковосинте-зуючої функції у пацієнта різко знизився вміст альбумінів у плазмі крові та онкотичний тиск плазми. Що буде наслідком цих змін?

Зменшення діурезу

Поява набряків

Збільшення об’єму циркулюючої крові

Збільшення густини крові

Зменшення ШОЕ

1534 / 4299
У хворого діагностовано грип. Після прийому антипіретиків стан його різко погіршився: свідомість потьмарена, АТ- 80/50 мм рт.ст., Ps- 140/хв, температура тіла різко знизилась до 35,8oC. Яке ускладнення виникло у даного хворого?

Гіпертермія

Гіповолемія

Алкалоз

Колапс

Ацидоз

1535 / 4299
У хворого лікар виявив накопичення рідини в плевральній порожнині справа над куполом діафрагми. У якому анатомічному утворі накопичилась рідина?

Реброво-середостінний синус

Діафрагмально-середостінний синус

Реброво-діафрагмальний лівий синус

Реброво-діафрагмальний правий синус

1536 / 4299
Пацієнт звернувся до лікаря з приводу того, що він втратив здатність розрізняти смаки на корені язика. Лікар встановив, що це пов’язано з ураженням нерва. Якого саме?

Блукаючий

Лицьовий

Верхньогортанний

Язикоглотковий

Трійчастий

1537 / 4299
Введення знеболюючого пацієнту перед екстракцією зуба призвело до розвитку анафілактичного шоку, який супроводжувався розвитком олігурії. Який патогенетичний механізм зумовив зменшення діурезу в даній клінічній ситуації?

Збільшення онкотичного тиску плазми крові

Підвищення гідростатичного тиску в капсулі Шумлянського-Боумена

Збільшення секреції вазопресину

Пошкодження клубочкового фільтру

Зниження гідростатичного тиску в капілярах ниркових тілець

1538 / 4299
При втручанні з метою лікування вивиху нижньої щелепи лікар повинен пам’ятати про м’яз, який при скороченні відтягує назовні капсулу і суглобовий диск скроневонижньощелепного суглоба. Який це м’яз?

M. pterygoideus medialis

M. temporalis

M. mylohyoideus

M. pterygoideus lateralis

M. masseter

1539 / 4299
Одним із методів зняття гострого болю при невралгії трійчастого нерва є створення депо анестетика в місці виходу гілок трійчастого нерва. В якій ділянці потрібно ввести анестетик для знечулення першої гілки трійчастого нерва?

Ділянка надперенісся

Підочноямковий край

Орбітальний край виличної кістки

Ділянка лобного відростка верхньої щелепи

Надочноямковий край

1540 / 4299
При макроскопічному дослідженні зуба у пульпі визначається маса сіро-чорного кольору з гнильним, смердючим запахом. Мікроскопічно пульпа являє собою безструктурні зернисті маси з великою кількістю мікробів. Яка патологія розвинулась у пульпі зуба?

Гнійне запалення

Гангрена

Серозне запалення

Скупчення гемосидерину

!нфаркт

1541 / 4299
У чоловіка 71-го року впродовж 10 днів спостерігався пронос з домішками у калі слизу та крові. Хворого було госпіталізовано у тяжкому стані, помер через 2 доби. При розтині тіла померлого виявлено: дифтеритичний коліт з множинними виразками неправильної форми різної глибини в сигмоподібній і прямій кишках. При бактеріологічному дослідженні висіяно шигели. Яке основне захворювання у хворого?

Черевний тиф

Неспецифічний виразковий коліт

Сальмонельоз

!єрсиніоз

Дизентерія

1542 / 4299
При мікроскопічному дослідженні біоптата з товстої кишки виявлена пухлина з призматичного епітелію, що формує атипові залозисті структури різної форми і величини. Базальна мембрана залоз зруйнована. Клітки пухлини поліморфні, ядра гіперхромні, відзначається велика кількість патологічних мітозів. Який діагноз найбільш вірогідний?

Аденокарцинома

Солідний рак

Слизовий рак

Базальноклітинний рак

Недиференційований рак

1543 / 4299
У пацієнта на слизовій оболонці ротової порожнини, носа та губах з’явились везикулярні пухирці. Стоматолог запідозрив везикулярний стоматит. Яке дослідження надасть можливість підтвердити діагноз?

Зараження тварин везикулярною рідиною

Постановка алергічної проби

Виділення бактерій з везикулярної рідини

Виділення вірусу з везикулярної рідини

Мікроскопія везикулярної рідини

1544 / 4299
Під час операції на щелепно-лицьовій ділянці для зниження секреції слинних залоз хворому ввели атропіну сульфат. Виникли тахікардія, сухість та гіперемія шкіри, параліч акомодації та підвищення внутрішньоочного тиску. Який антагоніст доцільно використати в даному випадку?

Скополаміну гідробромід

Прозерин

Дипіридамол

Атенолол

Платифіліну гідротартрат

1545 / 4299
У пацієнта 42-х років, що страждає на парадонтоз, у коронковій частині пульпи виявлено округлі звапновані утворення діаметром 2-3 мм. Назвіть ці структури:

Інтерглобулярні простори

Склерозований (прозорий) дентин

Мертвий дентин

Дентиклі

Зубні камені

1546 / 4299
При остеолатеризмі зменшується міцність колагену, що зумовлена помітним зменшенням утворення поперечних зшивок у колагенових фібрилах. Причиною цього явища є зниження активності такого ферменту:

Лізілгідроксилаза

Моноаміноксидаза

Колагеназа

Пролілгідроксилаза

Лізілоксидаза

1547 / 4299
Для морфологічного дослідження представлена ендокринна залоза, паренхіма якої складається з єпітєлію та нервової тканини. В епітеліальних тра-бекулах виявляється 2 типи клітин: хро-мофільні та хромофобні. Визначте даний орган:

Паращитоподібна залоза

Гіпоталамус

Щитоподібна залоза

Гіпофіз

Надниркова залоза

1548 / 4299
До приймального відділення надійшов хворий з ознаками гострої серцевої недостатності: блідість, акроціаноз, часте, поверхневе дихання. Який з перелічених засобів показаний у цьому випадку?

Корглікон

Нітрогліцерин

Адреналіну гідрохлорид

Дигітоксин

Кордіамін

1549 / 4299
У чоловіка 60-ти років спостерігається послаблення перистальтики кі-шечника. Який з перерахованих видів їжі буде стимулювати перистальтику в найбільшій мірі?

Сало

Чай

Чорний хліб

Білий хліб

М’ясо

1550 / 4299
Під час цитогенетичного обстеження пацієнта з порушеною репродуктивною функцією виявлено в деяких клітинах нормальний каріотип 46,ХУ але у більшості клітин каріотип синдрому Клайнфельтера - 47,ХХУ Яку назву носить таке явище неоднорідності клітин?

Мозаїцизм

Транспозиція

Інверсія

Дуплікація

Мономорфізм

1551 / 4299
У чоловіка і його сина інтенсивно росте волосся по краю вушних раковин. Це явище спостерігалося також у батька і дідуся за батьківською лінією. Який тип успадкування зумовлює це?

Аутосомно-домінантний

Рецесивний, зчеплений з Х- хромосомою

Аутосомно-рецесивний

Зчеплений з Y-хромосомою

Домінантний, зчеплений з Х-хромосомою

1552 / 4299
Сечокам’яна хвороба ускладнилася виходом конкременту з нирки. На якому рівні сечоводу, найімовірніше, він може зупинитися?

На межі черевної та тазової частин

У нирковій мисці

В середній черевній частині

На 5 см вище тазової частини

На 2 см вище впадіння в сечовий міхур

1553 / 4299
В гістологічному препараті нижньої щелепи ембріону виявляється зубний зачаток, в якому зубний сосочок утворений дрібними зірчастими базо-фільно забарвленими клітинами. Яка тканина утворює цю частину зубного зачатку?

Мезенхіма

Ретикулярна

Хрящова

Епітеліальна

Кісткова

1554 / 4299
Хворому 25-ти років з клінічною картиною нефротичного синдрому проведено пункційну біопсію нирки. Під час мікроскопічного дослідження клітини епітелію проксимальних канальців нефрону збільшені в об’ємі, у цитоплазмі вакуолі з прозорою рідиною, ядро зміщене до периферії. Яка дистрофія виявлена в епітелії каналь-ців?

Зерниста

Гідропічна

Рогова

Жирова

Гіаліново-крапельна

1555 / 4299
За професійними показаннями проведена вакцинація лікарів-стоматологів. Вакцина має захищати їх від вірусної інфекції, збудник якої може бути присутній у крові стоматологічних хворих, які перенесли інфекцію або є хронічними носіями. Яка вакцина була використана?

Генно-інженерний HBs-антиген

Субодинична грипозна вакцина

Жива корова вакцина

Антирабiчна вакцина

Інактивована вакцина проти гепатиту А

1556 / 4299
У хворого виявлено порушення секреторної функції піднижньощелепної слинної залози. Який нерв забезпечує її вегетативну іннервацію?

N. petrosus major

Chorda tympani

N. petrosus minor

N. mandibularis

N. auriculotemporalis

1557 / 4299
До лабораторії особливо небезпечних інфекцій доставлено матеріал хворого з підозрою на холеру. Який метод експрес діагностики може підтвердити цей діагноз?

Р!Ф

PЗК

РП

РА

РГА

1558 / 4299
Хворому на остєомієліт нижньої щелепи був призначений антибіотик тетрациклінової групи. Вкажіть цей препарат:

Амікацин

Доксицикліну гідрохлорид

Оксацилін

Рифампіцин

Стрептоміцин

1559 / 4299
При обстеженні хворого похилого віку після тромбозу судин головного мозку виявлено моторну афазію. Де локалізований осередок пошкодження?

Кутова звивина

Прецентральна звивина

Звивина Гешля

Центр Брока

Центр Верніке

1560 / 4299
При аналізі ЕКГ пацієнта встановлено збільшення тривалості зубця P. З чим це пов’язано?

Прискорене проведення збудження через атріовентрикулярний вузол

Уповільнене розповсюдження збудження передсердями

Прискорене розповсюдження збудження передсердями

Прискорене розповсюдження збудження шлуночками

Уповільнене розповсюдження збудження шлуночками

1561 / 4299
У новонародженої дитини виявлено вроджені вади розвитку травної системи, що пов’язано з дією тератоген-них факторів на початку вагітності. На який зародковий листок подіяв терато-ген?

Ендодерма і мезодерма

Ектодерма

Мезодерма

Ендодерма

Усі листки

1562 / 4299
Жінка під час вагітності хворіла на краснуху. Дитина народилась з вадами розвитку - незрощення губи і піднебіння. Генотип у дитини нормальний. Ці аномалії розвитку є проявом:

Модифікаційної мінливості

Анеуплодії

Хромосомної мутації

Поліплоїдії

Комбінативної мінливості

1563 / 4299
Хворому 67-ми років з діагнозом: ішемічна хвороба серця, стенокардія напруги, був призначений лікарський препарат з групи блокаторів калієвих каналів. Який препарат був призначений хворому?

Амлодипін

Карбокромен

Аміодарон

Молсидомін

Дипіридамол

1564 / 4299
У жінки, що тривало приймала антибіотики з приводу кишкової інфекції, розвинулося ускладнення з боку слизової порожнини рота у вигляді запального процесу і білого нальоту, у якому при бактеріологічному дослідженні були виявлені дріжджеподібні грибки Candida albicans. Який з перерахованих препаратів показаний для лікування цього ускладнення?

Бісептол

Поліміксин

Флуконазол

Фуразолідон

Тетрациклін

1565 / 4299
У хворої після сильного психоемоційного навантаження розвинувся неспецифічний патологічний процес, що передбачає 3 стадії: реакцію тривоги, стадію резистентності, стадію виснаження. Як він називається?

Компенсація

Регенерація

Адаптація

Стрес

Парабіоз

1566 / 4299
Людина вийшла з кондиціоновано-го приміщення назовні, де немає вітру, температура повітря +38oC, вологість 64%. За рахунок якого механізму буде здійснюватися віддача тепла з організму за цих умов?

Кондукція

Конвекція

Проведення

Випаровування поту

Радіація

1567 / 4299
У хворого виражений больовий синдром при невралгії. Який засіб з не-стероїдних протизапальних препаратів зменшить болесприйняття?

Диклофенак-натрій

Дроперидол

Кодеїну фосфат

Лідокаїну гідрохлорид

Кетаміну гідрохлорид

1568 / 4299
Пацієнта на дачі вжалила бджола. Об’єктивно: кисть лівої руки гаряча, рожева, набрякла, у місці укусу великий червоний піхур. Який механізм є провідним у розвитку набряку?

Зниження кровонаповнення судин

Зниження осмотичного тиску тканинної рідини

Зниження онкотичного тиску тканинної рідини

Підвищення проникливості судин

Пошкодження судин при жаленні

1569 / 4299
У пацієнта 32-х років має місце гіповітаміноз B2. Причиною виникнення специфічних симптомів (ураження епітелію, слизових, шкіри, рогівки ока) найбільш імовірно є дефіцит:

Цитохрому в

Цитохрому а1

Цитохрому с

Флавінових коферментів

Цитохромоксидази

1570 / 4299
У хворого при кашлі виділяється іржаво-коричневе харкотиння, в якому виявляються овальні, золотаво-коричневі яйця розміром близько 0,1 мм. Який діагноз може бути поставлений у даному випадку?

Дикроцеліоз

Парагонімоз

Шистосомоз

Фасціольоз

Опісторхоз

1571 / 4299
У пробірку, що містить 0,3% розчин NaCl, додали краплю крові. Що відбудеться з еритроцитами?

Механічний гемоліз

Осмотичний гемоліз

Біологічний гемоліз

Змін не буде

Зморшкування

1572 / 4299
При гістологічному дослідженні мікропрепарату шкіри людини виявляється тільки щільна неоформлена сполучна тканина. Який шар даного органу був представлений для вивчення?

Сітчастий шар дерми

Епідерміс

Базальний шар епідермісу

Підшкірна жирова клітковина (гіподерма)

Сосочковий шар дерми

1573 / 4299
Після перенесеної травми голови у хворого бувають напади інтенсивного болю в ділянці обличчя і судоми жувальних м’язів. Який нерв найімовірніше травмований?

N. olphactorius

N. oculomotorius

N. facialis

N. abducens

N. trigeminus

1574 / 4299
Після обстеження хворого лікар рекомендував йому вилучити з раціону наваристі м’ясні та овочеві бульйони, прянощі, копчені продукти, оскільки у хворого було виявлено:

Зменшення секреції хлористоводневої кислоти залозами шлунка

Зменшення слиновиділення

Збільшення секреції хлористоводневої кислоти залозами шлунка

Зменшення моторики шлунково-кишкового тракту

Дискінєзія жовчних шляхів

1575 / 4299
Виявлення здатності до капсу-лоутворення у багатьох збудників є важливим при проведенні експрес-діагностики та складовим бактеріологічного дослідження. Для виявлення капсули досліджуваної культури в умовах лабораторії можна застосовувати фарбування за таким простим методом, як:

Буррі

Лєффлера

Дроботько

Романовського-Гімзи

Буррі-Гінса

1576 / 4299
Лікар призначив хворому з гострою серцевою недостатністю не-глікозидний кардіотонічний засіб, який безпосередньо стимулює ві -адренорецептори міокарда, збільшує кровообіг, діурез та застосовується лише внутрішньовенно крапельно внаслідок швидкої інактивації в організмі. Який препарат призначив лікар?

Анаприлін

Дигоксин

Адреналін

Добутамін

Корглікон

1577 / 4299
У дитини 9-ти років, що хворіла на грип, на 5-ту добу захворювання з’явилися сильний головний біль, нудота, запаморочення, менінгеальні знаки. Смерть настала через добу від набряку мозку, що наростав. Під час розтину порожнини черепа м’які мозкові оболонки набряклі, повнокровні, дифузно просякнуті яскраво-червоного кольору рідиною. Про яке ускладнення грипу слід думати?

Геморагічний менінгіт

Венозна гіперемія оболонок мозку

Серозний менінгіт

Крововилив в мозок

Гнійний лептоменінгіт

1578 / 4299
У дитини 3-х років мати звернула увагу на утруднене дихання і слизові виділення з носа, що з’явилися після контакту з квітами. При біопсійному дослідженні у слизовій оболонці носа вияв- лено набряк і наявність великої кількості еозинофільних лейкоцитів. Який патологічний стан розвинувся у дитини?

Клітинно-опосередкована реакція

Атопічна реакція

Недостатність імунної системи

Антитільна реакція

Імунокомплексна реакція

1579 / 4299
На розтині тіла чоловіка 47-ми років, що помер від легеневої кровотечі, у 2 сегменті правої легені виявлена порожнина округлої форми з нерівними краями розміром 5,5 см; внутрішня поверхня її вкрита щільнуватими жовтуватими масами, що переходять у тканину легені. При гістологічному дослідженні - внутрішній шар складається з розплавлених казеозних мас, некро-тизованої тканини легені. Який процес розвинувся в легені?

Гострий абсцес

Хронічний абсцес

Гострий кавернозний туберкульоз

Рак легені, що розпадається

Хронічний кавернозний туберкульоз

1580 / 4299
Циклічні зміни слизової оболонки матки обумовлені дією гормонів яєчника на судини матки. Атрофія жовтого тіла, яке продукує гормон прогестерон, призводить до спазму судин. Які це судини?

Судини підслизового шару міометрію

Спіралеподібні артерії ендометрію

Судини надсудинного шару міоме-трію

Прямі артерії ендометрію

Судини судинного шару міометрію

1581 / 4299
Хворий повернувся із Африки з високою температурою (39oC), жовтяницею. Стан погіршувався, хворий помер. На розтині тіла виявлена жовтяниця, збільшені в розмірі аспидно-сірі печінка та селезінка, наявність гемомелані-ну в кістковому мозку. Реакція Перлса позитивна. Визначте імовірну причину смерті:

Бластомікоз

Малярія

Опісторхоз

Амебіаз

Ехінококоз

1582 / 4299
У хворого на пієлонефрит сеча тимчасово втратила бактерицидність через пошкодження клітин, які знахо- дяться у збиральних трубочках нирок та забезпечують секреторну фазу се-чоутворення. Які це клітини?

Пласкі епітеліоцити

Темні клітини

Світлі клітини

Гладенькі міоцити

Облямовані епітеліоцити

1583 / 4299
Хворий 37-ми років доставлений до лікарні у тяжкому стані. Об’єктивно: виражена набряклість язика, гортані, губ, повік. Дихання утруднене. Обличчя ціанотичне. Відомо, що 2 години тому лікував зуби у стоматолога. Про який алергічний стан йдеться?

Сезонний риніт

Феномен Артюса

Бронхіальна астма

Кропив’янка

Набряк Квінке

1584 / 4299
Хворий звернувся до лікаря зі скаргами на періодичні висипання герпети-чних пухирців на лінії губ і на крилах носа. Такий стан спостерігається впродовж 10-ти років, кожний раз після зниження захисних сил організму. Лікар встановив діагноз: лабіальний герпес. Як називається така форма інфекції?

Гостра

Персистенція

Екзогенна

Затяжна

Латентна

1585 / 4299
Для зменшення зубного болю хворому рекомендували приймати ненар-котичний анальгетик. Який засіб був призначений?

Парацетамол

Анестезин

Індерал

Метацин

Фізостигміну саліцилат

1586 / 4299
Хворому, що страждає на алергічний хейліт, призначена мазь преднізолону для змазування червоної облямівки і слизової губ. До якої групи засобів відноситься цей препарат?

Глюкокортикоїди

Антагоністи лейкотрієнових рецепторів

Блокатори серотонінових рецепторів

Мембраностабілізатори

Блокатори гістамінових рецепторів

1587 / 4299
У дівчинки 8-ми років раптово підвищилась температура, з’явились катаральні явища дихальних шляхів. На 5-й день хвороби виник параліч м’язів нижніх кінцівок і приєднались дихальні розлади. В передніх рогах спинного мозку виявлена проліферація глії навколо загиблих нейронів. Про яке захворювання йдеться?

Менінгококцемія

Скарлатина

Дифтерія

Поліомієліт

Кір

1588 / 4299
Хворий 52-х років зі стійким підвищенням артеріального тиску до 200/110 мм рт.ст. помер при явищах хронічної серцево-судинної недостатності. На розтині: маса серця 600 г, товщина стінки лівого шлуночка 2,2 см, порожнини серця розширені. Гістологічно виражений гіаліноз і склероз артеріол міокарда. Для якого захворювання характерні описані зміни?

Гіпертонічна хвороба

Атеросклероз артерій серця

Дилятаційна кардіоміопатія

Ендоміокардіальний фіброз

Гіпертрофічна кардіоміопатія

1589 / 4299
У хворого під час гіпертонічного кризу виник геморагічний інсульт, внаслідок чого спостерігається відсутність довільних рухів, підвищення сухожильних рефлексів та тонусу м’язів лівих руки та ноги. Як називається таке порушення рухової функції?

Моноплегія

Параплегія

Млявий параліч

Геміплегія

Тетраплегія

1590 / 4299
Дитині 5-ти років був діагностований гінгівіт. Лікар-стоматолог обробив місця ураження галогеновим антисептиком, який має протимікробну, фунгіцидну дію. Який це лікарський засіб?

Протаргол

Розчин кислоти борної

Розчин цинку сульфату

Розчин фуразолідону

Розчин йоду спиртовий

1591 / 4299
При лікуванні хворого на спадкову форму імунодефіциту було застосовано метод генотерапії: ген ферменту був внесений у клітини пацієнта за до- помогою ретровірусу. Яка властивість генетичного коду дозволяє використовувати ретровіруси у якості векторів функціональних гєнів?

Бєзпєрєрвність

Універсальність

Специфічність

Колінеарність

Надмiрнiсть

1592 / 4299
Скєлєтні м’язи прикріплюються до кісток за допомогою сухожиль, які здатні витримувати велике силове навантаження. Яким видом сполучної тканини утворені сухожилля?

Пухка волокниста

Щільна оформлена

Ретикулярна

Щільна неоформлена

Хрящова

1593 / 4299
Хворий звернувся з відчуттям серцебиття після стресу. ЧСС- 104/хв, тривалість інтервалу P — Q - 0,12 сек, QRS - без змін. Який тип аритмії у хворого?

Екстрасистолія

Синусова тахікардія

Синусова брадикардія

Синусова аритмія

Миготлива аритмія

1594 / 4299
При огляді порожнини рота лікар виявив каріозну порожнину на поверхні першого малого кутнього зуба, зверненої до ікла. Як називається уражена поверхня коронки?

Facies occlusalis

Facies vestibularis

Facies mesialis

Facies distalis

Facies lingualis

1595 / 4299
У дитини 6-ти років спостерігається затримка росту, порушення процесів окостеніння, декальцифікація зубів. Що може бути причиною цього?

Авітаміноз С

Дефіцит вітаміну D

Зменшення продукції глюкагону

Гіпертиреоз

Інсулінова недостатність

1596 / 4299
У хворого з алкогольним ураженням печінки порушені процеси біо-трансформації ксенобіотиків та ендогенних токсичних сполук. Зниження активності якого хромопротеїну може бути причиною цього?

Цитохром b

Цитохром с1

Цитохромоксидаза

Цитохром Р-450

Гемоглобін

1597 / 4299
Під час ембріонального розвитку зубів у деяких клітин, що продукують структури зуба, має місце інверсія ядер та органел з базального полюсу клітини до апікального. Яку назву мають ці клітини?

Цементобласти

Мезенхімні клітини

Енамелобласти

Фібробласти

Дентинобласти

1598 / 4299
На мікроскопічному препараті бронхів виявлено епітеліальний пласт одношарового багаторядного війчастого епітелію, в якому спостерігаються келихоподібні клітини. Яка їх функція?

Камбіальна

Всмоктувальна

Скоротлива

Залозиста

Опорна

1599 / 4299
На розтині тіла жінки, що тривало страждала на аденому паращитоподі-бних залоз із гіперпродукцією парат-гормону та померла від хронічної ниркової недостатності, виявлені відкладання вапна у шлунку, легенях, нирках. Яке за механізмом розвитку звапнуван-ня має місце?

Дистрофічне

Метаболічне

Метастатичне

Змішане

1600 / 4299
Від хворого виділені нерухомі бактерії овоїдної форми з біполярним забарвленням. В організмі утворюють ніжну капсулу. На агарі утворюють колонії з мутно-білим центром, оточеним фестончатою каймою, що нагадують мереживо. Продукують 'мишачий токсин'. Дані властивості притаманні для збудника:

Туляремії

Сибірки

Бруцельозу

Коклюшу

Чуми

1601 / 4299
Гіпоплазія емалі зумовлена домінантним геном, локалізованим в Х-хромосомі. Мати має нормальну емаль зубів, а у батька спостерігається гіпоплазія емалі. У кого з дітей буде виявлятися ця аномалія?

Тільки у дочок

У всіх дітей

У половини синів

У половини дочок

Тільки у синів

1602 / 4299
У 5-річної дівчинки на періанальних складках мати знайшла білих 'черв’ячків', які викликають у дитини свербіж і неспокій, і доставила їх до лабораторії. При дослідженні лікар побачив білих гельмінтів 0,5-1 см довжиною, ниткоподібної форми із загостреними кінцями, у деяких вони заокруглені. Який діагноз можна встановити?

Аскаридоз

Опісторхоз

Дифілоботріоз

Теніоз

Ентеробіоз

1603 / 4299
У фекаліях хворого з розладами травлення виявлені зрілі нерухомі членики ціп’яка; матка в них має 7-12 бічних відгалужень. Який це може бути вид гельмінта?

Ціп’як озброєний

Ціп’як карликовий

Ціп ’як неозброєний

Стьожак широкий

Ціп’як ехінокока

1604 / 4299
У клітин, які здатні до поділу, відбуваються процеси росту, формування органел, їх накопичення, завдяки активному синтезу білків, РНК, ліпідів, вуглеводів. Як називається період мітотичного циклу, в якому відбуваються вказані процеси, але не синтезується ДНК:

Телофаза

Анафаза

Премітотичний

Синтетичний

Пресинтетичний

1605 / 4299
Відомо, що ген, відповідальний за розвиток груп крові системи MN, має два алельних стани. Якщо ген М вважати вихідним, то поява алельного йому гена N відбулася внаслідок:

Реплікації ДНК

Мутації

Кросинговеру

Репарації ДНК

Комбінації генів

1606 / 4299
Проводиться каріотипування клітин здорової людини. В каріотипі знайдено дрібну акроцентричну непарну хромосому. Це може бути:

Y-хромосома

Х-хромосома

Хромосома групи В

Хромосома групи С

Хромосома групи А

1607 / 4299
До уролога звернувся чоловік 58-ми років зі скаргами на різкі болі при сечовиділенні та зменшення кількості сечі. Лікар припустив наявність сечокам’яної хвороби. В якій частині чоловічої уретри найбільш імовірне затримання каміння?

Pars intramuralis

Pars membranacea

Pars pelvina

Pars prostatica

Pars spongiosa

1608 / 4299
Хворий з запаленням слизової язика (глосит) скаржиться на розлад смакової чутливості передніх двох третин язика. Ураженням якого нерва воно викликане?

Язиковий

Барабанна струна

Барабанний

Малий кам’янистий

Язикоглотковий

1609 / 4299
До відділення хірургічної стоматології надійшла новонароджена дівчинка, яка при смоктанні починала поперху-ватись. При обстеженні виявлена розщілина твердого піднебіння, яка була наслідком незрощення середнього лобового відростка з верхньощелепним відростком I-ої зябрової дуги. Розщілина знаходилась у піднебінні між:

В длянц canalis incisivus

Processus palatinus maxillae et lamina horizontalis os palatinum

Os incisivum et processus palatinus maxillae

Lamina horizontalis os palatinum dextrum et sinistrum

Processus palatinus maxillae dextrae et sinistrae

1610 / 4299
Хворий звернувся до лікаря зі скаргою на утруднення під час жування. При обстеженні виявлена атрофія правих скроневого i жувального м’язів. При відкриванні рота щелепа відхиляється ліворуч. Який нерв уражений?

Верхньощелепний

Нижній альвеолярний

Лицевий

Щелепно-під’язиковий

Рухова частина нижньощелепного

1611 / 4299
Під час операції закриття природженої щілини піднебіння (уранопластики) при збиванні долотом крилоподібного гачка ушкоджено великий піднебінний канал. Виникла кровотеча. Яку артерію ушкоджено?

Висхідна піднебінна

Клиноподібна

Висхідна глоткова

Низхідна піднебінна

Задня верхня альвеолярна

1612 / 4299
У хворого виявлений фурункул у зовнішньому слуховому проході. Які з перелічених лімфатичних вузлів у першу чергу можуть відреагувати на запальний процес?

nodi lymphatici cervicales profundi

nodi lymphatici mandibulares

nodi lymphatici parotidei

nodi lymphatici cervicales superficiales

nodi lymphatici retropharyngeales

1613 / 4299
У хворого внаслідок пошкодження шкіри в середній ділянці грудинно-ключичнососкоподібного м’яза виникла повітряна емболія. Яка вена шиї була травмована?

Внутрішня яремна

Задня вушна

Передня яремна

Поперечна вена шиї

Зовнішня яремна

1614 / 4299
До лікаря звернувся хворий з запаленням комірок решітчастої кістки (етмоїдит). При обстеженні виявлено порушення кровопостачання кістки. Гілками якої артерії кровопостачаю-ться решітчасті комірки в нормі?

A. transversa faciei

A. infraorbitalis

A. facialis

A. ophthalmica

A. cerebri anterior

1615 / 4299
Юнака 18-ти років доставлено в лікарню з ознаками внутрішньої кровотечі. Під час гри в футбол отримав удар в ділянці лівого підребер’я. Ушкодження якого з органів, що проектуються в дану ділянку, може спричинити сильну кровотечу?

Lien

Ren sinistra

Fundus ventriculi

Flexura coli sinistra

Cauda pancreatis

1616 / 4299
Жінка звернулась до лікаря зі скаргами на набряклість та болючість нижньої кінцівки, припухлість вен та вузлів на медіальній поверхні стегна. Яка з вен уражена?

Велика підшкірна

Великогомілкова

Стегнова

Підколінна

Мала підшкірна

1617 / 4299
У військкоматі під час УЗ-діагностики у юнака 19-ти років було виявлено опущення нирки. На рівні яких хребців розташовані нирки в нормі?

IX-XII грудні

IV-V поперекові

IX-X грудні

XI грудний та III поперековий

XII грудний та I поперековий

1618 / 4299
В гістологічному препараті визначається слизова оболонка, вкрита багатошаровим плоским незроговілим, місцями - багатошаровим плоским зроговілим епітелієм. До складу слизової оболонки входить також власна пластинка, м’язова пластинка відсутня. Визначте місце локалізації такої слизової оболонки:

Ротова порожнина

Шлунок

Стравохід

Тонка кишка

Трахея

1619 / 4299
В гістологічному препараті молочного зуба дитини визначається гіпоплазія (недорозвинення) емалі. З діяльністю яких клітин пов’язані ці порушення?

Одонтобласти

Клітини пульпи емалевого органу

Клітини проміжного шару емалевого органу

Зовнішні емалеві клітини

Внутрішні емалеві клітини

1620 / 4299
У гістологічному препараті поздовжнього шліфу зуба в емалі визначаються чергування темних і світлих смуг шириною близько 100 мкм, орієнтованих перпендикулярно поверхні дентину. Назвіть дані структури емалі:

Перикіматії

Неонатальна лінія

Лінії Ретціуса

Емалеві призми

Смуги Іунтера-Шрегера

1621 / 4299
Під час гаструляції у зародку недостатньо сформувався первинний Іензе-новський вузлик. Розвиток якого осьового органу загальмується?

Мантійний шар нервової трубки

Нервові гребінці

Нервова трубка

Нервовий жолобок

Хорда

1622 / 4299
В результаті травми носа у чоловіка 32-х років пошкоджена слизова оболонка верхньої носової раковини. До яких наслідків це призвело?

Недостатнє зігрівання і зволоження повітря

Порушення очищення повітря

Недостатнє зволоження повітря

Недостатнє зігрівання повітря

Порушення нюху

1623 / 4299
Під час мікроскопічного дослідження органу ЦНС виявлена сіра речовина, у якій нейрони утворюють три шари: молекулярний, гангліонарний і зернистий. Назвіть нейрони, що формують другий шар:

Великі зірчасті

Клітини-зерна

Грушоподібні

Дрібні зірчасті

Кошикові

1624 / 4299
При утворенні зародка людини можна спостерігати появу у його складі порожнини, світлих дрібних бластомерів на периферії та темних великих бластомерів на одному з полюсів. Як називається зародок на цій стадії розвитку?

Зигота

Зародковий диск

Гаструла

Бластоциста

Морула

1625 / 4299
На препараті м’якої мозкової оболонки виявляється судина, у стінці якої відсутня середня оболонка, зовнішня оболонка зрощена з оточуючою тканиною, внутрішня оболонка побудована із базальної мембрани та ендотелію. Що це за судина?

Артерія змішаного типу

Артерія м’язового типу

Вена волокнистого типу

Артеріола

Вена м’язового типу зі слабким розвитком м’язових елементів

1626 / 4299
У першому критичному періоді в матковій трубі з невідомої причини в зародку відбулося розчинення оболонки запліднення. Яке ускладнення вагітності можливе в цьому випадку?

Утворення двох бластоцист

Загибель зародка

Інвагінація стінки бластоцисти

Повернення бластоцисти назад в ампулярну зону труби

Імплантація зародка в стінці труби

1627 / 4299
У людини час кровотечі збільшений до 10 хвилин. Причиною цього може бути:

Лейкопенія

Еритропенія

Лімфопенія

Гіпопротеінемія

Тромбоцитопенія

1628 / 4299
У чоловіка 47-ми років за медичними показаннями була видалена слинна залоза, після чого різко зменшився вміст амілази в слині. Яка залоза була видалена?

Привушна

Ясенні

Підщелепна

Підщочна

Під’язикова

1629 / 4299
В експерименті у тварини був перерізаний стовбур мозку, після чого у неї різко підвищився тонус м’язів-розгиначів (децеребраційна ригідність). Усунення впливу якої структури мозку на м’язи викликало цей стан?

Сірий горб

Смугасте тіло

Чорна субстанція

Червоне ядро

Блакитна пляма

1630 / 4299
При обстеженні хворого з травматичним пошкодженням головного мозку виявлено, що він втратив дотикову чутливість. Який відділ кори мозку пошкоджений?

Потилична частка кори

Тім’яна частка кори

Лобна частка кори

Задня центральна звивина

Передня центральна звивина

1631 / 4299
При підйомі пішки на 5-й поверх у людини підвищився артеріальний тиск. Причиною є збільшення:

Вмісту іонів в плазмі крові

Об’єму циркулюючої крові

В’язкості крові

Кількості функціонуючих капілярів

Хвилинного об’єму крові

1632 / 4299
При визначенні енерговитрат організму людини встановлено, що дихальний коефіцієнт дорівнює 1,0. Це означає, що у клітинах досліджуваного переважно окислюються:

Білки

Білки та вуглеводи

Вуглеводи та жири

Жири

Вуглеводи

1633 / 4299
У відповідь на розтягнення м’яза спостерігається його рефлекторне скорочення. З подразнення яких рецепторів починається ця рефлекторна реакція?

Суглобові рецептори

Дотикові рецептори

Больові рецептори

Сухожилкові рецептори Іольджі

М’язові веретена

1634 / 4299
При обробці перекисом водню слизової оболонки ротової порожнини хворого, кров пофарбувалась у коричневий колір замість піноутворення. При зниженні концентрації якого з перелічених ферментів це можливо?

Псевдохолінестераза

Ілюкозо-6-фосфатдегідрогеназа

Метгемоглобінредуктаза

Ацетилтрансфераза

Каталаза

1635 / 4299
Електрофоретичне дослідження сироватки крові хворого на пневмонію показало збільшення одної з білкових фракцій. Вкажіть її:

а1-глобуліни

Альбуміни

y-глобуліни

в-глобуліни

а2-глобуліни

1636 / 4299
У чоловіка 53-х років діагностовано сечокам’яну хворобу з утворенням уратів. Цьому пацієнту призначено алло-пурінол, який є конкурентним інгібітором ферменту:

Дигідроурацилдегідрогеназа

Уратоксидаза

Уреаза

Уриділтрансфераза

Ксантиноксидаза

1637 / 4299
Перетравлення білків у шлунку є початковою стадією розщеплення білків у травному каналі людини. Назвіть ферменти, які беруть участь в перетравлені білків у шлунку:

Трипсин та катепсини

Хімотрипсин та лізоцим

Ентеропептидаза та еластаза

Карбоксипептидаза та амінопептида-за

Пепсин та гастриксин

1638 / 4299
Після загоєння рани на її місці утворився рубець. Яка речовина є основним компонентом цього різновиду сполучної тканини?

Хондроїтинсульфат

Еластин

Колаген

Гіалуронова кислота

Кератансульфат

1639 / 4299
При ряді гемоглобінопатій відбуваються амінокислотні заміни у а- і в -ланцюгах гемоглобіну. Яка з них характерна для серпоподібноклітинної анемії?

Глутамат-валін

Аспартат-лізин

Аланін-серин

Метіонін-гістидин

Гліцин-серин

1640 / 4299
Для лікування деяких інфекційних захворювань, що викликаються бактеріями, застосовуються сульфаніламідні препарати, що блокують синтез фактора росту бактерій. Який механізм дії цих препаратів?

!нгібують всмоктування фолієвої кислоти

Є антивітамінами п-амінобензойної кислоти

Є алостеричними ферментами

Беруть участь в окисно-відновних процесах

Є алостеричними інгібіторами ферментів

1641 / 4299
Анаеробне розщеплення глюкози до молочної кислоти регулюється відповідними ферментами. Який фермент є головним регулятором цього процесу?

Глюкоза-6-фосфат-ізомераза

Альдолаза

Енолаза

Фосфофруктокіназа

Лактатдегідрогеназа

1642 / 4299
У хворого встановлено зниження синтезу вазопресину, що призводить до поліурії і, як наслідок, до вираженої дегідратації організму. Що з переліченого є провідним механізмом поліурії?

Порушення канальцієвої реабсорбції іонів Na

Порушення реабсорбції глюкози

Зниження канальцієвої реабсорбції білку

Зниження канальцієвої реабсорбції води

Підвищення гідростатичного тиску

1643 / 4299
Катіонні глікопротеїни є основними компонентами слини привушних залоз. Які амінокислоти обумовлюють їх позитивний заряд?

Аспартат, глутамат, гліцин

Лізин, аргінін, гістидин

Ілутамат, валін, лейцин

Цистеїн, гліцин, пролін

Аспартат, аргінін, глутамат

1644 / 4299
У хлопчика 4-х років після перенесеного важкого вірусного гепатиту спо- стерігаються блювання, епізоди непритомності, судоми. У крові - гіперамо-ніємія. Порушення якого біохімічного процесу в печінці викликало такий стан хворого?

Порушення знешкодження біогенних амінів

Пригнічення ферментів трансаміну-вання

Пригнічення синтезу білків

Активація декарбоксилювання амінокислот

Порушення знешкодження амоніаку

1645 / 4299
У ході катаболізму гістидину утворюється біогенний амін, що має потужну судинорозширювальну дію. Назвіть його:

Дофамін

Норадреналін

Гістамін

ДОФА

Серотонін

1646 / 4299
Ті організми, які в процесі еволюції не створили захисту від H202, можуть жити лише в анаеробних умовах. Які з перелічених ферментів можуть руйнувати пероксид водню?

Оксигенази та гідроксилази

Цитохромоксидаза, цитохром B5

Оксигеназа та каталаза

Пероксидаза та каталаза

Флавінзалежні оксидази

1647 / 4299
У пацієнта 50-ти років, що звернувся до стоматолога, був виявлений малиновий 'лакований'язик. У крові: знижена кількість еритроцитів і концентрації гемоглобіну, колірний показник 1,3, наявні ознаки мегалобластичного типу кровотворення, дегенеративні зміни білої крові. Яке захворювання крові було виявлене у хворого?

Мієлоїдний лейкоз

Апластична анемія

Залізодефіцитна анемія

Іемолітична анемія

Б12-фолієводефіцитна анемія

1648 / 4299
У вагітної жінки розвинувся токсикоз із важкими повторними блюваннями упродовж доби. До кінця доби почали проявлятися тетанічні судоми і зневоднення організму. Який зсув кислотно-лужної рівноваги викликав зазначені зміни?

Негазовий метаболічний ацидоз

Негазовий видільний ацидоз

Газовий алкалоз

Газовий ацидоз

Негазовий видільний алкалоз

1649 / 4299
Під час о6іду дитина поперхнулася i аспірувала їжу. Почався сильний кашель, шкіра і слизові ціанотичні, пульс прискорений, дихання рідке. Видих подовжений. Яке порушення зовнішнього дихання розвинулось у дитини?

Дихання Біота

Дихання Чейна-Стокса

Дихання Кусмауля

Стадія інспіраторної задишки при асфіксії

Стадія експіраторної задишки при асфіксії

1650 / 4299
Після занурення водолаза на глибину 60 м у нього з’явилися симптоми порушення функцій центральної нервової системи - збудження, ейфорія, ослаблення уваги, професійні помилки. Ці симптоми пов’язані з токсичною дією на нейрони:

Азоту

Амоніаку

Кисню

Вуглекислого газу

Лактату

1651 / 4299
У хворого у крові: лейкоцити -90 • 109/л. В лейкоцитарній формулі: е-0%, б- 0%, ю- 0%, п2%, с- 20%, лімфобласти - 1%, пролімфоцити - 2%, лімфоцити - 70%, м- 5%, клітини Боткіна-ГУмпрехта. Збільшені шийні, підщелепні лімфатичні вузли. Для якої патології характерна така картина?

Хронічний лімфолейкоз

Інфекційний мононуклеоз

Гострий лімфолейкоз

Лімфогранульоматоз

Хронічний мієлолейкоз

1652 / 4299
У хворого з пародонтитом відмічається набряк ясен. Вони мають темно-червоний колір. Яке місцеве порушення кровообігу переважає в яснах хворого?

Венозна гіперемія

Ішемія

Артеріальна гіперемія

Тромбоз

Емболія

1653 / 4299
Хвора 18-ти років скаржиться на загальну слабкість, швидку втомлюваність, пригнічений настрій. Має астенічний тип конституції. Ps- 68/хв., АТ-90/60 мм рт.ст. Встановлена первинна нейроциркуляторна артеріальна гіпотензія. Що є провідним фактором зниження артеріального тиску в хворої?

Зменшення серцевого викиду

Зменшення хвилинного об’єму крові

Зниження тонусу резистивних судин

Депонування крові в венах великого кола кровообігу

Гіповолемія

1654 / 4299
У жінки, яка протягом 15-ти років страждала на виражену гіпертензію, останнім часом з’явились задишка, серцебиття, трохи знизився систолічний тиск. Який основний механізм виникнення у хворої серцевої недостатності?

Порушення проведення імпульсу по міокарду

Порушення регуляції серцевої діяльності

Перевантаження серця збільшеним опором викиду крові

Ушкодження міокарда

Перевантаження серця збільшеним об’ємом крові

1655 / 4299
У хворого з черепно-мозковою травмою відзначається дихання, яке характеризується дихальними рухами, що наростають за глибиною, а потім спадають, після чого настає недовготривала пауза. Для якого патологічного дихання характерний даний тип?

Чейн-Стокса

Апнейстичне

Кусмауля

Гаспінг-дихання

Біота

1656 / 4299
Після травматичного видалення зуба хворий скаржиться на сильний, тупий, без чіткої локалізації біль у ясні, підвищення температури тіла до 37,5oC. Діагностований альвеоліт. Який вид болю в цього хворого?

Вісцеральний

Протопатичний

Відбитий

Фантомний

Епікритичний

1657 / 4299
Дитина 10-ти років під час гри порізала ногу відламком скла і була направлений у поліклініку для введення про- типравцевої сироватки. З метою попередження розвитку анафілактичного шоку лїкувальну сироватку вводили за Безредкою. Який механізм лежить в основі подїбного способу гіпосенсибілі-зацїї організму?

Зв’язування рецепторів до IgE на опасистих клітинах

Стимуляція синтезу антиген-специфічних IgG2

Стимуляція імунологічної толерантності до антигену

Блокування синтезу медіаторів опасистих клітин

Зв’язування фасованих на опасистих клітинах IgE

1658 / 4299
У хворого із порушенням серцевого ритму при обстеженні на ЕКГ спостерігається: ЧСС50/хв., ритм синусовий, неправильний, інтервал PQ подовжений, періодичне випадіння комплексу QRS. Яке порушення серцевого ритму має місце?

Повна AV-блокада

Блокада правої ніжки пучка Гіса

Неповна AV-блокада II ст

Синдром слабкості синусового вузла

Неповна AV-блокада I ст

1659 / 4299
Через деякий час після видалення зуба у пацієнта розвинулась атрофія країв зубної лунки. Назвіть цей вид атрофії:

Нейротична атрофія

Дисфункціональна атрофія

Атрофія внаслідок недостатності кровообігу

Атрофія від дії фізичних та хімічних факторів

Атрофія від тиску

1660 / 4299
На розтині трупа жінки 52-х років, яка тривалий час хворіла на хронічний гломерулонефрит, виявлено: значно зменшені, щільні, дрібнозернисті нирки, фібринозне запалення серозних і слизових оболонок, дистрофічні зміни паренхіматозних органів, набряк головного мозку. Яким ускладненням зумовлені описані зміни серозних оболонок і внутрішніх органів?

Уремія

Тромбоцитопенія

Сепсис

Анемія

ДВЗ -синдром

1661 / 4299
У жінки 35-ти років діагностована дифтерія зіву. Померла при явищах гострої серцевої недостатності. На розтині: порожнини серця розширені в поперечнику, м’яз серця тьмяний, в’ялий, строкатий на розрізі, з жовтуватими ділянками під ендокардом. Який вид дистрофії виявлений у кардіоміоцитах?

Балонна

Жирова

Гіаліново-крапельна

Гідропічна

Вуглеводна

1662 / 4299
У молодого чоловіка видалено збільшений надключичний лімфовузол. Гістологічно: у центрі - осередок сирчастого некрозу, на периферії - вал епітеліоїдних клітин та лімфоцитів, серед яких зустрічаються клітини Пирогова-Лангханса. Діагностуйте процес у лімфатичних вузлах:

Туберкульозний лімфаденіт

Метастаз Вірхова

Лімфогранульоматоз

Лепроматозний лімфаденіт

Сифілітичний лімфаденіт

1663 / 4299
У чоловіка 35-ти років під язиком, в області вуздечки виявлена бляшка білого кольору, розмірами 0,8х0,5 см, що виступає над поверхнею слизової оболонки, шорсткувата. Мікроскопічно: епітелій потовщений із гіперке-ратозом, акантозом. У підепітеліаль-ній сполучній тканині круглоклітинна інфільтрація з розростанням колагенових волокон. Який найбільш імовірний діагноз?

Плоскоклітинний рак із ороговінням

Афтозний стоматит

Хронічний виразковий стоматит

Лейкоплакія

Папілома

1664 / 4299
При гістологічному дослідженні стінки бронха і прилеглих ділянок легені виявлені пласти і тяжі атипового плоского епітелію. У клітинах - помірно виражені ознаки атипізму: поліморфізм, гіперхромія ядер, мітози. У центрі комплексів концентричні утворення рожевого кольору. Який найбільш імовірний діагноз?

Недиференційований рак

Аденокарцинома

Скірозний рак

Плоскоклітинний рак з ороговінням

Плоскоклітинний рак без ороговіння

1665 / 4299
При гістологічному дослідженні стінки кісти, що локалізується у ділянці верхньої щелепи, встановлено, що стінка кісти з середини вистелена багатошаровим плоским епітелієм з підлеглою грануляційною тканиною з лім-фолейкоцитарною інфільтрацією. Зовнішній шар представлений пухкою волокнистою сполучною тканиною, оточеною рубцевою фіброзною тканиною. Ці дані є підставою для встановлення такого діагнозу:

Кістогранульома

Проста гранульома

Амелобластома

Кератокіста

Епітеліальна гранульома

1666 / 4299
Хворий 59-ти років з трансмураль-ним інфарктом міокарда лівого тттлу-ночка, помер від справжнього розриву серця - тампонади серця. Який процес у зоні інфаркту міг сприяти розриву серця?

Формування рубця з витонченням лівого шлуночка

Тромбоутворення

Заміщення сполучної тканини зони інфаркту (організація)

Аутолітичні процеси розплавлення тканини міокарда (міомаляція)

1667 / 4299
У хворого 77-ми років, що страждає на атеросклероз, з’явився біль у правій ступні. Ступня збільшена в розмірі, шкірні покриви чорного кольору, маце-ровані, демаркаційна зона невиражена. Який патологічний процес у ступні діагностовано?

Нома

Суха гангрена

Секвестр

Коагуляційний некроз

Волога гангрена

1668 / 4299
Чоловік віком 42 роки помер при явищах вираженої інтоксикації та дихальної недостатності. На розрізі тканина легень у всіх відділах строката, з множинними дрібновогнищевими крововиливами та вогнищами емфіземи. Гістологічно у легенях: геморагічна бронхопневмонія з абсцедуванням, у цитоплазмі клітин епітелію бронхів еозинофільні та базофільні включення. Який найбільш імовірний діагноз?

Аденовірусна інфекція

Респіраторно-синцитіальна інфекція

Парагрип

Грип

Стафілококова бронхопневмонія

1669 / 4299
У дівчинки 6-ти років протягом 5-ти місяців розвинулася деформація обличчя внаслідок симетричного збільшення об’єму обох кутів нижньої щелепи. Мікроскопічно: кісткова тканина щелепи заміщена волокнистою сполучною тканиною з великою кількістю судин, примітивними кістковими балочками. Яке захворювання найбільш імовірно?

Цементома

Остеобластокластома

Еозинофільна гранульома

Фіброзна дисплазія

Херувізм

1670 / 4299
При розтині тіла чоловіка, померлого від внутрішньокишкової кровотечі, в клубовій кишці спостерігається некроз групових і солітарних фолікулів, імбібіція мертвих тканин жовчю і кров’ю; в нижньому відрізку кишки -явища секвестрації та відторгнення некротичних мас з утворенням дефектів. Який з перелічених діагнозів найбільш вірогідний?

Черевний тиф, стадія 'бру- дних'виразок

Черевний тиф, стадія некрозу

Черевнотифозна форма сальмонельозу

Хвороба Крона

Черевний тиф, стадія 'чи- стих'виразок

1671 / 4299
У хворого в обох щелепах рентгенологічно виявлено численні дефекти у вигляді гладкостінних округлих отворів. При гістологічному дослідженні - явища остеолізису і остеопорозу при явищах слабкого кісткоутворення. В сечі хворого знайдено білок БенсДжонса. Який найбільш імовірний діагноз?

Хронічний еритромієлоз

Мієломна хвороба

Хронічний мієлолейкоз

Гострий недиференційований лейкоз

Гострий мієлолейкоз

1672 / 4299
У хворого 23-х років після перенесеної ангіни розвинувся сечовий синдром (гематурія, протеїнурія, лейкоцитурія). В пункційному біоптаті нирок виявлена картина інтракапілярного проліфе- ративного гломерулонефриту, а еле-ктронномікроскопічно виявлені великі субепітеліальні депозити. Який патогенез цього захворювання?

Клітинно обумовлений цитоліз

Імунокомплексний механізм

Атопія, анафілаксія з утворенням IgE і фіксацією їх на опасистих клітинах

Цитотоксична, цитолітична дія антитіл

Гранулематоз

1673 / 4299
В сироватці крові новонародженого знайдено антитіла до вірусу кору. Про наявність якого імунітету це може свідчити?

Природний активний

Спадковий

Штучний пасивний

Штучний активний

Природний пасивний

1674 / 4299
Обстежуючи дитину, лікар-стоматолог виявив нальот на мигдаликах і запідозрив атипову форму дифтерії. Був підготовлений мазок, зроблений посів на поживні середовища і визначена токсичність виділеної культури. Яка реакція використана для визначення токсигенності виділеного штаму дифтерійної палички?

Реакція аглютинації на склі

Реакція преципітації у гелі

Реакція гемолізу

Реакція зв’язування комплементу

Реакція кольцепреципітації

1675 / 4299
Хворий надійшов до інфекційної лікарні на 8-й день зі скаргами на головний біль, нездужання, слабкість. Для серологічного дослідження взято кров. При проведенні реакція аглютинації Відаля встановлено, що вона позитивна в розведенні 1:200 з Одіагностикумом черевного тифу. Який діагноз можна встановити на підставі цього дослідження?

Лептоспіроз

Туберкульоз

Дизентерія

Черевний тиф

Холера

1676 / 4299
З фекалій хворого виділені шигели Зонне. Які потрібно провести додаткові дослідження для встановлення джерела інфекції?

Реакція нейтралізації

Реакція преципітації

Фаготипування виділеної чистої культури

Реакція зв’язування комплементу

Антибіотикограма

1677 / 4299
При мікроскопії мазка, взятого у хворого з гострим гнійним періоститом, лікар виявив грампозитивні бактерії, розташовані у вигляді скупчень, що нагадують виноградні грона. Які мікроорганізми мають дану морфологію?

Тетракоки

Сарцини

Стрептококи

Стафілококи

Гриби роду Кандида

1678 / 4299
Хворій жінці поставили клінічний діагноз 'гонорея'. Яке із перерахованих нижче досліджень можна застосувати для підтвердження діагнозу?

Зараження лабораторних тварин

Реакція гемаглютинації

Проба з бактеріофагом

Реакція іммобілізації

Мікроскопія патологічного матеріалу

1679 / 4299
В інфекційне відділення з ознаками ураження печінки поступила хвора, стоматолог за фахом. Які методи лабораторної діагностики необхідно призначити для встановлення діагнозу 'вірусний гепатит В'?

Виявлення HBsAg в сироватці крові

Вірусологічне дослідження фекалій

Виявлення активності ферментів (альдолази, трансаміназ та ін.)

Визначення функціональних проб печінки (білірубін і холестерин крові)

Вірусологічне дослідження сечі

1680 / 4299
У пацієнта з попереднім діагнозом 'сифіліс'лаборант взяв сироватку крові для постановки імунної реакції, яка основана на виявленні антитіл, які припиняють рух трепонем і призводять до їх загибелі. Яку реакцію було використано для діагностики?

Реакція преципітації

Реакція іммобілізації

Реакція аглютинації

Реакція зв’язування комплементу

Реакція нейтралізації

1681 / 4299
До стоматолога звернувся пацієнт із проявами кандидозу порожнини ро- та, який постійно рецидивує і не піддається лікуванню. При опитуванні з’ясувалось, що у пацієнта тривалий час відмічається підвищена температура, втрата маси тіла. Які дослідження треба провести у хворого?

Бактеріологічні дослідження на дисбактеріоз

Дослідити рівень специфічних антитіл до грибів Кандида

Імунологічні та серологічні тести на ВІЛ-інфекцію

Перевірити стан гуморального імунітету

Виділити чисту культуру збудника і дослідити чутливість до антибіотиків

1682 / 4299
Чоловік 49-ти років був доставлений в гастроентерологічне відділення зі скаргами на нудоту, біль у епігастрії. Хворому було призначено фамотидин. Вкажіть молекулярний субстрат його дії:

b-адренорецептори

Na+Н+-АТФаза

М-холінорецептори

а1-адренорецептори

Н2-гістамінорецептори

1683 / 4299
В комплексному лікуванні гінгівіту хворому призначили препарат, який за хімічною будовою відноситься до похідних піримідину, стимулює лейкопоез, прискорює загоєння ран, підсилює ріст та розмноження клітин (процеси проліферації), виявляє протизапальну дію. Застосовується при лейкопеніях різного генезу, в стоматологічній практиці при запальних захворюваннях слизової оболонки ротової порожнини. Визначте препарат:

Метилурацил

Коамід

Ціанокобаламін

Метотрексат

Меркаптопурин

1684 / 4299
У хворого на прийомі у стоматолога виник пароксизм тахікардії. Який з названих засобів слід використати для його купірування?

Верапаміл

Нітрогліцерин

Атропін

Дифенін

Ізадрин

1685 / 4299
Під час оперативного втручання анестезіолог для керованої гіпотонії за- стосував гангліоблокуючий засіб. Який препарат було призначено хворому в даному випадку?

Бензогексоній

Пірілен

Пахікарпін

Пентамін

Гігроній

1686 / 4299
До поліклініки звернувся хворий зі скаргами на біль за грудниною, задишку і серцебиття. Після обстеження лікар діагностував у хворого ІХС і призначив верапаміл. Який механізм дії даного препарату?

Блокує а-адренорецептори

Блокує калієві канали

Блокує в-адренорецептори

Блокує натрієві канали

Блокує кальцієві канали

1687 / 4299
Хворий на хронічну серцеву недостатність протягом декількох місяців приймав в амбулаторних умовах диго-ксин. На певному етапі лікування у нього виникли симптоми передозування препарату. Яке явище лежить в основі розвитку цього ускладнення?

Звикання

Тахіфілаксія

Функціональна кумуляція

Матеріальна кумуляція

Сенсибілізація

1688 / 4299
У людини травматичне пошкодження грудинно-ключично-соскоподібного м’яза. Це призвело до зменшення величини:

Дихальний об’єм

Функціональна залишкова ємність легенів

Резервний об’єм видиху

Залишковий об’єм

Резервний об’єм вдиху

1689 / 4299
У дитини діагностовано порушення формування емалі і дентину зубів внаслідок зниженого вмісту іонів Ca2+ в крові. Який гормональний препарат можна застосувати для корекції даного стану?

Тироксин

Кальцитонін

Преднізолон

Кортизон

Соматотропін

1690 / 4299
У хворого після застудного захво- рювання виникло порушення виділення сльози. Який вегетативний вузол найбільше при цьому постраждав?

Піднижньощелепний

Вушний

Під’язиковий

Війчастий

Крилопіднебінний

1691 / 4299
У відділення інтенсивної терапії доставлено жінку 50-ти років з діагнозом: інфаркт міокарда. Активність якого ферменту крові буде найбільш підвищена протягом перших двох діб захворювання?

Аланінамінопептидаза

Аспартатамінотрансфераза

ЛДГ4

Аланінамінотрансфераза

ЛДГ5

1692 / 4299
На гістологічному препараті нирки в дистальному звивистому канальці виявляються клітини, які щільно прилягають до ниркового тільця. Базальна мембрана їх дуже тонка і не утворює складок. Ці клітини чутливі до зміни вмісту натрію у сечі та впливають на секрецію реніну юкстагломерулярни-ми клітинами. Які це клітини?

Юкстагломерулярні клітини

Подоцити

Мезангіальні клітини

Клітини щільної плями

Ендотелій капілярів клубочка

1693 / 4299
У хворого 26-ти років виявлений великий фурункул м’яких тканин обличчя біля кореня носа та нижньої повіки. Грізним ускладненням цього захворювання може бути розповсюдження інфекції венозними сполученнями цього регіону до пазух твердої мозкової оболонки. Яка з пазух найбільш імовірно може бути уражена?

Верхня кам’яниста

Печериста

Сигмоподібна

Верхня сагітальна

Потилична

1694 / 4299
Тварині, сенсибілізованій туберкуліном, внутрішньоочеревенно введений туберкулін. Через 24 години при лапа-ратомії виявлено венозну гіперемію та набряк очеревини. У мазках-відбитках з очеревини велика кількість лімфоцитів та моноцитів. Який патологічний процес у тварини?

Гнійне запалення

Серозне запалення

Алергічне запалення

Асептичне запалення

Фібринозне запалення

1695 / 4299
У чоловіка 35-ти років через 30 хвилин після автомобільної аварії виявлена масивна травма нижніх кінцівок без значної зовнішньої крововтрати. По-страждалий знаходиться у збудженому стані. Який компонент патогенезу травматичного шоку є у пацієнта провідним і потребує негайного корегування?

Порушення функції органів

Внутрішня плазмовтрата

Інтоксикація

Внутрішня крововтрата

Біль

1696 / 4299
У процесі звапнування міжклітинної речовини кісткової тканини вздовж колагенових волокон відкладаються кристали гідроксиапатиту. Для реалізації цього процесу необхідна присутність у міжклітинній речовині лужної фосфатази. Яка клітина продукує цей фермент?

Остеоцит

Хондробласт

Остеобласт

Остеокласт

Хондроцит

1697 / 4299
Хвора звернулась до лікаря зі скаргами на болі та обмеження рухів у колінних суглобах. Який з нестероїдних протизапальних засобів краще призначити, враховуючи наявність в анамнезі хронічного гастродуоденіту?

Целекоксиб

Диклофенак-натрій

Промедол

Кислота ацетилсаліцилова

Парацетамол

1698 / 4299
Депресії, емоційні розлади є наслідком нестачі у головному мозку нора-дреналіну, серотоніну та інших біогенних амінів. Збільшення їх вмісту у си-напсах можна досягти за рахунок антидепресантів, які гальмують активність такого ферменту:

Оксидаза L-амінокислот

Моноамінооксидаза

Діамінооксидаза

Оксидаза D-амінокислот

Фенілаланін-4-монооксигеназа

1699 / 4299
Хворому 30-ти років для лікування пневмонії лікар на 3 дні призначив антибіотик з групи азалідів, що має бактерицидну дію, тривалий ефект, здатність зв’язуватись з фагоцитами і накопичуватись у вогнищах інфекції. Який препарат було призначено хворому?

Ізоніазид

Бензилпеніциліну натрієва сіль

Ципрофлоксацин

Еритроміцин

Азитроміцин

1700 / 4299
З метою оцінки адаптації до фізичного навантаження лікар провів обстеження робітників після виконання важкої праці. Які зміни в загальному аналізі крові можна виявити при цьому?

Перерозподільчий лейкоцитоз

Зсув лейкоцитарної формули вліво

Анемія

Гіпоальбумінемія

Лейкопенія

1701 / 4299
Серед лімфоцитів розрізняють популяцію клітин, що мають мембранні рецептори до IgM, вони активуються під впливом специфічних антигенів, мітотично розмножуються, диференціюються у плазматичні клітини, що виробляють антитіла (імуноглобуліни). Як називаються ці клітини?

Т-лімфоцити-кілери

Т-лімфоцити пам’яті

В-лімфоцити

Т-лімфоцити-супресори

1702 / 4299
У ході клінічного обстеження пацієнта виявлено збільшення щитоподібної залози (зоб), підвищення основного обміну, втрата маси тіла, порушення теплового балансу, підвищення апетиту, підвищення збудливості та дратівливості, екзофтальм і тахікардія. Яке ендокринне порушення призводить до появи даних симптомів?

Гіперфункція щитоподібної залози

Гіпофункція епіфізу

Гіпофункція щитоподібної залози

Гіпофункція паращитоподібних залоз

Гіперфункція гіпофізу

1703 / 4299
Пошкодження мозку призвело до порушення моторної функції мови. У якому відділі кори відбулося пошкодження?

Потилична ділянка кори

Передня центральна звивина

Скронева ділянка кори

Тім’яна ділянка кори

Нижня лобна звивина

1704 / 4299
Під час бійки у чоловіка виникла зупинка серця внаслідок сильного удару у верхню ділянку передньої черевної стінки. Які з зазначених рефлексів спричинили зупинку серця?

Парасимпатичні безумовні

Парасимпатичні умовні

Симпатичні умовні

Симпатичні безумовні

Периферичні

1705 / 4299
У пахвах людини виявлені дрібні (11,5 мм) сплющені у спинно-черевному напрямку, безкрилі кровосисні комахи. Їх личинки розвивалися тут же. Яке захворювання викликають ці комахи?

Поворотний тиф

Хвороба Чагаса

Фтиріоз

Сонна хвороба

Чума

1706 / 4299
У людини під впливом іонізуючого випромінювання в крові зменшилася кількість гранулоцитів. Який механізм лежить в основі цих змін?

Пригнічення лейкопоезу

Порушення виходу зрілих лейкоцитів з кісткового мозку

Підвищене руйнування лейкоцитів

Розвиток аутоімунного процесу

Збільшений перехід гранулоцитів в тканини

1707 / 4299
Хворому проведено лобектомію правої середньої частки легені. Які сегменти легені були уражені?

Бічний та присередній

Верхівковий, передній

Верхівково-задній та передній

Присередній і передній основні

Задній і бічний основні

1708 / 4299
Пацієнт доставлений до лікарні з симптомами запаморочення, сухості в роті, зіниці сильно розширену порушення акомодації, тахікардія, утруднення сечовипускання, атонія кишечнику. Передозування яким препаратом могло викликати дані симптоми?

Празозин

Атропіну сульфат

Клофелін

Каптоприл

Фуросемід

1709 / 4299
У чоловіка 60-ти років, який страждає на хронічну кишкову непрохідність, посилюється гниття білків у товстому кишечнику. Підтвердженням цього процесу є:

!ндиканурія

Глюкозурія

Креатинурія

Гіперурікурія

Білірубінурія

1710 / 4299
Хворий у непритомному стані доставлений бригадою швидкої допомоги до лікарні. Об’єктивно: рефлекси відсутні, періодично з’являються судоми, дихання нерівномірне. Після лабораторного обстеження було діагностовано печінкову кому. Нагромадження якого метаболіту є суттєвим для появи розладів центральної нервової системи?

Білірубін

Сечовина

Гістамін

Амоніак

Глутамін

1711 / 4299
Внаслідок переливання несумісної крові за антигеном Rh у хворої виникла гемолітична жовтяниця. Який лабораторний показник крові підтверджує цей тип жовтяниці?

Нагромадження некон’югованого білірубіну

Зменшення вмісту некон’югованого білірубіну

Зменшення вмісту стеркобіліну

Нагромадження уробіліногену

Зменшення вмісту кон’югованого білірубіну

1712 / 4299
Хворий госпіталізований з травмою медіальної групи м’язів стегна. Які види рухів він НЕ зможе робити?

Згинання стегна

Відведення стегна

Супінація стегна

Розгинання стегна

Приведення стегна

1713 / 4299
При обстеженні хворого з ендокринною патологією встановлено, що в плазмі крові підвищений рівень тестостерону. Які клітини в організмі чоловіка відповідальні за продукцію цього гормону?

Сперматогенні клітини

Клітини сім’яних міхурців

Сустентоцити сім’яників

Іландулоцити сім’яників

Клітини передміхурової залози

1714 / 4299
Курареподібні речовини (дитилін) роблять неможливим скорочення скелетних м’язів, оскільки вони блокують:

Центральні синапси

Пропріорецептори

Іангліонарні синапси

Нервово-м’язові синапси

Проведення збудження мембраною

1715 / 4299
Встановлено, що швидкість проведення збудження нервовими волокнами становить 120 м/сек. Зазначені волокна є:

Постгангліонарними парасимпатичними

Аксонами мотонейронів

Прегангліонарними парасимпатичними

Постгангліонарними симпатичними

Прегангліонарними симпатичними

1716 / 4299
У хворого при обстеженні виявлено глюкозурію, гіперглікемію. Скарги на сухість в роті, свербіння шкіри, часте сечовиділення, спрагу. Встановлений діагноз: цукровий діабет. Чим обумовлена поліурія у даного хворого?

Збільшення осмотичного тиску сечі

Зменшення серцевого викиду

Зменшення онкотичного тиску плазми

Збільшення фільтраційного тиску

Збільшення онкотичного тиску плазми

1717 / 4299
При сильному зубному болі хворому рекомендовано прийняти ненар-котичний анальгетик із групи похідних аніліну. Виберіть препарат:

Бутадіон

Анальгін

Кислота ацетилсаліцилова

Ібупрофен

Парацетамол

1718 / 4299
В лікарню доставлена жінка, у якої виникла необхідність в інтубації трахеї. Який з нижченаведених лікарських засобів доцільно застосувати у даному випадку?

Іентаміцину сульфат

Нітрогліцерин

Метронідазол

Атропіну сульфат

Дитилін

1719 / 4299
Жінці, що страждає на алергічний нейродерміт, призначено антигістамін-ний препарат II покоління, в якого відсутній пригнічуючий вплив на ЦНС. Вкажіть цей препарат:

Димедрол

Лоратадин

Тавегіл

Кетотіфен

Діазолін

1720 / 4299
Судово-медичному експерту надійшов орган для ідентифікації. При гістологічному дослідженні в ньому виявлені невпорядковано розміщені лім-фоїдні фолікули, які мають 4 зони і ексцентрично розташовану в них артерію. Який це орган?

Товста кишка

Апендикс

Селезінка

Лімфатичний вузол

Мигдалик

1721 / 4299
Хворий 37-ми років за останні три місяці схуд на 5 кг, скаржиться на тремор рук, підвищене потовиділення, екзофтальм, тахікардію. Збільшення секреції якого гормону може бути причиною цього?

Інсулін

Тироксин

Тиреокальцитонін

Кортизол

Глюкагон

1722 / 4299
У кішки під час експерименту подразнюють периферичний відрізок блукаючого нерва. Які з наведених змін будуть спостерігатися при цьому?

Збільшення частоти дихання

Зменшення частоти серцевих скорочень

Розширення зіниць

Збільшення частоти серцевих скорочень

Розширення бронхів

1723 / 4299
Після того, як людина випила 1,5 л води, кількість сечі значно збільшилась, а її відносна щільність зменшилась до 1,00І. Зазначені зміни є наслідком зменшення реабсорбції води в дистальних відділах нефронів внаслідок зменшення секреції:

Простагландинів

Вазопресину

Альдостерону

Реніну

Ангіотензину II

1724 / 4299
У хлопчика водянка яєчка (накопичування рідини між оболонками яєчка). Яка саме оболонка яєчка вміщує цю рідину?

Внутрішня сім’яна

Піхвова

М’ясиста

Зовнішня сім’яна

Білкова

1725 / 4299
При обстеженні хворого 6-ти років виникла підозра на погіршення прохідності дихальних шляхів. Який із методів дослідження дозволяє вірогідно визначити дану патологію?

Пневмографія

Спірографія

Спірометрія

Пневмотахометрія

Спірометаболографія

1726 / 4299
У чоловіка 60-ти років, який страждає на хронічний гепатит, часто спостерігались кровотечі з носа і ясен, спонтанно виникали геморагічні висипання на шкірі і слизових оболонках. Наслідком чого є ці явища?

Зменшення в крові рівня холінесте-рази

Зменшення синтезу протромбіну і фібриногену

Поява в крові макроглобулінів і кріоглобулінів

Підвищення вмісту амінотрансфераз плазми крові

Зменшення утворення сироваткових альбумінів

1727 / 4299
Провідними симптомами первинного гіперпаратиреозу є остеопороз та ураження нирок із розвитком сечокам’яної хвороби. Які речовини складають основу каменів при цьому захворюванні?

Цистін

Фосфат кальцію

Білірубін

Холестерин

Сечова кислота

1728 / 4299
У хворої встановлено порушення виділення тиреотропного гормону гіпофіза. Порушення функції якої частки гіпофіза має місце у хворої?

Lobus posterior

Infundibulum

Lobus anterior

Pars intermedia

1729 / 4299
У жінки 49-ти років, після тривалого стояння виявлено набряк ніг. Яка можлива причина появи набряків?

Зменшення гідростатичного тиску крові у венах

Підвищення артеріального тиску

Збільшення онкотичного тиску плазми крові

Підвищення гідростатичного тиску крові у венах

Зменшення гідростатичного тиску крові в артеріях

1730 / 4299
У хворого після прийому недоброякісної їжі розвинувся багаторазовий пронос. На наступний день у нього знизився артеріальний тиск, з’явилися тахікардія, екстрасистолія. рН крові 7,18. Ці порушення є наслідком:

Негазового ацидозу

Газового ацидозу

Газового алкалозу

Метаболічного алкалозу

Негазового алкалозу

1731 / 4299
Хворий переведений на безсольову дієту. Як у нього змінився поріг смакової чутливості до солоного?

Не змінився

Підвищився

Мало змінився

Знизився

Спочатку підвищився, а потім знизився

1732 / 4299
На аутопсії померлого, який три- вало страждав на гіпертонічну хворобу, патологоанатом виявив, що нирки різко зменшені у розмірах, щільні, поверхня їх рівномірно дрібнозерниста, на розрізі - паренхіма, особливо кіркова речовина, рівномірно виточені. Він дійшов висновку, що це:

Атеросклеротично зморщена нирка

Артеріолосклеротично зморщена нирка

Пієлонефритично зморщена нирка

Амілоїдно-зморщена нирка

1733 / 4299
Пацієнт звернувся до лікаря з приводу того, що він втратив здатність розрізняти смаки на корені язика. Лікар встановив, що це пов’язано з ураженням нерва. Якого саме?

Лицьовий

Трійчастий

Блукаючий

Язикоглотковий

Верхньогортанний

1734 / 4299
Хворий скаржиться на жовтяни-чність шкіри, шкірний свербіж, загальну слабкість. У сечі: відсутній уробілін. Яка патологія у хворого?

Механічна жовтяниця

Гостра печінкова недостатність

Хронічна печінкова недостатність

Паренхіматозна жовтяниця

Гемолітична жовтяниця

1735 / 4299
Введення знеболюючого пацієнту перед екстракцією зуба призвело до розвитку анафілактичного шоку, який супроводжувався розвитком олігурії. Який патогенетичний механізм зумовив зменшення діурезу в даній клінічній ситуації?

Підвищення гідростатичного тиску в капсулі Шумлянського-Боумена

Збільшення онкотичного тиску плазми крові

Зниження гідростатичного тиску в капілярах ниркових тілець

Збільшення секреції вазопресину

Пошкодження клубочкового фільтру

1736 / 4299
Для формування мінерального ма-триксу твердих тканин зуба необхідна висока концентрація фосфат-іонів, що утворюється в процесі гідролізу фо-сфорноефірних зв’язків за участю лужної фосфатази. Іони якого металу є активаторами цього процесу:

Ферум

Натрій

Цинк

Магній

Кальцій

1737 / 4299
При втручанні з метою лікування вивиху нижньої щелепи лікар повинен пам’ятати про м’яз, який при скороченні відтягує назовні капсулу і суглобовий диск скроневонижньощелепного суглоба. Який це м’яз?

M. pterygoideus medialis

M. masseter

M. mylohyoideus

M. temporalis

M. pterygoideus lateralis

1738 / 4299
У чоловіка 71-го року впродовж 10 днів спостерігався пронос з домішками у калі слизу та крові. Хворого було госпіталізовано у тяжкому стані, помер через 2 доби. При розтині тіла померлого виявлено: дифтеритичний коліт з множинними виразками неправильної форми різної глибини в сигмоподібній і прямій кишках. При бактеріологічному дослідженні висіяно шигели. Яке основне захворювання у хворого?

Ієрсиніоз

Черевний тиф

Сальмонельоз

Дизентерія

Неспецифічний виразковий коліт

1739 / 4299
Зірчастий ретикулум і зовнішній шар клітин емалевого органу піддаються інволюції і після завершення аме-логенезу разом з апікальними частинами амелобластів формують:

Пелікулу зуба

Зубний наліт (бляшки)

Кутикулу зуба

Муцинову плівку

1740 / 4299
При мікроскопічному дослідженні біоптата з товстої кишки виявлена пухлина з призматичного епітелію, що формує атипові залозисті структури різної форми і величини. Базальна мембрана залоз зруйнована. Клітки пухлини поліморфні, ядра гіперхромні, відзначається велика кількість патологічних мітозів. Який діагноз найбільш вірогідний?

Аденокарцинома

Слизовий рак

Солідний рак

Базальноклітинний рак

Недиференційований рак

1741 / 4299
При мікроскопії мазка з плівки, яка з’явилась на пептонній воді через 6 годин після посіву фекалій і культивування в термостаті, виявлені зігнуті у вигляді коми рухливі бактерії, грам-негативні, спор і капсул не утворюють. Які це мікроорганізми?

Коринебактерії

Спірили

Вібріони

Клостридії

Спірохети

1742 / 4299
Для морфологічного дослідження представлена ендокринна залоза, паренхіма якої складається з епітелію та нервової тканини. В епітеліальних тра-бекулах виявляється 2 типи клітин: хро-мофільні та хромофобні. Визначте даний орган:

Щитоподібна залоза

Надниркова залоза

Гіпофіз

Прищитоподібна залоза

Гіпоталамус

1743 / 4299
До приймального відділення надійшов хворий з ознаками гострої серцевої недостатності: блідість, акроціаноз, часте, поверхневе дихання. Який з перелічених засобів показаний у цьому випадку?

Корглікон

Нітрогліцерин

Адреналіну гідрохлорид

Дигітоксин

Кордіамін

1744 / 4299
В гістологічному препараті нижньої щелепи ембріону виявляється зубний зачаток, в якому зубний сосочок утворений дрібними зірчастими базо-фільно забарвленими клітинами. Яка тканина утворює цю частину зубного зачатку?

Мезенхіма

Ретикулярна

Кісткова

Хрящова

Епітеліальна

1745 / 4299
За професійними показаннями проведена вакцинація лікарів- стоматологів. Вакцина має захищати їх від вірусної інфекції, збудник якої може бути присутній у крові стоматологічних хворих, які перенесли інфекцію або є хронічними носіями. Яка вакцина була використана?

Субодинична грипозна вакцина

Жива корова вакцина

Генно-інженерний НВs-антиген

Антирабічна вакцина

!нактивована вакцина проти гепатиту А

1746 / 4299
У хворого виявлено порушення секреторної функції піднижньощелепної слинної залози. Який нерв забезпечує її вегетативну іннервацію?

N. auriculotemporalis

N. petrosus minor

Chorda tympani

N. petrosus major

N. mandibularis

1747 / 4299
При огляді новонародженої дитини педіатр виявив наявність в ротовій порожнині нижніх центральних різців. В якому віці вони прорізуються в нормі?

10-12 місяців

До 2-х років

До народження

6-8 місяців

Впродовж першого місяця життя

1748 / 4299
У хворого зі скаргами на часті рідкі випорожнення з кров’ю ('малинове желе') при мікроскопічному дослідженні були виявлені великі клітини з одним ядром та поглиненими еритроцитами. Для якого з найпростіших характерна така морфологічна форма?

Campylobacter jejuni

Toxoplasma gondii

Giardia lamblia

Balantidium coli

Entamoeba histolytica

1749 / 4299
У жінки 30-ти років з деформацією нижньої щелепи вилучена пухлина у вигляді вузла. Макроскопічно тканина її бура, з порожнинами; мікроскопічно представлена дрібними одноядер-ними клітинами овальної форми за типом остеобластів та гігантськими багатоядерними клітинами за типом остео-кластів, серед них багато дрібних незрілих судин, зерна гемосидерину, острівці остеоїдної речовини. Який найбільш імовірний діагноз?

Остеома

Гемангіома

Остеосаркома

Остеобластокластома

Фіброзна остеодисплазія

1750 / 4299
Хворому на остеомієліт нижньої щелепи був призначений антибіотик тетрациклінової групи. Вкажіть цей препарат:

Оксацилін

Доксицикліну гідрохлорид

Амікацин

Стрептоміцин

Рифампіцин

1751 / 4299
При огляді ротової порожнини на вестибулярній поверхні нижнього різця зліва виявлене утворення грибоподібної форми рожевого кольору до 2 см, яке широкою ніжкою фіксоване до надальвеолярної тканини. Під час гістологічного дослідження виявлено розгалужені судини капілярного типу з судинними бруньками, ділянками крововиливів та осередки гемосидерозу. Який найбільш імовірний діагноз?

Фіброматоз ясен

Кавернозна гемангіома

Гігантоклітинний епуліс

Ангіоматозний епуліс

Фіброзний епуліс

1752 / 4299
Пацієнту з метою підтримання роботи печінки був призначений препарат, якій містить метіонін. Завдяки цьому може бути забезпечено синтез:

Фосфатидилхоліну

Цитрату

Пірувату

Фосфатидилсерину

Лактату

1753 / 4299
Жінці 26-ти років, хворій на бронхіт, призначили засіб етіотропної терапії - антибіотик широкого спектру дії. Який це препарат?

Дексаметазон

Доксициклін

Інтерферон

Амброксол

БЦЖ-вакцина

1754 / 4299
У хворого герпетичний кон’юнктивіт. Який етіотропний лікарський засіб необхідно призначити?

Метисазон

Фурагін

Тетрациклін

Ацикловір

Ампіцилін

1755 / 4299
Пацієнт 68-ми років звернувся до кардіолога із скаргами на підвищення артеріального тиску, біль в ділянці серця, відчуття перебоїв у роботі серця. Призначте препарат з групи в1-адреноблокаторів для лікування даної патології:

Метопролол

Ноотропіл

Морфіну гідрохлорид

Бензилпеніцилін

Фенотерол

1756 / 4299
Жінка під час вагітності хворіла на краснуху. Дитина народилась з вадами розвитку - незрощення губи і піднебіння. Генотип у дитини нормальний. Ці аномалії розвитку є проявом:

Модифікаційної мінливості

Поліплоїдії

Анеуплоїдії

Хромосомної мутації

Комбінативної мінливості

1757 / 4299
У спекотну погоду в гарячих приміщеннях для нормалізації мікроклімату часто використовують вентилятори. При цьому посилюється віддача тепла тілом людини перш за все таким тттля-хом:

Радіація

Теплопроведення

Конвекція

Випаровування

Кондукція

1758 / 4299
Вагітній з кількома мимовільними абортами в анамнезі призначено терапію вітамінними препаратами. Який вітамін сприяє виношуванню плода?

Піридоксальфосфат

Альфа-токоферол

Ціанокобаламін

Рутин

Тіамін

1759 / 4299
З носоглотки здорового бакте-ріоносія виділено мікроорганізм, який за біологічними ознаками ідентичний Corynebacterium diphtheriae, але не продукує екзотоксин. В результаті якого процесу цей мікроорганізм здатний відновити експресію і продукцію екзото- ксину?

Фагова конверсія

Хромосомна мутація

Культивування на телуритовому середовищі

Культивування в присутності антитоксичної сироватки

Пасаж через організм чутливих тварин

1760 / 4299
Оперативно висічена сполучна тканина деформованого мітрального клапану при забарвленні гематоксиліном та еозином дає базофільну реакцію, а під впливом толуїдинового синього фарбується в бузковий колір (ме-тахромазія). Які зміни сполучної тканини виявляються такими типами реакцій?

Гіаліноз

Мукоїдний набряк

Петрифікація

Набряк сполучної тканини

Фібриноїдний некроз сполучної тканини

1761 / 4299
Внаслідок надмірного годування тварини вуглеводами в клітинах печінки при гістологічному дослідженні виявлена значна кількість гранул глікогену. До якої групи структур клітини відноситься глікоген?

Екскреторні включення

Секреторні включення

Пігментні включення

Органели спеціального призначення

Трофічні включення

1762 / 4299
У хворого виражений больовий синдром при невралгії. Який засіб з не-стероїдних протизапальних препаратів зменшить болесприйняття?

Дроперидол

Кетаміну гідрохлорид

Лідокаїну гідрохлорид

Кодеїну фосфат

Диклофенак-натрій

1763 / 4299
У хворого розвинувся птоз (опущення верхньої повіки). Ураження якого нерва має місце?

Блоковий

Трійчастий

Лицевий

Окоруховий

Відвідний

1764 / 4299
Стеатоз виникає внаслідок нако- пичення триацилгліцеролів у гепатоци-тах. Одним з механізмів розвитку цього захворювання є зменшення утилізації нейтрального жиру ЛПДНЩ. Які ліпо-тропні речовини попереджують розвиток стеатозу?

Аланін, B1, PP

Метіонін, B6, B12

Аргінін, B2, B3

Валін, B3, B2

Iзолейцин, B1, B2

1765 / 4299
До медико-генетичної консультації звернувся чоловік з кольоровою сліпотою (дальтонізм). З якою ймовірністю у його дітей виявиться дана ознака, якщо в генотипі його дружини даного алеля немає?

25%

50%

0%

100%

75%

1766 / 4299
Велика доза сечогінного засобу викликала у хворого падіння артеріального тиску. Яку групу засобів найбільш доцільно використати для його підвищення у цьому випадку?

Аналептики

Н-холіноміметики

Серцеві глікозиди

Адреноміметики

Плазмозамінники

1767 / 4299
В клітині відбувається процес трансляції. Коли рибосома доходить до кодонів УАА, УАГ або УГА, синтез по-ліпептидного ланцюга закінчується. Ці кодони у процесі біосинтезу поліпептиду не розпізнаються жодною т-РНК і тому є сигналом:

Термінації

Елонгації

Посттрансляційної модифікації

Початку транскрипції

Ініціації

1768 / 4299
Після початку лікування туберкульозу легень, хворий звернувся до лікаря зі скаргами на появу червоних сліз та сечі. Який препарат міг викликати такі зміни?

Бензилпеніциліну калієва сіль

Цефазолін

Бісептол-480

Рифампіцин

Бензилпеніциліну натрієва сіль

1769 / 4299
У хворого запальний процес у кри-лопіднебінній ямці. !нфекція поширилась в носову порожнину. Через яке анатомічне утворення розповсюдилась інфекція?

Foramen rotundum

Canalis ptherygoideus

Foramen sphenopalatinum

Canalis palatinus minor

Canalis palatinus major

1770 / 4299
У хворого запальний процес у крилопіднебінній ямці. !нфекція поширилась в середню черепну ямку. Через яке анатомічне утворення розповсюдилась інфекція?

Canalis palatinus major

Foramen rotundum

Foramen sphenopalatinum

Canalis palatinus minor

Canalis ptherygoideus

1771 / 4299
Гістологічне дослідження тканини виявило, що в ній відсутні кровоносні судини, а клітини щільно прилягають одна до одної, утворюючи пласти. Яка це тканина?

м’язова

хрящова

кісткова

епітеліальна

нервова

1772 / 4299
При гістологічному дослідженні мікропрепарату шкіри людини виявляється тільки щільна неоформлена сполучна тканина. Який шар даного органу був представлений для вивчення?

Сосочковий шар дерми

Підшкірна жирова клітковина

Сітчастий шар дерми

Базальний шар епідермісу

Епідерміс

1773 / 4299
Після ретельного обстеження хворого, який повернувся із Середньої Азії до України, було встановлено попередній діагноз: весняно-літній енцефаліт. Через укус якого членистоногого збудник міг потрапити до організму?

Москіт

Собачий кліщ

Селищний кліщ

Коростяний свербун

Тайговий кліщ

1774 / 4299
Хворому було поставлено попередні дiагноз: парагонімоз. Ця хвороба спричиняється легеневим сисуном. Збудник потрапив до організму хворого під час:

Вживання в їжу напівсирих раків i крабів

Контакту з хворою кішкою

Вживання в їжу немитих овочів

Вживання в їжу напівсирої чи в’яленої риби

Пиття сирої води з відкритих водойм

1775 / 4299
Після обстеження хворого лікар рекомендував йому вилучити з раціону наваристі м’ясні та овочеві бульйони, прянощі, копчені продукти, оскільки у хворого було виявлено:

Зменшення слиновиділення

Зменшення секреції хлористоводневої кислоти залозами шлунка

Збільшення секреції хлористоводневої кислоти залозами шлунка

Дискінезія жовчних шляхів

Зменшення моторики шлунково-кишкового тракту

1776 / 4299
У хворого відмічені ознаки атеросклерозу. Вміст яких транспортних форм ліпідів підвищений у плазмі крові хворого?

Хіломікрони

ЛППЩ

ЛПНЩ

ЛПВЩ

ЛПДНЩ

1777 / 4299
З метою перевірки якості чистоти повітря в операційній було проведено висів повітря на ряд поживних середовищ за допомогою апарата Кротова. Після інкубування на деяких з них виросли колонії, оточені чіткою зоною гемолізу. Які середовища було використано для перевірки повітря?

Середовище Ендо

Кров’яний МПА

Жовтково-сольовий агар

Сироватковий МПА

Вісмутсульфіт агар

1778 / 4299
Лікар-гематолог призначив пацієнту з кровотечею коагулянт, який діє шляхом підвищення синтезу протромбіну та інших факторів зсідання крові переважно в печінці, є синтетичним водорозчинним вітаміном. Який препарат призначив лікар?

Етамзилат

Кальцію хлорид

Вікасол

Гепарин

Тромбін

1779 / 4299
У хворого порушена функція м’язів гомілки. Він не може утримати своє тіло піднявшись навшпиньки. Функція якого м’яза постраждала?

M. flexor digitorum longus

M. tibialis anterior

M. tibialis posterior

M. extensor digitorum longus

M. triceps surae

1780 / 4299
Екзотоксин дифтерійної палички обробили 0,3-0,4% формаліном і витримали 30 днів у термостаті при температурі 40oC. Який препарат був отриманий у результаті проведених маніпуляцій?

Антитоксин

Діагностична сироватка

Анатоксин

Діагностикум

Лікувальна сироватка

1781 / 4299
Після отриманої травми хворий не може змістити нижню щелепу в бік та висунути її вперед. Функція яких м’язів імовірно була пошкоджена?

Скроневі

Щелепно-під’язикові

Жувальні

Щічні

Бічні крилоподібні

1782 / 4299
У хворого 30-ти років на нижній губі виявлена виразка із гладенькою лакованою поверхнею, червоного кольору, хрящоподібної консистенції. Біопсія місця ураження виявила лімфоплазмо-цитарну інфільтрацію, явища васкуліту. Який найбільш імовірний діагноз?

Вторинний сифіліс

Первинний сифіліс

Вроджений сифіліс

Вісцеральний сифіліс

Гума

1783 / 4299
До відділення хірургічної стоматології потрапив чоловік з травматичним розривом arcus venosus jugulae, яка знаходиться в spatium:

Retropharyngeale

Previscerale

Interscalenum

Interaponeuroticum suprasternale

Antescalenum

1784 / 4299
Аналіз ЕКГ хворого виявив відсутність зубця P. Тривалість та амплітуда комплексу QRS та зубця T відповідають нормі. Що є водієм ритму серця даного пацієнта?

Синусний вузол

Передсердно-шлуночковий вузол

Волокна Пуркін’є

Міокард шлуночків

Пучок Пса

1785 / 4299
Деякі білки слини виконують захисну функцію. Який з них захищає слизову оболонку ротової порожнини від механічних ушкоджень?

Муцин

Пероксидаза

Ренін

Каталаза

Лізоцим

1786 / 4299
У пацієнта для іонофорезу при бактеріальному періодонтиті застосували спиртовий розчин йоду. Вкажіть механізм терапевтичної дії цього засобу:

Відновлення нітрогрупи під впливом нітроредуктаз

Утворення альбумінатів

Гальмування утворення клітинної стінки

Заміна атомів водню при атомі азоту в аміногрупі білка

Зміна поверхневого натягу мембрани мікробної клітини

1787 / 4299
Чоловік повернувся з Лівану. Через деякий час він відчув біль і важкість у промежині і надлобковій ділянці. При обстежені йому було встановлено діагноз - урогенітальний шистосо-моз. Яким шляхом він міг заразитися?

Недоварене м’ясо раків і крабів

Недосмажене м’ясо великої рогатої худоби

Недостатньо просолена риба

Купання у заражених водоймах

Через немиті овочі та фрукти

1788 / 4299
Лікар виконує передній серединний розтин шиї для термінової трахео- томії. Про можливе пошкодження якої судини він повинен пам’ятати?

Arcus venosus juguli

Vthyroidea media

Vjugularis interna

V.jugularis externa

Vfacialis

1789 / 4299
До приймального відділення надійшов хворий з важкою щелепно-лицевою травмою. Який препарат йому необхідно ввести для зняття больового шоку?

Пантогам

Ібупрофен

Мідокалм

Сиднокарб

Промедол

1790 / 4299
При обробці фрезою великого кутнього зуба стоматолог інструментом, що зірвався, глибоко поранив щоку, пошкодивши при цьому не лише слизову оболонку але й м’яз. Який?

Щічний

Коловий м’яз рота

Жувальний

Великий виличний м’яз

Щелепно-під’язиковий м’яз

1791 / 4299
Під впливом іонізуючого опромінення або при авітамінозі Е в клітині спостерігається підвищення проникності мембран лізосом. До яких наслідків може призвести така патологія?

Формування веретена поділу

Інтенсивний синтез енергії

Інтенсивний синтез білків

Часткове чи повне руйнування клітини

Відновлення цитоплазматичної мембрани

1792 / 4299
При лікуванні хворого на спадкову форму імунодефіциту було застосовано метод генотерапії: ген ферменту був внесений у клітини пацієнта за допомогою ретровірусу. Яка властивість генетичного коду дозволяє використовувати ретровіруси у якості векторів функціональних генів?

Універсальність

Надмірність

Специфічність

Колінеарність

Безперервність

1793 / 4299
Провідну роль в процесі кальцифі- кації тканин зуба відіграє білок осте-окальцин, який має високу здатність зв’язувати іони кальцію, завдяки наявності в поліпептидному ланцюзі залишків модифікованої амінокислоти:

Аланін

Дельта-амінопропіонова

Іама-аміномасляна

Гама-карбоксиглутамінова

Карбоксиаспарагінова

1794 / 4299
Лікар виявив у дитини рахіт, зумовлений нестачею вітаміну D, але за своїм проявом подібний до спадкового вітаміностійкого рахіту (викривлення трубчастих кісток, деформація суглобів нижніх кінцівок, зубні абсцеси). Як називаються вади розвитку, які нагадують спадкові, але не успадковуються?

Фенокопії

Моносомії

Генокопії

Трисомії

Іенні хвороби

1795 / 4299
У хворого 69-ти років на шкірі в ділянці нижньої повіки з’явилося невелике бляшкоподібне утворення, з послі-дуючим виразкуванням, яке було оперативно видалене. При мікроскопічному дослідженні утворення: в дермі шкіри комплекси з атипових епітеліальних клітин, на периферії утворень клітини розташовані перпендикулярно до базальної мембрани. Клітини темні, призматичної полігональної форми; ядра гіперхромні з частими мітозами. Іноді зустрічаються утворення, подібні до волосяного фолікула. Яка гістологічна форма рака у хворого?

Плоскоклітинний з ороговінням

Недиференційований

Аденокарцинома

Плоскоклітинний без ороговіння

Базально-клітинний

1796 / 4299
Жінка 35-ти років хворіє на ВІЛ-інфекцію на стадії СНІД. На шкірі нижніх кінцівок, слизової оболонки піднебіння з’явились рудувато-червоні плями, яскраво-червоні вузлики різних розмірів. Один з вузликів взято на гістологічне дослідження. Виявлено багато хаотично розташованих тонко- стінних судин, вистелених ендотелієм; пучки веретеноподібних клітин з наявністю гемосидерину. Яка пухлина розвинулась у хворої?

Лімфома Беркіта

Фібросаркома

Гемангіома

Лімфангіома

Саркома Капоші

1797 / 4299
Хворий звернувся до лікаря з гострим зубним болем в нижньому лівому іклі. Поставлено діагноз - пульпіт. Який нерв забезпечує чутливість цього зуба?

Лицевий

Нижній комірковий

Виличний

Піднебінний

Верхній комірковий

1798 / 4299
Відомо, що стероїдні протизапальні препарати гальмують активність фо-сфоліпази А2, що необхідна для синтезу простагландинів. Яка речовина є попередником цих медіаторів запалення?

Проопіомеланокортин

Арахідонова кислота

Тирозин

Пальмітинова кислота

Холестерол

1799 / 4299
В стоматологічній практиці для дослідження збудливості тканин зубів використовують метод електроодонтоді-агностики. При цьому визначається:

Акомодація

Хронаксія

Корисний час

Лабільність

Поріг сили подразника

1800 / 4299
Студент перед іспитом скаржився на гострий зубний біль, який послабився під час складання іспиту. Яке гальмування зумовило зменшення больових відчуттів?

Диференціювальне

Зовнішнє

Запізніле

Згасаюче

Позамежне

1801 / 4299
У районах Південної Африки у людей розповсюджена серпоподібно-клітинна анемiя, при якій еритроцити мають форму серпа внаслідок зміни в молєкулі гемоглобіну амінокислоти глутаміну на валін. Причиною цієї хвороби є:

Трансдукція

Кросинговер

Геномні мутації

Генна мутація

Порушення механізмів реалізації генетичної інформації

1802 / 4299
При обстеженні дівчини 18-ти років знайдені наступні ознаки: недорозвинення яєчників, широкі плечі, вузький таз, вкорочення нижніх кінцівок, 'шия сфінкса', розумовий розвиток не порушений. Встановлено діагноз: синдром Шерешевського-Тернера. Яке хромосомне порушення у хворої?

Трисомія Х

Моносомія Х

Трисомія 13

Нульсомія Х

Трисомія 18

1803 / 4299
У генетично здорових батьків народилася дитина, хвора на фенілкетонурію (аутосомно-рецесивне спадкове захворювання). Які генотипи батьків?

АА х Аа

АА х АА

Аа х Аа

аа х аа

Аа х аа

1804 / 4299
До лікаря звернувся пацієнт 25-ти років з приводу сильного свербіжу шкіри, особливо між пальцями рук, у пахвових западинах, на нижній частині живота. При огляді шкіри хворого виявлено звивисті ходи білувато-брудного кольору. Який діагноз міг припустити лікар?

Демодекоз

Дерматотропний лейшманіоз

Скабієс

Міаз

Педикульоз

1805 / 4299
До приймальної медико-генетичної консультації звернулась пацієнтка. При огляді виявились наступні симптоми: трапецієподібна шийна складка (шия 'сфінкса'), широка грудна клітка, слабо розвинені молочні залози. Який найбільш імовірний діагноз?

Синдром Шерешевського-Тернера

Синдром Клайнфельтера

Синдром Мориса

Синдром 'крику кішки'

Синдром Патау

1806 / 4299
До жіночої консультації звернулася жінка 26-ти років, у якої було два мимовільні викидні. Яке протозойне захворювання могло спричинити невиношування вагітності?

Трипаносомоз

Лямбліоз

Лейшманіоз

Токсоплазмоз

Трихомонадоз

1807 / 4299
Після черепно-мозкової травми у хворого 38-ми років відсутнє відчуття нюху і смаку. Які коркові центри кінцевого мозку постраждали?

Operculum frontale

Sul. calcarinus

Gyrus temporalis superior

Gurus postcentralis

Uncus

1808 / 4299
До уролога звернувся чоловік 58-ми років зі скаргами на різкі болі при сечовиділенні та зменшення кількості сечі. Лікар припустив наявність сечокам’яної хвороби. В якій частині чоловічої уретри найбільш імовірне затримання каміння?

Pars pelvina

Pars intramuralis

Pars spongiosa

Pars prostatica

Pars membranacea

1809 / 4299
Оглядаючи ротову порожнину у хворого 19-ти років, стоматолог звернув увагу на зруйнований язиково-дистальний горбик жувальної поверхні лівого першого верхнього моляра. Цей горбик називається:

Гіпоконус

Метаконус

Мезоконус

Протоконус

Параконус

1810 / 4299
Хворий із запаленням слизової язика (глосит) скаржиться на розлад смакової чутливості передніх двох третин язика. Ураженням якого нерва воно викликане?

Малий кам’янистий

Барабанний

Барабанна струна

Язиковий

Язикоглотковий

1811 / 4299
До відділення хірургічної стоматології надійшла новонароджена дівчинка, яка при смоктанні починала поперхуватись. При обстеженні виявлена розщілина твердого піднебіння, яка була наслідком незрощен-ня середнього лобового відростка з верхньощелепним відростком I-ої зябрової дуги. Розщілина знаходилась у піднебінні між:

В дїлянщ canalis incisivus

Processus palatinus maxillae dextrae et sinistrae

Lamina horizontalis os palatinum dextrum et sinistrum

Os incisivum et processus palatinus maxillae

Processus palatinus maxillae et lamina horizontalis os palatinum

1812 / 4299
Чоловік 63-х років звернувся до стоматолога зі скаргою, що в нього нижня щелепа не рухається назад. Встановлено, що в нього через падіння пошкоджений такий м’яз:

Двочеревцевий

Присередній крилоподібний

Жувальний

Бічний крилоподібний

Скроневий

1813 / 4299
У новонародженої дитини протягом першої доби педіатр помітив відсутність акту дефекації. Про яку ваду розвитку свідчить цей факт?

Атрезія заднього проходу

Дивертикул стравоходу

Дивертикул порожньої кишки

Атрезія стравоходу

Заяча губа

1814 / 4299
Хворий звернувся до лікаря зі скаргами на утруднення під час жування. При обстеженні виявлена атрофія правих скроневого і жувального м’язів. При відкриванні рота щелепа відхиляється ліворуч. Який нерв уражений?

Лицевий

Верхньощелепний

Нижній альвеолярний

Щелепно-під’язиковий

Рухова частина нижньощелепного

1815 / 4299
У постраждалого 46-ти років визначається перелом у ділянці внутрішньої поверхні лівого гомілковосту-пневого суглоба. Де найбільш імовірно відбувся перелом?

Латеральна кісточка

Медіальна кісточка

П’яткова кістка

Нижня третина малогомілкової кістки

Надп’яткова кістка

1816 / 4299
При обстеженні хворого 40-ка років виявлено опущення верхньої повіки, розхідну косоокість, розширення зіниці, обмеженість рухливості очного яблука. Який нерв пошкоджений?

Очний

Відвідний

Блоковий

Зоровий

Окоруховий

1817 / 4299
У травмованого 27-ми років двосторонній вивих скронево-нижньощелепного суглоба. Які основні зв’язки суглоба будуть ушкоджені?

Крилоподібно-щелепні

Шило-щелепні

Внутрішньосуглобові

Клиноподібно-щелепні

Латеральні

1818 / 4299
У обстежуваного відзначається різке опущення кута рота зліва. Порушенням функції яких м’язів це ви- кликано?

Коловий м’яз рота

Виличні м’язи

М’яз, що опускає кут рота

М’яз сміху

М’яз, що підіймає верхню губу i крило носа

1819 / 4299
У гістопрепараті головного кінця зародка довжиною 6 мм на передньо-бічній поверхні шиї видні валикоподібні підвищення, що виникли за рахунок розростання мезенхіми. Як називаються дані утворення зябрового апарату?

Зяброві щілини

Зяброві дуги

Глоткові кишені

Зяброві перетинки

Зяброві кишені

1820 / 4299
В гістологічному препараті молочного зуба дитини визначається гіпоплазія (недорозвинення) емалі. З діяльністю яких клітин пов’язані ці порушення?

Зовнішні емалеві клітини

Одонтобласти

Клітини пульпи емалевого органу

Клітини проміжного шару емалевого органу

Внутрішні емалеві клітини

1821 / 4299
На електронній мікрофотографії поперечного шліфу зуба в емалі виявляються утворення овальної, полігональної і аркової форм, що складаються з щільно вкладених і упорядкованих кристалів гідроксиапатитів. Назвіть дане утворення:

Смуги Іунтера-Шрегера

Колагенове волокно

Лінії Ретціуса

Перикіматій

Емалева призма

1822 / 4299
При огляді ротової порожнини пацієнта 47-ми років стоматолог звернув увагу, що його язик має грубі гіпертрофовані сосочки, глибокі борозни. Лікар порадив хворому звернутися до гастроентеролога. При обстеженні з’ясувалося, що у нього значно підвищена кислотність шлункового соку. Гіперфункцією яких клі- тин у залозах слизової оболонки шлунка переважно обумовлений цей стан?

Келихоподібні клітини

Екзокриноцити шлункових залоз

Парієтальні екзокриноцити власних залоз шлунка

Екзокринні панкреатоцити

Додаткові мукоцити

1823 / 4299
У препараті червоного кісткового мозку людини визначаються скупчення гігантських клітин, розташованих в тісному контакті з синусоїдними капілярами. Назвіть формені елементи крові, які утворюються з цих клітин:

Лейкоцити

Кров’яні пластинки

Еритроцити

Моноцити

Лімфоцити

1824 / 4299
Гістологічна картина ендометрію має наступні характерні ознаки: потовщення, набряк, наявність звивистих залоз із розширеним просвітом, які секретують велику кількість слизу, мітози в клітинах не спостерігаються, у стромі наявні децидуальні клітини. Яка стадія менструального циклу відповідає описаній картині?

Секреторна (пременструальна)

Проліферативна

Відносного спокою

Регенераторна

Менструальна

1825 / 4299
У препараті в одному з судин мі-кроциркуляторного русла середня оболонка утворена 1-2 шарами гладеньких міоцитів, які розташовані поодинці і мають спіралеподібний напрямок. Зовнішня оболонка представлена тонким шаром пухкої сполучної тканини. Вкажіть вид судини:

Венула

Артеріола

Посткапіляр

Капіляр

Артеріоловенулярний анастомоз

1826 / 4299
В гістологічному препараті стінки очного яблука визначається структура, в якій відсутні кровоносні судини. Яке утворення характеризується даною морфологічною ознакою?

Цилiарне тіло

Сітківка

Судинна оболонка

Рогівка

Райдужна оболонка

1827 / 4299
Під час мікроскопічного дослідження органу ЦНС виявлена сіра речовина, у якій нейрони утворюють три шари: молекулярний, гангліонар-ний і зернистий. Назвіть нейрони, що формують другий шар:

Кошикові

Грушоподібні

Клітини-зерна

Дрібні зірчасті

Великі зірчасті

1828 / 4299
У процесі набуття клітинами специфічних для них морфологічних, біохімічних та функціональних особливостей клітини обмежуються у виборі можливостей шляхів розвитку. Яку назву має таке набуте обмеження?

Комітування

Капацитація

Детермінація

Рецепція

Адгезія

1829 / 4299
Накладення стоматологічного протезу викликало у пацієнта збільшення слиновиділення. Це обумовлено реалізацією таких механізмів регуляції:

Безумовні рефлекси

Безумовні та умовні рефлекси

Місцеві рефлекси

Умовні рефлекси

1830 / 4299
Необхідно оцінити перетравлюючі властивості слини. З яким субстратом для цього її треба змішати?

Жир

Крохмаль

Казеін

РНК

ДНК

1831 / 4299
У хворого з пересадженим серцем при фізичному навантаженні збіль- шився хвилинний об’єм крові. Який механізм регуляції забезпечує ці зміни?

Парасимпатичні умовні рефлекси

Парасимпатичні безумовні рефлекси

Симпатичні умовні рефлекси

Катехоламіни

Симпатичні безумовні рефлекси

1832 / 4299
У експериментальної тварини подразнювали периферичний відрізок симпатичних волокон, що іннерву-ють під’язикову слинну залозу. У результаті з фістули протоки залози виділяється:

Слина не виділяється

Багато рідкої слини

Мало рідкої слини

Мало в’язкої слини

1833 / 4299
При обстеженні хворого з травматичним пошкодженням головного мозку виявлено, що він втратив дотикову чутливість. Який відділ кори мозку пошкоджений?

Лобна частка кори

Тім’яна частка кори

Потилична частка кори

Задня центральна звивина

Передня центральна звивина

1834 / 4299
Студентка 18-ти років має масу тіла 50 кг. Робочий (загальний) обмін студентки складає 11 000 кДж/д. Якою повинна бути калорійність харчового раціону студентки, якщо вона не хоче змін маси тіла?

10 000 -11 000 кДж/д

9 000 -10 000 кДж/д

11 000 -12 000 кДж/д

12 000 -13 000 кДж/д

10 500 -11 500 кДж/д

1835 / 4299
У жінки 32-х років запалення ясен супроводжується їх гіпоксією. Утворення якого метаболіту вуглеводного обміну значно збільшується при цьому в тканинах пародонта?

НАДФ-Н

Глюкозо-6-фосфат

Рибозо-5-фосфат

Глікоген

Лактат

1836 / 4299
При обстеженні хворого виявлені дерматит, дiарея, деменція. Відсутність якого вітаміну є причиною цього стану?

Рутин

Нікотинамід

Аскорбінова кислота

Фолієва кислота

Біотин

1837 / 4299
У хворого, що страждає на стрептококову інфекцію, розвинувся геморагічний діатез. Яка причина підвищеної кровоточивості?

Збільшення кількості гепарину в плазмі крові

Посилений фібриноліз

Збільшення кількості калікреїну в плазмі крові

Нестача вітаміну С

Нестача вітаміну А

1838 / 4299
Еритроцити людини не містять мітохондрій. Який основний шлях утворення АТФ в цих клітинах?

Аеробний гліколіз

Окиснювальне фосфорилювання

Аденілаткіназна реакція

Креатинкіназна реакція

Анаеробний гліколіз

1839 / 4299
Хворому з печінковою недостатністю проведено дослідження електрофоретичного спектру білків сироватки крові. Які фізико-хімічні властивості білкових молекул лежать в основі цього методу?

Наявність заряду

Нездатність до діалізу

Гідрофільність

Оптична активність

Здатність набрякати

1840 / 4299
Педіатр під час огляду дитини відзначив відставання у фізичному і розумовому розвитку. В аналізі сечі був різко підвищений вміст кетокислоти, що дає якісну кольорову реакцію з хлорним залізом. Яке порушення обміну речовин було виявлене?

Цистинурія

Фенілкетонурія

Алкаптонурія

Альбінізм

Тирозинемія

1841 / 4299
У хворого 37-ми років після автомобільної катастрофи АТ- 70/40 мм рт.ст. Хворий у непритомному стані. За добу виділяє близько 300 мл сечі. Який механізм порушення сечоутво-рення у даному випадку?

Зменшення клубочкової фільтрації

Зменшення канальцевої секреції

Посилення клубочкової фільтрації

Зменшення канальцевої реабсорбції

Посилення канальцевої реабсорбції

1842 / 4299
Жінка із токсикозом вагітності страждає на гіперсалівацію, що призводить до втрати 3-4 літрів слини щоденно. Яке порушення водно-сольового обміну виникає при цьому?

Гіпонатріємія

Гіпогідратація гіпоосмолярна

Гіпокаліємія

Гіпогідратація ізоосмолярна

Гіпогідратація гіперосмолярна

1843 / 4299
Хвора поступила в клініку на обстеження. З дитинства відмічалось зниження гемоглобіну до 90-95 г/л. Лікування препаратами заліза було неефективним. У крові: ер- 3,21012/л, Hb- 85 г/л, КП- 0,78. В мазку: ані-зоцитоз, пойкілоцитоз, мішенеподі-бні еритроцити, ретикулоцити -16%. Поставлений діагноз - таласемія. До якого виду гемолітичних анемій можна віднести дане захворювання?

Спадкова гемоглобінопатія

Спадкова мембранопатія

Спадкова ферментопатія

Набута ферментопатія

Набута мембранопатія

1844 / 4299
У хворого на цукровий діабет відзначається високий рівень гіперглікемії, кетонурія, глюкозурія. Гіпер-стенурія і поліурія. Яка форма порушення кислотно-основної рівноваги має місце у даній ситуації?

Метаболічний ацидоз

Газовий алкалоз

Газовий ацидоз

Видільний алкалоз

Метаболiчний алкалоз

1845 / 4299
При електрокардіографічному дослідженні пацієнта 59-ти років, хворого на гіпертонічну хворобу, виявлено: ритм синусовий, правильний, ЧСС- 92/хв, тривалість РQ- 0,2 с, QRS- не змінений. У хворого порушена така властивість серця:

Скоротливість

Рефрактерність

Автоматизм

Провідність

Збудливість

1846 / 4299
У чоловіка 54-x років, який 12 років тому хворів на ревматичний міокардит та ендокардит, наявна недостатність мітрального клапану. Дослідження показали, що запального процесу зараз немає, хвилинний об’єм кровообігу достатній. Якому поняттю загальної нозології відповідає дана умова?

Патологічний процес

Патологічний стан

Патологічна реакція

Типовий патологічний процес

Компенсаторна реакція

1847 / 4299
Через 8 днів після опромінення у ліквідатора аварії на АЕС розвинулись виразково- некротичні зміни у порожнині рота. У крові: ер-3,2 • 1012/л, ретикулоцитів 0,0l%, Hb-60 г/л, лейк.- 2,3- 109/л, тромбоцитів 50 тис/л. Для якого періоду променевої хвороби характерні описані зміни?

Закінчення хвороби

Період розвинутих клінічних ознак

Період уявного благополуччя

Прихований період

Період первинних реакцій

1848 / 4299
Епідеміологічне дослідження розповсюдження пухлин виявило високу кореляцію розвитку пухлин легень з тютюнопалінням. З дією якого хімічного канцерогену найбільш імовірно пов’язано виникнення даного виду патології?

Ортоаміноазотолуол

Метилхолантрен

3,4-бензпірен

Діетілнітрозамін

Афлатоксин

1849 / 4299
При розвитку гострого пульпіту в хворого 24-х років відмічаються на-падоподібні болі у верхній лівій щелепі, що підсилюються вночі, гарячка; в крові - лейкоцитоз. Який вид лейкоцитозу можливий в даному випадку?

Нейтрофільний

Моноцитоз

Лімфоцитоз

Еозинофільний

Базофільний

1850 / 4299
Дитина 10-ти років під час гри порізала ногу відламком скла і була направлена у поліклініку для введення протиправцевої сироватки. З метою попередження розвитку анафілактичного шоку лікувальну сироватку вводили за Безредкою. Який механізм лежить в основі подібного способу гіпосенсибілізації організму?

Зв’язування рецепторів до IgE на опасистих клітинах

Зв’язування фіксованих на опасистих клітинах IgE

Блокування синтезу медіаторів опасистих клітин

Стимуляція синтезу антиген-специфічних IgG2

Стимуляція імунологічної толерантності до антигену

1851 / 4299
У пацієнта 46-ти років у ділянці 4 премоляра виявлено пухлину сіруватого кольору При гістологічному дослідженні: пухлина складається із острівців проліферуючего одонто- генного епітелію і пухкої тканини, яка нагадує тканину зубного сосочка сполучної тканини. Який з перелічених діагнозів найбільш імовірний?

Одонтоамелобластома

Одонтогенна міксома

Одонтогенна фіброма

Амелобластична одонтосаркома

Амелобластична фіброма

1852 / 4299
У хворого 67-ми років, що страждає на атеросклероз, з’явився 6іль у правій стопі. Стопа збільшена в розмірі, шкірні покриви чорного кольору, мацеровані, демаркаційна зона не виражена. Про який патологічний процес у стопі можна думати?

Нома

Секвестр

Коагуляційний некроз

Волога гангрена

Суха гангрена

1853 / 4299
При гістологічному дослідженні стінки кісти, що локалізується у ділянці верхньої щелепи, встановлено, що стінка кісти з середини вистелена багатошаровим плоским епітелієм з підлеглою грануляційною тканиною з лімфолейкоцитарною інфільтрацією. Зовнішній шар представлений пухкою волокнистою сполучною тканиною, оточеною рубцевою фіброзною тканиною. Ці дані є підставою для встановлення такого діагнозу:

Кератокіста

Кістогранульома

Амелобластома

Епітеліальна гранульома

Проста гранульома

1854 / 4299
При обстеженні у хворого на твердому піднебінні виявлена пухлина у вигляді невеликого щільного вузла сірого кольору без чітких меж. Після видалення пухлина досліджена гістологічно. Побудована з дрібних, кубічної форми клітин з гіперхромним ядром, які формують альвеоли, трабекули, солідні та кріброзні структури. Ріст пухлини - інвазивний. Назвіть пухлину:

Мономорфна аденома

Аденокістозна карцинома

Мукоепідермоїдний рак

Аденолімфома

Злоякісна плеоморфна аденома

1855 / 4299
Хворий 58-ми років впродовж багатьох років страждав на хронічний остеомієліт нижньої щелепи. Останнім часом в аналізі крові - гіпопроте-їнемія, диспротеїнемія; у сечі - про- теїнурія, білкові циліндри. Помер від хронічної ниркової недостатності. На розтині нирки збільшені, щільні, вос-коподібні. Який патологічний процес у нирках був виявлений на розтині?

Інтерстиційний нефрит

Хронічний піелонефрит

Гідронефроз

Амілоїдоз

Хронічний гломерулонефрит

1856 / 4299
У хворого в обох щелепах рентгенологічно виявлено численні дефекти у вигляді гладкостінних округлих отворів. При гістологічному дослідженні - явища остеолізису і остео-порозу при явищах слабкого кістко-утворення. В сечі хворого знайдено білок Бенс- Джонса. Який найбільш імовірний діагноз?

Хронічний мієлолейкоз

Хронічний еритромієлоз

Гострий недиференційований лейкоз

Гострий мієлолейкоз

Мієломна хвороба

1857 / 4299
У хворого 28-ми років верхня щелепа розширилась, потовщилась, деформуючи обличчя. При біопсії спостерігається волокниста фіброзна тканина, серед якої визначаються малозвапнені кісткові балки примітивної будови та остеоїдні балочки. Який найбільш імовірний діагноз?

Херувізм

Остеомієліт

Ревматоїдний артрит

Моноосальна фіброзна дисплазія

Поліосальна фіброзна дисплазія

1858 / 4299
Після введення лідокаїну в хворого 25-ти років з’явились задишка, бронхоспазм, різко знизився кров’яний тиск, що вимагало від лікаря-стоматолога застосування засобів негайної допомоги. Який механізм лежить в основі таких явищ?

Алергічні цитотоксичні реакції

Ефекти, зумовлені Т-кілерами

Гіперчутливість, зумовлена Т-лімфоцитами

Явище ідіосинкразії

Алергічні реакції за участю IgE

1859 / 4299
З випорожнень хворої дитини 6-місячного віку, яка знаходилась на штучному вигодовуванні, виділена культура кишкової палички з антигенною структурою 0-111. Який діагноз можна поставити?

Дизєнтєрієподі6нє захворювання

Холєроподі6нє захворювання

Колі-ентерит

Харчове отруєння

Гастроентерит

1860 / 4299
Бактеріолог при досліджєнні крові та слизу із носоглотки дотримувався певних заходів щодо збереження збудників у матеріалі. При бактеріо-скопічному дослідженні встановлено наявність грамнегативних коків, які нагадують кавові зерна і розташовані парами, або тетрадами. Назвіть збудника, який був ізольований бактеріологом:

Acinetobacter calcoaceticus

Neisseria meningitidis

Staphilococcus aureus

Neisseria gonorrhoeae

Moraxella lacunata

1861 / 4299
До інфекційного відділення госпіталізовано хворого 27-ми років зі скаргами на багаторазові пронос та блювання, біль у м’язах ніг, слабкість, запаморочення. Після огляду лікар поставив попередній діагноз 'холера'. Як необхідно досліджувати матеріал від хворого для експрес діагнозу?

Пряма і непряма РІФ

Серологічний метод

Біологічний метод

Бактеріологічний метод

РА

1862 / 4299
З виділень уретри хворого на уретрит в’ялого перебігу виділено чисту культуру кокоподібних мікроорганізмів. Виділений мікроорганізм в короткому строкатому ряду ферментує лише глюкозу до кислоти. Назвіть рід і вид виділеного мікроорганізму:

Streptococcus pyogenes

Staphylococcus aureus

Neisseria meningitides

Enterococcus faecalis

Neisseria gonorrhoeae

1863 / 4299
Хворому на туберкульоз 34-х років, в анамнезі якого була відкрита легенева форма захворювання, проведено мікроскопічне дослідження харкотиння з метою виявлення збудника. Який метод забарвлення доцільно використати при цьому?

Метод Нейссера

Метод Грама

Метод Буррі-Гінса

Метод Романовського-Гімзи

Метод Ціля-Нільсена

1864 / 4299
Лікар, мікроскопуючи мазок крові, що забарвлений за Романовським, виявив найпростіші у формі півмісяця, протоплазма яких вакуолізована і забарвлена у блакитний колір, а ядро - у червоний. Які найпростіші найбільш імовірно були у крові?

Токсоплазми

Трипаносоми

Лямблії

Балантидії

Лейшманії

1865 / 4299
У дитячому відділенні інфекційної клініки хлопчику 4-х років поставлено діагноз 'дифтерія'. Який препарат потрібно ввести хворому в першу чергу?

АДП

Протидифтерійна антитоксична сироватка

TABt

АКДП

Дифтерійний анатоксин

1866 / 4299
У пацієнта з попереднім діагнозом 'сифіліс'лаборант взяв сироватку крові для постановки імунної реакції, яка основана на виявленні антитіл, які припиняють рух трепонем і призводять до їх загибелі. Яку реакцію було використано для діагностики?

Реакція іммобілізації

Реакція преципітації

Реакція зв’язування комплементу

Реакція нейтралізації

Реакція аглютинації

1867 / 4299
При повторній постановці реакції аглютинації Відаля виявлено наростання у сироватці хворого титрів антитіл до О-антигенів S.typhi з 1:100 до 1:400. Як можна розцінити отри- мані результати?

Раніше перехворів на черевний тиф

Хворіє на черевний тиф

Раніше був щеплений проти черевного тифу

Є гострим носієм черевнотифозних мікробів

Є хронічним носієм черевнотифозних мікробів

1868 / 4299
На прийомі у лікаря-стоматолога у хворого 37-ми років з хронічним холециститом на фоні жовчнокам’яної хвороби розвинувся гострий напад болю у правому підребер’ї. Які препарати найбільш раціонально призначити?

Аллохол

Холензим

Сульфат магнію

Но-шпа

Анальгін

1869 / 4299
У хворої з артеріальною гіпертензією розвинувся напад бронхіальної астми. Який з нижче перерахованих бронхолітичних засобів може спровокувати гіпертонічний криз?

Сальбутамол

Ефедрину гідрохлорид

Еуфілін

Беротек

Ізадрин

1870 / 4299
Для корекції артеріального тиску при колаптоїдному стані хворому було введено мезатон. Який механізм дії даного препарату?

Стимулює а-адренорецептори

Стимулює а- в-адренорецептори

Блокує в-адренорецептори

Стимулює в-адренорецептори

Блокує а-адренорецептори

1871 / 4299
У хворого на прийомі у стоматолога виник пароксизм тахікардії. Який з названих засобів слід використати для його купірування?

Верапаміл

Дифенін

Ізадрин

Нітрогліцерин

Атропін

1872 / 4299
Чоловік 49-ти років звернувся до лікаря зі скаргами на біль за грудниною, який виникає у нього на вулиці, по дорозі на роботу, при фізичному напруженні. Хворіє протягом року. Спочатку біль зникав після прийому валідолу, але в останні дні його прийом став неефективним. Який препарат слід призначити хворому?

Нітрогліцерин

Клофелін

Калію хлорид

Сальбутамол

Октадин

1873 / 4299
Хворому 28-ми років на бактеріальну пневмонію призначили курс лікування еритроміцином. Відомо, що його антибактеріальні властивості зумовлені здатністю цього середника сполучатися з вільною 50S-субодиницею рибосоми. Синтез яких речовин блокує цей антибіотик у бактеріальних клітинах?

Жири

Полісахариди

ДНК

Білки

РНК

1874 / 4299
При огляді ротової порожнини у пацієнта 22-х років спостерігається нахил вперед верхніх та нижніх зубів, з перекриттям нижніх зубів верхніми. Якому прикусу характерне дане розміщення зубів?

Біпрогнатія

Ортогнатія

Прямий прикус

Прогенія

Закритий прикус

1875 / 4299
Для попередження сезонного підйому захворюваності на грип у лікувальних закладах міста санепідстан-ція зобов’язала провести імунізацію медпрацівників. Яким із перелічених препаратів слід проводити імунізацію?

Субодинична вакцина

Ремантадин

Амантадин

Інтерферон

Іамма-глобулін

1876 / 4299
Під час обстеження пацієнта 20- ти років ЛОР-лікар діагностував запалення верхньощелепних пазух. В якому носовому ході під час риноскопії був виявлений гній?

Загальний

Нижній

Верхній

Середній

Найвищий

1877 / 4299
Встановлено, що до складу пестициду входить арсенат натрію, який блокує ліпоєву кислоту. Вкажіть, активність яких ферментів порушується:

Метгемоглобінредуктаза

Мікросомальне окиснення

Ілутатіонпероксидаза

Ілутатіонредуктаза

ПВК-дегідрогеназньїй комплекс

1878 / 4299
До приймального відділення був доставлений хворий 56-ти років у непритомному стані з вираженим пригніченням дихання та серцевої діяльності, зниженням сухожильних рефлексів. За словами супроводжуючої його дружини, останнім часом він страждав на безсоння та приймав на ніч снодійні засоби. Який лікарський препарат необхідно ввести хворому для невідкладної допомоги?

Дипіроксим

Унітіол

Бемегрид

Протаміну сульфат

Атропіну сульфат

1879 / 4299
Хворий 44-х років скаржиться на часті нудоти, які нерідко завершуються блюванням. Порушення якої із функцій шлунка найімовірніше можна запідозрити у даного хворого?

Екскреторна

Евакуаторна

Всмоктувальна

Секреторна

Інкреторна

1880 / 4299
Дівчинка 11-ти років часто хворіє на гострі респіраторні інфекції, після яких спостерігаються множинні точкові крововиливі в місцях тертя одягу. Гіповітаміноз якого вітаміну має місце в дівчинки?

A

В2

Вб

Ві

C

1881 / 4299
Тварині, сенсибілізованій туберкуліном, внутрішньоочеревенно введений туберкулін. Через 24 години при лапаратомії виявлено венозну гіперемію та набряк очеревини. У мазках-відбитках з очеревини велика кількість лімфоцитів та моноцитів. Який патологічний процес у тварини?

Серозне запалення

Фібринозне запалення

Асептичне запалення

Алергічне запалення

Гнійне запалення

1882 / 4299
За даними ВООЗ щорічно на Землі малярією хворіють приблизно 250 млн чоловік. Ця хвороба зустрічається переважно у тропічних і субтропічних областях. Межі ії розповсюдження співпадають з ареалами комарів роду:

Кулізета

Анофелес

Мансоніа

Аедес

Кулекс

1883 / 4299
У жінки з резус-негативною кров’ю А (II) групи народилася дитина з АВ (IV) групою, у якої діагностували гемолітичну хворобу внаслідок резус-конфлікту. Яка група крові можлива у батька дитини?

III (В), резус-позитивна

II (А), резус-позитивна

III (В), резус-негативна

IV (АВ), резус-негативна

І (0), резус-позитивна

1884 / 4299
Який препарат слід призначити для лікування хворій 54-х років з хронічною серцевою недостатністю, що супроводжується тахіаритмією та набряками?

Спіронолактон

Метопролол

Дигоксин

Аміодарон

Анаприлін

1885 / 4299
У процесі звапнування міжклітинної речовини кісткової тканини вздовж колагенових волокон відкладаються кристали гідроксиапатиту. Для реалізації цього процесу необхідна присутність у міжклітинній речовині лужної фосфатази. Яка клітина продукує цей фермент?

Хондроцит

Остеокласт

Хондробласт

Остеобласт

Остеоцит

1886 / 4299
Хворий 46-ти років знаходиться в урологічному відділенні з приводу загострення хронічного циститу. Який з антисептичних засобів можна застосувати для промивання сечового міхура?

Ко-тримоксазол

Фурацилін

Перекис водню

Брильянтовий зелений

Розчин йоду спиртовий

1887 / 4299
З метою масового обстеження студентів на носійство S.aureus перед виробничою практикою у дитячому відділенні клінічної лікарні було використано елективне середовище з метою отримання чистої культури цього збудника. Яке з перерахованих середовищ було використано?

Кров’яний телуритовий агар

Середовище Вільсона-Блера

Середовище Ендо

М’ясо-пептонний агар

Жовтково-сольовий агар

1888 / 4299
Хворому 30-ти років для лікування пневмонії лікар на 3 дні призначив антибіотик з групи азалідів, що має бактерицидну дію, тривалий ефект, здатність зв’язуватись з фагоцитами і накопичуватись у вогнищах інфекції. Який препарат було призначено хворому?

Азитроміцин

Бензилпеніциліну натрієва сіль

Ципрофлоксацин

Еритроміцин

Ізоніазид

1889 / 4299
У хворого 35-ти років після перенесеного гепатиту розвинулася печінкова недостатність. Порушення якої із функцій печінки при цьому запускає механізм утворення набряків?

Ілікогенутворююча

Білковоутворююча

Жовчоутворююча

Антитоксична

Бар’єрна

1890 / 4299
У хворого 64-х років часті напади стенокардії. Два роки тому він переніс інфаркт міокарда. Вкажіть групу лікарських засобів, препарати якої доцільно призначити для профілактики розвитку повторного інфаркту міокарда:

Коагулянти

Фібринолітики

Антиагреганти

Антикоагулянти непрямої дії

Антагоністи гепарину

1891 / 4299
Хворий 41-го року звернувся до офтальмолога для обстеження очного дна. В кон’юнктивальний мішок ока лікар закрапав розчин, який викликав розширення зіниці і параліч акомодації. Розчин якого препарату закрапали в око?

Прозерин

Фурацилін

Іалантаміну гідробромід

Пілокарпіну гідрохлорид

Атропіну сульфат

1892 / 4299
У хворої 63-х років стався напад миготливої аритмії. При обстеженні лікар виявив гіпотонію (АТ- 80/45 мм рт.ст). Який препарат можна призначити разом з антиаритмічним для попередження колапсу?

Етимізол

Кофеїн-бензоат натрію

Адреналін

Мезатон

Теофілін

1893 / 4299
У хворого 55-ти років, що знахо- диться у кардіологічному відділенні з приводу серцевої недостатності, виявлені зміни показників гемодинамі-ки. Які з них найбільш інформативні для підтвердження вказаної патології?

Підвищення діастолічного артеріального тиску

Підвищення частоти серцевих скорочень

Зменшення хвилинного об’єму крові

Підвищення венозного тиску

Підвищення систолічного артеріального тиску

1894 / 4299
У дитини 12-ти років низький зріст при непропорційній будові тіла і розумовій відсталості. Недостатня секреція якого гормону може бути причиною таких порушень?

Кортизол

Інсулін

Тироксин

Глюкагон

Соматотропін

1895 / 4299
У реанімаційному відділенні знаходиться хворий 49-ти років у коматозному стані. При дослідженні крові відзначено збільшення концентрації K+, зменшення вмісту Ca++, ацидоз, збільшення рівня сечовини, сечової кислоти. Який вид коми за етіологією найбільш імовірний?

Гіпоглікемічна

Нейрогенна

Печінкова

Ниркова

Діабетична

1896 / 4299
При гістологічному дослідженні печінки визначається різке порушення часточкової будови з інтенсивним фіброзом і формуванням вузлів регенерації. Мікроскопічно: проліферація гепатоцитів, поява несправжніх часточок, дистрофія і некроз гепато-цитів. Який найбільш імовірний діагноз?

Вірусний гепатит

Цироз печінки

Алкогольний гепатит

Прогресуючий масивний некроз печінки

Жировий гепатоз

1897 / 4299
Який з легеневих об’ємів НЕМОЖЛИВО визначити за допомогою спірометрії?

Життєва ємність легенів

Дихальний об’єм

Залишковий об’єм

Резервний об’єм видиху

Резервний об’єм вдиху

1898 / 4299
Внаслідок переливання несумісної крові за антигеном Rh у хворої виникла гемолітична жовтяниця. Який лабораторний показник крові підтверджує цей тип жовтяниці?

Зменшення вмісту стеркобіліну

Нагромадження некон’югованого білірубіну

Зменшення вмісту кон’югованого білірубіну

Нагромадження уробіліногену

Зменшення вмісту некон’юговано-го білірубіну

1899 / 4299
Для лікування бронхіту дитині 8-ми років необхідно призначити антибактеріальний засіб. Який з вказаних препаратів з групи фторхіноло-нів ПРОТИПОКАЗАНИЙ в цьому віці?

Сульфадиметоксин

Ципрофлоксацин

Ампіцилін

Ампіокс

Амоксицилін

1900 / 4299
Чоловік 60-ти років страждає на атеросклероз судин головного мозку. При обстеженні виявлена гіперліпі-демія. Вміст якого класу ліпопротеї-дів найбільш імовірно буде підвищений при дослідженні сироватки крові?

Хіломікрони

Ліпопротеїди високої щільності

Ліпопротеїди низької щільності

Комплекси жирних кислот з альбумінами

Холестерин

1901 / 4299
Виділення гормонів кори на-днирників регулюється АКТГ аде-ногіпофізу. Які гормони виділяються наднирниками при дії останнього?

Глюкокортикоїди

Простагландини

Катехоламіни

Мінералокортикоїди

Андрогени

1902 / 4299
У хворого при обстеженні виявлено глюкозурію, гіперглікемію. Скарги на сухість в роті, свербіння шкіри, часте сечовиділення, спрагу. Встановлений діагноз: цукровий діабет. Чим обумовлена поліурія у даного хворого?

Збільшення осмотичного тиску сечі

Зменшення серцевого викиду

Зменшення онкотичного тиску плазми

Збільшення онкотичного тиску плазми

Збільшення фільтраційного тиску

1903 / 4299
В присутності 2,4-дінитрофенолу окиснення субстратів може тривати, але синтез молекул АТФ неможливий. Який механізм його дії?

Інгібування ферменту цитохромо-ксидаза

Переніс субстратів за межі міто-хондрії

Активація ферменту АТФ-ази

Стимуляція гідролізу утвореного АТФ

Роз’єднання окиснення і фосфори-лювання в мітохондріях

1904 / 4299
При функціональному виснаженні нейрона спостерігається ти-гроліз - просвітлення нейроплазми внаслідок розпаду базофільної субстанції. Зі зміною якої органели пов’язане це явище?

Лізосоми

Комплекс Іольджі

Мітохондрії

Гранулярна ендоплазматична сітка

Нейрофібрили

1905 / 4299
У кішки під час експерименту подразнюють периферичний відрізок блукаючого нерва. Які з наве- дених змін будуть спостерігатися при цьому?

Збільшення частоти дихання

Збільшення частоти серцевих скорочень

Розширення бронхів

Зменшення частоти серцевих скорочень

Розширення зіниць

1906 / 4299
У людини збільшений об’єм циркулюючої крові та зменшений осмотичний тиск плазми. Це супроводжується збільшенням діурезу, перш за все, внаслідок зменшеної секреції такого гормону:

Ренін

Вазопресин

Натрійуретичний

Альдостерон

Адреналін

1907 / 4299
Сімейний лікар призначив хворому 53-х років для лікування хронічної серцевої недостатності дигоксин. Через 1 місяць виявив симптоми глікозидної інтоксикації. Який препарат необхідно призначити для усунення інтоксикації?

Унітіол

Аміназин

Мезатон

Ціанокобаламін

Морфіну гідрохлорид

1908 / 4299
Під час проведення місцевої анестезії при екстракції зуба у хворого 33-х років виник анафілактичний шок. Виберіть препарат для усунення цього стану:

Димедрол

Кордіамін

Еуфілін

Атропіну сульфат

Адреналіну гідрохлорид

1909 / 4299
При огляді 6-місячної дитини лікар виявив незакрите заднє тім’ячко. При нормальному розвитку дитини воно закривається до:

Народження

Кінця другого року життя

Кінця першого року життя

3-х місяців

6-ти місяців

1910 / 4299
У хлопчика водянка яєчка (накопичування рідини між оболонками яєчка). Яка саме оболонка яєчка вміщує цю рідину?

Зовнішня сім’яна

Піхвова

М’ясиста

Внутрішня сім’яна

Білкова

1911 / 4299
При обстеженні хворого 6-ти років виникла підозра на погіршення прохідності дихальних шляхів. Який із методів дослідження дозволяє вірогідно визначити дану патологію?

Пневмографія

Спірографія

Пневмотахометрія

Спірометаболографія

Спірометрія

1912 / 4299
До ендокринолога звернулася жінка 48-ми років із ознаками вірилізму (розвиток вторинних статевих ознак чоловічої статі, зокрема вусів і бороди). У якій зоні кори наднирни-ків можливий патологічний процес?

Х-зона

Суданофобна

Клубочкова

Суданофільна

Сітчаста

1913 / 4299
У хворого 25-ти років спостерігається напад ядухи, експіраторна задишка, досить гучне, свистяче дихання. У акті дихання беруть участь допоміжні м’язи, над легенями вислуховуються сухі, свистячі хрипи. Головним механізмом у розвитку нападу є:

Порушення кровообігу в легенях

Ацидоз

Спазм дрібних бронхів та бронхіол

Зростання вмісту простагландину Е

Порушення діяльності центральної нервової системи

1914 / 4299
У потерпілого 24-х років з трав- мою голови у скроневій ділянці діагностовано епідуральну гематому. Яка з артерій найімовірніше пошкоджена?

Середня мозкова

Задня вушна

Середня оболонкова

Поверхнева скронева

Передня оболонкова

1915 / 4299
У пацієнта 28-ми років із синдромом Іценка-Кушинга виявлені гіперглікемія, глюкозурія. Основним механізмом гіперглікемії у даного хворого є стимуляція:

Ілюконеогенезу

Всмоктування глюкози в кишечнику

Ілікогенолізу в печінці

Синтезу глікогену

Ілікогенолізу в м’язах

1916 / 4299
У постраждалого 23-х років травма черепа з порушенням дірчастої пластинки решітчастої кістки. Який нерв може бути ушкоджений?

N. abducens

N. trochlearis

N. olfactorius

N. opticus

N. opthalmicus

1917 / 4299
У чоловіка 55-ти років діагностовано вікову далекозорість у зв’язку з послабленням акомодаційних властивостей кришталика та гладеньких м’язів судинної оболонки очного яблука. Тонус якого м’яза з віком послаблено?

M. dilatator pupillae

M. sphincter pupillae

M. obliquus superior

M. obliquus inferior

M. ciliaris

1918 / 4299
Хворого 21-го року госпіталізовано з проникаючою травмою дна порожнини рота. Який м’яз травмований?

Щито -під’язиковий

Щелепно-під’язиковий

Груднино-під’язиковий

Лопатково-під’язиковий

Шило-під’язиковий

1919 / 4299
При лікуванні сіалоаденітів (запалення слинних залоз) використовують препарати вітамінів. Який із наведених вітамінів відіграє важливу роль в антиоксидантному захисті?

Тіамін

Пантотенова кислота

Піридоксин

Рибофлавін

Токоферол

1920 / 4299
У хворого 27-ми років із стрептококовою інфекцією після видалення зуба спостерігалась дифузна кровотеча, яка є наслідком:

Недостатністю антикоагулянтів

Гіповітамінозом вітаміну К

Порушенням коагуляційної системи

Порушенням обміну кальцію

Активації фібринолізу

1921 / 4299
При остеолатеризмі зменшується міцність колагену, що зумовлена помітним зменшенням утворення поперечних зшивок у колагенових фібрилах. Причиною цього явища є зниження активності такого ферменту:

Лізілгідроксилаза

Лізілоксидаза

Пролілгідроксилаза

Колагеназа

Моноаміноксидаза

1922 / 4299
У хворого 43-х років після оперативного втручання на підшлунковій залозі розвинувся геморагічний синдром з порушенням третьої фази згортання крові. Що може бути найбільш імовірним механізмом порушення гемостазу?

Зниження синтезу фібриногену

Активація фібринолізу

Дефіцит фібринстабілізуючого фактора

Якісні аномалії фібриногенезу

Зниження синтезу протромбіну

1923 / 4299
Фармакологічні ефекти антидепресантів пов’язані з блокуванням (інгібуванням) ними ферменту, який каталізує розпад таких біогенних амінів, як норадреналін, серотонін в мітохондріях нейронів головно- го мозку. Який фермент бере участь у цьому процесі?

Декарбоксилаза

Ліаза

Пептидаза

Трансаміназа

Моноамінооксидаза

1924 / 4299
При остеохондрозі хребта перетискається судина, яка проходить через поперечні отвори шийних хребців. Яка це артерія?

Потилична

Внутрішня сонна

Задня мозкова

Хребтова

Зовнішня сонна

1925 / 4299
У хворого 35-ти років травма голови викликала втрату тактильних і температурних відчуттів. Яку звивину було пошкоджено внаслідок травми?

Поясна

Передцентральна

Зацентральна

Кутова

Надкрайова

1926 / 4299
У хворого 19-ти років після видалення верхнього лівого присередньо-го різця виникла кровотеча з комірки. Яка артерія зумовила кровотечу?

Задня верхня коміркова

Нижня коміркова

Висхідна піднебінна

Передня верхня коміркова

Низхідна піднебінна

1927 / 4299
При огляді порожнини рота на слизовій оболонці язика виявлена щільна біла пляма 1 см у діаметрі, що трохи потовщує поверхню органа. Мікроскопічно: в ділянці ураження багатошаровий плоский зроговілий епітелій з гіперплазією базального і зернистого шарів, з явищами гіпер-кератозу та акантозу, в дермі - лім-фогістіоцитарний інфільтрат. Який найбільш імовірний діагноз?

Лейкоплакія

Плоскоклітинний рак без зроговіння

Еритроплакія

Папілома

Плоскоклітинний рак зі зроговінням

1928 / 4299
У школярів 1-2 класів деяких регіонів Західної України при профо-гляді виявили крейдоподібні смужки і плями темно-жовтого, коричневого, аж до чорного кольору на емалі коронок центральних і бічних різців, частіше верхньої щелепи. У 1/3 дітей зуби крихкі, ламкі, з ерозіями, місцями з відламом окремих ділянок коронки. Про яке захворювання зубів слід думати?

Клиноподібні дефекти зубів

Флюороз

Некроз твердих тканин зубів

Гіпоплазія зубів

Гіперплазія зубів

1929 / 4299
У хворого 34-х років загострення хронічного апікального періодон-титу 38 зуба. З’явилась болюча припухлість на нижній щелепі, нориці в ротовій порожнині, вміст яких: грануляційна тканина та запальний інфільтрат, секвестри. Рентгенологічно - ділянки розрідження кістки щелепи з відокремленими її фрагментами. Який найбільш імовірний діагноз?

Хронічний остеомієліт

Періостит

Остеома

Остит

Фіброзна остеодисплазія

1930 / 4299
У жінки 36-ти років на рентгенограмі біля кореня 15 зуба визначається новоутворення щільної консистенції з чіткими межами. Біопсія показала, що воно складається з ділянок незрілого цементу. Який найбільш імовірний діагноз?

Остеобластокластома

Остеома

Остеоїдостеома

Цементома

Амелобластома

1931 / 4299
При лабораторному обстеженні у хворого виявили стеаторею. Вкажіть фермент, недостатність дії якого призвела до цього симптому?

Ліпаза

Амілаза

Пепсин

Хімотрипсин

Лактаза

1932 / 4299
У хворого 35-ти років з ураженням одного із відділів ЦНС спостерігається порушення координації та амплітуди рухів, тремтіння м’язів під час виконання довільних рухів, порушення тонусу м’язів. Який із відділів ЦНС уражений?

Довгастий мозок

Середній мозок

Проміжний мозок

Передній мозок

Мозочок

1933 / 4299
У спекотну погоду в гарячих приміщеннях для нормалізації мікроклімату часто використовують вентилятори. При цьому посилюється віддача тепла тілом людини, перш за все, таким шляхом:

Теплопроведення

Випаровування

Конвекція

Кондукція

Радіація

1934 / 4299
У жінки, що тривало приймала антибіотики з приводу кишкової інфекції, розвинулося ускладнення з боку слизової порожнини рота у вигляді запального процесу і білого нальоту, у якому при бактеріологічному дослідженні були виявлені дріжджеподібні грибки Candida albicans. Який з перерахованих препаратів показаний для лікування цього ускладнення?

Флуконазол

Поліміксин

Тетрациклін

Фуразолідон

Бісептол

1935 / 4299
У хворого 50-ти років під час медичного обстеження виявили стенокардію. Лікар призначив йому препарат метопролол, який знижує силу і частоту серцевих скорочень і, відповідно потребу міокарда в кисні. Механізм його лікуючої дії полягає у наступному:

Блокада М-холінорецепторів

Блокада Н-холінорецепторів

Стимуляція в1 -адренорецепторів

Блокада в2-адренорецепторів

Блокада в1-адренорецепторів

1936 / 4299
Мембранний потенціал спокою клітини змінився з -85 мВ до -90 мВ. Причиною цього може бути активація таких каналів мембрани клітини:

Калієві

Калієві та кальцієві

Калієві та натрієві

Кальцієві

Натрієві

1937 / 4299
Оперативно висічена сполучна тканина деформованого мітрального клапану при забарвленні гематоксиліном та еозином дає базофільну реакцію, а під впливом толуїдинового синього фарбується в бузковий колір (метахромазія). Які зміни сполучної тканини виявляються такими типами реакцій?

Гіаліноз

Фібриноїдний некроз сполучної тканини

Набряк сполучної тканини

Петрифікація

Мукоїдний набряк

1938 / 4299
Організми мають ядро, оточене ядерною мембраною. Генетичний матеріал зосереджений переважно в хромосомах, які складаються з ниток ДНК і білкових молекул. Діляться ці клітини мітотично. Це:

Бактерії

Бактеріофаги

Прокаріоти

Еукаріоти

Віруси

1939 / 4299
Внаслідок надмірного годування тварини вуглеводами в клітинах печінки при гістологічному дослідженні виявлена значна кількість гранул глікогену. До якої групи структур клітини відноситься глікоген?

Пігментні включення

Органели спеціального призначення

Секреторні включення

Екскреторні включення

Трофічні включення

1940 / 4299
У людини після довільної тривалої затримки дихання збільшилися частота й глибина дихання. Які зміни в крові стали причиною цього?

Підвищення pH

Підвищення pCO2

Зниження pCO2

Зниження pO2

Підвищення pO2

1941 / 4299
Жінка 37-ми років звернулася зі скаргами на загальну слабкість, сонливість, апатію, набряки. Після обстеження встановлений діагноз: ендемічний зоб. Дефіцит якого елементу може привести до цієї патології?

Кальцій

Магній

Йод

Фтор

Залізо

1942 / 4299
Під час аускультації хворого попросили глибоко дихати. Після 10 дихальних рухів хворий знепритомнів, що пов’язано з:

Респіраторним алкалозом

Зменшенням кисневої ємності крові

Респіраторним ацидозом

Еритроцитозом

Еритропенією

1943 / 4299
Хворому на виразкову хворобу шлунка 45-ти років призначено препарат в основі дії якого є антимікробна дія. Який це препарат?

Дитилін

Омепразол

Метронідазол

Фталазол

Бісакодил

1944 / 4299
Одна з тканин зуба постійно відкладається в області верхівки кореня, що обумовлює його видовження. Цей процес компенсує стирання коронки і забезпечує постійність загальної довжини зуба. Яка тканина зуба є репаративно активною у пост-натальному періоді?

Пухка сполучна

Цемент

Емаль

Щільна сполучна

Кісткова

1945 / 4299
Чоловіка 29-ти років непокоїть різкий 6іль шкіри обличчя. Який нерв уражений?

Язикоглотковий

Трійчастий

Лицевий

Блукаючий

Окоруховий

1946 / 4299
При лабораторному дослідженні крові пацієнта виявлено, що вміст білків у плазмі становить 40 г/л. Як це впливає на транскапілярний обмін води в мікроциркуляторному руслі?

Обмін не змінюється

Збільшується фільтрація, зменшується реабсорбція

Зменшуються фільтрація і реабсорбція

Збільшуються фільтрація і реабсорбція

Зменшується фільтрація, збільшується реабсорбція

1947 / 4299
У хворого запальний процес у крилопіднебінній ямці. !нфекція поширилась в носову порожнину. Через яке анатомічне утворення розповсюдилась інфекція?

Foramen sphenopalatinum

Canalis ptherygoideus

Canalis palatinus major

Foramen rotundum

Canalis palatinus minor

1948 / 4299
При дослідженні епітелію трубчастого органу з’ясувалося, що він складається з трьох типів клітин: призматичних клітин з війками (миготливі), коротких і довгих вставних (камбіальні елементи) і келихоподібних (секреторні). Який це епітелій?

Багатошаровий призматичний

Перехідний

Одношаровий призматичний залозистий

Одношаровий призматичний з облямівкою

Багаторядний призматичний війчастий

1949 / 4299
Гістологічне дослідження тканини виявило, що в ній відсутні кровоносні судини, а клітини щільно прилягають одна до одної, утворюючи пласти. Яка це тканина?

м’язова

кісткова

епітеліальна

нервова

хрящова

1950 / 4299
У пацієнта 32-х років має місце гіповітаміноз B2. Причиною виникнення специфічних симптомів (ураження епітелію, слизових, шкіри, рогівки ока) найбільш імовірно є дефіцит:

Флавінових коферментів

Цитохрому в

Цитохрому а1

Цитохрому с

Цитохромоксидази

1951 / 4299
Злоякісна пухлина привушної слинної залози спричинила пошкодження артерії, яка проходе через її паренхіму. Яка це артерія?

Поверхнева скронева

Задня вушна

Лицева

Висхідна глоткова

Верхньощелепна

1952 / 4299
Досліджуваний проходить функціональне дослідження у стані спокійної бадьорості із закритими очима. Який ритм електроенцефалограми переважно буде у нього реєструватися у потиличній ділянці?

Бета

Альфа

Дельта

Тета

Гамма

1953 / 4299
У шліфі зуба розрізняють структуру, яка в радіальному напрямку пронизана канальцями. Відростки яких клітин розміщені в цих каналь-цях?

Макрофаги

Амелобласти

Цементоцити

Фiбробласти

Одонтобласти

1954 / 4299
Машиною швидкої допомоги, доставлено хворого 49-ти років із алкогольним дєлірієм та зоровими галюцинаціями. Лікар призначив препарат, який блокує центральні но- радренергічні і дофамінергічні рецептори, та є похідним бутирофенону. Який нейролептик призначив лікар?

Діазепам

Аміназин

Хлорпротиксен

Етаперазин

Дроперидол

1955 / 4299
Жінка вживала антибіотики в першій половині вагітності. Це призвело до гіпоплазії зубів і зміни їх кольору у дитини. Генотип не змінився. Встановити вид мінливості, яка лежить в основі захворювання:

Мутаційна

Співвідносна

Рекомбінативна

Модифікаційна

Комбінативна

1956 / 4299
Запальний процес з ділянки ікло-вої ямки розповсюдився на м’які тканини очної ямки. Через який анатомічний утвір гній потрапив у очну ямку?

Foramen zygomaticotemporale

Foramen zygomaticoorbitale

Foramen zygomaticofaciale

Canalis nasolacrimalis

Canalis infraorbitale

1957 / 4299
На розтині тіла 68-річної жінки, що померла від гострої серцевої недостатності, у задній стінці лівого шлуночка серця виявлена ділянка неправильної форми, розмірами 6,5х4,5 см, в’яла, блідо-жовтуватого кольору, оточена зоною гіперемії. Як розцінив патологоанатом виявлені зміни?

Гострий інфаркт міокарда

Дифузний кардіосклероз

Вогнищевий міокардит

Аневризма серця

Післяінфарктний кардіосклероз

1958 / 4299
Лікар-стоматолог при огляді хворого на мигдаликах виявив накладення у вигляді плівок сіруватого кольору, що знімаються з утрудненням. Про яке захворювання можна думати?

Скарлатина

Фолікулярна ангіна

Кір

Дифтерія зіву

Грип

1959 / 4299
Екзотоксин дифтерійної палички обробили 0,3-0,4% формаліном і витримали 30 днів у термостаті при температурі 40oC. Який препарат був отриманий у результаті проведених маніпуляцій?

Анатоксин

Лікувальна сироватка

Діагностикум

Діагностична сироватка

Антитоксин

1960 / 4299
При гістологічному дослідженні біоптату, отриманого із нижньої третини стравоходу 67- річного чоловіка із симптомами тривалого рефлюксу шлункового вмісту, виявлено наступні зміни: у слизовій оболонці на місці багатошарового плоского епітелію визначається одношаровий залозистий призматичний епітелій з ознаками продукції слизу. Вкажіть патологічний процес, який виник у слизовій оболонці:

Регенерація

Гіперплазія

Організація

Гіпертрофія

Метаплазія

1961 / 4299
У хворого 31-го року після лікування флегмони щоки (через поріз під час гоління), довгий час були збільшені реґіонарні підщелепні лімфовузли. Біопсія одного з них виявила повнокров’я, набряк у корковому, мозковому шарах, в розширених центрах фолікулів - макрофагально-плазмоцитарна інфільтрація. В імунограмі - збільшений вміст В- лімфоцитів при зниженні рівня Т-лімфоцитів. Яка патологія в лімфовузлах?

Лімфаденіт

Лімфогрануломатоз

Лімфосаркома

Гіперплазія лімфовузла

1962 / 4299
Чоловік 53-х років скаржиться на почервоніння та невеликий гнійник на шкірі правої щоки. На розрізі вогнище щільне, жовто-зеленого кольору. У гної спостерігаються жовто- білі крупинки - зерна друз. Мікроскопічно - гнійник оточений дозріваючою грануляційною тканиною і зрілою сполучною тканиною, де виявляються плазматичні, ксантомні, епі-теліоїдні клітини. Назвіть можливий мікоз:

Актиномікоз

Аспергільоз

Бластомікоз

Споротрихоз

Стрептотрихоз

1963 / 4299
Хворому призначили антибіотик хлорамфенікол (левомецитин), який порушує у мікроорганізмів синтез білка шляхом гальмування процесу:

Елонгації трансляції

Утворення полірибосом

Ампліфікації генів

Транскрипції

Процесінгу

1964 / 4299
У хворого, який виходить зі стану тривалого голодування, визначили обмін азоту. Який результат можна очікувати?

Кетонемія

Азотний баланс не змінився

Азотна рівновага

Збільшення виділення азоту

Зниження виділення азоту

1965 / 4299
Для нормального перебігу процесу реплікації потрібні тимідилові нуклеотиди, синтез яких відбувається за участю тимідилатсинтетази, в яко- сті коферменту використовується:

Метилентетрагідрофолат

Нікотинамідаденіндинуклеотид

Карбоксибіотин

Тіаміндифосфат

Піридоксальфосфат

1966 / 4299
У дитини 13-ти років набряк в області гілки нижньої щелепи. Збільшені та малоболючі підщелепні та тттийні лімфовузли. Реакція Манту позитивна. На рентгенограмі гілки нижньої щелепи відмічається резорбція кістки з чіткими контурами, дрібні секвестри. При гістологічному дослідженні біоптату одного з лімфатичних вузлів виявлено вогнища казеозного некрозу, які оточені валами з епітеліоїдних клітин і лімфоцитів, між якими розташовані гігантські клітини Пирогова-Лангханса. Яка хвороба гілки нижньої щелепи у дитини?

Гострий остеомієліт

Саркома Юїнга

Туберкульоз

Остеобластокластома

Хронічний остеомієліт

1967 / 4299
У дитини 9-ти років швидко підвищилась температура тіла до 39oCС, з’явився біль у горлі. Об’єктивно: зів та мигдалики яскраво-червоного кольору. Язик набряклий малиново- червоного кольору зі збільшеними грибоподібними сосочками. Шкіра тіла та обличчя, крім носо-губного трикутника, вкрита густими червоними плямами величиною з макове зерно. Підщелепні лімфатичні вузли при пальпації болісні. Яке захворювання у дитини?

Поліомієліт

Скарлатина

Дифтерія

Менінгококовий назофарингіт

Кір

1968 / 4299
Чоловік 73-х років, який хворів на ессенціальну гіпертензію впродовж 22-х років, помер від хронічної ниркової недостатності. На аутопсії тіла померлого: нирки вдвічі зменшені в розмірі, щільні, з дрібнозернистою поверхнею, зі значно стоншеною кірковою речовиною. Яка пато- логія нирок виявлена?

Артеріолосклеротичний нефроци-роз

Атеросклеротичний нефроцироз

Амілоїдозно зморщені нирки

Пієлонефротичний нефроцироз

Хронічний гломерулонефрит

1969 / 4299
Чоловік повернувся з Лівану. Через деякий час він відчув 6іль і важкість у промежині і надлобковій ділянці. При обстежені йому було встановлено діагноз - урогенітальний ши- стосомоз. Яким шляхом він міг заразитися?

Недосмажене м’ясо великої рогатої худоби

Недоварене м’ясо раків і крабів

Через немиті овочі та фрукти

Недостатньо просолена риба

Купання у заражених водоймах

1970 / 4299
У хворого 48-ми років з обшир-ним інфарктом міокарда розвинулась серцева недостатність. Який патогенетичний механізм сприяв розвитку серцевої недостатності у хворої?

Реперфузійне ураження міокарда

Перевантаження тиском

Зменшення маси функціонуючих міокардіоцитів

Гостра тампонада серця

Перевантаження об’ємом

1971 / 4299
Лікар виконує передній серединний розтин шиї для термінової трахеотомії. Про можливе пошкодження якої судини він повинен пам’ятати?

Vjugularis interna

Vjugularis externa

Vthyroidea media

Arcus venosus juguli

Vfacialis

1972 / 4299
До стоматолога звернувся хворий 47-ми років з артритом скронево-нижньощелепного суглоба. Лікар призначив протизапальний препарат - селективний інгібітор ферменту циклооксигенази-2. Який препарат призначили хворому?

Бутадіон

Кислота мефенамова

Целекоксиб

Ібупрофен

Індометацин

1973 / 4299
До приймального відділення надійшов хворий з важкою щелепно-лицевою травмою. Який препарат йому необхідно ввести для зняття больового шоку?

Пантогам

Сиднокарб

Ібупрофен

Промедол

Мідокалм

1974 / 4299
Пацієнт 39-ти років із захворюванням першого верхнього різця зліва скаржиться на сильні болі шкіри в ділянці надбрівної дуги з того ж боку. Які рефлекси спричиняють вказані реакції?

Сомато-вісцеральні

Вісцеро-вісцеральні

Вісцеро-соматичні

Пропріоцептивні

Вісцеро-дермальні

1975 / 4299
У чоловіка 30-ти років при гістологічному дослідженні біоптату з шийного лімфатичного вузла виявлені гранульоми, які складаються з епітеліоїдних, лімфоїдних, багатоядерних гігантських клітин типу Пирогова-Лангханса. У центрі гранульом визначається некроз. Який збудник потрібно виявити в зоні некрозу для підтвердження діагнозу?

Мікобактерія Коха

Стафілокок

Бацила Волковича-Фріша

Сальмонела

Бліда трепонема

1976 / 4299
У хворого під час гіпертонічного кризу виник геморагічний інсульт, внаслідок чого спостерігається відсутність довільних рухів, підвищення сухожильних рефлексів та тонусу м’язів лівих руки та ноги. Як називається таке порушення рухової функції?

Параплегія

Тетраплегія

Геміплегія

Моноплегія

Млявий параліч

1977 / 4299
Хворому 37-ми років для обстеження очного дна був призначений атропіну сульфат у вигляді очних крапель, після чого для відновлення акомодації закрапали розчин пілокарпіну гідрохлориду, що не дало бажаного результату. Що спричинило відсутність ефекту?

Двосторонній антагонізм

Звикання

Односторонній антагонізм

Тахіфілаксія

Синергізм

1978 / 4299
В лабораторії вивчали вірулентність збудника дифтерії. При цьому проводили внутрішньоочеревен-не зараження лабораторних тварин. В ході експерименту встановлена доза бактерій, яка викликає загибель 95% тварин. Яку одиницю вимірювання вірулентності визначали в лабораторії?

ВД

DCL

LD 5

LD50

DLM

1979 / 4299
Шкіра людини дуже міцна на розрив. Відомо, що шкіра складається з епітеліальної тканини і двох видів сполучної тканини. Яка з нижче перерахованих тканин забезпечує міцність шкіри?

Перехідний епітелій

Щільна неоформлена сполучна

Одношаровий епітелій

Багатошаровий плоский епітелій

Пухка сполучна тканина

1980 / 4299
Перекомбінація генетичного матеріалу досягається декількома механізмами, одним з яких є кросинговер. На якій стадії профази першого мейотичного поділу він відбувається?

Диплонеми

Діакінезу

Лептонеми

Пахінеми

Зигонеми

1981 / 4299
Під час пресинтетичного періоду мітотичного циклу у клітині було порушено синтез ферменту ДНК-залежної-ДНК-полімерази. До яких наслідків це може призвести?

Скорочення тривалості мітозу

Порушення реплікації ДНК

Порушення формування веретена поділу

Порушення цитокінезу

1982 / 4299
Провідну роль в процесі каль-цифікації тканин зуба відіграє білок остеокальцин, який має високу здатність зв’язувати іони кальцію, завдяки наявності в поліпептидному ланцюзі залишків модифікованої амінокислоти:

Аланін

Карбоксиаспарагінова

Іама-аміномасляна

Іама-карбоксиглутамінова

Дельта-амінопропіонова

1983 / 4299
У людини артеріальний тиск становить: систолічний - 90 мм рт.ст., ді-астолічний - 70 мм рт.ст. Зменшення якого з наведених факторів найімовірніше обумовило таку величину артеріального тиску?

Розтяжність аорти

Тонус судин

Загальний периферичний опір

Насосна функція лівого серця

Насосна функція правого серця

1984 / 4299
У людини спостерігається зменшення компактної і губчастої речовин кісткової тканини, що проявляється у зміні лицьового відділу черепа, з’являється сивина, шкіра втрачає еластичність. На якому етапі онтогенезу виникають ці зміни?

Підлітковий вік

Дитинство

Юнацький вік

Старечий вік

Грудний вік

1985 / 4299
У біоптаті слизової оболонки хворого на бронхіальну астму виявлено значну кількість клітин з численними метахроматичними гранулами. Назвіть цю клітину:

Тканинний базофіл

Фібробласт

Ретикулоцит

Плазмоцит

Макрофаг

1986 / 4299
У хворого 45-ти років виявлено зміну функції привушної слинної залози. Який з вузлів вегетативної нервової системи віддає післявузло-ві симпатичні волокна для неї?

G. oticum

G. submandibulare

G. cervicothoracicum

G. pterygopalatinum

G.cervicale superius

1987 / 4299
У хворого 69-ти років на шкірі в ділянці нижньої повіки з’явилося невелике бляшкоподібне утворення, з послідуючим виразкуванням, яке було оперативно видалене. При мікроскопічному дослідженні утворення: в дермі шкіри комплекси з атипових епітеліальних клітин, на периферії утворень клітини розташовані перпендикулярно до базальної мембрани. Клітини темні, призматичної полігональної форми; ядра гіперхромні з частими мітозами. Іноді зустрічаються утворення, подібні до волосяного фолікула. Яка гістологічна форма рака у хворого?

Плоскоклітинний з ороговінням

Базально-клітинний

Недиференційований

Плоскоклітинний без ороговіння

Аденокарцинома

1988 / 4299
Жінка 35-ти років хворіє на ВІЛ-інфекцію на стадії СНІД. На шкірі нижніх кінцівок, слизової оболонки піднебіння з’явились рудувато-червоні плями, яскраво-червоні вузлики різних розмірів. Один з вузликів взято на гістологічне дослідження. Виявлено багато хаотично розташованих тонкостінних судин, вистелених ендотелієм, пучки веретеноподібних клітин з наявністю гемосиде-рину. Яка пухлина розвинулась у хво- рої?

Гемангіома

Саркома Капоші

Лімфома Беркіта

Лімфангіома

Фібросаркома

1989 / 4299
Рибалка наловив риби із річки, трошки підсмажив її на вогнищі і з’їв, майже напівсиру. Через декілька тижнів потому в нього з’явились ознаки ураження печінки і підшлункової залози. Лабораторний аналіз фекалій показав наявність дрібних яєць гельмінта. Яким трематодозом ймовірно заразився рибалка?

Дикроцеліоз

Шистосомоз

Опісторхоз

Парагонімоз

Фасціольоз

1990 / 4299
Під час спортивних змагань боксер отримав сильний удар у живіт, що привело до нокауту через короткочасне падіння артеріального тиску. Які фізіологічні механізми викликали цей стан?

Ішемія центральної нервової системи

Зміна транскапілярного обміну

Подразнення парасимпатичних нервів

Раптова зміна кількості рідини у організмі

Подразнення симпатичних нервів

1991 / 4299
У хворого після курсу лікування атеросклерозу в плазмі крові лабораторно доведено збільшення рівня ан-тиатерогенної фракції ліпопротеїнів. Збільшення рівня яких ліпопротеїнів підтверджує ефективність терапії захворювання?

ЛПДНЩ

ЛПНЩ

ЛПВЩ

ЛППЩ

Хіломікрони

1992 / 4299
При окисленні вуглеводів, ліпідів утворюється велика кількість енергії, основна частина якої утворюється завдяки окисленню ацетил-КоА. Скільки молекул АТФ утворю- ється при повному окисненні однієї молекули ацетіл-КоА?

38

8

24

36

12

1993 / 4299
У хворого з алкогольним ураженням пєчінки порушені процеси біотрансформації ксєно6іотиків та ендогенних токсичних сполук. Зниження активності якого хромопротеїну може бути причиною цього?

Цитохром Р-450

Гемоглобін

Цитохром с1

Цитохромоксидаза

Цитохром b

1994 / 4299
Реакції міжмолекулярного транспорту одновуглецевих радикалів є необхідними для синтезу білків та нуклеїнових кислот. З якого з перерахованих нижче вітамінів утворюється кофермент, потрібний для вказаних вище реакцій?

Аскорбінова кислота

Фолієва кислота

Тіамін

Пантотенова кислота

Рибофлавін

1995 / 4299
Піддослідному собаці ввели гормон, що призвело до збільшення швидкості клубочкової фільтрації за рахунок розширення приносної артеріоли і зменшення реабсорбції іонів натрію і води в канальцях нефрона. Який гормон було введено?

Передсердний натрійуретичний

Тестостерон

Тироксин

Окситоцин

Адреналін

1996 / 4299
Дівчина 19-ти років, хвора на цукровий діабет, чекає на донорську нирку. Яке ускладнення діабету є причиною хронічної ниркової недостатності?

Макроангіопатія

Атеросклероз

Нейропатія

Мікроангіопатія

Ретинопатія

1997 / 4299
Тривале лікування гіпофункції щитоподібної залози спричинило загальну дистрофію, карієс зубів, тахікардію, тремор кінцівок. Який засіб викликав побічні ефекти?

Хумулін

Паратиреоїдин

Тирокальцитонін

L-тироксин

Преднізолон

1998 / 4299
До медико-генетичної консультації звернувся чоловік з кольоровою сліпотою. Це зчеплена з Х-хромосомою рецесивна ознака. Яка імовірність появи в його родині дітей-дальтоників, якщо в генотипі його дружини такий алель відсутній?

75%

100%

50%

0%

25%

1999 / 4299
В яких лімфатичних вузлах може розвиватися запальна реакція при гінгівіті (запаленні слизової оболонки ясен)?

Nodi supraclaviculares

Nodi parotidei profundi

Nodi submandibulares

Nodi cervicales laterales

Nodi parotidei superficiales

2000 / 4299
У пацієнта 59-ти років, що знаходиться на обстеженні в стаціонарі, виявлено глюкозурію, рівень глюкози в крові 3,0 ммоль/л. Найбільш імовірною причиною глюкозурії може бути:

Мікседема

Пелагра

Захворювання нирок

Гіпертонічна хвороба

Нецукровий діабет

2001 / 4299
При цитологічних дослідженнях було виявлено велику кількість різних молекул т-РНК, які доставляють амінокислоти до рибосоми. Кількість рїзних типів т-РНК у клітині буде дорівнювати кількості:

Нуклеотидїв

Триплетів, що кодують амінокислоти

Амінокислот

Білків, синтезованих у клітині

Різних типів і-РНК

2002 / 4299
У районах Південної Африки у людей розповсюджена серпоподібноклїтинна анемія, при якій еритроцити мають форму серпа внаслідок зміни в молекулі гемоглобіну амінокислоти глутамату на валін. Причиною цієї хвороби є:

Геномна мутація

Генна мутація

Трансдукція

Порушення механізмів реалізації генетичної інформації

Кросинговер

2003 / 4299
При обстеженні дівчини 18-ти років знайдені наступні ознаки: недорозвинення яєчників, широкі плечі, вузький таз, вкорочення нижніх кінцівок, 'шия сфінкса', розумовий розвиток не порушений. Встановлено діагноз: синдром Шерешевського-Тернера. Яке хромосомне порушення у хворої?

Моносомїя Х

Трисомїя 13

Трисомїя 18

Трисомїя Х

Нульсомїя Х

2004 / 4299
У хворого виявлена короткочасна пневмонія. Міграція личинок якого гельмінта може призвести до цієї хвороби?

Гострик

Карликовий ціп’як

Волосоголовець

Аскарида

Альвеокок

2005 / 4299
При огляді хворого з ранами, що кровоточать, лікар виявив пошкодження тканин личинками, а також локальні місця нагноєння. Діагноз: облігатний мїаз. Збудником цього захворювання є:

Триатомовий клоп

Жигалка осіння

Муха цеце

Муха хатня

Муха вольфартовая

2006 / 4299
Дівчині 18-ти років встановлено попередній діагноз - синдром Шерешевського-Тернера. Це можна підтвердити за допомогою такого метода:

Дерматогліфіка

Цитогенетичний

Біохімічний

Близнюковий

Генеалогічний

2007 / 4299
У хворого з підозрою на одне з протозойних захворювань досліджено пунктат лімфатичного вузла. В препараті, забарвленому за Романовським-Гімзою, виявлено тільця півмісяцевої форми із загостреним кінцем, блакитною цитоплазмою, ядром червоного кольору. Яких найпростіших виявлено в мазках?

Малярійні плазмодії

Трипаносоми

Вісцеротропні лейшманії

Дерматотропні лейшманії

Токсоплазми

2008 / 4299
Після черепно-мозкової травми у хворого 38-ми років відсутнє відчуття нюху і смаку. Які коркові центри кінцевого мозку постраждали?

Operculum frontale

Sul. calcarinus

Uncus

Gyrus temporalis superior

Gurus postcentralis

2009 / 4299
Оглядаючи ротову порожнину у хворого 19-ти років, стоматолог звернув увагу на зруйнований язиково-дистальний горбик жувальної поверхні лівого першого верхнього моляра. Цей горбик називається:

Гіпоконус

Мезоконус

Параконус

Метаконус

Протоконус

2010 / 4299
Є потреба катетеризації сечового міхура у чоловіка. В якій частині уретри або структурі може виникати опір катетеру?

Простатична

Перетинчаста

Губчаста

Внутрішній отвір сечівника

Зовнішній отвір сечівника

2011 / 4299
При огляді хворого невропатолог відзначає асиметрію обличчя: згладженість складок шкіри лоба, око повністю не закривається, на одному боці очна щілина ширша, рот перекошений. Який нерв уражений?

Нижньощелепний

Додатковий

Очний

Верхньощелепний

Лицевий

2012 / 4299
У хворого відкритий рот, він не може його закрити і членороздільно говорити. Яке ураження у хворого?

Стрес

Ураження рухового центру мови

Перелом нижньої щелепи

Параліч жувальних м’язів

Вивих нижньої щелепи

2013 / 4299
Під час операції закриття природженої щілини піднебіння (уранопластики) при збиванні долотом крилоподібного гачка ушкоджено великий піднебінний канал. Виникла кровотеча. Яку артерію ушкоджено?

Висхідна глоткова

Задня верхня альвеолярна

Клиноподібна

Висхідна піднебінна

Низхідна піднебінна

2014 / 4299
У хворого 60-и років виявлено розширення вен стравоходу, прямої кишки та підшкірних вен передньої черевної стінки. Система якої вени ушкоджена?

Ворітна

Верхня брижова

Верхня порожниста вена

Непарна вена

Нижня порожниста вена

2015 / 4299
У травмованого 27-ми років двосторонній вивих скронево-нижньощелепного суглоба. Які основні зв’язки суглоба будуть ушкоджені?

Клиноподібно-щелепні

Шило-щелепні

Крилоподібно-щелепні

Внутрішньосуглобові

Латеральні

2016 / 4299
У хворого внаслідок пошкодження шкіри в середній ділянці грудинно-ключично- соскоподібного м’яза виникла повітряна емболія. Яка вена шиї була травмована?

Внутрішня яремна

Зовнішня яремна

Задня вушна

Передня яремна

Поперечна вена шиї

2017 / 4299
У хворого виникають болі при жуванні, утруднення при піднятті вверх нижньої щелепи (при закриванні рота). Функція яких м’язів порушена?

Латеральний і медіальний крилоподібні, жувальний

Скроневий, жувальний, щелепно-під’язиковий

Скроневий, жувальний, медіальний крилоподібний

Скроневий, латеральний і медіальний крилоподібні

Скроневий, жувальний, латеральний крилоподібний

2018 / 4299
У дитини 2-х років після перенесеного грипу з’явилися скарги на біль у вусі. Лікар виявив зниження слуху та запалення середнього вуха. Яким шляхом інфекція потрапила до середнього вуха?

Через foramen jugularis

Через слухову трубу

Через atrium mastoideum

Через canalis caroticus

Через canalis nasolacrimalis

2019 / 4299
В гістопрепараті представлена частина органу, що містить багатошаровий плоский незроговілий епітелій та нижче розташовані сполучнотканинні пластинки, які не містять судин. Який це орган?

Слизова оболонка ротової порожнини

Сітківка

Рогівка

Кришталик

Слизова оболонка стравоходу

2020 / 4299
У препараті червоного кісткового мозку людини визначаються скупчення гігантських клітин, розташованих в тісному контакті з синусоїдними капілярами. Назвіть формені елементи крові, які утворюються з цих клітин:

Моноцити

Лімфоцити

Кров’яні пластинки

Лейкоцити

Еритроцити

2021 / 4299
На гістологічному препараті пухкої сполучної тканини знайдено відносно великі клітини, заповнені базофільною метахроматичною зернистістю; гістохімічно встановлено, що гранули містять гепарин та гістамін. Що це за клітини?

Макрофаги

Фібробласти

Плазмоцити

Тканинні базофіли (тучні клітини).

Адипоцити

2022 / 4299
При ендоскопічному дослідженні у хворого з хронічним ентероколітом (запалення кишки) спостерігається відсутність специфічних структур рельєфу тонкої кишки. Які компоненти визначають особливості рельєфу слизової оболонки тонкої кишки в нормі?

Гаустри, ворсинки, крипти

Поля, ворсинки

Поля, складки, ямки

Циркулярні складки, ворсинки та крипти

Косо-спіральні складки

2023 / 4299
Морфологічні дослідження селезінки виявили активізацію імунних реакцій в організмі. В яких структурах даного органу починається антигензалежна проліферація Т-лімфоцитів?

Маргінальна зона білої пульпи

Центральна зона білої пульпи

Мантійна зона білої пульпи

Червона пульпа

Периартеріальна зона білої пульпи

2024 / 4299
У препараті в одному з судин мікроциркуляторного русла середня оболонка утворена 1-2 шарами гладеньких міоцитів, які розташовані поодинці і мають спіралеподібний напрямок. Зовнішня оболонка представлена тонким шаром пухкої сполучної тканини. Вкажіть вид судини:

Капіляр

Артеріола

Венула

Артеріоловенулярний анастомоз

Посткапіляр

2025 / 4299
Шкідливі екологічні чинники призвели до різкого падіння ендоцитозу і екзоцитозу в клітинах печінки та крові. Який шар плазмолеми постраждав найбільше?

Ліпопротеїновий

Надмембранний

Інтегральний

Кортикальний

Глікокалікс

2026 / 4299
У ході утворення плащового дентину у молочному зубі відбулося порушення секреторної активності одонтобластів. Утворення яких волокон зміниться?

Радіальні колагенові волокна Корфа

Нервові

Ретикулярні

Еластичні

Тангенційні колагенові волокна Ебнера

2027 / 4299
При утворенні зародка людини можна спостерігати появу у його складі порожнини, світлих дрібних бластомерів на периферії та темних великих бластомерів на одному з полюсів. Як називається зародок на цій стадії розвитку?

Морула

Бластоциста

Гаструла

Зародковий диск

Зигота

2028 / 4299
При непрямому гістогенезі кісткової тканини трубчастих кісток між епіфізарним та діафізарним центрами окостеніння утворюється пластинка, що в подальшому забезпечує ріст кісток у довжину. Як називається ця структура?

Шар внутрішніх генеральних пластинок

Остеон

Кісткова пластинка

Метафізарна пластинка

Кісткова манжетка

2029 / 4299
У хворого з пересадженим серцем при фізичному навантаженні збільшився хвилинний об’єм крові. Який механізм регуляції забезпечує ці зміни?

Симпатичні безумовні рефлекси

Парасимпатичні безумовні рефлекси

Симпатичні умовні рефлекси

Катехоламіни

Парасимпатичні умовні рефлекси

2030 / 4299
Хворому з гіперсекрецією шлункового соку лікар рекомендував виключити з харчового раціону:

Білий хліб

Солодке

М’ясні бульйони

Солоне

Молоко

2031 / 4299
Перед проведенням оперативного втручання з’ясовано, що у людини час кровотечі збільшений до 15 хвилин. Дефіцит яких формених елементів у складі крові може бути причиною таких змін?

Лімфоцити

Еритроцити

Тромбоцити

Лейкоцити

Моноцити

2032 / 4299
Досліджуються рецептори, інформація від яких прямує до кори головного мозку без участі таламусу. Які це рецептори?

Нюхові

Дотикові

Зорові

Смакові

Слухові

2033 / 4299
При дослідженні ізольованого кардіоміоциту встановлено, що він НЕ ГЕНЕРУЄ імпульси збудження автоматично. З якої структури серця отриманий кардіоміоцит?

Сино-атріальний вузол

Пучок Гіса

Атріовентрикулярний вузол

Шлуночок

Волокно Пуркін’є

2034 / 4299
Після руйнування структур ЦНС тварина втратила орієнтувальні рефлекси. Що саме зруйнували?

Чорна речовина

Медіальні ретикулярні ядра

Червоні ядра

Латеральні вестибулярні ядра

Чотиригорбкове тіло

2035 / 4299
При визначенні енерговитрат організму людини встановлено, що дихальний коефіцієнт дорівнює 1,0. Це означає, що у клітинах досліджуваного переважно окислюються:

Жири

Білки та вуглеводи

Вуглеводи

Вуглеводи та жири

Білки

2036 / 4299
У добовому раціоні дорослої здорової людини повинні бути жири, білки, вуглеводи, вітаміни, мінеральні солі та вода. Вкажіть добову кількість білку, яка забезпечує нормальну життєдіяльність організму:

10-20

100-120

50-60

70-80

40-50

2037 / 4299
При обстеженні хворого виявлені дерматит, діарея, деменція. Відсутність якого вітаміну є причиною цього стану?

Біотин

Нікотинамід

Рутин

Аскорбінова кислота

Фолієва кислота

2038 / 4299
Аміак є дуже отруйною речовиною, особливо для нервової системи. Яка речовина бере особливо активну участь у знешкодженні аміаку в тканинах мозку?

Аргінін

Глутамінова кислота

Сечовина

Аспарагінова кислота

Буферні системи

2039 / 4299
При обстеженні хворого виявлено підвищено вмісту в сироватці крові ліпопротеїнів низької щільності. Яке захворювання можна передбачити у цього хворого?

Запалення легень

Гастрит

Ураження нирок

Гострий панкреатит

Атеросклероз

2040 / 4299
У відділення інтенсивної терапії доставлено жінку 50-ти років з діагнозом: інфаркт міокарда. Активність якого ферменту буде найбільш підвищена впродовж перших двох діб?

Аспартатамінотрансфераза

Лужна фосфатаза

Аланінамінотрансфераза

Аланінамінопептидаза

Сорбітдегідрогеназа

2041 / 4299
У лікарню поступила робітниця хімічного підприємства з ознаками отруєння. У волоссі цієї жінки знайдено підвищену концентрацію арсенату, який блокує ліпоєву кислоту. Порушення якого процесу є найімовірною причиною порушень в організмі?

Відновлення метгемоглобіну

Знешкодження супероксидних іонів

Окислювальне декарбоксилювання ПВК

Мікросомальне окислення

Відновлення органічних перекисей

2042 / 4299
При захворюваннях підшлункової залози порушується утворення та секреція трипсину. Назвіть речовини, травлення яких буде порушене?

Білки

Нуклеїнові кислоти

Ліпіди

Фосфоліпіди

Вуглеводи

2043 / 4299
Еритроцити людини не містять мітохондрій. Який основний шлях утворення АТФ в цих клітинах?

Окиснювальне фосфорилювання

Анаеробний гліколіз

Аеробний гліколіз

Аденілаткіназна реакція

Креатинкіназна реакція

2044 / 4299
Катіонні глікопротеїни є основними компонентами слини привушних залоз. Які амінокислоти обумовлюють їх позитивний заряд?

Цистеїн, гліцин, пролін

Глутамат, валін, лейцин

Аспартат, аргінін, глутамат

Аспартат, глутамат, гліцин

Лізин, аргінін, гістидин

2045 / 4299
Однією із функцій слини є захисна, яка реалізується, зокрема, формуванням місцевого імунітету слизової оболонки за рахунок виділення привушними залозами такого білку:

Фібриноген

Еластин

Альбумін

Колаген

Секреторний імуноглобулін А

2046 / 4299
У хлопчика 4-х років після перенесеного важкого вірусного гепатиту спостерігаються блювання, епізоди непритомності, судоми. У крові - гіперамоніємія. Порушення якого біохімічного процесу в печінці викликало такий стан хворого?

Активація декарбоксилювання амінокислот

Порушення знешкодження біогенних амінів

Порушення знешкодження аміаку

Пригнічення синтезу білків

Пригнічення ферментів трансамінуван-ня

2047 / 4299
Хворому з печінковою недостатністю проведено дослідження електрофоретичного спектру білків сироватки крові. Які фізико-хімічні властивості білкових молекул лежать в основі цього методу?

Гідрофільність

Нездатність до діалізу

Оптична активність

Здатність набрякати

Наявність заряду

2048 / 4299
При спадковій оратацидурії виділення оротової кислоти в багато разів перевищує норму. Синтез яких речовин буде порушений при цій патології?

Сечовина

Сечова кислота

Пуринові нуклеотиди

Біогенні аміни

Піримідинові нуклеотиди

2049 / 4299
У хворого 37-ми років після автомобільної катастрофи АТ- 70/40 мм рт.ст. Хворий у непритомному стані. За добу виділяє близько 300 мл сечі. Який механізм порушення сечоутворення у даному випадку?

Зменшення клубочкової фільтрації

Посилення клубочкової фільтрації

Зменшення канальцевої секреції

Зменшення канальцевої реабсорбції

Посилення канальцевої реабсорбції

2050 / 4299
У хворого, якому було екстирповано зуб з приводу гострого гнійного періоститу, спостерігалася тривала кровотеча з комірки, яку не можна було спинити звичайними методами. У крові: ер.-2, 9 • 1012/л, НЬ- 90 г/л; КП- 0,9; тромб.-60 • 109/л; лейк.- 52 • 109/л.; базофіли - 0, еозинофіли -1%, моноцити - 0, нейтрофіли: юні - 0, паличкоядерні - 2%, сегменто-ядерні -18%, лімфоцити - 8%, моноцити -1%, мієлобласти - 70%. Яке захворювання крові наявне у цього хворого?

Промієлоцитарний лейкоз

Недиференційований лейкоз

Гострий мієлоїдний лейкоз

Хронічний мієлоїдний лейкоз

Еритромієлоз

2051 / 4299
Жінка із токсикозом вагітності страждає на гіперсалівацію, що призводить до втрати 3-4 літрів слини щоденно. Яке порушення водно-сольового обміну виникає при цьому?

Гіпогідратація ізоосмолярна

Гіпокаліємія

Гіпогідратація гіпоосмолярна

Гіпонатріємія

Гіпогідратація гіперосмолярна

2052 / 4299
У водолаза, що проводив роботи на великій глибині, при швидкому поверненні в умови нормального атмосферного тиску з’явилися біль у суглобах, свербіж шкіри, порушення зору, непритомність. Як називається описане явище?

Баротравма

Хвороба декомпресії

Стан невагомості

Синдром вибухової декомпресії

Гіпероксія

2053 / 4299
Через 8 днів після опромінення у ліквідатора аварії на АЕС розвинулись виразково- некротичні зміни у порожнині рота. У крові: ер- 3,2 • 1012/л, ретикулоцитів 0,01%, НЬ- 60 г/л, лейк.- 2,3 • 109/л, тромбоцитів 50 тис/л. Для якого періоду променевої хвороби характерні описані зміни?

Період розвинутих клінічних ознак

Закінчення хвороби

Прихований період

Період уявного благополуччя

Період первинних реакцій

2054 / 4299
Епідеміологічне дослідження розповсюдження пухлин виявило високу кореляцію розвитку пухлин легень з тютюнопалінням. З дією якого хімічного канцерогену найбільш імовірно пов’язано виникнення даного виду патології?

Метилхолантрен

Афлатоксин

Ортоаміноазотолуол

Діетілнітрозамін

3,4-бензпірен

2055 / 4299
У хворого виявлено ожиріння, гірсутизм, 'місяцеподібне' обличчя, рубці багряного кольору на шкірі стегон. АТ-180/110 мм рт.ст., глюкоза крові - 17,2 ммоль/л. При якій зміні секреції гормонів наднирників можлива така картина?

Гіпосекреція адреналіну

Гіпосекреція глюкокортикоїдів

Гіпосекреція мінералокортикоїдів

Гіперсекреція мінералокортикоїдів

Гіперсекреція глюкокортикоїдів

2056 / 4299
При мікроскопічному дослідження пунктату з осередку запалення у хворого із абсцесом шкіри знайдено велику кількість різних клітин крові. Які з цих клітин першими надходять із судин до тканин при запаленні?

Нейтрофіли

Моноцити

Еозинофіли

Базофіли

Лімфоцити

2057 / 4299
У жінки, яка протягом 15-ти років страждала на виражену гіпертензію, останнім часом з’явились задишка, серцебиття, дещо знизився систолічний тиск. Який основний механізм виникнення у хворої серцевої недостатності?

Ушкодження міокарда

Порушення проведення імпульсу по міокарду

Перевантаження серця збільшеним об’ємом крові

Порушення регуляції серцевої діяльності

Перевантаження серця збільшеним опором викиду крові

2058 / 4299
При гістологічному дослідженні стінки кісти, що локалізується у ділянці верхньої щелепи, встановлено, що стінка кісти з середини вистелена багатошаровим плоским епітелієм з підлеглою грануляційною тканиною з лімфолейкоцитарною інфільтрацією. Зовнішній шар представлений пухкою волокнистою сполучною тканиною, оточеною рубцевою фіброзною тканиною. Ці дані є підставою для встановлення такого діагнозу:

Амелобластома

Кератокіста

Епітеліальна гранульома

Кістогранульома

Проста гранульома

2059 / 4299
При обстеженні у хворого на твердому піднебінні виявлена пухлина у вигляді невеликого щільного вузла сірого кольору без чітких меж. Після видалення пухлина досліджена гістологічно. Побудована з дрібних, кубічної форми клітин з гіперхромним ядром, які формують альвеоли, трабекули, солідні та кріброзні структури. Ріст пухлини - інвазивний. Назвіть пухлину:

Мукоепідермоїдний рак

Аденолімфома

Мономорфна аденома

Аденокістозна карцинома

Злоякісна плеоморфна аденома

2060 / 4299
В біоптаті слизової носа знайдені епітеліоїдні клітини, плазмоцити, клітини Мікулича, еозинофільні тільця Русселя. Який найбільш імовірний діагноз?

Респіраторно-синцитіальна інфекція

Алергічний риніт

Риносклерома

Сифіліс

Туберкульоз

2061 / 4299
Хворий 58-ми років впродовж багатьох років страждав на хронічний остеомієліт нижньої щелепи. Останнім часом в аналізі крові - гіпопротеїнемія, диспротеїнемія; у сечі - протеїнурія, білкові циліндри. Помер від хронічної ниркової недостатності. На розтині нирки збільшені, щільні, воскоподібні. Який патологічний процес у нирках був виявлений на розтині?

Хронічний гломерулонефрит

Хронічний пієлонефрит

Інтерстиційний нефрит

Гідронефроз

Амілоїдоз

2062 / 4299
У хворого 77-ми років, що страждає на атеросклероз, з’явився біль у правій ступні. Ступня збільшена в розмірі, шкірні покриви чорного кольору, мацеровані, демаркаційна зона невиражена. Який патологічний процес у ступні діагностовано?

Волога гангрена

Секвестр

Коагуляційний некроз

Суха гангрена

Нома

2063 / 4299
При огляді хворого лікар-стоматолог відмітив, що краї зубної комірки, позбавленої зуба, значно зближені, діаметр її зменшений. Про який загальнопатологічний процес йдеться в даному випадку?

Гіпоплазія

Атрофія дисфункціональна

Атрофія від тиску

Атрофія нейротична (при денервації)

Атрофія, викликана недостатністю кровообігу

2064 / 4299
Чоловік віком 42 роки помер при явищах вираженої інтоксикації та дихальної недостатності. На розрізі тканина легень у всіх відділах строката, з множинними дрібновогнищевими крововиливами та вогнищами емфіземи. Гістологічно у легенях: геморагічна бронхопневмонія з абсцедуванням, у цитоплазмі клітин епітелію бронхів еозинофільні та базофільні включення. Який найбільш імовірний діагноз?

Стафілококова бронхопневмонія

Грип

Респіраторно-синцитіальна інфекція

Парагрип

Аденовірусна інфекція

2065 / 4299
Макроскопічно печінка збільшена в розмірах, ущільнена, тканина сірувато-жовтого кольору, з сальним блиском. Який патологічний процес лежить в основі описаних змін?

Мукоїдне набухання

Амілоїдоз

Гіаліноз

Жирова дистрофія

Гемахроматоз

2066 / 4299
При дослідженні біоптату збільшеного шийного лімфатичного вузла виявлена стертість малюнка, тканина його представлена великою кількістю проліферуючих лімфоцитів з домішкою поодиноких клітин Березовського-Штернберга. Ці зміни свідчать про таке захворювання:

Лімфогранулематоз з виснаженням лімфоїдної тканини

Змішано-клітинний варіант лімфогранулематозу

Лімфогранулематоз з переважанням лімфоїдної тканини

Нодулярно-склеротичний варіант лімфогранулематозу

Лімфосаркома

2067 / 4299
У жінки 45-ти років, яка померла від хронічної алкогольної інтоксикації, на аутопсії печінка різко збільшена, тістоподібної консистенції, жовтуватого кольору. Мікроскопічно в цитоплазмі гепатоцитів при фарбуванні гематоксиліном та еозином виявляються різних розмірів оптично порожні вакуолі. Який вид дистрофії має місце?

Паренхіматозна жирова

Гідропічна

Вуглеводна паренхіматозна

Гіаліново-крапельна

Мезенхімальна жирова

2068 / 4299
У хворого 23-х років після перенесеної ангіни розвинувся сечовий синдром (гематурія, протеїнурія, лейкоцитурія). В пункційному біоптаті нирок виявлена картина інтракапілярного проліферативного гломерулонефриту, а електронно-мікроскопічно виявлені великі субепітеліальні депозити. Який патогенез цього захворювання?

Клітинно обумовлений цитоліз

Гранулематоз

Атопія, анафілаксія з утворенням IgE і фіксацією їх на опасистих клітинах

Цитотоксична, цитолітична дія антитіл

Імунокомплексний механізм

2069 / 4299
З випорожнень хворої дитини 6-місячного віку, яка знаходиться на штучному вигодовуванні, виділена культура кишкової палички з антигенною структурою 0-111. Який діагноз можна поставити?

Дизентерієподібне захворювання

Холероподібне захворювання

Харчове отруєння

Гастроентерит

Колі-ентерит

2070 / 4299
До інфекційного відділення госпіталізовано хворого 27-ми років зі скаргами на багаторазові пронос та блювання, біль у м’язах ніг, слабкість, запаморочення. Після огляду лікар поставив попередній діагноз 'холера'. Як необхідно досліджувати матеріал від хворого для експрес діагнозу?

Біологічний метод

Серологічний метод

Бактеріологічний метод

РА

Пряма і непряма РІФ

2071 / 4299
З метою підтвердження діагнозу туляремії мисливцю, госпіталізованому на 5-й день хвороби, слід призначити для ранньої діагностики:

Реакція аглютинації

Реакція зв’язування комплементу

Реакція непрямої гемаглютинації

Алергічна проба

Реакція іммунофлюоресценції

2072 / 4299
Хворому на туберкульоз 34-х років, в анамнезі якого була відкрита легенева форма захворювання, проведено мікроскопічне дослідження харкотиння з метою виявлення збудника. Який метод забарвлення доцільно використати при цьому?

Метод Грама

Метод Ціля-Нільсена

Метод Нейссера

Метод Буррі-Гінса

Метод Романовського-Гімзи

2073 / 4299
Після посіву гною з уретри на спеціальне поживне середовище виросли ніжні блакитнуваті колонії. При мікроскопії препаратів з них виявленні грамнегативні бобоподібні диплококи. Збудником якої хвороби вони є?

Сифіліс

Хламідіоз

Гонорея

Туляремія

Меліоїдоз

2074 / 4299
У дитячому відділенні інфекційної клініки хлопчику 4-х років поставлено діагноз 'дифтерія'. Який препарат потрібно ввести хворому в першу чергу?

Дифтерійний анатоксин

ТАВї

АКДП

Протидифтерійна антитоксична сироватка

АДП

2075 / 4299
У пацієнта з попереднім діагнозом 'сифіліс' лаборант взяв сироватку крові для постановки імунної реакції, яка основана на виявленні антитіл, які припиняють рух трепонем і призводять до їх загибелі. Яку реакцію було використано для діагностики?

Реакція іммобілізації

Реакція нейтралізації

Реакція аглютинації

Реакція зв’язування комплементу

Реакція преципітації

2076 / 4299
При повторній постановці реакції аглютинації Відаля виявлено наростання у сироватці хворого титрів антитіл до О-антигенів S. typhi з 1:100 до 1:400. Як можна розцінити отримані результати?

Є хронічним носієм черевнотифозних мікробів

Раніше перехворів на черевний тиф

Хворіє на черевний тиф

Є гострим носієм черевнотифозних мікробів

Раніше був щеплений проти черевного тифу

2077 / 4299
При обстеженні на бактеріоносійство черевного тифу у сироватці крові кухарки шкільної їдальні виявлені Vi -антитіла. Яка з наведених реакцій була використана у даному випадку?

Реакція Відаля

Реакція непрямої гемаглютинації

Імуноферментний аналіз

Реакція імунофлюоресценції

Реакція зв’язування комплементу

2078 / 4299
Хворий з гіпертонічною хворобою II ст. за рекомендацією лікаря почав приймати антигіпертензивний препарат. Через кілька днів прийому препарату артеріальній тиск нормалізувався, але хворий помітив, що у нього з’явився сухий кашель. Який препарат був призначений?

Клофелін

Пропранолол (анаприлін)

Еналапріл

Резерпін

Верапаміл

2079 / 4299
У хворої з артеріальною гіпертензією розвинувся напад бронхіальної астми. Який з нижче перерахованих бронхолітичних засобів може спровокувати гіпертонічний криз?

Ізадрин

Еуфілін

Ефедрину гідрохлорид

Беротек

Сальбутамол

2080 / 4299
Для корекції артеріального тиску при колаптоїдному стані хворому було введено мезатон. Який механізм дії даного препарату?

Блокує β-адренорецептори

Стимулює β-адренорецептори

Стимулює α-адренорецептори

Блокує α-адренорецептори

Стимулює α- β-адренорецептори

2081 / 4299
Хворому 60-ти років було встановлено діагноз цироз печінки, причиною якого було токсичне ураження алкоголем. Який препарат краще за все призначити хворому?

Кислота глютамінова

Ессенціале

Вітамін Е

Рібоксин

Аскорбінова кислота

2082 / 4299
Хворий звернувся до лікаря зі скаргами на стоматит, який був лікований антисептичними засобами. Який препарат необхідно призначити хворому, якщо ураження слизової оболонки мають герпетичну природу?

Клотримазол

Ацикловір

Бісептол

Фуразолідон

Ремантадин

2083 / 4299
Хворому 28-ми років на бактеріальну пневмонію призначили курс лікування еритроміцином. Відомо, що його антибактеріальні властивості зумовлені здатністю цієї речовини сполучатися з вільною 508-субодиницею рибосоми. Синтез яких речовин блокує цей антибіотик у бактеріальних клітинах?

РНК

ДНК

Білки

Жири

Полісахариди

2084 / 4299
Небезпечно витискувати гноячки на обличчі, особливо вище ротової щілини. Через анастомози з якими венами обличчя може поширитися інфекція у порожнину черепа?

Середня і глибока скроневі вени

Вени крилоподібного сплетення

Передні вушні вени

Очні вени

Потилична вена

2085 / 4299
Дівчинка 11-ти років часто хворіє на гострі респіраторні інфекції, після яких спостерігаються множинні точкові крововиливі в місцях тертя одягу. Гіповітаміноз якого вітаміну має місце в дівчинки?

Вб

В1

А

В2

С

2086 / 4299
Хворого доставили до клініки у коматозному стані. В анамнезі цукровий діабет 2 типу впродовж 5-ти років. Об’єктивно: дихання шумне, глибоке, у видихуваному повітрі відчувається запах ацетону. Вміст глюкози у крові 15,2 ммоль/л, кетонових тіл - 100 мкмоль/л. Для якого ускладнення даного захворювання характерні такі розлади?

Гіпоглікемічна кома

Кетоацидотична кома

Печінкова кома

Гіперглікемічна кома

Гіперосмолярна кома

2087 / 4299
Хвора на хронічний гепатит скаржиться на підвищення чутливості до барбітуратів, які раніше вона переносила без симптомів інтоксикації. З порушенням якої функції печінки це пов’язане найбільше?

Фагоцитарна

Утворення жовчі

Гемопоетична

Гемодинамічна

Метаболічна

2088 / 4299
У лікарню потрапив хворий з високою температурою, маренням, розчухами на голові. На голові виявлені комахи, сірого кольору, довжиною 3 мм, із сплощеним у дорзовентральному напрямі тілом і трьома парами кінцівок. Для якої патології характерна описана картина?

Алергія

Демодекоз

Скабієс

Ураження шкіри клопами

Педикульоз

2089 / 4299
Під час прийому їжі у новонародженої дитини спостерігається закидання молока у носову порожнину. Вкажіть на можливу причину виникнення цього порушення.

Заяча губа

Викривлення носової перегородки вліво

Вовча паща

Перелом основи черепа

Викривлення носової перегородки вправо

2090 / 4299
Під час виконання фізичного навантаження людина менш чутлива до болю. Причиною цього є активація:

Функції наднирників

Антиноціцептивної системи

Ноціцептивної системи

Симпато-адреналової системи

Функції щитоподібної залози

2091 / 4299
За даними ВООЗ щорічно на Землі на малярію хворіють приблизно 250 млн. чоловік. Ця хвороба зустрічається переважно у тропічних і субтропічних областях. Межі її розповсюдження співпадають з ареалами комарів роду:

Анофелес

Аедес

Кулекс

Мансоніа

Кулізета

2092 / 4299
У хворого з патологією серцево-судинної системи розвинулись набряки на нижніх кінцівках. Який механізм розвитку серцевого набряку?

Підвищення гідростатичного тиску на артеріальному кінці капіляру

Підвищення гідростатичного тиску на венозному кінці капіляру

Зниження осмотичного тиску

Підвищення онкотичного тиску

Порушення лімфовідтоку

2093 / 4299
У хворого з’явилися жовтушність шкіри, склер та слизових оболонок. У плазмі крові підвищений рівень загального білірубіну, в калі - стеркобіліну, в сечі - уробіліну. Який вид жовтяниці у хворого?

Холестатична

Обтураційна

Хвороба Жільбера

Паренхіматозна

Гемолітична

2094 / 4299
У хворого лікар діагностував тромбоз селезінкової артерії (інфаркт селезінки). Звідки відходить уражена артерія?

Ліва шлункова артерія

Загальна печінкова артерія

Черевний стовбур

Верхня брижова артерія

Нижня брижова артерія

2095 / 4299
У хворого на хронічний пієлонефрит після неконтрольованого прийому сечогінних засобів виникли екстрасистоли та болі у ділянці серця. При дослідженні крові виявлено гіпокаліємію. Який з перерахованих препаратів слід призначити?

Ретинолу ацетат

Калію перманганат

Аспаркам

Гідрохлортіазид

Кальцію хлорид

2096 / 4299
Хворому 30-ти років для лікування пневмонії лікар на 3 дні призначив антибіотик з групи азалідів, що має бактерицидну дію, тривалий ефект, здатність зв’язуватись з фагоцитами і накопичуватись у вогнищах інфекції. Який препарат було призначено хворому?

Ізоніазид

Ципрофлоксацин

Бензилпеніциліну натрієва сіль

Еритроміцин

Азитроміцин

2097 / 4299
У хворого 64-х років часті напади стенокардії. Два роки тому він переніс інфаркт міокарда. Вкажіть групу лікарських засобів, препарати якої доцільно призначити для профілактики розвитку повторного інфаркту міокарда:

Коагулянти

Антиагреганти

Антикоагулянти непрямої дії

Антагоністи гепарину

Фібринолітики

2098 / 4299
У печінці хворого, який страждає на залізодефіцитну анемію виявлено порушення синтезу залізовмісного білка, який є джерелом заліза для синтезу гему. Як називається цей білок?

Гемосидерин

Трансферин

Гемоглобін

Церулоплазмін

Феритин

2099 / 4299
У ході клінічного обстеження пацієнта виявлено збільшення щитоподібної залози (зоб), підвищення основного обміну, втрата маси тіла, порушення теплового балансу, підвищення апетиту, підвищення збудливості та дратівливості, екзофтальм і тахікардія. Яке ендокринне порушення призводить до появи даних симптомів?

Гіпофункція епіфізу

Гіпофункція паращитоподібних залоз

Гіпофункція щитоподібної залози

Гіперфункція гіпофізу

Гіперфункція щитоподібної залози

2100 / 4299
При тестуванні на гіперчутливість пацієнту під шкіру ввели алерген, після чого спостерігалось почервоніння, набряк, біль внаслідок дії гістаміну. В результаті якого перетворення амінокислоти гістидину утворюється цей біогенний амін?

Декарбоксилювання

Фосфорилювання

Дезамінування

Ізомеризація

Метилювання

2101 / 4299
У дитини 12-ти років низький зріст при непропорційній будові тіла і розумовій відсталості. Недостатня секреція якого гормону може бути причиною таких порушень?

Тироксин

Інсулін

Глюкагон

Соматотропін

Кортизол

2102 / 4299
У реанімаційному відділенні знаходиться хворий 49-ти років у коматозному стані. При дослідженні крові відзначено збільшення концентрації К+, зменшення вмісту Сa++, ацидоз, збільшення рівня сечовини, сечової кислоти. Який вид коми за етіологією найбільш імовірний?

Ниркова

Гіпоглікемічна

Печінкова

Нейрогенна

Діабетична

2103 / 4299
До приймального відділення госпіталізовано чоловіка у непритомному стані. На зовнішні подразники не реагує, дихання періодичне по типу Чейн-Стокса, зіниці звужені, зіничний рефлекс відсутній. Було встановлено, що дані симптоми обумовлені застосуванням морфіну. Призначити антидотну терапію:

Унітіол

Кальцію хлорид

Апоморфіну гідрохлорид

Протаміну сульфат

Налоксон

2104 / 4299
У хворого із цирозом печінки відмічається стійка артеріальна гіпотензія. (АТ- 90/50 мм рт.ст.). Чим обумовлено зниження артеріального тиску при такій патології печінки?

Надмірна інактивація вазопресину

Активація калікреїн-кінінової системи

Посилення рефлекторного впливу із рецепторної зони дуги аорти

Зниження синтезу ангіотензиногену

Збільшення синтезу Na-уретичного гормону

2105 / 4299
Пацієнт доставлений до лікарні з симптомами запаморочення, сухості в роті, зіниці сильно розширені, порушення акомодації, тахікардія, утруднення сечовипускання, атонія кишечнику. Передозування яким препаратом могло викликати дані симптоми?

Фуросемід

Каптоприл

Празозин

Атропіну сульфат

Клофелін

2106 / 4299
Який з легеневих об’ємів НЕМОЖЛИВО визначити за допомогою спірометрії?

Життєва ємність легенів

Залишковий об’єм

Резервний об’єм вдиху

Дихальний об’єм

Резервний об’єм видиху

2107 / 4299
При обстеженні хворого з ендокринною патологією встановлено, що в плазмі крові підвищений рівень тестостерону. Які клітини в організмі чоловіка відповідальні за продукцію цього гормону?

Сперматогенні клітини

Клітини сім’яних міхурців

Гландулоцити сім’яників

Клітини передміхурової залози

Сустентоцити сім’яників

2108 / 4299
У хворого на енцефаліт спостерігаються дихальні рухи постійної амплітуди, які чергуються з періодами апное. Якому типу дихання це притаманно?

Куссмауля

Чейн-Стокса

Біота

Апнейстичне

Стенотичне

2109 / 4299
Чоловік 60-ти років страждає на атеросклероз судин головного мозку. При обстеженні виявлена гіперліпідемія. Вміст якого класу ліпопротеїдів найбільш імовірно буде підвищений при дослідженні сироватки крові?

Ліпопротеїди низької щільності

Холестерин

Хіломікрони

Комплекси жирних кислот з альбумінами

Ліпопротеїди високої щільності

2110 / 4299
Для покращання трофіки серцевого м’яза пацієнту призначено кардонат, до складу якого входить кокарбоксилаза (тіаміндифосфат) - коферментна форма вітаміну:

В2

В12

В10

В1

В5

2111 / 4299
Виділення гормонів кори наднирників регулюється АКТГ аденогіпофізу. Які гормони виділяються наднирниками при дії останнього?

Глюкокортикоїди

Катехоламіни

Мінералокортикоїди

Простагландини

Андрогени

2112 / 4299
До гінеколога звернулася жінка 28ми років з приводу безпліддя. При обстеженні знайдено: недорозвинені яєчники і матка, нерегулярний менструальний цикл. При дослідженні статевого хроматину у більшості соматичних клітин виявлено 2 тільця Бара. Яка хромосомна хвороба найбільш імовірна у жінки?

Синдром Клайнфельтера

Синдром Патау

Синдром трипло-Х

Синдром Шерешевського-Тернера

Синдром Едвардса

2113 / 4299
У пацієнтки з артеріальною гіпертензією виник гіпертензивний криз, який ускладнився набряком легень. Який сечогінний препарат необхідно застосувати у комплексній терапії даної хворої?

Фуросемід

Верошпірон

Тріамтерен

Еуфілін

Амілорид

2114 / 4299
У 2-річної дівчинки була діагностована важка форма міастенії. Який з перерахованих засобів показаний для лікуванні цього захворювання?

Цитітон

Галантаміну гідробромід

Допамін

Норадреналін

Скополамін

2115 / 4299
Сімейний лікар призначив хворому 53-х років для лікування хронічної серцевої недостатності дигоксин. Через 1 місяць виявив симптоми глікозидної інтоксикації. Який препарат необхідно призначити для усунення інтоксикації?

Унітіол

Ціанокобаламін

Морфіну гідрохлорид

Мезатон

Аміназин

2116 / 4299
При огляді 6-місячної дитини лікар виявив незакрите заднє тім’ячко. При нормальному розвитку дитини воно закривається до:

Народження

Кінця другого року життя

6-ти місяців

3-х місяців

Кінця першого року життя

2117 / 4299
У хлопчика водянка яєчка (накопичування рідини між оболонками яєчка). Яка саме оболонка яєчка вміщує цю рідину?

Білкова

Піхвова

М’ясиста

Зовнішня сім’яна

Внутрішня сім’яна

2118 / 4299
Хворому на хронічний гастрит зроблена внутрішньошлункова рН-метрія, за допомогою якої встановлено зменшення кислотності шлункового соку. Функція яких клітин знижена?

Головні екзокриноцити

Ендокриноцити

Парієтальні екзокриноцити

Додаткові клітини

Шийкові клітини

2119 / 4299
З метою визначення максимальної секреції хлористоводневої кислоти шлункового соку пацієнту 42-х років ввели розчин гістаміну. Це призвело до збільшення секреції підшлунковою залозою такого компоненту соку:

Ліпаза

Трипсиноген

Слиз

Амілаза

Бікарбонати

2120 / 4299
У хворого з облітеруючим атеросклерозом проведено десимпатизацію стегнової артерії в ділянці стегнового трикутника. Який вид артеріальної гіперемії виник внаслідок операції'?

Робоча

Реактивна

Нейропаралітична

Нейротонічна

Метаболічна

2121 / 4299
Хворий потрапив до лікарні після іонізуючого опромінення зі скаргами на блювання, анорексію, біль в різних ділянках живота, наявність крові у калі, підвищення температури тіла, млявість. Для якої форми гострої променевої хвороби характерна клінічна картина?

Токсемічна

Кістковомозкова

Змішана

Кишкова

Церебральна

2122 / 4299
В нефрологічній клініці у юнака 19-ти років була виявлена підвищена кількість калію у вторинній сечі. Підвищення секреції якого гормону, імовірно могло викликати такі зміни?

Адреналін

Глюкагон

Альдостерон

Тестостерон

Окситоцин

2123 / 4299
У хворого на інсульт спостерігається порушення заднього відділу задньої ніжки внутрішньої капсули. Які з провідних шляхів там проходять?

Tr Occipitopontinae

Tr. Parietopontinae

Tr. Corticonuclearis

Tr. Corticospinalis

Radiatio acustica et radiatio optica

2124 / 4299
У хворого 40-а років в результаті щелепно-лицьової травми порушилася функція під’язикової і підщелепної залоз зліва - залози почали секретувати невелику кількість густої слини. Функція якого нерва порушена?

Язикоглотковий

Трійчастий

Блукаючий

Лицевий

Під’язиковий

2125 / 4299
На аутопсії померлого, який тривало страждав на гіпертонічну хворобу, патологоанатом виявив, що нирки різко зменшені у розмірах, щільні, поверхня їх рівномірно дрібнозерниста, на розрізі - паренхіма, особливо кіркова речовина, рівномірно виточені. Він дійшов висновку, що це:

Атеросклеротично зморщена нирка

Артеріолосклеротично зморщена нирка

Амілоїдно-зморщена нирка

Пієлонефритично зморщена нирка

2126 / 4299
У хворого в порожнині рота спостерігається повна атрофія сосочків язика, язик червоний, 'лакований', вкритий виразками. В крові: лейкопенія, зсув формули вправо, КП-1,3, мегалоцити, тільця Жолі. Для якої патології це характерно?

Залізодефіцитна анемія

Агранулоцитоз

B12 -дефіцитна анемія

Залізорефрактерна анемія

Гемолітична анемія

2127 / 4299
При вивченні препарату трубчастого органу встановлено, що його середня оболонка утворена гіаліновою хрящовою тканиною, яка формує незамкнені кільця. Який це орган?

Малі бронхи

Великі бронхи

Трахея

Термінальні бронхіоли

Головні бронхи

2128 / 4299
В тканині видаленої слинної залози знайдено: дифузний склероз, прошарки сполучної тканини містять змішаноклі-тинний інфільтрат, часточки залози атрофовані, протоки розширені. Крім того, тканина залози містить кістозну порожнину великих розмірів, внутрішня поверхня її вкрита сплощеним двошаровим епітелієм, просвіт заповнений рідиною, оточуюча тканина склерозована. Діагностуйте захворювання:

Сіалолітіаз

Мономорфна аденома

Ретенційна кіста

Гострий сіалоаденіт

Плеоморфна аденома

2129 / 4299
При втручанні з метою лікування вивиху нижньої щелепи лікар повинен пам’ятати про м’яз, який при скороченні відтягує назовні капсулу і суглобовий диск скронево- нижньощелепного суглоба. Який це м’яз?

M. temporalis

M. mylohyoideus

M. pterygoideus medialis

M. pterygoideus lateralis

M. masseter

2130 / 4299
Зірчастий ретикулум і зовнішній шар клітин емалевого органу піддаються інволюції і після завершення амелогенезу разом з апікальними частинами амелобластів формують:

Зубний наліт (бляшки)

Пелікулу зуба

Муцинову плівку

Кутикулу зуба

2131 / 4299
У 4-річного хлопчика з’явились горбисті, щільні, підшкірно розташовані утворення в ділянці кутів та вздовж гілок двох нижніх щелеп, які зумовили деформацію обличчя. При гістологічному дослідженні біоптату виявлено: в міжбалковому просторі розростання васкуляризованої сполучної тканини, довкола судин маси ацидофільного матеріалу, багатоядерні гігантські клітини, окремі примітивні кісткові балочки. Діагностуйте захворювання:

Периферійна гігантоклітинна гранульома

Фіброзна дисплазія

Хронічний фіброзний періостит

Херувізм

Еозинофільна гранульома

2132 / 4299
При дослідженні бронхобіоптату встановлено атрофію слизової оболонки, кістозне перетворення залоз, осередкову метаплазію покривного призматичного епітелію в багатошаровий плоский, збільшення числа келихоподібних клітин, місцями у стінці бронха та особливо у слизовій оболонці різко виражена клітинна запальна інфільтрація і розростання грануляційної тканини, яка вибухає у просвіт бронха у вигляді поліпа. Який найбільш імовірний діагноз?

Бронхопневмонія

Гострий бронхіт

Гостра пневмонія

Проміжна пневмонія

Хронічний бронхіт

2133 / 4299
При остеолатеризмі зменшується міцність колагену, що зумовлена помітним зменшенням утворення поперечних зшивок у колагенових фібрилах. Причиною цього явища є зниження активності такого ферменту:

Колагеназа

Пролілгідроксилаза

Лізилоксидаза

Моноаміноксидаза

Лізилгідроксилаза

2134 / 4299
Під час мікроскопічного дослідження в залозі дитини 10-ти років були визначені тільки серозні кінцеві відділи, міжчасточкові протоки вистелені двошаровим чи багатошаровим епітелієм. Визначте залозу:

Підщелепна

Печінка

Привушна

Підшлункова

Під’язикова

2135 / 4299
Хвороба Хартнепа зумовлена точковою мутацією лише одного гена, наслідком чого є порушення всмоктування амінокислоти триптофану в кишечнику та реабсорбції її в ниркових канальцях. Це призводить до одночасних розладів у травній і сечовидільній системах. Яке генетичне явище спостерігається в цьому випадку?

Кодомінування

Плейотропія

Полімерія

Комплементарна взаємодія

Неповне домінування

2136 / 4299
Відомо, що частина диоксиду вуглецю використовується в організмі в біосинтезі жирних кислот, сечовини, глюконеогенезі тощо. Який вітамін утворює СO2-транспортуючу форму для цих реакцій?

Біотин

Ретинол

Нікотинамід

Рибофлавін

Тимін

2137 / 4299
У хворого, який страждає на вугрі та запальні зміни шкіри обличчя, при мікроскопії матеріалу з осередків ураження виявлені живі членистоногі розміром 0,2-0,5 мм. Вони мають витягнуту червоподібну форму, чотири пари коротких кінцівок, розміщених у середній частині тіла. Виявлені членистоногі викликають таке захворювання:

Шкірний міаз

Педикульоз

Фтиріоз

Демодекоз

Короста

2138 / 4299
У хворого 35-ти років травма голови викликала втрату тактильних і температурних відчуттів. Яку звивину було пошкоджено внаслідок травми?

Надкрайова

Зацентральна

Поясна

Кутова

Передцентральна

2139 / 4299
Сечокам’яна хвороба ускладнилася виходом конкременту з нирки. На якому рівні сечоводу, найімовірніше, він може зупинитися?

В середній черевній частині

На межі черевної та тазової частин

На 2 см вище впадіння в сечовий міхур

На 5 см вище тазової частини

У нирковій мисці

2140 / 4299
В гістологічному препараті нижньої щелепи ембріону виявляється зубний зачаток, в якому зубний сосочок утворений дрібними зірчастими базофільно забарвленими клітинами. Яка тканина утворює цю частину зубного зачатку?

Епітеліальна

Кісткова

Ретикулярна

Мезенхіма

Хрящова

2141 / 4299
При огляді ротової порожнини на вестибулярній поверхні нижнього різця зліва виявлене утворення грибоподібної форми рожевого кольору до 2 см, яке широкою ніжкою фіксоване до надальвео-лярної тканини. Під час гістологічного дослідження виявлено розгалужені судини капілярного типу з судинними бруньками, ділянками крововиливів та осередки гемосидерозу Який найбільш імовірний діагноз?

Фіброзний епуліс

Фіброматоз ясен

Кавернозна гемангіома

Ангіоматозний епуліс

Гігантоклітинний епуліс

2142 / 4299
Хворому з важким перебігом ревматоїдного артриту для постійного прийому був призначений преднізолон. Який режим прийому є оптимальним з урахуванням хронофармакологічних особливостей дії преднізолону та фізіологічних коливань секреції кортикостероїдів в організмі?

Вся доза ввечері

Усі наведені схеми рівноцінні

1/3 дози тричі на день

2/3 добової дози вранці, 1/3 дози вдень

Вся доза вранці

2143 / 4299
Пацієнт 68-ми років звернувся до кардіолога із скаргами на підвищення артеріального тиску, біль в ділянці серця, відчуття перебоїв у роботі серця. Призначте препарат з групи β1 -адреноблокаторів для лікування даної патології:

Морфіну гідрохлорид

Ноотропіл

Бензилпеніцилін

Метопролол

Фенотерол

2144 / 4299
Людина згинає та розгинає передпліччя без навантаження, спираючись ліктем на стіл. Який вид м’язового скорочення має місце у m. biceps brachii?

Гладкий тетанус

Зубчастий тетанус

Ауксотонічне

Ізометричне

Ізотонічне

2145 / 4299
У жінки, що тривало приймала антибіотики з приводу кишкової інфекції, розвинулося ускладнення з боку слизової порожнини рота у вигляді запального процесу і білого нальоту, у якому при бактеріологічному дослідженні були виявлені дріжджеподібні грибки Candida albicans. Який з перерахованих препаратів показаний для лікування цього ускладнення?

Флуконазол

Поліміксин

Тетрациклін

Фуразолідон

Бісептол

2146 / 4299
Мембранний потенціал спокою клітини змінився з -85 мВ до -90 м. Причиною цього може бути активація таких каналів мембрани клітини:

Калієві

Кальцієві

Калієві та натрієві

Натрієві

Калієві та кальцієві

2147 / 4299
Під час розтину тіла виявлено: легені щільні, коричневого кольору за рахунок відкладання ендогенного пігменту. Відомо, що за життя у хворого мав місце хронічний венозний застій в малому колі кровообігу. Який патологічний процес викликав таку картину?

Гемосидероз

Кальциноз

Жовтяниця

Меланоз

Порфірія

2148 / 4299
Організми мають ядро, оточене ядерною мембраною. Генетичний матеріал зосереджений переважно в хромосомах, які складаються з ниток ДНК і білкових молекул. Діляться ці клітини мітотично. Це:

Прокаріоти

Бактерії

Бактеріофаги

Еукаріоти

Віруси

2149 / 4299
Хвора звернулася зі скаргами на припухлість в передній ділянці шиї, схуднення, випуклість очей, тахікардію, неврівноваженість. Яка залоза уражена?

Гіпофіз

Прищитоподібні залози

Тимус

Епіфіз

Щитоподібна залоза

2150 / 4299
Під час аускультації хворого попросили глибоко дихати. Після 10 дихальних рухів хворий знепритомнів, що пов’язано з:

Респіраторним алкалозом

Зменшенням кисневої ємності крові

Респіраторним ацидозом

Еритроцитозом

Еритропенією

2151 / 4299
Хворому на виразкову хворобу шлунка 45-ти років призначено препарат в основі дії якого є антимікробний ефект. Який це препарат?

Фталазол

Бісакодил

Омепразол

Метронідазол

Дитилін

2152 / 4299
Одна з тканин зуба постійно відкладається в області верхівки кореня, що обумовлює його видовження. Цей процес компенсує стирання коронки і забезпечує постійність загальної довжини зуба. Яка тканина зуба є репаративно активною у постнатальному періоді?

Пухка сполучна

Кісткова

Емаль

Щільна сполучна

Цемент

2153 / 4299
Чоловіка 29-ти років непокоїть різкий біль шкіри обличчя. Який нерв уражений?

Трійчастий

Язикоглотковий

Лицевий

Блукаючий

Окоруховий

2154 / 4299
У людей, які проживають на великих висотах над рівнем моря, відмічається збільшення кількості еритроцитів, що спричиняється:

Посиленим утворенням нирками еритропоетину

Посиленим виділенням нирками реніну

Впливом гіпоксії на розпад еритроцитів

Підвищеним споживанням тканинами кисню

Гальмівним впливом гіпоксії на кістковий мозок

2155 / 4299
У бактеріологічній лабораторії проводилася експертиза шкур тварин за допомогою імунної реакції преципітації за Асколі. Позитивний результат цієї реакції свідчить про наявність:

Збудника бруцельозу

Антигенів збудника сибірки

Токсину анаеробної інфекції

Поверхневого антигену ієрсиній

Збудника чуми

2156 / 4299
Після вживання м’ясної консерви у школяра з’явилися неврологічні симптоми. Був поставлений діагноз: ботулізм. Які екстрені методи лікування необхідно використати?

Введення антибіотиків

Введення сульфаніламідних препаратів

Призначення проносних засобів

Введення антиботулінічної вакцини

Введення антиботулінічної сироватки

2157 / 4299
У хворого запальний процес у крилопіднебінній ямці. Інфекція поширилась в носову порожнину. Через яке анатомічне утворення розповсюдилась інфекція?

Foramen rotundum

Canalis ptherygoideus

Canalis palatinus minor

Foramen sphenopalatinum

Canalis palatinus major

2158 / 4299
У пацієнта 32-х років має місце гіповітаміноз B2. Причиною виникнення специфічних симптомів (ураження епітелію, слизових, шкіри, рогівки ока) найбільш імовірно є дефіцит:

Цитохромоксидази

Цитохрому в

Цитохрому с

Флавінових коферментів

Цитохрому а1

2159 / 4299
Жінці, яка скаржиться на постійне відчуття страху, тривоги, поставлено діагноз неврозу та призначено препарат з анксіолітичною дією. Який це препарат?

Кофеїн-бензоат натрію

Аміназин

Пірацетам

Настойка женьшеню

Діазепам

2160 / 4299
У пробірку, що містить 0,3% розчин NaCl, додали краплю крові. Що відбудеться з еритроцитами?

Біологічний гемоліз

Зморшкування

Осмотичний гемоліз

Змін не буде

Механічний гемоліз

2161 / 4299
Після перенесеної травми голови у хворого бувають напади інтенсивного болю в ділянці обличчя і судоми жувальних м’язів. Який нерв найімовірніше травмований?

N. abducens

N. oculomotorius

N. olphactorius

N. facialis

N. trigeminus

2162 / 4299
Вивчається родовід сім’ї, в якій спостерігаються надмірно великі зуби. Ця ознака трапляється в усіх поколіннях тільки у чоловіків і успадковується від батька до сина. Визначте тип успадкування :

Зчеплений з Y-хромосомою

Автосомно-домінантний

Автосомно-рецесивний

Зчеплений з Х-хромосомою домінантний

Зчеплений з Х-хромосомою рецесивний

2163 / 4299
У постраждалого перелом тіла нижньої щелепи. Кровотеча з рани. Яка артерія ушкоджена?

A. lingualis

A. facialis

A. maxillaris

A. carotis externa

A. alveolaris inferior

2164 / 4299
На розтині тіла 68-річної жінки, що померла від гострої серцевої недостатності, у задній стінці лівого шлуночка серця виявлена ділянка неправильної форми, розмірами 6,5х4,5 см, в’яла, блідо-жовтуватого кольору, оточена зоною гіперемії. Як розцінив патологоанатом виявлені зміни?

Аневризма серця

Післяінфарктний кардіосклероз

Дифузний кардіосклероз

Гострий інфаркт міокарда

Вогнищевий міокардит

2165 / 4299
На розтині тіла чоловіка 47-ми років, що помер від легеневої кровотечі, у 2 сегменті правої легені виявлена порожнина округлої форми з нерівними краями розміром 5,5 см; внутрішня поверхня її вкрита щільнуватими жовтуватими масами, що переходять у тканину легені. При гістологічному дослідженні - внутрішній шар складається з розплавлених казеозних мас, некротизованої тканини легені. Який процес розвинувся в легені?

Гострий кавернозний туберкульоз

Рак легені, що розпадається

Гострий абсцес

Хронічний кавернозний туберкульоз

Хронічний абсцес

2166 / 4299
Екзотоксин дифтерійної палички обробили 0,3-0,4% формаліном і витримали 30 днів у термостаті при температурі 40o Який препарат був отриманий у результаті проведених маніпуляцій?

Лікувальна сироватка

Антитоксин

Діагностикум

Анатоксин

Діагностична сироватка

2167 / 4299
В економічно розвинених країнах поширеним захворюванням є карієс зубів. Це захворювання уражає більш 95% населення. Що відіграє головну роль у демінералізації твердих тканин зуба при карієсі?

Розлад регуляції метаболізму

Органічні кислоти

Неповноцінне харчування

Недостатність вітаміну C

Екстремальні впливи на організм

2168 / 4299
При вивченні гістологічного препарату пульпи зуба було відмічено, що в сполучній тканині переважають пучки колагенових волокон, шар дентинобластів тонкий, проміжний шар виражений слабо. В якій ділянці зуба пульпа має такі особливості?

Периферичний шар пульпи

Коренева пульпа

Центральний шар пульпи

Коронкова пульпа

Шар Вейля

2169 / 4299
При огляді ротової порожнини стоматолог виявив каріозну порожнину на поверхні коронки другого малого кутнього зуба, яка межує з першим великим кутнім зубом. Назвіть пошкоджену поверхню коронки:

Facies vestibularis

Facies lingualis

Facies mesialis

Facies occlusalis

Facies distalis

2170 / 4299
У собаки вироблений умовний слиновидільний рефлекс на вмикання світла. Вмикання дзвоника під час виконання даного рефлексу призведе до розвитку наступного виду гальмування:

Диференціювання

Позамежне

Умовне гальмування

Зовнішнє

Згасання

2171 / 4299
Чоловік звернувся до стоматолога зі скаргою, що в нього нижня щелепа не рухається назад. Встановлено, що в нього пошкоджений такий м’яз :

Скроневий

Двочеревцевий

Бічний крилоподібний

Жувальний

Присередній крилоподібний

2172 / 4299
У пацієнта внаслідок запального процесу виникло надмірне збудження вушно-скроневого нерва. При цьому привушною слинною залозою буде виділятися:

Велика кількість в’язкої слини

Мала кількість в’язкої слини

Виділення слини припиниться

Велика кількість рідкої слини

Мала кількість рідкої слини

2173 / 4299
Хворому 55-ти років для комплексного лікування легеневої форми туберкульозу призначений протитуберкульозний засіб. Який з вказаних лікарських засобів проявляє свою антимікробну активність виключно по відношенню до мікобактерій туберкульозу?

Ізоніазид

Стрептоміцину сульфат

Циклосерин

Гатіфлоксацин

Канаміцину сульфат

2174 / 4299
У дитини 9-ти років швидко підвищилась температура тіла до 39°С, з’явився біль у горлі. Об’єктивно: зів та мигдалики яскраво-червоного кольору. Язик набряклий малиново- червоного кольору зі збільшеними грибоподібними сосочками. Шкіра тіла та обличчя, крім носо-губного трикутника, вкрита густими червоними плямами величиною з макове зерно. Підщелепні лімфатичні вузли при пальпації болісні. Яке захворювання у дитини?

Скарлатина

Дифтерія

Поліомієліт

Менінгококовий назофарингіт

Кір

2175 / 4299
Хвора 20-ти років звернулась до лікаря зі скаргами на кровоточивість та біль у яснах, що з’явилися через 2 дні після прийому сульфадимезину. Об’єктивно: ясенні сосочки та ясенний край гіперемовані, набряклі, кровоточать при незначному подразненні. Яка патологія розвинулась в яснах?

Гострий виразковий гінгівіт

Геморагічний гінгівіт

Епулід

Гострий катаральний гінгівіт

Хронічний катаральний гінгівіт

2176 / 4299
У хворого 48-ми років з обширним інфарктом міокарда розвинулась серцева недостатність. Який патогенетичний механізм сприяв розвитку серцевої недостатності у хворої?

Гостра тампонада серця

Зменшення маси функціонуючих міо-кардіоцитів

Перевантаження об’ємом

Реперфузійне ураження міокарда

Перевантаження тиском

2177 / 4299
До приймального відділення надійшов хворий з важкою щелепно-лицевою травмою. Який препарат йому необхідно ввести для зняття больового шоку?

Промедол

Ібупрофен

Пантогам

Сиднокарб

Мідокалм

2178 / 4299
У померлого внаслідок серцевої недостатності на шкірі відзначаються сліди висипу у вигляді плям і крапок. В області криж, остистих відростків хребців - пролежні. При мікроскопічному дослідженні ЦНС: в судинах мікроциркуляторного русла і дрібних артеріях - деструктивно-проліферативний ендотромбоваскуліт з наявністю гранульом Попова, в серці - інтерстиційний міокардит. Який з перелічених діагнозів найбільш імовірний?

Черевний тиф

Ку-гарячка

ВІЛ-інфекція

Вузликовий періартеріїт

Висипний тиф

2179 / 4299
У хворого під час гіпертонічного кризу виник геморагічний інсульт, внаслідок чого спостерігається відсутність довільних рухів, підвищення сухожильних рефлексів та тонусу м’язів лівих руки та ноги. Як називається таке порушення рухової функції?

Тетраплегія

Моноплегія

Параплегія

Геміплегія

Млявий параліч

2180 / 4299
В лабораторії вивчали вірулентність збудника дифтерії. При цьому проводили внутрішньоочеревинне зараження лабораторних тварин. В ході експерименту встановлена доза бактерій, яка викликає загибель 95% тварин. Яку одиницю вимірювання вірулентності визначали в лабораторії?

LD5

LD50

ІД

DLM

DCL

2181 / 4299
При електронній мікроскопії у цитоплазмі клітини, поблизу ядра, виявлена мембранна органела, яка складається з 5-10 пласких цистерн, з розширеними периферичними ділянками, від яких від’єднуються маленькі пухирці - лізосоми. Назвіть цю органелу:

Рибосома

Клітинний центр

Мітохондрія

Комплекс Гольджі

Цитоскелет

2182 / 4299
При обробці фрезою великого кутнього зуба стоматолог інструментом, що зірвався, глибоко поранив щоку, пошкодивши при цьому не лише слизову оболонку але й м’яз. Який?

Жувальний

Щелепно-під’язиковий м’яз

Щічний

Коловий м’яз рота

Великий виличний м’яз

2183 / 4299
Шкіра людини дуже міцна на розрив. Відомо, що шкіра складається з епітеліальної тканини і двох видів сполучної тканини. Яка з нижче перерахованих тканин забезпечує міцність шкіри?

Пухка сполучна тканина

Одношаровий епітелій

Багатошаровий плоский епітелій

Перехідний епітелій

Щільна неоформлена сполучна

2184 / 4299
У хворого на стоматологічному прийомі виник напад пароксизмальної тахікардії, у зв’язку з чим йому ввели лідокаїн. З яким механізмом дії лідокаїну пов’язаний його протиаритмічний ефект?

Сумація

Мембранно-іонний

Антагонізм

Кумуляція

Потенціювання

2185 / 4299
При огляді ротової порожнини стоматолог помітив появу у дитини перших постійних бічних різців. Який вік дитини?

6 років

9 років

7 років

13 років

1І років

2186 / 4299
Рибалка наловив риби із річки, трошки підсмажив її на вогнищі і з’їв, майже напівсиру. Через декілька тижнів потому в нього з’явились ознаки ураження печінки і підшлункової залози. Лабораторний аналіз фекалій показав наявність дрібних яєць гельмінта. Яким трематодозом найбільш імовірно заразився рибалка?

Шистосомоз

Дикроцеліоз

Опісторхоз

Парагонімоз

Фасціольоз

2187 / 4299
Вивчається мітотичний поділ клітин епітелію ротової порожнини. Встановлено, що в клітині диплоїдний набір хромосом. Кожна хромосома складається з двох максимально спіралізованих хроматид. Хромосоми розташовані у площині екватору клітини. Ця картина характерна для такої стадії мітозу:

Профаза

Прометафаза

Анафаза

Метафаза

Тілофаза

2188 / 4299
При огляді порожнини рота стоматолог виявив каріозну порожнину на поверхні коронки нижнього ікла, зверненої до першого малого кутнього зуба. Назвіть цю поверхню.

Facies mesialis

Facies lingualis

Facies distalis

Facies occlusalis

Facies vestibularis

2189 / 4299
В умовах експерименту проведено блокаду язико-глоткового нерву. При цьому буде спостерігатися зниження сприйняття такого подразника:

Солоне

Кисле

Гірке

Солодке

2190 / 4299
Під час спортивних змагань боксер отримав сильний удар у живіт, що привело до нокауту через короткочасне падіння артеріального тиску. Які фізіологічні механізми викликали цей стан?

Зміна транскапілярного обміну

Раптова зміна кількості рідини у організмі

Подразнення парасимпатичних нервів

Подразнення симпатичних нервів

Ішемія центральної нервової системи

2191 / 4299
У 12-річної дитини непереносимість ряду харчових продуктів. Їх вживання викликає алергічну реакцію у вигляді висипань на шкірі, що сверблять. Який антигістамінних засіб слід призначити, щоб не заважати шкільним заняттям дитини?

Ефедрин

Димедрол

Еуфілін

Лоратадин

Диклофенак

2192 / 4299
Чоловік 25-ти років звернувся до лікаря зі скаргами на біль в нижній щелепі справа, припухлість, високу температуру, озноб. При обстеженні макроскопічно відмічається відшарування окістя з накопиченням запального ексудату між ним і кісткою, з перифокальним набряком м’яких тканин та частково розплавленим окістям. Що розвинулося у хворого?

Локальний пародонтит

Гранулюючий періодонтит

Катаральний гінгівіт

Гнійний періостит

Гангренозний пульпіт

2193 / 4299
У хворого з алкогольним ураженням печінки порушені процеси біотрансформації ксенобіотиків та ендогенних токсичних сполук. Зниження активності якого хромопротеїну може бути причиною цього?

Цитохром b

Цитохромоксидаза

Цитохром Р-450

Цитохром с1

Гемоглобін

2194 / 4299
Піддослідному собаці ввели гормон, що призвело до збільшення швидкості клубочкової фільтрації за рахунок розширення приносної артеріоли і зменшення реабсорбції іонів натрію і води в канальцях нефрона. Який гормон було введено?

Тироксин

Адреналін

Передсердний натрійуретичний

Тестостерон

Окситоцин

2195 / 4299
Тривале лікування гіпофункції щитоподібної залози спричинило загальну дистрофію, карієс зубів, тахікардію, тремор кінцівок. Який засіб викликав побічні ефекти?

Хумулін

Преднізолон

Тирокальцитонін

L-тироксин

Паратиреоїдин

2196 / 4299
У жінки встановлено діагноз - рак шийки матки. З яким вірусом може бути асоційована ця патологія?

Varicella-Zoster вірус

Папілома вірус

Аренавірус

Цитомегаловірус

Вірус простого герпесу тип 2

2197 / 4299
У пацієнта 59-ти років, що знаходиться на обстеженні в стаціонарі, виявлено глюкозурію, рівень глюкози в крові 3,0 ммоль/л. Найбільш імовірною причиною глюкозурії може бути:

Гіпертонічна хвороба

Пелагра

Мікседема

Нецукровий діабет

Захворювання нирок

2198 / 4299
При обстеженні підлітка, що страждає на ксантоматоз, виявлено сімейну гіперхолестеролемію Концентрація яких транспортних форм ліпідів підвищується при цьому захворюванні?

Ліпопротеїди дуже низької щільності

Ліпопротеїди високої щільності

Хіломікрони

Ліпопротеїди низької щільності

2199 / 4299
У молодої жінки, яка увійшла до приміщення з високою концентрацією тютюнового диму, раптово виникли рефлекторні кашель та спазм бронхів. Подразнення яких рецепторів викликало дані захисні рефлекси?

Механорецептори легень

Центральні хеморецептори

Ірритантні рецептори

Юкстамедулярні рецептори

Рецептори плеври

2200 / 4299
У хворого після тривалого психоемоційного перенапруження спостерігається підвищення артеріального тиску, що супроводжується серцебиттям, кардіалгіями, головним болем, запамороченням. Який процес відіграє домінуючу роль у формуванні артеріальної гіпертензії у даному випадку?

Підвищення тонусу артеріол

Збільшення об’єму циркулюючої крові

Підвищення частоти серцевих скорочень

Збільшення серцевого викиду

Підвищення тонусу венул

2201 / 4299
При цитологічних дослідженнях було виявлено велику кількість різних молекул т-РНК, які доставляють амінокислоти до рибосоми. Кількість різних типів т-РНК у клітині буде дорівнювати кількості:

Нуклеотидів

Білків, синтезованих у клітині

Амінокислот

Триплетів, що кодують амінокислоти

Різних типів і-РНК

2202 / 4299
Відомо, що деякі гельмінти в личинковій стадії паразитують в м’язах риби. Вкажіть назву гельмінтозу, яким може заразитися людина, вживаючи рибу:

Теніарінхоз

Дифілоботріоз

Теніоз

Дикроцеліоз

Трихінельоз

2203 / 4299
До лікаря звернувся пацієнт 25-ти років з приводу сильного свербіжу шкіри, особливо між пальцями рук, у пахвових западинах, на нижній частині живота. При огляді шкіри хворого виявлено звивисті ходи білувато-брудного кольору. Який діагноз міг припустити лікар?

Дерматотропний лейшманіоз

Педикульоз

Скабієс

Демодекоз

Міаз

2204 / 4299
У хворого виявлена короткочасна пневмонія. Міграція личинок якого гельмінта може призвести до цієї хвороби?

Гострик

Альвеокок

Волосоголовець

Карликовий ціп’як

Аскарида

2205 / 4299
При огляді хворого з ранами, що кровоточать, лікар виявив пошкодження тканин личинками, а також локальні місця нагноєння. Діагноз: облігатний міаз. Збудником цього захворювання є:

Триатомовий клоп

Муха цеце

Жигалка осіння

Муха вольфартовая

Муха хатня

2206 / 4299
Є потреба катетеризації сечового міхура у чоловіка. В якій частині уретри або структурі може виникати опір катетеру?

Простатична

Перетинчаста

Губчаста

Зовнішній отвір сечівника

Внутрішній отвір сечівника

2207 / 4299
Хворий із запаленням слизової язика (глосит) скаржиться на розлад смакової чутливості передніх двох третин язика. Ураженням якого нерва воно викликане?

Язикоглотковий

Барабанна струна

Барабанний

Малий кам’янистий

Язиковий

2208 / 4299
Хворий звернувся до лікаря зі скаргою на утруднення під час жування. При обстеженні виявлена атрофія правих скроневого і жувального м’язів. При відкриванні рота щелепа відхиляється ліворуч. Який нерв уражений?

Нижній альвеолярний

Щелепно-під’язиковий

Лицевий

Верхньощелепний

Рухова частина нижньощелепного

2209 / 4299
У постраждалого 46-ти років визначається перелом у ділянці внутрішньої поверхні лівого гомілковоступневого суглоба. Де найбільш імовірно відбувся перелом?

Надп’яткова кістка

Нижня третина малогомілкової кістки

Латеральна кісточка

П’яткова кістка

Медіальна кісточка

2210 / 4299
У хворого 60-и років виявлено розширення вен стравоходу, прямої кишки та підшкірних вен передньої черевної стінки. Система якої вени ушкоджена?

Верхня брижова

Нижня порожниста вена

Ворітна

Верхня порожниста вена

Непарна вена

2211 / 4299
У дитини 2-х років після перенесеного грипу з’явилися скарги на 6іль у вусі. Лікар виявив зниження слуху та запалення середнього вуха. Яким шляхом інфекція потрапила до середнього вуха?

Через atrium mastoideum

Через canalis nasolacrimalis

Через слухову трубу

Через canalis caroticus

Через foramen jugularis

2212 / 4299
В гістологічному препараті визначається слизова оболонка, вкрита багатошаровим плоским незроговілим, місцями - багатошаровим плоским зроговілим епітелієм. До складу слизової оболонки входить також власна пластинка, м’язова пластинка відсутня. Визначте місце локалізації такої слизової оболонки:

Шлунок

Стравохід

Тонка кишка

Трахея

Ротова порожнина

2213 / 4299
На електронній мікрофотографії поперечного шліфу зуба в емалі виявляються утворення овальної, полігональної і аркової форм, що складаються з щільно вкладених і упорядкованих кристалів гідроксиапатитів. Назвіть дане утворення:

Смуги Гунтера-Шрегера

Лінії Ретціуса

Колагенове волокно

Перикіматій

Емалева призма

2214 / 4299
При обстеженні окуліст з’ясував, що пацієнт не розрізняє синій та зелений кольори при нормальному сприйнятті іншої кольорової гами. З порушенням функції яких структур сітківки це пов’язано?

Біполярні нейрони

Колбочкові нейрони

Амакринні нейрони

Горизонтальні нейрони

Паличкові нейрони

2215 / 4299
У ході утворення плащового дентину у молочному зубі відбулося порушення секреторної активності одонтобластів. Утворення яких волокон зміниться?

Тангенційні колагенові волокна Ебнера

Еластичні

Нервові

Радіальні колагенові волокна Кор-фа

Ретикулярні

2216 / 4299
У процесі набуття клітинами специфічних для них морфологічних, біохімічних та функціональних особливостей клітини обмежуються у виборі можливостей шляхів розвитку. Яку назву має таке набуте обмеження?

Рецепція

Адгезія

Детермінація

Комітування

Капацитація

2217 / 4299
При непрямому гістогенезі кісткової тканини трубчастих кісток між епіфізарним та діафізарним центрами окостеніння утворюється пластинка, що в подальшому забезпечує ріст кісток у довжину. Як називається ця структура?

Метафізарна пластинка

Шар внутрішніх генеральних пластинок

Остеон

Кісткова пластинка

Кісткова манжетка

2218 / 4299
При дослідженні оболонок головного мозку медичний експерт знайшов зяючі венозні судини, що зрощені з навколишніми тканинами. Назвіть, до яких вен відносяться ці судини:

Вени із слабким розвитком м’язових елементів

Вени із середнім розвитком м’язових елементів

Венули

Вени із сильним розвитком м’язових елементів

Вени безм’язового типу

2219 / 4299
У експериментальної тварини подразнювали периферичний відрізок симпатичних волокон, що іннервують під’язикову слинну залозу. У результаті з фістули протоки залози виділяється:

Багато рідкої слини

Мало в’язкої слини

Слина не виділяється

Мало рідкої слини

2220 / 4299
При обстеженні хворого з травматичним пошкодженням головного мозку виявлено, що він втратив дотикову чутливість. Який відділ кори мозку пошкоджений?

Потилична частка кори

Лобна частка кори

Передня центральна звивина

Задня центральна звивина

Тім’яна частка кори

2221 / 4299
Досліджуються рецептори, інформація від яких прямує до кори головного мозку без участі таламусу. Які це рецептори?

Зорові

Дотикові

Смакові

Слухові

Нюхові

2222 / 4299
При дослідженні ізольованого кардіоміоциту встановлено, що він НЕ ГЕНЕРУЄ імпульси збудження автоматично. З якої структури серця отриманий кардіоміоцит?

Пучок Гіса

Шлуночок

Волокно Пуркін’є

Атріовентрикулярний вузол

Сино-атріальний вузол

2223 / 4299
Після руйнування структур ЦНС тварина втратила орієнтувальні рефлекси. Що саме зруйнували?

Медіальні ретикулярні ядра

Червоні ядра

Чорна речовина

Чотиригорбкове тіло

Латеральні вестибулярні ядра

2224 / 4299
У жінки 32-х років запалення ясен супроводжується їх гіпоксією. Утворення якого метаболіту вуглеводного обміну значно збільшується при цьому в тканинах пародонта?

Глікоген

Рибозо-5-фосфат

Глюкозо-6-фосфат

Лактат

НАДФ-Н

2225 / 4299
У добовому раціоні дорослої здорової людини повинні бути жири, білки, вуглеводи, вітаміни, мінеральні солі та вода. Вкажіть добову кількість білку, яка забезпечує нормальну життєдіяльність організму:

10-20

70-80

100-120

50-60

40-50

2226 / 4299
Пацієнт звернувся зі скаргами на напади утрудненого дихання, запаморочення. Він працює на хімічному підприємстві з виробництва синильної кислоти. Порушенням функції якого ферменту можуть бути зумовлені зазначені симптоми?

Каталаза

Піруватдегідрогеназа

Сукцинатдегідрогеназа

Лактатдегідрогеназа

Цитохромоксидаза

2227 / 4299
Аміак є дуже отруйною речовиною, особливо для нервової системи. Яка речовина бере особливо активну участь у знешкодженні аміаку в тканинах мозку?

Аспарагінова кислота

Глутамінова кислота

Буферні системи

Сечовина

Аргінін

2228 / 4299
При захворюваннях підшлункової залози порушується утворення та секреція трипсину. Назвіть речовини, травлення яких буде порушене?

Ліпіди

Вуглеводи

Білки

Нуклеїнові кислоти

Фосфоліпіди

2229 / 4299
При спадковій оратацидурії виділення оротової кислоти в багато разів перевищує норму. Синтез яких речовин буде порушений при цій патології?

Сечова кислота

Біогенні аміни

Сечовина

Піримідинові нуклеотиди

Пуринові нуклеотиди

2230 / 4299
Чоловіку 37-ми років при лікуванні гострого пульпіту було введено розчин новокаїну. Через кілька хвилин у пацієнта розвинувся анафілактичний шок. З яким імуноглобуліном головним чином взаємодіє в організмі антиген при даній алергічній реакції?

IgE

IgG

IgA

IgM

IgD

2231 / 4299
У хворого, якому було екстирповано зуб з приводу гострого гнійного періоститу, спостерігалася тривала кровотеча з комірки, яку не можна було спинити звичайними методами. У крові: ер.- 2,9•1012/л, НЬ- 90 г/л; КП-0,9; тромб.- 60•109/л; лейк.- 52•109/л.; базофіли - 0, еозинофіли - 1%, моноцити - 0, нейтрофіли: юні - 0, пали-чкоядерні - 2%, сегментоядерні -18%, лімфоцити - 8%, моноцити - 1%, мі-єлобласти - 70%. Яке захворювання крові наявне у цього хворого?

Недиференційований лейкоз

Еритромієлоз

Хронічний мієлоїдний лейкоз

Гострий мієлоїдний лейкоз

Промієлоцитарний лейкоз

2232 / 4299
У водолаза, що проводив роботи на великій глибині, при швидкому поверненні в умови нормального атмосферного тиску з’явилися біль у суглобах, свербіж шкіри, порушення зору, непритомність. Як називається описане явище?

Стан невагомості

Баротравма

Гіпероксія

Синдром вибухової декомпресії

Хвороба декомпресії

2233 / 4299
При електрокардіографічному дослідженні пацієнта 59-ти років, хворого на гіпертонічну хворобу, виявлено: ритм синусовий, правильний, ЧСС-92/хв, тривалість PQ- 0,2 с, QRS- не змінений. У хворого порушена така властивість серця:

Скоротливість

Провідність

Автоматизм

Рефрактерність

Збудливість

2234 / 4299
У чоловіка 54-x років, який 12 років тому хворів на ревматичний міокардит та ендокардит, наявна недостатність мітрального клапану. Дослідження показали, що запального процесу зараз немає, хвилинний об’єм кровообігу достатній. Якому поняттю загальної нозології відповідає дана умова?

Компенсаторна реакція

Патологічний процес

Типовий патологічний процес

Патологічний стан

Патологічна реакція

2235 / 4299
Артеріальна гіпертензія у хворого обумовлена стенозом ниркових артерій. Активація якої системи є головною ланкою в патогенезі цієї форми гіпертензії?

Симпато-адреналова

Ренін-ангіотензинова

Гіпоталамо-гіпофізарна

Калікреїн-кінінова

Парасимпатична

2236 / 4299
У людини, яка тривалий час голодувала, розвинулись набряки. Який основний механізм виникнення цих набряків?

Збшьшення гідростатичного тиску венозної крові

Збільшення онкотичного тиску міжклітинної речовини

Зменшення об’єму циркулюючої крові

Зменшення онкотичного тиску плазми крові

Зменшення гідростатичного тиску міжклітинної речовини

2237 / 4299
В біоптаті слизової носа знайдені епітеліоїдні клітини, плазмоцити, клітини Мікулича, еозинофільні тільця Русселя. Який найбільш імовірний діагноз?

Алергічний риніт

Сифіліс

Риносклерома

Респіраторно-синцитіальна інфекція

Туберкульоз

2238 / 4299
Хворий 20-ти років звернувся із скаргами на появу короткочасних болісних відчуттів в області 2-го зуба справа на нижній щелепі під дією хімічних або механічних подразників. При огляді зуба виявлений середньої глибини дефект твердих тканин, що перетинає дентино-емалеву межу. Дентин пігментований, дещо розм’якшений. Який найбільш імовірний діагноз?

Середній карієс

Флюороз

Клиноподібний дефект

Поверхневий карієс

Глибокий карієс

2239 / 4299
6-річну дитину доставлено в стаціонар у стані асфіксії. У гортані виявлено сірувато-жовті плівки, що легко видаляються. Який вид запалення розвинувся?

Фібринозне

Катаральне

Десквамативно-некротичне

Геморагічне

Гнійне

2240 / 4299
Макроскопічно печінка збільшена в розмірах, ущільнена, тканина сірувато-жовтого кольору, з сальним блиском. Який патологічний процес лежить в основі описаних змін?

Мукоїдне набухання

Амілоїдоз

Гіаліноз

Гемахроматоз

Жирова дистрофія

2241 / 4299
При дослідженні біоптату збільшеного шийного лімфатичного вузла виявлена стертість малюнка, тканина його представлена великою кількістю проліферуючих лімфоцитів з домішкою поодиноких клітин Березовського-Штернберга. Ці зміни свідчать про таке захворювання:

Нодулярно-склеротичний варіант лімфогранулематозу

Лімфогранулематоз з переважанням лімфоїдної тканини

Лімфогранулематоз з виснаженням лімфоїдної тканини

Лімфосаркома

Змішано-клітинний варіант лімфогранулематозу

2242 / 4299
У жінки 45-ти років, яка померла від хронічної алкогольної інтоксикації, на аутопсії печінка різко збільшена, тістоподібної консистенції, жовтуватого кольору. Мікроскопічно в цитоплазмі гепатоцитів при фарбуванні гематоксиліном та еозином виявляються різних розмірів оптично порожні вакуолі. Який вид дистрофії має місце?

Гіаліново-крапельна

Паренхіматозна жирова

Вуглеводна паренхіматозна

Мезенхімальна жирова

Гідропічна

2243 / 4299
У дитини 5-ти років під час огляду зіву виявлено: слизова зіву і мигдаликів гіперемована, мигдалики збільшені, вкриті щільними, білуватими плівками, які видаляються з утрудненням. На місці видалення залишається глибокий дефект тканини. М’які тканини шиї набряклі, регіонарні лімфатичні вузли шиї збільшені, болісні. Про яке захворювання можна думати?

Паротит

Дифтерія

Кір

Аденовірусна інфекція

Скарлатина

2244 / 4299
У хворого 23-х років після перенесеної ангіни розвинувся сечовий синдром (гематурія, протеїнурія, лейкоцитурія). В пункційному бiоптатi нирок виявлена картина інтракапілярного пролiферативного гломерулонефриту, а електронномікроскопічно виявлєні вєликі субепiтелiальнi депозити. Який патогенез цього захворювання?

Клітинно обумовлений цитоліз

Атопія, анафілаксія з утворенням IgE і фіксацією їх на опасистих клітинах

Цитотоксична, цитолітична дія антитіл

Гранулематоз

Імунокомплексний механізм

2245 / 4299
В сироватці крові новонародженого знайдено антитіла до вірусу кору. Про наявність якого імунітету це може свідчити?

Штучний пасивний

Природний активний

Спадковий

Природний пасивний

Штучний активний

2246 / 4299
Бактеріолог при дослідженні крові та слизу із носоглотки дотримувався певних заходів щодо збереження збудників у матеріалі. При бактеріоскопічному дослідженні встановлено наявність грамнегативних коків, які нагадують кавові зерна і розташовані парами, або тетрадами. Назвіть збудника, який був ізольований бактеріологом:

Moraxella lacunata

Staphilococcus aureus

Acinetobacter calcoaceticus

Neisseria meningitidis

Neisseria gonorrhoeae

2247 / 4299
Мисливцю, госпіталізованому на 5-й день хвороби, з метою підтвердження діагнозу туляремія для ранньої діагностики слід призначити:

РІФ

Алергічну пробу

РЗК

РА

РНГА

2248 / 4299
З виділень уретри хворого на уретрит в’ялого перебігу виділено чисту культуру кокоподібних мікроорганізмів. Виділений мікроорганізм в короткому строкатому ряду ферментує лише глюкозу до кислоти. Назвіть рід і вид виділеного мікроорганізму:

Staphylococcus aureus

Neisseria gonorrhoeae

Neisseria meningitides

Enterococcus faecalis

Streptococcus pyogenes

2249 / 4299
Лікар, мікроскопуючи мазок крові, що забарвлений за Романовським, виявив найпростіші у формі півмісяця, протоплазма яких вакуолізована і забарвлена у блакитний колір, а ядро - у червоний. Які найпростіші найбільш імовірно були у крові?

Лейшманії

Лямблії

Трипаносоми

Токсоплазми

Балантидії

2250 / 4299
Пацієнт госпіталізований з попереднім діагнозом: гепатит В. Для діагностики захворювання здійснено постановку серологічної реакції, яка базується на взаємодії антигену з антитілом, хімічно зв’язаним з пероксидазою або лужною фосфатазою. Яку назву має використана серологічна реакція?

Реакція імобілізації

Реакція імунофлюоресценції

Імуноферментний аналіз

Радіоімунологічний метод

Реакція зв’язування комплементу

2251 / 4299
Студент звернувся до лікаря з проханням допомогти йому перебороти страх перед стоматологічними маніпуляціями. Лікар порадив йому вжити такий препарат:

Діазепам

Аміназин

Дімедрол

Пірацетам

Дроперидол

2252 / 4299
Який комбінований препарат з групи сульфаніламідів має високий бактерицидний ефект?

Бактрим

Етамбутол

Фталазол

Сульфадімезин

Салазопіридазин

2253 / 4299
Хворому призначено комплексне лікування пародонтозу з використанням антибіотика лінкоміцина. Позитивного ефекту не було. Призначте інший антибіотик, здатний накопичуватись в кістках, зубах i сполучній тканині:

Оксацилін

Левоміцетин

Ністатин

Еритроміцин

Тетрациклін

2254 / 4299
У дитини вроджена розщілина нижньої губи (”заяча губа”) i переднього відділу піднебіння (”вовча паща”). Незрощенням яких відростків викликаний цей дефект?

Нижньощелепний, носовий

Піднебінний, нижньощелепний

Верхньощелепний, лобний

Носовий, лобний

Носовий, піднебінний

2255 / 4299
У хворого після автомобільної катастрофи АТ- 70/40 мм рт.ст. Хворий у непритомному стані. За добу виділяє близько 300 мл сечі. Який механізм порушення сечоутворення у даному випадку?

Збільшення канальцевої реабсорбції

Зменшення канальцевої секреції

Зменшення клубочкової фільтрації

Зменшення канальцевої реабсорбції

Збільшення клубочкової фільтрації

2256 / 4299
При огляді ротової порожнини у пацієнта 22-х років спостерігається нахил вперед верхніх та нижніх зубів, з перекриттям нижніх зубів верхніми. Якому прикусу характерне дане розміщення зубів?

Прямий прикус

Ортогнатія

Прогенія

Біпрогнатія

Закритий прикус

2257 / 4299
Небезпечно видавлювати гноячки на обличчі, особливо вище ротової щілини. Через анастомози з якими венами обличчя може поширитися інфекція у порожнину черепа?

Потилична вена

Очні вени

Передні вушні вени

Середня і глибока скроневі вени

Вени крилоподібного сплетення

2258 / 4299
До приймального відділення був доставлений хворий 56-ти років у непритомному стані з вираженим пригніченням дихання та серцевої діяльності, зниженням сухожильних рефлексів. Зі слів супроводжуючої його дружини, останнім часом він страждав на безсоння та приймав на ніч снодійні засоби. Який лікарський препарат необхідно ввести хворому для невідкладної допомоги?

Унітіол

Дипіроксим

Атропіну сульфат

Протаміну сульфат

Бемегрид

2259 / 4299
Хворий 44-х років скаржиться на часті нудоти, які нерідко завершуються блюванням. Порушення якої із функцій шлунка найімовірніше можна запідозрити у даного хворого?

Інкреторна

Екскреторна

Секреторна

Всмоктувальна

Евакуаторна

2260 / 4299
Хвора 48-ми років з цукровим діабетом, поступила до лікарні у важкому прекоматозному стані. При дослідженні КОС виявлений метаболічний ацидоз. Хворій призначена комплексна терапія, у тому числі інсулін внутрішньом’язево і розчин бікарбонату натрію внутрішньовенно. Який найбільш імовірний механізм зумовив виявлені зміни КОС?

Виведення лужних компонентів крові

Зниження виведення С02

Утворення недоокислених продуктів

Порушення використання 02 у клітинах

Порушення буферних систем крові

2261 / 4299
При рентгенологічному дослідженні кісток основи черепа виявлено збільшення порожнини турецького сідла, витончення передніх нахилених відростків, руйнування різних ділянок турецького сідла. Пухлина якої ендокринної залози може спричинити таке руйнування кістки?

Гіпофіз

Наднирник

Епіфіз

Щитоподібна залоза

Вилочкова залоза

2262 / 4299
Аналізуються діти в одній сім’ї. Один з батьків гомозиготний по домінантному гену полідактилії, а другий - здоровий (гомозиготний по рецесивному гену) в цьому випадку у дітей проявиться закон:

Одноманітності гібридів першого покоління

Незалежного спадкування

Чистоти гамет

Зчеплене успадкування

Розщеплення гібридів

2263 / 4299
У ході експерименту з внутрішньовенним уведенням різних фізіологічно активних речовин зареєстрована тахікардія. Яка з цих речовин викликала таку реакцію?

Тироксин

Ацетилхолін

Калікреїн

Інсулін

Брадикінін

2264 / 4299
Який препарат слід призначити для лікування хворій 54-х років з хронічною серцевою недостатністю, що супроводжується тахіаритмією та набряками?

Аміодарон

Дигоксин

Анаприлін

Метопролол

Спіронолактон

2265 / 4299
У хворої при лікуванні гіпертонічної хвороби на фоні прийому препарату виник сухий кашель. Для якого препарату характерна така побічна дія?

Еналаприл

Но-шпа

Резерпін

Дротаверин

Гідрохлортіазид

2266 / 4299
У процесі звапнування міжклітинної речовини кісткової тканини вздовж колагенових волокон відкладаються кристали гідроксиапатиту. Для реалізації цього процесу необхідна присутність у міжклітинній речовині лужної фосфатази. Яка клітина продукує цей фермент?

Остеобласт

Остеоцит

Остеокласт

Хондробласт

Хондроцит

2267 / 4299
У хворого з’явилися жовтушність шкіри, склер та слизових оболонок. У плазмі крові підвищений рівень загального білірубіну, в калі - стеркобіліну, в сечі - уробіліну. Який вид жовтяниці у хворого?

Хвороба Жільбера

Обтураційна

Холестатична

Паренхіматозна

Гемолітична

2268 / 4299
У хворого на хронічний пієлонефрит після неконтрольованого прийому сечогінних засобів виникли екстрасистоли та болі у ділянці серця. При дослідженні крові виявлено гіпокаліємію. Який з перерахованих препаратів слід призначити?

Гідрохлортіазид

Кальцію хлорид

Аспаркам

Калію перманганат

Ретинолу ацетат

2269 / 4299
У хворого 35-ти років після перенесеного гепатиту розвинулася печінкова недостатність. Порушення якої із функцій печінки при цьому запускає механізм утворення набряків?

Жовчоутворююча

Білковоутворююча

Глікогенутворююча

Антитоксична

Бар’єрна

2270 / 4299
Хворий 41-го року звернувся до офтальмолога для обстеження очного дна. В кон’юнктивальний мішок ока лікар закрапав розчин, який викликав розширення зіниці і паралiч акомодації. Розчин якого препарату закрапали в око?

Атропіну сульфат

Пілокарпіну гідрохлорид

Фурацилін

Прозерин

Галантаміну гідробромід

2271 / 4299
У хворої 63-х років стався напад миготливої аритмії. При обстеженні лікар виявив гіпотонію (АТ- 80/45 мм рт.ст.). Який препарат можна призначити разом з антиаритмічним для попередження колапсу?

Кофеїн-бензоат натрію

Теофілін

Мезатон

Адреналін

Етимізол

2272 / 4299
При тестуванні на гіперчутливість пацієнту під шкіру ввели алерген, після чого спостерігалось почервоніння, набряк, біль внаслідок дії гістаміну. В результаті якого перетворення амінокислоти гістидину утворюється цей біогенний амін?

Фосфорилювання

Ізомеризація

Декарбоксилювання

Метилювання

Дезамінування

2273 / 4299
У хворого 55-ти років, що знаходиться у кардіологічному відділенні з приводу серцевої недостатності, виявлені зміни показників гемодинаміки. Які з них найбільш інформативні для підтвердження вказаної патології?

Зменшення хвилинного об’єму крові

Підвищення венозного тиску

Підвищення частоти серцевих скорочень

Підвищення систолічного артеріального тиску

Підвищення діастолічного артеріального тиску

2274 / 4299
У вагітної жінки визначили групу крові. Реакція аглютинації еритроцитів відбулася зі стандартними сирова- тками груп 0 (I), В (III) і не відбулася зі стандартною сироваткою групи А (II). Досліджувана кров належить до такої групи:

АВ (IV)

0 (I)

А (II)

В (III)

2275 / 4299
До приймального відділення госпіталізовано чоловіка у непритомному стані. На зовнішні подразники не реагує, дихання періодичне по типу Чейн-Стокса, зіниці звужені, зіничний рефлекс відсутній. Було встановлено, що дані симптоми обумовлені застосуванням морфіну. Призначити антидотну терапію:

Налоксон

Апоморфіну гідрохлорид

Кальцію хлорид

Унітіол

Протаміну сульфат

2276 / 4299
У хворого із цирозом печінки відмічається стійка артеріальна гіпотензія. (АТ- 90/50 мм рт.ст.). Чим обумовлено зниження артеріального тиску при такій патології печінки?

Надмірна інактивація вазопресину

Посилення рефлекторного впливу із рецепторної зони дуги аорти

Зниження синтезу ангіотензиногену

Збільшення синтезу Na-уретичного гормону

Активація калікреїн-кінінової системи

2277 / 4299
Для лікування бронхіту дитині 8ми років необхідно призначити антибактеріальний засіб. Який з вказаних препаратів з групи фторхінолонів ПРОТИПОКАЗАНИЙ в цьому віці?

Ципрофлоксацин

Ампіокс

Ампіцилін

Сульфадиметоксин

Амоксицилін

2278 / 4299
З досліджуваного матеріалу хворого виділений вірус, що належить до РНК-вірусів, які мають онкогенні властивості. До якої родини належить виділений від хворого вірус?

Коронавіруси

Пікорнавіруси

Рабдовіруси

Ретровіруси

Ортоміксовіруси

2279 / 4299
Для покращання трофіки серцевого м’яза пацієнту призначено кардонат, до складу якого входить кокарбоксилаза (тіаміндифосфат) - коферментна форма вітаміну:

B12

B6

B5

B1

B2

2280 / 4299
У хворого при обстеженні виявлено глюкозурію, гіперглікемію. Скарги на сухість в роті, свербіння шкіри, часте сечовиділення, спрагу. Встановлений діагноз: цукровий діабет. Чим обумовлена поліурія у даного хворого?

Зменшення серцевого викиду

Збільшення фільтраційного тиску

Зменшення онкотичного тиску плазми

Збільшення осмотичного тиску сечі

Збільшення онкотичного тиску плазми

2281 / 4299
В пульмонологічному відділенні хворому обструктивним бронхітом зі спастичним компонентом було призначено бронхолітик, терапевтичний ефект якого обумовлюється стимуляцією β2 -адренорецепторів гладенької мускулатури бронхів. Визначте цей препарат:

Еуфілін

Сальбутамол

Преднізолон

Метацин

Кромолін-натрій

2282 / 4299
В присутності 2,4-динітрофенолу окиснення субстратів може тривати, але синтез молекул АТФ неможливий. Який механізм його дії?

Активація ферменту АТФ-ази

Переніс субстратів за межі мітохон- дрії

Роз’єднання окиснення і фосфори-лювання в мітохондріях

Стимуляція гідролізу утвореного АТФ

Інгібування ферменту цитохромо-ксидаза

2283 / 4299
До акушерського відділення госпіталізовано породілля. Для знеболювання пологів жінці ввели наркотичний анальгетик. Назвіть препарат:

Промедол

Анальгін

Ацетилсаліцилова кислота

Диклофенак натрію

Целекоксиб

2284 / 4299
До гінеколога звернулася жінка 28-ми років з приводу безпліддя. При обстеженні знайдено: недорозвинені яєчники і матка, нерегулярний менструальний цикл. При дослідженні статевого хроматину у більшості соматичних клітин виявлено 2 тільця Бара. Яка хромосомна хвороба найбільш імовірна у жінки?

Синдром Едвардса

Синдром Клайнфельтера

Синдром трипло-Х

Синдром Патау

Синдром Шерешевського-Тернера

2285 / 4299
При дегельмінтизації у хворого виявлені довгі фрагменти гельмінта, що має членисту будову. Зрілі членики прямокутної форми, 30х12 мм, матка закритого типу у вигляді стовбура, від якого відходять 17-35 бічних відгалужень. Визначте вид гельмінта:

Альвеокок

Ціп’як озброєний

Ехінокок

Ціп’як неозброєний

Ціп’як карликовий

2286 / 4299
При дослідженні трубчастого органу встановлено, що його середня оболонка складається із гіалінових суцільних кілець. Який це орган?

Трахея

Головний бронх

Сечовий міхур

Гортань

Стравохід

2287 / 4299
У 2-річної дівчинки була діагностована важка форма міастенії. Який з перерахованих засобiв показаний для лікуванні цього захворювання?

Галантаміну гідробромід

Цитітон

Допамін

Норадреналін

Скополамін

2288 / 4299
Порушення процесів розщеплення ліпідів у тонкому кишечнику зумовлено порушенням активності ліпази. Який з наведених чинників активує ліпазу?

Жовчні кислоти

Пепсин

Солі Na+

Ентерокіназа

Соляна кислота

2289 / 4299
На гістологічному препараті спинномозкового вузла видно, що псевдо-уніполярні нейроцити оточені шаром специфічних клітин нейроглії. Вкажіть, які з наведених клітин є мантійними гліоцитами?

Епендимоцити

Шванівські клітини

Астроцити

Олігодендроцити

Фібробласти

2290 / 4299
При обстеженні хворого 6-ти років виникла підозра на погіршення прохідності дихальних шляхів. Який із методів дослідження дозволяє вирогідно визначити дану патологію?

Спірометрія

Спірометаболографія

Пневмотахометрія

Пневмографія

Спірографія

2291 / 4299
У новонародженої дитини спостерігається зниження інтенсивності смоктання, часте блювання, гіпотонія. У сечі та крові значно підвищена концентрація цитруліну. Який метаболічний процес порушений?

цтк

Орнітиновий цикл

Глюконеогенез

Гліколіз

Цикл Корі

2292 / 4299
У потерпілого 24-х років з травмою голови у скроневій ділянці діагностовано епідуральну гематому. Яка з артерій найімовірніше пошкоджена?

Поверхнева скронева

Середня оболонкова

Передня оболонкова

Середня мозкова

Задня вушна

2293 / 4299
У хворого з облітеруючим атеросклерозом проведено десимпатизацію стегнової артерії в ділянці стегнового трикутника. Який вид артеріальної гіперемії виник внаслідок операції?

Нейротонічна

Реактивна

Робоча

Нейропаралітична

Метаболічна

2294 / 4299
Чоловік скаржиться на порушення чутливості шкіри вушної раковини та зовнішнього слухового ходу. Пошкодження якого нерва викликало такий стан у хворого?

N. facialis

N. auricularis magnus

N. auricularis anteriores

N. auriculotemporalis

2295 / 4299
У чоловіка 55-ти років діагностовано вікову далекозорість у зв’язку з послабленням акомодаційних властивостей кришталика та гладеньких м’язів судинної оболонки очного яблука. Тонус якого м’яза з віком послаблено?

M. sphincter pupillae

M. obliquus superior

M. ciliaris

M. dilatator pupillae

M. obliquus inferior

2296 / 4299
При лікуванні сіалоаденітів (запалення слинних залоз) використовують препарати вітамінів. Який із наведених вітамінів відіграє важливу роль в антиоксидантному захисті?

Шридоксин

Тіамін

Токоферол

Пантотенова кислота

Рибофлавін

2297 / 4299
Одним із мєтодів зняття гострого болю при невралгії трійчастого нерва є створення депо анестетика в місці виходу гілок трійчастого нерва. В якій ділянці потрібно ввести анестетик для знечулення першої гілки трійчастого нерва?

Надочноямковий край

Ділянка лобного відростка верхньої щелепи

Підочноямковий край

Ділянка надперенісся

Орбітальний край виличної кістки

2298 / 4299
У 4-річного хлопчика з’явились горбисті, щільні, підшкірно розташовані утворення в ділянці кутів та вздовж гілок двох нижніх щелеп, які зумовили деформацію обличчя. При гістологічному дослідженні біоптату виявлено: в міжбалковому просторі розростання васкуляризованої сполучної тканини, довкола судин маси ацидофільного матеріалу, багатоядерні гігантські клітини, окремі примітивні кісткові балочки. Діагностуйте захворювання:

Периферійна гігантоклітинна гранульома

Херувізм

Еозинофільна гранульома

Хронічний фіброзний періостит

Фіброзна дисплазія

2299 / 4299
При дослідженні бронхобіоптату встановлено атрофію слизової оболонки, кістозне перетворення залоз, осередкову метаплазію покривного призматичного епітелію в багатошаровий плоский, збільшення числа келихоподібних клітин, місцями у стінці бронха та особливо у слизовій оболонці різко виражена клітинна запальна інфільтрація і розростання грануляційної тканини, яка вибухає у просвіт бронха у вигляді поліпа. Який найбільш імовірний діагноз?

Гострий бронхіт

Гостра пневмонія

Хронічний бронхіт

Бронхопневмонія

Проміжна пневмонія

2300 / 4299
У чоловіка 30-ти років було видалено зуб. При гістологічному дослідженні періодонта на верхівці зуба виявлено: ріст щільної волокнистої сполучної тканини з інфільтрацією її лімфоцитами, гістіоцитами та з великою кількістю нейтрофілів. Який різновид апікального періодонтиту у хворого?

Хронічний гранулюючий

Хронічний гранулематозний

Хронічний фіброзний з загостренням

Хронічний фіброзний без загострення

Гострий гнійний

2301 / 4299
На видаленому у хворого зубі лікар побачив три корені. Який це зуб?

Великий кутній верхньої щелепи

Великий кутній нижньої щелепи

Малий кутній нижньої щелепи

Ікло верхньої щелепи

Малий кутній верхньої щелепи

2302 / 4299
При остеолатеризмі зменшується міцність колагену, що зумовлена помітним зменшенням утворення поперечних зшивок у колагенових фібрилах. Причиною цього явища є зниження активності такого ферменту:

Колагеназа

Пролілгідроксилаза

Лізилоксидаза

Лізилгідроксилаза

Моноаміноксидаза

2303 / 4299
На м’якому піднебінні та слизовій щік 27-річної жінки лікар-стоматолог помітив висип у вигляді папул, що нагадує ураження при вторинному сифілісі. Який метод діагностики найдоцільніше використати для перевірки попереднього діагнозу?

Бактеріоскопічний

Алергічний

Серологічний

Біологічний

Бактеріологічний

2304 / 4299
Хвороба Хартнепа зумовлена точковою мутацією лише одного гена, наслідком чого є порушення всмоктування амінокислоти триптофану в кишечнику та реабсорбції її в ниркових канальцях. Це призводить до одночасних розладів у травній i сечовидільній системах. Яке генетичне явище спостерігається в цьому випадку?

Кодомінування

Полімєрія

Плейотропія

Комплементарна взаємодія

Неповне домінування

2305 / 4299
При остеохондрозі хребта перетискається судина, яка проходить через поперечні отвори шийних хребців. Яка це артерія?

Хребтова

Внутрішня сонна

Зовнішня сонна

Задня мозкова

Потилична

2306 / 4299
За професійними показаннями проведена вакцинація лікарів-стоматологів. Вакцина має захищати їх від вірусної інфекції, збудник якої може бути присутній у крові стоматологічних хворих, які перенесли інфекцію або є хронічними носіями. Яка вакцина була використана?

Антирабічна вакцина

Жива корова вакцина

Субодинична грипозна вакцина

Генно-інженерний НВs-антиген

Інактивована вакцина проти гепатиту А

2307 / 4299
У хворого 19-ти років після видалення верхнього лівого присереднього різця виникла кровотеча з комірки. Яка артерія зумовила кровотечу?

Нижня коміркова

Передня верхня коміркова

Низхідна піднебінна

Задня верхня коміркова

Висхідна піднебінна

2308 / 4299
У хворого у пародонтальних кишенях виявлена патогенна мікрофлора. Лікар-стоматолог призначив хіміотерапевтичний засіб - похідне імідазолу. Який це препарат?

Метронідазол

Ацикловір

Фуразолідон

Флюконазол

Метилурацил

2309 / 4299
На мікропрепараті, виготовленому зі спинномозкового вузла, спостерігаємо нейроцити, які мають тіло округлої форми і один відросток, який далі розгалужується на два. До якого різновиду нейронів за морфологічною класифікацією належать ці клітини?

Псевдоуніполярні

Нейробласти

Біполярні

Мультиполярні

Уніполярні

2310 / 4299
При мікроскопічному дослідженні фекалій від хворого з явищами профузного проносу, багаторазового блювання та наростаючої інтоксикації, було виявлено грамнегативні палички, що нагадували кому і розташовувались групами у вигляді табунців риб. Культура збудника була виділена через середовище накопичення - 1% пептонну воду, де вона утворювала ніжну плівку. Збудник якого захворювання був виділений з фекалій хворого?

Кишковий єрсиніоз

Псевдотуберкульоз

Холера

Сальмонельоз

Шигельоз

2311 / 4299
У молодої жінки діагностовано позаматкову вагітність. В якому з перерахованих органів відбувається запліднення?

Ампула маткової труби

Порожнина матки

Порожнина піхви

Перешийок маткової труби

Порожнина малого тазу

2312 / 4299
Хворому з важким перебігом ревматоїдного артриту для постійного прийому був призначений преднізолон. Який режим прийому є оптимальним з урахуванням хронофармакологічних особливостей дії преднізолону та фізіологічних коливань секреції кортикостероїдів в організмі?

Вся доза ввєчєрі

2/3 добової дози вранці, 1/3 дози вдень

1/3 дози тричі на день

Вся доза вранці

2313 / 4299
Метилмалонова ацидемiя характеризується надлишком метилмалонової кислоти в крові та сєчі, метаболічним ацидозом, затримкою розвитку дітей, ураженням нервової системи. Клінічні прояви спадкової метилмалонової ацидемії у деяких пацієнтів можна послабити вживанням підвищених кількостей одного з вітамінів. Якого саме?

B2

B1

В12

B6

В3

2314 / 4299
При дослідженні каріотипу п’ятирічного хлопчика виявлено 46 хромосом. Одна з хромосом 15-ої пари довша від звичайної, тому що до неї приєднана ділянка хромосоми з 21-ої пари. Вкажіть вид мутації, що має місце в цього хлопчика:

Поліплоїдія

Делеція

Транслокація

Інверсія

Дуплікація

2315 / 4299
Хворому зі скаргами на періодичні напади тахікардії, був призначений лікарський засіб із групи β-адреноблокаторів. Тахікардія зникла, але з’явилися напади задухи. Який засіб був призначений хворому?

Фентоламін

Резерпін

Октадин

Фенотерол

Анаприлін

2316 / 4299
Сульфаніламіди використовують у клінічній практиці як протимікробні засоби. Генетичний дефект якого ферменту пентозофосфатного обміну в еритроцитах під час лікування такими препаратами може призвести до розвитку гемолітичної анемії?

Гексокіназа

Трансальдолаза

Глюкозо-6-фосфатдегідрогеназа

Піруваткіназа

Транскетолаза

2317 / 4299
Під час футбольного матчу між вболівальниками різних команд виникла сутичка. На фоні негативних емоцій в одного учасника сутички були розширені зіниці й підвищене серцебиття. Активація якої системи регуляції функцій організму забезпечує такі вегетативні зміни при негативних емоціях?

Парасимпатична нервова

Метасимпатична нервова

Соматична нервова

Гіпоталамо-гіпофізарно-тиреоїдна

Симпато-адреналова

2318 / 4299
Під час розтину тіла виявлено: легені щільні, коричневого кольору за рахунок відкладання ендогенного пігменту. Відомо, що за життя у хворого мав місце хронічний венозний застій в малому колі кровообігу. Який патологічний процес викликав таку картину?

Жовтяниця

Гемосидероз

Кальциноз

Порфірія

Меланоз

2319 / 4299
У хворого на променеву хворобу з’явилися ознаки геморагічного синдрому. Які зміни в крові є вирішальними в патогенезі цього синдрому?

Нейтропенія

Лімфопенія

Тромбоцитопенія

Еритроцитоз

Еозинопенія

2320 / 4299
У постраждалого травма голови, яка супроводжується артеріальною кровотечею в ділянці тім’яної кістки. Яка артерія уражена?

A. temporalis superficialis

A. facialis

A. maxillaris

A. auricularis posterior

A. thyroidea superior

2321 / 4299
Вентилятор полегшує перебування в приміщенні з високою температурою, оскільки його робота суттєво збільшує віддачу організмом тепла таким шляхом:

Конвекція

Проведення

Радiацiя

Випаровування рідини

2322 / 4299
У людей, які проживають на великих висотах над рівнем моря, відмічається збільшення кількості еритроцитів, що спричиняється:

Посиленим утворенням нирками еритропоетину

Впливом гіпоксії на розпад еритроцитів

Посиленим виділенням нирками реніну

Підвищеним споживанням тканинами кисню

Гальмівним впливом гіпоксії на кістковий мозок

2323 / 4299
При лабораторному дослідженні крові пацієнта виявлено, що вміст білків у плазмі становить 40 г/л. Як це впливає на транскапілярний обмін води в мікроциркуляторному руслі?

Збільшуються фільтрація і реабсорбція

Зменшується фільтрація, збільшується реабсорбція

Збільшується фільтрація, зменшується реабсорбція

Зменшуються фільтрація і реабсорбція

Обмін не змінюється

2324 / 4299
У пухлинних клітинах переважає анаеробний гліколіз. Виявлення якої речовини у шлунковому соці може підтвердити діагноз ”рак шлунка” у пацієнта?

Молочна кислота

Глікохолева кислота

Білірубін

Гем

Таурохолева кислота

2325 / 4299
У бактеріологічній лабораторії проводилася експертиза шкур тварин за допомогою імунної реакції преципітації за Асколі. Позитивний результат цієї реакції свідчить про наявність:

Збудника бруцельозу

Збудника чуми

Поверхневого антигену ієрсиній

Антигенів збудника сибірки

Токсину анаеробної інфекції

2326 / 4299
Для профілактики і лікування тромбозу застосовуються засоби, що знижують згортання крові (антикоагулянти). Вкажіть антикоагулянт, при передозуванні якого як антагоніст застосовують протаміну сульфат:

Фенілін

Неодикумарин

Натрію гідроцитрат

Гепарін

Синкумар

2327 / 4299
Після вживання м’ясної консерви у школяра з’явилися неврологічні симптоми. Був поставлений діагноз: ботулізм. Які екстрені методи лікування необхідно використати?

Призначення проносних засобів

Введення антибіотиків

Введення антиботулінічної вакцини

Введення сульфаніламідних препаратів

Введення антиботулінічної сироватки

2328 / 4299
У хворого запальний процес у крилопіднебінній ямці. Інфекція поширилась в носову порожнину. Через яке анатомічне утворення розповсюдилась інфекція?

Canalis palatinus minor

Canalis palatinus major

Foramen sphenopalatinum

Canalis ptherygoideus

Foramen rotundum

2329 / 4299
У Південній та Центральній Америці зустрічається один з видів трипаносом, що є збудником хвороби Чагаса. Який з перелічених організмів є специфічним переносником збудника цього захворювання?

Комар

Триатомовий клоп

Москіт

Тарган

Муха цеце

2330 / 4299
Гістологічне дослідження тканини виявило, що в ній відсутні кровоносні судини, а клітини щільно прилягають одна до одної, утворюючи пласти. Яка це тканина?

М’язова

Епітеліальна

Нервова

Кїсткова

Хрящова

2331 / 4299
Лікарем швидкої допомоги у пацієнта було діагностовано анафілактичний шок, що супроводжувався бронхоспазмом. Виділення якої біологічно активної речовини тканинними базофілами зумовлює таку клінічну симптоматику?

Простагландини

Гістамін

Брадикінін

Гепарин

Лейкотрієни

2332 / 4299
16-тирічна дівчина має зріст 139 см, крилоподібну шию, нерозвинені грудні залози, первинну аменорею. Найбільш імовірно, вона має такий каріотип:

46, XY

46, ХХ

45, Х0

46, ХХ/46, XY

47, ХХХ

2333 / 4299
До пологового відділення надійшла пацієнтка з слабкістю пологової діяльності. Який гормон необхідно ввести пацієнтці для стимуляції скорочення гладеньких м’язів матки та лактації?

Кортизол

Синестрол

Альдостерон

Вазопрессин

Окситоцин

2334 / 4299
У травмованої жінки виявлено розрив лобкового симфізу. Який тип з’єднання кісток порушений?

Hemyartrosis

Synostosis

Syndesmosis

Synartrosis

Synchondrosis

2335 / 4299
У постраждалого перелом тіла нижньої щелепи. Кровотеча з рани. Яка артерія ушкоджена?

A. lingualis

A. carotis externa

A. alveolaris inferior

A. facialis

A. maxillaris

2336 / 4299
На розтині тіла чоловіка 47-ми років, що помер від легеневої кровотечі, у 2-му сегменті правої легені виявлена порожнина округлої форми з нерівними краями розміром 5,5 см; внутрішня поверхня її вкрита щільнуватими жовтуватими масами, що переходять у тканину легені. При гістологічному дослідженні - внутрішній шар складається з розплавлених казеозних мас, некротизованої тканини легені. Який процес розвинувся в легені?

Хронічний абсцес

Гострий кавернозний туберкульоз

Хронічний кавернозний туберкульоз

Гострий абсцес

Рак легені, що розпадається

2337 / 4299
Для лікування карієсу 36 зуба необхідно здійснити провідникову анестезію. Який нерв при цьому треба знеболити?

Нижній альвеолярний

Підборідний

Підочноямковий

Верхньощелепний

Suprafoveal

2338 / 4299
В економічно розвинених країнах поширеним захворюванням є карієс зубів. Це захворювання уражає більш 95% населення. Що відіграє головну роль у демінералізації твердих тканин зуба при карієсі?

Недостатність вітаміну C

Органічні кислоти

Розлад регуляції метаболізму

Неповноцінне харчування

Екстремальні впливи на організм

2339 / 4299
У гістологічному препараті яєчка в прошарках сполучної тканини між звивистими сім’яними канальцями виявлені відносно великі ацидофільні клітини, в цитоплазмі яких містяться глікопротеїнові включення та зерна глікогену, добре розвинута гладка ендоплазматична сітка та мітохондрії. Які це клітини?

Міоїдні

Сперматогонії

Підтримуючі

Інтерстиційні

Фібробласти

2340 / 4299
У собаки вироблений умовний слиновидільний рефлекс на вмикання світла. Вмикання дзвоника під час виконання даного рефлексу призведе до розвитку наступного виду гальмування:

Диференціювання

Згасання

Зовнішнє

Умовне гальмування

Позамежне

2341 / 4299
У пацієнта внаслідок запального процесу виникло надмірне збудження вушно-скроневого нерва. При цьому привушною слинною залозою буде виділятися:

Мала кількість в’язкої слини

Велика кількість рідкої слини

Виділення слини припиниться

Мала кількість рідкої слини

Велика кількість в’язкої слини

2342 / 4299
У пацієнта діагностований первинний туберкульоз легень. Призначення якого протитуберкульозного засобу з групи антибіотиків, що порушують синтез РНК мікобактерій, є бажаним?

Етамбутол

Рифампіцин

Канаміцину сульфат

ПАСК

Піразинамід

2343 / 4299
Хвора 18-ти років впродовж декількох тижнів скаржиться на біль та кровоточивість в області 35 зуба при вживанні твердої їжі. Об’єктивно: на жувальній поверхні цього зуба - велика каріозна порожнина, яка заповнена м’язоподібною тканиною. При зондуванні виникає кровотеча і біль в області з’єднання каріозної порожнини з пульпою. Який діагноз найбільш імовірний?

Хронічний фіброзний пульпіт

Хронічний гангренозний пульпіт

Хронічний глибокий карієс

Гострий гнійний пульпіт

Хронічний гіпертрофічний пульпіт

2344 / 4299
Хвора 20-ти років звернулась до лікаря зі скаргами на кровоточивість та біль у яснах, що з’явилися через 2 дні після прийому сульфадимезину. Об’єктивно: ясенні сосочки та ясенний край гіперемовані, набряклі, кровоточать при незначному подразненні. Яка патологія розвинулась в яснах?

Гострий виразковий гінгівіт

Хронічний катаральний гінгівіт

Епулід

Гострий катаральний гінгівіт

Геморагічний гінгівіт

2345 / 4299
До стоматолога звернувся хворий 47-ми років з артритом скронево-нижньощелепного суглоба. Лікар призначив протизапальний препарат - селективний інгібітор ферменту циклооксигенази-2. Який препарат призначили хворому?

Індометацин

Целекоксиб

Бутадіон

Кислота мефенамова

Ібупрофен

2346 / 4299
При вживані гарячої та холодної їжі для підтримання сталої температури в ротовій порожнині виникає однотипна судинна реакція її слизової. У чому вона полягає?

Звуження судин

Звуження артерій і розширення вен

Тонус судин не змінюється

Розширення судин

2347 / 4299
Пацієнт 39-ти років із захворюванням першого верхнього різця зліва скаржиться на сильні болі шкіри в ділянці надбрівної дуги з того ж боку. Які рефлекси спричиняють вказані реакції?

Пропріоцептивні

Вісцеро-дермальні

Вісцеро-соматичні

Сомато-вісцеральні

Вісцеро-вісцеральні

2348 / 4299
Збудники ряду інфекційних захворювань можуть передаватися через повітря. Наявність яких мікроорганізмів у повітрі закритих приміщень є свідченням прямої епідеміологічної небезпеки?

Бацили і гриби

Спори грибів

Стафілококи і нейсерії

Бацили і ентеробактерії

Гемолітичні стафілококи і стрептококи

2349 / 4299
Хірург прооперував пацієнта з приводу пахвинної грижі. Нижньою стінкою пахвинного каналу є пахвинна зв’язка. Апоневрозом якого м’язу живота вона утворена?

Зовнішній косий

Поперечний

Прямий

Внутрішній косий

2350 / 4299
У хворого 12-ти років діагностована тріада Гетчинсона: зуби діжкоподібної форми, паренхіматозний кератит та глухота. Для якої хвороби характерні виявлені зміни?

Туберкульоз

Токсоплазмоз

Опісторхоз

Сифіліс

Проказа

2351 / 4299
Назвіть вітаміни, при дефіциті яких в організмі людини виникає однаковий симптомокомплекс, зокрема - кровоточивість ясен:

В3 , B12

C, P

В1 , В2

PP, В6

H, D

2352 / 4299
При електронній мікроскопії у цитоплазмі клітини, поблизу ядра, виявлена мембранна органела, яка складається з 5-10 пласких цистерн, з розширеними периферичними ділянками, від яких від’єднуються маленькі пухирці - лізосоми. Назвіть цю орга-нелу:

Мітохондрія

Рибосома

Цитоскелет

Комплекс Гольджі

Клітинний центр

2353 / 4299
При мікроскопічному вивченні біоптата нирки в її кірковій речовині виявлені канальці близько 60 мкм в діаметрі, їх стінка утворена високим кубічним епітелієм з вираженою апікальною облямівкою і базальною складчастістю. Назвіть дані структурні утворення:

Проксимальні канальці

Капсула ниркового тільця

Збірна трубочка

Петля Генле

Дистальні канальці

2354 / 4299
При лікуванні хворого на спадкову форму імунодефіциту було застосовано метод генотерапії: ген ферменту був внесений у клітини пацієнта за допомогою ретровірусу. Яка властивість генетичного коду дозволяє використовувати ретровіруси у якості векторів функціональних генів?

Колінеарність

Специфічність

Безперервність

Універсальність

Надмірність

2355 / 4299
У хворого на стоматологічному прийомі виник напад пароксизмальної тахікардії, у зв’язку з чим йому ввели лідокаїн. З яким механізмом дії лідокаїну пов’язаний його протиаритмічний ефект?

Мембранно-іонний

Потенціювання

Кумуляція

Сумація

Антагонізм

2356 / 4299
Провідну роль в процесі кальцифікації тканин зуба відіграє білок остеокальцин, який має високу здатність зв’язувати іони кальцію, завдяки наявності в поліпептидному ланцюзі залишків модифікованої амінокислоти:

γ-карбоксиглутамінова

Аланін

γ-аміномасляна

Карбоксиаспарагінова

δ-амінопропіонова

2357 / 4299
Чоловік, у якого навіть у стані спокою часто бувають епізоди підвищення частоти скорочень серця до 180/хв., помітив, що після масажу шиї в ділянці пульсації сонних артерій частота скорочень серця та АТ зменшуються. Який рефлекс лежить в основі такої реакції хворого?

Спряжений пресорний

Умовний парасимпатичний

Безумовний симпатичний

Власний депресорний

Власний пресорний

2358 / 4299
У біоптаті слизової оболонки хворого на бронхіальну астму виявлено значну кількість клітин з численними метахроматичними гранулами. Назвіть цю клітину:

Плазмоцит

Макрофаг

Фібробласт

Тканинний базофіл

Ретикулоцит

2359 / 4299
У хворого 69-ти років на шкірі в ділянці нижньої повіки з’явилося невелике бляшкоподібне утворення, з послідуючим виразкуванням, яке було оперативно видалене. При мікроскопічному дослідженні утворення: в дермі шкіри комплекси з атипових епітеліальних клітин, на периферії утворень клітини розташовані перпендикулярно до базальної мембрани. Клітини темні, призматичної полігональної форми; ядра гіперхромні з частими мітозами. Іноді зустрічаються утворення, подібні до волосяного фолікула. Яка гістологічна форма рака у хворого?

Базально-клітинний

Недиференційований

Плоскоклітинний без ороговіння

Плоскоклітинний з ороговінням

Аденокарцинома

2360 / 4299
Вивчається мітотичний поділ клітин епітелію ротової порожнини. Встановлено, що в клітині диплоїдний набір хромосом. Кожна хромосома складається з двох максимально спіралізованих хроматид. Хромосоми розташовані у площині екватору клітини. Ця картина характерна для такої стадії мітозу:

Прометафаза

Метафаза

Анафаза

Телофаза

Профаза

2361 / 4299
У гістологічному препараті представлений орган ендокринної системи, структурно- функціональними одиницями якого є фолікули. Їх стінка утворена одношаровим кубічним епітелієм. Який орган представлений у препараті?

Наднирник

Щитоподібна залоза

Епіфіз

Білящитоподібна залоза

Гіпофіз

2362 / 4299
Хвора 65-ти років страждає на хронічний закреп внаслідок гіпотонії товстого кишечника. Який засіб лікування закрепу слід обрати за цих обставин?

Прозерин

Касторова олія

Бісакодил

Магнію сульфат

Метоклопрамід

2363 / 4299
Чоловік 25-ти років звернувся до лікаря зі скаргами на біль в нижній щелепі справа, припухлість, високу температуру, озноб. При обстеженні макроскопічно відмічається відшарування окістя з накопиченням запального ексудату між ним і кісткою, з пе- рифокальним набряком м’яких тканин та частково розплавленим окістям. Що розвинулося у хворого?

Катаральний гінгівіт

Гнійний періостит

Гангренозний пульпіт

Гранулюючий періодонтит

Локальний пародонтит

2364 / 4299
У альпініста, що піднявся на висоту 5200 м, розвинувся газовий алкалоз. Що є причиною розвитку алкалозу?

Введення кислот

Гіпервентиляція легенів

Гіповентиляція легенів

Введення лугів

Підвищення температури навколишнього середовища

2365 / 4299
У дорослої людини у стані спокою частота серцевих скорочень дорівнює 40/хв. Водієм ритму серця у людини є:

Атріовентрикулярний вузол

Синоатріальний вузол

Волокна Пуркін’є

Ніжки пучка Гіса

Пучок Гіса

2366 / 4299
У хворого хронічний нежить. Набряк слизової оболонки носової порожнини призводить до порушення функції рецепторів нюхового нерва, які розташовані в нюховій ділянці носової порожнини. Через яке утворення волокна цього нерва потрапляють в передню черепну ямку?

Foramen sphenopalatinum

Foramen incisivum

Foramen ethmoidale posterior

Lamina cribrosa os ethmoidale

Foramen ethmoidale anterior

2367 / 4299
Дівчина 19-ти років, хвора на цукровий діабет, чекає на донорську нирку. Яке ускладнення діабету є причиною хронічної ниркової недостатності?

Макроангіопатія

Мікроангіопатія

Нейропатія

Атеросклероз

Ретинопатія

2368 / 4299
Лікар-стоматолог під час огляду ротової порожнини на вестибулярній поверхні ясен в області 2-го премоляра виявив утворення у вигляді вузлика 0,8 см в діаметрі, буро- червоного кольору, м’якої консистенції на широкій ніжці. Гістологічно утворення багате судинами синусоїдного типу з великою кількістю видовжених одноядерних та гігантських багатоядерних клітин, місцями зустрічаються гранули гемосидерину. Результати макро- мікроскопічного дослідження характерні для такого захворювання:

Епуліс судинний

Епуліс фіброзний

Папілома

Афтозний стоматит

Епуліс гігантоклітинний

2369 / 4299
У жінки встановлено діагноз - рак шийки матки. З яким вірусом може бути асоційована ця патологія?

Цитомегаловірус

Аренавірус

Вірус простого герпесу тип 2

Varicella-Zoster вірус

Папілома вірус

2370 / 4299
В яких лімфатичних вузлах може розвиватися запальна реакція при гінгівіті (запаленні слизової оболонки ясен)?

Nodi supraclaviculares

Nodi submandibulares

Nodi cervicales laterales

Nodi parotidei superficiales

Nodi parotidei profundi

2371 / 4299
Студент дістав завдання розрахувати альвеолярну вентиляцію легень. Для цього йому необхідні знати наступні показники зовнішнього дихання:

Хвилинний об’єм дихання, дихальний об’єм, частота дихання

Об’ єм мертвого простору, життєва ємність легень, дихальний об’єм

Дихальний об’єм, резервний об’єм вдиху, резервний об’єм видиху

Життєва ємність легень, резервний об’єм вдиху, частота дихання

Дихальний об’єм, об’єм мертвого простору, частота дихання

2372 / 4299
У дитини 2-х років спостерігається відставання в розумовому розвитку, непереносимість білкової їжі, важка гіперамоніємія на тлі зниженого вмісту сечовини в плазмі крові, що пов’язано з вродженим дефіцитом такого ферменту мітохондрій:

Цитратсинтаза

Карбомоїлфосфатсинтетаза

Сукцинатдегідрогеназа

Моноамінооксидаза

Малатдегідрогеназа

2373 / 4299
Хвора 25-ти років звернулася до лікаря-алерголога зі скаргами на нежить, сльозотечу, чхання, утруднене дихання та сухий кашель після кожного прибирання у квартирі. Якій стадії патогенезу алергічних реакцій відповідає описана клінічна картина?

Сенсибілізація

Імунологічна

Патофізіологічна

Патохімічна

2374 / 4299
Хворому після дорожньо-транспортної пригоди було ампутовано нижню кінцівку. Протягом тривалого часу хворий відчував ампутовану кінцівку і сильний нестерпний біль в ній. Який вид болю виник у хворого?

Рефлекторний

Вісцеральний

Відбитий

Каузалгічний

Фантомний

2375 / 4299
При біохімічному аналізі еритроцитів немовляти встановлено виражену недостатність глутатіонпероксидази і низький рівень відновленого глутатіону. Яка анемія може розвинутись у цієї дитини?

Перніціозна

Залізодефіцитна

Мегалобластна

Серпоподібно-клітинна

Гемолітична

2376 / 4299
У молодої жінки, яка увійшла до приміщення з високою концентрацією тютюнового диму, раптово виникли рефлекторні кашель та спазм бронхів. Подразнення яких рецепторів викликало дані захисні рефлекси?

Механорецептори легень

Центральні хеморецептори

Юкстамедулярні рецептори

Ірритантні рецептори

Рецептори плеври

2377 / 4299
На розтині тіла чоловіка, що страждає на черевний тиф, у здухвинній кишці виявлені дефекти, що розташовуються по довжині кишки, краї їх рівні, дно утворене м’язовим шаром. Яка з стадій черевного тифу діагностована?

Загоєння

Мозкоподібного набухання

Некрозу

Чистих виразок

Утворення виразок

2378 / 4299
Під час іспиту студент побачив у гістологічному препараті орган, в кірковій речовині якого розташовані скупчення лімфатичних вузликів. У мозковій речовині знаходились тяжі клітин, що відходять від вузликів. Строму органа утворюють сполучна та ретикулярна тканини. Який орган досліджував студент?

Червоний кістковий мозок

Лімфатичний вузол

Мигдалик

Тимус

Селезінка

2379 / 4299
Хворого госпіталізовано з підозрою на пухлину верхнього відділу стравоходу. Під час рентгенологічного обстеження виявлено пухлинний процес на межі глотки та стравоходу. На рівні якого шийного хребця розташована пухлина?

1

6

4

2

3

2380 / 4299
Обмеження споживання води призвело до зневоднення організму. Який механізм активується за цих умов для збереження води в організмі?

Збільшення секреції соматостатину

Збільшення секреції альдостерону

Збільшення секреції вазопресину

Зменшення секреції вазопресину

Зменшення секреції альдостерону

2381 / 4299
На штучному вигодовуванні знаходиться дитина 9-ти місяців. Для вигодовування використовують суміші, які незбалансовані за вмістом вітаміну B6 . У дитини спостерігаються судоми, причиною яких може бути порушення утворення такої речовини:

Дофамін

β-аланін

ГАМК

Серотонін

Гістамін

2382 / 4299
Під дією медіатора на постсинаптичну мембрану нервової клітини розвинулася гіперполяризація. Збільшення проникності мембрани для якого іона може викликати такі зміни?

Кальцій

Калій

Магній

Натрій

Натрій і калій

2383 / 4299
Лікар дав жінці рекомендацію продовжити низькокалорійну дієту. Вона вирішила отримувати ту ж саму кількість калорій, але замінити вуглеводи на жири. Рівень якої з наведених ліпопротеїнових фракцій буде підвищеним внаслідок цієї дієти?

ЛПНЩ

Хіломікрони

ЛПВЩ

ЛППЩ

ЛПДНЩ

2384 / 4299
У пацієнта 28-ми років, хворого на пневмонію, розвинувся набряк легень. Об’єктивно спостерігається швидке зростання глибини та частоти дихання з переважанням фази вдиху над фазою видиху, загальне збудження, розширення зіниць, тахікардія, підвищення артеріального тиску, судоми. Який стан розвився у хворого?

Перший період асфіксії

Апное

Третій період асфіксії

Гіпопное

Другий період асфіксії

2385 / 4299
В хромосомному наборі жінки виявлено хромосому, в якій плечі p і q мають однакову довжину. До якого морфологічного типу належить ця хромосома?

Телоцентричний

Субметацентричний

Акроцентричний

Субакроцентричний

Метацентричний

2386 / 4299
У хворого 50-ти років раптово виникло сильне серцебиття, біль у серці, різка слабкість, підвищення артеріального тиску, спостерігається дефіцит пульсу. На ЕКГ виявлено відсутність зубця Р і різні інтервали R-R. Яке порушення серцевого ритму розвинулося у хворого?

Екстрасистолія

Пароксизмальна тахікардія

Миготлива аритмія

Дихальна аритмія

Поперечна блокада серця

2387 / 4299
На місці механічного пошкодження шкіри завдяки процесам регенерації формується пухка волокниста сполучна тканина. Які клітини продукують білки (колаген, еластин) та компоненти міжклітинної речовини?

Макрофаги

Адипоцити

Тканинні базофіли

Фібробласти

Плазмоцити

2388 / 4299
При дослідженні слини людини необхідно оцінити її гідролітичні властивості. Що з наведеного потрібно при цьому використати як субстрат?

Крохмаль

Клітковина

Жири

Білки

Амінокислоти

2389 / 4299
На електронограмі кінцевих секреторних відділів слинної залози визначаються клітини пірамідної форми з центрально розміщеним ядром та білковими секреторними гранулами. Визначте, що це за клітини:

Адипоцити

Ендотеліоцити

Міоепітеліальні клітини

Мукоцити

Сероцити

2390 / 4299
Вітамін D резистентний рахіт визначається домінантним геном, який локалізований в Х- хромосомі. Який генотип має здоровий хлопчик у родині, де мати здорова, а у батька діагностовано дану форму рахіту?

аа

Аа

AA

ХаУ

ХАУ

2391 / 4299
У хворого після видалення зуба виник напад стенокардії. Який антиангінальний засіб дасть швидкий ефект при сублінгвальному застосуванні?

Клофелін

Триметазідин

Панангін

Нітроглицерин

Ізадрин

2392 / 4299
У постраждалого глибока різана рана у ділянці зовнішньої поверхні кута нижньої щелепи. Який м’яз при цьому буде пошкоджено?

M. masseter

M. orbicularis oris

M. buccinator

M. zygomaticus

M. depressor anguli oris

2393 / 4299
На розтині тіла чоловіка 72-х років, який поступив до неврологічного відділення з геморагічним інсультом і помер, виявлено крововилив у стовбур головного мозку, гіпертрофію міокарду лівого шлуночка серця до 3 см, маленькі нирки з дрібнозернистою поверхнею. Гістологічно у нирці склероз і гіаліноз клубочків, атрофія канальців, склероз артеріол. Про яке захворювання йдеться?

Ішемічна хвороба серця

Атеросклероз

Гіпертонічна хвороба

Амілоїдоз нирок

Ревматизм

2394 / 4299
До консультації стоматолога звернулася жінка 25-ти років з гострим зубним болем. У хворої спостерігались симптоми кон’юнктивиту і бронхіту. При огляді ротової порожнини лікар поставив діагноз гострого пульпіту правого верхнього моляра, а також звернув увагу на набухлі, повнокровні слизові оболонки зіва і наявність на слизовій оболонці щік відповідно малим нижнім кутнім зубам білих плям (плям Більшовського-Філатова- Коплика). Жінка працює у дитячому садочку. З анамнезу відомо, що вона в дитинстві не вакцинувалася у зв’язку з відмовою батьків, ’’дитячими” інфекціями не хворіла. Яке інфекційне захворювання найвірогідніше розвинулося у жінки?

Коклюш

Епідемічний паротит

Скарлатина

Дифтерія

Кір

2395 / 4299
У приймальне відділення лікарні доставлений хворий з черепно-мозковою травмою, у якого серед інших симптомів встановлено порушення ковтання. Який відділ ЦНС у нього імовірно уражений?

Спинний мозок на рівні CIII-IV

Спинний мозок на рівні CV-VII

Довгастий мозок

Гіпоталамус

Спинний мозок на рівні ThX-XII

2396 / 4299
До лікаря звернувся пацієнт зі скаргами на біль та обмеженість зміщення нижньої щелепи праворуч. Функція якого м’язу порушена?

M. zygomaticus major

M. ptherygoideus lateralis sinister

M. ptherygoideus lateralis dexter

M. masseter sinister

M. temporalis dexter

2397 / 4299
У молодої жінки на обличчі явища дерматиту у вигляді ”червоного метелика”. При мікроскопічному дослідженні біоптату шкіри виявлені зміни, представлені проліферативно-деструктивним васкулітом, набряком сосочкового шару дерми, вогнищевою, переважно периваскулярною, лімфогістіоцитарною інфільтрацією. Встановіть діагноз:

Ревматоїдний артрит

Дерматоміозит

Ревматизм

Системний червоний вовчак

Склеродермія

2398 / 4299
Стоматолог після обробки зуба призначив хворому знеболюючий препарат із групи нестероїдних протизапальних засобів. Вкажіть його:

Феназепам

Галантаміну гідробромід

Буторфанол

Диклофенак натрію

Галоперидол

2399 / 4299
Лікар встановив, що у хворої втрачена смакова чутливість у ділянці задньої третини язика. Функція якої пари черепних нервів порушена?

XII

VIII

V

IX

XI

2400 / 4299
При гістологічному дослідженні періапікальної тканини, видаленої у хворого, що тривалий час хворів на хронічний періодонтит, було виявлено грануляційну тканину з тяжами плоского епітелію, оточену фіброзною капсулою. Який найбільш імовірний діагноз?

Гранулюючий періодонтит

Кістогранульома

Проста гранульома

Абсцедуючий періодонтит

Складна гранульома

2401 / 4299
У населеному пункті, розташованому на березі Дніпра, виявлені випадки опісторхозу. З метою профілактики санстанція зобов’язана попередити жителів, що необхідно:

Добре проварювати і прожарювати рибу

Обдавати овочі і фрукти окропом

Достатньо проварювати яловичину

Достатньо проварювати свинину

Кип’ ятити питну воду

2402 / 4299
При обстеженні дівчини 18-ти років знайдені наступні ознаки: недорозвинення яєчників, широкі плечі, вузький таз, вкорочення нижніх кінцівок, 'шия сфінкса' розумовий розвиток не порушено. Встановлено діагноз: синдром Шерешевського-Тернера. Яке хромосомне порушення у хворої?

Трисомія Х

Трисомія 13

Трисомія 18

Моносомія Х

Нульсомія Х

2403 / 4299
При вивченні родоводу сім’ї, в якій спостерігається гіпертрихоз (надмірне оволосіння) вушних раковин, виявлена ознака трапляється в усіх поколіннях тільки у чоловіків і успадковується від батька до сина. Визначте тип успадкування гіпертрихозу:

Аутосомно-рецесивний

Зчеплений з Х-хромосомою домінантний

Зчеплений з Х-хромосомою рецесивний

Аутосомно-домінантний

Зчеплений з Y-хромосомою

2404 / 4299
На клітину подіяли речовиною, яка спричинила порушення цілісності мембран лізосом. Що відбудеться з клітиною внаслідок цього?

Трансформація

Спеціалізація

Диференціація

Автоліз

Дегенерація

2405 / 4299
У хворого затримка сечі в сечовому міхурі. Яка статева залоза пошкоджена?

Придаток яєчка

Яєчко

Цибулино-сечівникова залоза

Сім ’яний міхурець

Передміхурова залоза

2406 / 4299
До відділення хірургічної стоматології надійшла новонароджена дівчинка, яка при смоктанні починала поперхува- тись. При обстеженні виявлена розщілина твердого піднебіння, яка була наслідком незрощення середнього лобового відростка з верхньощелепним відростком I-ої зябрової дуги. Розщілина знаходилась у піднебінні між:

Lamina horizontalis os palatinum dextrum et sinistrum

Processus palatinus maxillae et lamina horizontalis os palatinum

В ділянці canalis incisivus

Processus palatinus maxillae dextrae et sinistrae

Os incisivum et processus palatinus maxillae

2407 / 4299
У хворого фронтит. В анамнезі - запалення верхньощелепної пазухи. Через який відділ носової порожнини могла потрапити інфекція в лобову пазуху?

Середній носовий хід

Присінок носової порожнини

Верхній носовий хід

Нижній носовий хід

Решітчасто-клиноподібна кишеня

2408 / 4299
До стоматолога звернувся хворий зі скаргами на асиметрію лиця. Лікар побачив, що на лівій половині обличчя брова стоїть нижче, на лобі немає складок, віко вужче, очне яблуко виступає вперед. Функція якої пари черепно-мозкових нервів уражена?

I

IV

VI

VII

V

2409 / 4299
Після травми м’яких тканин в області задньої поверхні медіального виростка плеча у постраждалого виникло відчуття поколювання шкіри медіальної поверхні передпліччя. Який з наведених нервів знаходиться в зоні ушкодження?

N. ulnaris

N. dorsalis scapularis

N. subscapularis

N. musculocutaneus

N. radialis

2410 / 4299
На гістологічному зрізі однієї з ендокринних залоз видно округлі структури різних розмірів, стінка яких утворена одним шаром епітеліальних клітин на базальній мембрані, всередині ці структури містять гомогенну неклітинну масу. Яка це залоза?

Прищитоподібна

Передня частка гіпофізу

Щитоподібна

Надниркова, кіркова речовина

Задня частка гіпофізу

2411 / 4299
При обстеженні окуліст з’ясував, що пацієнт не розрізняє синій та зелений кольори при нормальному сприйнятті іншої кольорової гами. З порушенням функції яких структур сітківки це пов’язано?

Паличкові нейрони

Біполярні нейрони

Колбочкові нейрони

Амакринні нейрони

Горизонтальні нейрони

2412 / 4299
Під час тренування у спортсмена була травмована нижня кінцівка. Лікар травматолог встановив діагноз: розрив сухожилка. До якого типу сполучної тканини належить тканина, що утворює сухожилок?

Пухка волокниста

Щільна оформлена волокниста

Ретикулярна

Хрящова

Щільна неоформлена волокнистої

2413 / 4299
При гістохімічному дослідженні лейкоцитів мазку крові визначаються клітини, у цитоплазмі яких знаходяться гранули, що містять гістамін і гепарин. Які це клітини?

Нейтрофіли

Еозинофіли

Еритроцити

Базофіли

Моноцити

2414 / 4299
При електронномікроскопічному дослідженні гіалінового хряща виявляються клітини з добре розвиненою гранулярною ендоплазматичною сіткою, комплексом Гольджі. Яку функцію виконують ці клітини?

Депонування глікогену

Руйнування міжклітинної речовини хряща

Депонування жиру

Утворення міжклітинної речовини

Трофіка хрящової тканини

2415 / 4299
Після антигенної стимуляції на гістологічному зрізі лімфовузла експериментальної тварини у мозкових тяжах знайдено велику кількість клітин такої морфології: інтенсивно базофільна цитоплазма, ексцентрично розміщене ядро з хроматином, що розташований у вигляді 'спиць колеса' та світлою ділянкою ци-топлазми біля нього. Які це клітини?

Плазмоцити

Тканинні базофіли (опасисті клітини)

Фібробласти

Адипоцити

Макрофаги

2416 / 4299
В препараті діагностується тканина, в якій клітини розміщуються поодинці та ізогрупами, а в міжклітинній речовині не видно волокнистих структур. Яка тканина присутня в препараті?

Гіалінова хрящова

Волокниста хрящова

Епітеліальна

Гладенька м’язова

Кісткова

2417 / 4299
Сполучна тканина побудована з паралельно розташованих колагенових волокон, розмежованих фібробластами. Цей тип сполучної тканини називається:

Пухка

Ретикулярна

Щільна неоформлена

Слизова

Щільна оформлена

2418 / 4299
Недорозвиненість яких відділів лицьового черепу в ембріональний період приводить до появи такої вади розвитку, як 'вовча паща'?

Лобні і верхньощелепні відростки

Піднебінні відростки

Нижньощелепні відростки

Лобні відростки

Нижньощелепні і піднебінні відростки

2419 / 4299
У навколопульпарному дентині декальцінованого зуба дорослої людини визначаються ділянки з невпорядкова- ним розташуванням дентинних трубочок і колагенових фібрил. Назвіть даний вид дентину:

Первинний

Третинний (іррегулярний)

Склерозований

Вторинний (регулярний)

Мертві шляхи

2420 / 4299
Імплантація зародка у слизову оболонку матки складається з двох фаз - адгезії та інвазії. Перша фаза супроводжується:

Активізацією секреції маткових залоз

Руйнуванням ЄПІТЄЛІОЦИТІВ слизової оболонки (ендометрію) матки

Прикріпленням бластоцисти до поверхні ендометрію

Пригніченням секреції маткових залоз

Руйнуванням сполучної тканини ендо- МЄТРІЮ

2421 / 4299
У хворого з пересадженим серцем при фізичному навантаженні збільшився хвилинний об’єм крові. Який механізм регуляції забезпечує ці зміни?

Парасимпатичні безумовні рефлекси

Парасимпатичні умовні рефлекси

Катехоламіни

Симпатичні безумовні рефлекси

Симпатичні умовні рефлекси

2422 / 4299
Пацієнт звернувся до стоматолога зі скаргами на металевий присмак у роті і печіння язика після протезування. Які дослідження необхідно провести з метою визначення причини?

Оклюзіографія

Рентгенографія

Мастикаціографія

Гальванометрія

Електроміографія

2423 / 4299
При обробці перекисом водню слизової оболонки ротової порожнини хворого, кров пофарбувалась у коричневий колір замість піноутворення. При зниженні концентрації якого з перелічених ферментів це можливо?

Ілюкозо-6-фосфатдегідрогеназа

Метгемоглобінредуктаза

Каталаза

Псевдохолінестераза

Ацетилтрансфераза

2424 / 4299
При операції на щитоподібній залозі з приводу захворювання на Базедову хворобу помилково були видалені паращи- топодібні залози. Виникли судоми, тетанія. Обмін якого біоелемента було порушено?

Магній

Залізо

Кальцій

Калій

Натрій

2425 / 4299
Мати помітила темну сечу у її 5-річної дитини. Жовчних пігментів у сечі не виявлено. Встановлено діагноз: алкаптону- рія. Дефіцит якого ферменту має місце?

Тирозиназа

Фенілаланінгідроксилаза

Оксидаза гомогентизинової кислоти

Оксидаза оксифенілпірувату

Декарбоксилаза фенілпірувату

2426 / 4299
Ціаністий калій є отрутою, смерть організму наступає миттєво. Назвіть, на які ферменти в мітохондріях діє цианістий калій:

Цитохром В5

Флавінові ферменти

Цитохромоксидаза [аа3]

Цитохром Р-450

НАД+ - залежні дегідрогенази

2427 / 4299
У хворого, що проходить курс лікувального голодування, нормальний рівень глюкози у крові підтримується, головним чином, за рахунок глюконеогенезу. З якої амінокислоти у печінці людини при цьому найбільш активно синтезується глюкоза?

Ілутамінова кислота

Аланін

Лізин

Валін

Лейцин

2428 / 4299
Хворий хворіє на цукровий діабет, що супроводжується гіперглікемією натще понад 7,2 ммоль/л. Рівень якого білка пла-зми крові дозволяє ретроспективно (за попередні 4-8 тижні до обстеження) оцінити рівень глікемії у хворого?

Фібриноген

С-реактивний білок

Альбумін

Церулоплазмін

Ілікозильований гемоглобін

2429 / 4299
У 8-місячної дитини спостерігаються блювання та діарея після прийому фруктових соків. Навантаження фруктозою призвело до гіпоглікемії. Спадкова недостатність якого ферменту є причиною стану дитини?

Фруктозо-1-фосфатальдолаза

Фруктокіназа

Фосфофруктокіназа

Гексокіназа

Фруктозо-1,6-дифосфатаза

2430 / 4299
При підвищенні концентрації чадного газу в повітрі може наступити отруєння. При цьому порушується транспортування гемоглобіном кисню від легень до тканин. Накопичення в крові якого похідного гемоглобіну є причиною цього?

Карбоксигемоглобін

Іемохромоген

Оксигемоглобін

Метгемоглобін

Карбгемоглобін

2431 / 4299
Надмірна концентрація глюкози в ротовій рідині при цукровому діабеті призводить до розвитку:

Множинного карієсу

Флюорозу

Гіпоплазії емалі

Посиленої кальцифікації емалі

Гіперплазії емалі

2432 / 4299
У 5-ти річної дитини спостерігається недостатнє звапнування емалі, карієс зубів. Гіповітаміноз якого вітаміну зумовлює розвиток такого процесу?

Кальциферол

Біотин

Фолієва кислота

Токоферол

Нікотинова кислота

2433 / 4299
З віком знижується секреторна активність привушних слинних залоз. Активність якого ферменту слини буде різко зменшуватись?

Гексокіназа

Лізоцим

Фосфатаза

Амілаза

Мальтаза

2434 / 4299
В експерименті кролю ввели нефроцитотоксичну сироватку морської свинки. Яке захворювання нирок моделювалося в цьому досліді?

Нефротичний синдром

Хронічна ниркова недостатність

Гострий дифузний гломерулонефрит

Хронічний пієлонефрит

Гострий пієлонефрит

2435 / 4299
При профілактичному огляді дітей в закарпатському селищі у багатьох знайдено множинний карієс. З недостатністю якого мінералу в їжі можна пов’язати розвиток карієсу?

Залізо

Фтор

Йод

Молібден

Кобальт

2436 / 4299
У людини, яка тривалий час голодувала, розвинулись набряки. Який основний механізм виникнення цих набряків?

Зменшення об’єму циркулюючої крові

Збільшення онкотичного тиску міжклітинної речовини

Зменшення гідростатичного тиску міжклітинної речовини

Збільшення гідростатичного тиску венозної крові

Зменшення онкотичного тиску плазми крові

2437 / 4299
У хворого діагностовано хронічний гломерулонефрит. Внаслідок значних склеротичних змін маса функціонуючих нефронів зменшилася до 10%. Яке з перерахованих нижче порушень лежить в основі розвитку в хворого уремічного синдрому?

Азотемія

Артеріальна гіпертензія

Порушення водного гомеостазу

Ниркова остеодистрофія

Порушення осмотичного гомеостазу

2438 / 4299
Хворий 38-ми років скаржиться на спрагу (випиває до 8 л води на добу), поліурію, схуднення, загальну слабкість. Хворіє впродовж 6 місяців. У сечі: питома вага - 1,001, лейкоцити - 1-2 в полі зору, білок - сліди. Яка причина постійної поліурії у хворого?

Підвищення осмотичного тиску сечі

Зменшення продукції АДГ

Ураження канальців нирок

Підвищення онкотичного тиску сечі

Ураження клубочків нирок

2439 / 4299
При запальних процесах в організмі починається синтез білків 'гострої фази' Які речовини є стимуляторами їх синтезу?

Ангіотензини

Інтерлейкін-1

Інтерферони

Імуноглобуліни

Біогенні аміни

2440 / 4299
При розтині тіла померлого у прямій і сигмоподібній кишках видно дефекти слизової оболонки неправильної форми з нерівними контурами, вони зливаються між собою, залишаючи невеликі острівці слизової оболонки, що збереглася. Про який різновид коліту можна думати?

Гнійний

Катаральний

Фолікулярний

Виразковий

Фібринозний

2441 / 4299
На секції виявлено: множинні геморагічні інфаркти легень, у деяких судинах легень буруватого кольору щільні маси, які не прикріплені до стінки судин, варикозне розширення вен нижніх кінцівок, в яких наявні тромби. Про який патологічний процес йдеться?

Тромбоемболія легеневої артерії

Застійний тромбоз легеневої артерії

Тканинна емболія легеневої артерії

Геморагічна бронхопневмонія

Жирова емболія легеневої артерії

2442 / 4299
6-річну дитину доставлено в стаціонар у стані асфіксії. У гортані виявлено сірувато-жовті плівки, що легко видаляються. Який вид запалення розвинувся?

Фібринозне

Гнійне

Десквамативно-некротичне

Катаральне

Геморагічне

2443 / 4299
У хворого на туберкульоз в біопта- ті нирки при гістологічному дослідженні у вогнищі казеозного некрозу виявлені безпорядно розсипані дрібні зерна хроматину. Наслідком чого є виявлені зміни?

Апоптоз

Пікноз ядер

Каріорексис

Мітотична активність ядер

Каріолізис

2444 / 4299
У хворого 23-х років після перенесеної ангіни розвинувся сечовий синдром (гематурія, протеїнурія, лейкоцитурія). В пункційному біоптаті нирок виявлена картина інтракапілярного проліфератив- ного гломерулонефриту, а електронно- мікроскопічно виявлені великі субепіте- ліальні депозити. Який патогенез цього захворювання?

Цитотоксична, цитолітична дія антитіл

Імунокомплексний механізм

Атопія, анафілаксія з утворенням IgE і фіксацією їх на тучних клітинах

Клітинно обумовлений цитоліз

Гранулематоз

2445 / 4299
В інфекційну лікарню поступив хворий з ознаками пневмонії, яка розвинулась на 6-й день захворювання грипом. Який метод найвірогідніше підтверджує грипозну етіологію пневмонії?

Виявлення антитіл проти гемаглютинінів вірусу грипу

Зараження курячих ембріонів

Дослідження парних сироваток

Імунолюмінісцентне дослідження мазків-відбитків з носових ходів

Виявлення антигенів віруса грипу в харкотинні методом !ФА

2446 / 4299
Від хворого на гостру кишкову інфекцію виділено вірус, який віднесено до роду ентеровірусів. Для встановлення серотипу віруса застосовують діагностичні сироватки. Ці сироватки повинні містити антитіла проти:

Білків капсиду

Вірусних гемаглютинінів

Вірусних ферментів

Білків суперкапсидної оболонки

Неструктурних білків віруса

2447 / 4299
В інфекційну лікарню поступив пацієнт з клінічними ознаками енцефаліту. В анамнезі - укус кліща. В реакції гальмування гемаглютинації виявлено антитіла проти збудника кліщового енцефаліту в розведенні 1:20, що НЕ Є діагностичним. Вкажіть наступні дії лікаря після одержання вказаного результату:

Дослідити цю ж сироватку повторно

Використати чутливішу реакцію

Відхилити діагноз кліщового енцефаліту

Повторити дослідження з іншим діа- гностикумом

Повторити дослідження із сироваткою, взятою через 10 днів

2448 / 4299
В анотації до препарату вказано, що він містить антигени збудника черевного тифу, адсорбовані на стабілізованих еритроцитах барана. З якою метою використовують цей препарат?

Для виявлення антитіл в реакції непрямої гемаглютинації

Для виявлення антитіл в реакції Відаля

Для виявлення антитіл в реакції гальмування гемаглютинації

Для виявлення антитіл в реакції зв’язування комплементу

Для серологічної ідентифікації збудника черевного тифу

2449 / 4299
Хворому з афтами слизової оболонки призначили препарат, діючим агентом котрого є галоген, а також поверхнево активна речовина, що має дезінфікуючу, дезодоруючу дію. Застосовується для де-зінфекції неметалевого інструментарію, рук, предметів догляду за інфекційними хворими. Як антисептик використовується для лікування інфікованих ран, слизової оболонки ротової порожнини, патологічних зубоясеневих кишень, дезінфекції кореневих каналів. Визначте препарат:

Калію перманганат

Кислота борна

Хлоргексидину біглюконат

Перекис водню

Діамантовий зелений

2450 / 4299
Для ремiнералiзуючої терапії початкового карієсу ЗУ6ІВ була призначена СІЛЬ лужного металу. Визначте препарат:

Калію бромид

Натрію фторид

Натрію бромид

Натрію хлорид

Калію хлорид

2451 / 4299
Для корекції артеріального тиску при колаптоїдному стані хворому було введено мезатон. Який механізм дії даного препарату?

Стимулює а- ^-адренорецептори

Блокує а-адренорецептори

Стимулює ^-адренорецептори

Стимулює а-адренорецептори

Блокує ^-адренорецептори

2452 / 4299
Після закапування в око крапель у хворого на глаукому розвинувся міоз і короткозорість. Внутрішньоочний тиск знизився. Яка група препаратів здатна викликати такий ефект?

М-холіноблокатори

Іангліоблокатори

Н-холіноміметики

а-адреноміметики

М-холіноміметики

2453 / 4299
До стоматолога звернулася хвора зі скаргами на біль, відчуття печіння у ясні від гарячого, кислого, соленого, солодкого, кровоточивість ясен під час прийому їжі і чищення зубів. Діагноз: гострий катаральний гінгівіт. Оберіть препарат з в’яжучим механізмом дії:

Іаласкорбін

Хлоргексидин

Натрій гідрокарбонат

Відвар кори дуба

Цитраль

2454 / 4299
В умовах запалення знижується сила місцевоанестезуючої дії новокаїну. Якою є причина порушення гідролізу солі новокаїну і вивільнення активного анестетика-основи в осередку запалення?

Пригнічення карбангідрази

Локальний тканинний алкалоз

Пригнічення окислювального фосфо- рилювання

Активація сукцинатдегідрогенази

Локальний тканинний ацидоз

2455 / 4299
Хворому на миготливу аритмію, в анамнезі у котрого бронхіальна астма, треба призначити протиаритмічний засіб. Який препарат з цієї групи ПРОТИПОКАЗАНИЙ хворому?

Анаприлін

Новокаїнамід

Верапаміл

Ніфедипін

Аймалін

2456 / 4299
Дитина 5-ти років поступила в ЛОР- відділення клінічної лікарні з діагнозом: гнійне запалення середнього вуха. Захворювання розпочалось з запалення носоглотки. Через який канал (каналець) скроневої кістки інфекція потрапила в барабанну порожнину?

Барабанної струни

М’язовотрубний

Сонний

Сонно-барабанні

Барабанний

2457 / 4299
При проведенні дуоденального зондування зонд не проходить зі шлунка в дванадцятипалу кишку. В якому відділі шлунка є перешкода (пухлина)?

Кардіальний відділ

Воротар

Тіло

Дно

Мала кривизна

2458 / 4299
У хворого на шкірі живота, грудної клітки з’явився висип у вигляді розеол і петехій, дрібноточковий кон’юнктиваль- ний висип. Смерть настала при явищах ураження мозку. При мікроскопічному дослідженні секційного матеріалу у ЦНС (довгастий мозок, міст), у шкірі і нирках, міокарді виявлений деструктивно- проліферативний ендотромбоваскуліт. Про яке захворювання слід думати?

Системний червоний вовчак

Сепсис

Бруцельоз

Вузликовий періартеріїт

Висипний тиф

2459 / 4299
Хворий тривалий час лікувався з приводу пневмонії нез’ясованої етіології, стійкої до стандартної терапії. З анамнезу встановлено, що він тривалий час знаходився у службовому відрядженні у США, де отримав травму, лікувався у шпиталі, після одужання повернувся на батьківщину. Оцінюючи анамнез, клінічну картину захворювання, лікар запідозрив у хворого СНІД. Результати якого методу лабораторної діагностики дозво- ляють підтвердити попередньо встановлений діагноз у даного пацієнта?

Імуноферментний аналiз

РГГА - реакція гальмування гемаглютинації

Реакщя зв’язування комплементу

Реакція Вiдаля

Електронна мікроскопія

2460 / 4299
У дитячому садку через кілька годин після вживання сирної маси майже у всіх дітей раптово з’явилися симптоми гастроентериту. При бактеріологічному дослідженні блювотних мас та залишків сирної маси було виділено золотистий стафілокок. Як доцільно продовжити дослідження для уточнення джерела інфекції?

Визначити здатність штамів до токси- ноутворення

Провести дослідження обладнання харчоблоку

Вивчити наявність антитіл у хворих дітей

Поставити алергічну пробу

Провести фаготипування виділених штамів

2461 / 4299
Хворий 67-ми років впродовж 20-ти років страждав на гіпертонічну хворобу. Помер від хронічної ниркової недостатності. Який вид мали нирки при розтині тіла?

Великі червоні

Великі з множинними тонкостінними кістами

Маленькі, щільні, поверхня дрібнозерниста

Великі строкаті

Великі білі

2462 / 4299
При обстеженні у юнака 16-ти років було виявлено прискорення серцебиття під час вдиху, сповільнене - під час видиху. На ЕКГ відмічалося: вкорочення інтервалу RR під час вдиху та подовження його під час видиху. Назвіть вид аритмії:

Синусова брадикардія

Синусова тахікардія

Миготлива аритмія

Синусова аритмія

Ідіовентрикулярний ритм

2463 / 4299
Для полегшення вправлення вивиху в плечовому суглобі з метою розслаблення м’язів хворому був введений міорела- ксант дитилін. При цьому наступило 'а- пное' Що необхідно ввести хворому?

Галантаміну гідробромід

Дипіроксим

Ізонітрозин

Бемегрид

Свіжа цитратна кров

2464 / 4299
При обстеженні хворого 35-ти років проведено гістологічне дослідження пун- ктату червоного кісткового мозку і виявлено значне зменшення кількості мегакаріоцитів. До яких змін периферичної крові це призведе?

Тромбоцитоз

Агранулоцитоз

Лейкопенія

Тромбоцитопенія

Лейкоцитоз

2465 / 4299
Аналізуються діти в одній сім’ї. Один з батьків гомозиготний по домінантному гену полідактилії, а другий - здоровий (гомозиготний по рецесивному гену). В цьому випадку у дітей проявиться закон:

Розщеплення гібридів

Одноманітності гібридів першого покоління

Чистоти гамет

Зчеплене успадкування

Незалежного спадкування

2466 / 4299
У ході експерименту з внутрішньовенним уведенням різних фізіологічно активних речовин зареєстрована тахікардія. Яка з цих речовин викликала таку реакцію?

Брадикінін

Калікреїн

Ацетилхолін

Інсулін

Тироксин

2467 / 4299
У жінки з резус-негативною кров’ю А (II) групи народилася дитина з АВ (IV) групою, у якої діагностували гемолітичну хворобу внаслідок резус-конфлікту. Яка група крові можлива у батька дитини?

III (В), резус-негативна

III (В), резус-позитивна

II (А), резус-позитивна

І (0), резус-позитивна

IV (АВ), резус-негативна

2468 / 4299
До реанімації лікарні потрапив хворий з набряком легень. Який з перелічених препаратів необхідно використати для проведення форсованого діурезу?

Спіронолактон

Гідрохлортіазид

Еуфілін

Фуросемід

Тріамтерен

2469 / 4299
У хворого з’явилися жовтушність шкіри, склер та слизових оболонок. У плазмі крові підвищений рівень загального білірубіну, в калі - стеркобіліну, в сечі - уробіліну. Який вид жовтяниці у хворого?

Гемолітична

Паренхіматозна

Обтураційна

Хвороба Жільбера

Холестатична

2470 / 4299
У пацієнта порушена функція нирок. Для перевірки стану фільтраційної здатності нирок йому призначено визначення кліренсу:

Сечової кислоти

Глутаміну

Індолу

Гідрокарбонату

Креатиніну

2471 / 4299
В експерименті після обробки нервово-м’язового препарата жаби курареподібною речовиною скорочення м’яза у відповідь на електричну стимуляцію нерва зникли. Яка функція клітинної мембрани м’яза порушується курареподібними препаратами?

Рецепція медіаторів у нервово- м’язовому синапсі

Зміна проникності для різних речовин

Підтримання внутрішньої структури клітини, її цитоскелету

Створення бар’єру між середовищем клітини та навколишньою міжклітинною рідиною

Створення електричних потенціалів по обидва боки мембрани

2472 / 4299
Хворий поступив у клініку зі струсом мозку. На фоні неврологічних симптомів у крові збільшується концентрація амі-аку. Яку речовину слід призначити для знешкодження цієї речовини у мозковій тканини?

Глутамінова кислота

Гістамін

Нікотинова кислота

Аскорбінова кислота

Серотонін

2473 / 4299
У біоптаті з правого головного бронха чоловіка 63 р., курця, виявлено пухлину, що складається з груп атипових епітеліальних клітин, які проникають поза межі базальної мембрани слизового шару, формують 'гнізда' та тяжі, в центральних частинах яких розташовані концентричні, яскраво-еозинофільні маси - 'ра- кові перлини'. Діагностуйте захворювання:

Дрібноклітинний рак

Аденокарцинома

Плоскоклітинний незроговілий рак

Плоскоклітинний зроговілий рак

Плоскоклітинна зроговіла папілома

2474 / 4299
У дитини 12-ти років низький зріст при непропорційній будові тіла і розумовій відсталості. Недостатня секреція якого гормону може бути причиною таких порушень?

Тироксин

Інсулін

Соматотропін

Ілюкагон

Кортизол

2475 / 4299
У вагітної жінки визначили групу крові. Реакція аглютинації еритроцитів відбулася зі стандартними сироватками груп 0 (І), В (III) і не відбулася зі стандартною сироваткою групи А (II). Досліджувана кров належить до такої групи:

В (III)

АВ (IV)

А (II)

0 (I)

2476 / 4299
Після фармакологічної блокади іонних каналів мембрани нервового волокна потенціал спокою зменшився з -90 до -80 мВ. Які канали було заблоковано?

Натрієві

Магнієві

Хлорні

Кальцієві

Калієві

2477 / 4299
Хворому на ревматизм призначили нестероїдний протизапальний засіб диклофенак- натрій. Через загострення супутнього захворювання диклофенак- натрій відмінили. Яке захворювання є протипоказом до призначення диклофенаку-натрію?

Гіпертонічна хвороба

Бронхіт

Виразкова хвороба шлунка

Стенокардія

Цукровий діабет

2478 / 4299
У хворого із цирозом печінки відмічається стійка артеріальна гіпотензія. (АТ- 90/50 мм рт.ст.). Чим обумовлено знижен- ня артеріального тиску при такій патології печінки?

Активація калікреїн-кінінової системи

Збільшення синтезу Аа-уретичного гормону

Посилення рефлекторного впливу ІЗ рецепторної зони дуги аорти

Зниження синтезу ангіотензиногену

Надмiрна інактивація вазопресину

2479 / 4299
У приймальне відділення поступив хворий із блідою шкірою, температура тіла знижена, пульс слабкий, малого наповнення, артеріальний тиск знижений. Дана симптоматика характерна для го-строго отруєння морфіном. Який із перелічених препаратів слід застосувати в першу чергу?

Налоксон

Ізадрин

Атропіну сульфат

Адреналіну гідрохлорид

Кодеїну сульфат

2480 / 4299
У хлопчика 5-ти років з’явилися біль під час ковтання, набряк шиї, висока температура. При огляді на мигдаликах виявлені плівки сіро-білого кольору, які знімаються з великим зусиллям. Про яке захворювання йде мова?

Кір

Скарлатина

Менінгококова інфекція

Дифтерія

2481 / 4299
При обстеженні хворого з ендокринною патологією встановлено, що в плазмі крові підвищений рівень тестостерону. Які клітини в організмі чоловіка відповідальні за продукцію цього гормону?

Сустентоцити сім’яників

Сперматогенні клітини

Іландулоцити сім’яників

Клітини передміхурової залози

Клітини сім’яних міхурців

2482 / 4299
Дитині віком 6-ти років, у якої запідозрено активний туберкульозний процес, проведено діагностичну реакцію Манту. Який імунобіологічний препарат при цьому було введено?

Вакцина БЦЖ

Вакцина АКДП

Тулярін

Вакцина АДП

Туберкулін

2483 / 4299
Хворому 68-ми років в комплекс лікування атеросклерозу, ускладненого ішемічною хворобою серця, лікар включив гіполіпідемічний засіб, який знижує вміст в крові переважно тригліцеридів. Який із вказаних препаратів було призначено хворому?

Інсулін

Кислота нікотинова

Фенофібрат

Ілібенкламід

Преднізолон

2484 / 4299
Хворий звернувся до лікаря зі скаргами на ригідність м’язів, скутість рухів, постійний тремор рук. На основі обстеження лікар встановив діагноз - хвороба Паркінсона. Зробіть раціональний вибір препарату:

Етосуксимід

Сибазон

Фенобарбітал

Дифенін

Леводопа

2485 / 4299
Хворому на ревматоїдний артрит для попередження можливого негативного впливу на слизову шлунка призначили препарат із групи нестероїдних протизапальних засобів - селективний інгібітор ЦОГ-2. Вкажіть препарат:

Ібупрофен

Анальгін

Бутадіон

Ацетилсаліцилова кислота

Целекоксиб

2486 / 4299
У хворого зі скаргами на біль у шлунку встановлено зменшення його секреторної функції, що супроводжується анемією. Нестатність якої речовини обумовлює розвиток у хворого гіповітамінозу B12 та виникнення анемії?

Фактор Кастла

Біотин

Піридоксин

Тіамін

Кальциферол

2487 / 4299
В експерименті на тварині здійснили перерізку блукаючих нервів з двох боків. Як при цьому зміниться характер дихання?

Стане поверхневим і частим

Стане глибоким і частим

Стане глибоким і рідким

Стане поверхневим і рідким

Дихання не зміниться

2488 / 4299
У дитини, хворої на серпоподібно- клітинну анемію спостерігається кілька патологічних ознак: анемія, збільшена селезінка, враження шкіри, серця, нирок і мозку. Як називається цей випадок мно- жинної дії одного гена?

Епістаз

Комплементарність

Плейотропія

Кодомінування

Полімерія

2489 / 4299
У дівчинки 15-ти років виявлено блідість шкірних покровів, глосит, гінгівіт. У крові: еритроцити - 3, 3 • 1012/л, гемоглобін - 70 г/л, кольоровий показник - 0,5. У мазку крові: гіпохромія, мікроцитоз, пой- кілоцитоз. Яка анемія спостерігається у хворої?

Таласемія

Б12-фолієводефіцитна

Залізодефіцитна

Гемолітична

Серпоподібно-клітинна

2490 / 4299
В нефрологічній клініці у юнака 19-ти років була виявлена підвищена кількість калію у вторинній сечі. Підвищення секреції якого гормону, імовірно могло викликати такі зміни?

Тестостерон

Адреналін

Глюкагон

Альдостерон

Окситоцин

2491 / 4299
В онкологічне відділення поступив хворий з підозрою на пухлину легень. При обстеженні виявили локалізацію патології в нижній частці правої легені. Скільки бронхолегеневих сегментів має ця частка?

5

2

6

3

4

2492 / 4299
У жінки, що знаходиться на лікуванні з приводу тиреотоксикозу, спостерігається підвищення температури тіла. Що лежить в основі цього явища?

Порушення синтезу глікогену

Зниження окислення жирів у печінці

Порушення дезамінування амінокислот

Зниження утилізації глюкози тканинами

Роз’єднання окисного фосфорилюван- ня

2493 / 4299
У хворого наприкінці весни і на початку літа при екскурсії поза місто, а іноді і в місті спостерігаються риніт і кон’юнктивіт, що виникають раптово, супроводжуються рясним витікання рідкого сли- зу з носа і сльозотечею. Який тип алергічних реакцій спостерігається у даному випадку?

I тип анафілактичний

III тип імунокомплексний

II тип цитотоксичний

IV тип ГУТ

2494 / 4299
У жінки 29 р. виявлено вузлоподібну пухлину нижньої щелепи із вираженою деформацією щелепи, рентгенологічними ознаками руйнування кістки. При гістологічному дослідженні пухлини спостерігаються множинні атипові одно- ядерні клітини овальної форми, поодинокі гігантські багатоядерні клітини, хаотично розміщуються кісткові балочки. Діагностуйте захворювання:

Фолікулярна амелобластома

Злоякісна амелобластома

Амелобластична фіброма

Внутрішньокісткова карцинома

Остеобластокластома

2495 / 4299
При втручанні з метою лікування вивиху нижньої щелепи лікар повинен пам’ятати про м’яз, який при скороченні відтягує назовні капсулу і суглобовий диск скронево- нижньощелепного суглоба. Який це м’яз?

M. pterygoideus medialis

M. temporalis

M. pterygoideus lateralis

M. masseter

M. mylohyoideus

2496 / 4299
Одним із методів зняття гострого болю при невралгії трійчастого нерва є створення депо анестетика в місці виходу гілок трійчастого нерва. В якій ділянці потрібно ввести анестетик для знечулен- ня першої гілки трійчастого нерва?

Ділянка надперенісся

Ділянка лобного відростка верхньої щелепи

Підочноямковий край

Орбітальний край виличної кістки

Надочноямковий край

2497 / 4299
З метою серологічної діагностики інфекційного захворювання лікарю необхідно здійснити реакцію непрямої (пасивної) гемаглютинації. Що потрібно використати для постановки цієї реакції крім сироватки хворого?

Еритроцитарний діагностикум

Інтерферон

Діагностична сироватка

Анатоксин

Гемолітична сироватка

2498 / 4299
ПІСЛЯ утворення плащового дентину живлення ВНУТРІШНІХ КЛІТИН емалевого органу порушується. Який біологічний механізм забезпечує відновлення функціонального стану і живлення амелобластів?

Інверсія

Ретрузія

Інвазія

Прогресія

Регресія

2499 / 4299
При мікроскопічному дослідженні бі- оптата з товстої кишки виявлена пухлина з призматичного епітелію, що формує атипові залозисті структури різної форми і величини. Базальна мембрана залоз зруйнована. Клітки пухлини поліморфні, ядра гіперхромні, відзначається велика кількість патологічних мітозів. Який діа-гноз найбільш імовірний?

Базальноклітинний рак

Недиференційований рак

Аденокарцинома

Солідний рак

Слизовий рак

2500 / 4299
При остеолатеризмі зменшується міцність колагену, що зумовлена помітним зменшенням утворення поперечних зшивок у колагенових фібрилах. Причиною цього явища є зниження активності такого ферменту:

Пролілгідроксилаза

Лізилгідроксилаза

Колагеназа

Моноаміноксидаза

Лізилоксидаза

2501 / 4299
У маленької дівчинки 5-ти років лікар видалив зуб на якому було два корені. Який це зуб?

Малий кутній верхньої щелепи

Малий кутній нижньої щелепи

Великий кутній верхньої щелепи

Ікло верхньої щелепи

Великий кутній нижньої щелепи

2502 / 4299
У видаленій матці жінки 55-ти років патологоанатом у товщі міометрію виявив щільний вузол діаметром 5 см з чіткими межами, на розрізі тканина вузла волокниста, сіро- рожевого кольору, з хаотичним розташуванням пучків волокон. Мікроскопічно пухлина складається з гладком’язевих клітин, що утворюють пучки різної товщини, що йдуть у різних напрямках, і прошарків місцями гіаліні- зованої сполучної тканини. Яка пухлина розвинулась у хворої?

Міосаркома

Фіброміома

Фібросаркома

Фіброма

Рабдоміома

2503 / 4299
Після травми хребта у пацієнта 18- ти років впродовж місяця спостерігалась відсутність самовільного сечовипускання, яке пізніше відновилося. Який відділ спинного мозку був пошкоджений?

Поперековий

Грудний

Крижовий

Поперековий і крижовий

Шийний

2504 / 4299
Сечокам’яна хвороба ускладнилася виходом конкременту з нирки. На якому рівні сечоводу, найімовірніше, він може зупинитися?

На межі черевної та тазової частин

На 5 см вище тазової частини

На 2 см вище впадіння в сечовий міхур

В середній черевній частині

У нирковій мисці

2505 / 4299
В результаті надмірної рухливості плода відбулося перетиснення пуповини, але кровообіг між плодом і матір’ю не порушився. Наявність яких структур у пуповині сприяла цьому в першу чергу?

Залишок жовткового стебельця

Оболонка вени

Залишок алантоїса

Слизова тканина

Оболонка артерій

2506 / 4299
Солдати, які отримали поранення у розпал битви, можуть не відчувати болю до її завершення. Які гормони опіатної антиноціцептивної системи зменшують відчуття болю?

Ендорфіни

Вазопресин

Окситоцин

Серотоніни

Альдостерон

2507 / 4299
У хворого виявлено порушення секреторної функції піднижньощелепної слинної залози. Який нерв забезпечує її вегетативну іннервацію?

N. mandibularis

N. petrosus minor

N. auriculotemporalis

N. petrosus major

Chorda tympani

2508 / 4299
Проведено розтин тіла чоловіка, який помер від сепсису. Виявлено гнійне розплавлення тканин шиї та КЛІТКОВИНИ переднього середостіння. Гістологічно спостерігається багато дрібних абсцесів стільникової будови, у центрі яких розташовані базофільні гомогенні утвори, до яких одним кінцем прикріплені короткі паличковидні структури. Який найбільш імовірний діагноз?

Туберкульоз

Лейшманіоз

Краснуха

Дифтерія

Актиномікоз

2509 / 4299
При обстеженні хворого виявлено погіршення кровопостачання переднього відділу міжшлуночкової перегородки серця. В якій артерії знижений кровотік?

Задня міжшлуночкова

Права вінцева

Передня міжшлуночкова

Огинаюча

2510 / 4299
При огляді ротової порожнини на вестибулярній поверхні нижнього різця зліва виявлене утворення грибоподібної форми рожевого кольору до 2 см, яке широкою ніжкою фіксоване до надальвео- лярної тканини. Під час гістологічного дослідження виявлено розгалужені судини капілярного типу з судинними бруньками, ділянками крововиливів та осередки гемосидерозу. Який найбільш імовірний діагноз?

Гігантоклітинний епуліс

Фіброматоз ясен

Фіброзний епуліс

Ангіоматозний епуліс

Кавернозна гемангіома

2511 / 4299
У пацієнта при оцінці основного обміну, визначеного методом непрямої калориметрії необхідно врахувати належний рівень обміну речовин та енергії. Найбільш точно визначити його величину можна при врахуванні:

Поверхні тіла та маси

Дихального коефіцієнту й поверхні тіла

Зросту й дихального коефіцієнту

Дихального коефіцієнту й калоричного коефіцієнту кисню

Статі, віку, зросту й маси тіла

2512 / 4299
При мікроскопічному дослідженні фекалій від хворого з явищами профузного проносу, багаторазового блювання та наростаючої інтоксикації, було виявлено грамнегативні палички, що нагадували кому і розташовувались групами у вигляді табунців риб. Культура збудника була виділена через середовище накопичення - 1% пептонну воду, де вона утворювала ніжну плівку. Збудник якого захворювання був виділений з фекалій хворого?

Кишковий єрсиніоз

Псевдотуберкульоз

Холера

Сальмонельоз

Шигельоз

2513 / 4299
У молодої жінки діагностовано позаматкову вагітність. В якому з перерахованих органів відбувається запліднення?

Перешийок маткової труби

Порожнина піхви

Порожнина малого тазу

Ампула маткової труби

Порожнина матки

2514 / 4299
Хворий на бронхіальну астму тривалий час приймає преднізолон. Який механізм дії препарату?

Блокада натрієвих каналів

Блокада лейкотрієнових рецепторів

Гальмування активності фосфоліпази А

Пригнічення активності дигідрофола- тредуктази

Блокада гістамінових рецепторів

2515 / 4299
У органах хворого запідозрили паразитування декількох видів паразитів. Як називається така сукупність паразитів, що населяють організм?

Фітоценоз

Біотоп

Екосистема

Паразитоценоз

Біогеоценоз

2516 / 4299
У молодої жінки виявлена аномалія рефракції ока, при якій фокусування зображення можливе за сітківкою. Як називається ця аномалія?

Еметропія

Гіперметропія

Міопія

Астигматизм

Пресбіопія

2517 / 4299
У жінки, що тривало приймала антибіотики з приводу кишкової інфекції, розвинулося ускладнення з боку слизової порожнини рота у вигляді запального процесу і білого нальоту, у якому при бактеріологічному дослідженні були виявлені дріжджеподібні грибки Candida albicans. Який з перерахованих препаратів показаний для лікування цього ускладнення?

Фуразолідон

Тетрациклін

Поліміксин

Бісептол

Флуконазол

2518 / 4299
У чоловіка 30-ти років виявлено хронічний гастрит із підвищеною кислото- утворюючою функцією шлунка. Який препарат найбільш доцільно застосовувати для профілактики виразкової хвороби?

Атропін

Фамотидин

Пілокарпіну гідрохлорид

Но-шпа

Метоклопрамід

2519 / 4299
Серпоподібноклітинна анемія у людини супроводжується появою в крові аномального гемоглобіну, зміною форми еритроцитів, розвитком анемії. Дане захворювання є результатом:

Поліплоїдії

Хромосомної аберації

Генної мутації

Мітохондріальної мутації

Політенії

2520 / 4299
Однією з патогенетичних ланок у розвитку променевої хвороби є інтенсифікація процесів вільно радикального окиснення речовин. Які речовини є основним джерелом утворення вільних радикалів?

Білки

Гормони

Ліпіди

Вуглеводи

Вода

2521 / 4299
В епідермісі є клітини, що виконують захисну функцію і мають моноци- тарний генез. Які це клітини?

Меланоцити

Кератиноцити зернистого шару

Клітини Лангерганса

Кератиноцити базального шару

Кератиноцити остистого шару

2522 / 4299
Одним з найбільш небезпечних і типових побічних ефектів антиаритмічних засобів є негативна інотропна дія. Який із перелічених антаритмічних засобів НЕ ЗМЕНШУЄ силу серцевих скорочень?

Верапаміл

Дигоксин

Пропранолол

Метопролол

Хінідину сульфат

2523 / 4299
У хворого встановлений діагноз: заглотковий абсцес. Куди може поширитися гнійне запалення?

Передтрахеальний простір шиї

Середнє нижнє середостіння

Надгруднинний міжфасціальний простір шиї

Переднє нижнє середостіння

Заднє нижнє середостіння

2524 / 4299
У хворого 26-ти років з ознаками колі-ентериту, виділено чисту культуру бактерій, яка за морфологічними, куль- туральними та біохімічними властивостями віднесена до роду шигел. Яку з названих реакцій доцільно застосувати для серологічної ідентифікації збудника?

Аглютинації

Преципітації

Гальмування гемаглютинації можливих збудників захворювання

Непрямої гемаглютинації

Зв’язування комплементу

2525 / 4299
При гістологічному дослідженні ділянки тканини виявлені явища каріопікнозу, каріорексису, каріолізису у ядрах клітин, а також плазмоліз - у цитоплазмі клітин. Який патологічний процес має місце у даному випадку?

Гіаліноз

Апоптоз

Атрофія

Некроз

Дистрофія

2526 / 4299
У судово-медичних, антропологічних та археологічних дослідженнях для визначення віку людини аналізують загальну масу та товщину тканини, яка впродовж усього життя відкладається на поверхні дентина кореня зуба. Про яку тканину йдеться?

Емаль

Кісткова

Пухка сполучна

Цемент

Щільна сполучна

2527 / 4299
У ЖІНКИ 42-х РОКІВ має МІСЦЄ цукровий дiабет із підвищеною концентрацією глюкози в крові натще (11,5 ммоль/л). Яке з перелічених порушень буде характерне для цього захворювання?

Аміноацидурія

Метаболічний алкалоз

Глюкозурія

Гіперкапнія

Респіраторний ацидоз

2528 / 4299
Аналізується каріотип жіночого організму з синдромом трисомії - (47, ХХХ). При складанні ідіограми у цьому наборі буде така кількість пар гомологічних хромосом:

24 пари

22 пари

47 пар

23 пари

21 пара

2529 / 4299
Після початку лікування туберкульозу легень, хворий звернувся до лікаря зі скаргами на появу червоних сліз та сечі. Який препарат міг викликати такі зміни?

Бензилпеніциліну калієва сіль

Бісептол-480

Рифампіцин

Бензилпеніциліну натрієва сіль

Цефазолін

2530 / 4299
Хворий звернувся до стоматолога із симптомами запалення слизової оболонки ротової порожнини. У мазках, отриманих з пародонтальних кишень, виявлено найпростіші з непостійною формою тіла, розміром 6-60 мкм, здатні утворювати псевдоподії. Які це найпростіші?

Trichomonas hominis

Entamoeba gingivalis

Entamoeba histolytica

Entamoeba coli

Lamblia intestinalis

2531 / 4299
Жінці, яка скаржиться на постійне відчуття страху, тривоги, поставлено діагноз неврозу та призначено препарат з анксіолітичною дією. Який це препарат?

Настойка женьшеню

Пірацетам

Аміназин

Кофеїн-бензоат натрію

Діазепам

2532 / 4299
До лікаря звернувся хворий із скаргами на слабкість, нудоту, недокрів’я. Хворий зазначив, що 3 місяці тому використовував в їжу свіжепосолену ікру. На який гельмінтоз могла захворіти людина?

Дифілоботріоз

Трихінельоз

Теніаринхоз

Дикроцеоліоз

Теніоз

2533 / 4299
16-тирічна дівчина має зріст 139 см, крилоподібну шию, нерозвинені грудні залози, первинну аменорею. Найбільш імовірно, вона має такий каріотип:

46, ХХ/46, XY

46, XY

45, Х0

47, ХХХ

46, ХХ

2534 / 4299
В шліфі зуба виявляється структура, в якій розрізняють чергування світлих і темних смуг, розташованих перпендикулярно до її поверхні, а також тонкі паралельні лінії росту. Яка це структура?

Безклітинний цемент

Емаль

Дентин

Клітинний цемент

Пульпа

2535 / 4299
Порушення процесів мієлінізації нервових волокон призводить до неврологічних розладів і розумової відсталості. Такі симптоми характерні для спадкових і набутих порушень обміну:

Сфінголіпідів

Вищих жирних кислот

Холестерину

Фосфатидної кислоти

Нейтральних жирів

2536 / 4299
У гістологічному препараті сечоводу кілька оболонок. Яким епітелієм вистелена слизова оболонка цього органу?

Багатошаровий перехідний

Одношаровий кубічний

Багатошаровий плоский незроговілий

Одношаровий призматичний з облямівкою

Одношаровий плаский

2537 / 4299
У хворого початкова стадія гінгіві- ту. Спостерігається гіперемія ясен у при- шийкових областях зубів внаслідок роз-ширення судин мікроциркуляторного русла, що приносять кров. Яка речовина тучних клітин забезпечила вказані зміни?

Субстанція Р

Ендорфіни

Ацетилхолін

Адреналін

Гістамін

2538 / 4299
У дитини, яку годували синтетичними сумішами, з’явились ознаки недостатності вітаміну В\. В яких реакціях бере участь цей вітамін?

Окислювальне декарбоксилування кетокислот

Гідроксилювання проліну

Декарбоксилювання амінокислот

Трансамінування амінокислот

Окислювально-відновні реакції

2539 / 4299
Яка група організмів мають кільцеві та лінійні молекули ДНК, що формують хромосоми простої будови (не мають гі- стонів)?

Бактерії

Бактеріофаги

Найпростіші

Віруси

Гриби

2540 / 4299
До лікаря-стоматолога звернувся хворий зі скаргами на сухість в ротовій порожнині, як в стані спокою, так і під час прийому їжі. При обстеженні встановлено порушення секреторної діяльності під’язикової і підщелепної слинних залоз. В якому випадку може бути таке явище?

Пошкодження блокового нерва

Пошкодження піднижньощелепного нерва

Пошкодження язикоглоткового нерва

Пошкодження окорухового нерва

Пошкодження вегетативних волокон барабанної струни лицевого нерва

2541 / 4299
У потерпілого травма верхньої щелепи, вибитий перший малий кутній зуб. Який відросток верхньої щелепи пошкоджений?

Піднебінний

Альвеолярний

Лобовий

Виличний

2542 / 4299
Хвора 18-ти років впродовж декількох тижнів скаржиться на біль та крово- точивість в області 35 зуба при вживанні твердої їжі. Об’єктивно: на жувальній поверхні цього зуба - велика каріозна порожнина, яка заповнена м’язоподібною тканиною. При зондуванні виникає кровотеча і біль в області з’єднання каріозної порожнини з пульпою. Який діагноз найбільш імовірний?

Хронічний гангренозний пульпіт

Хронічний фіброзний пульпіт

Хронічний гіпертрофічний пульпіт

Гострий гнійний пульпіт

Хронічний глибокий карієс

2543 / 4299
Із щелепної кістки пацієнта 45-ти років оперативно видалено порожнисте утворення діаметром - 2,5 см, заповнене жовтуватою рідиною з кристалами холестерину, яке було розташоване у верхівці кореня каріозного зуба. При мікроскопічному дослідженні внутрішня поверхня порожнини вкрита багатошаровим плоским епітелієм з акантозом, без керати- нізації. Стінка порожнини фіброзна, з кристалами холестерину, інфільтрована лімфоцитами, плазмоцитами, макрофагами, що резорбують жири. Який діагноз найбільш імовірний?

Фолікулярна кіста

Остеобластокластома

Кератокіста

Амелобластома

Радикулярна кіста

2544 / 4299
Хворий звернувся до лікаря зі скаргами на періодичні висипання герпети- чних пухирців на лінії губ і на крилах носа. Такий стан спостерігається впродовж 10-ти років, кожний раз після зниження захисних сил організму. Лікар встановив діагноз: лабіальний герпес. Як називається така форма інфекції?

Екзогенна

Гостра

Латентна

Затяжна

Персистенція

2545 / 4299
У хворого флегмона у ділянці крило-піднебінної ямки з ураженням однойменного вегетативного вузла. Функція якої залози буде порушена у першу чергу?

Навколовушна

Піднижньощелепна

Слізна

Під’язикова

2546 / 4299
У хворого травма у ділянці передньої поверхні переднього драбинчастого м’яза. Функція якого нерва може бути порушена?

Блукаючий

Плечового сплетення

Діафрагмальний

ЗВОРОТНІЙ гортанний

Додатковий

2547 / 4299
При ОГЛЯДІ ротової порожнини хворого виявлено карієс коронки зуба, яка звернена до власне ротової порожнини. Яка поверхня уражена?

Facies contactus

Facies mesialis

Facies vestibularis

Facies lingualis

Facies distalis

2548 / 4299
У хворого порушено формування відчуття солодкого та солоного смаку на ВЄРХІВЦІ та по краях язика. ЯКІ сосочки язика уражені?

Papillae foliatae

Papillae fungiformes

Papillae filiformes

Papillae vallatae

Papillae conicae

2549 / 4299
Хворий 46-ти років з ревматичним стенозом помер від хронічної легенево- серцевої недостатності. На розтині виявлені щільні коричневого кольору легені. Який пігмент зумовив забарвлення легенів?

Іемозоїн

Ліпофусцин

Порфірин

Меланін

Іемосидерин

2550 / 4299
Велику частину коронки, шийки і кореня зуба складає дентин, товщина якого з віком може збільшуватися, можливе також його часткове відновлення після пошкодження. Які структури забезпечують ці процеси?

Одонтобласти

Амелобласти

Цементобласти

Дентинні канальці

Перитубулярний дентин

2551 / 4299
Хворий скаржиться, що при згадуванні про минулі трагічні події в його житті, у нього виникають тахікардія, задишка і різкий підйом артеріального тиску. Які структури ЦНС забезпечують за-значені кардіореспіраторні реакції у даного хворого?

Латеральні ядра гіпоталамуса

Кора великих півкуль

Мозочок

Чотиригорбкове тіло середнього мозку

Специфічні ядра таламуса

2552 / 4299
Хворий звернувся з відчуттям серцебиття після стресу. ЧСС- 104/хв., тривалість інтервалу P — Q - 0,12 сек., QRS - без змін. Який тип аритмії у хворого?

Синусова брадикардія

Екстрасистолія

Миготлива аритмія

Синусова тахікардія

Синусова аритмія

2553 / 4299
Провідну роль в процесі кальцифіка- ції тканин зуба відіграє білок остеокальцин, який має високу здатність зв’язувати іони кальцію, завдяки наявності в поліпе- птидному ланцюзі залишків модифікованої амінокислоти:

7-карбоксиглутамінова

Аланін

^-амінопропіонова

Y-аміномасляна

Карбоксиаспарагінова

2554 / 4299
У плазмі крові пацієнта зпідвищи- лась активність ізоферментів ЛДІ і ЛДГ2. Про патологію якого органа це свідчить?

Мозок

Нирки

Печінка

Міокард

Скелетні м’язи

2555 / 4299
В медичній практиці застосовують антикоагулянти, що посилюють дію інгі- бітора факторів коагуляції антитромбіну III. Такий ефект притаманний:

Дерматан-сульфату

Кератан-сульфату

Гіалуроновій кислоті

Колагену

Іепарину

2556 / 4299
У дорослої людини у стані спокою частота серцевих скорочень дорівнює 40/хв. Водієм ритму серця у людини є:

Волокна Пуркін’є

Ніжки пучка Гіса

Синоатріальний вузол

Атріовентрикулярний вузол

Пучок Гіса

2557 / 4299
При видаленні верхнього лівого третього моляра виникла значна кровотеча. Ураження якої артерії призвело до кровотечі?

A. facialis

A. alveolaris inferior

A. alveolaris superioris anterioris

A. infraobitalis

A. alveolaris superioris posterioris

2558 / 4299
Після екстракції зуба виникла гостра коміркова кровотеча. Який засіб при внутрішньовенному введені забезпечить гемостаз?

Кальцію хлорид

Натрію хлорид

Кислота амінокапронова

Вікасол

Тромбін

2559 / 4299
У пацієнта після операції з застосуванням апарату штучного кровообігу з’явилася гемоглобінурія, причиною якої може бути:

Біологічний гемоліз

Хімічний гемоліз

Імунний гемоліз

Механічний гемоліз

Термічний гемоліз

2560 / 4299
У потерпілого спостерігається кровотеча в ділянці верхньої губи. Яка артерія пошкоджена внаслідок травми?

Верхньощелепна

Язикова

Лицева

Кутова

Поверхнева скронева

2561 / 4299
Від хворого виділені нерухомі бактерії овоїдної форми з біполярним забарвленням. В організмі утворюють ніжну капсулу. На агарі утворюють колонії з мутно-білим центром, оточеним фестончатою облямівкою, що нагадують мереживо. Продукують 'мишачий токсин' Дані властивості притаманні для збудника:

Сибірки

Коклюшу

Туляремії

Чуми

Бруцельозу

2562 / 4299
У хворого відмічається м’язова слабкість та біль в м’язах, що обумовлено порушенням утворення карнозину та ансе- рину із амінокислоти в-аланіну. В організмі людини утворення вказаної амінокислоти відбувається в процесі катаболізму:

АМФ

УМФ

ГМФ

ІМФ

ёАМФ

2563 / 4299
Хворий 33-х років, що страждає на інсулінозалежний цукровий діабет, на фоні ін’єкції інсуліну втамував спрагу великою кількістю води, що призвело до розвитку гіпоглікемічної коми. Який вид порушення водно-сольового обміну супроводжує даний стан?

Гіперосмолярна гіпогідратація

Гіперосмолярна гіпергідратація

Гіпоосмолярна гіпергідратація

Гіпоосмолярна гіпогідратація

Ізоосмолярная гіпергідратація

2564 / 4299
Хворий 84-х років страждає на паркінсонізм, одним з патогенетичних ланок якого є дефіцит медіатора в окремих структурах мозку. Якого медіатора насамперед?

Норадреналін

Адреналін

Гістамін

Дофамін

Ацетилхолін

2565 / 4299
Епідеміологічні дослідження показують, що у людей з групою крові І (0), частіше, ніж у людей з рештою груп крові, виникає виразкова хвороба шлунка і 12-палої кишки. До проявів якого виду реактивності можна віднести дане явище?

Індивідуальна

Гіпергічна

Гіперергічна

Видова

Групова

2566 / 4299
При лабораторному обстеженні крові людини, яку вкусила змія, виявлено гемоліз еритроцитів, гемоглобінурію. Дія зміїної отрути зумовлена наявністю в ній ферменту:

Фосфоліпаза Д

Фосфоліпаза А1

Фосфоліпаза С

Сфінгомієліназа

Фосфоліпаза А2

2567 / 4299
В медичній практиці широкого застосування набули курареподібні речовини (міорелаксанти). Про яку побічну дію повинен пам’ятати лікар при їх застосуванні?

Розлади мозкового кровообігу

Зупинка серця

Тромбоутворення

Розслаблення дихальних м’язів

Приступи судом

2568 / 4299
При ПІДЙОМІ в гори у людини збільшується частота дихання i прискорюється серцебиття. ЯКІ ЗМІНІ в КРОВІ спричиняють це?

Зниження рН

Підвищення осмотичного тиску

Зниження парціального тиску О2

Підвищення парціального тиску С02

Підвищення рН

2569 / 4299
У досліді вивчали просторовий поріг шкірної чутливості. Він буде найбільшим у шкірі:

Плеча

Спини

Тильної поверхні кисті

Гомілки

Обличчя

2570 / 4299
Чоловіку 35-ти років, хворому на атопічний дерматит, був призначений лоратадин. Визначте механізм дії цього препарату:

Стимулює М-холінорецептори

Блокує ГАМК-рецептори

Блокує Ні-гістамінові рецептори

Блокує ^-адренорецептори

Стимулює дофамінові рецептори

2571 / 4299
До лікарні доставлено хворого з отруєнням бертолетовою сіллю (KCLO3). Яка форма гіпоксії розвинулася у нього?

Респіраторна

Циркуляторна

Тканинна

Гіпоксична

Гемічна

2572 / 4299
Під час іспиту студент побачив у гістологічному препараті орган, в кірковій речовині якого розташовані скупчення лімфатичних вузликів. У мозковій речовині знаходились тяжі клітин, що відхо-дять від вузликів. Строму органа утворюють сполучна та ретикулярна тканини. Який орган досліджував студент?

Червоний кістковий мозок

Лімфатичний вузол

Тимус

Мигдалик

Селезінка

2573 / 4299
Проведене медичне обстеження населення високогірного селища. Який по казник буде характерним для цього адаптивного екологічного типу людей?

Зниження показників основного обміну

Підвищення концентрації холестерину в крові

Підсилення потовиділення

Зменшення рівня гемоглобіну

Зростання кисневої ємності крові

2574 / 4299
Оксид азоту відіграє важливу роль у релаксації гладеньких м’язів судин і зниженні артеріального тиску, розширенні коронарних артерій. NO в організмі може утворюватись з:

Метіоніну

Лізину

Глутаміну

Аргініну

Проліну

2575 / 4299
Під час патологоанатомічного розтину в померлого виявлено гній у задньому середостінні. Через який простір поширився запальний процес, що був локалізований у передньому відділі шиї?

Передвісцеральний

Надгруднинний

Переддрабинчатий

Позавісцеральний

Міждрабинчатий

2576 / 4299
Альпініст на вершині гори Ельбрус відчуває нестачу кисню, порушення дихання, серцебиття, оніміння кінцівок. Який вид гіпоксії розвинувся?

Тканинна

Гіпоксична

Серцева

Гемічна

Циркуляторна

2577 / 4299
У пацієнта 28-ми років, хворого на пневмонію, розвинувся набряк легень. Об’єктивно спостерігається швидке зростання глибини та частоти дихання з переважанням фази вдиху над фазою видиху, загальне збудження, розширення зіниць, тахікардія, підвищення артеріального тиску, судоми. Який стан розвинувся у хворого?

Другий період асфіксії

Апное

Перший період асфіксії

Гіпопное

Третій період асфіксії

2578 / 4299
У приймальне відділення доставлено хворого з гострою кровотечею. У разі втрати якого об’єму циркулюючої крові може настати летальний кінець у такого хворого?

12%

75%

50%

33%

3%

2579 / 4299
У чоловіка 30-ти років множинні переломи КІСТОК КІНЦІВОК в результат автомобільної катастрофи. В стаціонар доставлений через годину після травми в важкому стані: свідомість затьмарена, шкіра бліда, вкрита липким потом, зіниці вузькі із слабкою реакцією на світло, дихання рідке, поверхневе, тони серця приглушені, пульс ниткоподібний, АТ- 60/40 мм рт.ст. Ознак внутрішньої чи зовнішньої крововтрати немає. Який стан розвинувся у хворого?

Опіковий шок

Постгеморагічний шок

Травматичний шок

Кардіогенний шок

Анафілактичний шок

2580 / 4299
При мікроскопії зубного нальоту клінічно здорової дитини 10-ти років виявлені Гр+ і Гр- мікроорганізми. Який саме етап забарвлення за методом Грама дозволив віддиференціювати Гр+ бактерії від Гр-?

Обробка водним розчином фуксину

Обробка генціанвіолетом

Обробка розчином Люголя

Обробка спиртом

Обробка сірчаною кислотою

2581 / 4299
Хворому на туберкульоз легень призначений препарат першого ряду, який спричинив розвиток невриту лицьового нерва та порушення рівноваги. Вкажіть цей лікарський засіб:

Фуразолідон

Ізоніазид

Левоміцетин

Цефазолін

Бісептол

2582 / 4299
Хворому 45-ти років з діагнозом 'амебна дизентерія' призначено протипротозойний засіб з групи нітроімідазолу. Вкажіть препарат:

Поліміксин

Бійохінол

Цефалексин

Фуразолідон

Метронідазол

2583 / 4299
У постраждалого - різана рана у ділянці краю нижньої щелепи попереду жувального м’яза. З рани виливається кров темно-червоного кольору. Яку судину пошкоджено?

V lingualis

V facialis

V retromandibularis

V jugularis anterior

V jugularis externa

2584 / 4299
У постраждалого поверхневе поранення передньої ділянки шиї, спостерігається кровотеча. Кров темного кольору. Яку судину пошкоджено?

V jugularis interna

A. thyroidea superior

V jugularis externa

A. carotis externa

V jugularis anterior

2585 / 4299
Педіатр, оглядаючи порожнину рота дитини, виявив 8 зубів. Дитина розвивається нормально. Визначте вік дитини:

10-12 місяців

12-15 місяців

8-9 місяців

6-8 місяців

15-20 місяців

2586 / 4299
У хворого зареєстрували ЕКГ За яким її елементом лікар може оцінити процеси розповсюдження деполяризації передсердями?

Зубець P

Зубець S

Зубець T

Зубець Q

Зубець R

2587 / 4299
Глюкагон - це гормон, що підвищує рівень глюкози в крові шляхом активації:

Гліколізу

Глікогенсинтази

Глікогенфосфорилази в міоцитах

Глюкокінази

Глікогенфосфорилази в гепатоцитах

2588 / 4299
Хворий 47-ми років впродовж останніх 3-х років хворіє на туберкульоз легень, скаржиться на задишку, тяжкість в області правого боку грудної клітки; температура тіла - 37,7oC. Виявлено правосторонній ексудативний плеврит. Який тип клітин очікується у плевральному пунктаті?

Еозинофіли

Еритроцити

Лімфоцити

Атипові клітини

Нейтрофіли

2589 / 4299
У пацієнта при рентгенологічному обстеженні виявлено грижу міжхребцевого диска грудного відділу хребта. Який вид з’єднання МІЖ хребцями зазнав патологічних ЗМІН?

Синостоз

Синхондроз

Геміартроз

Діартроз

Синдесмоз

2590 / 4299
У наркотизованого собаки зареєстровано електрокардіограму (ЕКГ). Збільшення яких елементів ЕКГ відбулося після подразнення гілочки лівого блукаючого нерва, яка йде до серця?

Тривалість сегменту S-T

Амплітуда зубців комплексу QRS

Тривалість зубця Т

Тривалість інтервалу P-Q

Амплітуда всіх зубців ЕКГ

2591 / 4299
При огляді хворого лікар- стоматолог виявив наслідки опіку сли-зової ротової порожнини агресивною речовиною. Зі слів хворого, він випадково випив концентровану оцтову кислоту. Вказані зміни відповідають поняттю:

Патологічний стан

Патологічна реакція

Типовий патологічний процес

Патологічний процес

Хвороба

2592 / 4299
При дослідженні біологічного матеріалу пацієнта, хворого на грип, виявлено збудник, який відносять до неклітинної форми життя. У своєму складі збудник містить РНК, оточену білками оболонки. Даний збудник належить до:

Гельмінтів

Бактерій

Грибів

Вірусів

Пріонів

2593 / 4299
Хворий 53-х років страждає на множинний карієс зубів. Що з перерахованого може бути причиною даного захворювання?

Стрептокок групи А

Механічне подразнення зубними протезами

Гіпосалівація

Харчування переважно вуглеводною їжею

Гіперсалівація

2594 / 4299
При вивченні зрізу зуба з ознаками глибокого карієсу на межі з пульпою виявлена зона замісного дентину, у якій багато одонтобластів, невелика кількість дентинних канальців, які мають широкий просвіт, розташовуються рідко і мають звивистість хода. Який загальнопатоло- гічний процес лежить в основі утворення зони замісного дентину?

Замісна гіперплазія

Патологічна регенерація

Робоча гіперплазія

Захисна гіперплазія

Гормональна гіперплазія

2595 / 4299
У постраждалого глибока різана рана у ділянці зовнішньої поверхні кута нижньої щелепи. Який м’яз при цьому буде пошкоджено?

M. buccinator

M. orbicularis oris

M. masseter

M. depressor anguli oris

M. zygomaticus

2596 / 4299
У хворого на слизовій оболонці піднебіння з’явилася безболісна виразка з щільними краями і сальним червоним дном. Мікроскопічно відзначається лімфо-плазмоцитарна інфільтрація, гігантські клітини типу Пирогова-Лангханса і виражений продуктивний васкуліт. Визначте вид гранульоми:

Ревматична

Сифілітична

Туберкульозна

Склеромна

Лепрозна

2597 / 4299
До лікаря звернувся пацієнт зі скаргами на біль та обмеженість зміщення нижньої щелепи праворуч. Функція якого м’язу порушена?

M. temporalis dexter

M. masseter sinister

M. ptherygoideus lateralis sinister

M. zygomaticus major

M. ptherygoideus lateralis dexter

2598 / 4299
Лікар-стоматолог під час огляду пацієнта побачив 'білі зони' демінералізації зубів. Який мікроорганізм міг спричинити такі зміни емалі?

Neisseria meningitidis

Staphylococcus aureus

Streptococcus mutans

Staphylococcus epidermidis

Streptococcus pyogenes

2599 / 4299
У дитини 10-ти років раптово підвищилася температура до 39oC, з’явилися нежить, кашель, світлобоязнь. Під час огляду на слизовій оболонці порожнини рота виявлені плями Філатова-Копліка. Який з діагнозів найбільш імовірний?

Краснуха

Вітряна віспа

Кір

Скарлатина

Мононуклеоз

2600 / 4299
На РОЗТИНІ тіла чоловіка, що помер ВІД серцевої декомпенсації, виявлено РІЗКО збільшене серце масою 960 г, ('бичаче серце'), у порожнині перикарду 90 мл солом’яного кольору рідини. Порожнини серця різко розширені, міокард в’ялий, товщина стінки лівого шлуночка - 2,3 см. Нирки зменшені у розмірах, масою по 70 г, поверхня їх дрібнозерниста. Під час мікроскопічного дослідження внутрішніх органів виявлений гіаліноз артеріол і гіпертрофія м’язового шару більш крупних артерій. Назвіть основне захворю-вання:

Ревматизм

Атеросклероз

Ішемічна хвороба серця

Карідоміопатія

Гіпертонічна хвороба

2601 / 4299
Педіатр, оглядаючи порожнину рота дитини, виявив 8 зубів. Дитина розвивається нормально. Визначте вік дитини:

10-12 місяців

12-15 місяців

15-20 місяців

6-8 місяців

8-9 місяців

2602 / 4299
Відомо, що ротенон викликає інгібування дихального ланцюгу. Який комплекс дихального ланцюга мітохондрій інгібується цією речовиною?

Коензим Q-цитохром с-редуктаза

Цитохромоксидаза

АТФ -синтетаза

Сукцинат-коензим Q-редуктаза

НАДН-коензим Q-редуктаза

2603 / 4299
За даними ВООЗ, щорічно на Землі на малярію хворіють приблизно 250 млн. чоловік. Ця хвороба зустрічається переважно у тропічних і субтропічних областях. Межі її розповсюдження співпадають з ареалами комарів роду:

Анофелес

Кулекс

Мансоніа

Кулізета

Аедес

2604 / 4299
Мисливець напився сирої води із ставка. Яким трематодозом він міг заразитися?

Клонорхоз

Фасціольоз

Дикроцеліоз

Опісторхоз

Парагонімоз

2605 / 4299
На клітину подіяли речовиною, яка спричинила порушення цілісності мембран лізосом. Що відбудеться з клітиною внаслідок цього?

Диференціація

Автоліз

Трансформація

Дегенерація

Спеціалізація

2606 / 4299
Пацієнт скаржиться на виділення сечі під час статевого акту. Який орган уражений?

Сечовий міхур

Передміхурова залоза

Придаток яєчка

Яєчко

Сім’яні міхурці

2607 / 4299
У хворого зареєстрували ЕКГ За яким її елементом лікар може оцінити процеси розповсюдження деполяризації передсердями?

Зубець S

Зубець Q

Зубець T

Зубець P

Зубець R

2608 / 4299
Рентгенологічно встановлено скупчення гною у клиноподібній пазусі. В який носовий хід виділяється гній?

Правий та лівий верхні

Правий нижній

Лівий середній

Лівий нижній

Правий середній

2609 / 4299
У пацієнта при рентгенологічному обстеженні виявлено грижу міжхребцевого диска грудного відділу хребта. Який вид з’єднання між хребцями зазнав патологічних змін?

Синостоз

Синхондроз

Діартроз

І^міартроз

Синдесмоз

2610 / 4299
До стоматолога звернувся хворий зі скаргами на асиметрію обличчя. Лікар побачив, що на лівій половині обличчя брова стоїть нижче, на лобі немає складок, повіка вужча, очне яблуко виступає вперед. Функція якої пари черепно-мозкових нервів уражена?

I

VII

IV

VI

V

2611 / 4299
Хворий скаржиться на болі під час жування, особливо при висуненні нижньої щелепи вперед і зміщенні її вбік. Функція яких м’язів порушена?

Жувальні

Щелепно-під’язикові

Скроневі

Латеральні крилоподібні

Медіальні крилоподібні

2612 / 4299
Лікарі-стоматологи мають великий ризик захворіти на гепатит В, тому підлягають обов’язковій імунізації. За допомогою якої вакцини це може бути зроблено?

Анатоксин

Хімічна

Інактивована

Рекомбінантна

Жива

2613 / 4299
Після травми м’яких тканин в області задньої поверхні медіального виростка плеча у постраждалого виникло відчуття поколювання шкіри медіальної поверхні передпліччя. Який з наведених нервів знаходиться в зоні ушкодження?

N. ulnaris

N. radialis

N. dorsalis scapularis

N. musculocutaneus

N. subscapularis

2614 / 4299
Диференціювання В-лімфоцитів в плазматичні клітини призводить до вироблення імуноглобулінів, які відповідають за специфічну імунну відповідь організму. У якому органі імунної системи відбувається диференціювання В-лімфоцитів?

Червоний кістковий мозок

Щитоподібна залоза

Тимус

Печінка

Мигдалики

2615 / 4299
На шліфі зуба в області верхівки кореня визначається тканина, що складається з клітин відростчатої форми, які оточені мінералізованою міжклітинною речовиною. Назвіть дану тканину:

Емаль

Клітинний цемент

Періодонт

Дентин плащовий

Ретикулофіброзна кісткова тканина

2616 / 4299
При електронній мікроскопії нирки виявлені канальці, які вистелені кубічним епітелієм. В епітелії розрізняють світлі та темні клітини. В світлих клітинах мало органел. Цитоплазма утворює складки. Ці клітини забезпечують реабсорбцію води з первинної сечі у кров. Темні клітини за будовою і функцією нагадують парієтальні клітини шлунка. Які канальці представлені у мікроскопічному препараті?

Висхідні канальці петлі Генле

Збірні ниркові трубочки

Дистальні канальці

Низхідні канальці петлі Генле

Проксимальні канальці

2617 / 4299
При гістохімічному дослідженні лейкоцитів мазку крові визначаються кліти- ни, у цитоплазмі яких знаходяться гранули, що містять гістамін і гепарин. Які це клітини?

Базофіли

Еозинофіли

Моноцити

Нейтрофіли

Еритроцити

2618 / 4299
Хірург пошкодив нерв, що іннервує щелепно-під’язиковий м’яз. Назвіть пошкоджений нерв:

N. accessorius

N. hypoglossus

N. facialis

N. trigeminus

N. glossopharyngeus

2619 / 4299
В емалі на межі з дентином зустрічаються незвапновані ділянки, що часто стають місцем проникнення інфекції в зуб. Як називають такі утворення?

Енамелобласти

Волокна Томса

Емалеві призми

Емалеві пучки

Дентинобласти

2620 / 4299
На мікропрепараті яєчника представлено округле утворення, залозисті клітини якого містять ліпідні краплі. Визначте цю структуру:

Примордіальний фолікул

Зрілий фолікул

Атретичне тіло

Жовте тіло

Первинний фолікул

2621 / 4299
Імплантація зародка у слизову оболонку матки складається з двох фаз - адгезії та інвазії. Перша фаза супроводжується:

Активізацією секреції маткових залоз

Прикріпленням бластоцисти до поверхні ендометрію

Пригніченням секреції маткових залоз

Руйнуванням епітеліоцитів слизової оболонки (ендометрію) матки

Руйнуванням сполучної тканини ендометрію

2622 / 4299
На поверхні коронки другого великого кутнього зуба, яка контактує з щокою, лікар виявив каріозну порожнину. Як називається вражена поверхня коронки?

Facies distalis

Facies mesialis

Facies lingualis

Facies vestibularis

Facies occlusalis

2623 / 4299
На розтині тіла чоловіка 46-ти років, що не лікувався від гострої кишкової інфекції і помер від сепсису, виявлені: флегмона параректальної клітковини, множинні виразки прямої і сигмоподібної кишок, окремі з перфорацією, слизова цих відділів кишечника потовщена, вкрита сіруватого кольору плівками, що важко знімаються. Про яке захворювання можна думати?

Туберкульоз

Амебіаз

Дизентерія

Холера

Черевний тиф

2624 / 4299
У хворої набряки. У сечі велика кількість білку. Про порушення функції якого відділу нефрону це свідчить?

Висхідна частина петлі Генле

Дистальний звивистий каналець

Проксимальний звивистий каналець

Ниркове тільце

Низхідна частина петлі Генле

2625 / 4299
Мати помітила темну сечу у її 5-річної дитини. Жовчних пігментів у сечі не виявлено. Встановлено діагноз: алкаптонурія. Дефіцит якого ферменту має місце?

Оксидаза оксифенілпірувату

Оксидаза гомогентизинової кислоти

Декарбоксилаза фенілпірувату

Тирозиназа

Фенілаланінгідроксилаза

2626 / 4299
Ціанистий калій є отрутою, смерть організму наступає миттєво. Назвіть, на які ферменти в мітохондріях діє ціанистий калій:

Цитохром Р-450

Цитохромоксидаза [аа3]

НАД+ - залежні дегідрогенази

Флавінові ферменти

Цитохром В5

2627 / 4299
У хворого, що проходить курс лікувального голодування, нормальний рівень глюкози у крові підтримується, головним чином, за рахунок глюконеогенезу. З якої амінокислоти у печінці людини при цьому найбільш активно синтезується глюкоза?

Аланін

Лейцин

Глутамінова кислота

Лізин

Валін

2628 / 4299
На розтині тіла померлого чоловіка 52-х років виявлені зміни в легенях у вигляді сегментарної ділянки казеозного некрозу, які зливаються та займають верхню частину правої легені. Легеня збільшена, щільна, на розрізі - жовтуватого кольору, на плеврі - фібринозні плівки. Назвіть форму туберкульозу:

Інфільтративний туберкульоз

Туберкульома

Гострий кавернозний туберкульоз

Казеозна пневмонія

Циротичний туберкульоз

2629 / 4299
У дитини 3-х років впродовж 6-ти місяців розвинулась деформація обличчя у результаті симетричного збільшення об’єму кутів нижньої щелепи. Мікроскопічно: між кістковими балками розташована сполучна тканина з великою кількістю судин і примітивними кістковими балочками. Яке захворювання найбільш імовірне?

Херувізм

Еозинофільна гранульома

Остеосаркома

Фіброма

Остеобластокластома

2630 / 4299
На автопсії у померлого 72-х років з повторним трансмуральним інфарктом міокарда оболонки епікарду та перикарду набухлі, потовщені, кострубаті, неначе вкриті волосяним покровом. Назвіть вид запалення у оболонках серця:

Гнійне

Дифтеритичне

Крупозне

Серозне

Катаральне

2631 / 4299
На розтині хворого, що помер від серцево-судинної недостатності, темний колір ступні правої нижньої кінцівки. В судинах стегна згустки сірувато-червоного кольору, що частково перекривають просвіти. Також на стінках відмічаються ділянки жовто-сірого кольору та фіброзні бляшки, деякі з яких кам’яної щільності. Ускладнення якої клініко- анатомічної форми атеросклерозу було у хворого?

Атеросклероз артерій нижніх кінцівок

Атеросклероз артерій кишечника

Атеросклероз артерій головного мозку

Атеросклероз ниркових артерій

Атеросклероз аорти

2632 / 4299
У хворого на слизовій оболонці піднебіння з’явилася безболісна виразка з щільними краями і сальним червоним дном. Мікроскопічно відзначається лімфо-плазмоцитарна інфільтрація, гігантські клітини типу Пирогова-Лангханса і виражений продуктивний васкуліт. Визначте вид гранульоми:

Лепрозна

Сифілітична

Склеромна

Туберкульозна

Ревматична

2633 / 4299
При розтині тіла 7-річної дитини, що померла внаслідок декомпенсації вродженої вади серця, виявлено збільшення маси та об’єму тимусу. Мікроскопічним дослідженням виявлено нормальну будову тимусу. Який патологічний процес має місце в тимусі?

Тимома

Дисплазія тимусу

Акцидентальна інволюція

Агенезія тимусу

Вроджена тимомегалія

2634 / 4299
При запальних процесах в організмі починається синтез білків 'гострої фази'. Які речовини є стимуляторами їх синтезу?

Імуноглобуліни

Інтерлейкін-1

Ангіотензини

Біогенні аміни

Інтерферони

2635 / 4299
Хворий звернувся до лікаря зі скаргами на загальну слабкість, порушення сну. Об’єктивно: шкіра має жовтий колір. У крові: збільшена кількість прямого білірубіну, жовчні кислоти. Кал ахолічний. Для якого стану характерні ці зміни?

Синдром Жильбера

Механічна жовтяниця

Паренхіматозна жовтяниця

Гемолітична жовтяниця

Хронічний холецистит

2636 / 4299
При розтині тіла померлого у прямій і сигмоподібній кишках видно дефекти слизової оболонки неправильної форми з нерівними контурами, вони зливаються між собою, залишаючи невеликі острівці слизової оболонки, що збереглася. Про який різновид коліту можна думати?

Гнійний

Фібринозний

Катаральний

Фолікулярний

Виразковий

2637 / 4299
На секції виявлено: множинні геморагічні інфаркти легень, у деяких судинах легень буруватого кольору щільні маси, які не прикріплені до стінки судин, варикозне розширення вен нижніх кінцівок, в яких наявні тромби. Про який патологічний процес йдеться?

Жирова емболія легеневої артерії

Застійний тромбоз легеневої артерії

Тромбоемболія легеневої артерії

Геморагічна бронхопневмонія

Тканинна емболія легеневої артерії

2638 / 4299
Водолаз, який занурився на глибину 75 метрів, відчув симптоми порушення функцій ЦНС - збудження, послаблення уваги, ейфорія, професійні помилки. Токсичною дією на нейрони якої речовини зумовлені ці симптоми?

Лактат

Кисень

Азот

Вуглекислий газ

Аміак

2639 / 4299
Істотним недоліком мікроскопічного методу діагностики інфекцій є його недостатня інформативність у зв’язку з морфологічною подібністю багатьох видів мікроорганізмів. Яка імунологічна реакція дозволяє значно підвищити інформативність цього методу?

Радіоімунний аналіз

Імуноферментний аналіз

Реакція опсонізації

Реакція Кумбса

Реакція імунофлюоресценції

2640 / 4299
Лікар-стоматолог під час огляду пацієнта побачив 'білі зони' демінералізації зубів. Який мікроорганізм міг спричинити такі зміни емалі?

Staphylococcus aureus

Neisseria meningitidis

Staphylococcus epidermidis

Streptococcus pyogenes

Streptococcus mutans

2641 / 4299
У дитини 10-ти років раптово підвищилася температура до 39°C, з’явилися нежить, кашель, світлобоязнь. Під час огляду на слизовій оболонці порожнини рота виявлені плями Філатова-Копліка. Який з діагнозів найбільш імовірний?

Кір

Мононуклеоз

Скарлатина

Краснуха

Вітряна віспа

2642 / 4299
В закритому колективі виникла необхідність перевірити стан імунітету проти дифтерії, щоб обґрунтувати необхідність вакцинації. Які дослідження слід провести з такою метою?

Перевірити стан імунітету щодо дифтерійної палички

Перевірити медичну документацію щодо вакцинації

Встановити рівень антитіл проти дифтерійної палички

Встановити титр антитоксинів в РНГА

Перевірити членів колективу на носійство палички дифтерії

2643 / 4299
Для ремінералізуючої терапії початкового карієсу зубів була призначена сіль лужного металу. Визначте препарат:

Калію хлорид

Калію бромід

Натрію фторид

Натрію хлорид

Натрію бромід

2644 / 4299
У хворого із вивихом щелепи лікар застосував засіб для короткочасного розслаблення м’язів з групи міорелаксантів. Виберіть цей засіб:

Новокаїн

Піридостигміну гідробромід

Дитилін

Цититон

Папаверину гідрохлорид

2645 / 4299
Хворій для терапії остеомієліту було призначено антибіотик, який здатен добре проникати в кісткову тканину. Назвіть цей препарат:

Лінкоміцину гідрохлорид

Цефазолін

Стрептоміцину сульфат

Поліміксин В

Амфотерицин В

2646 / 4299
Стоматологічному хворому для пригнічення страху перед болем призначили психоседативний засіб. Який препарат найбільш ефективний в цьому випадку?

Настоянка валеріани

Аміназин

Натрію бромід

Діазепам

Літію карбонат

2647 / 4299
Після закапування в око крапель у хворого на глаукому розвинувся міоз і короткозорість. Внутрішньоочний тиск знизився. Яка група препаратів здатна викликати такий ефект?

Гангліоблокатори

М-холіноблокатори

Н-холіноміметики

М-холіноміметики

α-адреноміметики

2648 / 4299
У лікарню було привезено хворого з опіками шкіри. Для очищення ран від мертвих тканин та слизу лікар для локального лікування призначив ферментний препарат. Назвіть цей препарат:

Трипсин

Пепсин

Аспарагіназа

Стрептокіназа

Панзинорм

2649 / 4299
Під час стану емоційного напруження у людини з’явились поперечні зморшки на лобі посередині. Скорочення якого м’яза відбулося?

M. procerus

M. auricularis anterior

M. temporoparietalis

M. occipitofrontalis

M. corrugator supercilii

2650 / 4299
Новонароджений не зробив перший вдих. Під час патологоанатомічного розтину тіла встановлено, що при вільних дихальних шляхах легені не розправилися. Що з наведеного могло бути причиною цього?

Звуження бронхів

Збільшення розміру альвеол

Потовщення плеври

Розрив бронхів

Відсутність сурфактанту

2651 / 4299
При гострому запаленні привушної слинної залози спостерігається пошкодження клітин секреторних відділів. Які клітини страждають при цьому?

Серозні, клітини з базальною посмугованістю, зірчасті

Білкові, серозні, слизові

Білково-слизові

Облямовані епітеліоцити, клітини з базальною посмугованістю

Серозні, міоепітеліальні

2652 / 4299
При пародонтозі хворому призначили жиророзчинний вітамінний препарат, що бере активну участь в окисно-відновних процесах в організмі. Антиоксидант є фактором росту, антиксерофтальмічним, забезпечує нормальний зір. В стоматологічній практиці використовується для прискорення епітелізації при захворюваннях слизових оболонок при пародонтозі. Визначте цей препарат:

Ергокальциферол

Вікасол

Ціанокобаламін

Токоферолу ацетат

Ретинолу ацетат

2653 / 4299
Дитина 5-ти років поступила в ЛОР-відділення клінічної лікарні з діагнозом: гнійне запалення середнього вуха. Захворювання розпочалось з запалення носоглотки. Через який канал (каналець) скроневої кістки інфекція потрапила в барабанну порожнину?

Сонний

Барабанної струни

М’язовотрубний

Барабанний

Сонно-барабанні

2654 / 4299
У хворого - глибока рвана рана із нерівними краями, вкрита гноєм. У крайових відділах - соковита грануляційна тканина, яка не здіймається над рівнем рани. Назвіть вид загоєння рани:

Безпосереднє закриття дефекту епітеліальної тканини

Організація рани

Загоювання первинним натягом

Загоювання вторинним натягом

Загоювання під струпом

2655 / 4299
При проведенні дуоденального зондування зонд не проходить зі шлунка в дванадцятипалу кишку. В якому відділі шлунка є перешкода (пухлина)?

Дно

Воротар

Кардіальний відділ

Мала кривизна

Тіло

2656 / 4299
До приймального відділення був доставлений хворий 56-ти років у непритомному стані з вираженим пригніченням дихання та серцевої діяльності, зниженням сухожильних рефлексів. Зі слів супроводжуючої його дружини, останнім часом він страждав на безсоння та приймав на ніч снодійні засоби. Який лікарський препарат необхідно ввести хворому для невідкладної допомоги?

Дипіроксим

Бемегрид

Атропіну сульфат

Протаміну сульфат

Унітіол

2657 / 4299
Хвора 30-ти років скаржиться на сильну спрагу, сухість у роті, які з’явилися після сильного нервового потрясіння. При лабораторному обстеженні виявлено збільшення цукру в крові до 10 ммоль/л. Захворювання якої ендокринної залози у хворої?

Підшлункова

Щитоподібна

Епіфіз

Статеві

Наднирники

2658 / 4299
В клініку звернувся чоловік 45-ти років із скаргами на втрату чутливості в ділянці задньої 1/3 язика. Функція якої пари черепно-мозкових нервів порушена?

V

Х

VIII

IX

XII

2659 / 4299
У дитячому садку через кілька годин після вживання сирної маси майже у всіх дітей раптово з’явилися симптоми гастроентериту. При бактеріологічному дослідженні блювотних мас та залишків сирної маси було виділено золотистий стафілокок. Як доцільно продовжити дослідження для уточнення джерела інфекції?

Визначити здатність штамів до токсиноутворення

Провести фаготипування виділених штамів

Поставити алергічну пробу

Провести дослідження обладнання харчоблоку

Вивчити наявність антитіл у хворих дітей

2660 / 4299
У хворого виявлено порушення прохідності дихальних шляхів на рівні дрібних і середніх бронхів. Які зміни кислотно-основної рівноваги можуть розвинутись у пацієнта?

Метаболiчний алкалоз

Респіраторний ацидоз

КОР не зміниться

Респіраторний алкалоз

Метаболiчний ацидоз

2661 / 4299
Аналізуються діти в одній сім’ї. Один з батьків гомозиготний по домінантному гену полідактилії, а другий - здоровий (гомозиготний по рецесивному гену) в цьому випадку у дітей проявиться закон:

Зчеплене успадкування

Одноманітності гібридів першого покоління

Чистоти гамет

Розщеплення гібридів

Незалежного спадкування

2662 / 4299
У пацієнта відмічена висока концентрація вазопресину (АДГ) у крові. До яких змін діурезу це призведе?

Поліурія

Олігоурія

Анурія

Глюкозурія

Натрійурія

2663 / 4299
Який препарат слід призначити для лікування хворій 54-х років з хронічною серцевою недостатністю, що супроводжується тахіаритмією та набряками?

Метопролол

Спіронолактон

Дигоксин

Аміодарон

Анаприлін

2664 / 4299
У дівчинки спостерігаються висока температура і біль у горлі. Об’єктивно: набряк м’якого піднебіння, на мигдаликах сірі плівки, які важко відокремлюються, залишаючи глибокі кровоточиві дефекти тканини. Яке захворювання найбільш імовірне?

Інфекційний мононуклеоз

Дифтерія зіву

Лакунарна ангіна

Ангіна Симановського-Венсана

Некротична ангіна

2665 / 4299
У хворого спостерігається остеопороз кісток, в крові - гіперкальціємія, гіпофосфатемія. Яка причина такого стану?

Посилена секреція тироксину

Посилена секреція паратгормону

Посилена секреція кортикостероїдів

Пригнічення секреції паратгормону

Пригнічення секреції кортикостероїдів

2666 / 4299
Під час експедиції до Середньої Азії студенти виявили членистоногу тварину завдовжки 7 см. Тіло поділяється на головогруди з 4 парами ходильних ніг і сегментоване черевце, в останньому сегменті якого є дві отруйні залози, що відкриваються отворами на кінці гачкоподібного жала. Було встановлено, що ця тварина є нічним хижаком і отруйною для людини. Учасники експедиції віднесли її до ряду:

Acarina

Aranei

Aphaniptera

Solpugae

Scorpiones

2667 / 4299
У пацієнта, який тривалий час знаходився на незбалансованому харчуванні з маленькою кількістю білка, розвинулась жирова інфільтрація печінки. Назвіть речовину, відсутність якої у їжі могла бути причиною цього стану:

Біотин

Аланін

Холестерин

Арахідонова кислота

Метіонін

2668 / 4299
У хворого необхідно зменшити насосну функцію серця. Які мембранні циторецептори доцільно для цього заблокувати?

Н-холінорецептори

М-холінорецептори

α-адренорецептори

α- та βадренорецептори

β-адренорецептори

2669 / 4299
Окисне декарбоксилювання піровиноградної кислоти каталізується складним поліферментним комплексом з участю кількох функціонально зв’язаних коферментів. Вкажіть цей комплекс:

КоА-SH, ФАД, ПАЛФ, ТГФК, карні-тин

ТДФ, ФАД, КоА-SH, НАД, ліпоєва кислота

ФАД, ТГФК, ПАЛФ, ТДФ, холін

НАД, ПАЛФ, ТДФ, метилкобаламін, біотин

Ліпоєва кислота, ТГФК, ПАЛФ, метилкобаламін

2670 / 4299
Щоб взяти спинномозкову рідину для дослідження, лікар повинен зробити пункцію підпавутинного простору спинного мозку. Між якими хребцями треба ввести голку, щоб не пошкодити спинний мозок?

III і IV поперекові

I і II поперекові

IV і V грудні

XI і XII грудні

XII грудний і I поперековий

2671 / 4299
До відділення реанімації поступив хворий з гострим отруєнням невідомим лікарським засобом. Для швидкого виведення отрути з організму провели форсований діурез. Який з перерахованих засобів використали для цієї процедури?

Гідрохлортіазид

Спіронолактон

Омепразол

Дитилін

Фуросемід

2672 / 4299
Відомо, що в метаболізмі катехо-ламінових медіаторів особлива роль належить ферменту моноаміноксидазі (МАО). Яким шляхом цей фермент ін-активує медіатори (норадреналін, адреналін, дофамін)?

Приєднання аміногрупи

Карбоксилювання

Видалення метильної групи

Окисне дезамінування

Гідроліз

2673 / 4299
У хворого на гострий панкреатит у сечі суттєво підвищено вміст діастази. Який засіб з групи інгібіторів протеолізу необхідно включити до складу комплексної терапії цього хворого?

Дигестал

Фестал

Панкреатин

Контрикал

Мезим форте

2674 / 4299
В бактеріологічну лабораторію з вогнища харчового отруєння доставлена в’ялена риба, при дослідженні якої на середовищі Кітта-Тароцці бактеріолог виявив мікроорганізми, подібні до 'тенісної ракетки'. Збудником якої хвороби вони могли бути?

Дизентерія

Ботулізм

Сальмонельоз

Дифтерія

Черевний тиф

2675 / 4299
Дитині віком 6-ти років, у якої запідозрено активний туберкульозний процес, проведено діагностичну реакцію Манту. Який імунобіологічний препарат при цьому було введено?

Вакцина АДП

Вакцина АКДП

Вакцина БЦЖ

Тулярін

Туберкулін

2676 / 4299
В клініку потрапив хворий з ознаками спазматичної кишкової непрохідності. При наданні медичної допомоги з кишечника хворого виділено гельмінтів, які належать до класу круглих червів, розмірами 25-40 см. Визначте вид гельмінту:

Кривоголовка дванадцятипала

Аскарида людська

Гострик

Вугриця кишкова

Волосоголовець людський

2677 / 4299
Хворому на гострий бронхіт з утрудненим відхаркуванням, призначили ацетилцистеїн. Яка дія засобу забезпечить лікувальний ефект?

Активація миготливого епітелію бронхів

Рефлекторна стимуляція перистальтики бронхіол

Подразнення бронхіальних залоз

Олужнювання харкотиння

Деполімеризація мукопротеїдів

2678 / 4299
У хворого на ішемічну хворобу серця визначено підвищений вміст в плазмі крові тригліцеридів і ліпопротеїнів дуже низької щільності. Який препарат слід призначити?

Добутамін

Аміодарон

Фамотидин

Фенофібрат

Лізиноприл

2679 / 4299
Серед органічних речовин клітини знайдено полімер, який складається з десятків, сотень і тисяч мономерів. Молекула здатна самовідтворюватися та бути носієм інформації. За допомогою рентгеноструктурного аналізу виявлено, що молекула складається з двох спірально закручених ниток. Вкажіть цю сполуку:

Гормон

РНК

Целюлоза

Вуглевод

ДНК

2680 / 4299
Харчовий раціон жінки 30-ти років, яка годує груддю, містить 1000 мг кальцію, 1300 мг фосфору та 20 мг заліза на добу. Яким чином слід відкоригувати вміст мінеральних речовин у цьому харчовому раціоні?

Зменшити вміст фтору

Зменшити вміст заліза

Збільшити вміст заліза

Збільшити вміст фосфору

Збільшити вміст кальцію

2681 / 4299
У пацієнта має місце пошкодження волокон дев’ятої пари черепних нервів (язикоглотковий). Формування якого відчуття буде порушено?

Гірке

Кисле

Солодке

Солоне

Усіх смакових відчуттів

2682 / 4299
У дитини, хворої на серпоподібно-клітинну анемію спостерігається кілька патологічних ознак: анемія, збільшена селезінка, враження шкіри, серця, нирок і мозку. Як називається цей випадок множинної дії одного гена?

Кодомінування

Епістаз

Полімерія

Плейотропія

Комплементарність

2683 / 4299
У дівчинки 15-ти років виявлено блідість шкірних покровів, глосит, гінгівіт. У крові: еритроцити - 3,3 • 1012/л, гемоглобін - 70 г/л, кольоровий показник -0,5. У мазку крові: гіпохромія, мікроцитоз, пойкілоцитоз. Яка анемія спостерігається у хворої?

Залізодефіцитна

Серпоподібно-клітинна

Б12-фолієводефіцитна

Гемолітична

Таласемія

2684 / 4299
У хворого 49-ти років виявлено: набряк обличчя, значна протеїнурія, гіпо-протеїнемія, диспротеїнемія, гіперліпідемія. Який попередній діагноз?

Нефротичний синдром

Сечокам’яна хвороба

Цистит

Пієлонефрит

Простатит

2685 / 4299
У жінки, що знаходиться на лікуванні з приводу тиреотоксикозу, спостерігається підвищення температури тіла. Що лежить в основі цього явища?

Порушення дезамінування амінокислот

Зниження утилізації глюкози тканинами

Порушення синтезу глікогену

Зниження окислення жирів у печінці

Роз’єднання окисного фосфорилюван-ня

2686 / 4299
Лікар-стоматолог призначив пацієнту з артритом щелепно-лицевого суглоба диклофенак- натрій. Який механізм дії цього препарату?

Пригнічення каталази

Активація опіатних рецепторів

Блокада опіатних рецепторів

Активація фосфодіестерази

Пригнічення циклооксигенази-2

2687 / 4299
Хірургу необхідно провести видалення частини травмованої ступні по лінії Лісфранкова суглоба. Яку зв’язку необхідно перетнути?

П ’ятково-човноподібна

Роздвоєна

Таранно-п’яткова

Таранно-човноподібна

Медіальна міжкісткова заплесно-плеснова

2688 / 4299
При мікроскопічному дослідженні бляшкоподібного утвору, видаленого з бокової поверхні язика у чоловіка із зубними протезами, виявлено значне потовщення епітеліального шару з явищами паракератозу, гіперкератозу та акантозу, у сполучній тканині - дрібні кругло- клітинні інфільтрати. Діагностуйте патологічний стан:

Хронічний стоматит

Іхтіоз

Хронічний глосит

Лейкоплакія

Гунтерівський глосит

2689 / 4299
Хворий обстежується за клінічними показаннями. Проведені дослідження по діагностиці вірусних гепатитів. У сироватці крові виявлені тільки антитіла до HbsAg. Такий результат свідчить про:

Перенесений гепатит В

Гострий гепатит В

Гепатит А

Гострий гепатит С

Хронічний гепатит С

2690 / 4299
Для лікування туберкульозу призначений антибіотик, який забарвлює сечу в червоний колір. Вкажіть цей антибіотик:

Нітроксолін

Амоксицилін

Цефотаксим

Еритроміцин

Рифампіцин

2691 / 4299
Відомо, що сальні залози мають голокриновий тип секреції. За рахунок яких структурних компонентів поновлюються клітини цієї залози?

Клітини-себоцити

Багатошаровий плоский епітелій вивідної протоки

Одношаровий кубічний епітелій вивідної протоки

Клітини гермінативного шару

Міоепітеліальні клітини

2692 / 4299
В експерименті при вивченні процесів всмоктування продуктів гідролізу їжі і води було встановлено, що основним відділом шлунково-кишкового тракту, де відбуваються ці процеси, є:

Товста кишка

Пряма кишка

Ротова порожнина

Тонка кишка

Шлунок

2693 / 4299
Детоксикація білірубіну відбувається в мембранах ендоплазматичного ретикулума гепатоцитів. Основна частина білірубіну секретується гепатоцитами в жовч у формі:

Моноглюкуроніда

Диглюкуроніда

Вільного

Непрямого

Некон’югованого

2694 / 4299
У пацієнта в ділянці нижньої щелепи виник болючий вузлик червоного кольору. Гістологічно: скупчення гнійного ексудату в декількох волосяних фолікулах. Яка клініко- морфологічна форма запалення має місце?

Флегмона

Карбункул

Фурункул

Абсцес

Натічник

2695 / 4299
У чоловіка, що загинув раптово при явищах гострого порушення мозкового кровообігу, на розтині виявлений розрив аневризми середньої мозкової артерії і округла порожнина діаметром 4 см, заповнена кров’ю, у лобній частці головного мозку. Як називається такий вид крововиливу?

Геморагічна інфільтрація

Кровопідтік

Петехії

Гематома

2696 / 4299
Солдати, які отримали поранення у розпал битви, можуть не відчувати болю до її завершення. Які гормони опіатної антиноціцептивної системи зменшують відчуття болю?

Вазопресин

Ендорфіни

Серотоніни

Окситоцин

Альдостерон

2697 / 4299
У хворого у пародонтальних кишенях виявлена патогенна мікрофлора. Лікар-стоматолог призначив хіміотерапевтичний засіб - похідне імідазолу. Який це препарат?

Флюконазол

Ацикловір

Фуразолідон

Метронідазол

Метилурацил

2698 / 4299
Після введення в експлуатацію системи подачі води у новий мікрорайон міста працівники санепідслужби визначили загальне мікробне число води. Яке значення цього показника є гранично допустимим для питної води?

1000

500

400

10

100

2699 / 4299
При дослідженні каріотипу п’ятирічного хлопчика виявлено 46 хромосом. Одна з хромосом 15-ої пари довша від звичайної, тому що до неї приєднана ділянка хромосоми з 21-ої пари. Вкажіть вид мутації, що має місце в цього хлопчика:

Інверсія

Делеція

Транслокація

Поліплоїдія

Дуплікація

2700 / 4299
Розщеплення цАМФ та цГМФ до звичайних, нециклічних нуклеозидмоно-фосфатів каталізується таким ферментом:

Глікогенфосфорилаза

Фосфодіестераза

Протеїнкіназа

Аденілатциклаза

Глюкозо-6-фосфатаза

2701 / 4299
В експерименті на тварині після пе-рерізки блукаючих нервів спостерігають постійну тахікардію. Який вплив парасимпатичної нервової системи на роботу серця демонструє цей експеримент?

Змішаний вплив

Збуджувальний

Гальмівний

Парадоксальний

Сумація збуджень

2702 / 4299
До лікаря-травматолога звернувся хворий, у якого після травми стало утрудненим активне розгинання руки в ліктьовому суглобі. Який м’яз імовірно ушкоджений?

M. coracobrachialis

M. deltoideus

M. latissimus dorsi

M. pectoralis minor

M. triceps brachii

2703 / 4299
У жінки 42-х років має місце цукровий діабет із підвищеною концентрацією глюкози в крові натще (11,5 ммоль/л). Яке з перелічених порушень буде характерне для цього захворювання?

Респіраторний ацидоз

Глюкозурія

Гіперкапнія

Аміноацидурія

Метаболічний алкалоз

2704 / 4299
У хворого скарги на гострий біль в животі переймоподібного характеру, часті позиви на дефекацію, рідкі кров’янисті випорожнення із слизом. Лабораторним дослідженням мазків фекалій виявлено організми непостійної форми, що містять еритроцити. Яке можливе захворювання?

Кишковий трихомоноз

Шистосомоз

Лямбліоз

Балантидіаз

Амебіаз

2705 / 4299
Пасажири автобуса у спекотну погоду попросили відкрити люки. Який шлях тепловіддачі при цьому активується?

Теплопроведення

Випромінювання та теплопроведення

Випаровування поту

Конвекція

Випромінювання

2706 / 4299
Наприкінці зими студент, який останнім часом відзначав нервове перенапруження, після переохолодження захворів на гостре респіраторне захворювання. Що є причиною захворювання?

Гіповітаміноз

Нераціональне харчування

Нервове перенапруження

Патогенний збудник

Переохолодження

2707 / 4299
У жінки 38-ми років розвинувся напад бронхіальної астми. Який з перерахованих бронхолітиків, ефективний для надання невідкладної допомоги, належить до групи β - 2 адреноміметиків?

Сальбутамол

Адреналін

Платифілін

Іпратропію бромід

Атропін

2708 / 4299
В шліфі зуба виявляється структура, в якій розрізняють чергування світлих і темних смуг, розташованих перпендикулярно до її поверхні, а також тонкі паралельні лінії росту. Яка це структура?

Безклітинний цемент

Пульпа

Дентин

Емаль

Клітинний цемент

2709 / 4299
Під час огляду хворого при проведенні аускультації лікар оцінює роботу мітрального клапана. Де вислуховують тон цього клапану?

У края груднини в другому міжребер’ї зліва

У края груднини зліва напроти хряща 5 ребра

У края груднини справа напроти хряща 5 ребра

На верхівці серця

У края груднини в другому міжребер’ї справа

2710 / 4299
У хворого встановлено діагноз - синдром Клайнфельтера. Каріотип при цьому захворюванні буде - (47, XXY). В цьому наборі буде така кількість статевих хромосом:

Дві

Сорок чотири

Одна

Нуль

Три

2711 / 4299
У хворого при огляді порожнини рота встановлено різкє почервоніння слизової оболонки кореня язика. Визначте, яке утворення залучене в запальний процес:

Піднебінний мигдалик

Піднебінна завіска

Язиковий мигдалик

Трубний мигдалик

Глотковий мигдалик

2712 / 4299
У травмованого рана скроневої ділянки, з якої яскраво-червоною цівкою втікає кров. Яка судина пошкоджена?

A. maxillaris

A. auricularis posterior

A. temporalis superficialis

A. facialis

A. occipitalis

2713 / 4299
При гістологічному дослідженні біоптату печінки жінки, що тривало страждає на вірусний гепатит В, виявлено: дифузний фіброз тканини печінки з утворенням фіброзних порто-портальних і порто-центральних септ і порушення часточкової будови печінки (поява несправжніх часточок). Для якого процесу характерні описані морфологічні зміни?

Холестаз

Кпатоцелюлярний рак

Гострий гепатит

Хронічний гепатит

Цироз печінки

2714 / 4299
При вивченні гістологічного препарату слизової оболонки ротової порожнини було виявлено, що багатошаровий плоский незроговілий епітелій інфільтрований лімфоцитами. Слизова оболонка якої ділянки ротової порожнини найбільш імовірно представлена на препараті?

Тверде піднебіння

Губа

Мигдалик

Ясна

Щока

2715 / 4299
До лікаря-стоматолога звернувся хворий зі скаргами на сухість в ротовій порожнині, як в стані спокою, так і під час прийому їжі. При обстеженні встановлено порушення секреторної діяльності під’язикової і підщелепної слинних залоз. В якому випадку може бути таке явище?

Пошкодження блокового нерва

Пошкодження окорухового нерва

Пошкодження вегетативних волокон барабанної струни лицевого нерва

Пошкодження язикоглоткового нерва

Пошкодження піднижньощелепного нерва

2716 / 4299
Під час обстеження лікар-невролог при ударі неврологічним молоточком по сухожилку нижче колінної чашечки оцінює рефлекторне розгинання в колінному суглобі. З подразненням яких рецепторів пов’язане виникнення цього рефлексу?

Ноцицептивні рецептори

М’язові веретена

Суглобові рецептори

Сухожилкові рецептори Іольджі

Тактильні рецептори

2717 / 4299
В експерименті вивчалися головні показники гемодинаміки. Який з нижче перерахованих показників гемодинаміки є однаковим для великого й малого кіл кровообігу?

Лінійна швидкість кровотоку

Опір кровотоку

Середній артеріальний тиск

Діастолічний артеріальний тиск

Об’ємна швидкість кровотоку

2718 / 4299
Чоловік 50-ти років на прийомі у стоматолога відмовився від знеболення. Після сильного болю у нього виникла анурія внаслідок різкого збільшення продукції:

Тимозину

Тироксину

Ілюкагону

Реніну

Адреналіну

2719 / 4299
Із щелепної кістки пацієнта 45-ти років оперативно видалено порожнисте утворення діаметром - 2,5 см, заповнене жовтуватою рідиною з кристалами холестерину, яке було розташоване у верхівці кореня каріозного зуба. При мікроскопічному дослідженні внутрішня поверхня порожнини вкрита багатошаровим плоским епітелієм з акантозом, без кератинізації. Стінка порожнини фіброзна, з кристалами холестерину, інфільтрована лімфоцитами, плазмоцитами, макрофагами, що резорбують жири. Який діагноз найбільш імовірний?

Кератокіста

Остеобластокластома

Амелобластома

Фолікулярна кіста

Радикулярна кіста

2720 / 4299
У пацієнта через 30 хвилин після лікування у стоматолога з’явилися червоні плями на шкірі обличчя і слизовій рота, що сверблять. Був встановлений діагноз: кропивниця. Яка з біологічно активних речовин, що викликають розширення судин, появу свербежу, виділяється при цьому типі алергічної реакції'?

Гістамін

Брадикінін

Інтерлейкін-1

Простагландин Е2

Лейкотрієн В4

2721 / 4299
У пацієнтки 26-ти років висипання на шкірі, свербіж після вживання цитрусових. Призначте лікарський засіб з групи блокаторів Н1-гістамінорецепторів:

Вікасол

Кислота ацетилсаліцилова

Анальгін

Димедрол

Парацетамол

2722 / 4299
Хворому на акне призначений до-ксицикліну гідрохлорид. Які застереження повинен дати лікар хворому при застосуванні цього препарату?

Запивати великою кількістю рідини, бажано молоком

Приймати перед їжею

Не приймати разом з вітамінними препаратами

Уникати тривалого перебування на сонці

Курс лікування не повинен перевищувати 1 день

2723 / 4299
Хворий на мегалобластну анемію приймав препарат з групи водорозчинних вітамінних засобів. Визначте цей препарат:

Ціанокобаламін

Тіаміну хлорид

Аскорбінова кислота

Піридоксин

Токоферолу ацетат

2724 / 4299
В матеріалі, взятому від хворої людини, знайдено декілька видів мікроорганізмів (стафілококи та стрептококи різних видів), які стали причиною захворювання. Як називається такий вид інфекції?

Змішана інфекція

Реінфекція

Коінфекція

Суперінфекція

Вторинна інфекція

2725 / 4299
У хворого після травми хребта спостерігається відсутність довільних рухів, сухожилкових рефлексів, чутливості тільки нижніх кінцівок. Який механізм порушень та у якому відділі хребта була травма?

Центральний параліч, куприковий відділ

Спінальний шок, грудний відділ

Периферичний параліч, шийний відділ

Спінальний шок, шийний відділ

2726 / 4299
У потерпілого від електротравми в ділянці шиї сформувався патологічний фіксований нахил голови у бік пошкодження у поєднанні з поворотом обличчя в протилежний бік. Який м’яз шиї піддався рубцевим змінам?

Передній драбинчастий

Трапецієподібний

Двочеревцевий

Під’язиково-лопатковий

Груднино-ключично-соскоподібний

2727 / 4299
Велику частину коронки, шийки і кореня зуба складає дентин, товщина якого з віком може збільшуватися, можливе також його часткове відновлення після пошкодження. Які структури забезпечують ці процеси?

Амелобласти

Цементобласти

Одонтобласти

Дентинні канальці

Перитубулярний дентин

2728 / 4299
Хронічне запалення тканин ясен завершилося надмірним розростанням волокон сполучної тканини. Які клітинні елементи відіграли в цьому процесі провідну роль?

Макрофаги

Фібробласти

Остеобласти

Фіброцити

Остеокласти

2729 / 4299
Хвора 20-ти років звернулася до лікаря зі скаргами на загальне схуднення, зниження апетиту, слабкість, появу незвичайного кольору шкіри, що нагадує південну 'бронзову засмагу'. При обстеженні у клініці, окрім гіперпігментації, виявлений двобічний туберкульоз наднирників. Надлишкове накопичення якої речовини зумовило гіперпігментацію шкіри?

Кмомеланін

Ліпофусцин

Меланін

Адренохром

Білірубін

2730 / 4299
Хворий скаржиться, що при згадуванні про минулі трагічні події в його житті, у нього виникають тахікардія, задишка і різкий підйом артеріального тиску. Які структури ЦНС забезпечують зазначені кардіореспіраторні реакції у даного хворого?

Мозочок

Кора великих півкуль

Чотиригорбкове тіло середнього мозку

Латеральні ядра гіпоталамуса

Специфічні ядра таламуса

2731 / 4299
Хворий звернувся з відчуттям серцебиття після стресу. ЧСС- 104/хв., тривалість інтервалу P — Q - 0,12 сек., QRS -без змін. Який тип аритмії у хворого?

Синусова тахікардія

Миготлива аритмія

Синусова брадикардія

Екстрасистолія

Синусова аритмія

2732 / 4299
Під час дослідження клітин було встановлено в їх цитоплазмі високий вміст ферменту аміноацил-тРНК-синтетаза. Він забезпечує в клітині такий процес:

Репарація

Транскрипція

Активація амінокислот

Елонгація

Реплікація

2733 / 4299
Який фермент має демінералізуючу дію - посилює розщеплення мінеральних компонентів тканин зуба?

Лужна фосфатаза

Глюкозо-6-фосфатаза

Кисла фосфатаза

Глікогенфосфорилаза

Фосфотрансфераза

2734 / 4299
У гістологічному препараті кіркової речовини нирок можна бачити ниркові тільця та канальці нефронів. Відомо, що в канальцях нефрону відбувається реабсорбція речовин. Яка тканина нефрону бере участь у цьому процесі?

Епітеліальна

Слизова

Ретикулярна

Хрящова

Власне сполучна

2735 / 4299
Хворий 45-ти років, що переніс лівосторонню крупозну пневмонію, загинув від множинних травм в результаті автотранспортної катастрофи. На розтині нижня частка лівої легені в області задньо-бокової стінки зрощена із грудною стінкою фіброзними спайками. Об’єм частки зменшений, вона щільна, на розрізі м’ясистого вигляду, сірувато- рожевого кольору, її шматочки тонуть у воді. При гістологічному дослідженні в цих ділянках відмічається дифузне розростання волокнистої сполучної тканини. Ускладнення крупозної пневмонії:

Ателектаз

Емфізема

Абсцес

Карніфікація

Гангрена

2736 / 4299
У дівчинки 16-ти років, яка тривалий час намагалась знизити масу свого тіла голодуванням, виник набряк. Яка головна причина цього явища?

Гіпоглікемія, зумовлена порушенням синтезу глікогену

Зменшення вироблення вазопресину в гіпоталамусі

Венозний застій і підвищення венозного тиску

Зменшення швидкості клубочкової фільтрації

Гіпопротеїнемія, зумовлена порушенням синтезу білків

2737 / 4299
У постраждалого з ножовим пораненням шиї кровотеча. При первинній обробці рани встановлено, що пошкоджена судина, яка розташована вздовж латерального краю груднино-ключично-соскоподібного м’яза. Визначте цю судину:

V. jugularis anterior

A. carotis externa

V. jugularis interna

V. jugularis externa

A. carotis interna

2738 / 4299
В еритроцитах пацієнта, хворого на гемолітичну анемію, була значно знижена активність піруваткінази. Який метаболічний процес порушений за цих умов?

Синтез глікогену

Гліколіз

Глікогеноліз

Глюконеогенез

Пентозофосфатний шлях окислення глюкози

2739 / 4299
У плазмі крові пацієнта підвищилась активність ізоферментів ЛДГ1 і ЛДГ2. Про патологію якого органа це свідчить?

Скелетні м’язи

Мозок

Міокард

Печінка

Нирки

2740 / 4299
Для визначення функціонального стану печінки у хворого досліджували екскрецію тваринного індикану у сечі, який утворюється при детоксикації продуктів гниття амінокислоти в товстій кишці. Назвіть цю амінокислоту:

Серин

Цистеїн

Валін

Триптофан

Гліцин

2741 / 4299
При вивченні порівняльної радіочутливості тканин була виявлена неоднакова їх чутливість до дії іонізуючого випромінювання. Яка з перерахованих тканин є найбільш радіочутливою?

М’язова

Нервова

Хрящова

Кісткова

Кровотворна

2742 / 4299
Під час огляду порожнини рота на губній і язиковій поверхні зубів виявлені темно-жовті і коричневі плями і смуги, що займають більше половини поверхні зубів; емаль і дентин зруйновані. Який найбільш імовірний діагноз?

Дистрофічне звапнування

Зубний камінь

Клиноподібні дефекти зубів

Флюороз

Метастатичне звапнування

2743 / 4299
На поздовжньому шліфі зуба видно тканину, що утворює основу зуба і складається з колагенових волокон, мінералізованого матриксу і трубочок, в яких проходять відростки дентинобластів. З чого розвивається представлена тканина?

Внутрішні клітини емалевого органа

Зовнішні клітини емалевого органа

Зубний мішечок

Периферійна частина зубного сосочка

Проміжні клітини емалевого органа

2744 / 4299
В експерименті необхідно вивчити процес збудження у м’язі. З цією метою необхідно зареєструвати:

Силу скорочення

Концентрацію іонів

Механоміограму

Тривалість скорочення

Електроміограму

2745 / 4299
При видаленні верхнього лівого третього моляра виникла значна кровотеча. Ураження якої артерії призвело до кровотечі?

A. alveolaris superioris posterioris

A. alveolaris inferior

A. facialis

A. alveolaris superioris anterioris

A. infraobitalis

2746 / 4299
У пацієнта після операції з застосуванням апарату штучного кровообігу з’явилася гемоглобінурія, причиною якої може бути:

Термічний гемоліз

Механічний гемоліз

Хімічний гемоліз

Імунний гемоліз

Біологічний гемоліз

2747 / 4299
При збільшенні частоти стимуляції ізольованого серця кроля відмічається неповне розслаблення шлуночків серця внаслідок:

Пригнічення K — Na насосу

Накопичення кальцію у кардіоміоци-тах

Збільшення вмісту калію в інтерстиції

Збільшення вмісту натрію у кардіоміо-цитах

Збільшення вмісту калію у кардіоміо-цитах

2748 / 4299
Через декілька годин після травми зуба у пульпі спостерігається гіперемія судин, виражений набряк тканини з поодинокими нейтрофілами, лімфоцитами, у нервових волокнах - незначні дистрофічні зміни. Який найбільш імовірний діагноз?

Гранулюючий пульпіт

Гангренозний пульпіт

Серозний пульпіт

Гнійний пульпіт

Фіброзний пульпіт

2749 / 4299
Вади розвитку плоду можуть спричинити такі хвороби матері як краснуха, сифіліс, токсоплазмоз, цитомегалія, герпес, хламідіоз. До якої форми мінливості відносять такі вади розвитку?

Модифікаційна

Мутаційна

Геномного імпринтингу

Епімутаційна

Комбінативна

2750 / 4299
Захворювання Бері-Бері - це класична форма недостатності вітаміну тіаміну. Активна форма його синтезується за допомогою ферменту з класу:

Трансфераз

Ізомераз

Ліаз

Гідролаз

Оксидоредуктаз

2751 / 4299
У чоловіка 25-ти років після ушкодження периферичних нервів втратились всі види чутливості. Як називається цей вид порушення?

Гіперстезія

Анестезія

Атаксія

Гіпостезія

2752 / 4299
У жінки 60-ти років з цирозом печінки виник геморагічний синдром. Який механізм це зумовив?

Зменшення запасів глікогену в печінці

Підвищення тиску в системі ворітної вени

Поява у крові нейротоксичних речовин

Зниження онкотичного тиску в крові

Зменшення синтезу протромбіну і фібриногену

2753 / 4299
Сироватка крові хворого має молочний вигляд. При біохімічному дослідженні виявлено високий рівень триа-цилгліцеролів і хіломікронів. Спадковий дефект якого ферменту викликає цей стан?

Фосфодіестераза

Панкреатична ліпаза

Тканинна гормон-чутлива ліпаза

Ліпопротеїнліпаза

Фосфоліпаза

2754 / 4299
Для лікування хвороби Паркінсо-на застосовують попередник дофаміну - ДОФА. З якої амінокислоти утворюється ця активна речовина?

Тирозин

Аланін

Гістидин

Цистеїн

Триптофан

2755 / 4299
У чоловіка гнійна рана у ділянці соскоподібного відростка черепа, внаслідок чого розвинулося запалення оболонок головного мозку. Яким шляхом проникла інфекція в порожнину черепа?

V emissariae mastoidea

Vv tympanicae

Vv labirinthi

V facialis

V auricularis

2756 / 4299
При лабораторному обстеженні крові людини, яку вкусила змія, виявлено гемоліз еритроцитів, гемоглобінурію. Дія зміїної отрути зумовлена наявністю в ній ферменту:

Сфінгомієліназа

Фосфоліпаза А2

Фосфоліпаза С

Фосфоліпаза А1

Фосфоліпаза Д

2757 / 4299
У людини виявлена пухлина одного з відділів головного мозку, внаслідок чого в неї порушена здатність підтримувати нормальну температуру тіла. Яка структура головного мозку пошкоджена?

Гіпоталамус

Стріатум

Мозочок

Таламус

Чорна субстанція

2758 / 4299
В експерименті досліджували поріг подразнення тактильних рецепторів різними подразниками. Для якого з наведених подразників поріг буде найменшим?

Хімічний

Механічний

Тепловий

Холодовий

Світловий

2759 / 4299
У обстежуваного в II міжребер’ї по парастернальній лінії справа при аускультації краще прослуховується II тон, ніж I. Закриттям якого клапану зумовлено формування II тону?

Півмісяцевий клапан аорти

Півмісяцевий клапан легеневого стовбура

Правий трьохстулковий клапан

Лівий двостулковий клапан

Двостулковий та трьохстулковий клапани

2760 / 4299
Робота шахтарів у забої часто спричинює антракоз. Який вид дихальної недостатності може розвинутись при цьому?

Обструктивний

Торакальний

Дисрегуляторний

Рестриктивний

Діафрагмальний

2761 / 4299
Зародок ланцетника перебуває на одній із стадій розвитку, під час якої кількість його клітин збільшується, але загальний об’єм зародка практично не змінюється. На якій стадії розвитку знаходиться зародок?

Органогенезу

Нейруляції

Гаструляції

Гістогенезу

Дроблення

2762 / 4299
В гістологічному препараті ендометрію видно окремі епітеліальні клітини, в яких хромосоми формують ’’пластинку” що розташована в екваторіальній площині. В якому періоді клітинного циклу перебувають такі клітини?

Телофаза

Метафаза

Анафаза

Профаза

Інтерфаза

2763 / 4299
У пацієнта з атеросклерозом під час ультразвукового обстеження діагностовано двобічний стеноз ниркових артерій. Вкажіть, яка біологічно активна речовина є ключовою ланкою патогенезу артеріальної гіпертензії у даному випадку:

Ренін

Тироксин

Адреналін

Кортизол

Вазопресин

2764 / 4299
У 72-річного хворого з цирозом печінки розвинулась печінкова кома. Її розвиток зумовлений надходженням у загальний кровотік через портокавальні анастомози речовин, що знешкоджуються в печінці (синдром портальної гіпертензії) та некрозом гепатоцитів. Для якого виду печінкової коми це характерно?

Змішана

Печінково-клітинна

Шунтова

Паренхіматозна

Кетоацидотична

2765 / 4299
Чоловік в результаті ДТП втратив багато крові, свідомість затьмарена, низький кров’яний тиск. При цьому у нього компенсаторно активується ренін-ангіотензинова система, що призводить до:

Гіперпродукції вазопресину

Гіперпродукції альдостерону

Посилення еритропоезу

Посилення серцевих скорочень

Підвищення згортання крові

2766 / 4299
При лабораторному обстеженні у пацієнта виявлено дефіцит УДФ- глюкуронілтрансферази. Які показники крові є підтвердженням даної ензимопатії?

Кетоацидоз

Індиканурія

Гіпербілірубінемія

Фенілкетонурія

Уремія

2767 / 4299
При обстеженні хворого виявлено зниження секреторної функції залоз носової порожнини. Який нерв забезпечує парасимпатичну іннервацію даних залоз?

N. petrosus minor

N. chorda tympani

N. maxillaris

N. petrosus profundus

N. petrosus major

2768 / 4299
Хворий 67-ми років був доставлений в кардіологічне відділення зі скаргами на періодичні болі у серці, задишку при незначному фізичному навантаженні, ціаноз та набряки. При ЕКГ-обстеженні виявлені позачергові збудження шлуночків серця. Як називається таке порушення ритму?

Тріпотіння

Екстрасистолія

Брадикардія

Фібриляція

Тахікардія

2769 / 4299
Хвора 40-ка років знаходиться на лікуванні в терапевтичному відділенні. В температурному листі хворої відмічаються цикли підвищення температури, які чергуються з періодами її нормалізації, що тривають кілька діб. До якого із типів температурних кривих відносяться дані показники?

Febris continua

Febris intermittent

Febris remittens

Febris recurrens

2770 / 4299
В приймальне відділення доставили хворого з тепловим ударом. Які з наведених захисно- компенсаторних реакцій розвиваються при цьому?

Стійка гіперглікемія

Підвищення ЧСС

Звуження периферійних судин

Спазм вінцевих судин

Розширення периферійних судин

2771 / 4299
Клітини чутливих спинномозкових нервових вузлів входять до складу рефлекторних дуг. До якого типу нейронів вони відносяться?

Псевдоуніполярні

Уніполярні

Біполярні

Мультиполярні

2772 / 4299
У хворого мисливця через 2 дні після розділки тушки ховраха підвищилась температура тіла до 39°C, збільшились лімфовузли. У подальшому у нього розвинулася пневмонія з серозно-геморагічним ексудатом, у якому виявлені овоїдної форми мікроорганізми з біполярним забарвленням. Який діагноз можна припустити у даного хворого?

Правець

Чума

Псевдотуберкульоз

Бруцельоз

Сибірка

2773 / 4299
У піддослідної тварини досліджували види скорочення м’язів травного тракту та виявили різну їх функціональну спрямованість. Було встановлено, що лише один тип рухової активності здійснюється циркуляторним та повздовжніми м’язами. Назвіть його:

Маятникоподібне скорочення

Жування

Тонічне скорочення сфінктерів

Перистальтика

Ритмічна сегментація

2774 / 4299
У дитини рана позаду соскоподібного відростка черепа. З рани витікає яскраво-червона кров. Гілки якої артерії пошкоджені?

A. maxillaris

A. temporalis superior

A. carotis externa

A. carotis interna

A. occipitalis

2775 / 4299
An 84-year-old patient suffers from parkmsomsm. One of the pathogenetic development elements of tMs disease is defidency of a certam mediator іп some of the bram structures. Name tMs mediator:

Dopamine

Acetylcholine

Adrenaline

Histamine

Noradrenaline

2776 / 4299
Several patients whh smtiar complati nts came to the doctor. They all presented whh weakness, pam іп the abdomen, mdti gestion. Stool ana^s revealed the need for urgent hosphahzation of the patient, who had microbialcysts whh four nuclei detected іn his samples. Such cysts are characteristic of the followmg protozoon:

Lamblia giardia

Entamoeba coli

Trichomonas hominis

Balantidium coli

Entamoeba histolytica

2777 / 4299
After farial trauma the patient developed a buccal hematoma. What sativatory gland has hs outflow blocked by the hematoma?

Sublingual

Parotid

Lingual

Buccal

Submandibular

2778 / 4299
A woman complains of painful chewing, especially when she moves her lower jaw backwards. What muscles are affected?

Lateral pterygoid muscles

Posterior bundles of the temporal muscles

Medial pterygoid muscles

Anterior bundles of the temporal muscles

Masseter muscles

2779 / 4299
Due to sustained trauma the patient presents with unevenly dilated pupils (ani-socoria). What muscle is blocked?

Musculus rectus superior

Musculus sphincter pupillae

Musculus dilatator pupillae

Musculus ciliaris

Musculus rectus inferior

2780 / 4299
Microphotogram made with electron microscope shows alveolar cells that form blood-air barrier. Name this cells:

Villous epithelial cells

Clara cells (club cells)

Alveolar respiratory epithelial cells

Alveolar secretory epithelial cells

Alveolar macrophages

2781 / 4299
During an experiment, a myotome was destroyed in a rabbit fetus. This manipulation will result in malformation of the following structure:

Skeletal muscles

Dermal connective tissue

Smooth muscles

Axial skeleton

Serous membranes

2782 / 4299
The specimen shows a tissue with cells that are located separately and in isogroups; no fibrous structures can be detected within its intercellular substance. What tissue is presented by this specimen?

Epithelial tissue

Hyaline cartilaginous tissue

Fibrous cartilaginous tissue

Bone tissue

Smooth muscular tissue

2783 / 4299
After inhalation of dust a person develops cough, which results from stumulation of:

Irritant receptors

Nociceptors

Pulmonary chemoreceptors

Pulmonary thermoreceptors

Juxtacapillary receptors

2784 / 4299
An experimental animal, a dog, received a weak solution of hydrochloric acid through a tube inserted into the duodenum. Primarily it will result in increased secretion of the following hormone:

Histamine

Secretin

Neurotensin

Gastrin

Cholecystokinin

2785 / 4299
A patient suffers from diabetes melli-tus with fasting hyperglycemia over 7.2 mmol/L. What blood plasma protein would allow to assess the patient’s glycemia level retrospectively (4-l2 weeks prior to examination)?

C-reactive protein

Fibrinogen

Glycated hemoglobin

Albumin

Ceruloplasmin

2786 / 4299
The patient exhausted by starvation presents with intensification of the following process in the liver and kidneys:

Urea synthesis

Uric acid synthesis

Bilirubin synthesis

Hippuric acid synthesis

Gluconeogenesis

2787 / 4299
A diabetus mellitus patient developed unconsciousness and convulsions after administration of insulin. What result of blood glucose analysis is the most likely in this case?

3.3 mmol/L

8 mmol/L

5.5 mmol/L

1.5 mmol/L

10 mmol/L

2788 / 4299
In the process of an experiment, a rabbit received guinea pig antikidney serum. What renal disease was modeled in this experiment?

Acute diffuse glomerulonephritis

Chronic pyelonephritis

Chronic renal failure

Acute pyelonephritis

Nephrotic syndrome

2789 / 4299
A 13-year-old girl is an in-patient at the hematology department of the regi- onal children’s hospital. She was diagnosed with iron-deficiency anemia. What type of hypoxia does this patient have?

Tissue

Hemic

Mixed

Circulatory

Respiratory

2790 / 4299
A 55-year-old man was diagnosed with acute glomerulonephritis. Name the main mechanism of anemia development in this case:

Decreased erythropoietin synthesis

Decreased synthesis of renal prostaglandins

Decreased glomerular filtration

Decreased tubular reabsorption

Renal azotemia

2791 / 4299
A man, who for a long time has been suffering from chronic mandibular osteomyelitis, died of chronic kidney disease. Autopsy revealed large lardaceous kidneys. What process had occurred in the kidneys?

Contracted kidney

Glomerulonephritis

Renal amyloidosis

Arterial nephrosclerosis

Necrotic nephrosis

2792 / 4299
On autopsy of a 69-year-old woman, who for a long time had been suffering from hypertension, the pathologist determined that both of her kidneys are dense, markedly diminished, with fine- grained surface. These changes are indicative of:

Compression atrophy

Senile renal atrophy

Dysfunctional atrophy

Hypoplasia

Atrophy due to inadequate blood supply

2793 / 4299
What drugs are used for specific treatment of diphtheria?

Antibiotics

Antitoxic serum

Anatoxin

Native plasma

Placental gamma globulin

2794 / 4299
An ophthalmologist suspects blennorrhea (gonococcal conjunctivitis) in a child with signs of suppurative keratoconjunctivitis. What laboratory diagnostics should be conducted to confirm the diagnosis?

Microscopy and bacteriological analysis

Microscopy and serum diagnostics

Serum diagnostics and allergy test

Biological analysis and phagodiagnostics

Biological analysis and allergy test

2795 / 4299
For two weeks a woman has been taking the mixture for neurasthenia, which was prescribed by a neurologist. Her general state slightly improved but shortly she started complaining of rhinitis, conjunctivitis, skin rashes, fatigue, and memory impairment. What group of drugs can have such a side effect?

Motherwort preparations

Bromine salts

Hop preparations

Adaptogens

Valerian preparations

2796 / 4299
A patient developed burning sensation in the oral cavity and white fuzzy coating on the tongue. Oral thrush is diagnosed. What drug of those listed below should be used?

Nystatin

Griseofulvin

Tetracycline

Amphotericin

Gentamicin

2797 / 4299
A patient suffering from ciliary arrhythmia with anamnesis of bronchial asthma should be prescribed an antiarrhythmic drug. What antiarrhythmic drug is CONTRAINDICATED in this case?

Verapamil

Ajmaline

Nifedipine

Anaprilin (Propranolol)

Novocainamide (Procainamide)

2798 / 4299
In the process of an experiment, a specimen of neuromuscular tissue of a frog had been processed with a curariform agent, which resulted in disappearance of muscle contractions in response to electrical stimulation of the nerve. What function of the muscle cell membrane is disturbed by curariform agents?

Mediator reception in the neuromuscular junction

Maintenance of the barrier between the cellular environment and interstitial fluid

Maintenance of the cell’s internal structure, its cytoskeleton

Maintenance of electric potentials on the both sides of the membrane

Different permeability to different substances

2799 / 4299
A patient was diagnosed with a monogenic hereditary disease. Name this disease:

Hypertension

Hemophilia

Peptic ulcer disease of the stomach

Poliomyelitis

Hymenolepiasis

2800 / 4299
A patient is diagnosed with frontitis. He also has a history of maxillary sinusitis. What structure of the nasal cavity is the most likely pathway through which the infection penetrated into the frontal sinus?

Inferior nasal meatus

Ethmoid and sphenoid sinus

Superior nasal meatus

Vestibule of the nose

Middle nasal meatus

2801 / 4299
В популяцiях людини у деяких осiб впродовж життя спостерiгається не двi, а три генерацiї зубiв. Це прояв такого закону:

Гомологiчних рядiв спадкової мiнливостi

Хардi-Вайнберга

Ембрiональної iндукцiї

Незалежного успадкування

Бiогенетичний

2802 / 4299
У хворого, що страждає впродовж тижня на пневмонiю, при мiкроскопуваннi харкотиння виявлено личинки гельмiнта. В кровi - еозинофiлiя. Про який дiагноз можна думати в цьому випадку?

Тенiоз

Аскаридоз

Ехiнококоз

Парагонiмоз

Фасцiольоз

2803 / 4299
На клiтину подiяли речовиною, яка спричинила порушення цiлiсностi мембран лiзосом. Що вiдбудеться з клiтиною внаслiдок цього?

Спецiалiзацiя

Трансформацiя

Диференцiацiя

Автолiз

Дегенерацiя

2804 / 4299
До гастроентеролога звернулась мама з дитиною 12-ти рокiв зi скаргами на зниження апетиту в дитини, метеоризм. При ендоскопiчному обстеженнi дiагностовано дискiнезiю жовчних проток, а у дуоденальному вмiстi виявлено найпростiшi грушоподiбної форми з двома ядрами та багатьма джгутиками. Яке захворювання найбiльш iмовiрно у дитини?

Лямблiоз

Токсоплазмоз

Трихомоноз

Амебiаз

Балантидiаз

2805 / 4299
Бiохiмiчний аналiз амiнокислотного складу щойно синтезованих полiпептидiв показав, що в процесi трансляцiї перша амiнокислота в кожному бiлку одна i та ж. Назвiть її:

Гiстидин

Метiонiн

Iзолейцин

Серин

Фенiлаланiн

2806 / 4299
У хворого скарги на гострий бiль в животi переймоподiбного характеру, частi позиви на дефекацiю, рiдкi кров’янистi випорожнення iз слизом. Лабораторним дослiдженням мазкiв фекалiй виявлено органiзми непостiйної форми, що мiстять еритроцити. Яке найбiльш iмовiрне захворювання?

Кишковий трихомоноз

Балантидiаз

Амебiаз

Лямблiоз

Шистосомоз

2807 / 4299
Рiст у людини контролюють кiлька неалельних домiнантних генiв. Встановлено, що при збiльшеннi кiлькостi цих генiв рiст збiльшується. Який тип взаємодiї мiж цими генами?

Комплементарнiсть

Епiстаз

Кодомiнування

Полiмерiя

Плейотропiя

2808 / 4299
У медико-генетичну консультацiю звернулась мати 2-рiчної дитини з вiдставанням у фiзичному i розумовому розвитку. Який метод дослiдження дозволить виключити хромосомну патологiю?

Цитогенетичний

Цитологiчний

Популяцiйно-статистичний

Бiохiмiчний

Генеалогiчний

2809 / 4299
На заняттях з молекулярної бiологiї йде розгляд мутацiй, якi призводять до утворення аномального гемоглобiну. Яка замiна амiнокислот вiдбувається при утвореннi S-гемоглобiну, що спричиняє виникнення серпоподiбноклiтинної анемiї?

Глiцин аспарагiном

Глутамiнова кислота валiном

Треонiн лiзином

Гiстидiн аргiнiном

Лiзин глутамiном

2810 / 4299
Людям, що мешкають в умовах арктичного клiмату, незалежно вiд їхньої раси, притаманний ряд пристосувань до умов iснування. Для представникiв арктичного адаптивного типу порiвняно з корiнними жителями Центральної Африки характерним є:

Бiльша довжина нiг, нiж рук

Худорляве тiло

Менша потреба в жирах

Пiдвищений шар пiдшкiрного жиру

Пiдвищення потовидiлення

2811 / 4299
Зародок ланцетника перебуває на однiй iз стадiй розвитку, пiд час якої кiлькiсть його клiтин збiльшується, але загальний об’єм зародка практично не змiнюється. На якiй стадiї розвитку знаходиться зародок?

Гаструляцiї

Нейруляцiї

Органогенезу

Гiстогенезу

Дроблення

2812 / 4299
Турист, який перебував в однiй з країн Далекого Сходу, госпiталiзований у терапевтичне вiддiлення з пiдозрою на запалення легень. Пiд час дослiдження харкотиння та фекалiй виявлено яйця легеневого сисуна. Внаслiдок вживання яких продуктiв збудник цiєї хвороби мiг потрапити до органiзму хворого?

Сирi овочi i фрукти 

Недостатньо термiчно оброблена свинина

Недостатньо термiчно оброблена яловичина

Термiчно необробленi прiсноводнi краби

Недостатньо термiчно обробленi яйця

2813 / 4299
до хiрурга з варикозним розширенням вен лiвої ноги. Вузли вен розташованi на заднiй поверхнi шкiри гомiлки, на заднiй та переднiй поверхнях шкiри стегна. Якi поверхневi вени нижньої кiнцiвки пошкодженi у хворого?

Пiдколiнна, поверхнева пiдшкiрна вени

Задня великогомiлкова вена, велика пiдшкiрна вена

Мала пiдшкiрна вена, глибока вена стегна

Стегнова вена, велика та мала пiдшкiрнi вени

Велика та мала пiдшкiрнi вени

2814 / 4299
У пацiєнта аспермiя. Який орган не працює?

Простата

Придаток яєчка

Яєчко

Передмiхурова залоза

Сiм’янi мiхурцi

2815 / 4299
Хворiй дитинi 10 днiв вiд народження провели операцiю з приводу розщеплення верхньої губи (”заяча губа”). Розщеплення верхньої губи є наслiдком незрощення:

Лобового i верхньощелепних вiдросткiв I-ої зябрової дуги

Третьої зябрової дуги

Пiднебiнних валикiв верхньощелепних вiдросткiв I-ої зябрової дуги

Верхньо- i нижньощелепних вiдросткiв I-ої зябрової дуги

Другої зябрової дуги

2816 / 4299
У хворого пiсля застудного захворювання з’явилося порушення больової i температурної чутливостi переднiх 2/3 язика. Який iз нервiв при цьому постраждав?

Трiйчастий

Блукаючий

Барабанна струна

Дiафрагмальний

Пiд’язиковий

2817 / 4299
Чоловiк 42-х рокiв звернувся до медпункту з приводу рiзаної рани нижньої частини передньої поверхнi плеча. Об’єктивно: утруднене згинання передплiччя. Якi з наведених м’язiв ймовiрно ушкодженi у хворого?

M. biceps brachii, m. anconeus

M. deltoideus, m. biceps brachii

M. brachialis, m. biceps brachii

M. coracobrachialis, m. supraspinatus

M. deltoideus, m. infraspinatus

2818 / 4299
При операцiї на шлунку хiрург пересiк лiву шлункову артерiю, перев’язав її, але кров продовжує витiкати з протилежного кiнця артерiї. Яка артерiя анастомозує з нею?

Права шлункова

Селезiнкова

Права шлунково-чепцева

Верхня пiдшлунково-дванадцятипала

Лiва шлунково-чепцева

2819 / 4299
Хвору прооперовано з приводу стегнової кили. Де проектується вихiдний отвiр цiєї кили?

Пахвинна дiлянка

Сiднична дiлянка

Лобкова дiлянка

Стегновий трикутник

2820 / 4299
На прийом до лiкаря прийшов хворий високого росту, з вiдвислою нижньою губою i великим носом, та з великими кiнцiвками. Пiдвищення секрецiї якої залози можна запiдозрити у цього хворого?

Передня частка гiпофiза

Щитоподiбна

Прищитоподiбнi

Епiфiз

2821 / 4299
При операцiї на шиї з правого боку порушилася екскурсiя правого склепiння дiафрагми. Це сталося в результатi пошкодження такого нерва:

Правий дiафрагмальний

Лiвий дiафрагмальний

Надключичний нерв

Правий поперечний нерв шиї

Лiвий поперечний нерв шиї

2822 / 4299
У хворого при оглядi порожнини рота встановлено рiзке почервонiння слизової оболонки кореня язика. Визначте, яке утворення залучене в запальний процес:

Язиковий мигдалик

Глотковий мигдалик

Пiднебiнний мигдалик

Пiднебiнна завiска

Трубний мигдалик

2823 / 4299
При обстеженнi у хворої виявлений заглотковий абсцес. В який простiр шиї повинен проникнути хiрург при розтинi абсцесу?

Мiждрабинчастий

Надгруднинний

Передвiсцеральний

Позавiсцеральний

Переддрабинчастий

2824 / 4299
У хворого флегмона у дiлянцi крилопiднебiнної ямки з ураженням однойменного вегетативного вузла. Функцiя якої залози буде порушена у першу чергу?

Пiд’язикова

Навколовушна

Слiзна

Пiднижньощелепна

2825 / 4299
У хворого травма у дiлянцi передньої поверхнi переднього драбинчастого м’яза. Функцiя якого нерва може бути порушена?

Зворотнiй гортанний

Додатковий

Блукаючий

Дiафрагмальний

Плечового сплетення

2826 / 4299
При оглядi ротової порожнини хворого виявлено карiєс коронки зуба, яка звернена до власне ротової порожнини. Яка поверхня уражена?

Facies lingualis

Facies distalis

Facies mesialis

Facies vestibularis

Facies contactus

2827 / 4299
У хворого дiагностовано пошкодження мiжхребцевого диска в поперековому вiддiлi хребта. До якого виду сполучення кiсток належить це з’єднання?

Синостоз

Синдесмоз

Гемiартроз

Дiартроз

Синхондроз

2828 / 4299
Хворий скаржиться на сильний нежить та втрату вiдчуття запахiв. Де в носовiй порожнинi пошкодженi рецептори нюхового аналiзатору?

Хоани

Середнiй носовий хiд

Верхнiй носовий хiд

Загальний носовий хiд

Нижнiй носовий хiд

2829 / 4299
Студент перкуторно визначає межу серця, яка проектується на передню грудну клiтку на рiвнi хрящiв третього ребра. Яку межу серця визначив студент?

Нижня

Права

Верхня

Лiва

Верхiвка

2830 / 4299
У чоловiка гнiйна рана у дiлянцi соскоподiбного вiдростка черепа, внаслiдок чого розвинулося запалення оболонок головного мозку. Яким шляхом проникла iнфекцiя в порожнину черепа?

V.v. labirinthi

V. emissariae mastoidea

V. facialis

V. auricularis

V.v. tympanicae

2831 / 4299
У постраждалого пошкоджена кам’яниста частина скроневої кiстки. Лiнiя перелому пройшла позаду вiд внутрiшнього слухового отвору. Який канал скроневої кiстки був пошкоджений?

М’язово-трубний

Каналець барабанної струни

Барабанний

Лицевий

Сонний

2832 / 4299
Рентгенологiчно встановлено скупчення гною у клиноподiбнiй пазусi. В який носовий хiд видiляється гнiй?

Лiвий нижнiй

Правий середнiй

Лiвий середнiй

Правий нижнiй

Правий та лiвий верхнi

2833 / 4299
Результати обстеження хворого з порушенням слиновидiлення привушною залозою вказують на можливiсть ушкодження вушного вузла. Який нерв утворює цей вузол?

N. auricularis magnus

N. vagus

N. hypoglossus

N. petrosus major

N. petrosus minor

2834 / 4299
Чоловiку видалили зуб. Коронка зуба долотоподiбна, широка, край вузький. Корiнь конусоподiбний, стиснутий з бокiв. Який зуб видалили?

Нижнiй рiзець

Нижнє iкло

Нижнiй премоляр

Верхнiй премоляр

Верхнiй рiзець

2835 / 4299
Хворий не може пiдняти опущену нижню щелепу. Якi м’язи голови НЕ МОЖУТЬ виконати свою функцiю?

Верхнi вушнi м’язи

Малi виличнi

Великi виличнi

Щiчнi

Жувальнi

2836 / 4299
мазку червоного кiсткового мозку людини серед клiтин мiєлоїдного ряду та адипоцитiв зустрiчаються клiтини зiрчастої форми з оксифiльною цитоплазмою, якi контактують своїми вiдростками. Якi це клiтини?

Ретикулярнi

Дендритнi клiтини

Остеоцити

Фiбробласти

Макрофаги

2837 / 4299
На мiкропрепаратi з контурами бобоподiбного органу спостерiгається кiркова та мозкова речовина. Кiркова речовина представлена окремими кулястими вузликами дiаметром 0,5-1 мм, а мозкова - мозковими тяжами. З якого органа зроблено гiстологiчний зрiз?

Селезiнка

Тимус

Лiмфатичний вузол

Нирки

Наднирник

2838 / 4299
Сполучна тканина побудована з паралельно розташованих колагенових волокон, розмежованих фiбробластами. Цей тип сполучної тканини називається:

Пухка

Щiльна оформлена

Щiльна неоформлена

Слизова

Ретикулярна

2839 / 4299
На мiкропрепаратi серця розрiзняємо клiтини прямокутної форми з центрально розташованим ядром, розвиненими мiофiбрилами, зв’язанi мiж собою вставними дисками. З цими клiтинами пов’язана така функцiя серця:

Скорочення

Регенераторна

Ендокринна

Захисна

Проведення iмпульсу

2840 / 4299
У хворої набряки. У сечi велика кiлькiсть бiлка. Про порушення функцiї якого вiддiлу нефрону це свiдчить?

Дистальний звивистий каналець

Ниркове тiльце

Проксимальний звивистий каналець

Низхiдна частина петлi Генле

Висхiдна частина петлi Генле

2841 / 4299
У юнака 17-ти рокiв взяли катетером сечу з сечового мiхура для дослiдження. Клiтини якого епiтелiю, що вистеляє сечовий мiхур, можуть бути виявленi при мiкроскопiї осаду сечi?

Багатошаровий незроговiлий

Одношаровий кубiчний

Одношаровий призматичний

Перехiдний

Багатошаровий зроговiлий

2842 / 4299
У препаратi щитоподiбної залози при обробцi солями срiбла видно великi аргiрофiльнi клiтини, якi розташованi в стiнцi фолiкулiв. Який гормон синтезується даними клiтинам?

Кальцитонiн

Паратирин

Тироксин

Альдостерон

Адреналiн

2843 / 4299
На гiстологiчному препаратi представленi три нейрони: псевдоунiполярний, бiполярний та мультиполярний. Скiльки аксонiв можливо визначити у кожного з перерахованих типiв клiтин?

Багато

Один

Три

Два

Жодного

2844 / 4299
Дослiджено ембрiональний орган, в якому формуються першi форменi елементи кровi як тканини. Назвiть цей орган:

Селезiнка

Тимус

Печiнка

Червоний кiстковий мозок

Жовтковий мiшок

2845 / 4299
При вивченнi гiстологiчного препарату слизової оболонки ротової порожнини було виявлено, що багатошаровий плоский незроговiлий епiтелiй iнфiльтрований лiмфоцитами. Слизова оболонка якої дiлянки ротової порожнини найбiльш iмовiрно представлена на препаратi?

Ясна

Мигдалик

Тверде пiднебiння

Губа

Щока

2846 / 4299
Вiдомо, що фiброзно-хрящова оболонка трахеї складається з незамкнених на заднiй стiнцi кiлець гiалiнового хряща. Яка тканина з’єднує їх вiльнi кiнцi?

Жирова сполучна

Щiльна неоформлена волокниста сполучна

Гладка м’язова

Пухка волокниста сполучна

Посмугована м’язова

2847 / 4299
На мiсцi механiчного пошкодження шкiри завдяки процесам регенерацiї формується пухка волокниста сполучна тканина. Якi клiтини продукують бiлки (колаген, еластин) та компоненти мiжклiтинної речовини?

Адипоцити

Тканиннi базофiли

Фiбробласти

Макрофаги

Плазмоцити

2848 / 4299
На гiстологiчному препаратi виявлено клiтини, якi утворюють iзогеннi групи, у мiжклiтиннiй речовинi виявлено глiкопротеїни, протеоглiкани та колагеновi волокна. Яку тканину виявлено?

Слизова

Бура жирова

Кiсткова

Хрящова

Бiла жирова

2849 / 4299
У хворого на хронiчний гепатит в аналiзi кровi на бiлковi фракцiї виявили зниження загальної кiлькостi бiлка. Це означає, що у клiтинах печiнки порушена функцiя таких органел:

Комплекс Гольджi

Гранулярна ендоплазматична сiтка

Лiзосоми

Мiтохондрiї

Цитоскелет

2850 / 4299
На гiстологiчному препаратi слизової оболонки органу визначається багатошаровий епiтелiй, що складається з 20-25 клiтинних шарiв, поверхневi клiтини мають плоску форму. Який це орган?

Дванадцятипала кишка 

Дно шлунка

Стравохiд

Тонка кишка

Товста кишка

2851 / 4299
на стартi перед змаганнями вiдмiчається пiдвищення артерiального тиску i частоти серцевих скорочень. Впливом яких вiддiлiв ЦНС можливо пояснити вказанi змiни?

Середнiй мозок

Кора великих пiвкуль

Промiжний мозок

Довгастий мозок

Гiпоталамус

2852 / 4299
У пацiєнта вiдмiчена висока концентрацiя вазопресину (АДГ) у кровi. До яких змiн дiурезу це призведе?

Полiурiя

Анурiя

Глюкозурiя

Олiгоурiя

Натрiйурiя

2853 / 4299
Хворий скаржиться на тривалу кровотечу навiть при незначному травматичному пошкодженi. Лабораторний аналiз показав порушення складу кровi, а саме зменшення кiлькостi таких формених елементiв:

Моноцити

Тромбоцити

Еритроцити

Лiмфоцити

Нейтрофiли

2854 / 4299
В експериментi у тварини подразнювали периферiйний вiдрiзок симпатичного нерва, що iннервує пiд’язикову слинну залозу. При цьому залоза видiляє:

Багато рiдкої слини

Мало рiдкої слини

Мало в’язкої слини

Слина не видiляється

Багато в’язкої слини

2855 / 4299
У спортсмена пiсля перевантаження пiд час тренування виникла м’язова контрактура. При цьому м’яз втрачає гнучкiсть та поступово стає твердим, бо не має можливостi розслабитися. Вкажiть iмовiрну причину контрактури:

Змiни у структурi тропомiозину

Пiдвищення молочної кислоти у кровi

Недостатнiсть АТФ

Збiльшення K+ у кровi

Зниження Ca++ у кровi

2856 / 4299
У потерпiлого з травмою грудної клiтки рiзко погiршується стан: наростає задуха, зблiднення обличчя, тахiкардiя. Що може бути причиною вказаних розладiв?

Пневмоторакс

Переляк

Перелом ребер

Реакцiя на больовий подразник

Забiй грудної клiтки

2857 / 4299
В експериментi при вивченнi процесiв всмоктування продуктiв гiдролiзу їжi i води було встановлено, що основним вiддiлом шлунково-кишкового тракту, де вiдбуваються цi процеси, є:

Тонка кишка

Ротова порожнина

Пряма кишка

Шлунок

Товста кишка

2858 / 4299
Вимушенi пози, яких набуває людина при захворюваннях внутрiшнiх органiв (наприклад, згинання i приведення нижнiх кiнцiвок до живота), формуються внаслiдок реалiзацiї таких рефлексiв:

Вiсцеромоторнi

Вiсцеродермальнi

Дерматовiсцеральнi

Вiсцеровiсцеральнi

Моторновiсцеральнi

2859 / 4299
У жiнки 30-ти рокiв з’явилися ознаки вiрилiзму (рiст волосся на тiлi, облисiння скронь, порушення менструального циклу). Гiперпродукцiя якого гормону може спричинити такий стан?

Релаксин

Пролактин

Окситоцин

Естрiол

Тестостерон

2860 / 4299
У значної кiлькостi пацiєнтiв перед вiдвiдуванням стоматолога виникає тривожнiсть, страх, пригнiчений настрiй. Посилення секрецiї якого медiатора центральної нервової системи може зменшити цi змiни психо-емоцiйного стану у людини?

Ацетилхолiн

Дофамiн

ГАМК

Норадреналiн

Серотонiн

2861 / 4299
Пiд час роботи лiкарю-стоматологу доводиться довго стояти на ногах, що може призвести до застою кровi у венах нижнiх кiнцiвок i до їх варикозного розширення. Провiдним механiзмом формування застою у даному випадку є зменшення:

Скорочення скелетних м’язiв нижнiх кiнцiвок

Градiєнта тиску кровi у венозних судинах

Присмоктувального ефекту грудної клiтки

Присмоктувально-насосного ефекту дiафрагми на органи черевної порожнини

Залишкової рушiйної сила серця

2862 / 4299
У потерпiлого в автомобiльнiй катастрофi гематома спинного мозку супроводжується загруднинними болями, тахiкардiєю i пiдвищенням артерiального тиску. Ураження яких сегментiв спинного мозку є причиною стану хворого?

C6-C8

L1-L3

S1-S3

Th1-Th5

2863 / 4299
В експериментi при електричному подразненнi блукаючого нерва збiльшується вихiд в синаптичну щiлину ацетилхолiну, що зменшує ЧСС через наступний механiзм:

Зменшення тривалостi потенцiалу дiї

Гiперполяризацiя мембрани кардiомiоцитiв

Збiльшення тривалостi потенцiалу дiї

Збiльшення швидкостi проведення збудження в АВ-вузлi

Деполяризацiя мембрани кардiомiоцитiв

2864 / 4299
В медичнiй практицi широкого застосування набули курареподiбнi речовини (мiорелаксанти). Про яку побiчну дiю повинен пам’ятати лiкар при їх застосуваннi?

Напади судом

Розслаблення дихальних м’язiв

Тромбоутворення

Розлади мозкового кровообiгу

Зупинка серця

2865 / 4299
Людина в спекотну погоду тривалий час була позбавлена можливостi пиття, що спричинило виражене вiдчуття спраги. Змiна якого гомеостатичного показника кровi стала першопричиною цього?

Осмотичний тиск плазми

Рiвень глюкози

Онкотичний тиск плазми

Гемактокрит

рН

2866 / 4299
У людини виявлена пухлина одного з вiддiлiв головного мозку, внаслiдок чого в неї порушена здатнiсть пiдтримувати нормальну температуру тiла. Яка структура головного мозку пошкоджена?

Стрiатум

Гiпоталамус

Мозочок

Таламус

Чорна субстанцiя

2867 / 4299
Обмеження споживання води призвело до зневоднення органiзму. Який механiзм активується за цих умов для збереження води в органiзмi?

Зменшення секрецiї вазопресину

Збiльшення секрецiї соматостатину

Збiльшення секрецiї вазопресину

Збiльшення секрецiї альдостерону

Зменшення секрецiї альдостерону

2868 / 4299
Пiд час операцiї для знерухомлення пацiєнта використовують курареподiбнi фармакологiчнi препарати. Механiзм їх дiї полягає у блокуваннi:

Проведення збудження нервовими волокнами

Видiлення ацетилхолiну у синаптичну щiлину

Н-холiнорецепторiв скелетних м’язiв

М-холiнорецепторiв гладких м’язiв

Видiлення норадреналiну у синаптичну щiлину

2869 / 4299
антикоагулянтiв використовують рiзноманiтнi речовини, в тому числi полiсахарид природного походження, а саме:

Декстран

Хондроiтинсульфат

Гiалуронова кислота

Дерматансульфат

Гепарин

2870 / 4299
У хворого спостерiгається остеопороз кiсток, в кровi - гiперкальцiємiя, гiпофосфатемiя. Яка причина такого стану?

Посилена секрецiя тироксину

Пригнiчення секрецiї кортикостероїдiв

Пригнiчення секрецiї паратгормону

Посилена секрецiя кортикостероїдiв

Посилена секрецiя паратгормону

2871 / 4299
Окисне декарбоксилювання пiровиноградної кислоти каталiзується складним полiферментним комплексом з участю кiлькох функцiонально зв’язаних коферментiв. Вкажiть цей комплекс:

КоА-SН, ФАД, ПАЛФ, ТГФК, карнiтин

ТДФ, ФАД, КоА-SН, НАД, лiпоєва кислота

ФАД, ТГФК, ПАЛФ, ТДФ, холiн

Лiпоєва кислота, ТГФК, ПАЛФ, метилкобаламiн

НАД, ПАЛФ, ТДФ, метилкобаламiн, бiотин

2872 / 4299
Вiдомо, що в метаболiзмi катехоламiнових медiаторiв особлива роль належить ферменту моноамiноксидазi (МАО). Яким шляхом цей фермент iнактивує медiатори (норадреналiн, адреналiн, дофамiн)?

Окисне дезамiнування

Карбоксилювання

Видалення метильної групи

Гiдролiз

Приєднання амiногрупи

2873 / 4299
Хворому на туберкульоз легень призначено рифампiцин, який пригнiчує фермент РНК-полiмеразу на стадiї iнiцiацiї такого процесу:

Термiнацiя

Транскрипцiя

Трансляцiя

Елонгацiя

Реплiкацiя

2874 / 4299
При дослiдженнi на лабораторних щурах властивостей одного з вiтамiнiв при його дефiцитi спостерiгалося одночасне порушення репродуктивної функцiї та дистрофiя скелетних м’язiв. Про який вiтамiн йдеться?

D

K

E

B2

A

2875 / 4299
У кровi хворого виявили С-реактивний бiлок, який за хiмiчною природою є глiкопротеїном. Про яку патологiю це свiдчить?

Порфiрiя

Лейкопенiя

Ревматизм

Анемiя

Тромбоцитопенiя

2876 / 4299
У 8-ми мiсячної дитини спостерiгаються частi та сильнi пiдшкiрнi крововиливи. Призначення синтетичного аналога вiтамiну K (вiкасолу) дало позитивний ефект. Цей вiтамiн бере участь у γкарбоксилуваннi глутамiнової кислоти такого бiлка зсiдання кровi:

Фактор Хагемана

Фактор Розенталя

Протромбiн

Проконвертин

Фiбриноген

2877 / 4299
У дитини спостерiгається гепатомегалiя, гiпоглiкемiя, судоми, особливо натщесерце та при стресових ситуацiях. Дiагноз: хвороба Гiрке. Генетичний дефект якого фермента має мiсце при данiй хворобi?

Амiло-1,6-глiкозидаза

Глюкокiназа

Глiкогенфосфорилаза

Глюкозо-6-фосфатаза

Фосфоглюкомутаза

2878 / 4299
Формування великої кiлькостi iмуноглобулiнiв з рiзною антигенною специфiчнiстю з невеликої кiлькостi генiв вiдбувається внаслiдок:

Рекомбiнацiї генiв

Делецiї

Реплiкацiї

Транслокацiї

Транскрипцiї

2879 / 4299
Який фермент має демiнералiзуючу дiю - посилює розщеплення мiнеральних компонентiв тканин зуба?

Кисла фосфатаза

Глюкозо-6-фосфатаза

Фосфотрансфераза

Лужна фосфатаза

Глiкогенфосфорилаза

2880 / 4299
Для визначення функцiонального стану печiнки у хворого дослiджували екскрецiю тваринного iндикану у сечi, який утворюється при детоксикацiї продуктiв гниття амiнокислоти в товстiй кишцi. Назвiть цю амiнокислоту:

Валiн

Серин

Глiцин

Цистеїн

Триптофан

2881 / 4299
Синтез глюкози з невуглеводних компонентiв є важливим бiохiмiчним процесом. Глюконеогенез з амiнокислот найбiльш активно вiдбувається за умов бiлкового харчування. Яка амiнокислота з наведених є найбiльш глюкогенною?

Iзолейцин

Лiзин

Валiн

Аланiн

Лейцин

2882 / 4299
У дiвчинки 6-ти рокiв вираженi ознаки гемолiтичної анемiї. При бiохiмiчному аналiзi еритроцитiв встановлено дефiцит ферменту глюкозо-6фосфатдегiдрогенази. Порушення якого метаболiчного процесу вiдiграє головну роль у розвитку цiєї патологiї?

Тканинне дихання

Окисне фосфорилювання

Глюконеогенез

Анаеробний глiколiз

Пентозофосфатний шлях

2883 / 4299
Лiкар встановив у хворого наявнiсть генетичного дефекту лiпопротеїнлiпази. Що буде характерно для бiохiмiчного аналiзу кровi даного пацiєнта?

Гiпохiломiкронемiя

Гiперглiкемiя

Гiпертриацилглiцеролемiя

Гiпотриацилглiцеролемiя

Гiпоглiкемiя

2884 / 4299
У хворого з синдромом Iценко-Кушинга спостерiгається стiйка гiперглiкемiя та глюкозурiя. Синтез та секрецiя якого гормону збiльшенi у цього хворого?

Альдостерон

Тироксин

Кортизол

Адреналiн

Глюкагон

2885 / 4299
Юнак 25-ти рокiв звернувся до лiкаря зi скаргами на загальну слабкiсть, швидку втомлюванiсть, дратiвливiсть, зниження працездатностi, кровоточивiсть ясен. Недостатнiсть якого вiтамiну може мати мiсце у даному випадку?

Ретинол

Тiамiн

Рибофлавiн

Фолiєва кислота

Аскорбiнова кислота

2886 / 4299
У жiнки 37-ми рокiв було виявлено фруктоземiю, фруктозурiю. Вмiст глюкози у кровi - 2,1 ммоль/л. Дiагностовано непереносимiсть фруктози. Природжена недостатнiсть якого ферменту є молекулярною основою цього захворювання?

Фруктозо-1-фосфатальдолаза

Гексокiназа

Трiозофосфатiзомераза

Фосфофруктокiназа

Фосфоглюкомутаза

2887 / 4299
50-ти рокiв, який лiкувався вiд виразкової хвороби шлунка, нормалiзувалося травлення, зникли болi, полiпшився настрiй. Але через кiлька тижнiв знову з’явились болi в епiгастрiї, печiя, вiдрижка кислим. Як можна характеризувати такий перебiг хвороби?

Рецидив хвороби

Термiнальний стан

Латентний перiод

Перiод ремiсiї

Продромальний перiод

2888 / 4299
Пiсля тотальної резекцiї шлунка у хворого розвинулася тяжка B12-дефiцитна анемiя з порушенням кровотворення i появою в кровi змiнених еритроцитiв. Свiдченням її була наявнiсть в кровi:

Овалоцитiв

Мегалоцитiв

Нормоцитiв

Мiкроцитiв

Анулоцитiв

2889 / 4299
У хворого, що страждає на гiпертонiчну хворобу, виявленi добовi коливання загального периферичного опору судин току кровi. Якi судини беруть у цьому найбiльшу участь?

Вени

Артерiоли

Аорта

Капiляри

Артерiоло-венулярнi анастомози

2890 / 4299
При клiнiчному обстеженнi в жiнки встановлено: пiдвищення потовидiлення, тахiкардiя, схуднення, тремор. Яка ендокринна патологiя може це спричинити?

Гiпогонадизм

Гiпоальдостеронiзм

Гiпертиреоз

Гiпотиреоз

Гiпергонадизм

2891 / 4299
У хворого виявлено порушення прохiдностi дихальних шляхiв на рiвнi дрiбних i середнiх бронхiв. Якi змiни кислотноосновної рiвноваги можуть розвинутись у пацiєнта?

Респiраторний ацидоз

КОР не змiниться

Респiраторний алкалоз

Метаболiчний ацидоз

Метаболiчний алкалоз

2892 / 4299
У хворого 49-ти рокiв виявлено: набряк обличчя, значна протеїнурiя, гiпопротеїнемiя, диспротеїнемiя, гiперлiпiдемiя. Який попереднiй дiагноз?

Пiєлонефрит

Сечокам’яна хвороба

Простатит

Нефротичний синдром

Цистит

2893 / 4299
У хворого наприкiнцi весни i на початку лiта при екскурсiї поза мiсто, а iнодi i в мiстi спостерiгаються ринiт i кон’юнктивiт, що виникають раптово, супроводжуються рясним витiкання рiдкого слизу з носа i сльозотечею. Який тип алергiчних реакцiй спостерiгається у даному випадку?

IV тип ГУТ

II тип цитотоксичний

I тип анафiлактичний

III тип iмунокомплексний

2894 / 4299
У хворого з варикозним розширенням вен, пiд час огляду нижнiх кiнцiвок вiдзначається: цiаноз, пастознiсть, зниження температури шкiри, поодинокi петехiї. Який розлад гемодинамiки наявний у хворого?

Компресiйна iшемiя

Венозна гiперемiя

Обтурацiйна iшемiя

Артерiальна гiперемiя

Тромбоемболiя

2895 / 4299
Пiсля механiчної травми хворому наклали джгут на руку, щоб зупинити кровотечу. Нижче джгута рука зблiдла, з’явилося вiдчуття онiмiння. Цей стан є наслiдком:

Венозного застою

Обтурацiйної iшемiї

Ангiоспастичної iшемiї

Компресiйної iшемiї

Тромбозу

2896 / 4299
У чоловiка 36-ти рокiв, який прибув на вiдпочинок в гори (висота бiльше 2000 м над рiвнем моря) спостерiгались збiльшення частоти дихання, тахiкардiя, незначне запаморочення. Зазначенi симптоми зникли через двi доби. Цей процес називається:

Адаптацiя

Пролiферацiя

Регенерацiя

Компенсацiя

Гальмування

2897 / 4299
У потерпiлого внаслiдок ДТП лiкар констатував вiдсутнiсть дихання i серцевої дiяльностi протягом 1 хвилини. Якому термiнальному стану вiдповiдає ця картина?

Преагонiя

Клiнiчна смерть

Травматичний шок, еректильна фаза

Агонiя

Травматичний шок, торпiдна фаза

2898 / 4299
У чоловiка 50-ти рокiв, який декiлька рокiв страждав на хронiчну печiнкову недостатнiсть, виник асцит. Який механiзм є головним у виникненнi цього порушення?

Зменшення синтезу альбумiнiв i глобулiнiв в печiнцi

Пiдвищення онкотичного тиску в кровi

Пiдвищення тиску в системi воротної вени

Поява в кровi нейротоксичних речовин

Збiльшення вмiсту в кровi ЛПНЩ та ЛПДНЩ

2899 / 4299
Хворий 33-х рокiв, що страждає на iнсулiнозалежний цукровий дiабет, на фонi iн’єкцiї iнсулiну втамував спрагу великою кiлькiстю води, що призвело до розвитку гiпоглiкемiчної коми. Який вид порушення водно-сольового обмiну супроводжує даний стан?

Iзоосмолярная гiпергiдратацiя

Гiперосмолярна гiпергiдратацiя

Гiпоосмолярна гiпогiдратацiя

Гiперосмолярна гiпогiдратацiя

Гiпоосмолярна гiпергiдратацiя

2900 / 4299
Чоловiк 56-ти рокiв з вадою серця скаржиться на набряки нижнiх кiнцiвок, що з’явилися останнiм часом. Мiсцевим патогенетичним фактором набряку є:

Пiдвищення тканинного тиску

Зниження гiдродинамiчного тиску кровi

Пiдвищення гiдродинамiчного тиску кровi

Зниження проникливостi стiнки судини

Пiдвищення онкотичного тиску кровi

2901 / 4299
У хворої дiагностували виразку шлунка. З анамнезу вiдомо, що вона тривалий час хворiє на ревматоїдний артрит. Прийом яких препаратiв найбiльш iмовiрно призвiв до розвитку даного захворювання?

Антигiпертензивнi препарати

Антигiстамiннi препарати

Блокатори Н2-рецепторiв

Глюкокортикоїди

Антибiотики

2902 / 4299
Хворий 67-ми рокiв був доставлений в кардiологiчне вiддiлення зi скаргами на перiодичнi болi у серцi, задишку при незначному фiзичному навантаженнi, цiаноз та набряки. При ЕКГ-обстеженнi виявленi позачерговi збудження шлуночкiв серця. Як називається таке порушення ритму?

Фiбриляцiя

Тахiкардiя

Брадикардiя

Екстрасистолiя

Трiпотiння

2903 / 4299
Пiд час аварiї на атомному пiдводному човнi солдат строкової служби одержав дозу опромiнення 5 Гр. Скаржиться на головний бiль, нудоту, запаморочення. Якi змiни в кiлькостi лейкоцитiв можна очiкувати у хворого пiсля опромiнення?

Лейкопенiя

Еозинофiлiя

Нейтрофiльний лейкоцитоз

Агранулоцитоз

Лiмфоцитоз

2904 / 4299
Хворий доставлений у лiкарню iз рваною раною щелепно-лицевої дiлянки, що супроводжувалася сильною кровотечею, яку довго не могли зупинити. Яке порушення загального об’єму кровi виникне протягом першої години пiсля крововтрати?

Порушень об’єму кровi не буде

Гiперволемiя

Гiповолемiя полiцитемiчна

Гiповолемiя нормоцитемiчна

Гiповолемiя олiгоцитемiчна

2905 / 4299
двох автомобiлiв у одного з водiїв вiдзначається деформацiя у середнiй третинi лiвої гомiлки, сильний бiль, особливо при спробi рухати лiвою гомiлкою. З рани виступають кiнцi кiстки тригранного сiчення, при рухах посилюється кровотеча. Яка кiстка може бути пошкоджена?

Малогомiлкова

Наколiнок

Стегнова

Великогомiлкова

Надп’яткова

2906 / 4299
При якому захворюваннi iнфекцiйноалергiчної або невстановленої природи розвивається двостороннє дифузне або вогнищеве негнiйне запалення клубочкового апарату нирок з характерними нирковими i позанирковими симптомами?

Полiкiстоз нирок

Нефросклероз

Гломерулонефрит

Пiєлонефрит

Нефролiтiаз

2907 / 4299
Хворий захворiв гостро: висока температура, збiльшена, болiсна селезiнка. На 10-й день на шкiрi живота з’явилася розеольозно-папульозна висипка. На 21-й день настала смерть вiд перитонiту. При патологоанатомiчному дослiдженнi трупа у здухвиннiй кишцi виявленi глибокi виразки у дiлянцi некротизованих групових лiмфоїдних фолiкулiв (пейєрових бляшок). Одна з виразок перфорувала, наявний фiбринозно-гнiйний розлитий перитонiт. Про яке захворювання можна подумати у даному випадку?

Черевний тиф

Холера

Амебiаз кишечника

Сальмонельоз

Дизентерiя

2908 / 4299
У померлої жiнки 86-ти рокiв, яка страждала на атеросклероз судин головного мозку, на розтинi виявлена атрофiя кори головного мозку. Як називається ця атрофiя вiдносно причини?

Дисфункцiональна

Нейротична

Вiд тиску

Вiд недостатнього кровопостачання

Вiд дiї фiзичних та хiмiчних факторiв

2909 / 4299
На розтинi тiла хворого 42-х рокiв, що страждав на хронiчний дифузний бронхiт i помер вiд легенево-серцевої недостатностi, виявленi великi, пiдвищеної повiтряностi легенi, що своїми краями прикривають середостiння, не спадаються, блiдосiрого кольору, рiжуться з хрустом, при натискуваннi пальцем на поверхнi легень залишається ямка. З просвiту бронхiв видiляється слизово-гнiйний ексудат. Який найбiльш iмовiрний дiагноз?

Хронiчна дифузна обструктивна емфiзема

Хронiчна вогнищева емфiзема

Вiкарна компенсаторна емфiзема

Iнтерстицiйна, промiжна емфiзема

Первинна iдiопатична емфiзема

2910 / 4299
Пiд час огляду вагiтної на слизовiй оболонцi порожнини рота стоматолог виявив 3 округлi утворення, якi з’явилися 3 днi тому, з червоним обiдком, поверхня їх бiлосiрого кольору до 1 см у дiаметрi. Стоматолог дiагностував таке захворювання:

Афтозний стоматит

Катаральний стоматит

Гангренозний стоматит

Виразково-некротичний стоматит

Лейкоплакiя

2911 / 4299
У видаленiй матцi жiнки 55-ти рокiв патологоанатом у товщi мiометрiю виявив щiльний вузол дiаметром 5 см з чiткими межами, на розрiзi тканина вузла волокниста, сiро-рожевого кольору, з хаотичним розташуванням пучкiв волокон. Мiкроскопiчно пухлина складається з гладком’язевих клiтин, що утворюють пучки рiзної товщини, що йдуть у рiзних напрямках, i прошаркiв мiсцями гiалiнiзованої сполучної тканини. Яка пухлина розвинулась у хворої?

Мiосаркома

Фiбромiома

Рабдомiома

Фiбросаркома

Фiброма

2912 / 4299
У хворого 28-ми рокiв вiдзначалися пiдвищення артерiального тиску, гематурiя i набряки на обличчi. Незважаючи на лiкування, почали наростати явища ниркової недостатностi. Через 6 мiсяцiв хворий помер вiд уремiї. Мiкроскопiчно при дослiдженнi нирок в клубочках виявлена пролiферацiя нефротелiю капсули, подоцитiв з утворенням ”напiвмiсяцiв”, склероз i гiалiноз клубочкiв. Який найбiльш iмовiрний дiагноз?

Пiдгострий гломерулонефрит

Гострий пiєлонефрит

Нефротичний синдром

Гострий гломерулонефрит

Хронiчний гломерулонефрит

2913 / 4299
У чоловiка 23-х рокiв виникла перфорацiя твердого пiднебiння, в областi якого виявлене щiльне утворення iз чiткими межами. Пiсля операцiї при мiкроскопiчному дослiдженнi цього утворення було виявлено: значне вогнище казеозного некрозу, оточене грануляцiйною тканиною з ендоваскулiтом, клiтинним iнфiльтратом, що складається з лiмфоцитiв, епiтелiоїдних клiтин, з переважанням плазмоцитiв. Яке найбiльш iмовiрне захворювання у хворого?

Лепра

Туберкульоз

Сифiлiс

Склерома

Саркома

2914 / 4299
Жiнка 53-х рокiв скаржиться на болiсну припухлiсть у лiвiй привушнiй дiлянцi, яка з’явилась 5 днiв тому. Об’єктивно: в цiй дiлянцi шкiра слабко гiперемована, болiсна. Iз вивiдної протоки слинної залози у малiй кiлькостi видiляється мутна, тягуча жовтувато-зеленувата рiдина. При цьому мiкроскопiчно визначається дифузна iнфiльтрацiя залози сегментоядерними нейтрофiлами. Встановiть дiагноз:

Епiдемiчний паротит

Гострий серозний паротит

Аденома залози

Сухий синдром Шегрена

Гострий гнiйний паротит

2915 / 4299
Чоловiк 65-ти рокiв захворiв на гострий остеомiєлiт нижньої щелепи. Через 3 днi з’явилися рiзко виражений набряк шкiри та пiдщелепних м’яких тканин шиї. В них мiкроскопiчно виявляється дифузна iнфiльтрацiя нейтрофiлами. Яке ускладнення основного захворювання розвинулось у шкiрi хворого?

Карбункул

Флегмона

Актиномiкоз

Абсцес

Фурункул

2916 / 4299
Через декiлька годин пiсля травми зуба у пульпi спостерiгається гiперемiя судин, виражений набряк тканини з поодинокими нейтрофiлами, лiмфоцитами, у нервових волокнах - незначнi дистрофiчнi змiни. Який найбiльш iмовiрний дiагноз?

Гангренозний пульпiт

Гнiйний пульпiт

Фiброзний пульпiт

Гранулюючий пульпiт

Серозний пульпiт

2917 / 4299
На розтинi тiла чоловiка 46-ти рокiв, що не лiкувався вiд гострої кишкової iнфекцiї i помер вiд сепсису, виявленi: флегмона параректальної клiтковини, множиннi виразки прямої i сигмоподiбної кишок, окремi з перфорацiєю, слизова цих вiддiлiв кишечника потовщена, вкрита сiруватого кольору плiвками, що важко знiмаються. Про яке захворювання можна думати?

Черевний тиф

Амебiаз

Туберкульоз

Холера

Дизентерiя

2918 / 4299
Мама дитини 4-х рокiв звернулася по медичну допомогу зi скаргами на пiдвищення температури тiла, тенезми, розвиток дiареї, на переймоподiбнi болi в животi у дитини. Дитина вiдвiдує дитячий колектив. Пiд час дослiдження у калових масах: слиз, домiшка кровi. Як називаються змiни у ШКТ при дизентерiї?

Ентероколiт

Гастроентерит

Колiт

Гастрит

Ентерит

2919 / 4299
На автопсiї у померлого 72-х рокiв з повторним трансмуральним iнфарктом мiокарда оболонки епiкарду та перикарду набухлi, потовщенi, кострубатi, неначе вкритi волосяним покровом. Назвiть вид запалення у оболонках серця:

Серозне

Катаральне

Дифтеритичне

Крупозне

Гнiйне

2920 / 4299
У хворого 35-ти рокiв, який тривалий час страждав на бронхiальну астму, розвинувся астматичний статус з летальним наслiдком. При дослiдженнi секцiйного матерiалу в легенях виявлено спазм бронхiол, в їх стiнках - виражену клiтинну iнфiльтрацiю з переважанням еозинофiльних лейкоцитiв та лiмфоцитiв, лаброцити з явищами дегрануляцiї. Який механiзм гiперчутливостi обумовив зазначенi змiни?

Iмунокомплексна

Реагiнова реакцiя

Антитiлозалежна

Клiтинна цитотоксичнiсть

2921 / 4299
На розтинi тiла чоловiка, що помер вiд серцевої декомпенсацiї, виявлено рiзко збiльшене серце масою 960 г, (”бичаче серце”), у порожнинi перикарду 90 мл солом’яного кольору рiдини. Порожнини серця рiзко розширенi, мiокард в’ялий, товщина стiнки лiвого шлуночка - 2 , 3 см. Нирки зменшенi у розмiрах, масою по 70 г, поверхня їх дрiбнозерниста. Пiд час мiкроскопiчного дослiдження внутрiшнiх органiв виявлений гiалiноз артерiол i гiпертрофiя м’язового шару бiльш крупних артерiй. Назвiть основне захворювання:

Iшемiчна хвороба серця

Гiпертонiчна хвороба

Кардiомiопатiя

Атеросклероз

Ревматизм

2922 / 4299
Пiд час огляду зуба у його коронцi виявлена велика порожнина, дном якої є вузький шар розм’якшеного дентину, що вiддiляє цю порожнину вiд пульпи. Який найбiльш iмовiрний дiагноз?

Перiодонтит 

Поверхневий карiєс

Глибокий карiєс

Пульпiт

Середнiй карiєс

2923 / 4299
До лiкарнi надiйшла дитина з дiагнозом дифтерiя. Якими препаратами для специфiчної терапiї Ви скористуєтесь?

Протидифтерiйна антитоксична сироватка, антибiотики

Дифтерiйний бактерiофаг

Дифтерiйнi вакцини: АКДП, АДП, АД

Дифтерiйний анатоксин, антибiотики

Вакцина ”Кодивак”, сульфанiламiди

2924 / 4299
В iнфекцiйне вiддiлення лiкарнi госпiталiзовано хворого зi скаргами на нудоту, рiдкi випорожнення зi слизом i прожилками кровi, пiдвищення температури, слабкiсть. Лiкар запiдозрив дизентерiю. Який метод лабораторної дiагностики найдоцiльнiше призначити для пiдтвердження дiагнозу?

Серологiчний

Бактерiологiчний

Мiкологiчний

Мiкроскопiчний

2925 / 4299
В хiрургiчний кабiнет звернулась людина, яку покусав невiдомий собака. Широкi рванi рани локалiзованi на обличчi. Яку лiкувально-профiлактичну допомогу потрiбно надати для профiлактики сказу?

Термiново ввести нормальний γ-глобулiн

Термiново ввести вакцину АКДП

Призначити комбiновану антибiотикотерапiю

Госпiталiзувати хворого i тримати пiд наглядом лiкаря

Розпочати iмунiзацiю антирабiчною вакциною

2926 / 4299
У бактерiологiчнiй лабораторiї проводиться дослiдження якостi питної води. Її мiкробне число виявилося близько 100. Якi мiкроорганiзми враховувалися при цьому?

Умовно-патогеннi мiкроорганiзми

Ентеропатогеннi бактерiї та вiруси

Бактерiї групи кишкової палички

Бактерiї, патогеннi для людей та тварин

Всi бактерiї, що виросли на живильному середовищi

2927 / 4299
При пiдозрi на туберкульоз хворiй дитинi зробили пробу Манту. Через 24 години у мiсцi введення алергену з’явилися припухлiсть, гiперемiя i болiснiсть. Якi основнi компоненти визначають цю реакцiю органiзму?

Гранулоцити, Т-лiмфоцити i Ig G

Макрофаги, В-лiмфоцити i моноцити

Мононуклеари, Т-лiмфоцити i лiмфокiни

Плазматичнi клiтини, Т-лiмфоцити i лiмфокiни

В-лiмфоцити, Ig М

2928 / 4299
У трирiчної дiвчинки краснуха. Її десятирiчна сестричка не заразилась цiєю iнфекцiйною хворобою, хоча весь час контактувала з нею. Педiатр з’ясував, що вона хворiла на краснуху п’ять рокiв тому. Який вид iмунiтету лежить в основi захисту старшої сестри?

Видовий

Природний активний

Штучний активний

Штучний пасивний

Природний пасивний

2929 / 4299
Хворий звернувся до лiкаря зi скаргами на перiодичнi висипання герпетичних пухирцiв на лiнiї губ i на крилах носа. Такий стан спостерiгається впродовж 10-ти рокiв, кожний раз пiсля зниження захисних сил органiзму. Лiкар встановив дiагноз: лабiальний герпес. Як називається така форма iнфекцiї?

Гостра

Екзогенна

Латентна

Затяжна

Персистенцiя

2930 / 4299
При iдентифiкацiї чистої культури мiкроорганiзмiв найважливiшою є серологiчна iдентифiкацiя i для цього використовують реакцiю аглютинацiї. Виберiть, якi компоненти необхiднi для постановки цiєї реакцiї:

Термоекстракт, специфiчна сироватка

Специфiчний антиген, вiдоме антитiло, бактерiї

Невiдомi антитiла, неспецифiчний антиген

Невiдома культура бактерiй, специфiчнi антитiла

Специфiчний антиген, сироватка хворого

2931 / 4299
При проведеннi лабораторної дiагностики гепатиту С визначають наявнiсть антитiл до вiрусу гепатиту С у сироватцi кровi хворого. Яке дослiдження при цьому слiд провести?

Метод нуклеїнових зондiв

Метод гiбридизацiї нуклеїнових кислот

IФА

Метод лiгазної ланцюгової реакцiї

Метод гiбридизацiї нуклеїнових кислот з посиленням сигналу

2932 / 4299
Лiкарi-стоматологи мають великий ризик захворiти на гепатит В, тому пiдлягають обов’язковiй iмунiзацiї. За допомогою якої вакцини це може бути зроблено?

Анатоксин

Хiмiчна

Iнактивована

Рекомбiнантна

Жива

2933 / 4299
У хворого пiсля екстракцiї зуба розвинулася гостра серцева недостатнiсть. Який препарат доцiльно призначити?

Кордiгiт

Строфантин

Дигiтоксин

Настойку конвалiї

Адонiзид

2934 / 4299
Хворому призначено комплексне лiкування пародонтозу з використанням антибiотика лiнкомiцина. Позитивного ефекту не було. Призначте iнший антибiотик, здатний накопичуватись в кiстках, зубах i сполучнiй тканинi:

Нiстатин

Тетрациклiн

Еритромiцин

Левомiцетин

Оксацилiн

2935 / 4299
Хворому для лiкування iшемiчної хвороби серця (IХС) був призначений βадреноблокатор, через деякий час у нього з’явився кашель, бронхоспазм. У якого з перелiчених засобiв є така побiчна дiя?

Анаприлiн

Фенiгiдин

Метопролол

Атенолол

Талiнол

2936 / 4299
При пародонтозi хворому призначили жиророзчинний вiтамiнний препарат, що бере активну участь в окисно-вiдновних процесах в органiзмi. Антиоксидант є фактором росту, антиксерофтальмiчним, забезпечує нормальний зiр. В стоматологiчнiй практицi використовується для прискорення епiтелiзацiї при захворюваннях слизових оболонок при пародонтозi. Визначте цей препарат:

Ретинолу ацетат

Вiкасол

Цiанокобаламiн

Ергокальциферол

Токоферолу ацетат

2937 / 4299
Хворому на ревматизм призначили нестероїдний протизапальний засiб диклофенак-натрiй. Через загострення супутнього захворювання диклофенакнатрiй вiдмiнили. Яке захворювання є протипоказом до його призначення?

Бронхiт

Стенокардiя

Гiпертонiчна хвороба

Цукровий дiабет

Виразкова хвороба шлунка

2938 / 4299
Хворому з явищами тривоги, страху, невпевненостi, психiчної напруги, призначено дiазепам. Який можливий механiзм його транквiлiзуючої дiї?

Взаємодiя з бензодiазепiновими рецепторами

Взаємодiя з серотонiновими рецепторами

Взаємодiя з холiнорецепторами

Взаємодiя з дофамiновими рецепторами

Взаємодiя з адренорецепторами

2939 / 4299
Пацiєнту з гiпохромною анемiєю призначено залiзовмiсний препарат, який вводять тiльки внутрiшньовенно. Який це засiб?

Дихлотiазид

Ферковен

Кислота етакринова

Манiт

Фуросемiд

2940 / 4299
У дитини спостерiгається сухий кашель. Який протикашльовий засiб ненаркотичної дiї полегшить стан хворого?

Морфiну гiдрохлорид

Глауцину гiдрохлорид

Коренi алтеї лiкарської

Кодеїну фосфат

Калiю йодид

2941 / 4299
Хворому на виразкову хворобу шлунка призначено препарат в основi дiї якого є блокада H2-гiстамiнових рецепторiв. Який це препарат?

Омепразол

Бiсакодил

Атропiну сульфат

Фамотидин

Дитилiн

2942 / 4299
Хворому з кардiогенним шоком треба ввести неглiкозидний кардiотонiк. Вкажiть препарат вибору у данiй ситуацiї:

Амрiнон

Кофеїн

Кордiамiн

Етiмiзол

Добутамiн

2943 / 4299
Для дегiдратацiйної терапiї у випадку набряку мозку та легень призначають препарат з потужною натрiйуретичною дiєю. Вкажiть цей препарат:

Кислота етакринова

Теофiлiн

Фуросемiд

Спiронолактон

Манiт

2944 / 4299
Хворому зi злоякiсною пухлиною для усунення нестерпного болю призначили наркотичний анальгетик. Який механiзм розвитку протибольової дiї цього засобу?

Гальмування холiнергiчних рецепторiв

Активацiя Д2-дофамiнових рецепторiв

Активацiя опiатних рецепторiв

Гальмування гiстамiнергiчних рецепторiв

Гальмування серотонiнергiчних рецепторiв

2945 / 4299
Хворому на стрептококову пневмонiю призначено протимiкробний засiб, що порушує побудову мiкробної оболонки. Який це препарат?

Гентамiцину сульфат

Азитромiцин

Бензилпенiцилiну натрiєва сiль

Доксициклiну гiдрохлорид

Еритромiцин

2946 / 4299
У пацiєнтки 26-ти рокiв висипання на шкiрi, свербiж пiсля вживання цитрусових. Призначте лiкарський засiб з групи блокаторiв Н1-гiстамiнорецепторiв:

Димедрол

Парацетамол

Кислота ацетилсалiцилова

Вiкасол

Анальгiн

2947 / 4299
Хворий на мегалобластну анемiю приймав препарат з групи водорозчинних вiтамiнних засобiв. Визначте цей препарат:

Цiанокобаламiн

Пiридоксин

Аскорбiнова кислота

Тiамiну хлорид

Токоферолу ацетат

2948 / 4299
До комплексної терапiї хворого на бронхопневмонiю, що супроводжується виснажливим сухим кашлем, лiкар включив муколiтичний препарат, який деполiмеризує мукопротеїди. Вкажiть цей препарат:

Ацетилцистеїн

Кодеїн

Неодикумарин

Атенолол

Строфантин

2949 / 4299
Перед проведенням екстракцiї зуба з використанням мiсцевої анестезiї хворому було проведено пробу на чутливiсть до новокаїну, яка виявилась позитивною. Який препарат можна використати для анестезiї у даному випадку?

Кислота ацетилсалiцилова

Новокаїнамiд

Натрiю вальпроат

Лiдокаїн

Анальгiн

2950 / 4299
З метою усунення нападу бронхiальної астми, який розвинувся при екстракцiї зуба, пацiєнту призначили сальбутамол. До якої фармакологiчної групи вiдноситься цей препарат?

Адаптогени

М-холiномiметики

Аналептики

β2-адреномiметики

Наркотичнi анальгетики

2951 / 4299
Хворий напередодні операції був у стані стресу. Збільшення концентрації якого гормону в крові супроводжує цей стан?

Прогестерон

Адреналін

Пролактин

Iнсулін

Глюкагон

2952 / 4299
Пацієнт із сильним зубним болем впродовж декількох днів не звертався до лікаря і займався самолікуванням. В результаті цього виникла необхідність екстракції зуба. Застосування якого анальгетика збільшує імовірність розвитку кровотечі після екстракції зуба?

Парацетамолу

Анальгіну

Кодеїну фосфату

Ацетилсаліцилової кислоти

Димедролу

2953 / 4299
Відомо, що слиз завжди вкривас епітелій власне носової порожнини. Вкажіть, які клітини епітелію слизової оболонки носової порожнини синтезують слиз:

Базальні

Келихоподібні

Biйчасті

Мікроворсинчасті

2954 / 4299
Вибрати концентрацію етилового спирту, що має найактивнішу протимікробну дію за наявності білка в середовищі:

40%

96%

60%

15%

70%

2955 / 4299
У хворої дитини спостерігаються ознаки ахондроплазії (карликовості). Відомо, що це моногенне захворювання, і ген, який відповідає за розвиток такої аномалії є домінантним. У рідного брата цієї дитини розвиток нормальний. За генотипом здорова дитина буде:

ААВВ

АаВb

aa

Aa

АА

2956 / 4299
У хворого аритмія. Який лікарський засіб слід призначити хворому?

Нітрогліцерин

Iмізин

Аміодарон

Eyфілін

Кавінтон

2957 / 4299
У хворого діагностовано перелом вінцевого відростка нижньої щелепи зі зміщенням. Який м'яз змістить відросток?

Скроневий

Медiальний крилоподібний

Латеральний крилоподібний

Жувальний

2958 / 4299
У хворого на цукровий діабет з'явився біль у правій нозі, тканини першого пальця стали набряклими, чорного кольору, епідерміс відшарувався, з'явилися виділення з неприємним запахом. Визначте патологічний процес:

Секвестр

Iнфаркт

Коагуляційний некроз

Волога гангрена

Суха гангрена

2959 / 4299
У хворого на СНIД в клітинах, уражених ВІЛ-інфекцією, виявлено активність ферменту ревертази. Яка нуклеїнова кислота синтезується заучастю цього ферменту?

рPHK

мPHK

тPHK

ДНК

пре-мРНК

2960 / 4299
Під час ультразвукового дослідження дитини 1,5 років встановлено незарощення овального отвору. Де у серці розташоване це анатомічне утворення?

Стінка лівого шлуночка

Міжшлуночкова перегородка

Міжпередсердна перегородка

Стінка правого шлуночка

2961 / 4299
Хворий 38-ми років з хронічним алкоголізмом помер від прогресуючої серцевої недостатності. Нa розтині: лобарна плевропневмонія нижньої часточки правої легені. При гістологічному дослідженні в альвеолах фібринозний ексудат та сегментоядерні лейкоцити. Визначте стадію крупозної пневмонії:

Завершення

Сiрої гепатизації

Червоної гепатизації

Припливу

2962 / 4299
В плазмі крові здорової людини містяться декілька десятків білків. При захворюванні організму з'являються нові білки, зокрема 'білок гострої фази'. Таким білком є:

Фібриноген

C-рeактивний білок

Імуноглобулін А

Протромбін

Iмуноглобулін G

2963 / 4299
Під час фізичного й емоційного навантаження людина є менш чутливою до болю. Причиною цього є активація:

Hоцицептивної системи

Функції щитоподібної залози

Парасимпатичної системи

Антиноцицептивної системи

Функції надниркових залоз

2964 / 4299
У приймальне відділення надійшов хворий з ознаками гострої серцевої недостатності: блідістю, акроціанозом, частим поверхневим диханням. Який з перерахованих засобів показаний в цьому випадку?

Корглікон

Адреналіну гідрохлорид

Дигітоксин

Кордіамін

Hітрогліцерин

2965 / 4299
Вивчення відбитків виступів епідермiсy пальців рук (дактилоскопія) використовується криміналістами для іденитфікаці особи, а також для діагностики генетичних аномалій, зокрема хвороби Дауна. Який шар шкіри визначає індивідуальність відбитків?

Сітчастий

Cоcочковий

Блискучий

Базальний

Роговий

2966 / 4299
В хірургічний кабінет звернулася людина, яку покусав невідомий собака. Широкі рвані рани локалізовані на обличчі. Яку лікувально-профілактичну допомогу потрібно надати для профілактики сказу?

Призначити комбіновану антибіотикотерапію

Tерміново ввести вакцину АКДП

Розпочати імунізацію антирабічною вакциною

Госпіталізувати хворого і тримати під наглядом лікаря

Терміново ввести нормальний у-глобулін

2967 / 4299
При запальних процесах в організмі починається синтез білків 'гострої фази'. Яка речовина є стимулятором іх синтезу?

Iнтерферон

Інтерлейкін-1

Iмуноглобуліни

Ангіотензин

Біогенні аміни

2968 / 4299
У новонародженої дитини щелепи добре розвинені та в кожній є зачатки молочних і постійних зубів. Скільки зачатків зубів у новонародженого в одній щелепі?

20 молочних та 10 постійних

10 молочних та 8 постійних

10 молочних та 10 постійних

10 молочних та 16 постійних

20 молочних

2969 / 4299
У людей похилого віку спостерігається надмірна втрата маси кісткової тканини, яка відображає розвиток остеопорозу. Активація яких клітин кісткової тканини зумовлює розвиток цього захворювання?

Остеокластів

Тканинних базофілів

Остеобластів

Макрофагів

Остеоцитів

2970 / 4299
В бактеріологічну лабораторію направлено харкотиння хворого на туберкульоз. Для бактеріоскопічного дослідження препаратів-мазків і виявлення туберкульозної палички потрібно використати один із вказаних методів фарбування:

Pомановського

Грама

Здродовського

Ціля-Нільсена

Буррі-Гінса

2971 / 4299
В ході стоматологічних маніпуляцій були пошкоджені волокна 12-ї пари черепномозкових нервів. У чому це проявилося?

Порушення скорочення м'язів гортані

Порушення скорочення м'язів, що піднімають під'язикову кістку

Порушення функції м'язів язика

Порушення скорочення м'язів глотки

Порушення скорочення м'язів м'якого піднебіння

2972 / 4299
При виконанні оперативного втручання на діафрагмі ротової порожнини хірургу необхідно виділити ділянку, що має назву 'піднижньощелепний трикутник'. Який з м'язів його обмежує?

М. digastricus

М. stylohyoideus

М. geniohyoideus

М. hуoglossus

2973 / 4299
У похилої людини зареєстрували зміну сили серцевих скорочень та фізичних властивостей судин, що чітко відобразилося на графічному записі пульсових хвиль сонної артерії. Який метод було застосовано?

Флебографія

Плетизмографія

Mioгpафія

Рeографія

Сфiгмографія

2974 / 4299
Чоловік звернувся до стоматолога зі скаргами на біль при жуванні і висуванні щелепи вперед. Які жувальні м'язи запалені?

M.m. buccalis

М.m. pterigoidei laterales

М.m. temporales

М.m. plerigoidei mediales

2975 / 4299
Хворого доставлено у лікарню з попереднім діагнозом «ботулізм». Яку серологічну реакцію слід застосувати для виявлення ботулінічного токсину в досліджуваному матеріалі?

Реакцію преципітації

Реакцію зв'язування комлементу

Реакцію аглютинації

Реакцію імунофлюоресценції

Реакцію нейтралізації

2976 / 4299
Гостра крововтрата викликала падіння системного артеріального тиску. Посилення секреції якого гормону може стабілізувати цю ситуацію?

Реніну

Iнсуліну

Глюкагону

Гастрину

Tестостерону

2977 / 4299
У дівчинки 16-ти років, яка тривалий час намагалась знизити масу свого тіла голодуванням, виник набряк. Яка головна причина цього явища?

Зменшення синтезу вазопресину в гіпоталамусі

Зменшення швидкості клубочкової фільтрації

Венозний застій і підвищення венозного тиску

Гіпоглікемія, зумовлена порушенням синтезу глікогену

Гiпопротеїнемія, зумовлена порушенням синтезу білків

2978 / 4299
У людини виявлена пухлина одного з відділів головного мозку, внаслідок чого в неї порушена здатність підтримувати нормальну температуру тіла. Яка структура головного мозку пошкоджена?

Стріатум

Мозочок

Tаламус

Чорна субстанція

Гіпоталамус

2979 / 4299
У посіві гною з фурункулу знайдено кулястої форми мікроорганізми, розташовані у вигляді 'грона винограду' Які мікроби виявлено?

Диплококи

Тетракоки

Мікрококи

Стрептококи

Стафілококи

2980 / 4299
При деяких спадкових хворобах (наприклад синдром Кернса-Сейра) спостерігається деструкція мітохондрій. Які процеси у клітині можуть бути порушені внаслідок цього?

Синтез білків

Поділ ядра

Кросинговер

Синтез ліпідів

Синтез АТФ

2981 / 4299
У дитини 1,5 років при огляді виявили осередкові потовщения в ділянці ребер, зап'ястків, викривлення ніжок. Стоматолог вказав на пізнє прорізування зубів, порушення порядку прорізування зубів, нерівномірну мінералізацію емалі та дентину, конфігурацію верхньої щелепи в горизонтальному напрямку у вигляді 'високого піднебіння' Яке захворювання розвинулося у дитини?

Рахіт

Подагра

Флюороз

Ciaлолітіаз

Остеопороз

2982 / 4299
В ході розтину тіла померлого у прямій і сигмоподібній кишках видно дефекти слизової оболонки неправильної форми з нерівними контурами, які зливаються між собою, залишаючи невеликі острівці слизової оболонки,що збереглася. Який різновид коліту можна припустити?

Фолікулярний

Виразковий

Гнійний

Фібринозний

Катаральний

2983 / 4299
У хворого діагностована пневмонія мікоплазмової етіології. Які антибіотики за механізмом дії НЕ СЛІД використовувати для лікування?

Антибіотики, які пригнічують синтез компонентів клітинної стінки

Антибіотики, які порушують процеси окислювального фосфорилювання

Антибіотики, які порушують синтез білка

Антибіотики, які порушують синтез нуклеїнових кислот

Антибіотики, які порушують проникність цитоплазматичної мембрани

2984 / 4299
У хворого запалення привушної слинної залози. Визначте, який нерв залучений в запальний процес:

N. tympaniсus

N. mandibularis

N. Facialis

N linqualis

N. mаxillaris

2985 / 4299
При гістохімічному дослідженні лейкоцитів у мазку крові визначаються клітини, у цитоплазмі яких с гранули, що містять гістамін і гепарин. Які це клітини?

Еозинофіли

Моноцити

Нейтрофіли

Базофіли

Еритроцити

2986 / 4299
Чоловік 56-ти років з вадою серця скаржиться на набряки нижніх кінцівок, що з'явилися останнім часом. Mісцевим патогенетичним фактором набряку є:

Зниження проникності стінки судини

Підвищення гідродинамічного тиску крові

Підвищення онкотичного тиску крові

Підвищення тканинного тиску

2987 / 4299
Під час проведення стоматологічної лікувальної процедури у пацієнта виник напад стенокарді. Препарати якої групи необхідно призначити для надання швидкої допомоги?

Антигіпертензивні засоби

Антиангінальні засоби

Кардіотоніки

Стимулятори дихання

Протиаритмічні засоби

2988 / 4299
В медичній практиці широкого застосування набули курареподібні речовини (міорелаксанти). Про яку побічну дію повинен пам'ятати лікар при їх застосуванні?

Напади судом

Зупинка серця

Тромбоутворення

Розслаблення дихальних м'язів

Pозлади мозкового кровообігу

2989 / 4299
На електронній мікрофотографії eпiтеліальної тканини ідентифікується структура, що лежить під епітеліоцитами і має вигляд тривимірної сітки. Як вона називається?

Десмосома

Цитолема

Hапiвдесмосома

Власна пластинка

Базальна мембрана

2990 / 4299
В ході розтину тіла 4-pічної дівчинки, яка тривалий час страждала і померла від зливної пневмонії, виявлено, що маса тимуса становить 2 грами. Під час гістологічного досліджения тимуса виявлено різкий спад лімфоцитів, колапс строми залози і нечисленні обвапнені кістоподібно розширені тільця Гассаля. Який патологічний процес розвинувся в тимусі?

Атрофія тимуса

Tимомегалія

Гіперплазія тимуса

Дисплазія тимуса

2991 / 4299
При патогістологічному дослідженні тканин видаленого зуба відзначається заповнення більшої частини порожнини зуба сполучною тканиною зі значною кількістю колагенових волокон та клітинними інфільтратами, які складаються з лімфоцитів і плазматичних клітин. Для якої форми пулыіту характерні зазначені зміни?

Гнійний пульпіт

Фіброзний пульпіт

Гранулюючий пульпіт

Гангренозний пульпіт

2992 / 4299
Хворий скаржиться на сильний нежить та втрату відчуття запахів. Де в носовій порожнині розташовані рецептори нюхового аналізатору?

Нижній носовий хід

Верхній носовий хід

Середній носовий хід

Загальний носовий хід

Хоани

2993 / 4299
При дослідженні на лабораторних щурах властивостей одного з вітамінів при його дефіциті спостерігалося одночасне порушення репродуктивної функції та дистрофія скелетних м'язів. Про який вітамін йдеться?

А

B2

D

E

K

2994 / 4299
До гастроентеролога звернулася мати з дитиною 12-ти років зі скаргами на зниження апетиту в дитини, метеоризм. При ендоскопічному обстеженні діагностовано дискінезію жовчних проток, а у дуоденальному вмісті виявлено найпростіші грушоподібної форми з двома ядрами та численними джгутиками. Яке захворювання найбільш імовірне у дитини?

Трихомоноз

Балантидіаз

Лямбліоз

Амебіаз

Tоксоплазмоз

2995 / 4299
В експерименті при вивченні процесів всмоктування продуктів гідролізу іжі та води було встановлено, що основним відділом шлунково-кишкового тракту, де відбуваються ці процеси, є:

Товста кишка

Pотова порожнина

Тонка кишка

Шлунок

Пряма кишка

2996 / 4299
У хворого встановлено діагноз синдром Клайнфельтера. Каріотип при цьому захворюванні буде (47, XXҮ). В цьому наборі буде така кількість статевих хромосом:

Tpи

Сорок чотири

Дві

Одна

Нуль

2997 / 4299
При огляді ротової порожнини чоловіка 50-ти років, який тривалий час палить, на слизовій оболонці язика виявлено неправильної форми бляшку білого кольору. Гістологічно виявлено потовщення багатошарового плоского епітелію, паракератоз, гіперкератоз та акантоз. Вкажіть вид патологічного процесу:

Kератоакантома

Лейкоплакія

Хронічний стоматит

Гіпертрофічний глосит

Авітаміноз А

2998 / 4299
Хворому на ревматизм призначили нестероїдний протизапальний засіб диклофенак-натрій. Через загострення супутнього захворювання диклофенакнатрій відмінили. Яке захворювання с протипоказанням для призначення диклофенаку-натрію?

Гіпертонічна хвороба

Стенокардія

Цукровий діaбет

Виразкова хвороба шлунку

Бронхіт

2999 / 4299
Для проведення аналізу кров пацієнта відібрали у присутності гепарину. Цей антикоагулянт за хімічною структурою належить до:

Гемопротеїнів

Триацилгліцеролів

Фосфолітідів

Простих білків

Глікозаміногліканів

3000 / 4299
При гістологічному дослідженні ділянки тканини виявлені явища каpіопiкнозу, каpiopексису, карiолізису у ядрах клітин, а також плазмолізис і плазморексис у цитоплазмі клітин. Який патолоrічний процес має місце у цьому випадку?

Некроз

Атрофія

Дистрофія

Гіаліноз

3001 / 4299
Дитині 13-ти років видалили другий молочний великий кутній зуб. Який постійний зуб виросте на його місці?

Перший великий кутній зуб

Третій великий кутній зуб

Перший малий кутній зуб

Другий великий кутній зуб

Другий малий кутній зуб

3002 / 4299
У хворого після застудного захворювання з'явилося порушення больової i температурної чутливості передніх 2/3 язика. Який з нервів при цьому постраждав?

Під’язиковий

Блукаючий

Барабанна струна

Дiафрагмальний

Трійчастий

3003 / 4299
У дитини 10-ти років раптово підвищилася температура до 39°С, 3'явилися нежить, кашель, світлобоязнь, Під час огляду на слизовій оболонці порожнини рота виявлені плями Філатова-Копліка. Який з діагнозів найбільш iмовiрний?

Мононуклеоз

Скарлатина

Вітряна віспа

Краснуха

Kip

3004 / 4299
Людина в спекотну погоду тривалий час була позбавлена можливості пиття, що призвело до вираженого відчуття спраги. Зміна якого гомеостатичного показника крові стала першопричиною цього?

Осмотичний тиск плазми

pH

Онкотичний тиск плазми

Pівень глюкози

Гематокрит

3005 / 4299
У хворого, який тривалий час не звертався до лікаря-стоматолога для лікування хронічного каріссу в 36 зубі, з'явився різкий біль в нижній щелепі, припухлість щоки, температура тіла піднялася до 38°C. Яких змін в аналізі крові слід очікувати у хворого?

Анемія

Нейтрофільоз

Еозинофілія

Моноцитоз

Лейкопенія

3006 / 4299
У недоношених новонароджених порушений синтез сурфактанту. Які функції він виконує в легенях?

Зменшує поверхневий натяг стівокальвеол

Підвищує опір дихальних шляхів

Погіршує дифузію О2 через аерогематичний бар'єр

Збільшує поверхневий натяг стінок альвеол

Полегшує екскурсію діафрагми

3007 / 4299
Хворому на виразкову хворобу шлунку призначено препарат, в основі дії якогоє блокада Н2-гістамінових рецепторів. Який це препарат?

Омепразол

Фамотидин

Бісакодил

Атропіну сульфат

Дитилін

3008 / 4299
Порекомендуйте препарат з групи нітратів хворому на ІXC для профілактики нападів стенокардії:

Лізиногліцерин

Нітрогліцерин

Ментол

Ловастатин

lзocopбіду мононітрат

3009 / 4299
Хворому 60-ти років, який стражає на цукровий діабет, був призначений інсулін. Про який вид медикаментозної терапії йде мова?

Симптоматична

Замісна

Патогенетична

Eтiотропна

Профілактична

3010 / 4299
У хворої діагностовано крововилив в задні роги спинного мозку. Якими вони є за функцією?

Чутливими

Парасимпатичними

Симпатичними

Руховими

3011 / 4299
У хворого менінгіт. Показана пункція підпавутинного простору. Визначте, між якими структурами він розташований:

Окістям та твердою мозковою оболонкою

Твердою та павутинною оболонками

Павутинною та м'якою оболонками

Окістям та павутинною оболонкою

3012 / 4299
Стоматолог при огляді хворих зазначив, що у багатьох з них зубна емаль без блиску, з фарфороподібними та пігментованими плямами. У окремих хворих є поодинокі та множинні дефекти емалі у вигляді безбарвних або пirментованих ерозій. Надмірне надходження в організм якої речовини призвело до розвитку таких змін в зy6ax?

Калію

Магнію

Фтору

Кальцію

Натрію

3013 / 4299
При досліджениі 16 зуба на жувальній поверхні виявлено порожнину з вузьким отвором, заповнену розм'якшеним дентином. Мікроскопічно: в розширених дентинних канальцях бактері, руйнування окремих канальців, злиття порожнин у каверни, явища декальцинації емалі та дентину без утворення замісного дентину. Поставте діагноз:

Гострий поверхневий карієс зуба

Карієс зуба у стадії плями

Гострий глибокий карієс зуба

Хронічний поверхневий карієс зуба

Хронічний глибокий карієс зуба

3014 / 4299
У хворого під час відвідування стоматолога трапився напад бронхіальної астми. Що необхідно призначити хворому для зняття бронхоспазму?

Сальбутамол

Метацин

Анаприлін

Дроперидол

Бензогексоній

3015 / 4299
У хворого, який помер в результаті легенево-серцевої недостатності, серце збільшене в розмірах, стінка правого шлуночку на розтині потовщена, порожнина розширена. Визначте характер патологічного процесу:

Гіперплазія

Склероз

Гіпертрофія

Метаплазія

Атрофія

3016 / 4299
У трирічної дівчинки краснуха. Її десятирічна сестра не заразилася цією iнфекційною хворобою, хоча весь час контактувала з хворою. Педіатр з'ясував, що вона хворіла на краснуху п'ять років тому. Який вид імунітету лежить в основі захисту старшої сестри?

Видовий

Природний активний

Природний пасивний

Штучний активний

Штучний пасивний

3017 / 4299
У пацієнта виявлено різке розширення підшкірних вен в ділянці передньої черевної стінки навколо пупка. Підвищений тиск у якій судині сприяє цій симптоматиці?

V. рortae heрatis

V. mesenterica inferior

V. cava superior

V. сava inferior

V. mesenterica superior

3018 / 4299
У хворого зі скаргами на біль у шлунку встановлено зменшення його секреторної функції, що супроводжується анемією. Недостатність якої peчовини зумовлює розвиток у хворого гіповітамінозу В12 та виникнення анемії?

Фактор Касла

Tiaмін

Піридоксин

Біотин

Кальциферол

3019 / 4299
На аутопсії у померлого 72-х років з повторним трансмуральним інфарктом міокарда оболонки серця епікард та перикард набряклі, потовщені, неpівні, неначе вкриті волосяним покривом. Назвіть вид запалення в оболонках серця:

Крупозне

Гнійне

Катаральне

Серозне

Дифтеритичне

3020 / 4299
На шостому місяці вагітності в жінки з'явилася виражена залізодефіцитна анемія. Її діагностичною ознакою була поява в крові:

Пойкілоцитів

Макроцитів

Анулоцитів

Нормоцитів

Ретикулоцитів

3021 / 4299
Відомо, що iони калыцію разом з іншими факторами забезпечують скорочення м’язової тканини. 3 якими структурами взаємодіє кальцій під час скорочення?

Білком міозином товстих фібрил

Актоміозиновим комплексом сарколеми

Білком актином тонких фібрил

Білком кальсеквестрином

Білком тропоніном тонких фібрил

3022 / 4299
У дитини діагностовано глистну інвазію. Яких змін лейкоцитарної формули слід при цьому очікувати?

Збільшення кількості моноцитів

Збільшення кількості нейтрофілів

Збільшення кількості еритроцитів

3більшення кількості лімфоцитів

Збільшення кількості еозинофілів

3023 / 4299
При лікуванні пародонтиту хворому призначили жиророзчинний вітамінний препарат, що бере активну участь в окисно-відновних процесах в організмі. Антиоксидант, який є фактором росту, антиксерофтальмічним, забезпечує нормальний зір. В стоматологічній практиці використовується для прискорення епітелізації при захворюваннях слизової оболонки порожнини рота та тканин паподонту. Buзначте цей препарат:

Ергокальциферол

Ретинолу ацетат

Biкасол

Ціанокобаламін

Токоферолу ацетат

3024 / 4299
У хворого на цукровий діабет після iн'єкції інсуліну настала втрата свідомості, судоми. Який результат може дати біохімічний аналіз крові на вміст цукру?

10,0 ммоль/л

3,3 ммоль/л

1,5 ммоль/л

8,0 ммоль/л

5,5 ммоль/л

3025 / 4299
У пацієнта через 2 місяці після операції трансплантації нирки погіршився стан. На основі лабораторного обстеження констатовано, що розпочалася реакція відторгнення трансплантату. Який фактор імунної системи відіграє вирішальну роль в цій реакції?

T-xeлпepu

Інтерлейкін-1

Природні кілери

В-лімфоцити

T-кілери

3026 / 4299
У хворого фронтит. В анамнезі-задалення верхньощелепної пазухи. Через який відділ носової порожнини інфекцій могла потрапити до лобової пазухи?

Нижній носовий хід

Клино-решітчастий закуток

Верхний носовий хід

Присінок носової порожнини

Середній носовий хід

3027 / 4299
У дитини 4-х років виявлено численні каріозні порожнини, жовте забарвлення зубів. Відомо, що мати під час вагітності лікувалася антибіотиками. Який препарат неймовірніше приймала мати дитини?

Еритроміцин

Доксициклін

Стрептоміцину сульфат

Цефазолін

Ампіцилін

3028 / 4299
Після припинення вживання морфіну за умов його тривалого застосування виникають тяжкі психічні, неврологічні і соматичні порушення. Як називається цей стан?

Tолерантність

Кумуляція

Ідіосинкразія

Сенсибілізація

Абстиненція

3029 / 4299
У хворого зареєстрували ЕKГ. За яким її елементом лікар може оцінити процеси розповсюдження деполяризації передсердями?

Зубець S

Зубець Т

Зубець Р

Зубець R

Зубець Q

3030 / 4299
У хворого з синдромом Іценка-Кушинга спостерігаються стійка гіперглікемія та глюкозурія. Синтез та секреція якого гормону збільшені у цього хворого?

Адреналім

Альдостерон

Tироксин

Глюкагон

Кортизол

3031 / 4299
При якому захворюванні інфекційно-алергічної або невстановленої природи розвивається двостороннє дифузне або вогнищеве негнійне запалення клубочкового апарату нирок з характерними нирковими і позанирковими симптомами?

Пієлонефрит

Полікістоз нирок

Нефролітіаз

Нефросклероз

Гломерулонефрит

3032 / 4299
Лікар-стоматолог застосував в якості антисептика розчин калію перманганату. Бактерицидний ефект препарату забезпечує:

Атомарний кисень

Mангану оксид

Калію оксид

Калій

Гідроксид калію

3033 / 4299
До лікаря-офтальмолога звернувся підліток з порушенням гостроти зору. Лікар пояснив це спазмом акомодації. Який з названих компонентів очного яблука входить до акомодаційного aпарату ока?

Скловидне тіло

Сітківка

Рогівка

Склера

Циліарний м'яз

3034 / 4299
Піддослідному змастили кiнчик язика місцевим анестетиком. Це призведе до відсутності сприйняття смаку.

Солоного

Кислого

Солодкого

Гіркого

Кислого та солоного

3035 / 4299
У вертикальному положенні пацієнт, заплющуючи очі, втрачає рівновагу. Які структури мозку у нього найімовірніше уражені?

Прецентральна звивина кори великих півкуль

Лімбічна система

Таламус

Базальні ганглії

Мозочок

3036 / 4299
До основних способів підвищення резистентності емалі належить фторування. Механізм протикаріозної дії фтору пов'язаний з:

Синтезом гідроксиапатиту

Синтезом фторапатиту

Синтезом хлорапатиту

Демінералізацією зуба

3037 / 4299
У хворого наприкінці весни і на початку літа під час екскурсій за межі міста, а іноді й в місті спостерігаються риніт і кон'юнктивіт, що виникають раптово, супроводжуються рясним витIкання рідкого слизу з носа і сльозотечею. Який тип алергічних реакцій спостерігастьси у цьому випадку?

IIІ тип імунокомплексний

I тип анафілактичний

II тип цитотоксичний

IV тип ГУТ

3038 / 4299
Bідбулося пошкодження структурного гена - ділянки молекули ДНК. Але це не призвело до заміни амінокислот у білку, тому що через деякий час пошкодження було ліквідовано. Це прояв такої властивості ДНК, як здатність до:

Мутації

Реплікації

Peпарації

Транскрипції

Зворотної транскрипції

3039 / 4299
У дитини, хворої на серповидноклітинну анемію, спостерігаються кілька патологічних ознак: анемія, збільшена селезінка, ураження шкіри, серця, нирок і мозку. Як називається цей випадок множинної дії одного гена?

Епiстаз

Плейотропія

Кодомінування

Полімерія

Комплементарність

3040 / 4299
В сечі хворого знайдені кетонові тіла. При якому захворюванні в сечі з'являються кетонові тіла?

Інфаркт нирки

Туберкульоз нирки

Цукровий діабет

Гострий гломерулонефрит

Сечокам'яна хвороба

3041 / 4299
Під час автомобільної аварії людина отримала сильний удар в епігастральну ділянку, внаслідок чого виникла зупинка серця. Що могло стати причиною таких змін серцевої діяльності?

Підвищення тонусу блукаючого нерва

Виділення кортизолу

Виділення адреналіну

Виділення альдостерону

Підвищення тонусу симпатичної нервової системи

3042 / 4299
На мікропрепараті зародка людини, взятого з мимовільного викидня, виявлено зародковий щиток, в якому розпізнаються два шари клітин: ендо- і ектодерма. На якому етапi ембріонального розвитку був ембріон?

Прогенезу

Гаструляції

Oрганогенезу

Нейруляції

Гістогенезу

3043 / 4299
Після механічної травми хворому наклали джгут на руку, щоб зупинити кровотечу. Нижче джгута рука зблідла, з'явилося відчуття заніміння. Цей стан є наслідком:

Венозного застою

Ангіоспастичної ішемії

Тромбозу

Обтураційної ішемії

Компресійної ішемії

3044 / 4299
Однією з реакцій матричного синтезу є реплікація. Яка нова молекула утворюється внаслідок цього з молекули ДНК?

Про-іPНК

іPHK

рPHK

ДНК

тРНК

3045 / 4299
До вірусологічної лабораторії iнфекційної лікарні звернувся хворий з метою пройти обстеження на ВІЛ- iнфекцію. Які з методів лабораторної діагностики ВІЛ-інфекції та СНIДу використовуються наразі в Україні?

Серологічний

Алергічний

Біологічний

Вiрусологічний

Бактеріологічний

3046 / 4299
Для проведення оперативного втручання в щелепно-лицевій ділянці з метою зменшення салівації застосовують холінергічні засоби. Який препарат з названих нижче Ви запропонуєте?

Адреналіну гідрохлорид

Дитилін

Прозерин

Атротіну сульфат

Лобеліну гідрохлорид

3047 / 4299
У хворого, що потрапив до неврологічного відділення, було виявлено посилення процесів гальмування в пентральній нервовій системі. Надлишок якого медіатора може призвести до цього?

Дофамін

Ацетилхолін

ГАМК

Адреналін

Норадреналін

3048 / 4299
Під час профілактичного огляду в одного працівника їдальні у фекаліях виявлено восьмиядерні цисти. Кому з найпростіших вони належать?

Токсоплазмі

Балантидію

Трихомонаді кишковій

Амебі кишковій

Лямблії

3049 / 4299
У хворого на малярію під час розтину спостерігається виражена жовтушність шкіри, склер та слизових оболонок. Селезінка збільшена в розмірі, аспідно-сірого кольору. Таке забарвлення селезінки обумовлене наявністю:

Лiпофусцину

Гемосидерину

Гематопорфірину

Меланіну

Гeмомеланіну

3050 / 4299
У хворого на rіпертонічну хворобу виявлено в крові збільшення концентрації вазопресину. На функцію якого органу впливає цей гормон?

Легень

Нирок

Печінки

Наднирників

Серця

3051 / 4299
У чоловіка 58-ми років клінічна картина гострого панкреатиту. Підвищення в сечі якої з перерахованих нижче речовин буде підтвердженням діагнозу?

Cечової кислоти

Амілази

Сечовини

Залишкового азоту

Альбуміну

3052 / 4299
У хворого видалено новоутворення шкіри, яке мас вигляд вузла щільної консистенції з сосочковою поверхнею, що нагадує цвітну капусту. Мікроскопічно пухлина складається з багатьох сосочків. Паренхіма сформована з покривного епітелію зі збільшеною кількістю шарів. В eпітелії збережені полярність клітин, стратифікація, цілісність власної мембрани. Строма пухлини розташована в центрі сосочків. Ваш діагноз:

Папілома

Аденома

Фіброаденома

Цистаденома

Фіброма

3053 / 4299
В зв'язку з наявністю злоякісної пухлини на язику хворому необхідно його видалити. В якому місці легко знайти язичну артерію та перев'язати її?

В сонному трикутнику

В лопаточно-трапецієвидному трикутнику

В лопаточно-підключичному трикутнику

В трикутнику Пирогова

В лопаточно-трахейному трикутнику

3054 / 4299
При огляді у хворого коронка 47 зуба торкається глибокого дефекту слизової оболонки щоки, краї якого щільні, чіткі, дно сірого кольору. Мікроскопічне дослідження біопсійного матеріалу зі стінки дефекту виявило на дні гнійний ексудат, під яким зона некротизованої тканини, глибшегрануляційна тканина, яка переходить у зрілу фіброзну. Яка патологія розвинулась на щоці?

Рак

Гостра виразка

Хронічна ерозія

Гостра ерозія

Хронічна виразка

3055 / 4299
На ЕКГ хворого виявлено скорочення тривалості інтервалу R-R. Як при цьому зміниться діяльність серця?

Зменшиться частота серцевих скорочень

Сповільниться частота і знизиться сила серцевих скорочень

Збільшиться частота серцевих скорочень

Зменшиться сила серцевих скорочень

Збільшиться сила серцевих скорочень

3056 / 4299
Дитина 5-ти років надійшла в ЛОР-відділення клінічної лікарні з діагнозом гнійне запалення середнього вуха. Захворювання розпочалося з запалення носоглотки. Крізь який канал(каналець) скроневої кістки інфекція потрапила в барабанну порожнину?

Барабанний

Сонний

Сонно-барабанні

М'язово-трубний

Барабанної струни

3057 / 4299
У хворої суглоби збільшені, болючі. У крові пацієнтки підвищений рівень уратів. Як називається така патологія?

Подагра

Цинга

Пелагра

Карiєс

Рахіт

3058 / 4299
У потертілого глибока різана рана у задній ділянці шиї. Який м'яз пошкоджено?

М. digastricus

M. levator scарulae

М. trapezius

M. mylohyoideus

М. sternocleidomastoideus

3059 / 4299
Для працівників атомної електростанці проводять періодичні медогляди, під час яких в першу чергу обстежують найбільш чутливу до іонізуючого випромінювання систему організму. Назвіть цю систему:

Кісткова

М'язова

Кровотворна

Епiтеліальні тканини

Hервова

3060 / 4299
У водолазів при швидкому підйомі з глибини на поверхню існує ймовірність розвитку декомпресійної хвороби, що може призвести до смерті внаслідок газової емболії. Який газ при цьому виділяється?

CO

CO2

N2

NO2

02

3061 / 4299
Хворому після радіоактивного опромінення лікар рекомендував збільшити в раціоні вміст рослинних олій - джерела полієнових жирних кислот. Назвіть кислоту, що містить три подвійні зв'язки:

Пальмітинова

Стеаринова

Ліноленова

Арахідонова

Олеїнова

3062 / 4299
У пацієнта 25-ти років спостерігається виражена м'язова слабкість. Вміст яких електролітів в плазмі крові доцільно визначити в першу чергу?

Iонів магнію

Іонів натрію

Іонів калію

Iонів калыцію

lонів хлору

3063 / 4299
У соматичних клітинах абортивного плода людини виявлено делецію короткого плеча 5-ї хромосоми. Вкажіть число аутосом у каріотипі цього організму:

45

46

44

48

47

3064 / 4299
У процесі старіння у людини спостерігається зменшення синтезу та секреції підшлункового соку, зменшення вмісту в ньому трипсину. Це призводить до порушення розщеплення:

Фосфоліпідів

Білків

Полісахаридів

Нуклеїнових кислот

Ліпідів

3065 / 4299
. Тирозин використовується в якості субстрату в процесі синтезу тироксину. Вкажіть хімічний елемент, який бере участь в цьому процесі:

Цинк

Залізо

Miдь

Кальцій

Йод

3066 / 4299
При посіві мікроорганізмів кишкової групи на середовище Ендо виростають або забарвлені, або безбарвні колонії. Ферментацією якого вуглеводу зумовлений цей процес?

Сахарози

Глюкози

Мальтози

Арабінози

Лактози

3067 / 4299
, як називається новий стан біосфери, у якому визначальним фактором розвитку є розумова діяльність людини:

Літocфера

Aтмосфера

Гідросфера

Hоocфера

Тропосфера

3068 / 4299
У пацієнта на місці гнійного запaлення шкіри (карбункул) утворився кeлoїдний рубець. На якій стадії запалення це відбувається?

Альтерації первинної

Ексудації

Альтерації вторинної

Проліферації

3069 / 4299
У хворого спостерігається втрата загальної чутливості на окремих ділянках тіла справа. Яка зі звивин великих півкуль головного мозку уражена?

Верхня скронева

Нижня скронева

Зацентральна

Передцентральна

Середня скронева

3070 / 4299
У хворого спостерігається остеопороз кісток, в крові - rіперкальциємія, гiпофосфатемія. Яка причина такого стану?

Пригнічення секреції паратгормону

Пригнічення секреції кортикостероїдів

Посилена секреція параттормону

Посилена секреція кортикостерoйдів

Посилена секреція тироксину

3071 / 4299
У пацієнта при обстеженні у неврoпатолога виявлена втрата чутливості тильної поверхні лівої кисті. Як називасться це явище?

Атонія

Алексія

Aстенія

Анестезія

Атаксія

3072 / 4299
До стоматолога звернувся хворий зі скаргами на асиметрію обличчя. Лікар зазначив, що на лівій половині обличчя брова стоїть нижче, на лобі немає складок, очна щілина вужча, очне яблуко виступає вперед. Функція якої пари черепно-мозкових нервів порушена?

VI

I

V

IV

VIІ

3073 / 4299
Пiсля обстеження у хворого виявлені симптоми акромегалії, Яка ендокринна залоза залучена до патологічного процесу?

Нейрогіпофіз

Наднирники

Аденогіпофіз

Щитоподібна залоза

Епіфіз

3074 / 4299
У юнака 17-ти років взяли катетером сечу з сечового міхура для дослідження. Клітини якого епітелію, що вистилають сечовий міхур, можуть бути виявлені при мікроскопії осаду сечі?

Одношаровий призматичний

Одношаровий кубічний

Перехідний

Багатошаровий зроговілий

3075 / 4299
Чоловіку 35-ти років, хворому на атопічний дерматит, був призначений лоратадин. Вкажіть механізм дії цього препарату:

Стимулює дофамінові рецептори

Блокує Н1-riстамінові рецептори

Блокує ГАМК-рецептори

Стимулює М-холінорецентори

Блокує β-адренорецептори

3076 / 4299
У хворого діагностовано стоматит, викликаний вірусом простого герпесу типу 1 і 2. Який засіб, що є аналогом нуклеозидів і перетворюється під впливом тимідинкінази, забезпечить вибіркову високоефективну противірусну терапію?

Ацетилцистеїн

Лаферон

Оксолін

Ремантадин

Ацикловір

3077 / 4299
У хворої 43-х років на тлі септичного шоку відзначаються тромбоцитопенія, зменшення фібриногену, поява в крові продуктів деградації фібрину, поява петехiальних крововиливів. Вкажіть причину виникнення цих змін:

ДВЗ-синдром

Eкзогенна iнтоксикація

Геморагічний діатез

Порушення продукування тромбоцитів

Аутоiммунна тромбоцитопенія

3078 / 4299
В ембріональному матеріалі виявлено порушення диференціації ентодерми. Зміни в розвитку яких органів можуть виникнути при даному процеci?

Слинних залоз

Нирок

Аорти

Серця

Шлунку

3079 / 4299
Під час приготування мазка з зубного нальоту і фарбування його за методом Грама студент в ході мікроскопії знайшов різноманітні мікроорганізми фіолетового і рожевого кольору. Який структурний компонент мiкроорганізму зумовлює різне сприйняття барвників?

Цитоплазма

Внутрішній периплазматичний простір

Клітинна стінка

Цитоплазматична мембрана

Зовнішня мембрана

3080 / 4299
У хворого важка нефропатія з масивним набряковим синдромом, що ускладнила хронічну бронхоектатичну хворобу. Лабораторні дослідження виявляють виражену протеїнурію, циліндрурію, значне зниження вмісту білка в сироватці крові, гіперліпемію, riпокаліємію та інші відхилення. Що є первинною і найбільш суттєвою ланкою в патогенезі набряків у цього хвоporo?

Підвищення гідростатичного тиску крові

Підвищення тиску позаклітинної рідини

Блокада лімфовідтоку

Зниження онкотичного тиску крові

Підвищення проникності мікросудин

3081 / 4299
У жінки народилася хвора на токсоплазмоз дитина. Жінка вважає, що заразилася токсоплазмою від подруги, яка нещодавно також народила хвору дитину. Який спосіб зараження людини токсоплазмою є НЕМОЖЛИВИМ?

Вживання напівсирого м'яса зараженої свійської тварини

Споживання немитих овочів

Пиття води, зараженої ооцитами

Контакт з кішкою

Контакт з хворою людиною

3082 / 4299
У хворого із вивихом щелепи лікар застосував засіб для короткочасного розслаблення м'язів з групи міорелаксантів. Виберіть цей засіб:

Піридостигміну бромід

Папаверину гідрохлорид

Дитилін

Новокаїн

Цититон

3083 / 4299
Щоб взяти спинномозкову рідину для дослідження, лікар повинен зробити пункцію підпавутинного простору спинного мозку. Між якими хребцями треба ввести голку, щоб не пошкодити спинний мозок?

l i ll поперекові

XII грудний i l поперековий

IV і V rpyдні

III i IV поперекові

XI і XIІ грудні

3084 / 4299
У хворого з крововиливом у передній гіпоталамус виникла поліурія. Не достатність якого гормону вплинула на зменшення реабсорбції води в канальцях нирок?

Альдостерону

Вазопресину

Окситоцину

Кальцитоніну

Адреналіну

3085 / 4299
І.M. Сєченов встановив, що втомлена кінцівка відновлює працездатнiсть швидше, якщо в період відпочинку друга кінцівка працює. Це дало можливість розробити вчення про:

Оптимум

Песимум

Втому

Парабіоз

Активний відпочинок

3086 / 4299
Деякі триплети iPHK (УАА, УАГ, УГА) не кодують амінокислоти, а є термінаторами в процесі зчитування iнформаці, тобто здатні припинити транскрипцію. Ці триплети мають назву:

Оператори

lнтрони

Eкзони

Стоп-кодони

Антикодони

3087 / 4299
У пацієнта діагностували виразкову хворобу шлунка та призначили антибактеріальне лікування. На який збудник воно спрямоване?

Е. сolli

CL. perfringens

Н. руlori

St. aureus

Cl. trachomatis

3088 / 4299
Для уточнення діагнозу хворому 15-ти років потрібно зробити сіалографію привушної залози. Де розташований отвір, крізь який буде введена рентгеноконтрастна маса?

На щоці в ділянці навпроти 2-го нижнього малого кутнього зуба

На щоці в ділянці навпроти 2-го верхнього великого кутнього зуба

Hа щоці в ділянці навпроти 2-го нижнього великого кутнього зуба

На щоці в ділянці навпроти 2-го верхнього малого кутнього зуба

3089 / 4299
Як відрізнясться рH венозної крові від артеріальної і чим це пояснити?

Більший, завдяки більшому вмісту СО2 в крові

Більший, завдяки виділенню О2 із організму

Менший, завдяки більшому вмісту СО2 в крові

Mенший, завдяки виділенню О2 із організму

Не відрізняється

3090 / 4299
У хворого, який перебуває на лікуванні з приводу вірусного гепатиту В, з'явилися ознаки печінкової недостатності. Які зміни в аналізі крові, що свідчать про порушення білкового обміну, найімовірніше будуть спостері гатися в цьому випадку?

Абсолютна гіпоальбумінемія

Абсолютна rіперфібриногенемія

Абсолютна гіперальбумінемія

Білковий склад крові не змінений

Абсолютна гіперглобулінемія

3091 / 4299
Досліджено ембріональний орган, в якому формуються перші формені елементи крові як тканини. Назвіть цей орган:

Тимус

Жовтковий мішок

Печінка

Селезінка

Червоний кістковий мозок

3092 / 4299
Для покращення мінералізації зубів лікарі-стоматологи призначають препарати Са2+. На які процеси він НЕ ВПЛИВАЄ в організмі?

Проведення збудження через синапс

Гемостаз

М'язове скорочення

Створення онкотичного тиску

Розвиток деполяризації в міокарді

3093 / 4299
У хворого на цукровий діабет після застосування цукрознижувальної терапії виник гіпоглікемічний стан. Надмірне вживання якого гормону могло призвести до цього стану?

Iнсуліну

Адреналіну

Кортизолу

Тироксину

Глюкагону

3094 / 4299
На гістологічному препараті представлені три нейрони: псевдоуніполярний, біполярний та мультиполярний. Скільки аксонів можна визначити у кожного з перерахованих типів клітин?

Жодного

Два

Три

Один

Багато

3095 / 4299
На прийомі у стоматолога під час маніпуляцій в ротовій порожнині жінка відчула себе погано: виник головний біль, посилилось серцебиття. При вимірюванні AТ встановлено, що систолічний тиск становить 170 мм рт.ст. Яка нормальна величина систолічного АТ у людини (мм рт.ст.)?

90-100

60-80

140-160

100-120

160-180

3096 / 4299
Який фермент має демінералізуючу дію - посилює розщеплення мінеральних компонентів тканин зуба?

Глюкозо-6-фосфатаза

Гiкorенфосфорилаза

Фосфотрансфераза

Лужна фосфатаза

Кисла фосфатаза

3097 / 4299
При досліджені трупа жінки 59-ти років, яка померла від гострої серцевої недостатності, в стінці лівого шлуночка виявлено ділянку жовтого кольору 2,5х2 см, неправильної форми, тістоподібної консистенції. На ендокарді, відповідно до неї, утворився тромб, а на епікарді – фібринозні нашарування. Яка локалізація інфаркту стосовно стінки серця мала місце?

Субендокардіальний

Трансмуральний

Iнтрамуральний

Субепікардіальний

3098 / 4299
В ході мікроскопії з імерсійною системою вивчено препарат-мазок з культури стрептобацил, зафарбований за методом Ожешко. Яку структурну особливість бактерій досліджено?

Включення

Джгутики

Спори

Будова клітинної стінки

Kапсула

3099 / 4299
Хворого оперують з приводу травми скронево-нижньощелепного суглоба. Під час розсічення виявлена структура, що доповнює конгруентність суглобових поверхонь. Що це за структура?

Зв'язка

Диск

Складка

Губа

Меніск

3100 / 4299
Який метод мікробіологічної діагностики треба використати, щоб підтвердити або спростувати діагноз: холера?

Бактеріологічний

Алергічний

Біологічний

Вірусологічний

Бактеріоскопічннй

3101 / 4299
У десятирічної дитини виявлено гельмінтоз. Які зміни у лейкоцитарній формулі крові будуть спостерігатися у цьому разі?

Зросте кылькысть сегментоядерних нейтрофілів

Зросте кількість еритроцитів

Зросте кількість тромбоцитів

Зросте кількість еозинофілів

Зросте кількість базофілів

3102 / 4299
Жінка з групою крові AB(O) Rh(-) вагітна вдруге рсзус-позитивним плодом. Після перших пологів резус-позитивним плодом імунопрофілактика не проводилася. Яке ускладнення може виникнути під час другої вагітності?

Маткова кровотеча

Передчасні пологи

Внутрішньосудинний гемоліз еритроцитів плода

Внутрішньосуднний гемоліз еритроцитів матері

3103 / 4299
У підтриманні певної пози задіяні повільні скелетні м’язи, які мають значні резерви кисню. Яка речовина із нижченаведених бере участь в запасанні кисню у цих м'язах?

Міоглобін

Цитохром

Гемоглобін

Кальмодулін

Креатинфосфат

3104 / 4299
В умовному експерименті дія токсичної речовини порушує механізм передавання нервового імпульсу. Яка структура забезпечує виконання цієї функції?

Нейролема

Синапс

Нейрофібрили

Мітохондрія

Субстанція Ніссля

3105 / 4299
Пацієнту встановлено діагноз; гострий риногенний гайморит (запалення верхньощелепної пазухи). Як інфекція моїла проникнути в гайморову пазуху?

Крізь верхній носовий хід

Крізь середній носовий хід

Крізь нижній носовий хід

Крізь передню черепну ямку

Крізь середню черепну ямку

3106 / 4299
У чоловіка спостерігаються збільшені і болючі суглоби, у сироватці крові виявлено підвищений вміст уратів. Обмін яких речовин порушений у пацієента?

Пуринів

Холестерину

Фенілаланіну

Глюкози

3107 / 4299
Пацієнту проведено хімічний аналіз слини з метою оцінювання її антибактеріальної активності. Який компонент слини відповідає за ці властивості?

Холестерол

Церулоплазмін

Амілаза

Лізоцим

Паротин

3108 / 4299
У жінки з ендокринною патологією виявлено підвищення основного обміну на 15%. Збільшення продукування якого гормону може обумовлювати цей патологічний етап?

Альдостерону

Глюкагону

Трийодтироніну

Соматостатину

Тиреокальцитоніну

3109 / 4299
Під час розтину тіла чоловіка віком 76 років, який номер від серцево-судинної недостатності, у інтимі аорти виявлені об'ємні біло-жовті бляшки з дрібнозернистими аморфними масами в центрі, які заглиблюються в товщину стінки. Для якої стадії атеросклерозу характерні такі патологічні зміни стінки аорти?

Атероматозу

Атерокальцинозу

Ліпоїдозу

Звиразкування

Ліпосклерозу

3110 / 4299
Пацієнту з діагнозом: ішемічна хвороба серця, для профілакіики тромбоутворення був при призначений лікарський засіб із фармакотерапевтичної групи нестероїдних протизапальних препаратів. Вкажіть цей лікарський іасіб.

Індометацнн

Кислота мефенамова

Ібупрофен

Кислота ацетилсаліцилова

Піроксикам

3111 / 4299
Піддослідному собаці через зонд у дванадцятипалу кишку ввели слабкий розчин хлористоводневої кислоти. До підсилення секреції якого гормону це призведе?

Гастрину

Секретину

Холсцистокініну

Гістаміну

Нейротензину

3112 / 4299
Який полісахарид природного походження використовують як антикоагулянт?

Гіалуронова кислота

Гепарин

Дерматансульфат

Декстран

Хондроітинсульфат

3113 / 4299
У пацієнта виявили збільшення активності G-клітин шлунку. Які зміни травлення у шлунку при цьому виникають?

Зменшення кислотності

Підвищення секреції гастрину

Зниження активності ферментів

Утворення великої кількості слизу

Пригнічення моторики шлункау

3114 / 4299
Чоловік віком 35 років скаржиться на біль у ділянці печінки. З’ясовано, що хворий захоплюється риболовлею і часто вживає недосмажену на вогнищі рибу. У фекаліях виявлені дуже маленькі яйця гельмінту, темного кольору, з кришечкою овальної форми Який вид гельмінтозу найімовірнипийу цьому разі?

Фасціольоз

Дикроцеліоз

Шистосомоз

Парагонімоз

Опісторхоз

3115 / 4299
Кровотечу, що виникла у дитини після видалення зуба, не вдавалося зупинити упродовж 6 годин. Під час дослідження системи гемостазу виявлено різке зменшення вмісту VIII фактору зсідання крові. За яким типом успадковується це захворювання?

Аутосомно-рецесивний

Неповне домінування

Полігонний

Зчеплене зі статевою хромосомою

Аутосомно-домінантний

3116 / 4299
До комплексної терапії хворого на бронхопневмонію, що супроводжується виснажливим сухим кашлем, лікар включив муколітичний препарат, який деполімеризує мукопротеїди. Укажіть цей препарат.

Неодикумарин

Кодеїн

Ацетилцистсін

Атенолол

Строфантин

3117 / 4299
У жінки виникла загроза передчасного переривання вагітності. З недостатністю якого гормону це пов’язано?

Окситоцину

Естрадіолу

Тестостерону

Альдостерону

Прогестерону

3118 / 4299
Пацієнтка, яка хворіє на бронхіальну астму, для лікування захворювання щоденно 3 рази на день приймала таблетки під язик. Через деякий час вона поскаржилася на тахікардію, біль у ділянці серця, запаморочення, зниження артеріального тиску, швидку втомлюваність. Який лікарський засіб приймала жінка?

Дигітоксин

Анаприлін

Адреналін

Ізадрин

3119 / 4299
Під час тренувань спортсмени-підводники на тривалий час затримують дихання. З якою метою це робиться?

Зниження чутливості нейронів дихального центру до О2

Підвищення чутливості нейронів дихального центру до О2

Підвищення чутливості нейронів дихального центру до СО2

Зниження чутливості нейронів дихального центру до СО2

Зниження чутливості рецепторів розтягу легень

3120 / 4299
У жінки віком 45 років через кілька років після переїзду до Закарпаття з’явилися слабкість, сонливість, апатія, зниження пам’яті, набряки. Після обстеження встановлено діагноз: ендемічний зоб. Нестача якого мікроелементу у воді та їжі може бути причиною цього захворювання?

Кальцію

Магнію

Фтору

Заліза

Йоду

3121 / 4299
Водій віком ЗО років під час ДТП отримав черепно-мозкову травму. Він став скаржитися на спрагу та збільшення добової кількості сечі. Який відділ головного мозку в нього уражений?

Гіпоталамус

Ретикулярна формація

Мозочок

Тім'яна частка мозку

Передня центральна звивина

3122 / 4299
Розчин хлораміну Б широко застосовують у медичній практиці. Укажіть, яка фармакологічна дія відсутня у цього засобу.

Протиалергічна

Дезодоруюча

Антисептична

Знебарвлювальна

Сперматоцидна

3123 / 4299
Вкажіть, який серцевий глікозид швидкої дії застосовують під час гострої серцевої недостатності.

Валідол

Аміодарон

Строфантин

Кордіамін

Нітрогліцерин

3124 / 4299
Для профілактики та лікування якого захворювання використовуються антитоксичної сироватки?

Дизентерії

Гонореї

Туберкульозу

Кашлюка

Дифтерії

3125 / 4299
У людини спостерігається дефіцит вітаміну D, що призводить до недостатнього всмоктування:

Хлору

Заліза

Води

Натрію

Кальцію

3126 / 4299
У пацієнтки, яка хворіє на вторинний сифіліс, з’явилися вогнища депігментації шкіри у верхніх відділах спини. Яка патологія шкіри розвинулася у жінки?

Метаплазія

Лейкодерма

Дисплазія

Лейкоплакія

Параксратоз

3127 / 4299
Пацієнт скаржиться на біль під час жування, особливо, коли висуває нижню щелепу вперед і зміщує її у бік. Функція яких м’язів порушена?

Щелепно-під'язикових

Медіальних крилоподібних

Латеральних крилоподібних

Жувальних

Скроневих

3128 / 4299
На рибосомі відбувається трансляція. Рибосома дійшла до кодону УАА. Цей кодон у процесі біосинтезу поліпептиду не розпізнається жодною т-РНК, тому синтез поліпептидного ланцюга закінчився. Сигналом якого процесу є кодон УАА?

Термінації

Посттрансляційної модифікації

Елонгації

Ініціації

3129 / 4299
Пацієнту віком 25 років для уточнення діагнозу треба зробити сіалографію привушної залози. Де розташований отвір, через який буде введена рентгеноконтрастна маса?

На щоці в ділянці навпроти 2-го верхнього великого кутнього зуба

На plica sublingualis

На caruncula sublingualis

На щоці в ділянці навпроти 2-го нижнього великого кутнього зуба

На щоці в ділянці навпроти 2-го верхнього малого кутнього зуба

3130 / 4299
У пацієнта хронічний неврит трійчастого нерва. Який із травних процесів буде порушено найбільше у цьому разі?

Формування смакових відчуттів

Слиноутворення

Жування

Ковтання

Слиновиділення

3131 / 4299
Під час операції на щитоподібній залозі через захворювання на Базедову хворобу помилково були видалені паращитоподібні залози. У пацієнтки виникли судоми та тетанія. Обмін якого біоелементу було порушено?

Калію

Заліза

Кальцію

Натрію

Магнію

3132 / 4299
Під час огляду порожнини рота у жінки віком 52 роки спостерігаються гіперемія, набряк та кровоточивість ясен нижньої щелепи. Різці нижньої щелепи мають рухомість ІІ ступеня, наявні зубо-ясенні кишені, під час натискування з-під слизової ясен виділяється жовто- зелений ексудат. Для якого захворювання характерні ці симптоми?

Пародонтит

Періостит

Стоматит

Остеомієліт

Періодонтит

3133 / 4299
Пацієнту зі злоякісною пухлиною для усунення нестерпного болю призначили наркотичний анальгетик. Який механізм знеболюючої дії цього лікарського засобу?

Гальмування гістамінергічних рецепторів

Активація В2-дофамінових рецепторів

Активація опіатних рецепторів

Гальмування холінергічних рецепторів

Гальмувапня серотонінергічних рецепторів

3134 / 4299
Звуження великої судини спричинило погіршення відтоку крові з лівого шлуночка серця. Яка судина зазнала патологічних змін?

Нижня порожниста вена

Легеневий стовбур

Легенева вена

Верхня порожниста вена

Аорта

3135 / 4299
Після видалення зуба у чоловіка віком 40 років відбулася активна регенерація ранової поверхні. Враховуючи функції органсл клітини, визначте, які з них найперші забезпечили процес регенерації.

Мітохондрії

Рибосоми

Пероксисоми

Лізосоми

3136 / 4299
Дитина неспокійно спить, уві сні скрегоче зубами, часто розчухує ділянку анального отвору. Під час лабораторного досліджепня у зішкрібах із периаиальних складок дитини були виявлені гельмінти довжиною до 1 см, білого кольору. Який гельмінт спричинив такі симптоми у дитини?

Трихінела

Гострик

Вугриця кишкова

Ціп'як карликовий

Аскарида людська

3137 / 4299
У препараті сполучної тканини, забарвленому гематоксиліном-еозином, спостерігаються ізогенні групи клітин, оточені базофільною міжклітинною речовиною. Волокнисті структури не виявлені. Яка це сполучна тканина?

Пластинчаста кісткова

Щільна волокниста

Гіалінова хрящова

Еластична хрящова

Пухка волокниста

3138 / 4299
Досить часто причиною набутих імунодефіцитів є інфекційне ураження організму, під час якого збудники безпосередньо розмножуються в клітинах імунної системи і руйнують їх. Виберіть серед нижченаведених ті захворювання, за яких спостерігаються такі патологічні процеси.

Дизентерія, холера

Інфекційний мононуклеоз, СНІД

Туберкульоз, мікобактеріоз

Ку-гарячка, висипний тиф

Поліомієліт, гепатит А

3139 / 4299
Пацієнт упродовж тривалого часу вживав антибіотики широкого спектру дії, що викликало зниження апетиту, нудоту, пронос з гнилісним запахом випорожнень, схуднення. Яка побічна дія антибіотиків виникла у процесі лікування?

Пряма подразнювальна дія

Дисбактеріоз

Нсфротоксична дія

Гепатотоксична дія

Алергічна реакція

3140 / 4299
На певному етапі онтогенезу людини між кровоносними системами матері та плода виникає фізіологічний зв’язок. Який провізорний орган виконує цю функцію?

Жовтковий мішок

Амніон

Плацента

Алантоїс

Серозна оболонка

3141 / 4299
Під час обстеження порожнини рота у пацієнтки спостерігається гіперемія слизової оболонки щік, дна порожнини рота, м’якого та твердого піднебіння. Слизова оболонка тьмяна, з великою кількістю дрібних виразок, вкрита непрозорим слизом. Який імовірний діагноз?

Хронічний рсцидивуючий афтозний стоматит

Катаральний стоматит

Виразковий стоматит

Гангренозний стоматит

Гнійний стоматит

3142 / 4299
Під час аналізу крові лікар-лаборант зробив додатковий висновок, що вона належить людині жіночої статі. Особливості будови яких форменних елементів дають змогу зробити такий висновок?

Моноцитів

Нейтрофільних лейкоцитів

Еритроцитів

Базофільпих лейкоцитів

Лімфоцитів

3143 / 4299
Під час розтину тіла шестимісячної дитини, яка померла внаслідок тривалого інфекційного захворювання, виявлено зменшення маси та розмірів тимусу. Під час мікроскопічного дослідження в тимусі виявлено атрофію часточок, значне зменшення кількості лімфоцитів, інверсію шарів, збільшення кількості тимічних тілець Який патологічний процес найімовірніше розвинувся в тимусі?

Акцидентальна трансформація

Тимома

Агенезія тимусу

Вікова інволюція

3144 / 4299
У якому внутрішньому органі відбувається біотрансформація (метаболічні перетворення) більшості лікарських речовин під час їх надходження в організм людини?

Легені

Нирки

Кишечник

Шкіра

Печінка

3145 / 4299
Під час ендоскопічного дослідження лікар виявив порушення цілісності стінки шлунку в межах слизової оболонки. Укажіть, яким типом епітелію в нормі вистелена зсередини стінка шлунку.

Одношаровим призматичним залозистим

Багатошаровим плоским зроговілим

Псевдобагатошаровим

Багатошаровим плоским незроговілим

Перехідним

3146 / 4299
На мікропрспараті серця представлені клітини прямокутної форми з центрально розташованим ядром та розвиненими міофібрилами, зв'язані між собою вставними дисками. Яка функція серця пов’язана з цими клітинами?

Скорочення серця

Регенераторна

Ендокринна

Проведення імпульсу

Захисна

3147 / 4299
Яка речовина в складі слини синтезується в слинних залозах та поза ними і надає слині густий слизовий характер?

Сульфати

Муцин

Мальтаза

Фосфати

Амілаза

3148 / 4299
Для вправлення вивиху нижньої щелепи треба відвести її донизу. Наявність якої анатомічної структури обумовлює таку дію?

Вирізки нижньої щелепи

Нижньощелепної ямки скроневої кістки

Виросткового відростка нижньої щелепи

Суглобового горбка скроневої кістки

3149 / 4299
У потерпілого після ДТП гематома спинного мозку супроводжується загрудинним болем, тахікардією і підвищенням артеріального тиску. Ураження яких сегментів спинного мозку є причиною такого стану пацієнта?

L1-L3

C6-C8

Th1-Th5

S1-S3

3150 / 4299
Жінці, якій встановлено діагноз сифіліс, треба призначити лікування. З анамнезу пацієнтки відомо, що вона має алергію на препарати пеніцилінового ряду. Який антибіотик треба призначити у цьому разі?

Цефтріаксон

Ампіцилін

Ампіокс

Амоксицилін

Аугментин

3151 / 4299
Для полоскання ротової порожнини пацієнту призначили лікарський засіб із групи інтрофуранів у таблетках для зовнішнього застосування. Який із нижченаведеннх препаратів був призначений хворому?

Фурагін

Фурацилів

Фуразолідон

Фуроссмід

Фурадонін

3152 / 4299
Адреналін використовується для подовження дії новокаїну під час інфільтраційної анестезії. З якою дією адреналіну пов'язаний цей ефект?

Розширення судин

Пригнічення тканинних естераз

Звуження судин

Пригнічення функцій нервових закінчень і провідників

Потенціювання дії новокаїну на рівні ЦНС

3153 / 4299
До клініки госпіталізовано пацієнта, який має сильний головний біль, ригідність м’язів потилиці, повторне блювання, біль під час перкусії черепа, підвищену чутливість до світлових подразників. Установлено попередній діагноз: менінгіт. Для підтвердження діагнозу призначено проведення спинномозкової пункції. Визначте анатомічне місце її проведення.

Між 12 грудним і 1 поперековим хребцями

Між 11 і 12 грудними хребцями

Між 3 і 4 поперековими хребцями

Між 5 поперековим хребцем і основою крижів

Між 1 і 2 поперековими хребцями

3154 / 4299
Пацієнтка, якій діагностовано ішемічну хворобу серця, отримувала антиангінальний препарат. Він має такі властивості: розширює вінцеві артерії та периферичні судини (артеріальні й венозні), зменшує погребу міокарда в кисні, поліпшує ендокардіальний кровообіг. Укажіть цей лікарський засіб.

Валідол

Папаверин

Нітрогліцерин

Дибазол

Еуфілін

3155 / 4299
Під час розтину трупа чоловіка віком 46 років, який тривалий час хворів на хронічний бронхіт, виявлено, що легені збільшені в об’ємі, не спадаються, підвищеної повітряності, бліді, тістоподібної консистеніцї, ріжуться з хрускотом. Під час мікроскопічного дослідження встановлено, що на всіх ділянках легень проксимальні відділи ацинусів різко розширені, стінки альвеол стоншені, капіляри в них недокрівні. Якій клініко-морфологічній формі емфіземи відповідає така морфологічна картина?

Старечій

Вікарній

Хронічній вогнищевій

Хронічній дифузній обструктивній

Ідіопатичній

3156 / 4299
Під час операції на головному мозку подразнення кори великих півкуль викликало у пацієнта тактильні та температурні відчуття. Яку звивину подразнювали?

Прецентральну

Верхню латеральну

Постцентральну

Парагіпокампальну

Поясну

3157 / 4299
Під час обстеження головного мозку на комп'ютерному томографі лікар виявив пухлину, що локалізується на ділянці нижнього слиновидільного ядра. Який це відділ головного мозку?

Довгастий мозок

Проміжний мозок

Мозочок

Міст

Середній мозок

3158 / 4299
У травмованого чоловіка рана скроневої ділянки, з якої яскраво-червоною цівкою витікає кров. Яка судина пошкоджена?

A. auricularis posterior

А. facialis

А. temporalis superficialis

A. occipitalis

A. maxillaris

3159 / 4299
Яка кислота - проміжний продукт циклу трикарбонових кислот — бере участь у зв’язуванні іонів кальцію?

Янтарна

Яблучна

Оцтова

Лимонна

3160 / 4299
Пацієнт з діагнозом: цукровий діабет після введення інсуліну втратив свідомість, виникли судоми. Який імовірний результат біохімічного аналізу крові на вміст цукру у цьому разі?

8 ммоль/л

5.5 ммоль/л

33 ммоль/л

1.5 ммоль/л

10 ммоль/л

3161 / 4299
У пацієнта об’єктивно спостерігається: права носогубна складка розгладжена, права очноямкова щілина розширена (її не вдається закрити під час примружування, тому що повіки не змикаються), виникають утруднення під час розмови та вживання їжі (їжа застрягає між щокою і зубами). Який нерв уражено?

N. glossopharyngeus sinister

N. facialis dexter

N. abducens dexter

N. vagus dexter

N. trigeminus dexter

3162 / 4299
У чоловіка, якому діагностовано: глосит, на язиці спостерігається зникнення сосочків, почервоніння та печіння. Під час лабораторного аналізу крові виявлено: еритроцити — 2,2 1012/л, гемоглобін — 103 г/л, колірний показник — 1,4. Який вид анемії спостерігається у пацієнта?

Залізорефрактерна

α-таласемія

В12-фолієводефіцитна

β-таласемія

Залізодефіцитна

3163 / 4299
За ЗО хвилин після лікування у лікаря-стоматолога на шкірі обличчя і слизовій рота пацієнта з’явилися червоні плями, що сверблять. Установлений діагноз: кропивниця. Яка з біологічно активних речовин, що викликають розширення судин та появу свербежу, виділяється при цьому типі алергічної реакції?

Гістамін

Інтерлейкін-1

Брадикінін

Лсйкотрієн В4

Простагландин Е2

3164 / 4299
Чоловік хворіє на цукровий діабет, що супроводжується гіперглікемією натще понад 12 ммоль/л. Рівень якого білка плазми крові дає можливість ретроспективно (за попередні 4-8 тижнів до обстеження) оцінити рівень глікемії у пацієнта?

Фібриногену

Глікозильованого гемоглобіну

Альбуміну

С-реактивного білка

Цсрулоплазміну

3165 / 4299
У видаленій матці жінки віком 55 ро-ків патологоанатом у товщі міометрію виявив щільний вузол діаметром 5 см із чіткими межами. На розрізі тканина вузла волокниста, сіро-рожевого кольору, з хаотичним розташуванням пучків волокон. Мікроскопічно пухлина складається з гладком’язових клітин, що утворюють пучки різної товщини, які йдуть у різних напрямках, і прошарків місцями гіалінізованої сполучної тканини. Яка пухлина розвинулася у жінки?

Міосаркома

Фіброміома

Фібросаркома

Фіброма

Рабдоміома

3166 / 4299
Чоловік має високий зріст, великі кінцівки, відвислу нижню губу та великий ніс. Визначте, підвищення секреції якої залози у цього пацієнта.

Епіфіза

Щитоподібної

Прищитоподібної

Передньої частки гіпофіза

3167 / 4299
Під час аналізу крові виявлено знижений вміст гемоглобіну. Яка функція крові тоді порушується у цьому разі?

Транспорт гормонів

Зсідання

Забезпечення імунітету

Транспорт газів

Транспорт поживних речовин

3168 / 4299
Який представник типу Членистоногі є переносником збудників бубонної чуми?

Клоп

Москіт

Блоха

Комар

Воша

3169 / 4299
Який представник типу Членистоногі є переносником збудників бубонної чуми?

Москіт

Клоп

Блоха

Воша

Комар

3170 / 4299
Під час бактсріоскопічного дослідження мазків спинномозкової рідини, забарвлених за Романовським-Гімзою, виявлено найпростіших у формі півмісяця з загостреними кінцями, блакитною цитоплазмою та рубіново-червовим ядром. Для збудника якого захворювання характерні ці ознаки?

Токсоплазмоз

Лейшманіоз

Трипаносомоз

Малярія

Амебіаз

3171 / 4299
На зрізі лімфатичного вузла в мозкових тяжах виявлено велику кількість клітин із такою морфологією: розміри від 7 до 15 мкм, базофільна цитоплазма, ядро розташоване ексцентрично, з малюнком хроматину, що нагадує спиці в колесі. Відомо, що ці клітини - продуценти антитіл. Які клітини мають структуру?

Макрофаги

Адипоцити (жирові клітини)

Фібробласти

Тучні клітини

Плазмоцити

3172 / 4299
Унаслідок пухлини гіпофіза під час рентгенографії виявлено руйнування і збільшення ямки турецького сідла. Яка кісткова порожнина при цьому уражена?

Барабанна порожнина

Лицевий канал

Зоровий канал

Пазуха клиноподібної кістки

Сонний канал

3173 / 4299
До кардіологічного відділення лікарні госпіталізовано чоловіка віком 49 років з інфарктом міокарда. Які зміни клітинного складу периферичної крові індуковапі некротичними змінами в міокарді?

Нейтрофільний лейкоцитоз

Тромбоцитопенія

Лімфопенія

Моноцитоз

Еозинофільний лейкоцитоз

3174 / 4299
У людини під час емоційного збудження спостерігається прискорення та посилення частоти серцевих скорочень, підвищення тиску, розширення зіниць, збільшення вентиляції легень. Який гормон є причиною такого стану людини?

Інсулін

Альдостерон

Адреналін

Тестостерон

Паратгормон

3175 / 4299
Під час аускультації серця було встановлено роздвоєння І тону в V міжреберному просторі по ссрсдньоключичній лінії. Патологію якого клапана аускультував лікар?

Клапана верхньої порожнистої вени

Двостулкового

Тристулкового

Клапана аорти

Клапана легеневого стовбура

3176 / 4299
Пацієнту з великими опіками зробили пересадку донорської шкіри. Але на 8-му добу трансплантат набряк, його колір змінився і на 11-ту добу розпочалася реакція відторгнення. Які клітини беруть участь у цьому процесі?

В-лімфоцити

Еризроцити

Т-лімфоцити

Еозинофіли

Базофіли

3177 / 4299
У чоловіка виявлено твердий шанкр на статевих органах. Попередній діагноз — сифіліс. Яка форма інфекції у цього пацієнта?

Первинна інфекція

Вторинна інфекція

Змішана інфекція

Рецидив

Суперіпфекція

3178 / 4299
У процесі диференціації зародкової ектодерми утворюються: нервова трубка, нервові гребені, плакоди, шкірна ектодерма та прехордальна пластинка. Як називається процес формування нервової трубки?

Гаструляція

Нейруляція

Сомітогенез

Гістогенез

Органогенез

3179 / 4299
До лікаря звернулася пацієнтка віком 38 років зі скаргами на те, що після перенесеного гострого вірусного респіраторного захворювання вона не відчуває дотику їжі, а також болю та температури на передній частині (2/3) язика (обпекла язик гарячим часм). Визначте, яка з гілок якого нерва була уражена.

Язикові гілки язикоглоткового нерва

Верхній гортанний нерв блукаючого нерва

Язиковий нерв нижньощелепного нерва трійчастого нерва

Барабанна струна лицевого нерва

Язикові гілки під'язикового нерва

3180 / 4299
Визначення якого ферменту в крові є найбільш інформативним у перші години після виникнення інфаркту міокарда?

Аланінамінотрансфсрази

Глутаматдегідрогенази

Лактатдегідрогенази

Креатинфосфокінази

Аспартатамінотрансферази

3181 / 4299
Скільки сегментів у шийному відділі спинного мозку?

5

8

2

7

12

3182 / 4299
Експериментальне дослідження мембранних іонних струмів у динаміці розвитку потенціалу дії показало, що лавиноподібний вхід іонів натрію до клітини спостерігається у фазі:

Гіперполяризаційного слідового потенціалу

Реверсполяризації

Деполяризаційного слідового потенціалу

Реполяризації

Деполяризації

3183 / 4299
До кардіологічного відділення госпіталізована жінка, якій діагностовано ішемічну хворобу серця. Який лікарський засіб треба ввести хворій для припинення нападу миготливої аритмії?

Левоміцетин

Біцилін-5

Дигоксин

Адреналіну гідрохлорид

Кислота аскорбінова

3184 / 4299
Під час захворювань дихальної системи та розладів кровообігу порушується транспорт кисню, що супроводжується гіпоксією. За таких умов енергетичний обмін здійснюється шляхом анаеробного гліколізу. Який продукт у цьому разі утворюється та накопичується у крові?

Фумарова кислота

Лимонна кислота

Піровиноградна кислота

Молочна кислота

Глутамінова кислота

3185 / 4299
У пацієнта, який отримав радіаційне ураження, з'явилися ознаки геморагічного синдрому. Який механізм має найбільше значення у патогенезі цього захворювання?

Еритропенія

Нейтропенія

Еозинопенія

Тромбоцитопенія

Лімфопенія

3186 / 4299
Пацієнта з опіками тіла оглянув лікар і встановив, що ураження шкіри є неглибокими. Укажіть, за допомогою клітин якого шару буде відбуватися регенерація ураженого епідермісу.

Базального

Зернистого

Рогового

Блискучого

3187 / 4299
Під час повного (з водою) аліментарного голодування у людини розвиваються генералізовані набряки. Який із патогенетичних факторів у цьому разі є провідним?

Зниження гідростатичного тиску міжклітинної рідини

Підвищення осмотичного тиску міжклітинної рідини

Зниження онкотичного тиску плазми крові

Зниження осмотичного тиску плазми крові

Підвищення онкотичного тиску тканинної рідини

3188 / 4299
Під час вивчення розвитку зубів у людини в ембріональний і постембріональний період було встановлено, що вони є похідними:

Ектодерми і ентодерми

Мезодерми

Ектодерми

Ектодерми і мезодерми

Ентодерми і мезодерми

3189 / 4299
Під час мікроскопічного дослідження бляшкоподібного утвору, видаленого з бокової поверхні язика, у чоловіка із зубними протезами виявлено значне потовщення епітеліального шару з явищами папакератозу, гіперкератозу та акантозу. У сполучній тканині виявлені дрібні круглоклітинні інфільтрати. Діагностуйте це захворювання.

Хронічний стоматит

Лейкоплакія

Хронічний глосит

Іхтіоз

Гунтерівський глосит

3190 / 4299
Під час видалення верхнього лівого третього моляра у пацієнта виникла значна кровотеча. Ураження якої артерії спричинило кровотечу?

Aa.alveolares superiores anteriores

А. facialis

А. alveolaris inferior

А. alveolaris superior posterior

А. infraobitalis

3191 / 4299
Під час операції на щелепно-лицьовій ділянці пацієнту для зниження секреції слинних залоз увели атропіну сульфат. У нього виникла тахікардія, спостерігається сухість і гіперемія шкіри, параліч акомодації та підвищення внутрішньоочного тиску. Який антагоніст треба використати у цьому разі?

Скополаміну гідробромід

Прозерін

Платифіліну гідротартрат

Атенолол

Дипіридамол

3192 / 4299
У пацієнта з діагнозом: ниркова недостатність, виявлено різке зниження вмісту натрію в сироватці крові. Спостерігаються бліді пухкі набряки обличчя, які з'являються зранку. Визначте речовину, що входить до міжклітинного матриксу сполучної тканини та зв’язує іони натрію, які надходять із кровоносного русла.

Колаген

Гіалуронова кислота

Фібронектин

Проколаген

Еластин

3193 / 4299
На клітину подіяли речовиною, яка спричинила порушення цілісності мембран лізосом. Який процес відбуватиметься з клітиною у цьому разі?

Дегенерація

Диференціація

Автоліз

Спеціалізація

Трансформація

3194 / 4299
Лікар-стоматолог-хірург видалив у пацієнта кісту нижньої щелепи в ділянці другого премоляра. Кіста була сформована в альвеолярному краю щелепи, стінка її тонка, у порожнині розташований рудиментарний зуб. Мікроскопічно порожнину кісти вистилає плоский багатошаровий епітелій. Яке захворювання розвинулося у пацієнта?

Радикулярна кіста

Кістогранульома

Резидуальна кіста

Фолікулярна кіста

Парадентальна кіста

3195 / 4299
Чоловік віком ЗО років звернувся до лікаря зі скаргами на підвищення температури тіла до 38°(7, слабкість, біль у горлі. Під час обстеження з'ясувалося, що язик хворого вкритий білим нальотом. Які гістологічні структури язика беруть участь в утворенні цього нальоту?

Сполучнотканинна основа всіх сосочків язика

Епітелій ниткоподібішх сосочків

Епітелій грибоподібних сосочків

Епітелій листоподібних сосочків

Епітелій жолобкуватих сосочків

3196 / 4299
Яка амінокислота із нижченаведених кодується одним триплетом?

Лейцин

Метіонін

Аланін

Серин

Лізин

3197 / 4299
У пацієнта порушена парасимпатична іннервація привушної слинної залози. З боку якого вузла можлива патологія?

Ganglion geniculi

Ganglion ptcrygopalatiniun

Ganglion submandibular?

Ganglion trigeminale

Ganglion oticum

3198 / 4299
У чоловіка, який переніс інсульт та перебуває у нерухомому положенні спостерігається недостатність кровообігу, шкіра та м’які тканини над крижами почорніли і набрякли. Після відторгнення епідермісу в чорних тканинах з'явилися виразки. Який процес розвинувся у пацієнта?

Суха гангрена

Інфаркт

Флегмона

Пролежні

Абсцес

3199 / 4299
За 8 днів після хірургічної операції у пацієнта розвинувся правець. Лікар припустив, що збудником правця став контамінований шовний матеріал, який відправили до бактеріологічної лабораторії для проведення мікробіологічного дослідження. Яке живильне середовище треба використати для первинного посіву шовного матеріалу?

Кітта-Тароцці

ЖСА

Гіса

Ендо

Сабуро

3200 / 4299
Під час морфологічного дослідження скелетних м’язів мишей, які тривалий час плавали у басейні, виявлено збільшення кількості мітохопдрій із багатьма кристами і просвітленим матриксом. Яка функція клітини перебуває у надзвичайно напруженому стані?

Синтетична

Захисна

Транспортна

Секреторна

Енергетична

3201 / 4299
Через декілька годин після травми зуба в пульпі спостерігаються гіперемія судин та виражений набряк тканини з поодинокими нейтрофілами та лімфоцитами. У нервових волокнах незначні дистрофічні зміни. Який найбільш імовірний діагноз?

Гранулюючий пульпіт

Фіброзний пульпіт

Гангренозний пульпіт

Серозний пульпіт

Гнійний пульпіт

3202 / 4299
Під час дослідження кровотоку в судинах головного мозку лікар встановлює датчик над артерією, що проходить у поперечних отворах шийних хребців. Яку артерію обстежує лікар?

A. vеrtebralis

A. cerebri anterior

A. carotis externa

A. carotis interna

A. basilarіs

3203 / 4299
У пацієнтки віком 18 років фолікулярна ангіна супроводжується болем у горлі, ознаками інтоксикації, слабкістю, гіпертермією. Який жарознижувальний препарат варто призначити у цьому разі?

Окситоцин

Аскорбінова кислота

Парацетамол

Ціанокобаламін

Лоратадин

3204 / 4299
Серед органічних речовин клітини знайдено полімер, який складається з десятків, сотень і тисяч мономерів. Молекула здатна самовідтворюватися та бути носієм інформації. За допомогою рентгеноструктурного аналізу виявлено, що молекула складається з двох спірально закручених ниток. Вкажіть цю сполуку.

Целюлоза

РНК

ГЬрмон

ДНК

Вуглевод

3205 / 4299
У нормі при цистоскопії слизова оболонка сечового міхура утворює складки. Винятком є одна ділянка трикутної форми, де слизова оболонка гладенька. В якій частині сечового міхура розташований цей трикутник?

Верхівка міхура

Шийка міхура

Дно міхура

Перешийок міхура

Тіло міхура

3206 / 4299
Чоловік віком 70 років скаржиться на біль у дрібних суглобах рук і ніг. Суглоби деформовані та болючі. Під час лабораторного дослідження виявлено підвищений рівень солей сечової кислоти в крові та сечі. Порушення обміну яких речовин характеризують ці показники?

Калію

Кальцію

Нуклеопротеїдів

Ліпопротеїдів

Хромопротеідів

3207 / 4299
Для патогенетичного лікування невралгій, серед інших і альвеолярних нервів, використовують ненарко тичні анальгетики. Визначте найбільш імовірний механізм їх дії.

Інактивація циклооксигенази та припинення синтезу просагландинів

Інактивація ліпоксигснази та пригнічення синтезу лейкотриєнів

Взаємодія з опіатними рецепторами

Антибрадикінінова активність

3208 / 4299
Під час забарвлення методом Ціля-Нільссна бакпрепаратів, виготовлених із мокротиння, було виявлено наявність яскраво-червоних тонких паличок, які розміщуються поодиноко або групами та нечутливі до дії кислот. На живильних середовищах перші ознаки росту з’являються на 10-15 добу. До якої родини відносяться ці бактерії?

Histoplasma dubrosii

Coxiella burneitii

Yersinia pseudotuberculosis

Klebsiella rhinoscleromatis

Micobacterium tuberculosis

3209 / 4299
Під час обстеження пацієнта виявлено підвищення вмісту в сироватці крові ліпопротеїнів низької щільності. Яке захворювання у цього пацієнта?

Анемія

Атеросклероз

Панкреатит

Запалення легень

Гаст рит

3210 / 4299
До основних способів підвищення резистентності емалі відноситься фторування. З якими процесами пов'язаний механізм протикаріссної дії фтору?

Синтезу хлорапатиту

Синтезом органічного матриксу зуба

Синтезом гідроксиапатиту

Синтезом фторапатиту

Демінералізацією зуба

3211 / 4299
За 5 днів після екстракції зуба пацієнту в клініці хірургічної стоамтаології розкрили гнійний абсцес. Після цього у нього піднялася висока температура, а при посіві крові був виділений патогенний стафілокок. Як називається така форма поширення мікробів в організмі.

Токсинемія

Вірусемія

Паразитемія

Септицемія

Антигенемія

3212 / 4299
У чоловіка діагностували анацидний гастрит. Для поліпшення роботи шлунку йому призначили таблетки ацидин-пепсину. Який вид лікування призначений пацієнту?

Етіотроппа терапія

Симптоматична терапія

Замісна терапія

Психотерапія

Профілактичне застосування

3213 / 4299
Вади розвитку плода можуть бути наслідками таких хвороб матері як краснуха, сифіліс, токсоплазмоз, цитомегалія, герпес, хламідіоз. До якої форми мінливості відносяться такі вади розвитку?

Мутаційна

Модифікаційна

Комбінативна

Епімутаційна

Геномного імприптингу

3214 / 4299
Під час обстеження дитини лікар-педіатр виявив набряк та гіперемію лімфоїдної тканини на ділянці мигдаликової ямки. Який із мигдаликів розташований у нормі в цьому місці?

Tonsilla tubaria

Tonsilla palatina

Tonsilla lingualis

Tonsilla pharyngealis

3215 / 4299
Психологічне дослідження встановило, що людина має добру здатність швидко пристосовуватися до нових умов, гарну пам’ять, емоційну стійкість, високу працездатність. Визначте ймовірний тип темпераменту цієї людини.

Меланхолік

Сангвінік

Флегматик

Холерик

Флегматик з елементами меланхоліка

3216 / 4299
Відторгнення трансплантату відбувається через його антигенну відмінність від комплексу антигенів реципієнта. За якого виду трансплантації ця проблема відсутня?

Гомотрансплантація

Аутотрансплантація

Гетеротрансплантація

Ксенотрансплантація

Алотрансплантація

3217 / 4299
Під час огляду плода виявлено розщеплення твердого піднебіння. Встановіть імовірний діагноз вродженої вади обличчя.

Палатосхізис

Мікрогнатія

Хейлосхізис

Гіпертелоризм

Макростомія

3218 / 4299
Пацієнтка віком ЗО років скаржиться на сильну спрагу та сухість у роті, які з'явилися після сильного нервового напруження. Під час лабораторного обстеження виявлено збільшення цукру в крові до 10 ммоль/л. Захворювання якої ендокринної залози спостерігається у жінки?

Статевої

Наднирників

Епіфіза

Підшлункової

Щитоподібної

3219 / 4299
У чоловіка 50 років, який хворіє на фіброз легень, спостерігається недостатність дихання. Який механізм є основною причиною цього явища?

Порушення дифузії газів у легенях

Порушення функції дихального центру

Рестриктивна недостатність

Зменшення загального току крові в легенях

Обструктивна недостатність

3220 / 4299
На місці механічного пошкодження шкіри завдяки процесам регенерації формується пухка волокниста сполучна тканина. Які клітини продукують білки (колаген, еластин) та компоненти міжклітинної речовини?

Фібробласти

Адипоцити

Макрофаги

Плазмоцити

Тканинні базофіли

3221 / 4299
У пацієнта за 2 доби після розвитку інфаркту міокарда відбулося різке падіння систолічного АТ до 60 мм рт.ст. з тахікардією 140/хв., задишкою та непритомністю. Який механізм має вирішальне значення у патогенезі шоку, що розвинувся?

Розвиток пароксизмальної тахікардії

Підвищення збудливості міокарда продуктами некротичного розпаду

Зниження об’єму циркулюючої крові

Зменшення хвилинного об'єму крові

Розвиток анафілактичної реакції на міокардіальні білки

3222 / 4299
Під час огляду семирічної дитини, яка проживає в районі із вмістом фтору в питній воді понад 2 мг на літр води, лікар-стоматолог виявив на ділянці різців верхньої та нижньої щелеп поодинокі пігментовані, фарфороподібні та крейдонодібні плями і смужки, що вкривають близько половини поверхні коронок зубів. Ураження локалізоване лише в емалі і не зачіпає дентин. Яке захворювання розвинулося у дитини?

Місцева гіпоплазія

Флюороз

Поверхневий карієс

Рахіт

3223 / 4299
Пацієнт отримав травму зовнішньої поверхні обличчя і скроневої ділянки. Установлено попередній діагноз: перелом виличної дуги. Відростки яких кісток черепа пошкоджені у цьому разі?

Скроневий відросток виличної кістки та виличний відросгок лобової кістки

Виличний відросток лобової кістки та виличний відросток скроневої кістки

Виличний відросток верхньої щелепи та виличний відросток лобової кістки

Скроневий відросток виличної кістки та виличний відросток скроневої кістки

Виличний відросток верхньої щелепи та виличний відросток скроневої кістки

3224 / 4299
У пацієнта запалення верхньощелепного нерва. Через який отвір черепа проходить цей нерв?

Остистий

Овальний

Круглий

Яремний

3225 / 4299
Гідроксипролін є важливою амінокислотою у складі колагену. За участю якого вітаміну відбувається утворення цієї амінокислоти шляхом гідроксилювання проліну?

B2

C

D

B1

B6

3226 / 4299
У пацієента з діагнозом: атеросклероз, під час ультразвукового обстеження виявлено стеноз ниркових артерій. Укажіть, яка біологічно активна речовина є ключовою ланкою патогенезу артеріальної гіпертензії у цьому разі

Адреналін

Вазопресин

Кортизол

Ренін

Тироксин

3227 / 4299
З віком у людини втрачається еластичність кришталика. Який основний симптом характерний для цього захворювання?

Порушення кольорового зору

Віддалення найближчої точки чіткого бачення

Порушення сутінкового зору

Астигматизм

Погіршення кровопостачання сітківки

3228 / 4299
Під час експерименту тварині провели оперативне втручання - перев'язування загальної жовчної протоки, що супроводжувалося холестазом та виникненням жовтяниці. Який вид жовтяниці та механізмом виникнення розвинувся в експериментальної тварини?

Механічна

Кон'югаційна

Печінкова

Гемолнична

3229 / 4299
Під час обстеження пацієнта у відділенні щелепно-лицевої хірургії лікар вивчає контрфорси верхньої щелепи. Скільки основних контрфорсів вона мас?

З

5

2

4

6

3230 / 4299
У організмі людини лімфоцитами та інтими клітинами синтезуються універсальні противірусні агенти у відповідь на надходження вірусів. Вкажіть групу білків, що маюгь потужну противірусну активність.

Інтерлейкін-2

Цитокіни

Інтрелейкін-4

Інтреферони

3231 / 4299
Який матеріал найчастіше використовують для дослідження каріотипу методом винчения статевого хроматину?

Нервові клітини

Епідерміс шкіри

Статеві клііини

Епітелій ротової порожнини

Еритроцити

3232 / 4299
Під час розтину тіла дівчинки, яка померла від асфіксії, виявлено, що слизова трахеї і бронхів укрити біло-сірою плівкою, яка слабко з'єднана з підлеглими тканинами і легко знімається пінцетом. Просвіт сегментарних бронхів заповнений пухкими масами сіро-білого кольору. Який трахеобронхіт за характером ексудату виявлено під час розгину?

Дифтеритичний

Крупозний

Катаральний

Гнійний

3233 / 4299
Чоловік отримав поранення та втратив 25% об’єму циркулюючої крові. Вкажіть терміновий механізм компенсації крововтрати.

Активація еритропоезу

Відновлення числа еритроцитів

Збільшення числа ретикулоцитів

Відновлення білкового складу крові

Надходження міжтканинної рідини до судини

3234 / 4299
У пацієнта діагностовано хронічну недостатність кіркової речовини надниркових залоз (Аддісонона або бронзова хвороба). Недостатність якого гормону спостерігається при цьому патологічному процесі?

Тироксину

Вазопресину

Адреналіну

Альдостерону

Інсуліну

3235 / 4299
У маленької п'ятирічної дівчинки лікар-сгоматолог видалив зуб. який мав дна корені. Який це зуб?

Малий кутній нижньої щелепи

Малий кутній верхньої щелепи

Ікло верхньої щелепи

Великий кутній верхньої щелепи

Великий кутній нижньої щелепи

3236 / 4299
У пацієнта виявлено руйнування стінки барабанної порожнини з розповсюдженням гною до задньої черепної ямки. Яка зі стінок зруйнована?

Paries jugularіs

Paries tegmentalis

Paries labyrirahicus

Paries mastoideus

Paries membranaceus

3237 / 4299
У пацієнта виявлено руйнування стінки барабанної порожнини з розповсюдженням гною до задньої черепної ямки. Яка зі стінок зруйнована?

Paries mastoideus

Paries membranaceus

Paries labyrirahicus

Paries jugularіs

Paries tegmentalis

3238 / 4299
Який показник є ознакою гострої ниркової недостатності?

Гіперглікемія

Глюкозурія

Гіперазотемія

Протеїнурія

Гематурія

3239 / 4299
Під час сходження в гори у альпініста розвинулися головний біль, запаморочення, ейфорія, серцебиття, задишка, яка чергувалася з апное. Яке порушення кислотно-основного стану розвинулося у альпініста?

Метаболічний алкалоз

Негазовий ацидоз

Газовий алкалоз

Газовий ацидоз

Негазовий алкалоз

3240 / 4299
Під час вивчення екзаменаційного гістологічного препарату декальцинованого каріозного зуба здобувач відзначив поперечні тріщини дентину та розширені дентинні канальці темно-синього кольору з потовщеннями, що зливаються в порожнини (каверни). Для якої зони каріозного дентину характерні ці морфологічні зміни?

Зона розм'якшення

Зона заміспого дентину

Зона вторинного дентину

Зона прозорого дентину

3241 / 4299
У пацієнта, який упродовж тривалого часу вживає препарати, що блокують вироблення ангіотензину II, виникли брадикардія та порушення серцевого ритму. Яка клінічна проблема є причиною цих розладів?

Гіпокальціємія

Гіперкальціємія

Гіпокаліємія

Гіперпатріємія

Гіперкаліємія

3242 / 4299
Під час абсолютного голодування єдиним джерелом води для організму є процес окислення органічних сполук. Яка з нижченаведених речовин за цих умов є основним джерелом води?

Жири

Білки

Вуглеводи

Ліпопротеїни

Глікопротеїни

3243 / 4299
У здорового подружжя народилася дитина з розщепленнями губи та піднебіння, аномаліями великих пальців кисті та мікроцефалією. Каріотип дитини: 47, 18+. Який тип мутації спричинив цю спадкову хворобу?

Моносомія за Х-хромосомою

Трисомія за аутосомою

Моносомія за аутосомою

Поліплоїдія

Нулісомія

3244 / 4299
Пацієнта віком 38 років госпіталізовано з ознаками гіпоксії, що розвилася після отруєння чадним газом. Стан середньої тяжкості, тахікардія, задишка, АТ — 160/100 мм рт.ст. Який механізм токсичної дії окису вуглецю на організм людини?

Блокада кальцієвих каналів еритроцитів

Утворення карбгемоглобіну

Утворення карбоксигемоглобіну

Порушення дисоціації оксигемоглобіну

Утворення метгемоглобіну

3245 / 4299
У пацієнтки віком 35 років діагностовано дифтерію зіву. Жінка померла через гостру серцеву недостатність. Посмертний розтин тіла показав, що порожнини серця розширені в поперечнику, м’яз серця тьмяним, в'ялий, строкатий на розрізі, з жовтуватими ділянками під ендокардом. Який вид дистрофії виявлено у кардіоміоцитах?

Вуглеводна

Балонна

Гіаліново-крапельна

Жирова

Гідрапічна

3246 / 4299
У гістопрепараті представлено кровоносну судину. Її внутрішня оболонка складається з ендотелію, субендотелію і внутрішньої еластичної мембрани. У середній оболонці переважають гладенькі міоцити. Зовнішня оболонка складається з пухкої волокнистої сполучної тканини. Визначте, для якої судини характерні ці морфологічні ознаки.

Артерії еластичного типу

Вени безм'язового типу

Артерії м'язового типу

Артерії змішаного типу

Вени м'язового типу

3247 / 4299
У чоловіка, який хворіє на туберкульоз та перебуває на лікуванні, погіршився слух. Який препарат міг викликати таку побічну дію?

Етіонамід

Ізоніазид

Рифампіцин

Стрептоміцин

Канаміцииу сульфат

3248 / 4299
Під час визначення групової належності крові за системою АВ(0) аглютинацію еритроцитів досліджуваної крові викликали стандартні сироватки першої та другої груп і не викликала сироватка третьої групи. До якої групи відноситься досліджувана кров?

А(II) 0 β

0(1) α, β

АВ(IV)

В(III) α

3249 / 4299
У гістологічному препараті розвитку зуба в одній із тканин, пронизаній канальцями, в міжклітинній речовині видно колагенові волокна, які мають радіальний і тангенціальний напрямок. Визначте, для гістогенезу якої тканини це характерно.

Дентину

Щільної оформленої сполучної тканини

Емалі

Цементу

Пульпи

3250 / 4299
До лікаря потрапив чоловік 35 років зі скаргами на біль у ділянці печінки. З'ясовано, що хворий захоплюється риболовлею і часто вживає недосмажену на вогнищі рибу. У фекаліях виявлені дуже маленькі яйця гельмінту, темного кольору, з кришечкою овальної форми. Який гельмінтоз найімовірніший?

Опісторхоз

Шистосомоз

Фасціольоз

Парагонімоз

Дикроцеліоз

3251 / 4299
У дитини 10 років виявлений гельмінтоз. На які зміни у лейкоцитарній формулі можна чекати?

Зросте кількість еозинофілів

Зросте кількість еритроцитів

Зросте кількість тромбоцитів

Зросте кількість сегментоядерних нейтрофілів

Зросте кількість базофілів

3252 / 4299
У пацієнта аспермія. Який орган не працює?

Передміхурова залоза

Придаток яєчка

Яєчко

Простата

Сім'яні міхурці

3253 / 4299
У крові хворого виявили С-реактивний білок, який за хімічною природою є глікопротеїном. Про яку патологію це свідчить?

Ревматизм

Лейкопенія

Порфірія

Анемія

Тромбоцитопенія

3254 / 4299
Лікар встановив пацієнту діагноз: менінгококовий назофарингіт. Який метод лабораторної діагностики раціональніше провести для підтвердження діагнозу?

Мікроскопічний

Бактеріологічний

Алергічний

Серологічний

Біологічний

3255 / 4299
При проведенні дуоденального зондування зонд не проходить зі шлунка в дванадцятипалу кишку. В якому відділі шлунка є перешкода (пухлина)?

Кардіальний відділ

Мала кривизна

Воротар

Дно

Тіло

3256 / 4299
Хворий із запаленням слизової язика (глосит) скаржиться на розлад смакової чутливості передніх двох третин язика. Ураженням якого нерва воно викликане?

Барабанна струна

Язиковий

Малий кам'янистий

Язикоглотковий

Барабанний

3257 / 4299
Для визначення функціонального стану печінки у хворого досліджували екскрецію тваринного індикану у сечі, який утворюється при детоксикації продуктів гниття амінокислоти в товстій кишці. Назвіть цю амінокислоту:

Гліцин

Валін

Триптофан

Серин

Цистеїн

3258 / 4299
На автопсії у померлого 72-х років з повторним трансмуральним інфарктом міокарда оболонки епікарду та перикарду набухлі, потовщені, кострубаті, неначе вкриті волосяним покровом. Назвіть вид запалення у оболонках серця:

Дифтеритичне

Гнійне

Крупозне

Катаральне

Серозне

3259 / 4299
У навколопульпарному дентині декальцінованого зуба дорослої людини визначаються ділянки з невпорядкованим розташуванням дентинних трубочок і колагенових фібрил. Назвіть даний вид дентину:

Склерозований

Мертві шляхи

Первинний

Третинний (іррегулярний)

Вторинний (регулярний)

3260 / 4299
На основі патофізіологічної характеристики, яка враховує стан обміну речовин і енергії, повне голодування можна розділити на три періоди. Що є характерним для третього (термінального) періоду голодування?

Розвиток негазового ацидозу

Посилення розпаду білків життєво важливих органів

Посилення катаболізму білків у м'язах та глюконеогенезу у печінці

Активація ліполізу в жировій тканині

Підвищення утворення кетонових тіл у печінці

3261 / 4299
У дитини 5-ти років під час огляду зіву виявлено: слизова зіву і мигдаликів гіперемована, мигдалики збільшені, вкриті щільними, білуватими плівками, які видаляються з утрудненням. На місці видалення залишається глибокий дефект тканини. М'які тканини шиї набряклі, регіонарні лімфатичні вузли шиї збільшені, болісні. Про яке захворювання можна думати?

Аденовірусна інфекція

Скарлатина

Дифтерія

Паротит

Кір

3262 / 4299
У XIV сторіччі бубонна чума протягом 20 років забрала життя 75% населення Європи та Азії. Який представник типу Членистоногі є переносником збудників цієї хвороби?

Клоп

Комар

Блоха

Воша

Москіт

3263 / 4299
До інфекційного відділення госпіталізовано хворого 27-ми років зі скаргами на багаторазові пронос та блювання, біль у м'язах ніг, слабкість, запаморочення. Після огляду лікар поставив попередній діагноз'холер'. Як необхідно досліджувати матеріал від хворого для експрес діагнозу?

Біологічний метод

Бактеріологічний метод

Пряма і непряма РІФ

РА

Серологічний метод

3264 / 4299
Екзотоксин дифтерійної палички обробили 0,3-0,4% формаліном і витримали 30 днів у термостаті при температурі 40oЯкий препарат був отриманий у результаті проведених маніпуляцій?

Лікувальна сироватка

Антитоксин

Анатоксин

Діагностикум

Діагностична сироватка

3265 / 4299
При мікроскопічному вивченні біоптата нирки в її кірковій речовині виявлені канальці близько 60 мкм в діаметрі, їх стінка утворена високим кубічним епітелієм з вираженою апікальною облямівкою і базальною складчастістю. Назвіть дані структурні утворення.

Проксимальні канальці

Капсула ниркового тільця

Петля Генле

Дистальні канальці

Збірна трубочка

3266 / 4299
У гістологічному препараті сечоводу кілька оболонок. Яким епітелієм вистелена слизова оболонка цього органу?

Одношаровий кубічний

Одношаровий плаский

Одношаровий призматичний з облямівкою

Багатошаровий перехідний

Багатошаровий плоский незроговілий

3267 / 4299
У пацієнта з атеросклерозом під час ультразвукового обстеження діагностовано двобічний стеноз ниркових артерій. Укажіть, яка біологічно активна речовина є ключовою ланкою патогенезу артеріальної гіпертензії у цьому разі.

Тироксин

Ренін

Кортизол

Адреналін

Вазопресин

3268 / 4299
У ході клінічного обстеження пацієнта виявлено збільшення щитоподібної залози (зоб), підвищення основного обміну, втрата маси тіла, порушення теплового балансу, підвищення апетиту, підвищення збудливості та дратівливості, екзофтальм і тахікардія. Яке ендокринне порушення призводить до появи даних симптомів?

Гіперфункція щитоподібної залози

Гіпофункція паращитоподібних залоз

Гіпофункція щитоподібної залози

Гіперфункція гіпофізу

Гіпофункція епіфізу

3269 / 4299
У дитини 2-х років спостерігається відставання в розумовому розвитку, непереносимість білкової їжі, важка гіперамоніємія на тлі зниженого вмісту сечовини в плазмі крові, що пов'язано з вродженим дефіцитом такого ферменту мітохондрій:

Карбомоїлфосфатсинтетаза

Моноамінооксидаза

Малатдегідрогеназа

Цитратсинтаза

Сукцинатдегідрогеназа

3270 / 4299
Аналізується каріотип жіночого організму з синдромом трисомії - (47, ХХХ). При складанні ідіограми у цьому наборі буде така кількість пар гомологічних хромосом:

22 пари

21 пара

47 пар

23 пари

24 пари

3271 / 4299
У ході утворення плащового дентину у молочному зубі відбулося порушення секреторної активності одонтобластів. Утворення яких волокон зміниться?

Радіальні колагенові волокна Корфа

Еластичні

Нервові

Тангенційні колагенові волокна Ебнера

Ретикулярні

3272 / 4299
Зірчастий ретикулум і зовнішній шар клітин емалевого органу піддаються інволюції і після завершення амелогенезу разом з апікальними частинами амелобластів формують:

Муцинову плівку

Кутикулу зуба

Зубний наліт (бляшки)

Пелікулу зуба

3273 / 4299
Пацієнту віком 50 років встановлено діагноз: мікседема. Порушення утворення яких гормонів спричиняє розвиток цієї патології?

Окситоцину і вазопресину

Кортизолу і альдостерону

АКТГ і СТГ

Тироксину і трийодтироніну

Інсуліну і глюкагону

3274 / 4299
У хворого на стоматологічному прийомі виник напад пароксизмальної тахікардії, у зв'язку з чим йому ввели лідокаїн. З яким механізмом дії лідокаїну пов'язаний його протиаритмічний ефект?

Потенціювання

Мембранно-іонний

Кумуляція

Сумація

Антагонізм

3275 / 4299
Під час операції для знерухомлення пацієнта використовують курареподібні фармакологічні препарати. Механізм їх дії полягає у блокуванні:

Н-холінорецепторів скелетних м'язів

Виділення норадреналіну у синаптичну щілину

Виділення ацетилхоліну у синаптичну щілину

Проведення збудження нервовими волокнами

М-холінорецепторів гладких м'язів

3276 / 4299
У хворої 63-х років стався напад миготливої аритмії. При обстеженні лікар виявив гіпотонію (АТ- 80/45 мм рт.ст.). Який препарат можна призначити разом з антиаритмічним для попередження колапсу?

Етимізол

Адреналін

Кофеїн-бензоат натрію

Мезатон

Теофілін

3277 / 4299
Для лікування туберкульозу призначений антибіотик, який забарвлює сечу в червоний колір. Вкажіть цей антибіотик:

Амоксицилін

Рифампіцин

Цефотаксим

Нітроксолін

Еритроміцин

3278 / 4299
Жінка із токсикозом вагітності страждає на гіперсалівацію, що призводить до втрати 3-4 літрів слини щоденно. Яке порушення водно-сольового обміну виникає при цьому?

Гіпогідратація гіпоосмолярна

Гіпогідратація ізоосмолярна

Гіпогідратація гіперосмолярна

Гіпонатріємія

Гіпокаліємія

3279 / 4299
При вивченні препарату трубчастого органу встановлено, що його середня оболонка утворена гіаліновою хрящовою тканиною, яка формує незамкнені кільця. Який це орган?

Термінальні бронхіоли

Великі бронхи

Малі бронхи

Трахея

Головні бронхи

3280 / 4299
При біохімічному аналізі еритроцитів немовляти встановлено виражену недостатність глутатіонпероксидази і низький рівень відновленого глутатіону. Яка анемія може розвинутись у цієї дитини?

Серпоподібно-клітинна

Перніціозна

Мегалобластна

Залізодефіцитна

Гемолітична

3281 / 4299
В підтриманні певної пози беруть участь повільні скелетні м'язи, які мають значні резерви кисню. Яка речовина бере участь в запасанні кисню в цих м'язах?

Кальмодулін

Креатинфосфат

Цитохром

Міоглобін

Гемоглобін

3282 / 4299
У хворого з'явилися жовтушність шкіри, склер та слизових оболонок. У плазмі крові підвищений рівень загального білірубіну, в калі - стеркобіліну, в сечі - уробіліну. Який вид жовтяниці у хворого?

Хвороба Жільбера

Холестатична

Обтураційна

Паренхіматозна

Гемолітична

3283 / 4299
У хворого на ішемічну хворобу серця визначено підвищений вміст в плазмі крові тригліцеридів і ліпопротеїнів дуже низької щільності. Який препарат слід призначити?

Лізиноприл

Фамотидин

Добутамін

Фенофібрат

Аміодарон

3284 / 4299
У лікарню було привезено хворого з опіками шкіри. Для очищення ран від мертвих тканин та слизу лікар для локального лікування призначив ферментний препарат. Назвіть цей препарат:

Трипсин

Пепсин

Аспарагіназа

Стрептокіназа

Панзинорм

3285 / 4299
При збільшенні частоти стимуляції ізольованого серця кроля відмічається неповне розслаблення шлуночків серця внаслідок:

Збільшення вмісту калію в інтерстиції

Збільшення вмісту натрію у кардіоміоцитах

Накопичення кальцію у кардіоміоцитах

Збільшення вмісту калію у кардіоміоцитах

Пригнічення K-Na насосу

3286 / 4299
При мікроскопічному дослідженні фекалій від хворого з явищами профузного проносу, багаторазового блювання та наростаючої інтоксикації, було виявлено грамнегативні палички, що нагадували кому і розташовувались групами у вигляді табунців риб. Культура збудника була виділена через середовище накопичення - 1% пептонну воду, де вона утворювала ніжну плівку. Збудник якого захворювання був виділений з фекалій хворого?

Сальмонельоз

Кишковий єрсиніоз

Псевдотуберкульоз

Холера

Шигельоз

3287 / 4299
До приймального відділення госпіталізовано чоловіка у непритомному стані. На зовнішні подразники не реагує, дихання періодичне по типу Чейна-Стокса, зіниці звужені, зіничний рефлекс відсутній. Було встановлено, що дані симптоми обумовлені застосуванням морфіну. Призначити антидотну терапію:

Унітіол

Налоксон

Кальцію хлорид

Апоморфіну гідрохлорид

Протаміну сульфат

3288 / 4299
Унаслідок поранення чоловік втратив 25% об'єму циркулюючої крові. Назвіть терміновий механізм компенсації крововтрати.

Збільшення числа ретикулоцитів

Надходження міжтканинної рідини до судини

Відновлення числа еритроцитів

Активація еритропоезу

Відновлення білкового складу крові

3289 / 4299
В біоптаті слизової носа знайдені епітеліоїдні клітини, плазмоцити, клітини Мікулича, еозинофільні тільця Русселя. Який найбільш імовірний діагноз?

Алергічний риніт

Респіраторно-синцитіальна інфекція

Риносклерома

Туберкульоз

Сифіліс

3290 / 4299
Порушення процесів розщеплення ліпідів у тонкому кишечнику зумовлено порушенням активності ліпази. Який з наведених чинників активує ліпазу?

Соляна кислота

Пепсин

Ентерокіназа

Жовчні кислоти

Солі Na+

3291 / 4299
У пацієнта при рентгенологічному обстеженні виявлено грижу міжхребцевого диска грудного відділу хребта. Який вид з'єднання між хребцями зазнав патологічних змін?

Геміартроз

Діартроз

Синдесмоз

Синостоз

Синхондроз

3292 / 4299
Досліджено ембріональний орган, в якому формуються перші формені елементи крові як тканини. Назвіть цей орган:

Жовтковий мішок

Червоний кістковий мозок

Селезінка

Тимус

Печінка

3293 / 4299
В анотації до препарату вказано, що він містить антигени збудника черевного тифу, адсорбовані на стабілізованих еритроцитах барана. З якою метою використовують цей препарат?

Для виявлення антитіл в реакції гальмування гемаглютинації

Для виявлення антитіл в реакції зв'язування комплементу

Для серологічної ідентифікації збудника черевного тифу

Для виявлення антитіл в реакції Відаля

Для виявлення антитіл в реакції непрямої гемаглютинації

3294 / 4299
У жінки 60-ти років з цирозом печінки виник геморагічний синдром. Який механізм це зумовив?

Зменшення синтезу протромбіну і фібриногену

Зменшення запасів глікогену в печінці

Поява у крові нейротоксичних речовин

Зниження онкотичного тиску в крові

Підвищення тиску в системі ворітної вени

3295 / 4299
Ціанистий калій є отрутою, смерть організму наступає миттєво. Назвіть, на які ферменти в мітохондріях діє ціанистий калій:

Цитохром B₅

Флавінові ферменти

Цитохромоксидаза [аа3]

НАД+ - залежні дегідрогенази

Цитохром Р-450

3296 / 4299
При огляді ротової порожнини у пацієнта 22-х років спостерігається нахил вперед верхніх та нижніх зубів, з перекриттям нижніх зубів верхніми. Якому прикусу характерне дане розміщення зубів?

Прогенія

Закритий прикус

Ортогнатія

Прямий прикус

Біпрогнатія

3297 / 4299
Хвора 20-ти років звернулась до лікаря зі скаргами на кровоточивість та біль у яснах, що з'явилися через 2 дні після прийому сульфадимезину. Об'єктивно: ясенні сосочки та ясенний край гіперемовані, набряклі, кровоточать при незначному подразненні. Яка патологія розвинулась в яснах?

Гострий виразковий гінгівіт

Хронічний катаральний гінгівіт

Епулід

Гострий катаральний гінгівіт

Геморагічний гінгівіт

3298 / 4299
Пацієнту зі злоякісною пухлиною для усунення нестерпного болю призначили наркотичний анальгетик. Який механізм протибольової дії цього засобу?

Активація D2-дофамінових рецепторів

Гальмування серотонінергічних рецепторів

Активація опіатних рецепторів

Гальмування гістамінергічних рецепторів

Гальмування холінергічних рецепторів

3299 / 4299
У хворого 55-ти років, що знаходиться у кардіологічному відділенні з приводу серцевої недостатності, виявлені зміни показників гемодинаміки. Які з них найбільш інформативні для підтвердження вказаної патології?

Підвищення діастолічного артеріального тиску

Підвищення венозного тиску

Зменшення хвилинного об'єму крові

Підвищення частоти серцевих скорочень

Підвищення систолічного артеріального тиску

3300 / 4299
У альпініста, що піднявся на висоту 5200 м, розвинувся газовий алкалоз. Що є причиною розвитку алкалозу?

Введення лугів

Гіпервентиляція легенів

Введення кислот

Гіповентиляція легенів

Підвищення температури навколишнього середовища

3301 / 4299
При остеохондрозі хребта перетискається судина, яка проходить через поперечні отвори шийних хребців. Яка це артерія?

Задня мозкова

Хребтова

Зовнішня сонна

Внутрішня сонна

Потилична

3302 / 4299
У чоловіка гнійна рана у ділянці соскоподібного відростка черепа, внаслідок чого розвинулося запалення оболонок головного мозку. Яким шляхом проникла інфекція в порожнину черепа?

V. auricularis

V.v. tympanicae

V. emissariae mastoidea

V. facialis

V.v. labirinthi

3303 / 4299
Хворій дитині 10 днів від народження провели операцію з приводу розщеплення верхньої губи ('заяча губа'). Розщеплення верхньої губи є наслідком незрощення:

Верхньо- і нижньощелепних відростків І-ої зябрової дуги

Піднебінних валиків верхньощелепних відростків І-ої зябрової дуги

Другої зябрової дуги

Третьої зябрової дуги

Лобового і верхньощелепних відростків І-ої зябрової дуги

3304 / 4299
У приймальне відділення поступив хворий із блідою шкірою, температура тіла знижена, пульс слабкий, малого наповнення, артеріальний тиск знижений. Дана симптоматика характерна для гострого отруєння морфіном. Який із перелічених препаратів слід застосувати в першу чергу?

Ізадрин

Атропіну сульфат

Налоксон

Кодеїну сульфат

Адреналіну гідрохлорид

3305 / 4299
До відділення реанімації поступив хворий з гострим отруєнням невідомим лікарським засобом. Для швидкого виведення отрути з організму провели форсований діурез. Який з перерахованих засобів використали для цієї процедури?

Гідрохлортіазид

Спіронолактон

Фуросемід

Дитилін

Омепразол

3306 / 4299
Хворий доставлений у лікарню із рваною раною щелепно-лицевої ділянки, що супроводжувалася сильною кровотечею, яку довго не могли зупинити. Яке порушення загального об'єму крові виникне протягом першої години після крововтрати?

Гіповолемія олігоцитемічна

Гіповолемія нормоцитемічна

Гіповолемія поліцитемічна

Гіперволемія

Порушень об'єму крові не буде

3307 / 4299
Небезпечно видавлювати гноячки на обличчі, особливо вище ротової щілини. Через анастомози з якими венами обличчя може поширитися інфекція у порожнину черепа?

Передні вушні вени

Очні вени

Потилична вена

Середня і глибока скроневі вени

Вени крилоподібного сплетення

3308 / 4299
Пацієнту видалили зуб. Коронка зуба долотоподібна, широка, край вузький. Корінь конусоподібний, стиснутий із боків. Який зуб видалили?

Верхній різець

Нижній різець

Нижній премоляр

Нижнє ікло

Верхній премоляр

3309 / 4299
При розтині тіла померлого у прямій і сигмоподібній кишках видно дефекти слизової оболонки неправильної форми з нерівними контурами, вони зливаються між собою, залишаючи невеликі острівці слизової оболонки, що збереглася. Про який різновид коліту можна думати?

Фібринозний

Катаральний

Фолікулярний

Гнійний

Виразковий

3310 / 4299
У травмованого чоловіка рана скроневої ділянки, з якої яскраво-червоною цівкою витікає кров. Яка судина була пошкоджена?

facialis

temporalis superficialis

occipitalis

auricularis posterior

maxillaris

3311 / 4299
У чоловіка 30-ти років множинні переломи кісток кінцівок в результаті автомобільної катастрофи. В стаціонар доставлений через годину після травми в важкому стані: свідомість затьмарена, шкіра бліда, вкрита липким потом, зіниці вузькі із слабкою реакцією на світло, дихання рідке, поверхневе, тони серця приглушені, пульс ниткоподібний, АТ- 60/40 мм рт.ст. Ознак внутрішньої чи зовнішньої крововтрати немає. Який стан розвинувся у хворого?

Постгеморагічний шок

Опіковий шок

Травматичний шок

Кардіогенний шок

Анафілактичний шок

3312 / 4299
У новонародженої дитини спостерігається зниження інтенсивності смоктання, часте блювання, гіпотонія. У сечі та крові значно підвищена концентрація цитруліну. Який метаболічний процес порушений?

Гліколіз

Глюконеогенез

Орнітиновий цикл

Цикл Корі

ЦТК

3313 / 4299
Хвора звернулася зі скаргами на припухлість в передній ділянці шиї, схуднення, випуклість очей, тахікардію, неврівноваженість. Яка залоза уражена?

Прищитоподібні залози

Щитоподібна залоза

Гіпофіз

Епіфіз

Тимус

3314 / 4299
Хворому на акне призначений доксицикліну гідрохлорид. Які застереження повинен дати лікар хворому при застосуванні цього препарату?

Курс лікування не повинен перевищувати 1 день

Не приймати разом з вітамінними препаратами

Приймати перед їжею

Запивати великою кількістю рідини, бажано молоком

Уникати тривалого перебування на сонці

3315 / 4299
В клініку звернувся чоловік 45-ти років із скаргами на втрату чутливості в ділянці задньої 1/3 язика. Функція якої пари черепно-мозкових нервів порушена?

V

Х

XII

ІХ

VIII

3316 / 4299
У хворого відкритий рот, він не може його закрити і членороздільно говорити. Яке ураження у хворого?

Стрес

Вивих нижньої щелепи

Перелом нижньої щелепи

Ураження рухового центру мови

Параліч жувальних м'язів

3317 / 4299
Хвора на хронічний гепатит скаржиться на підвищення чутливості до барбітуратів, які раніше вона переносила без симптомів інтоксикації. З порушенням якої функції печінки це пов'язане найбільше?

Фагоцитарна

Гемодинамічна

Метаболічна

Утворення жовчі

Гемопоетична

3318 / 4299
Пацієнта госпіталізовано до лікарні з пошкодженням шийних сегментів спинного мозку. Скільки сегментів має цей відділ спинного мозку?

8

7

12

5

2

3319 / 4299
У дівчинки спостерігаються висока температура і біль у горлі. Об'єктивно: набряк м'якого піднебіння, на мигдаликах сірі плівки, які важко відокремлюються, залишаючи глибокі кровоточиві дефекти тканини. Яке захворювання найбільш імовірне?

Дифтерія зіву

Інфекційний мононуклеоз

Ангіна Симановського-Венсана

Некротична ангіна

Лакунарна ангіна

3320 / 4299
Чоловік звернувся до хірурга з варикозним розширенням вен лівої ноги. Вузли вен розташовані на задній поверхні шкіри гомілки, на задній та передній поверхнях шкіри стегна. Які поверхневі вени нижньої кінцівки пошкоджені у хворого?

Велика та мала підшкірні вени

Задня великогомілкова вена, велика підшкірна вена

Мала підшкірна вена, глибока вена стегна

Стегнова вена, велика та мала підшкірні вени

Підколінна, поверхнева підшкірна вени

3321 / 4299
У дитини 3-х років впродовж 6-ти місяців розвинулась деформація обличчя у результаті симетричного збільшення об'єму кутів нижньої щелепи. Мікроскопічно: між кістковими балками розташована сполучна тканина з великою кількістю судин і примітивними кістковими балочками. Яке захворювання найбільш імовірне?

Остеобластокластома

Фіброма

Херувізм

Еозинофільна гранульома

Остеосаркома

3322 / 4299
Від хворого на гостру кишкову інфекцію виділено вірус, який віднесено до роду ентеровірусів. Для встановлення серотипу віруса застосовують діагностичні сироватки. Ці сироватки повинні містити антитіла проти:

Вірусних ферментів

Білків капсиду

Вірусних гемаглютинінів

Білків суперкапсидної оболонки

Неструктурних білків віруса

3323 / 4299
У хлопчика 5-ти років з'явилися біль під час ковтання, набряк шиї, висока температура. При огляді на мигдаликах виявлені плівки сіро-білого кольору, які знімаються з великим зусиллям. Про яке захворювання йде мова?

Менінгококова інфекція

Скарлатина

Кір

Дифтерія

3324 / 4299
При обстеженні хворого з ендокринною патологією встановлено, що в плазмі крові підвищений рівень тестостерону. Які клітини в організмі чоловіка відповідальні за продукцію цього гормону?

Клітини передміхурової залози

Сперматогенні клітини

Клітини сім'яних міхурців

Гландулоцити сім'яників

Сустентоцити сім'яників

3325 / 4299
Як антикоагулянти використовують різні речовини, зокрема полісахарид природного походження, а саме:

Гепарин

Дерматансульфат

Декстран

Гіалуронова кислота

Хондроїтинсульфат

3326 / 4299
Велику частину коронки, шийки і кореня зуба складає дентин, товщина якого з віком може збільшуватися, можливе також його часткове відновлення після пошкодження. Які структури забезпечують ці процеси?

Перитубулярний дентин

Цементобласти

Амелобласти

Дентинні канальці

Одонтобласти

3327 / 4299
При мікроскопічному дослідження пунктату з осередку запалення у хворого із абсцесом шкіри знайдено велику кількість різних клітин крові. Які з цих клітин першими надходять із судин до тканин при запаленні?

Лімфоцити

Базофіли

Нейтрофіли

Моноцити

Еозинофіли

3328 / 4299
У хворого після травми хребта спостерігається відсутність довільних рухів, сухожилкових рефлексів, чутливості тільки нижніх кінцівок. Який механізм порушень та у якому відділі хребта була травма?

Периферичний параліч, шийний відділ

Спінальний шок, грудний відділ

Спінальний шок, шийний відділ

Центральний параліч, куприковий відділ

3329 / 4299
У постраждалого - різана рана у ділянці краю нижньої щелепи попереду жувального м'яза. З рани виливається кров темно-червоного кольору. Яку судину пошкоджено?

V. jugularis externa

V. lingualis

V. jugularis anterior

V. retromandibularis

V. facialis

3330 / 4299
У пацієнта має місце пошкодження волокон дев'ятої пари черепних нервів (язикоглотковий). Формування якого відчуття буде порушено?

Солодке

Гірке

Кисле

Усіх смакових відчуттів

Солоне

3331 / 4299
У хворого на хронічний гепатит в аналізі крові на білкові фракції виявили зниження загальної кількості білка. Це означає, що у клітинах печінки порушена функція таких органел:

Цитоскелет

Комплекс Гольджі

Гранулярна ендоплазматична сітка

Лізосоми

Мітохондрії

3332 / 4299
У жінки 32-х років запалення ясен супроводжується їх гіпоксією. Утворення якого метаболіту вуглеводного обміну значно збільшується при цьому в тканинах пародонта?

Лактат

НАДФ-Н

Глюкозо-6-фосфат

Глікоген

Рибозо-5-фосфат

3333 / 4299
У крові пацієнта за 10 тижнів після жовтяниці виявлено HBsAg. Для якого виду вірусного гепатиту це характерно?

С

В

А

Е

D

3334 / 4299
На мікропрепараті серця розрізняємо клітини прямокутної форми з центрально розташованим ядром, розвиненими міофібрилами, зв'язані між собою вставними дисками. З цими клітинами пов'язана така функція серця:

Ендокринна

Проведення імпульсу

Регенераторна

Скорочення

Захисна

3335 / 4299
У препараті сполучної тканини, забарвленому гематоксиліном-еозином, спостерігаються ізогенні групи клітин, оточені базофільною міжклітинною речовиною. Волокнисті структури не виявлені. Яка це сполучна тканина?

Пухка волокниста

Пластинчаста кісткова

Еластична хрящова

Щільна волокниста

Гіалінова хрящова

3336 / 4299
На прийом до лікаря прийшов хворий високого росту, з відвислою нижньою губою і великим носом, та з великими кінцівками. Підвищення секреції якої залози можна запідозрити у цього хворого?

Щитоподібна

Прищитоподібні

Передня частка гіпофіза

Епіфіз

3337 / 4299
До комплексної терапії хворого на бронхопневмонію, що супроводжується виснажливим сухим кашлем, лікар включив муколітичний препарат, який деполімеризує мукопротеїди. Вкажіть цей препарат:

Строфантин

Атенолол

Ацетилцистеїн

Неодикумарин

Кодеїн

3338 / 4299
Одна з тканин зуба постійно відкладається в області верхівки кореня, що обумовлює його видовження. Цей процес компенсує стирання коронки і забезпечує постійність загальної довжини зуба. Яка тканина зуба є репаративно активною у постнатальному періоді?

Щільна сполучна

Цемент

Емаль

Кісткова

Пухка сполучна

3339 / 4299
Пацієнт скаржиться на виділення сечі під час статевого акту. Який орган уражений?

Сечовий міхур

Сім'яні міхурці

Придаток яєчка

Яєчко

Передміхурова залоза

3340 / 4299
В тканині видаленої слинної залози знайдено: дифузний склероз, прошарки сполучної тканини містять змішаноклітинний інфільтрат, часточки залози атрофовані, протоки розширені. Крім того, тканина залози містить кістозну порожнину великих розмірів, внутрішня поверхня її вкрита сплощеним двошаровим епітелієм, просвіт заповнений рідиною, оточуюча тканина склерозована. Діагностуйте захворювання:

Сіалолітіаз

Плеоморфна аденома

Гострий сіалоаденіт

Ретенційна кіста

Мономорфна аденома

3341 / 4299
У плазмі крові пацієнта підвищилась активність ізоферментів ЛДГ₁ і ЛДГ₂. Про патологію якого органа це свідчить?

Нирки

Скелетні м'язи

Мозок

Печінка

Міокард

3342 / 4299
Вітамін D резистентний рахіт визначається домінантним геном, який локалізований в Х-хромосомі. Який генотип має здоровий хлопчик у родині, де мати здорова, а у батька діагностовано дану форму рахіту?

AA

XАY

XаY

Аа

аа

3343 / 4299
Однією із функцій слини є захисна, яка реалізується, зокрема, формуванням місцевого імунітету слизової оболонки за рахунок виділення привушними залозами такого білку:

Секреторний імуноглобулін A

Еластин

Колаген

Альбумін

Фібриноген

3344 / 4299
Пасажири автобуса у спекотну погоду попросили відкрити люки. Який шлях тепловіддачі при цьому активується?

Конвекція

Випромінювання

Випромінювання та теплопроведення

Випаровування поту

Теплопроведення

3345 / 4299
Вивчається родовід сім'ї, в якій спостерігаються надмірно великі зуби. Ця ознака трапляється в усіх поколіннях тільки у чоловіків і успадковується від батька до сина. Визначте тип успадкування :

Зчеплений з Y-хромосомою

Автосомно-домінантний

Автосомно-рецесивний

Зчеплений з Х-хромосомою рецесивний

Зчеплений з Х-хромосомою домінантний

3346 / 4299
Хірург пошкодив нерв, що іннервує щелепно-під'язиковий м'яз. Назвіть пошкоджений нерв:

N. glossopharyngeus

N. accessorius

N. hypoglossus

N. facialis

N. trigeminus

3347 / 4299
На шліфі зуба в області верхівки кореня визначається тканина, що складається з клітин відростчатої форми, які оточені мінералізованою міжклітинною речовиною. Назвіть дану тканину:

Емаль

Клітинний цемент

Ретикулофіброзна кісткова тканина

Дентин плащовий

Періодонт

3348 / 4299
В еритроцитах пацієнта, хворого на гемолітичну анемію, була значно знижена активність піруваткінази. Який метаболічний процес порушений за цих умов?

Глюконеогенез

Синтез глікогену

Пентозофосфатний шлях окислення глюкози

Глікогеноліз

Гліколіз

3349 / 4299
У пацієнтки 26-ти років висипання на шкірі, свербіж після вживання цитрусових. Призначте лікарський засіб з групи блокаторів Н₁-гістамінорецепторів:

Вікасол

Димедрол

Парацетамол

Анальгін

Кислота ацетилсаліцилова

3350 / 4299
У жінки віком 35 років діагностовано дифтерію зіву. Вона померла від гострої серцевої недостатності. Розтин тіла показав: порожнини серця розширені в поперечнику, м'яз серця тьмяний, в'ялий, строкатий на розрізі, з жовтуватими ділянками під ендокардом. Який вид дистрофії виявлений у кардіоміоцитах?

Жирова

Вуглеводна

Гідропічна

Гіаліново-крапельна

Балонна

3351 / 4299
Обмеження споживання води призвело до зневоднення організму. Який механізм активується за цих умов для збереження води в організмі?

Збільшення секреції вазопресину

Зменшення секреції альдостерону

Зменшення секреції вазопресину

Збільшення секреції соматостатину

Збільшення секреції альдостерону

3352 / 4299
У пацієнтки віком 30 років після видалення зуба на нижній щелепі, спостерігалося підвищення температури, згодом виникла припухлість в області шиї. Під час розтину шкіри шиї встановлено, що підшкірно-жирова клітковина просякнута непрозорою жовто-зеленою рідиною з неприємним запахом. Який запальний процес розвинувся у жировій клітковині?

Фібринозне запалення

Серозне запалення

Геморагічне запалення

Флегмона

Абсцес

3353 / 4299
Під час роботи лікарю-стоматологу доводиться довго стояти на ногах, що може призвести до застою крові у венах нижніх кінцівок і до їх варикозного розширення. Провідним механізмом формування застою у даному випадку є зменшення:

Залишкової рушійної сила серця

Скорочення скелетних м'язів нижніх кінцівок

Присмоктувально-насосного ефекту діафрагми на органи черевної порожнини

Присмоктувального ефекту грудної клітки

Градієнта тиску крові у венозних судинах

3354 / 4299
Аналізуються діти в одній сім'ї. Один з батьків гомозиготний по домінантному гену полідактилії, а другий - здоровий (гомозиготний по рецесивному гену) в цьому випадку у дітей проявиться закон:

Незалежного спадкування

Розщеплення гібридів

Одноманітності гібридів першого покоління

Чистоти гамет

Зчеплене успадкування

3355 / 4299
В експерименті вивчалися головні показники гемодинаміки. Який з нижче перерахованих показників гемодинаміки є однаковим для великого й малого кіл кровообігу?

Лінійна швидкість кровотоку

Діастолічний артеріальний тиск

Об'ємна швидкість кровотоку

Середній артеріальний тиск

Опір кровотоку

3356 / 4299
У пацієнта, який хворіє на туберкульоз та перебуває на лікуванні, погіршився слух. Який лікарський засіб викликав таке ускладнення?

Канаміцину сульфат

Етіонамід

Рифампіцин

Стрептоміцин

Ізоніазид

3357 / 4299
В лабораторії вивчали вірулентність збудника дифтерії. При цьому проводили внутрішньоочеревинне зараження лабораторних тварин. В ході експерименту встановлена доза бактерій, яка викликає загибель 95% тварин. Яку одиницю вимірювання вірулентності визначали в лабораторії?

LD50

DCL

LD 5

DLM

ІД

3358 / 4299
Під час операції на головному мозку подразнення кори великих півкуль викликало у пацієнта тактильні та температурні відчуття. Яку звивину подразнювали?

Постцентральну

Парагіпокампальну

Верхню латеральну

Прецентральну

Поясну

3359 / 4299
У хворого з алкогольним ураженням печінки порушені процеси біотрансформації ксенобіотиків та ендогенних токсичних сполук. Зниження активності якого хромопротеїну може бути причиною цього?

Гемоглобін

Цитохромоксидаза

Цитохром Р-450

Цитохром b

Цитохром с1

3360 / 4299
Для патогенетичного лікування невралгій, серед інших і альвеолярних нервів, використовують ненаркотичні анальгетики. Виберіть найбільш імовірний механізм їх дії.

Взаємодія з опіатними рецепторами

Інактивація ліпоксигенази та пригнічення синтезу лейкотрієнів

Інактивація циклооксигенази та пригнічення синтезу простагландинів

Антибрадикінінова активність

3361 / 4299
Під час проведення бактеріоскопії мазка з уретри у пацієнта виявлено гонорею. Враховуючи, що препаратами вибору для лікування гонореї є фторхінолони, який лікарський засіб із нижченаведених треба призначити пацієнту?

Фуразолідон

Фторурацил

Ципрофлоксацин

Цефазолін

Уросульфан

3362 / 4299
У тварини під час проведення експерименту викликали отруєння уретаном. Який вид гіпоксії виник?

Дихальна

Гіпоксична

Гемічна

Тканинна

Циркуляторна

3363 / 4299
У пацієнта діагноз: гострий риногенний гайморит (запалення верхньощелепної пазухи). Яким чином інфекція могла проникнути в гайморову пазуху?

Крізь нижній носовий хід

Крізь передню черепну ямку

Крізь середній носовий хід

Крізь середню черепну ямку

Крізь верхній носовий хід

3364 / 4299
З метою визначення максимальної секреції хлористоводневої кислоти шлункового соку пацієнту 42-х років ввели розчин гістаміну. Це призвело до збільшення секреції підшлунковою залозою такого компоненту соку:

Трипсиноген

Амілаза

Ліпаза

Слиз

Бікарбонати

3365 / 4299
При гістологічному дослідженні періапікальної тканини, видаленої у хворого, що тривалий час хворів на хронічний періодонтит, було виявлено грануляційну тканину з тяжами плоского епітелію, оточену фіброзною капсулою. Який найбільш імовірний діагноз?

Абсцедуючий періодонтит

Гранулюючий періодонтит

Проста гранульома

Складна гранульома

Кістогранульома

3366 / 4299
Пацієнту з гіпохромною анемією призначено залізовмісний препарат, який вводять тільки внутрішньовенно. Який це засіб?

Ферковен

Кислота етакринова

Маніт

Фуросемід

Дихлотіазид

3367 / 4299
У хворого спостерігається остеопороз кісток, в крові - гіперкальціємія, гіпофосфатемія. Яка причина такого стану?

Посилена секреція тироксину

Пригнічення секреції паратгормону

Посилена секреція паратгормону

Пригнічення секреції кортикостероїдів

Посилена секреція кортикостероїдів

3368 / 4299
При мікроскопічному дослідженні бляшкоподібного утвору, видаленого з бокової поверхні язика у чоловіка із зубними протезами, виявлено значне потовщення епітеліального шару з явищами паракератозу, гіперкератозу та акантозу, у сполучній тканині - дрібні круглоклітинні інфільтрати. Діагностуйте патологічний стан:

Хронічний глосит

Лейкоплакія

Іхтіоз

Гунтерівський глосит

Хронічний стоматит

3369 / 4299
В емалі на межі з дентином зустрічаються незвапновані ділянки, що часто стають місцем проникнення інфекції в зуб. Як називають такі утворення?

Емалеві призми

Енамелобласти

Емалеві пучки

Волокна Томса

Дентинобласти

3370 / 4299
У дівчинки 6-ти років виражені ознаки гемолітичної анемії. При біохімічному аналізі еритроцитів встановлено дефіцит ферменту глюкозо-6-фосфатдегідрогенази. Порушення якого метаболічного процесу відіграє головну роль у розвитку цієї патології?

Окисне фосфорилювання

Пентозофосфатний шлях

Глюконеогенез

Анаеробний гліколіз

Тканинне дихання

3371 / 4299
У хворого зареєстрували ЕКГ. За яким її елементом лікар може оцінити процеси розповсюдження деполяризації передсердями?

Зубець S

Зубець Q

Зубець P

Зубець T

Зубець R

3372 / 4299
У 4-річного хлопчика з'явились горбисті, щільні, підшкірно розташовані утворення в ділянці кутів та вздовж гілок двох нижніх щелеп, які зумовили деформацію обличчя. При гістологічному дослідженні біоптату виявлено: в міжбалковому просторі розростання васкуляризованої сполучної тканини, довкола судин маси ацидофільного матеріалу, багатоядерні гігантські клітини, окремі примітивні кісткові балочки. Діагностуйте захворювання.

Периферійна гігантоклітинна гранульома

Хронічний фіброзний періостит

Фіброзна дисплазія

Херувізм

Еозинофільна гранульома

3373 / 4299
Регуляція експресії генів здійснюється за рахунок різних механізмів. Укажіть ділянку ДНК при індукції якої активується експресія гену.

Спейсер

Атенюатор

Термінатор

Сайленсер

Енхансер

3374 / 4299
Під час обстеження порожнини рота у хворої жінки визначається гіперемія слизової оболонки щік, дна порожнини рота, м'якого та твердого піднебіння. Слизова оболонка тьмяна, з великою кількістю дрібних виразок, вкрита непрозорим слизом. Встановіть імовірний діагноз.

Гнійний стоматит

Хронічний рецидивуючий афтозний стоматит

Виразковий стоматит

Гангренозний стоматит

Катаральний стоматит

3375 / 4299
У людини виявлена пухлина одного з відділів головного мозку, внаслідок чого в неї порушена здатність підтримувати нормальну температуру тіла. Яка структура головного мозку пошкоджена?

Мозочок

Чорна субстанція

Стріатум

Гіпоталамус

Таламус

3376 / 4299
Дівчині 18-ти років встановлено попередній діагноз – синдром Шерешевського-Тернера. Це можна підтвердити за допомогою такого метода:

Біохімічний

Цитогенетичний

Близнюковий

Генеалогічний

Дерматогліфіка

3377 / 4299
У пацієнта в ділянці нижньої щелепи виник болючий вузлик червоного кольору. Гістологічно: скупчення гнійного ексудату в декількох волосяних фолікулах. Яка клініко-морфологічна форма запалення має місце?

Абсцес

Фурункул

Флегмона

Натічник

Карбункул

3378 / 4299
У пацієнта виявили збільшення активності G-клітин шлунку. Які з перерахованих змін травлення при цьому виникають в шлунку?

Утворення великої кількості слизу

Зниження активності ферментів

Зменшення кислотності

Пригнічення моторики шлунка

Підвищена секреція гастрину

3379 / 4299
При дослідженні біоптату збільшеного шийного лімфатичного вузла виявлена стертість малюнка, тканина його представлена великою кількістю проліферуючих лімфоцитів з домішкою поодиноких клітин Березовського-Штернберга. Ці зміни свідчать про таке захворювання:

Лімфосаркома

Змішано-клітинний варіант лімфогранулематозу

Лімфогранулематоз з виснаженням лімфоїдної тканини

Лімфогранулематоз з переважанням лімфоїдної тканини

Нодулярно-склеротичний варіант лімфогранулематозу

3380 / 4299
Робота шахтарів у забої часто спричинює антракоз. Який вид дихальної недостатності може розвинутись при цьому?

Дисрегуляторний

Рестриктивний

Торакальний

Обструктивний

Діафрагмальний

3381 / 4299
У чоловіка 55-ти років діагностовано вікову далекозорість у зв'язку з послабленням акомодаційних властивостей кришталика та гладеньких м'язів судинної оболонки очного яблука. Тонус якого м'яза з віком послаблено?

M. sphincter pupillae

M. ciliarіs

M. obliquus superior

M. dilatator pupillae

M. obliquus inferior

3382 / 4299
Водолаз, який занурився на глибину 75 метрів, відчув симптоми порушення функцій ЦНС - збудження, послаблення уваги, ейфорія, професійні помилки. Токсичною дією на нейрони якої речовини зумовлені ці симптоми?

Вуглекислий газ

Азот

Аміак

Кисень

Лактат

3383 / 4299
Сполучна тканина побудована з паралельно розташованих колагенових волокон, розмежованих фібробластами. Цей тип сполучної тканини називається:

Щільна неоформлена

Ретикулярна

Пухка

Слизова

Щільна оформлена

3384 / 4299
У гістологічному препараті кіркової речовини нирок можна бачити ниркові тільця та канальці нефронів. Відомо, що в канальцях нефрону відбувається реабсорбція речовин. Яка тканина нефрону бере участь у цьому процесі?

Слизова

Епітеліальна

Власне сполучна

Хрящова

Ретикулярна

3385 / 4299
У хворого після автомобільної катастрофи АТ- 70/40 мм рт.ст. Хворий у непритомному стані. За добу виділяє близько 300 мл сечі. Який механізм порушення сечоутворення у даному випадку?

Зменшення клубочкової фільтрації

Зменшення канальцевої реабсорбції

Збільшення канальцевої реабсорбції

Збільшення клубочкової фільтрації

Зменшення канальцевої секреції

3386 / 4299
На мікропрепараті з контурами бобоподібного органу спостерігається кіркова та мозкова речовина. Кіркова речовина представлена окремими кулястими вузликами діаметром 0,5-1 мм, а мозкова - мозковими тяжами. З якого органа зроблено гістологічний зріз?

Селезінка

Нирки

Лімфатичний вузол

Наднирник

Тимус

3387 / 4299
При ендоскопічному дослідженні у хворого з хронічним ентероколітом (запалення кишки) спостерігається відсутність специфічних структур рельєфу тонкої кишки. Які компоненти визначають особливості рельєфу слизової оболонки тонкої кишки в нормі?

Гаустри, ворсинки, крипти

Поля, ворсинки

Циркулярні складки, ворсинки та крипти

Косо-спіральні складки

Поля, складки, ямки

3388 / 4299
У процесі набуття клітинами специфічних для них морфологічних, біохімічних та функціональних особливостей клітини обмежуються у виборі можливостей шляхів розвитку. Яку назву має таке набуте обмеження?

Детермінація

Капацитація

Рецепція

Адгезія

Комітування

3389 / 4299
Антитоксичні сироватки використовуються для профілактики та лікування:

Туберкульозу

Кашлюка

Дизентерії

Гонореї

Дифтерії

3390 / 4299
Хворий на мегалобластну анемію приймав препарат з групи водорозчинних вітамінних засобів. Визначте цей препарат:

Ціанокобаламін

Аскорбінова кислота

Токоферолу ацетат

Піридоксин

Тіаміну хлорид

3391 / 4299
Лікар-стоматолог призначив пацієнту з артритом щелепно-лицевого суглоба диклофенак-натрій. Який механізм дії цього препарату?

Активація опіатних рецепторів

Блокада опіатних рецепторів

Пригнічення циклооксигенази-2

Пригнічення каталази

Активація фосфодіестерази

3392 / 4299
Хворий на бронхіальну астму тривалий час приймає преднізолон. Який механізм дії препарату?

Блокада гістамінових рецепторів

Пригнічення активності дигідрофолатредуктази

Блокада лейкотрієнових рецепторів

Гальмування активності фосфоліпази А

Блокада натрієвих каналів

3393 / 4299
Людина згинає та розгинає передпліччя без навантаження, спираючись ліктем на стіл. Який вид м'язового скорочення має місце у m.biceps brachii?

Зубчастий тетанус

Ізотонічне

Гладкий тетанус

Ауксотонічне

Ізометричне

3394 / 4299
Під час аналізу лікар-лаборант зробив додатковий висновок, що кров належить людині жіночої статі. Особливості будови яких форменних елементів дають змогу зробити такий висновок?

Моноцитів

Лімфоцитів

Еритроцитів

Базофільних лейкоцитів

Нейтрофільних лейкоцитів

3395 / 4299
На гістологічному препараті представлені три нейрони: псевдоуніполярний, біполярний та мультиполярний. Скільки аксонів можливо визначити у кожного з перерахованих типів клітин?

Один

Два

Три

Жодного

Багато

3396 / 4299
Хворий 53-х років страждає на множинний карієс зубів. Що з перерахованого може бути причиною даного захворювання?

Стрептокок групи А

Харчування переважно вуглеводною їжею

Механічне подразнення зубними протезами

Гіпосалівація

Гіперсалівація

3397 / 4299
У нашому організмі лімфоцитами та іншими клітинами синтезуються універсальні противірусні агенти у відповідь на надходження вірусів. Назвіть ці білкові фактори.

Інтерферон

Цитокіни

Інтерлейкін-4

Фактор некрозу пухлин

Інтерлейкін-2

3398 / 4299
У вагітної пацієнтки віком 20 років на вестибулярній поверхні ясен різця виявлено кругле формування червонуватого кольору з виразкуванням на поверхні. Під час мікроскопічного дослідження спостерігається схожість із капілярною гемангіомою. Яке утворення виникло у пацієнтки на яснах?

Фіброма

Гігантоклітинний епуліс

Ангіоматозний епуліс

Папілома

Фіброзний епуліс

3399 / 4299
При захворюваннях підшлункової залози порушується утворення та секреція трипсину. Назвіть речовини, травлення яких буде порушене?

Фосфоліпіди

Вуглеводи

Ліпіди

Білки

Нуклеїнові кислоти

3400 / 4299
У чоловіка під час крововиливу ушкоджені нижні горбики середнього мозку. Який рефлекс втратиться внаслідок цього у хворого?

Орієнтувальний на звукові сигнали

Рефлекс випрямлення голови

Орієнтувальний на тактильні подразники

Статокинетичний - очний ністагм

Орієнтувальний на світлові сигнали

3401 / 4299
Хворого з підвищеним згортанням крові тривалий час лікували саліцилатами. До порушення якого метаболічного процесу це може призвести?

Мікросомального окислення

Окисного фосфорилювання

Синтезу простагландинів

Тканинного дихання

Спряження тканинного дихання і окисного фосфорилювання

3402 / 4299
Відомо, що синовіальна рідина зменшує тертя суглобових поверхонь. При ревматизмі чи артриті її в'язкість знижується внаслідок деполімеризації такої речовини:

Глікогену

Гепарину

Гіалуронової кислоти

Альбуміну

Колагену

3403 / 4299
У хлопчика група крові І (I^0 I^0), а в його сестри -- ІV (I^A I^B). Які групи крові у батьків цих дітей?

ІІ (І^А І^А) і III (І^В І^0) групи

I (І^0 І^0) і IV (I^A I^B) групи

ІІ (І^А І^0) і III (І^В І^0) групи

III (І^В І^0) i IV (I^A I^B) групи

I (І^0 І^0) і III (І^В І^0) групи

3404 / 4299
При збільшенні частоти стимуляції ізольованого серця кроля відмічається неповне розслаблення шлуночків серця внаслідок:

Накопичення кальцію у кардіоміоцитах

Збільшення вмісту натрію у кардіоміоцитах

Збільшення вмісту калію в інтерстиції

Збільшення вмісту калію у кардіоміоцитах

Пригнічення K-Na насосу

3405 / 4299
У біоптаті слизової оболонки ротової порожнини визначаються морфологічні ознаки ясен. Які особливості будови слизової оболонки ясен можна спостерігати в нормі?

Пухко зрощена з окістям, добре виражена м'язова пластинка

Відсутня м'язова пластинка, підслизова основа добре розвинута

Містить багато дрібних слинних залоз

Нерухомо зрощена з окістям, власна пластинка утворює високі сосочки, відсутня м'язова пластинка

Власна та м'язова пластинки відсутні

3406 / 4299
У чоловіка 52 років діагностовано системний амебіаз з ураженням кишечника, печінки та легень. Який препарат слід застосувати?

Ентеросептол

Тетрациклін

Хінгамін

Метронідазол

Хініофон

3407 / 4299
У хворого важка нефропатія з масивним набряковим синдромом, що ускладнила хронічну бронхоектатичну хворобу. Лабораторні дослідження виявляють виражену протеїнурію, циліндрурію, значне зниження вмісту білка в сироватці крові, гіперліпемію, гіпокаліємію та інші відхилення. Що є первинною і найбільш суттєвою ланкою в патогенезі набряків у цього хворого?

Підвищення гідростатичного тиску крові

Блокада лімфовідтоку

Зниження онкотичного тиску крові

Підвищення проникності мікросудин

Підвищення тиску позаклітинної рідини

3408 / 4299
На електронній мікрофотографії епітеліальної тканини ідентифікується структура, що лежить під епітеліоцитами і має вигляд тривимірної сітки. Як вона називається?

Базальна мембрана

Цитолема

Напівдесмосома

Десмосома

Власна пластинка

3409 / 4299
У жінки 78 років під час фізичного навантаження раптово з'явився біль у животі, який супроводжувалася зблідненням і падінням артеріального тиску до 70/40 мм рт. ст. Смерть настала при явищах гострої серцевої недостатності. Під час аутопсії в черевному відділі аорти разом із вираженим атеросклерозом виявлено мішкоподібне випинання стінки судини, діаметром 16 см, заповнене згустками крові. У стінці випинання виявлено щілиноподібний наскрізний отвір. Визначте патологію в аорті:

--

Неспецифічний аортит

Сифілітичний мезаортит

Аневризма аорти з розривом

Дисплазія стінки аорти

3410 / 4299
Відомо, що ротенон викликає інгібування дихального ланцюгу. Який комплекс дихального ланцюга мітохондрій інгібується цією речовиною?

Цитохромоксидаза

НАДН-коензим Q-редуктаза

АТФ-синтетаза

Коензим Q-цитохром с-редуктаза

Сукцинат-коензим Q-редуктаза

3411 / 4299
У пацієнта порушена парасимпатична іннервація привушної слинної залози. З боку якого вузла можлива патологія?

Ganglion geniculi

Ganglion trigeminale

Ganglion pterygopalatinum

Ganglion submandibulare

Ganglion oticum

3412 / 4299
У пацієнта з дігнозом: подагра, виявлено значне підвищення рівня сечової кислоти у крові. Кінцевим продуктом обміну яких речовин є сечова кислота?

Альбумінів

Тригліцеридів

Глобулінів

Пуринових основ

Жирових кислот

3413 / 4299
У жінки 32-х років з безсимптомним перебігом хвороби вдруге народилася мертва дитина з вираженою мікроцефалією. Яку хворобу в першу чергу може припустити лікар?

Бруцельоз

Лістеріоз

Токсоплазмоз

Гістоплазмоз

Сифіліс

3414 / 4299
До лікаря звернулася пацієнтка віком 38 років зі скаргами на те, що після перенесеного гострого вірусного респіраторного захворювання вона не відчуває дотику їжі, а також болю та температури на передній частині (2/3) язика (обпекла язик гарячим чаєм). Визначте, яка з гілок якого нерва була уражена.

Язикові гілки язикоглоткового нерва

Язиковий нерв нижньощелепного нерва трійчастого нерва

Язикові гілки під'язикового нерва

Верхній гортанний нерв блукаючого нерва

Барабанна струна лицевого нерва

3415 / 4299
Яка частина клітини є основною мішенню під час дії іонізуючої радіації?

Саркоплазматичний ретикулум

Рибосоми

ДНК

Цитоплазматична мембрана

Мітохондрії

3416 / 4299
До приймального відділення звернувся пацієнт зі скаргами на сухість у роті, світлобоязнь та порушення зору. Під час об'єктивного обстеження встановлено: шкіра гіперемована, суха, зіниці розширені, тахікардія. Поставлено діагноз: отруєння алкалоїдами красавки. Який із лікарських засобів доцільно застосувати у цьому разі?

Прозерин

Ацеклідин

Діпіроксим

Армін

Пілокарпін

3417 / 4299
Хворому на туберкульоз легень призначено рифампіцин, який пригнічує фермент РНК-полімеразу на стадії ініціації такого процесу:

Транскрипція

Реплікація

Елонгація

Термінація

Трансляція

3418 / 4299
Під час бактеріологічного дослідження випорожнень кухаря ресторану, в якого клінічні прояви захворювання відсутні, на вісмут-сульфіт агарі виросли дрібні колонії з металевим відблиском. Укажіть, які це найімовірніші мікроорганізми.

Сальмонели

Стафілококи

Стрептококи

Шигели

Ешерихії

3419 / 4299
До складу харчових раціонів обов'язково входять продукти, в яких є клітковина. Відомо, що вона не перетравлюється ферментами травного тракту й не засвоюється організмом. Яку роль відіграє ця речовина?

Гальмує секреторну функцію травного каналу

Гальмує всмоктувальну функцію травного каналу

Гальмує процеси виділення ферментів травних соків

Гальмує моторну функцію травного каналу

Стимулює моторну функцію травного каналу

3420 / 4299
Під час експерименту культуру клітин людини опромінювали протонами. Унаслідок опромінення було виявлено пошкодження ядерець. Утворення яких органел клітини порушиться у цьому разі?

Лізосом

Мікротрубочок

Рибосом

Ендоплазматичної сітки

Комплексу Гольджі

3421 / 4299
Анальгін ефективно знижує біль при пульпіті не лише під час резорбтивного, але й під час місцевого застосування. Якою дією анальгіну пояснюється знеболення в останньому разі?

Пригніченням ЦОГ-2

Відволікальною дією

Пригніченням утворення альгогенних кінінів

Місцевоанестезіючою активністю анальгіну

Пригніченням вивільнення речовини Р

3422 / 4299
Хворому за кілька днів до планового оперативного втручання з приводу виразкової хвороби шлунка призначено вікасол. Який механізм дії даного лікарського засобу?

Пригнічує фібриноліз

Знижує проникність стінки кровоносних судин

Збільшує згортання крові підвищенням синтезу протромбіну

Пригнічує агрегацію тромбоцитів

Зв'язує вільні йони кальцію, витісняючи його з реакції коагуляції

3423 / 4299
Хворому з кардіогенним шоком треба ввести неглікозидний кардіотонік. Вкажіть препарат вибору у даній ситуації:

Кофеїн

Етімізол

Амрінон

Кордіамін

Добутамін

3424 / 4299
У дитини спостерігається сухий кашель. Який протикашльовий засіб ненаркотичної дії полегшить стан хворого?

Кодеїну фосфат

Калію йодид

Корені алтеї лікарської

Глауцину гідрохлорид

Морфіну гідрохлорид

3425 / 4299
В ембріональному матеріалі виявлено порушення диференціації ентодерми. Зміни в розвитку яких органів можуть виникнути при даному процесі?

Аорти

Слинних залоз

Серця

Нирок

Шлунку

3426 / 4299
Під час операції на органах черевної порожнини у пацієнта відбулася рефлекторна зупинка серця. Укажіть місце розташування центру цього рефлексу?

У спинному мозку

У середньому мозку

У проміжному мозку

У корі великих півкуль

У довгастому мозку

3427 / 4299
У пацієнтки віком 35 років під час обстеження виявлено підвищення основного обміну. Надлишок якого з нижченаведених гормонів імовірно зумовив цей стан у пацієнтки?

Соматотропіну

Інсуліну

Кортизолу

Трийодтироніну

Глюкагону

3428 / 4299
На розтині тіла хворого 42-х років, що страждав на хронічний дифузний бронхіт і помер від легенево-серцевої недостатності, виявлені великі, підвищеної повітряності легені, що своїми краями прикривають середостіння, не спадаються, блідо-сірого кольору, ріжуться з хрустом, при натискуванні пальцем на поверхні легень залишається ямка. З просвіту бронхів виділяється слизово-гнійний ексудат. Який найбільш імовірний діагноз?

Первинна ідіопатична емфізема

Хронічна вогнищева емфізема

Вікарна компенсаторна емфізема

Хронічна дифузна обструктивна емфізема

Інтерстиційна, проміжна емфізема

3429 / 4299
Хворому на стрептококову пневмонію призначено протимікробний засіб, що порушує побудову мікробної оболонки. Який це препарат?

Еритроміцин

Доксицикліну гідрохлорид

Бензилпеніциліну натрієва сіль

Гентаміцину сульфат

Азитроміцин

3430 / 4299
У хворого взято кров для аналізу. Виявлено, що 30% еритроцитів мають неправильну форму. Як називається це явище?

Патологічний пойкілоцитоз

Мікроцитоз

Анізоцитоз

Фізіологічний пойкілоцитоз

Макроцитоз

3431 / 4299
У хворого, який перебуває на лікуванні з приводу вірусного гепатиту В, з'явилися ознаки печінкової недостатності. Які зміни в аналізі крові, що свідчать про порушення білкового обміну, найімовірніше будуть спостерігатися в цьому випадку?

Абсолютна гіперальбумінемія

Білковий склад крові не змінений

Абсолютна гіперглобулінемія

Абсолютна гіпоальбумінемія

Абсолютна гіперфібриногенемія

3432 / 4299
Вади розвитку плоду можуть спричинити такі хвороби матері як краснуха, сифіліс, токсоплазмоз, цитомегалія, герпес, хламідіоз. До якої форми мінливості відносять такі вади розвитку?

Комбінативна

Мутаційна

Геномного імпринтингу

Модифікаційна

Епімутаційна

3433 / 4299
Лужна фосфатаза каталізує реакції гідролізу фосфорноефірних зв'язків із вивільненням фосфат-іонів, які відіграють важливу роль у формуванні мінерального матриксу кісткової тканини. Які фактори забезпечують перебіг таких реакцій?

Fe^3+, рН=7,0--7,4

Zn^2+, рН=5,0--5,5

Zn^2+, рН=7,0--7,4

Fe^3+, рН=5,0--5,5

Fe^2+, рН=7,0--7,4

3434 / 4299
Вживання рослин і грибів, зібраних уздовж автомобільних трас, є небезпечним через можливість отруєння свинцем. Що є головним джерелом забруднення довкілля цим хімічним елементом?

Хімічні добрива

Кислотні дощі

Гербіциди

Вихлопні гази

Каналізаційні стоки

3435 / 4299
Визначення у пацієнта групи крові за допомогою моноклональних тест-реагентів виявило позитивну реакцію аглютинації з реагентами анти-А і анти- В та негативну з анти-D) Якої групи кров у цього пацієнта?

B (III) Rh (-)

А (ІІ) Rh (+)

АВ (ІV) Rh (+)

АВ (ІV) Rh (-)

0 (I) Rh (+)

3436 / 4299
Пацієнт звернувся до сімейного лікаря зі скаргами на слабкість, втрату ваги та збільшення шийних лімфатичних вузлів. Під час мікроскопічного дослідження біопсії лімфовузла виявлено гігантські багатоядерні клітини Рід-Штернберга, лімфоцити, плазмоцити, гістіоцити, еозинофіли, ділянки некрозу і склерозу. Для якого захворювання характерні такі клініко-лабораторні показники?

Туберкульозу

Лімфогранульоматозу (лімфома Ходжкіна)

Саркоїдозу (хвороба Беньє-Бека-Шаумана)

Лімфолейкозу

Лімфосаркоми

3437 / 4299
У західних регіонах Європи майже половина всіх природжених вад розвитку припадає на тих новонароджених, яких було зачато в період інтенсивного застосування в цих районах пестицидів. Наслідком якого впливу є такі хворобливі стани дітей?

Тератогенний

Мутагенний

Малігнізація

Канцерогенний

Механічний

3438 / 4299
У пацієнта, який отримав радіаційне ураження, з'явилися ознаки геморагічного синдрому. Який механізм має найбільше значення у патогенезі цього захворювання?

Тромбоцитопенія

Нейтропенія

Еозинопенія

Еритропенія

Лімфопенія

3439 / 4299
При розвитку стомлення жувальних м'язів може настати їх сповільнене розслаблення, при якому порушується механічна обробка їжі. Як називається цей стан?

Гальванізм

Гіподинамія

Тетанус

Гальванізація

Контрактура

3440 / 4299
Хвора 20-ти років звернулася до лікаря зі скаргами на загальне схуднення, зниження апетиту, слабкість, появу незвичайного кольору шкіри, що нагадує південну ''бронзову засмагу''. При обстеженні у клініці, окрім гіперпігментації, виявлений двобічний туберкульоз наднирників. Надлишкове накопичення якої речовини зумовило гіперпігментацію шкіри?

Білірубін

Ліпофусцин

Меланін

Гемомеланін

Адренохром

3441 / 4299
І.М. Сєченов встановив, що втомлена кінцівка відновлює працездатність швидше, якщо в період відпочинку друга кінцівка працює. Це дало можливість розробити вчення про:

Активний відпочинок

Парабіоз

Втому

Песимум

Оптимум

3442 / 4299
Під час ендоскопічного дослідження лікар виявив порушення цілісності стінки шлунку в межах слизової оболонки. Укажіть, яким типом епітелію в нормі вистелена зсередини стінка шлунку.

Багатошаровим плоским зроговілим

Псевдобагатошаровим

Багатошаровим плоским незроговілим

Одношаровим призматичним залозистим

Перехідним

3443 / 4299
Глюкозурія розвивається внаслідок порушення ниркових функцій. Укажіть, який патологічний процес може спричинити розвиток глюкозурії?

Зменшення фільтрації глюкози в клубочках

Зменшення реабсорбції глюкози в дистальних канальцях

Зменшення реабсорбції глюкози в проксимальних канальцях

Збільшення фільтрації глюкози в клубочках

Збільшення канальцевої секреції глюкози

3444 / 4299
Із нижченаведених амінокислот, що містять гідроксильну групу, одна має найбільше значення у формуванні структури колагену та органічного матриксу зуба. Яка це амінокислота?

Серин

Треонін

Гомосерин

Тирозин

Оксипролін

3445 / 4299
Відбулось пошкодження структурного гена - ділянки молекули ДНК. Однак, це не призвело до заміни амінокислот у білку, тому що через деякий час пошкодження було ліквідовано за допомогою специфічних ферментів. Це здатність ДНК до:

Транскрипції

Реплікації

Репарації

Мутації

Зворотної транскрипції

3446 / 4299
В ході мікроскопічного дослідження операційного матеріалу (частина губи з виразкою) біля країв і під дном виразкового дефекту у сполучній тканині слизової оболонки виявлені епітеліальні комплекси з атипового багатошарового епітелію з фігурами патологічних мітозів. У центрі комплексів спостерігаються нагромадження яскраво-рожевих концентричних утворень. Яка патологія розвинулася?

Пласкоклітинний рак зі зроговінням

Папілома

Базальноклітинний рак

Перехідноклітинний рак

Пласкоклітинний рак без зроговіння

3447 / 4299
Пацієнт отримав травму зовнішньої поверхні обличчя і скроневої ділянки. Установлено попередній діагноз: перелом виличної дуги. Відростки яких кісток черепа пошкоджені у цьому разі?

Виличний відросток верхньої щелепи та виличний відросток лобової кістки

Скроневий відросток виличної кістки та виличний відросток скроневої кістки

Скроневий відросток виличної кістки та виличний відросток лобової кістки

Виличний відросток верхньої щелепи та виличний відросток скроневої кістки

Виличний відросток лобової кістки та виличний відросток скроневої кістки

3448 / 4299
Диференціювання В-лімфоцитів в плазматичні клітини призводить до вироблення імуноглобулінів, які відповідають за специфічну імунну відповідь організму. У якому органі імунної системи відбувається диференціювання В-лімфоцитів?

Щитоподібна залоза

Тимус

Червоний кістковий мозок

Печінка

Мигдалики

3449 / 4299
У водолазів при швидкому підйомі з глибини на поверхню існує ймовірність розвитку декомпресійної хвороби, що може призвести до смерті внаслідок газової емболії. Який газ при цьому виділяється?

СО

N_2

О_2

CО_2

NО_2

3450 / 4299
Двомісячній дитині встановлено діагноз: синдром <<котячого крику>>. Причиною цієї хвороби є делеція короткого плеча 5-ї автосоми. Яку загальну кількість хромосом виявлено у дитини?

45

47

46

43

44

3451 / 4299
При огляді плода спостерігається розщеплення верхньої губи. Поставте діагноз вродженої вади обличчя:

Гіпертелоризм

Мікрогнатія

Палатосхізис

Макростомія

Хейлосхізис

3452 / 4299
Пацієнту з гіпертонічною хворобою призначено каптоприл. Утворення якої речовини при цьому знижується?

Серотоніну

Реніну

Брадикініну

Гістаміну

Ангіотензину ІІ

3453 / 4299
Є декілька шляхів знешкодження аміаку в організмі, але для окремих органів існують специфічні. Який шлях знешкодження аміаку характерний для клітин головного мозку?

Утворення креатину

Утворення аспарагіну

Утворення сечовини

Утворення глутаміну

Утворення NH4+

3454 / 4299
Турист, який перебував в одній з країн Далекого Сходу, госпіталізований у терапевтичне відділення з підозрою на запалення легень. Під час дослідження харкотиння та фекалій виявлено яйця легеневого сисуна. Внаслідок вживання яких продуктів збудник цієї хвороби міг потрапити до організму хворого?

Термічно необроблені прісноводні краби

Сирі овочі і фрукти

Недостатньо термічно оброблена яловичина

Недостатньо термічно оброблені яйця

Недостатньо термічно оброблена свинина

3455 / 4299
Піддослідному змастили кінчик язика місцевим анестетиком. Це призведе до відсутності сприйняття смаку:

Кислого

Солодкого

Гіркого

Солоного

Кислого та солоного

3456 / 4299
У дитини 3-х років впродовж 6-ти місяців розвинулась деформація обличчя у результаті симетричного збільшення об'єму кутів нижньої щелепи. Мікроскопічно: між кістковими балками розташована сполучна тканина з великою кількістю судин і примітивними кістковими балочками. Яке захворювання найбільш імовірне?

Еозинофільна гранульома

Херувізм

Остеосаркома

Остеобластокластома

Фіброма

3457 / 4299
Хворий захворів гостро: висока температура, збільшена, болісна селезінка. На 10-й день на шкірі живота з'явилася розеольозно-папульозна висипка. На 21-й день настала смерть від перитоніту. При патологоанатомічному дослідженні трупа у здухвинній кишці виявлені глибокі виразки у ділянці некротизованих групових лімфоїдних фолікулів (пейєрових бляшок). Одна з виразок перфорувала, наявний фібринозно-гнійний розлитий перитоніт. Про яке захворювання можна подумати у даному випадку?

Холера

Сальмонельоз

Дизентерія

Черевний тиф

Амебіаз кишечника

3458 / 4299
Після травми хворий не може розігнути руку в ліктьовому суглобі. Порушенням функції якого з основного м'язів це може бути викликано?

Musculus triceps brachii

Musculus teres major

Musculus infraspinatus

Musculus subscapularis

Musculus levator scapule

3459 / 4299
Із випорожнень хворого на гострий гастроентерит виділена чиста культура рухливих, дрібних, дещо зігнутих грам-негативних паличок, які впродовж 6 годин дають ріст на лужній 1% пептонній воді у вигляді ніжної голубуватої плівки. Яким мікроорганізмам притаманні такі властивості?

Спірохетам

Вібріонам

Клостридіям

Спірилам

Бацилам

3460 / 4299
Враховуючи, що шкірний лейшманіоз міського типу характеризується циклічним перебігом, лікар припустив, що тривалість хвороби пацієнта близько 3--6 місяців. Поява яких патолого-анатомічних змін характеризує цей цикл захворювання?

Горбка

Виразок

Туберкулоїдних шкірних проявів

Рубця

Первинної лейшманіоми

3461 / 4299
Порушення слухової функції можуть обумовлюватися зміною будови клітин Кортієвого органу, що сприймають подразнення. Які це клітини?

Пограничні

Підтримуючі

Волоскові

Фалангові

Клітини-стовпи

3462 / 4299
У хворої 32 років видалено новоутворення ясен грибоподібної форми, буруватого кольору. Мікроскопічно виявлено: сполучна тканина, багата на судини синусоїдного типу, великі багатоядерні клітини та дрібні одноядерні клітини. Спостерігаються дрібні крововиливи, накопичення гемосидерину. Назвіть це новоутворення:

Гіпертрофічний гінгівіт

Гігантоклітинний епуліс

Фіброматозний епуліс

Ангіоматозний епуліс

Фіброматоз ясен

3463 / 4299
Перед пірнанням пiд воду шукачі перлин роблять декілька глибоких вдихів та видихів. З якою метою це відбувається?

Для збільшення дифузійної здатності легень

Для збільшення кровотоку в малому колі кровообігу

Для максимально можливого виведення азоту з організму

Для забезпечення організму запасом кисню

Для максимально можливого виведення СО_2 з організму

3464 / 4299
Під час огляду порожнини рота на губній та язиковій поверхні зубів знайдено темно-жовті та коричневі плями, які вкривають більше ніж половину поверхні зубів; емаль та дентин зруйновані. Який із наведених діагнозів найімовірніший?

Клиноподібні дефекти

Флюороз

Ерозія зубів

Глибокий карієс

Карієс емалі

3465 / 4299
У пацієнта при обстеженні у невропатолога виявлена втрата чутливості тильної поверхні лівої кисті. Як називається це явище?

Анестезія

Атаксія

Астенія

Алексія

Атонія

3466 / 4299
У пацієнта під час видалення зуба стався напад судом. Який препарат для надання першої допомоги треба використати?

Настоянка валеріани

Корвалол

Фенобарбітал

Димедрол

Сибазон

3467 / 4299
У чоловіка виявлено: високий рівень зв'язаного з білком тироксину (Т4) та нормальна концентрація вільного Т3. Яким буде основний обмін у цього пацієн-та?

Пониженим

Нормальним

Надто високим

---

Підвищеним

3468 / 4299
У пацієнта з діагнозом: системний червоний вовчак, виявлено ураження нирок із нефротичним синдромом. Що з нижченаведеного викликликало цей патологічний стан?

Ішемічне пошкодження нирок

Механічне пошкодження сечових шляхів

Аутоімунне пошкодження клубочків нефронів

Гломерулосклероз

Гіперпротеінемія

3469 / 4299
Здатність зубів протистояти дії кислоти залежить від співвідношення кальцію та фосфору в емалі. Яке співвідношення цих елементів має бути в нормі?

1,1

0,5

0,8

0,9

1,67

3470 / 4299
У хворого діагностовано стоматит, викликаний вірусом простого герпесу типу 1 і 2. Який засіб, що є аналогом нуклеозидів і перетворюється під впливом тимідинкінази, забезпечить вибіркову високоефективну противірусну терапію?

Ремантадин

Ацетилцистеїн

Лаферон

Ацикловір

Оксолін

3471 / 4299
Чоловік 59 років мав ознаки паренхіматозної жовтяниці та портальної гіпертензії. Під час гістологічного дослідження пункційного біоптату печінки було знайдено таке: балково-часточкова будова порушена, частина гепатоцитів має ознаки жирової дистрофії, утворюються порто-портальні сполучнотканинні септи з формуванням псевдочасточок та наявністю перипортальних лімфо-макрофагальних інфільтратів. Діагностуйте захворювання:

Хронічний гепатоз

Токсична дистрофія

Алкогольний гепатит

Вірусний гепатит

Цироз печінки

3472 / 4299
При якому захворюванні інфекційно-алергічної або невстановленої природи розвивається двостороннє дифузне або вогнищеве негнійне запалення клубочкового апарату нирок з характерними нирковими і позанирковими симптомами?

Пієлонефрит

Нефролітіаз

Нефросклероз

Гломерулонефрит

Полікістоз нирок

3473 / 4299
У хворого видалено новоутворення шкіри, яке має вигляд вузла щільної консистенції з сосочковою поверхнею, що нагадує цвітну капусту. Мікроскопічно пухлина складається з багатьох сосочків. Паренхіма сформована з покривного епітелію зі збільшеною кількістю шарів. В епітелії збережені полярність клітин, стратифікація, цілісність власної мембрани. Строма пухлини розташована в центрі сосочків. Ваш діагноз:

Цистаденома

Аденома

Фіброаденома

Фіброма

Папілома

3474 / 4299
У п'ятирічного хлопчика спостерігаються: сильний головний біль, блювання, ригідність м'язів потилиці, блювання без нудоти, герпетичний висип на обличчі, лихоманка. Бактеріологічне дослідження якого патологічного матеріалу треба провести для підтвердження попереднього діагнозу цереброспінальний менінгіт?

Дослідження блювотиння

Пункцію спинномозкової рідини

Виділення уринокультур N. meningitidis

Виділення бактерій N. meningitidis зі слизової оболонки сечостатевої системи

Виділення копрокультур N. meningitidis

3475 / 4299
Під час аналізу крові виявлено знижений вміст гемоглобіну. Яка функція крові тоді порушується у цьому разі?

Транспорт газів

Транспорт поживних речовин

Зсідання

Транспорт гормонів

Забезпечення імунітету

3476 / 4299
Останніми роками під час проведення лабораторної діагностики гепатиту В у крові хворого визначають наявність вірусної ДНК. За допомогою якої з наведених реакцій це визначають?

Реакція гальмування гемаглютинації

Реакція непрямої гемаглютинації

Ланцюгова полімеразна реакція

Імуноферментний аналіз

Реакція зв'язування комплементу

3477 / 4299
Пацієнт віком 60 років помер від легенево-серцевої недостатності. У нижніх частках обох легень стінки бронхів різної товщини з мішкоподібними розширеннями. У частині розширених бронхів просвіти заповнені гнійними масами. Під час гістологічного дослідження у стінках бронхів виявлено деструкцію непосмугованих м'язових та еластичних волокон, хронічну запальну інфільтрацію тканини. Для якого захворювання характерні такі патологічні зміни?

Бронхогенного раку легені

Метаплазії епітелію бронха

Бронхоектатичної хвороби

Гострого бронхіту

Хронічного бронхіту

3478 / 4299
В епітелії повітроносних шляхів є клітини з куполоподібною апікальною частиною, на поверхні якої розміщуються мікроворсинки. У клітині виявляється добре розвинений синтетичний апарат, а в апікальній частині – секреторні гранули. Укажіть ці клітини.

Клітина без облямівки

Клітина Клара

Ендокринна

Келихоподібна

Камбіальна

3479 / 4299
Після споживання їжі виникає аліментарна (харчова) гіперглікемія, яка стимулює секрецію такого гормону:

Адреналін

Норадреналін

Кортизол

Глюкагон

Інсулін

3480 / 4299
Гормони білково-пептидної природи, що мають мембранний тип дії, регулюють обмін речовин у клітинах за участю внутрішньоклітинних посередників (месенджерів). Через утворення якого продукту АКТГ реалізує внутрішньоклітинні ефекти?

цГМФ

цАМФ

---

ІТФ

Кальмодулін

3481 / 4299
Яким видом терапії є призначення кислоти ацетилсаліцилової для усунення лихоманки під час гострої респіраторної вірусної інфекції?

Етіотропним

Стимулюючим

Симптоматичним

Замісним

Профілактичним

3482 / 4299
До ЛОР-відділення звернувся пацієнт із запаленням слизової оболонки порожнини носа і порушенням нюху. Яка зона слизової оболонки носа найімовірніше вражена?

Загальний носовий хід

Перетинка носа

Верхній носовий хід

Середній носовий хід

Нижній носовий хід

3483 / 4299
У пацієнта спостерігається порушення засвоєння жирів. Лікар призначив йому жовчогінний препарат для покращення травлення жирної їжі. Які компоненти жовчі беруть участь у цьому процесі?

Солі жовчних кислот

Білірубінглюкуроніди

Дигліцериди

Холестерин і його ефіри

Насичені жирні кислоти

3484 / 4299
У гістопрепараті представлено кровоносну судину. Її внутрішня оболонка складається з ендотелію, субендотелію і внутрішньої еластичної мембрани. У середній оболонці переважають гладенькі міоцити. Зовнішня оболонка складається з пухкої волокнистої сполучної тканини. Визначте, для якої судини характерні ці морфологічні ознаки.

Артерії м'язового типу

Вени безм'язового типу

Артерії еластичного типу

Артерії змішаного типу

Вени м'язового типу

3485 / 4299
У похилої людини зареєстрували зміну сили серцевих скорочень та фізичних властивостей судин, що чітко відобразилося на графічному записі пульсових хвиль сонної артерії. Який метод було застосовано?

Реографія

Міографія

Сфігмографія

Плетизмографія

Флебографія

3486 / 4299
На електронній мікрофотографії червоного кісткового мозку визначається мегакаріоцит, у периферичній частині цитоплазми якого виявляються демаркаційні канали. Яка функція цих структур?

Руйнування клітини

Збільшення площі поверхні клітин

Поділ клітини

Збільшення кількості іонних каналів

Утворення тромбоцитів

3487 / 4299
Білому щуру ввели під шкіру сулему в дозі 5 мг/кг маси тіла. За 24 години концентрація креатиніну в плазмі крові збільшилася в декілька разів. Який механізм ретенційної азотемії спостерігається в цьому разі?

Зростання клубочкової фільтрації

Збільшення утворення креатиніну в м'язах

Збільшення секреції креатиніну в канальцях нирок

Зростання реабсорбції креатиніну

Зниження клубочкової фільтрації

3488 / 4299
У юнака 20-ти років, який розпочав систематично тренуватися з легкої атлетики, в крові у стані спокою виявили: кількість еритроцитів - 5,5cdot 10^12/л, ретикулоцитів - 12% від загальної кількості еритроцитів, гемоглобіну - 160 г/л, колірний показник - 1,03. Такі показники крові свідчать про стимуляцію еритропоезу внаслідок виникнення при тренуваннях:

Гіпервентиляції

Гіперглікемії

Гіперкапнії

Гіпоксемії

Фізичного навантаження

3489 / 4299
Агрегати муцину затримують воду, що забезпечує їх в'язкість та захисну дію. Це можливо тому, що до структури муцину входять:

Олігосахариди

Глікозаміноглікани

Глюкоза

Гомополісахариди

Дисахариди

3490 / 4299
За три доби після пломбування першого правого малого кутнього зуба пацієнт відчув біль під правим оком та стійке закладення носа, що супроводжувалося підвищенням температури тіла до 38^oC і виділенням гнійного слизу із правого носового ходу. Якої ймовірної помилки припустився лікар?

Перфорації правої верхньощелепної пазухи

Перфорації правої стінки носової порожнини

Перелому міжкоміркової перегородки

Перфорації клиноподібної пазухи

Перфорації підочноямкового каналу

3491 / 4299
Хворому на гострий бронхіт з утрудненим відхаркуванням, призначили ацетилцистеїн. Яка дія засобу забезпечить лікувальний ефект?

Активація миготливого епітелію бронхів

Деполімеризація мукопротеїдів

Рефлекторна стимуляція перистальтики бронхіол

Подразнення бронхіальних залоз

Олужнювання харкотиння

3492 / 4299
У спортсмена на старті перед змаганнями відмічається підвищення артеріального тиску і частоти серцевих скорочень. Впливом яких відділів ЦНС можливо пояснити вказані зміни?

Довгастий мозок

Гіпоталамус

Середній мозок

Проміжний мозок

Кора великих півкуль

3493 / 4299
У пацієнта через 2 місяці після операції трансплантації нирки погіршився стан. На основі лабораторного обстеження констатовано, що розпочалася реакція відторгнення трансплантату. Який фактор імунної системи відіграє вирішальну роль в цій реакції?

Т-хелпери 2

В-лімфоцити

Т-кілери

Інтерлейкін-1

Природні кілери

3494 / 4299
Лікар-стоматолог виявив у пацієнта каріозну порожнину на поверхні коронки нижнього другого малого кутнього зуба, яка уворилась біля першого малого кутнього зуба. Укажіть уражену поверхню коронки.

Facies distalis

Facies occlusalis

Facies lingualis

Facies mesialis

Facies vestibularis

3495 / 4299
В якості антикоагулянтів використовують різноманітні речовини, в тому числі полісахарид природного походження, а саме:

Гепарин

Хондроітинсульфат

Гіалуронова кислота

Декстран

Дерматансульфат

3496 / 4299
У чоловіка 66-ти років діагностована злоякісна епітеліальна пухлина, що походить з бронху середнього калібру. Який епітелій є джерелом розвитку цієї пухлини?

Одношаровий призматичний

Багатошаровий незроговілий

Одношаровий багаторядний перехідний

Одношаровий багаторядний війчастий

Багатошаровий зроговілий

3497 / 4299
З рото-глотки хлопчика, який хворіє на хронічний тонзиліт виділили культуру кокових бактерій. У мазках вони розташовувалися у вигляді ланцюжків. Які це можуть бути бактерії?

Ешерихії

Стафілококи

Клостридії

Вібріони

Стрептококи

3498 / 4299
Серологічна діагностика інфекційних захворювань заснована на специфічній взаємодії антитіл з антигенами. Як називається реакція осадження з розчину антигену при впливі на нього імунної сироватки й електроліту?

--

Реакція зв'язування комплементу

Реакція гемадсорбції

Реакція нейтралізації

Реакція преципітації

3499 / 4299
Під час дослідження кровотоку в судинах головного мозку лікар встановлює датчик над артерією, що проходить у поперечних отворах шийних хребців. Яку артерію обстежує лікар?

A. cerebri anterior

A. basilaris

A. vertebralis

A. carotis externa

3500 / 4299
Робота шахтарів у забої часто спричинює антракоз. Який вид дихальної недостатності може розвинутись при цьому?

Дисрегуляторний

Торакальний

Обструктивний

Рестриктивний

3501 / 4299
Жінка 53-х років скаржиться на болісну припухлість у лівій привушній ділянці, яка з'явилась 5 днів тому. Об'єктивно: в цій ділянці шкіра слабко гіперемована, болісна. Із вивідної протоки слинної залози у малій кількості виділяється мутна, тягуча жовтувато-зеленувата рідина. При цьому мікроскопічно визначається дифузна інфільтрація залози сегментоядерними нейтрофілами. Встановіть діагноз:

Епідемічний паротит

Гострий гнійний паротит

Аденома залози

Сухий синдром Шегрена

3502 / 4299
Хірургу необхідно провести видалення частини травмованої ступні по лінії Лісфранкова суглоба. Яку зв'язку необхідно перетнути?

Таранно-човноподібна

Роздвоєна

Медіальна міжкісткова заплесно-плеснова

Таранно-п'яткова

3503 / 4299
Хвора 45-ти років протягом 8-ми років хворіла на туберкульоз, померла в стаціонарі з ознаками хронічної ниркової недостатності. На розтині: нирки збільшені, поверхня розрізу має сальний вигляд. Гістологічно виявлено масивні відкладання безструктурних гомогенних еозинофільних мас, в результаті забарвлення Конго-рот спостерігається їх виражена метахромазія. Який патологічний процес розвинувся у нирках?

Вторинний амілоїдоз

Нефросклероз

Токсичний нефрит на тлі антибіотикотерапії

Гематогенний туберкульоз нирок

3504 / 4299
Хворому із сечокам'яною хворобою ввели наркотичний анальгетик, що має спазмолітичний ефект. Який це був препарат?

Промедол

Кислота мефенамова

Ібупрофен

Анальгін

3505 / 4299
Процес дроблення зиготи завершується утворенням бластули. Який тип бластули характерний для людини?

Целобластула

Амфібластула

Морула

Бластоциста

3506 / 4299
На електронній мікрофотографії клітини видно дві різні органели, які руйнують білки. Назвіть ці органели:

Ендоплазматична сітка та мікрофіламенти

Рибосома

Комплекс Гольджі та мікротрубочки

Лізосоми та протеасоми

3507 / 4299
У крові жінки з негативним резусом під час вагітності виявлено специфічні білки, здатні зруйнувати резус-позитивні еритроцити плода. Як називається цей захисний компонент організму матері?

Сироватка

Антитіло

Антиген

Фермент

3508 / 4299
При перевірці стерильності наборів стоматологічних інструментів в одному випадку були виділені грампозитивні коки, що розташовуються у вигляді скупчень, що дають позитивну реакцію плазмокоагуляції, ферментують маніт в анаеробних умовах і мають лецитіназну активність. Який мікроорганізм виділений у даному випадку?

Staph. aureus

Str. pyogenes

Corinebacterium xerosis

St. epidermidis

3509 / 4299
Для дослідження очного дна пацієнту закапали в кон'юнктивальну щілину мідріатик, що не порушує акомодацію. Укажіть цей препарат.

Тропікамід

Платифілін

Атропін

Мезатон

3510 / 4299
Щоб взяти спинномозкову рідину для дослідження, лікар повинен зробити пункцію підпавутинного простору спинного мозку. Між якими хребцями треба ввести голку, щоб не пошкодити спинний мозок?

ІІІ і IV поперекові

IV і V грудні

I і II поперекові

XI і XII грудні

3511 / 4299
Після черепно-мозкової травми жінці віком 45 років встановлено діагноз: синдром верхньої очноямкової щілини. Це симптомокомплекс, що виникає внаслідок ураження пар черепних нервів, які проходять через однойменну щілину. Які пари нервів уражено?

N. oculomotorius, n. trochlearis, n. abducens, r. ophthalmicus n. trigemini

N. olfactorius, n. opticus

N. vestibulocochlearis, n. glossopharyngeus

N. facialis, n. trochlearis, n. abducens

3512 / 4299
Під час захворювань дихальної системи та розладів кровообігу порушується транспорт кисню, що супроводжується гіпоксією. За таких умов енергетичний обмін здійснюється шляхом анаеробного гліколізу. Який продукт у цьому разі утворюється та накопичується у крові?

Фумарова кислота

Молочна кислота

Глутамінова кислота

Лимонна кислота

3513 / 4299
Чоловік 50-ти років на прийомі у стоматолога відмовився від знеболення. Після сильного болю у нього виникла анурія внаслідок різкого збільшення продукції:

Тироксину

Тимозину

Адреналіну

Глюкагону

3514 / 4299
Пацієнт віком 42 роки захворів за тиждень після того, як обробляв шкірку лисиці. Захворювання проявилося нервовим збудженням, гідрофобією та судомами. Під час аутопсії виявлено енцефаліт із ураженням стовбурової частини головного мозку стінок ІІІ шлуночка та гіпокампа. Проявами енцефаліту було скупчення лімфоцитів та мікрогліальних клітин довкола загиблих нейронів і судин. У нервових клітинах гіпокампа виявлено еозинофільні включення - тільця Бабеша-Негрі. Яке захворювання діагностовано у померлого?

Чума

Сказ

Сибірка

Бруцельоз

3515 / 4299
У хворого травма голови в ділянці з'єднання тім'яних кісток між собою. Яка пазуха твердої оболонки головного мозку може бути ушкоджена?

Нижня стрілова

Верхня стрілова

Сигмоподібна

Поперечна

3516 / 4299
З метою диференційної діагностики менінгітів проводять дослідження спинномозкової рідини. В якому місці люмбальна пункція є безпечною?

L IV - L V

L I - L II

Th XII - L I

L III - L IV

3517 / 4299
У дівчинки 16-ти років, яка тривалий час намагалась знизити масу свого тіла голодуванням, виник набряк. Яка головна причина цього явища?

Зменшення вироблення вазопресину в гіпоталамусі

Гіпоглікемія, зумовлена порушенням синтезу глікогену

Венозний застій і підвищення венозного тиску

Гіпопротеїнемія, зумовлена порушенням синтезу білків

3518 / 4299
У пацієнта діагностували виразкову хворобу шлунка та призначили антибактеріальне лікування. На який збудник воно спрямоване?

H. pylori

St. aureus

Cl. perfringens

E) colli

3519 / 4299
Хворий обстежується за клінічними показаннями. Проведені дослідження по діагностиці вірусних гепатитів. У сироватці крові виявлені тільки антитіла до HbsAg. Такий результат свідчить про:

Хронічний гепатит С

Гепатит А

Перенесений гепатит В

Гострий гепатит С

3520 / 4299
Хворому на акне призначений доксицикліну гідрохлорид. Які застереження повинен дати лікар хворому при застосуванні цього препарату?

Запивати великою кількістю рідини, бажано молоком

Курс лікування не повинен перевищувати 1 день

Уникати тривалого перебування на сонці

Приймати перед їжею

Не приймати разом з вітамінними препаратами

3521 / 4299
На гістологічному препараті виявлено клітини, які утворюють ізогенні групи, у міжклітинній речовині виявлено глікопротеїни, протеоглікани та колагенові волокна. Яку тканину виявлено?

Кісткова

Хрящова

Біла жирова

Бура жирова

3522 / 4299
Під час операції для знерухомлення пацієнта використовують курареподібні фармакологічні препарати. Механізм їх дії полягає у блокуванні:

Виділення ацетилхоліну у синаптичну щілину

Н-холінорецепторів скелетних м'язів

Виділення норадреналіну у синаптичну щілину

Проведення збудження нервовими волокнами

3523 / 4299
Під час забарвлення методом Ціля-Нільсена бакпрепаратів, виготовлених із мокротиння, було виявлено наявність яскраво-червоних тонких паличок, які розміщуються поодиноко або групами та нечутливі до дії кислот. На живильних середовищах перші ознаки росту з'являються на 10--15 добу. До якої родини відносяться ці бактерії?

Micobacterium tuberculosis

Klebsiella rhinoscleromatis

Coxiella burnettii

Histoplasma dubrosii

3524 / 4299
При обстеженні дитини виявлено незарощення овального отвору. Де розташований цей отвір?

В ділянці мітрального клапана

Між лівим передсердям і лівим шлуночком

Між лівим і правим шлуночками

Між лівим і правим передсердями

Між правим передсердям і правим шлуночком

3525 / 4299
Хворий скаржиться на тривалу кровотечу навіть при незначному травматичному пошкоджені. Лабораторний аналіз показав порушення складу крові, а саме зменшення кількості таких формених елементів:

Моноцити

Еритроцити

Тромбоцити

Нейтрофіли

Лімфоцити

3526 / 4299
Хворому з явищами тривоги, страху, невпевненості, психічної напруги, призначено діазепам. Який можливий механізм його транквілізуючої дії?

Взаємодія з адренорецепторами

Взаємодія з дофаміновими рецепторами

Взаємодія з бензодіазепіновими рецепторами

Взаємодія з серотоніновими рецепторами

Взаємодія з холінорецепторами

3527 / 4299
При мікроскопічному дослідженні препаратів із зіву, зафарбованих за Нейссером, було виявлено палички з потовщеннями на полюсах, що розташовувались під кутом одна до одної. До якого виду можуть належати дані мікроорганізми?

Leptospira interrhogans

Mycobacterium tuberculosis

Streptococcus pyogenes

Corynebacterium diphtheriae

3528 / 4299
При обстеженні пацієнта в відділенні щелепно-лицевої хірургії лікар вивчає контрфорси нижньої щелепи. Скільки основних контрфорсів вона має?

1

4

2

3

3529 / 4299
Підвищення кров'яного тиску в аорті спричинило навантаження на серцевий м'яз. М'язова стінка якої ділянки серця реагує на подразнення?

Лівий шлуночок

Правий шлуночок

Ліве передсердя

Праве передсердя

3530 / 4299
У чоловіка, його сина та дочки відсутні малі корінні зуби. Така аномалія спостерігалася також у дідуся за батьківською лінією. Який імовірний тип успадкування цієї аномалії?

Аутосомно-домінантний

Зчеплений з У-хромосомою

Рецесивний, зчеплений з Х-хромосомою

Домінантний, зчеплений з Х-хромосомою

3531 / 4299
У пацієнта перед складною хірургічною операцією відзначається блідість шкірних покривів, збільшення частоти серцевих скорочень та дихання, підвищення артеріального тиску, сухість слизової рота внаслідок активації:

Парасимпатичної нервової системи

Метасимпатичної нервової системи

Соматичної нервової системи

Симпатичної нервової системи

3532 / 4299
Після черепно-мозкової травми у хворого спостерігається порушення функції сечовидільної системи - поліурія. Порушення виділення якого гормону може спричинити поліурію у хворого?

АКТГ

Мінералокортикоїди

Інсулін

Вазопресин

3533 / 4299
У хворого на цукровий діабет вміст молочної кислоти в крові 2,5 ммоль/л. Яке ускладнення розвинулось?

Гіперосмолярна кома

Гіпоглікемічна кома

Гіперглікемічна кома

Лактацидемічна кома

Гіперкетонемічна кома

3534 / 4299
На мікропрепараті з контурами бобоподібного органу спостерігається кіркова та мозкова речовина. Кіркова речовина представлена окремими кулястими вузликами діаметром 0,5-1 мм, а мозкова - мозковими тяжами. З якого органа зроблено гістологічний зріз?

Нирки

Наднирник

Тимус

Лімфатичний вузол

3535 / 4299
У пацієнта з відкритою раною обличчя із підритими краями спостерігається некроз тканин із поступовим частковим гангренозним процесом, що майже доходить до кісткової тканини. У рані під час детального обстеженя виявлено живі личинки. Пацієнту виставлено діагноз: тканинний міаз. Личинки яких двокрилих викликали цю хворобу?

Phlebotomus pappataci

Stomoxys calcitrans

Glossina palpalis

Musca domestica

Wohlfahrtia magnifica

3536 / 4299
До лікаря-травматолога звернувся хворий, у якого після травми стало утрудненим активне розгинання руки в ліктьовому суглобі. Який м'яз імовірно ушкоджений?

M. deltoideus

M. triceps brachii

M. pectoralis minor

M. latissimus dorsi

3537 / 4299
Що потрібно додати до донорської крові, законсервованої цитратом натрію, щоб викликати її зсідання?

Іони кальцію

Фібриноген

Протромбін

Вітамін K

3538 / 4299
У закладі дошкільної освіти зареєс-тровано спалах кишкової інфекції. Бактеріологічне дослідження випорожнень хворихпатогенних бактерій не виявило. Під час електронної мікроскопії встановлено: утворення округлої форми з чітким обідком і товстою втулкою, які нагадують колесо. Укажіть найімовірніший збудник цієї інфекції.

Adenovirus

E) coli

Rotavirus

P. vulgaris

Coxsacki-virus

3539 / 4299
У чоловіка 62 років, який помер при наростаючих явищах серцевої недостатності, на розтині знайдено збільшене в об'ємі серце. Серце дряблої консистенції, камери розтягнуті, міокард на розрізі тьмяний, глинисто-жовтий. З боку ендокарда видно жовто-білу посмугованість, яка особливо виражена в сосочкових м'язах. Який патологічний процес найбільш імовірний?

Жирова дистрофія міокарда

Ожиріння серця

Міомаляція

Дилатаційна кардіоміопатія

3540 / 4299
До лікаря звернувся чоловік віком 27 років. Під час огляду було виявлено: збільшення кистей рук, стоп та нижньої щелепи, деформація суглобів (kiphosis), гормональні порушення (імпотенція та атрофія яєчок). Про порушення функції якої залози свідчать ці симптоми?

Надниркових залоз

Паращитоподібних залоз

Шишкоподібного тіла

Передньої частки гіпофізу

3541 / 4299
У хворого 12-ти років запальний процес у внутрішньому вусі викликав розлите ураження оболонок мозку. Лікар припустив, що процес розповсюдився через сполучення між субарахноїдальним простором головного мозку та перилімфатичним простором внутрішнього вуха. Через яке анатомічне утворення розповсюдилось запалення?

Hiatus canalis n. petrosi minoris

Hiatus canalis n. petrosi majoris

Аqueductus vestibuli

Fossa subarcuata

3542 / 4299
До хірургічного відділення надійшов хворий, у якого діагностовано абсцес печінки. З анамнезу відомо, що у нього неодноразово траплялися шлунково-кишкові розлади. Під час лабораторного дослідження фекалій виявлено клітини округлої форми, що містять по 4 ядра. Який представник найпростіших паразитує у хворого?

Balantidium coli

Entamoeba gingivalis

Entamoeba hystolytica

Trichomonas hominis

3543 / 4299
Під час обстеження периферичної крові пацієнта виявлено: еритроцити --- 3.0cdot10^12/л, Hb --- 80 г/л, лейкоцити --- 21cdot10^9/л. У лейкоцитарній формулі спостерігається: базофіли --- 0%, еозинофіли --- 0%, мієлобласти --- 54%, промієлоцити --- 1%, мієлоцити --- 0%, метамієлоцити --- 0%, паличкоядерні --- 1%, сегментоядерні --- 28%, лімфоцити --- 13%, моноцити --- 3%. Якій патології відповідають вищенаведені результати обстеження?

Гострому мієлобластному лейкозу

Хронічному мієлолейкозу

Лейкемоїдній реакції

Недиференційованому лейкозу

3544 / 4299
Лікар-стоматолог для лікування гінгівіту призначив пацієнту антибактеріальний протипротозойний засіб, що може викликати відразу до алкоголю. Укажіть препарат, який призначив лікар.

Лінкоміцину гідрохлорид

Метронідазол

Тетрациклін

Левоміцетин

Цефтріаксон

3545 / 4299
Для лікування абсцедивного пародонтозу призначено ферментний препарат із групи протеолітичних ферментів. Назвіть цей лікарський засіб.

Стрептоліаза

Лідаза

Контрикал

Трипсин кристалічний

3546 / 4299
Мама дитини 4-х років звернулася по медичну допомогу зі скаргами на підвищення температури тіла, тенезми, розвиток діареї, на переймоподібні болі в животі у дитини. Дитина відвідує дитячий колектив. Під час дослідження у калових масах: слиз, домішка крові. Як називаються зміни у ШКТ при дизентерії?

Гастроентерит

Гастрит

Коліт

Ентероколіт

3547 / 4299
Зчитування спадкової інформації з гена розпочинається з синтезу про-іРНК на фрагменті матричного ланцюга ДНК. Де відбувається цей процес у клітинах еукаріот?

Цитоплазма

Ядро

Комплекс Гольджі

Центросоми

3548 / 4299
Які рецептори реагують на газовий склад крові, що надходить до головного мозку?

Каротидних синусів

Аортальні

Бульбарні

Дихального центру

3549 / 4299
Пацієнту для ввідного наркозу застосовано тіопентал-натрій. Після цього у нього виникли гіперсалівація та ларингоспазм. Уведенням якого препарату можна було попередити ці ефекти?

Анальгіну

Атропіну сульфат

Пірацетаму

Дитиліну

Адреналіну гідрохлорид

3550 / 4299
Хворому для лікування ішемічної хвороби серця (ІХС) був призначений beta-адреноблокатор, через деякий час у нього з'явився кашель, бронхоспазм. У якого з перелічених засобів є така побічна дія?

Анаприлін

Фенігідин

Метопролол

Талінол

Атенолол

3551 / 4299
Які рецептори реагують на газовий склад крові, що надходить до головного мозку?

Каротидних синусів

Ноцицептори

Механорецептори

Аортальні

Бульбарні

3552 / 4299
У чоловіка підвищений основний обмін речовин, висока температура тіла та тахікардія у стані спокою. Підвищена функція якої залози ймовірно є причиною цього стану пацієнта?

Надниркової

Статевої

Підшлункової

Щитовидної

3553 / 4299
У пацієнта віком 50 років раптово з'явилися головний біль, запаморочення голови та нудота. АТ - 220/110 мм рт. ст. Після введення 0,1% розчину гігронію в/в крапельно самопочуття пацієнта покращилося. Який механізм дії цього препарату?

Блокада N-холінорцепторів гангліїв

Блокада каналів Са^++

Блокада АПФ

Активація alpha_2-адренорецепторів

Блокада beta_1-адренорецепторів

3554 / 4299
Припинення кровотечі після пологів пов'язано з дією окситоцину на стінки матки. Яка оболонка органа реагує на дію цього препарату?

Підслизова

Ендометрій

Параметрій

Міометрій

Периметрій

3555 / 4299
Після тривалого застосування ізоніазиду лікар виявив у пацієнта явища поліневриту, парестезії, розлади пам'яті, судоми. З яким механізмом можуть бути пов'язані вказані побічні ефекти препарату?

Порушення синтезу клітинної стінки

Антагонізм з ПАБК

Пригнічення синтезу білка

Пригнічення утворення піридоксальфосфату

Пригнічення синтезу РНК

3556 / 4299
Яка зі складових частин пародонта виконує сенсорну функцію, що забезпечує регуляцію сили жувального тиску на зуби?

Періодонт

Окістя

Цемент

Кістки альвеолярного відростку

Ясна

3557 / 4299
Хворому 65-ти років для зняття гіпертонічного кризу було призначено препарат, що пригнічує реабсорбцію натрію хлориду на рівні товстого сегменту висхідної частини петлі Генле, що має виражений діуретичний ефект. Визначте препарат:

Фуросемід

Тріамтерен

Маніт

Гідрохлортіазид

Спіронолактон

3558 / 4299
Стоматологічному хворому для пригнічення страху перед болем призначили психоседативний засіб. Який препарат найбільш ефективний в цьому випадку?

Діазепам

Літію карбонат

Аміназин

Натрію бромід

Настоянка валеріани

3559 / 4299
Малотоксичний антибіотик, який рідко викликає побічні ефекти та відноситься до антибіотиків резерву з групи макролідів. Механізм його дії полягає у пригніченні синтезу білка рибосомами бактерії у результаті пригнічення фермента пептидтранслокази. Укажіь цей препарат.

Ампіцилін

Азитроміцин

Тетрациклін

Сизоміцин

Левоміцетин

3560 / 4299
Жінка з алергічним дерматитом протягом тижня приймала протиалергічний препарат, після якого в неї розвивалася виражена сонливість. Назвіть препарат:

Аміназин

Кромолін натрію

Димедрол

Адреналіну гідрохлорид

Лоратадин

3561 / 4299
У мазку периферійної крові серед лейкоцитів переважають округлі клітини з посегментованими ядрами. Дрібна зернистість в їх цитоплазмі фарбується як кислими, так і основними барвниками. Як називаються ці клітини?

Юні нейтрофіли

Еозинофіли

Сегментоядерні нейтрофіли

Моноцити

Базофіли

3562 / 4299
У піддослідної тварини під час проведення експерименту зруйновано відділ головного мозку, що викликало перехід тварини з гомойотермного в пойкілотермний стан. Який відділ мозку зруйновано?

Гіпоталамус

Довгастий мозок

Епіфіз

Гіпофіз

Середній мозок

3563 / 4299
У хворого, який працює на підземних гірничих виробках, розвинувся фіброз легенів. Проведене спірометричне обстеження показало:

Збільшення життєвої ємності легень

Зменшення опору повітроносних шляхів

Нормальний опір повітроносних шляхів

Зменшення життєвої ємності легень

Збільшення опору повітроносних шляхів

3564 / 4299
В інфекційне відділення лікарні госпіталізовано хворого зі скаргами на нудоту, рідкі випорожнення зі слизом і прожилками крові, підвищення температури, слабкість. Лікар запідозрив дизентерію. Який метод лабораторної діагностики найдоцільніше призначити для підтвердження діагнозу?

Мікроскопічний

Мікологічний

Серологічний

Бактеріологічний

3565 / 4299
Під час емоційного стресу в жировій тканині активується гормончутливий фермент - тригліцеридліпаза. Який вторинний месенджер бере участь в активації ферменту?

цГМФ

Са^2+

Інозитолтрифосфат

Диацилгліцерол

цАМФ

3566 / 4299
У п'ятирічної дитини діагностовано міопатію Дюшена. Батьки здорові. Дядько дитини і син тітки за материнською лінією також хворі на міопатію. Визначте тип успадкування захворювання.

Аутосомно-рецесивний

Домінантний, зчеплений з Х-хромосомою

Аутосомно-домінантний

Рецесивний, зчеплений з Y-хромосомою

Рецесивний, зчеплений з Х-хромосомою

3567 / 4299
У хворого, який помер в результаті легенево-серцевої недостатності, серце збільшене в розмірах, стінка правого шлуночку на розтині потовщена, порожнина розширена. Визначте характер патологічного процесу:

Атрофiя

Склероз

Гiперплазія

Метаплазія

Гiпертрофiя

3568 / 4299
Лікар обговорював із колегами застосування нового протиепілептичного засобу - натрію вальпроату. Укажіть можливий механізм дії препарату.

Стимуляція активності Са^2+залежної АТФ-ази

Пригнічення активності ферменту ГАМК-трансферази

Пригнічення моноамінооксидази

Пригнічення активності Са^2+залежної АТФ-ази

Стимуляція активності ферменту ГАМК-трансферази

3569 / 4299
У пацієнта з діагнозом: ниркова недостатність, виявлено різке зниження вмісту натрію в сироватці крові. Спостерігаються бліді пухкі набряки обличчя, які з'являються зранку. Визначте речовину, що входить до міжклітинного матриксу сполучної тканини та зв'язує іони натрію, які надходять із кровоносного русла.

Проколаген

Фібронектин

Гіалуронова кислота

Колаген

Еластин

3570 / 4299
Серед органічних речовин клітини знайдено полімер, який складається з десятків, сотень і тисяч мономерів. Молекула здатна самовідтворюватися та бути носієм інформації. За допомогою рентгеноструктурного аналізу виявлено, що молекула складається з двох спірально закручених ниток. Вкажіть цю сполуку:

Гормон

РНК

ДНК

Целюлоза

Вуглевод

3571 / 4299
В еритроцитах пацієнта, хворого на гемолітичну анемію, була значно знижена активність піруваткінази. Який метаболічний процес порушений за цих умов?

Пентозофосфатний шлях окислення глюкози

Глюконеогенез

Синтез глікогену

Глікогеноліз

Гліколіз

3572 / 4299
За даними ВООЗ, щорічно на Землі на малярію хворіють приблизно 250 млн. чоловік. Ця хвороба зустрічається переважно у тропічних і субтропічних областях. Межі її розповсюдження співпадають з ареалами комарів роду:

Кулекс

Мансоніа

Аедес

Кулізета

Анофелес

3573 / 4299
Під час обстеження пацієнта у відділенні щелепно-лицевої хірургії лікар вивчає контрфорси верхньої щелепи. Скільки основних контрфорсів вона має?

6

2

4

3

5

3574 / 4299
Пацієнта шпиталізовано з високою концентрацією азоту у крові. Яку дію викликає азот в організмі людини при його високій концентрації?

Хімічну

Алергічну

Фізичну

Наркотичну

Токсичну

3575 / 4299
Відомо, що слиз завжди вкриває епітелій власне носової порожнини. Вкажіть, які клітини епітелію слизової оболонки носової порожнини синтезують слиз:

Базальні

Мікроворсинчасті

Келихоподібні

Війчасті

3576 / 4299
Жінці віком 52 років зробили ін'єкцію місцевого анестетика перед видаленням зуба. Який механізм дії лежить в основі знеболюючого ефекту цього препарату?

Порушення ізольованого проведення збудження в нервових волокнах

Порушення фізіологічної цілісності в нервових волокнах

Порушення функціонування мікротрубочок в нервових волокнах

Порушення анатомічної цілісності в нервових волокнах

Порушення аксонного транспорту в нервових волокнах

3577 / 4299
У шестирічної дитини на слизовій оболонці щік на рівні премолярів лікар-стоматолог виявив сіро-білі плями діаметром до 1-го мм. Із анамнезу відомо, що дитина не була вакцинована у віці 12 місяців. Лікар-стоматолог зробив припущення, що ураження слизової викликав складний РНК-умісний вірус із гемаглютинуючими властивостями, який не має нейрамінідазної активності і не культивується в курячих ембріонах. Який вірус спричинив розвиток захворювання?

Кору

Коксакі А

Вітряної віспи

Паротиту

Простого герпесу

3578 / 4299
Чоловік віком 24 роки помер унаслідок гострої легенево-серцевої недостатності. Протягом двох останніх днів скаржився на кашель із незначним <<іржа-вим>> мокротинням, біль у грудній клітці справа, який різко посилювався під час дихання, підвищення температури до 39^oС. Під час аутопсії виявлено: нижні частки легень червоні, щільні і безповітряні, плевра вкрита нитками і плівками фібрину. Встановлено діагноз: двостороння нижньочасткова плевропневмонія. Яка стадія розвитку пневмонії найімовірніше була у чоловіка?

Приплив

---

Червоне спечінкування

Сіре спечінкування

Видужання

3579 / 4299
В гістологічному препараті ендометрію видно окремі епітеліальні клітини, в яких хромосоми формують ''пластинку'', що розташована в екваторіальній площині. В якому періоді клітинного циклу перебувають такі клітини?

Профаза

Анафаза

Інтерфаза

Телофаза

Метафаза

3580 / 4299
Пацієнту з великими опіками зробили пересадку донорської шкіри. Але на 8-му добу трансплантат набряк, його колір змінився і на 11-ту добу розпочалася реакція відторгнення. Які клітини беруть участь у цьому процесі?

Т-лімфоцити

В-лімфоцити

Еритроцити

Базофіли

Еозинофіли

3581 / 4299
Під час проведення стоматологічної лікувальної процедури у пацієнта виник напад стенокардії. Препарати якої групи необхідно призначити для надання швидкої допомоги?

Антигіпертензивні засоби

Антиангінальні засоби

Протиаритмічні засоби

Стимулятори дихання

Кардіотоніки

3582 / 4299
Хворій для терапії остеомієліту було призначено антибіотик, який здатен добре проникати в кісткову тканину. Назвіть цей препарат:

Поліміксин В

Цефазолін

Амфотерицин В

Стрептоміцину сульфат

Лінкоміцину гідрохлорид

3583 / 4299
Який патологічний стан може розвинутися під час вливання великих обсягів ізотонічних розчинів?

Гіперволемія олігоцитемічна

Проста гіперволемія

Гіповолемія олігоцитемічна

Гіперволемія поліцитемічна

Гіповолемія поліцитемічна

3584 / 4299
При огляді педіатром дитини 10-ти років виявлено множинні петехії на шкірі, кровоточивість ясен та знижений рівень вітаміну С в сечі. Який процес порушився в цьому разі?

Розпад протеогліканів

Активація гіалуронідази

Розпад колагену

Синтез колагену

Синтез протеогліканів

3585 / 4299
У здорового подружжя народилася дитина з розщепленнями губи та піднебіння, аномаліями великих пальців кисті та мікроцефалією. Каріотип дитини: 47, 18+. Який тип мутації спричинив цю спадкову хворобу?

Моносомія за аутосомою

Поліплоїдія

Трисомія за аутосомою

Нулісомія

Моносомія за Х-хромосомою

3586 / 4299
У хворого виявлено порушення прохідності дихальних шляхів на рівні дрібних і середніх бронхів. Які порушення кислотно-лужної рівноваги можна виявити в крові у цьому разі?

Респіраторний алкалоз

Метаболічний алкалоз

Метаболічний ацидоз

Респіраторний ацидоз

--

3587 / 4299
У стоматологічній практиці широко використовується місцевий анестетик --- лідокаїн. Який механізм його дії?

Блокує лігандкеровані натрієві канали

Активує потенціалзалежні калієві канали

Блокує потенціалзалежні калієві канали

Активує потенціалзалежні кальцієві канали

Блокує потенціалзалежні натрієві канали

3588 / 4299
В експерименті тварині проводили електростимуляцію нейронів головного мозку, внаслідок чого у неї виникла гіпофагія (відмова від прийому їжі). В яку ділянку головного мозку було введено електроди?

Нейрогіпофіз

Червоне ядро

Аденогіпофіз

Гіпоталамус

Таламус

3589 / 4299
У деяких одноклітинних організмів, наприклад амеб, живлення здійснюється шляхом фагоцитозу. В яких клітинах організму людини таке явище не є способом живлення, а здійснює захист організму від чужорідних компонентів (мікроорганізмів, пилу тощо)?

Лейкоцити

Еритроцити

Епітеліоцити

Тромбоцити

Міоцити

3590 / 4299
Пацієнтка віком 36 років звернулася до лікаря-стоматолога із приводу набряку під правим оком. Після обстеження встановлено діагноз: флегмона підочноямкової ділянки. Від яких зубів інфекція найчастіше поширюється в цю ділянку обличчя?

Другого премоляра та першого моляра

Верхніх першого та другого молярів

Верхнього ікла й першого премоляра

Верхніх бічних та присередніх різців

Верхнього присереднього різця

3591 / 4299
У чоловіка спостерігаються збільшені і болючі суглоби, у сироватці крові виявлено підвищений вміст уратів. Обмін яких речовин порушений у пацієнта?

Холестерину

Глюкози

---

Фенілаланіну

Пуринів

3592 / 4299
В ході мікроскопії з імерсійною системою вивчено препарат-мазок з культури стрептобацил, зафарбований за методом Ожешко. Яку структурну особливість бактерій досліджено?

Спори

Включення

Капсула

Будова клітинної стінки

Джгутики

3593 / 4299
При дослідженні 16 зуба на жувальній поверхні виявлено порожнину з вузьким отвором, заповнену розм'якшеним дентином. Мікроскопічно: в розширених дентинних канальцях бактерії, руйнування окремих канальців, злиття порожнин у каверни, явища декальцинації емалі та дентину без утворення замісного дентину. Поставте діагноз:

Хронічний глибокий карієс зуба

Гострий поверхневий карієс зуба

Хронічний поверхневий карієс зуба

Гострий глибокий карієс зуба

Карієс зуба у стадії плями

3594 / 4299
Чоловік звернувся до хірурга з варикозним розширенням вен лівої ноги. Вузли вен розташовані на задній поверхні шкіри гомілки, на задній та передній поверхнях шкіри стегна. Які поверхневі вени нижньої кінцівки пошкоджені у хворого?

Задня великогомілкова вена, велика підшкірна вена

Стегнова вена, велика та мала підшкірні вени

Підколінна, поверхнева підшкірна вени

Мала підшкірна вена, глибока вена стегна

Велика та мала підшкірні вени

3595 / 4299
Хворий скаржиться на кашель, виділення харкотиння, загальну слабкість. Під час лабораторного дослідження харкотиння виявлені личинки. Це характерно для:

Цистицеркозу

Опісторхозу

Ентеробіозу

Теніозу

Аскаридозу

3596 / 4299
У пацієнта на електрокардіограмі відзначено зниження амплітуди зубця R. Що означає цей зубець на ЕКГ?

Поширення збудження від передсердь до шлуночків

Поширення збудження по основі шлуночків

Поширення збудження по передсердях

Реполяризацію шлуночків

Електричну діастолу серця

3597 / 4299
Студент перкуторно визначає межу серця, яка проектується на передню грудну клітку на рівні хрящів третього ребра. Яку межу серця визначив студент?

Верхня

Ліва

Права

Верхівка

Нижня

3598 / 4299
У дитини 5-ти років відбулася інвазія гельмінтів, що призвело до сенсибілізації організму. Які показники лейкоцитарної формули підтвердять цей процес?

Збільшення кількості нейтрофілів

Зменшення кількості базофілів

Збільшення кількості базофілів

Збільшення кількості еозинофілів

Зменшення кількості еозинофілів

3599 / 4299
При мікроскопічному дослідженні видаленого зуба виявлено руйнування емалі та дентино-емалевого сполучення; дентинні канальці розширені, заповнені мікробними масами. Відростки одонтобластів у стані дистрофії та некрозу; трапляються осередки демінералізації дентину. Який із перерахованих діагнозів найімовірніший?

Середній карієс

Глибокий карієс

Поверхневий карієс

Карієс цементу

Флюороз

3600 / 4299
Пацієнт відвідав лікаря-стоматолога для екстракції зуба. Після видалення зуба кровотеча з лунки не припинялася протягом 15 хв. З анамнезу відомо, що пацієнт хворіє на хронічний активний гепатит. Яка можлива причина подовження часу кровотечі?

Тромбоцитопенія

Підвищення активності антикоагулянтної системи

Зниження вмісту фібриногену в крові

Зниження вмісту альбумінів у крові

Гіпокальціємія

3601 / 4299
Машиною швидкої допомоги в лікарню доставлено непритомного чоловіка після отруєння чадним газом. Гіпоксія у нього зумовлена нагромадженням у крові:

Сульфгемоглобіну

Оксигемоглобіну

Метгемоглобіну

Карбоксигемоглобіну

Карбгемоглобіну

3602 / 4299
Під час огляду зубів на боковій поверхні першого верхнього моляра зліва виявлено каріозну порожнину у вигляді конуса, орієнтованого основою до поверхні, а верхівкою - у товщу зуба. На дні каріозної порожнини видно розм'якшений дентин. Поставте діагноз:

Карієс емалі

Карієс дентину

Карієс цементу

Ерозія зуба

3603 / 4299
Укажіть хрящевидні утворення, що доповнюють конгруентність суглобових поверхонь.

Губи

Складки

Зв'язки

Меніски

Диски

3604 / 4299
У юнака 17-ти років через 2 тижні після перенесеної ангіни розвинувся гострий дифузний гломерулонефрит. Що є найбільш частою причиною цього ускладнення?

Віруси

Стафілококи

Стрептококи

Гриби роду Candida

Мікобактерії туберкульозу

3605 / 4299
Типові кардіоміоцити мають специфічну фазу потенціалу дії:

Систолічну реполяризацію

Повільну діастолічну реполяризацію

Швидку діастолічну деполяризацію

Швидку систолічну деполяризацію

Повільну реполяризацію (плато)

3606 / 4299
У пацієнта спостерігається порушення функції білявушної слинної залози. Який нерв посилює її секрецію?

N.petrosus profundus

N.petrosus minor

N.auricularis minor

N.petrosus major

N.auricularis major

3607 / 4299
Хвора 30-ти років скаржиться на сильну спрагу, сухість у роті, які з'явилися після сильного нервового потрясіння. При лабораторному обстеженні виявлено збільшення цукру в крові до 10 ммоль/л. Захворювання якої ендокринної залози у хворої?

Статеві

Наднирники

Епіфіз

Щитоподібна

Підшлункова

3608 / 4299
При клінічному обстеженні в жінки встановлено: підвищення потовиділення, тахікардія, схуднення, тремор. Яка ендокринна патологія може це спричинити?

Гіпоальдостеронізм

Гіпогонадизм

Гіпотиреоз

Гіпертиреоз

Гіпергонадизм

3609 / 4299
При операції на шиї з правого боку порушилася екскурсія правого склепіння діафрагми. Це сталося в результаті пошкодження такого нерва:

Лівий поперечний нерв шиї

Лівий діафрагмальний

Надключичний нерв

Правий діафрагмальний

Правий поперечний нерв шиї

3610 / 4299
Пацієнту встановлно діагноз ка-рієс ускладнений хронічним пульпітом. Лікар-стоматолог під час огляду виявив розростання тканини м'якої консистенції яскраво-рожевого кольору у вигляді поліпу над збереженою частиною коронки зуба. Для якого захворювання характерні такі симптоми?

Серозний пульпіт

Гангренозний пульпіт

Дифузний гнійний пульпіт

Гранулюючий пульпіт

Фіброзний пульпіт

3611 / 4299
Під час розтину тіла шестимісячної дитини, яка померла внаслідок тривалого інфекційного захворювання, виявлено зменшення маси та розмірів тимусу. Під час мікроскопічного дослідження в тимусі виявлено атрофію часточок, значне зменшення кількості лімфоцитів, інверсію шарів, збільшення кількості тимічних тілець. Який патологічний процес найімовірніше розвинувся в тимусі?

Акцидентальна трансформація

Тимома

Вікова інволюція

Агенезія тимусу

3612 / 4299
Визначте, як називається новий стан біосфери, у якому визначальним фактором розвитку є розумова діяльність людини:

Ноосфера

Літосфера

Атмосфера

Гідросфера

Тропосфера

3613 / 4299
Відомо, що сальні залози мають голокриновий тип секреції. За рахунок яких структурних компонентів поновлюються клітини цієї залози?

Клітини-себоцити

Одношаровий кубічний епітелій вивідної протоки

Міоепітеліальні клітини

Багатошаровий плоский епітелій вивідної протоки

Клітини гермінативного шару

3614 / 4299
Один із синдромів проявляється ураженням зубів, волосся і кісток. У кожному поколінні є хворі, з однаковою частотою хворіють представники чоловічої і жіночої статі. Визначте тип успадкування синдрому:

Зчеплений з Х-хромосомою рецесивний

Аутосомно-домінантний

Зчеплений з Х-хромосомою домінантний

Зчеплений з Y-хромосомою

Аутосомно-рецесивний

3615 / 4299
З віком у людини втрачається еластичність кришталика. Який основний симптом характерний для цього захворювання?

Порушення кольорового зору

Віддалення найближчої точки чіткого бачення

Астигматизм

Порушення сутінкового зору

Погіршення кровопостачання сітківки

3616 / 4299
На гістологічному препараті представлені три нейрони: псевдоуніполярний, біполярний та мультиполярний. Скільки аксонів можливо визначити у кожного з перерахованих типів клітин?

Один

Три

Жодного

Багато

Два

3617 / 4299
При тривалому голодуванні зростає секреція глюкокортикоїдів корою наднирникових залоз. Глюкокортикоїди посилюють у печінці синтез ферментів глюконеогенезу. Термінальним ферментом цього процесу є:

Глюкозо-1-фосфатаза

Фруктозо-2,6-дифосфатаза

Фруктозо-1,6-дифосфатаза

Глюкозо-6-фосфатаза

Фруктозо-6-фосфатаза

3618 / 4299
Який лікарський засіб, що здатен проникати в кістову тканину і кістковий мозок, доцільно призначити для лікування інфекцій кісткової системи (остеомієліт, остеїт)?

Лінкоміцин

Бензилпеніцилін

Гентаміцин

Біцилін-3

Синтоміцин

3619 / 4299
Після травми ока виникло нагноєння м’яких тканин орбіти. Через яке анатомічне утворення запальний процес може поширитися у крило-піднебінну ямку?

Верхню очноямкову щілину

Нижню очноямкову щілину

Вилично-очноямковий отвір

Клино-піднебінний отвір

Круглий отвір

3620 / 4299
У складі мітохондріального дихального ланцюга містяться складні білки цитохроми. Який тип реакцій вони каталізують?

Дезамінування

Трансамінування

Декарбоксилювання

Гідратації

Окисно-відновні

3621 / 4299
Пацієнту встановили діагноз: тромбоз нижньої брижової артерії. Який відділ кишечника буде уражено в цьому разі?

Сигмовидна кишка

Червоподібний відросток

Клубова кишка

Дванадцятипала кишка

Порожня кишка

3622 / 4299
Який фермент має демінералізуючу дію - посилює розщеплення мінеральних компонентів тканин зуба?

Кисла фосфатаза

Лужна фосфатаза

Глюкозо-6-фосфатаза

Фосфотрансфераза

Глікогенфосфорилаза

3623 / 4299
Фермент здійснює перенесення функціональної групи від одного субстрату до іншого. Вкажіть клас цього ферменту:

Гідролаза

Лігаза

Оксидоредуктаза

Трансфераза

Ізомераза

3624 / 4299
Пацієнт, який приймав засіб для зниження АТ, звернувся до лікаря зі скаргою на підвищену сухість у роті. Який гіпотензивний засіб має таку побічну дію?

Анаприлін

Дибазол

Клофелін

Верапаміл

Адельфан

3625 / 4299
Під час дослідження клітин епітелію слизової оболонки щоки чоловіка, у більшості ядер виявлено по одному тільцю Барра. Для якого синдрому характерні такі показники?

Патау

Клайнфельтера

Едвардса

Шерешевського-Тернера

Дауна

3626 / 4299
При дослідженні на лабораторних щурах властивостей одного з вітамінів при його дефіциті спостерігалося одночасне порушення репродуктивної функції та дистрофія скелетних м'язів. Про який вітамін йдеться?

B_2

D

A

E

K

3627 / 4299
Для працівників атомної електростанції проводять періодичні медогляди, під час яких в першу чергу обстежують найбільш чутливу до іонізуючого випромінювання систему організму. Назвіть цю систему:

Нервова

Епітеліальні тканини

М'язова

Кровотворна

Кісткова

3628 / 4299
Нормальну оклюзію зубних дуг можна посилити, відтягнувши нижню щелепу назад. Який м'яз виконує цю функцію?

Жувальний

Грудинно-ключично-соскоподібний

Скроневий

Латеральний крилоподібний

Медіальний крилоподібний

3629 / 4299
Пацієнта госпіталізували до лікарні з явищами гострої крововтрати. Що є провідним в патогенезі постгеморагічного шоку?

Анемія

Гіповолемія

Гіпоксія

Зменшення серцевого викиду

Зменшення тонусу судин

3630 / 4299
У дитини через 30 хвилин після вживання соку манго раптово виникла обмежена припухлість у ділянці м'якого піднебіння, яка заважала ковтанню, а пізніше і диханню. Слизова оболонка в області припухлості гіперемована, безболісна. Результати аналізу крові показали наявність еозинофілії. Температура тіла нормальна. З анамнезу відомо, що у старшої сестри виникали напади бронхіальної астми. Який вид набряку найіморвініше розвинувся у дитини?

Запальний

Алергічний

Аліментарний

Печінковий

Серцевий

3631 / 4299
Для профілактики та лікування якого захворювання використовуються антитоксичні сироватки?

Дизентерії

Туберкульозу

Дифтерії

Кашлюка

Гонореї

3632 / 4299
Яка частота серцевих скорочень у пацієнта з діагнозом: пароксизмальна тахікардія?

110--120/хв.

<140/хв.

90--100/хв.

100--110/хв.

120--130/хв.

3633 / 4299
У хворого на гострий апендицит виявлено зростання числа лейкоцитів у крові. Який вид лейкоцитозу може мати місце при цьому діагнозі?

Нейтрофільний

Лімфоцитоз

Базофільний

Еозинофільний

Моноцитоз

3634 / 4299
В ендокринологічному відділенні перебуває хлопчик 9 років, у якого вже декілька разів були переломи кінцівок, пов'язані із крихкістю кісток. Функція якої ендокринної залози порушена?

Надниркові залози

Щитоподібна залоза

Епіфіз

Тимус

Паращитоподібна залоза

3635 / 4299
Під час операції на головному мозку подразнення у пацієнта кори великих півкуль викликало тактильні і температурні відчуття. Яку звивину подразнювали?

Постцентральна

Поясна

Прецентральна

Верхня латеральна

Парагіпокампальна

3636 / 4299
Розщеплення цАМФ та цГМФ до звичайних, нециклічних нуклеозидмонофосфатів каталізується таким ферментом:

Протеїнкіназа

Глюкозо-6-фосфатаза

Аденілатциклаза

Фосфодіестераза

Глікогенфосфорилаза

3637 / 4299
При операції на шлунку хірург пересік ліву шлункову артерію, перев'язав її, але кров продовжує витікати з протилежного кінця артерії. Яка артерія анастомозує з нею?

Права шлунково-чепцева

Селезінкова

Верхня підшлунково-дванадцятипала

Ліва шлунково-чепцева

Права шлункова

3638 / 4299
Під час сходження в гори у альпініста розвинулися головний біль, запаморочення, ейфорія, серцебиття, задишка, яка чергувалася з апное. Яке порушення кислотно-основного стану розвинулося у альпініста?

Негазовий ацидоз

Газовий ацидоз

Метаболічний алкалоз

Негазовий алкалоз

Газовий алкалоз

3639 / 4299
Чоловіку віком 63 роки встановили діагноз тромбофлебіт глибоких вен гомілки. Який шар цих судин пошкоджено у цьому разі?

Шар гладеньких м'язових клітин

Підендотеліальний

Ендотеліальний

Шар еластичних волокон

Шар сполучної тканини

3640 / 4299
У хворого діагностовано пошкодження міжхребцевого диска в поперековому відділі хребта. До якого виду сполучення кісток належить це з'єднання?

Діартроз

Геміартроз

Синхондроз

Синдесмоз

Синостоз

3641 / 4299
При виконанні оперативного втручання на діафрагмі ротової порожнини хірургу необхідно виділити ділянку, що має назву ''піднижньощелепний трикутник''. Який з м'язів його обмежує?

M. stylohyoideus

M. geniohyoideus

M. hyoglossus

M. digastricus

3642 / 4299
На мікропрепараті серця розрізняємо клітини прямокутної форми з центрально розташованим ядром, розвиненими міофібрилами, зв'язані між собою вставними дисками. З цими клітинами пов'язана така функція серця:

Проведення імпульсу

Регенераторна

Скорочення

Ендокринна

Захисна

3643 / 4299
На ЕКГ хворого виявлено скорочення тривалості інтервалу R-R. Як при цьому зміниться діяльність серця?

Збільшиться сила серцевих скорочень

Сповільниться частота і знизиться сила серцевих скорочень

Зменшиться частота серцевих скорочень

Зменшиться сила серцевих скорочень

Збільшиться частота серцевих скорочень

3644 / 4299
Вегетативні ефекти подразнення черепно-мозкового нерва виявлено в гортані, стравоході, серці, шлунку, кишечнику та травних залозах. Ядро якого нерва подразнено?

Під'язичного

Лицьового

Язикоглоточного

Блукаючого

Додаткового

3645 / 4299
У дитини 1,5 років при огляді виявили осередкові потовщення в ділянці ребер, зап'ястків, викривлення ніжок. Стоматолог вказав на пізнє прорізування зубів, порушення порядку прорізування зубів, нерівномірну мінералізацію емалі та дентину, конфігурацію верхньої щелепи в горизонтальному напрямку у вигляді ''високого піднебіння''. Яке захворювання розвинулося у дитини?

Флюороз

Остеопороз

Сіалолітіаз

Подагра

Рахіт

3646 / 4299
Під час секвенування ДНК і біохімічного аналізу поліпептиду було встановлено, що лінійна послідовність триплетів нуклеотидів відповідає послідовності амінокислот у поліпептидному ланцюзі. Визначте, яка властивість генетичного коду була встановлена:

Триплетність

Колінеарність

Виродженість

Неперекривність

Універсальність

3647 / 4299
Внаслідок гнійного отиту гноєм зруйновано верхню стінку барабанної порожнини. У яку ямку черепа розповсюдився гній з барабанної порожнини?

Задня черепна

Середня черепна

Крило-піднебінна

Очна

Передня черепна

3648 / 4299
У пацієнта діагностовано хронічну недостатність кіркової речовини надниркових залоз (Аддісонова або бронзова хвороба). Недостатність якого гормону спостерігається при цьому патологічному процесі?

Інсуліну

Альдостерону

Тироксину

Вазопресину

Адреналіну

3649 / 4299
Жінка скаржиться на головний біль, біль у м'язах під час ковтання, жування та обертання очей, слабкість, підвищену температуру, набряк повік і обличчя. За 1,5-2 місяці до появи цих симптомів жінка споживала свинину, яка не пройшла ветеринарно-санітарну експертизу. Який гельмінт викликає вказані симптоми?

Анкілостома

Гострик

Аскарида людська

Трихінела

3650 / 4299
Прискорений синтез якого полісахариду передує відкладанню мінеральних солей в органічний матрикс зуба?

Гепарину

Хондроїтинсульфату

Глікогену

Дерматансульфату

3651 / 4299
Наприкінці зими студент, який останнім часом відзначав нервове перенапруження, після переохолодження захворів на гостре респіраторне захворювання. Що є причиною захворювання?

Гіповітаміноз

Патогенний збудник

Нераціональне харчування

Нервове перенапруження

3652 / 4299
Стоматолог при обстеженні ротової порожнини пацієнта звернув увагу, що його язик вкритий білуватим нальотом. Які гістологічні структури беруть участь в його утворенні?

Епітелій грибоподібних сосочків

Епітелій жолобкуватих сосочків

Епітелій ниткоподібних сосочків

Епітелій листоподібних сосочків

3653 / 4299
Трирічну дитину пригостили полуницею. Незабаром на шкірі дитини з'явилися висипання. Які зміни виявлено в лейкоцитарній формулі крові цієї дитини?

Лімфоцитопенія

Еозинофілія

Нейтрофільний лейкоцитоз

Моноцитоз

3654 / 4299
У пацієнта внаслідок травми утворився дефект шкіри. З метою ліквідації дефекта на це місце хірурги перемістили ділянку шкіри з іншої частини тіла цього ж хворого. Який вид трансплантації було здійснено?

Ксенотрансплантацію

Аллотрансплантацію

Аутотрансплантацію

Гомотрансплантацію

3655 / 4299
У дитини 6-ти місяців на шкірі спини виявлено плаский вузол діаметром 3 см, червоного кольору, який блідне при натисканні. Який діагноз найімовірніший?

Пігментний невус

Лімфангіома

Гемангіома

Меланома

3656 / 4299
У недоношених новонароджених порушений синтез сурфактанту. Які функції він виконує в легенях?

Погіршує дифузію O_2 через аерогематичний бар'єр

Зменшує поверхневий натяг стінок альвеол

Полегшує екскурсію діафрагми

Збільшує поверхневий натяг стінок альвеол

3657 / 4299
У чоловіка гнійна рана у ділянці соскоподібного відростка черепа, внаслідок чого розвинулося запалення оболонок головного мозку. Яким шляхом проникла інфекція в порожнину черепа?

V.v. labirinthi

V. facialis

V. emissariae mastoidea

V.v. tympanicae

3658 / 4299
У дитини від народження знижена функція щитоподібної залози. Який патологічний стан може розвинутися в дитини внаслідок цього?

Нанізм

Кретинізм

Гіперпігментація шкіри

Гігантизм

3659 / 4299
З метою усунення нападу бронхіальної астми, який розвинувся при екстракції зуба, пацієнту призначили сальбутамол. До якої фармакологічної групи відноситься цей препарат?

beta_2-адреноміметики

М-холіноміметики

Аналептики

Адаптогени

3660 / 4299
Під час експедиції до Середньої Азії студенти виявили членистоногу тварину завдовжки 7 см. Тіло поділяється на головогруди з 4 парами ходильних ніг і сегментоване черевце, в останньому сегменті якого є дві отруйні залози, що відкриваються отворами на кінці гачкоподібного жала. Було встановлено, що ця тварина є нічним хижаком і отруйною для людини. Учасники експедиції віднесли її до ряду:

Aphaniptera

Aranei

Acarina

Scorpiones

3661 / 4299
В закритому колективі виникла необхідність перевірити стан імунітету проти дифтерії, щоб обґрунтувати необхідність вакцинації. Які дослідження слід провести з такою метою?

Перевірити членів колективу на носійство палички дифтерії

Встановити титр антитоксинів в РНГА

Встановити рівень антитіл проти дифтерійної палички

Перевірити медичну документацію щодо вакцинації

3662 / 4299
При бактеріологічному обстеженні групи хворих із карієсом зубів були виділені різні мікроорганізми. Якому з наведених нижче мікроорганізмів належить провідна роль у виникненні карієсу у цих хворих?

St. salivarius

Candida albicans

Streptococcus mutans

Staphylococcus aureus

3663 / 4299
У пацієнтки віком 30 років після видалення зуба на нижній щелепі, спостерігалося підвищення температури, згодом виникла припухлість в області шиї. Під час розтину шкіри шиї встановлено, що підшкірно-жирова клітковина просякнута непрозорою жовто-зеленою рідиною з неприємним запахом. Який запальний процес розвинувся у жировій клітковині?

Геморагічне запалення

Фібринозне запалення

Флегмона

Абсцес

3664 / 4299
Для видалення зуба розчин новокаїну вводять у ділянку проходження чутливого нерва, що веде до знеболення внаслідок порушення:

Утворення медіаторів болю

Збудливості больових рецепторів

рН тканин

Аксонального транспорту

Проведення больових імпульсів

3665 / 4299
У чоловіка, який переніс інсульт та перебуває у нерухомому положенні спостерігається недостатність кровообігу, шкіра та м'які тканини над крижами почорніли і набрякли. Після відторгнення епідермісу в чорних тканинах з'явилися виразки. Який процес розвинувся у пацієнта?

Флегмона

Інфаркт

Пролежні

Абсцес

3666 / 4299
При досліджені трупа жінки 59-ти років, яка померла від гострої серцевої недостатності, в стінці лівого шлуночка виявлено ділянку жовтого кольору 2,5х2 см, неправильної форми, тістоподібної консистенції. На ендокарді, відповідно до неї, утворився тромб, а на епікарді - фібринозні нашарування. Яка локалізація інфаркту стосовно стінки серця мала місце?

Трансмуральний

Субендокардіальний

Інтрамуральний

Субепікардіальний

3667 / 4299
На поздовжньому шліфі зуба видно тканину, що утворює основу зуба і складається з колагенових волокон, мінералізованого матриксу і трубочок, в яких проходять відростки дентинобластів. З чого розвивається представлена тканина?

Проміжні клітини емалевого органа

Периферійна частина зубного сосочка

Зовнішні клітини емалевого органа

Зубний мішечок

3668 / 4299
Під час розтину тіла жінки, яка померла від уремії, виявлено: нирки розмірами 7х3,5х2 см з рівномірно дрібнозернистою поверхнею і стоншеним до 0,3 см блідим корковим шаром. Під час гістологічного дослідження встановлено: склероз і гіаліноз більшості клубочків, гіаліноз артеріол, склероз стінок артерій з концентричним звуженням просвіту, атрофічні зміни в канальцях. Для якої патології характерні такі макро- і мікроскопічні зміни в нирках?

Для вторинно-зморщеної нирки

Для первинно-зморщеної нирки

Для амілоїдно-зморщеної нирки

Для пієлонефритично зморщеної нирки

3669 / 4299
Досліджується гістологічний препарат кровотворного органа, який складається з часточок, що відрізняються за формою. Кожна часточка має кіркову і мозкову речовини. Для якого органа характерні ці ознаки?

Мигдаликів

Лімфатичного вузла

Червоподібного відростка

Тимусу

Селезінки

3670 / 4299
Студент, який погано навчався протягом семестру, під час складання підсумкового контролю перебуває у стані емоційного напруження. Провідним механізмом розвитку емоційного напруження в даному випадку є дефіцит:

Часу і енергії

Часу

Інформації

Енергії та інформації

3671 / 4299
У пацієнта віком 43 років спостерігається: стоматит, глосит, гладенький язик малинового кольору. В загальному аналізі крові виявлено: Hb - 100 г/л, еритроцити - 2,3cdot10^12/л, кольоровий показник - 1,30. Для якого патологічного стану характерні такі клініко-лабораторні показники?

Дефіцит вітаміну В_12

Гемоліз еритроцитів

Порушення синтезу порфіринів

Гіпоплазія червоного кісткового мозку

3672 / 4299
Оксид азоту (ІІ) - нестійка молекула, яка бере участь у вазодилятації, імунних процесах, нейротрансмісії. Назвіть фермент, за допомогою якого відбувається утворення NO з аргініну:

Аргіназа

Аргініносукцинатсинтетаза

Орнітинкарбамоїлтрансфераза

NO-синтаза

3673 / 4299
Під час мікроскопічного дослідження зародкового матеріалу в препараті визначається жовтковий мішок. Укажіть основну функцію цього органу у людини:

Продукція навколоплідних вод

Трофічна

Захисна

Кровотворна

3674 / 4299
На гістологічному препараті представлений зріз судини, що характеризується правильною круглою формою; зіяє, стінка складається з 3-х оболонок, де у середній спостерігається наявність 40-50 вікончастих еластичних мембран. Яка судина представлена на препараті?

Кровоносний капіляр

Артерія змішаного типу

Артерія еластичного типу

Вена м'язового типу

3675 / 4299
Для лікування хвороби Паркінсона застосовують попередник дофаміну - ДОФА. З якої амінокислоти утворюється ця активна речовина?

Цистеїн

Гістидин

Тирозин

Аланін

Триптофан

3676 / 4299
Пасажири автобуса у спекотну погоду попросили відкрити люки. Який шлях тепловіддачі при цьому активується?

Конвекція

Теплопроведення

Випромінювання

Випаровування поту

Випромінювання та теплопроведення

3677 / 4299
Людина в спекотну погоду тривалий час була позбавлена можливості пиття, що спричинило виражене відчуття спраги. Зміна якого гомеостатичного показника крові стала першопричиною цього?

Рівень глюкози

рН

Онкотичний тиск плазми

Осмотичний тиск плазми

3678 / 4299
У пацієнта віком 30 років діагностовано пухлину тіла нижньої щелепи, яка з'явилася кілька місяців тому. Макроскопічно пухлина представлена щільною білуватою тканиною, яка руйнує кістку щелепи. Під час мікроскопічного дослідження видаленої пухлини виявлено, що вона побудована із сітки тяжів одонтогенного епітелію з різноманітними розгалудженнями. Укажіть цю пухлину.

Акантоматозна амелобластома

Плексиформна амелобластома

Фолікулярна амелобластома

Гранулярно-клітинна амелобластома

Базальноклітинна амелобластома

3679 / 4299
Пацієнтці хірургічним шляхом видалено утворення біля верхівки зуба, яке мало фіброзну капсулу. Під час мікроскопічного дослідження виявлено: утворення складається з фібробластів, макрофагів, невеликої кількості лімфоцитів, плазматичних та ксантомних клітин, кристалів холестерину, поодиноких клітин сторонніх тіл, а також тяжів багатошарового епітелію. Назвіть це утворення.

Проста гранульома

Складна гранульома

Кістогранульома

Фолікулярна кіста щелепи

Радікулярна кіста щелепи

3680 / 4299
До комплексної терапії хворого на бронхопневмонію, що супроводжується виснажливим сухим кашлем, лікар включив муколітичний препарат, який деполімеризує мукопротеїди. Вкажіть цей препарат:

Кодеїн

Атенолол

Строфантин

Неодикумарин

Ацетилцистеїн

3681 / 4299
При деяких захворюваннях товстої кишки змінюються кількісні співвідношення між епітеліоцитами слизової оболонки. Які типи клітин у нормі переважають в епітелії крипт товстої кишки?

Малодиференційовані клітини

Ендокриноцити

Келихоподібні клітини

Клітини з ацидофільними гранулами

3682 / 4299
У хворого під час прийому виникла атріовентрикулярна блокада. Яка з наведених речовин придатна для надання невідкладної допомоги у цьому разі?

Атропін

Атенолол

Платифілін

Пірензепін

3683 / 4299
Яка біологічно активна речовина стимулює виділення гідрокарбонатних іонів клітинами проток підшлункової залози?

---

Гастрин

Гістамін

Секретин

3684 / 4299
Під час повного (з водою) аліментарного голодування у людини розвиваються генералізовані набряки. Який із патогенетичних факторів у цьому разі є провідним?

Зниження гідростатичного тиску міжклітинної рідини

Зниження осмотичного тиску плазми крові

Підвищення онкотичного тиску тканинної рідини

Зниження онкотичного тиску плазми крові

3685 / 4299
У хворого на підставі визначення порфіринів у слині діагнозована порфірія. Синтез якої сполуки порушується при цьому захворюванні?

Креатину

Гему

Сечової кислоти

Фосфоліпідів

3686 / 4299
Водій віком 30 років під час ДТП отримав черепно-мозкову травму. Він став скаржитися на спрагу та збільшення добової кількості сечі. Який відділ головного мозку в нього уражений?

Мозочок

Передня центральна звивина

Гіпоталамус

Ретикулярна формація

Тім'яна частка мозку

3687 / 4299
Пацієнта шпиталізовано на 5-й день хвороби з проявами жовтяниці, болями у м'язах, ознобом і носовими кровотечами. Під час проведення лабораторної діагностики виконано темнопольну мікроскопію краплини крові пацієнта. Укажіть збудників хвороби.

Calymmatobacterium granulomatis

Leptospira interrogans

Rickettsia mooseri

Borrelia dutlonii

Bartonella bacilloformis

3688 / 4299
У пацієнта на прийомі у лікаря-стоматолога виникла гіперсалівація. Препарати якої групи мають властивість пригнічувати вироблення слини?

Холіноблокатори

В`яжучі препарати

Холіноміметики

Адреноблокатори

3689 / 4299
Після закапування в око крапель у хворого розвинувся мідріаз і параліч акомодації. Яка група препаратів здатна викликати такий ефект?

Антихолінестеразні

beta-адреноміметики

М-холінолітики

alpha-адреноблокатори

3690 / 4299
У процесі старіння у людини спостерігається зменшення синтезу та секреції підшлункового соку, зменшення вмісту в ньому трипсину. Це призводить до порушення розщеплення:

Фосфоліпідів

Нуклеїнових кислот

Полісахаридів

Ліпідів

Білків

3691 / 4299
Деякими регіонами України поширилися місцеві випадки малярії. З якими комахами це пов'язано?

Мокреці родини Ceratopogonidae

Комарі роду Anofeles

Мошки роду Simullium

Гедзі родини Tabanidae

3692 / 4299
У вертикальному положенні пацієнт, заплющуючи очі, втрачає рівновагу. Які структури мозку у нього найімовірніше уражені?

Лімбічна система

Таламус

Мозочок

Базальні ганглії

3693 / 4299
Виявлено, що у пацієнтів із COVID-19 легеневими клітинами-мішенями для коронавірусу SarsCov-2 є пневмоцити ІІ-го типу. Яка функція альвеолярного епітелію насамперед порушується у разі вірусного ураження цих клітин?

Продукція слизу

Доочистка повітря в альвеолах

Газообмін

Синтез сурфактанту

3694 / 4299
Пацієнту віком 40 років встановлено діагноз: герпетичний стоматит. Який противірусний препарат потрібно призначити в цьому разі?

Ацикловір

Тінідазол

Оксацилін натрію

ПАСК

3695 / 4299
При деяких спадкових хворобах (наприклад синдром Кернса-Сейра) спостерігається деструкція мітохондрій. Які процеси у клітині можуть бути порушені внаслідок цього?

Поділ ядра

Синтез ліпідів

Синтез АТФ

Синтез білків

3696 / 4299
Укажіть, що не характерно для низькомолекулярних гепаринів (еноксапарин, фраксипарин та ін.).

Біодоступність вище, ніж у гепарину

Уводять підшкірно 1--2 рази на добу

Посилюють гальмівну дію антитромбіну III на фактор Ха

Відсутня інгібуюча дія на тромбін

Антиагрегантна і антикоагулянтна активність

3697 / 4299
У жінки з ендокринною патологією виявлено підвищення основного обміну на 15%. Збільшення продукування якого гормону може обумовлювати цей патологічний стан?

Глюкагону

Трийодтироніну

Тиреокальцитоніну

Соматостатину

3698 / 4299
Укажіть серологічну реакцію, для проведення якої необхідні 5 інгредієнтів: антиген, антитіло і комплемент (перша система), еритроцити барана та гемолітична сироватка (друга система).

Нейтралізації

Гальмування гемаглютинації

Зв'язування комплементу

Пасивної (непрямої) гемаглютинації

3699 / 4299
Визначення якого ферменту в крові є найбільш інформативним у перші години після виникнення інфаркту міокарда?

Глутаматдегідрогенази

Креатинфосфокінази

Лактатдегідрогенази

Аланінамінотрансферази

Аспартатамінотрансферази

3700 / 4299
У пацієнта віком 60 років виявлено погіршення сприйняття звуків високої частоти. Порушення стану яких структур слухового аналізатора зумовлює ці зміни?

Барабанної перетинки

Основної мембрани завитки біля овального віконця

М’язів середнього вуха

Євстахієвої труби

Основної мембрани завитки біля гелікотреми

3701 / 4299
На розтині тіла чоловіка 46-ти років, що не лікувався від гострої кишкової інфекції і помер від сепсису, виявлені: флегмона параректальної клітковини, множинні виразки прямої і сигмоподібної кишок, окремі з перфорацією, слизова цих відділів кишечника потовщена, вкрита сіруватого кольору плівками, що важко знімаються. Про яке захворювання можна думати?

Туберкульоз

Дизентерія

Амебіаз

Холера

Черевний тиф

3702 / 4299
При розтині тіла 2-річної дитини, померлої від менінгіту, виявлена відсутність тимусу, Т-залежних зон в периферичній лімфоїдній тканині. Який імунодефіцитний синдром характеризується такими змінами?

Синдром недостатності клітинного імунітету

Синдром недостатності гуморального імунітету

Вторинний імунодефіцитний синдром

Комбінований імунодефіцитний синдром

Синдром недостатності моноцитарних фагоцитів

3703 / 4299
У хворого, який переніс травму і подальший геморагічний бурсит лівого колінного суглоба, при огляді через 3 місяці відзначається обмеження обсягу рухів у цьому суглобі внаслідок утворення рубця. Який компонент запалення є основою розвитку цього ускладнення?

Порушення мікроциркуляції

Альтерація первинна

Проліферація

Альтерація вторинна

Ексудація

3704 / 4299
Після обстеження у хворого виявлені симптоми акромегалії. Яка ендокринна залоза залучена до патологічного процесу?

Аденогіпофіз

Наднирники

Щитоподібна залоза

Нейрогіпофіз

Епіфіз

3705 / 4299
У чоловіка діагностували анацидний гастрит. Для поліпшення роботи шлунку йому призначили таблетки ацидин-пепсину. Який вид лікування призначений пацієнту?

Психотерапія

Профілактичне застосування

Етіотропна терапія

Симптоматична терапія

Замісна терапія

3706 / 4299
У хворого порушено процес акомодації ока. Який м'яз пошкоджений?

Musculus dilatator pupillae

Musculus rectus superior

Musculus sphincter pupillae

Musculus rectus inferior

Musculus ciliaris

3707 / 4299
У хворого спостерігається запальний процес нижньої стінки очної ямки. Крізь яке анатомічне утворення можливо розповсюдження запального процесу до крило-піднебінної ямки?

Надочноямкова вирізка

Зоровий канал

Нижня очноямкова щілина

Задні решітчасті отвори

Верхня очноямкова щілина

3708 / 4299
Хворому з бронхопневмонією лікар призначив ацетилцистеїн. Визначте показання для його застосування:

Бронхіальна астма

Серцева недостатність

Бронхіт з харкотинням

Асфіксія новонароджених

Судоми

3709 / 4299
Під час профілактичного огляду в чоловіка виявлено: збільшення щитовидної залози, екзофтальмія, підвищення температури тіла до 37,3^oС, тахікардія, тремтіння пальців рук. Яка патологія щитоподібної залози розвинулась у пацієнта?

Спорадичний кретинізм

Мікседема

Аденома щитоподібної залози

Хвороба Грейвса

Ендемічний зоб

3710 / 4299
Хворий отримав травму голови. При обстеженні виявлена підшкірна гематома скроневої ділянки. Пошкодження якої судини призвело до появи гематоми?

А. occipitalis

А. buccalis

А. temporalis superficialis

А. auricularis posterior

А. maxillaris

3711 / 4299
У людини під час емоційного збудження спостерігається прискорення та посилення частоти серцевих скорочень, підвищення тиску, розширення зіниць, збільшення вентиляції легень. Який гормон є причиною такого стану людини?

Тестостерон

Паратгормон

Адреналін

Інсулін

Альдостерон

3712 / 4299
Гідроксипролін є важливою амінокислотою у складі колагену. За участю якого вітаміну відбувається утворення цієї амінокислоти шляхом гідроксилювання проліну?

B_6

D

B_1

B_2

C

3713 / 4299
Гістологічне дослідження тканини виявило, що в ній відсутні кровоносні судини, а клітини щільно прилягають одна до одної, утворюючи пласти. Яка це тканина?

Хрящова

Епітеліальна

М'язова

Нервова

Кісткова

3714 / 4299
У пацієнта під час огляду спостерігається синдром CREST, що поєднує: кальциноз шкіри, синдром Рейно, порушення моторики стравоходу, склеродактилії та телеангіоектазії. Для якого захворювання характерні ці симптоми?

Системного червоного вовчака

Системної склеродермії

Ревматоїдного артриту

Дерматоміозиту

Подагричного артриту

3715 / 4299
У десятирічної дитини виявлено гельмінтоз. Які зміни у лейкоцитарній формулі крові будуть спостерігатися у цьому разі?

Зросте кількість тромбоцитів

Зросте кількість сегментоядерних нейтрофілів

Зросте кількість базофілів

Зросте кількість еритроцитів

Зросте кількість еозинофілів

3716 / 4299
У людини з четвертою групою крові (генотип IAIB) в еритроцитах одночасно присутні антиген А, який контролюється алелем IA, і антиген В - продукт експресії алеля IB) Прикладом якої взаємодії генів є дане явище?

Епістаз

Кодомінування

Полімерія

Комплементарність

Неповне домінування

3717 / 4299
Хворий на мегалобластну анемію приймав препарат з групи водорозчинних вітамінних засобів. Визначте цей препарат:

Тіаміну хлорид

Аскорбінова кислота

Ціанокобаламін

Піридоксин

Токоферолу ацетат

3718 / 4299
У дорослої людини протягом життя у деяких клітинах не відбувається мітоз і кількісний вміст ДНК залишається постійним. Укажіть ці клітини.

Ендотелію

Нервові

Кровотворні

М'язові (гладкі)

Епідермісу

3719 / 4299
При обстеженні людини дуже маленького зросту (карлика) виявлені дитячі риси обличчя, нормальні пропорції тіла, недорозвинення вторинних статевих ознак. У такої людини знижена гормональна активність:

Задньої частки гіпофіза

Передньої частки гіпофіза

Щитоподібної залози

Тимуса

Середньої частки гіпофіза

3720 / 4299
У хірургічному відділенні перев'язувальні матеріали стерилізували в автоклаві. Через недогляд медсестри режим стерилізації був порушений і температура в автоклаві досягала лише 100^oC замість належних 120^oC) Які мікроорганізми могли зберегти життєздатність за таких умов?

Бацили і клостридії

Цвілеві і дріжджоподібні грибки

Стафілококи і стрептококи

Сальмонели і клебсієли

Коринебактерії і мікобактерії

3721 / 4299
У жінки 30-ти років з'явилися ознаки вірилізму (ріст волосся на тілі, облисіння скронь, порушення менструального циклу). Гіперпродукція якого гормону може спричинити такий стан?

Пролактин

Естріол

Тестостерон

Окситоцин

Релаксин

3722 / 4299
Психологічне дослідження встановило: у людини добра здатність швидко пристосовуватися до нової обстановки, добра пам'ять, емоційна стійкість, висока працездатність. Найімовірніше, ця людина:

Сангвінік

Флегматик

Холерик

Флегматик з елементами меланхоліка

Меланхолік

3723 / 4299
Під час експерименту у тварини виведено назовні загальну жовчну протоку. Які процеси травлення порушуються у цьому разі?

Гідроліз і всмоктування жирів

Всмоктування води

Гідроліз і всмоктування вуглеводів

Гідроліз і всмоктування жирів, білків та вуглеводів

Гідроліз і всмоктування білків

3724 / 4299
Пацієнта віком 35 років шпиталізовано зі скаргами на біль у нижній щелепі справа, високу температуру тіла, озноб та припухлість. Під час обстеження виявлено відшарування окістя з накопиченням запального ексудату між ним і кісткою, із перифокальним набряком м'яких тканин та частково розплавленим окістям. Для якого захворювання характерні ці симптоми?

Фіброзного періодонтиту

Гранулюючого періодонтиту

Гнійного періоститу

Пародонтозу

Локального пародонтиту

3725 / 4299
Пацієнтам із ішемічною хворобою серця призначають невеликі дози аспірину, який інгібує синтез активатора агрегації тромбоцитів тромбоксану А2. Укажіть кислоту, із якої утворюється тромбоксан А2.

Арахідонова

Гомогентизинова

Оцтова

Малонова

Глутамінова

3726 / 4299
Хворого оперують з приводу травми скронево-нижньощелепного суглоба. Під час розсічення виявлена структура, що доповнює конгруентність суглобових поверхонь. Що це за структура?

Меніск

Зв'язка

Губа

Складка

Диск

3727 / 4299
Під впливом іонізуючого випромінення або у разі авітамінозу Е в клітині спостерігається підвищення проникності мембран лізосом. До яких наслідків може призвести така патологія?

До інтенсивного синтезу білків

До відновлення цитоплазматичної мембрани

До формування веретена поділу

До часткового чи повного руйнування клітини

До інтенсивного синтезу енергії

3728 / 4299
Регуляція експресії генів здійснюється за рахунок різних механізмів. Укажіть ділянку ДНК при індукції якої активується експресія гену.

Сайленсер

Термінатор

Атенюатор

Спейсер

Енхансер

3729 / 4299
У пацієнта розвинувся анафілактичний шок після застосування новокаїну для провідникової анестезії. Який з наведених препаратів є препаратом вибору для його усунення?

Адреналіну гідрохлорид

Норадреналіну гідротартрат

Преднізолон

Димедрол

Супрастин

3730 / 4299
У хворого стенокардія. Який антиангінальний засіб textbfПРОТИПОКАЗАНИЙ для лікування, якщо в пацієнта алергія на йод?

Нітросорбід

Нітрогліцерин

Аміодарон

Верапаміл

Дротаверин

3731 / 4299
Під час ультразвукового дослідження дитини 1,5 років встановлено незарощення овального отвору. Де у серці розташоване це анатомічне утворення?

Міжшлуночкова перегородка

Стінка лівого шлуночка

Міжпередсердна перегородка

Стінка правого шлуночка

3732 / 4299
Під час аутопсії жінки віком 64 років, яка хворіла на туберкульоз, виявлено щільну, збільшену в розмірі селезінку, із множинними сіро-білими дрібними вогнищами. За результатами мікроскопічного дослідження встановлено: вогнища мають казеозний некроз у центрі, навколо якого знаходяться епітеліоїдні, багатоядерні гігантські клітини та лімфоцити. Яке ураження селезінки розвинулося у цієї жінки?

Сагова селезінка

Септична селезінка

Парфірова селезінка

Міліарний туберкульоз селезінки

Сальна селезінка

3733 / 4299
Під час аускультації серця був визначений систолічний шум у ІІ міжреберному проміжку на лівій пригрудинній лінії. Патологію якого клапана вислухав лікар?

Двостулкового клапана

Клапана легеневого стовбура

Клапана верхньої порожнистої вени

Клапана аорти

Тристулкового клапана

3734 / 4299
Під час обстеження головного мозку на комп'ютерному томографі лікар виявив пухлину, що локалізується на ділянці нижнього слиновидільного ядра. Який це відділ головного мозку?

Мозочок

Проміжний мозок

Міст

Середній мозок

Довгастий мозок

3735 / 4299
На аутопсії жінки, яка померла від гострого інфаркту міокарда, у вені лівої гомілки виявлений тромб. При мікроскопічному дослідженні тромба виявлено, що він заміщений сполучною тканиною, в якій відзначається дифузне відкладення солей кальцію. Дайте назву такому результату тромбозу:

Петрифікація

Септичний аутоліз

Організація і каналізація

Організація

Асептичний аутоліз

3736 / 4299
У хворого мисливця через 2 дні після розділки тушки ховраха підвищилась температура тіла до 39^oC, збільшились лімфовузли. У подальшому у нього розвинулася пневмонія з серозно-геморагічним ексудатом, у якому виявлені овоїдної форми мікроорганізми з біполярним забарвленням. Який діагноз можна припустити у даного хворого?

Псевдотуберкульоз

Правець

Чума

Бруцельоз

Сибірка

3737 / 4299
Під час мікроскопічного дослідження прямої кишки виявлені великі вогнища некрозу слизової оболонки. Некротичні маси просякнуті фібрином і утворюють плівку. Слизова і підслизова оболонки на периферії ділянок некрозу повнокровні, набряклі, з крововиливами та лейкоцитарною інфільтрацією. Яке захворювання можна припустити?

Амебіаз

Дизентерія

Черевний тиф

Холера

Сальмонельоз

3738 / 4299
Пацієнт скаржиться на біль очних яблук. Під час огляду виявлено підвищення внутрішньоочного тиску. Порушення відтоку якої рідини спричинило цей стан?

Перилімфи

Ендолімфи

Лімфи

Сльози

Водянистої вологи

3739 / 4299
У хворого запалення привушної слинної залози. Визначте, який нерв залучений в запальний процес:

N. facialis

N. maxillaris

N. linqualis

N. tympanicus

N. mandibularis

3740 / 4299
У хворого, який страждає на жовчнокам'яну хворобу, внаслідок обтурації жовчовивідних шляхів виділяється знебарвлений жирний кал. Відсутність якого компонента жовчі обумовлює явище стеатореї?

Жирних кислот

Жовчних кислот

Лужної фосфатази

Жовчних пігментів

Холестерину

3741 / 4299
Мазок із зубного нальоту було зафарбовано за методом Буррі-Гінса. Під час мікроскопічного дослідження препарату виявлено: на темно-коричневому тлі спостерігаються червоні клітини мікроорганізмів, деякі бактерії оточені світлим ореолом. Яку структуру мікроорганізмів виявлено?

Зовнішня мембрана

Капсула

Протопласт

Екзоферменти, пов'язані з клітинною стінкою

Шар пептидоглікану

3742 / 4299
У 8-ми місячної дитини спостерігаються часті та сильні підшкірні крововиливи. Призначення синтетичного аналога вітаміну K (вікасолу) дало позитивний ефект. Цей вітамін бере участь у gamma-карбоксилуванні глутамінової кислоти такого білка зсідання крові:

Фактор Розенталя

Проконвертин

Фактор Хагемана

Фібриноген

Протромбін

3743 / 4299
У пацієнта хронічний неврит трійчастого нерва. Який із травних процесів буде порушено найбільше у цьому разі?

Ковтання

Слиновиділення

Жування

Слиноутворення

Формування смакових відчуттів

3744 / 4299
Лікар-стоматолог застосував в якості антисептика розчин калію перманганату. Бактерицидний ефект препарату забезпечує:

Гідроксид калію

Мангану оксид

Калій

Атомарний кисень

Калію оксид

3745 / 4299
Хворому вживлено зубні імпланти. За 3 тижні відбулося їх відторгнення. Які клітини крові беруть найбільшу участь у цьому патологічному процесі?

Імуноглобуліни М

Плазмоцити

Імуноглобуліни E

Т-лімфоцити

В-лімфоцити

3746 / 4299
Під час проведення експерименту на збудливу клітину подіяли тетраетиламонієм, який блокує калієві іоноселективні канали. Як це позначиться на мембранному потенціалі клітини?

Розвинеться деполяризація

Потенціал спокою зникне

Потенціал спокою збільшиться

Потенціал дії не виникне

Розвинеться гіперполяризація

3747 / 4299
Яку артерію можна пошкодити при виконанні провідникової анестезії в ділянці отвору нижньої щелепи?

Нижня альвеолярна

Щічна

Середня оболонна

Криловидні гілки

Язикова

3748 / 4299
Людина потрапила в ситуацію, що пов'язана з емоційним напруженням. У результаті цього в неї в крові підвищився рівень адреналіну, і як наслідок, збільшилась сила серцевих скорочень. Яким чином адреналін збільшує силу серцевих скорочень?

Активує барорецептори судин

Знижує тонус блукаючих нервів

Знижує збудливість клітин-пейсмекерів

Активує beta-адренорецептори серця

Активує периферичні хеморецептори

3749 / 4299
Який вид апатиту складає найбільшу частку мінерального компоненту зубів людини?

Стронцієвий апатит

Хлорапатит

Карбонатний апатит

Гідроксиапатит

Фторапатит

3750 / 4299
Чоловік з інфільтративною формою туберкульозу легень лікувався ізоніазидом. За деякий час у нього з'явилися симптоми В6-гіповітаминозу. Яка причина цього стану?

Прискорюється біотрансформація вітаміну В6

Сповільнюється всмоктування вітаміну В6

Утворюється міцний зв'язок вітаміну з білками плазми крові

Ізоніазид є антагоністом вітаміну В6

Прискорюється елімінація вітаміну В6

3751 / 4299
Для лікування гінгівіту лікар-стоматолог призначив пацієнту препарат із протипротозойною та антибактеріальною діями, що може викликати відразу до алкоголю. Укажіть призначений лікарем препарат.

Цефтріаксон

Метронідазол

Левоміцетин

Тетрациклін

Лінкоміцину гідрохлорид

3752 / 4299
При обстеженні хворої дитини лікар виявив часткову відсутність сосочків на бічній поверхні язика. Які це сосочки?

Листоподібні

Грибоподібні

Ниткоподібні

Конічні

Жолобуваті

3753 / 4299
Під час обстеження пацієнта спостерігається: потовщення шиї, екзофтальм, підвищення температури тіла, пульс - 110/хв. Рівень яких гормонів у крові пацієнта потрібно визначити?

Кортизолу

Інсуліну

Тироксину

Статевих гормонів

Катехоламінів

3754 / 4299
При введенні хворому з метою проведення провідникової анестезії засобу, який використовується в хірургічній стоматології, виникли симптоми отруєння: збудження ЦНС з наступним паралічем, гостра серцево-судинна недостатність (колапс), в патогенезі якого має значення сенсибілізація до даного препарату. Виникли також алергічні реакції (свербіж, набряклість, еритема). Визначте препарат:

Тіопентал-натрій

Дитилін

Тубокурарину хлорид

Ардуан

Лідокаїн

3755 / 4299
У гістологічному препараті стінки серця між ендокардом та міокардом виявляються крупні клітини зі світлою цитоплазмою та ексцентрично розміщеним ядром. Які клітини серця мають вищенаведені морфологічні ознаки?

Ендокринні клітини

Ліпоцити

Пейсмекерові клітини

Клітини Пуркін'є

Скоротливі кардіоміоцити

3756 / 4299
У постраждалого з травмою основи черепа встановлене пошкодження гачкоподібного відростка присередньої пластинки крилоподібного відростка клиноподібної кістки. Функція якого м’яза м’якого піднебіння буде порушена?

М’яза язичка

Піднебінно-язикового

Підіймача піднебінної завіски

Напрягача піднебінної завіски

Піднебінно-глоткового

3757 / 4299
У значної кількості пацієнтів перед відвідуванням стоматолога виникає тривожність, страх, пригнічений настрій. Посилення секреції якого медіатора центральної нервової системи може зменшити ці зміни психо-емоційного стану у людини?

Дофамін

Серотонін

Ацетилхолін

Норадреналін

ГАМК

3758 / 4299
Пацієнт упродовж тривалого часу вживав антибіотики широкого спектру дії, що викликало зниження апетиту, нудоту, пронос з гнилісним запахом випорожнень, схуднення. Яка побічна дія антибіотиків виникла у процесі лікування?

Дисбактеріоз

Гепатотоксична дія

Пряма подразнювальна дія

Нефротоксична дія

Алергічна реакція

3759 / 4299
У хворого з гіпертонічною хворобою виявлено значне збільшення маси міокарда лівого шлуночка. Це сталося внаслідок:

Жирової інфільтрації міокарда

Збільшення об'єму кардіоміоцитів

Затримки води в міокарді

Розростання сполучної тканини

Збільшення кількості кардіоміоцитів

3760 / 4299
Потерпілому з резус-негативною кров'ю після гострої крововтрати помилково перелили резус-позитивну кров. Які зміни в крові відбудуться у цьому разі?

Агрегація еритроцитів донора

Еритроцитоз

Гемоліз еритроцитів донора

Агрегація тромбоцитів

Гемоліз еритроцитів реципієнта

3761 / 4299
У хворого 26-ти років на фоні хронічного гастриту з недостатністю внутрішнього фактору Кастла констатована анемія. Який тип анемії характерний для даного випадку?

Гіпопластична

Таласемія

Хронічна постгеморагічна

Залізодефіцитна

B_12-фолієводефіцитна

3762 / 4299
Чоловік звернувся до лікаря зі скаргами на біль у горлі. Під час обстеження пацієнта виявлено гіпертрофію лімфоїдного органу, який знаходиться в fossa tonsillaris. Укажіть цей орган.

Tonsilla palatina

Tonsilla adenoidea

Tonsilla pharyngea

Tonsilla lingualis

Tonsilla tubaria

3763 / 4299
До відділення інтенсивної терапії шпиталізовано пацієнта з нападом стенокардії. Який препарат необхідно застосувати пацієнту для усунення нападу?

Гепарин

Вікасол

Фуросемід

Кальцію хлорид

Нітрогліцерин

3764 / 4299
Від хворого з підозрою на черевний тиф була виділена грамнегативна рухлива паличка. З метою ідентифікації виділену культуру посіяли на напіврідкі середовища Гіса. За якою ознакою буде враховуватися розкладання мікробом вуглеводів до кислоти?

Утворення осаду

Розрідження середовища

Зміна забарвлення індикатора

Утворення бульбашок газу

Помутніння живильного середовища

3765 / 4299
У хворого сифілісом на шкірі шиї виникли плями, що відрізняються більш блідим кольором. Назвіть розлад пігментного обміну:

Лентіго

Меланодермія

Лейкодерма

Порфірія

Ксеродерма

3766 / 4299
Людина після ДТП надійшла в реанімаційне відділення у важкому стані, який характеризується визначенням: важкий патологічний процес, що супроводжується виснаженням життєво важливих функцій організму і приводить його на межу життя і смерті через критичне зменшення капілярного кровообігу в уражених органах. В якому стані перебуває людина?

Шок

Преагонія

Кома

Агонія

Колапс

3767 / 4299
Під час експерименту внаслідок перерізки блукаючого нерва у тварини різко підвищився рівень глюкози в крові, що зумовлено:

Підвищенням секреції глюкагону

Підвищенням секреції інсуліну

Підвищенням секреції соматостатину

Зниженням секреції інсуліну

Зниженням секреції глюкагону

3768 / 4299
У порожнині матки виявлено mboxембріон людини, який не прикріплений до ендометрію. На якій стадії розвитку перебуває зародок?

Морули

Бластоцисти

Гаструли

Зиготи

Нейрули

3769 / 4299
Хворий 40-ка років після багатомісячного плавання в районах Західної Африки повернувся додому. Через 15 днів відчув слабкість, головний біль, підвищилась температура, з'явилась лихоманка. Лікарем поставлений діагноз ''малярія''. Якими методами лабораторної діагностики можна підтвердити цей діагноз?

Бактеріологічний, алергічний

Мікроскопічний, культуральний

Мікроскопічний, серологічний

Бактеріоскопічний, біологічний

Серологічний, біологічний

3770 / 4299
Методом амніоцентезу у клітинах епітелію плода визначено каріотип 45, Х0. Мати і батько здорові. Який синдром імовірно розвинувся у плода?

Трисомії Х

<<Котячого крику>>

Шерешевського-Тернера

Едвардса

Патау

3771 / 4299
У 72-річного хворого з цирозом печінки розвинулась печінкова кома. Її розвиток зумовлений надходженням у загальний кровотік через портокавальні анастомози речовин, що знешкоджуються в печінці (синдром портальної гіпертензії) та некрозом гепатоцитів. Для якого виду печінкової коми це характерно?

Кетоацидотична

Печінково-клітинна

Паренхіматозна

Змішана

Шунтова

3772 / 4299
Хворому на злоякісну пухлину язика хірург перев'язує A) Lingualis у ділянці трикутника Пирогова. Водночас він особливу увагу звертає на:

N. glossopharynseus

N. hypoglossus

Ansa cervicalis

N. sublingualis

N. lingualis

3773 / 4299
Під час проведення бактеріологічного дослідження зубного нальоту з ротової порожнини десятирічної дитини виявлено Streptococcus mutans у великій кількості. Розвиток якого патологічного процесу може спричинити цей мікроорганізм?

Карієсу

Виразково-гангренозного стоматиту

Пародонтозу

Везикулярного стоматиту

Хронічного пульпиту

3774 / 4299
Яка з нижчеперерахованих особливо небезпечних інфекцій передається від хворої тварини людині через укуси бліх і характеризується лімфогенним розповсюдженням збудника з характерним геморагічним запаленням реґіонарних лімфатичних вузлів?

Чума

Туберкульоз

Туляремія

Сибірська виразка

Холера

3775 / 4299
У стоматологічній практиці застосовують місцеві анестетики, які блокують такі іонні канали:

Повільні кальцієві

Хлорні

Калієві

Швидкі кальцієві

Натрієві

3776 / 4299
Лізоцим - фермент з класу гідролаз, що визначає захисну функцію слини. Його антибактеріальні властивості зумовлені здатністю порушувати структурну цілісність клітинної стінки бактерій, гідролізуючи:

Пептидні зв'язки білків

Ефірні зв'язки ліпідів

Глікозидні зв'язки мукополісахаридів

Глікозидні зв'язки азотистих основ і пентоз

Антигени і ендотоксини, розташовані у стінці

3777 / 4299
Під час трепанації черепа на скроневій кістці лікар-нейрохірург виявив трійчастий вузол. Де знаходиться цей вузол?

На лускатій частині

На передній поверхні кам’янистої частини

На задній поверхні кам’янистої частини

На нижній поверхні кам’янистої частини

На барабанній частині

3778 / 4299
Аналіз сечі хворого на цукровий діабет показав наявність глюкозурії. Нирковий поріг для глюкози становить:

1,0 ммоль/л

8,88 ммоль/л

20,0 ммоль/л

5,55 ммоль/л

15,5 ммоль/л

3779 / 4299
В сечі новонародженого визначається цитрулін та високий рівень аміаку. Вкажіть, утворення якої речовини найімовірніше порушене у цього малюка:

Сечовина

Аміак

Креатинін

Креатин

Сечова кислота

3780 / 4299
Досліджено ембріональний орган, в якому формуються перші формені елементи крові як тканини. Назвіть цей орган:

Жовтковий мішок

Селезінка

Печінка

Червоний кістковий мозок

Тимус

3781 / 4299
Під час аварії на атомному підводному човні солдат строкової служби одержав дозу опромінення 5 Гр. Скаржиться на головний біль, нудоту, запаморочення. Які зміни в кількості лейкоцитів можна очікувати у хворого після опромінення?

Еозинофілія

Агранулоцитоз

Лімфоцитоз

Лейкопенія

Нейтрофільний лейкоцитоз

3782 / 4299
Під час огляду хворого при проведенні аускультації лікар оцінює роботу мітрального клапана. Де вислуховують тон цього клапану?

У края груднини в другому міжребер'ї зліва

На верхівці серця

У края груднини справа напроти хряща 5 ребра

У края груднини в другому міжребер'ї справа

У края груднини зліва напроти хряща 5 ребра

3783 / 4299
У пацієнта запалення верхньощелепного нерва. Через який отвір черепа проходить цей нерв?

Яремний

Остистий

Круглий

Овальний

3784 / 4299
Під час морської прогулянки у чоловіка виникли прояви хвороби руху: збліднення, пітливість, запаморочення, нудота, прискорене дихання, зниження артеріального тиску. Укажіть причину цього стану у чоловіка.

Активація симпатичного відділу автономної нервової системи

Активація парасимпатичного відділу автономної нервової системи

Надмірне подразнення вестибулярного апарата

Надмірне подразнення вісцерорецепторів черевної порожнини

Порушення координації між зоровою і руховою системами

3785 / 4299
Хворий доставлений у лікарню із рваною раною щелепно-лицевої ділянки, що супроводжувалася сильною кровотечею, яку довго не могли зупинити. Яке порушення загального об'єму крові виникне протягом першої години після крововтрати?

Гіповолемія олігоцитемічна

Гіповолемія нормоцитемічна

Гіперволемія

Гіповолемія поліцитемічна

Порушень об'єму крові не буде

3786 / 4299
Під час аналізу крові лікар-лаборант зробив додатковий висновок, що вона належить людині жіночої статі. Особливості будови яких форменних елементів дають змогу зробити такий висновок?

Базофільних лейкоцитів

Моноцитів

Нейтрофільних лейкоцитів

Еритроцитів

Лімфоцитів

3787 / 4299
Пацієнту з гіпохромною анемією призначено залізовмісний препарат, який вводять тільки внутрішньовенно. Який це засіб?

Маніт

Ферковен

Фуросемід

Дихлотіазид

Кислота етакринова

3788 / 4299
У хворого діагностовано пухлину паращитоподібної залози. Генералізована фіброзна остеодистрофія. Періодичні напади ниркових кольок. Ультразвуковим обстеженням виявлено дрібні камені в нирках. Яка найбільш імовірна причина утворення цих каменів?

Гіперхолестеринемія

Гіперурикемія

Гіперкальціємія

Гіпокальціємія

Гіперфосфатемія

3789 / 4299
В матеріалі, взятому від хворої людини, знайдено декілька видів мікроорганізмів (стафілококи та стрептококи різних видів), які стали причиною захворювання. Як називається такий вид інфекції?

Вторинна інфекція

Змішана інфекція

Реінфекція

Коінфекція

Суперінфекція

3790 / 4299
Хворий скаржиться на сильний нежить та втрату відчуття запахів. Де в носовій порожнині пошкоджені рецептори нюхового аналізатору?

Хоани

Загальний носовий хід

Нижній носовий хід

Верхній носовий хід

Середній носовий хід

3791 / 4299
Під час огляду вагітної на слизовій оболонці порожнини рота стоматолог виявив 3 округлі утворення, які з'явилися 3 дні тому, з червоним обідком, поверхня їх біло-сірого кольору до 1 см у діаметрі. Стоматолог діагностував таке захворювання:

Гангренозний стоматит

Лейкоплакія

Виразково-некротичний стоматит

Катаральний стоматит

Афтозний стоматит

3792 / 4299
У потерпілого в автомобільній катастрофі гематома спинного мозку супроводжується загруднинними болями, тахікардією і підвищенням артеріального тиску. Ураження яких сегментів спинного мозку є причиною стану хворого?

C6-C8

S1-S3

Th1-Th5

L1-L3

3793 / 4299
Який із наведених сечогінних засобів не буде проявляти діуретичного ефекту у пацієнта із хворобою Аддісона?

Фуросемід

Гідрохлортіазид

Триамтерен

Етакринова кислота

Спіронолактон

3794 / 4299
Хворій 66-ти років для припинення гіпертонічного кризу було внутрішньовенно введено розчин магнію сульфату. Проте артеріальний тиск не знизився, а після повторного введення препарату з'явилися млявість, загальмованість, пригнічення свідомості та дихання. Який препарат є антагоністом магнію сульфату та усуває симптоми його передозування?

Кальцію хлорид

Калію перманганат

Натрію хлорид

Вугілля активоване

Калію хлорид

3795 / 4299
У пацієнта відмічена висока концентрація вазопресину (АДГ) у крові. До яких змін діурезу це призведе?

Анурія

Натрійурія

Глюкозурія

Олігоурія

Поліурія

3796 / 4299
У пацієнта карієс ускладнився пульпітом, що супроводжувався нестерпним болем. Укажіть основну причину виникнення цього болю при запаленні пульпи?

Ексудація

Проліферація

Еміграція лейкоцитів

Ішемія

Первинна альтерація

3797 / 4299
У хворого необхідно зменшити насосну функцію серця. Які мембранні циторецептори доцільно для цього заблокувати?

beta-адренорецептори

Н-холінорецептори

alpha-адренорецептори

alpha- та beta-адренорецептори

М-холінорецептори

3798 / 4299
У крові хворого на цукровий діабет збільшується вміст кетонових тіл, що призводить до метаболічного ацидозу. Із якої речовини синтезуються кетонові тіла?

Сукциніл-КоА

Пропіоніл-КоА

Малоніл-КоА

Ацетил-КоА

Метилмалоніл-КоА

3799 / 4299
Хірург пошкодив нерв, що іннервує щелепно-під'язиковий м'яз. Назвіть пошкоджений нерв:

N. facialis

N. accessorius

N. hypoglossus

N. trigeminus

N. glossopharyngeus

3800 / 4299
Речовини виводяться з клітини в результаті з'єднання мембранної структури апарату Гольджі з цитолемою. Вміст такої структури викидається за межі клітини. Цей процес має назву:

Ендоцитоз

Екзоцитоз

--

Полегшена дифузія

3801 / 4299
Юнак 25-ти років звернувся до лікаря зі скаргами на загальну слабкість, швидку втомлюваність, дратівливість, зниження працездатності, кровоточивість ясен. Недостатність якого вітаміну може мати місце у даному випадку?

Фолієва кислота

Аскорбінова кислота

Рибофлавін

Тіамін

3802 / 4299
У пацієнта, який тривалий час знаходився на незбалансованому харчуванні з маленькою кількістю білка, розвинулась жирова інфільтрація печінки. Назвіть речовину, відсутність якої у їжі могла бути причиною цього стану:

Арахідонова кислота

Біотин

Аланін

Метіонін

3803 / 4299
У хворого 49-ти років виявлено непропорційне збільшення кистей рук, стоп, носа, вух, надбрівних дуг і виличних кісток. У крові - гіперглікемія, порушення тесту толерантності до глюкози. Причиною розвитку даної патології найбільш імовірно є:

Гіперсекреція гормонів нейрогіпофізу

Гіперсекреція соматотропного гормону

Гіпосекреція інсуліну

Гіпосекреція вазопресину

3804 / 4299
Під час гістологічного дослідження видаленого зуба, виявлено меншу кількість та зменшені розміри одонтобластів і пульпоцитів зі склерозом сполучнотканинної основи пульпи. Яка патологія пульпи спостерігається?

Гострий пульпіт

Гіаліноз

Атрофія

Дистрофія

3805 / 4299
У хворого з патологією серцево-судинної системи спостерігається надмірне утворення ангіотензину ІІ, який синтезується за участю такого ферменту:

Ангіотензинперетворюючий фермент

Циклооксигеназа

Урокіназа

Кініназа

3806 / 4299
Під час експерименту, до ізольованої нервової клітини ссавця повністю зупинили надходження кисню. Як зміниться потенціал спокою у цьому разі?

Дещо збільшиться

Зникне

Суттєво збільшиться

Суттєво зменшиться

3807 / 4299
При лабораторному обстеженні у пацієнта виявлено дефіцит УДФ-глюкуронілтрансферази. Які показники крові є підтвердженням даної ензимопатії?

Уремія

Фенілкетонурія

Кетоацидоз

Гіпербілірубінемія

3808 / 4299
У дитини виявили порушення остеогенезу і розвиток карієсу, внаслідок недостатнього надходження в організм певного мікроелементу. Назвіть цей мікроелемент.

Залізо

Йод

Кобальт

Фтор

3809 / 4299
У пацієнта має місце пошкодження волокон дев'ятої пари черепних нервів (язикоглотковий). Формування якого відчуття буде порушено?

Солоне

Кисле

Гірке

Усіх смакових відчуттів

Солодке

3810 / 4299
При видаленні зуба руйнують зв'язки між цементом та зубною альвеолою. Визначте цю структуру:

Дентин

Ясна

Періодонт

Емаль

3811 / 4299
Хворого доставлено у лікарню з попереднім діагнозом ''ботулізм''. Яку серологічну реакцію слід застосувати для виявлення ботулінічного токсину в досліджуваному матеріалі?

Реакцію імунофлюоресценції

Реакцію преципітації

Реакцію нейтралізації

Реакцію аглютинації

3812 / 4299
У чоловіка 58-ми років клінічна картина гострого панкреатиту. Підвищення в сечі якої з перерахованих нижче речовин буде підтвердженням діагнозу?

Залишкового азоту

Сечовини

Альбуміну

Амілази

3813 / 4299
У хворого під час комбінованої терапії хронічної серцевої недостатності дигітоксином та фуросемідом розвинулася різка м'язова слабкість. Які електролітні порушення можна виявити в крові?

Гіперкаліємія

--

Гіперкальціємія

Гіпокаліємія

3814 / 4299
У пацієнта виявили збільшення активності G-клітин шлунку. Які зміни травлення у шлунку при цьому виникають?

Утворення великої кількості слизу

Підвищення секреції гастрину

Пригнічення моторики шлункау

Зниження активності ферментів

3815 / 4299
Після пошкодження мозку у людини порушене сприйняття зорової інформації. В якому відділі кори сталося пошкодження?

Передня центральна звивина

Задня центральна звивина

Потилична ділянка кори

Тім'яна ділянка кори

3816 / 4299
Який представник типу Членистоногі є переносником збудників бубонної чуми?

Блоха

Клоп

Москіт

Комар

3817 / 4299
Який показник є ознакою гострої ниркової недостатності?

Протеїнурія

Гематурія

Гіперазотемія

Глюкозурія

Гіперглікемія

3818 / 4299
В організмі самки комара роду Anopheles, малярійний плазмодій розмножується копуляцією (різновид статевого процесу). Яким хазяїном є ця комаха для малярійного плазмодія?

Резервуарним

Факультативним

Додатковим

Остаточним

3819 / 4299
У хворого на ішемічну хворобу серця визначено підвищений вміст в плазмі крові тригліцеридів і ліпопротеїнів дуже низької щільності. Який препарат слід призначити?

Фамотидин

Фенофібрат

Лізиноприл

Добутамін

3820 / 4299
Хворий 22-х років надійшов у стаціонар зі скаргами на лихоманку, слабкість. Один із збільшених шийних лімфатичних вузлів взятий на гістологічне дослідження. Результати: у тканині лімфовузла виявлені вогнища некрозу, оточені епітеліоїдними клітинами, гігантськими багатоядерними клітинами Пирогова-Лангханса, а також лімфоцитами. Яке захворювання можна припустити у даному випадку?

Лімфолейкоз

Саркоїдоз

Туберкульоз

Сифіліс

3821 / 4299
При мікроскопічному дослідженні бляшкоподібного утвору, видаленого з бокової поверхні язика у чоловіка із зубними протезами, виявлено значне потовщення епітеліального шару з явищами паракератозу, гіперкератозу та акантозу, у сполучній тканині - дрібні круглоклітинні інфільтрати. Діагностуйте патологічний стан:

Лейкоплакія

Іхтіоз

Хронічний глосит

Гунтерівський глосит

3822 / 4299
В експерименті у тварини подразнювали периферійний відрізок симпатичного нерва, що іннервує під'язикову слинну залозу. При цьому залоза виділяє:

Мало рідкої слини

Мало в'язкої слини

Багато рідкої слини

Слина не виділяється

3823 / 4299
Лікар-стоматолог виявив на слизовій оболонці порожнини рота семирічної дівчинки виразку, діаметром 1,5 см, із підритими нерівними краями і сірим дном. Після забарвлення зіскобу з виразки за Цілем-Нільсеном у препараті виявлено: тонкі палички рубіново-червоного кольору, розташовані у безладних скупченнях та поодинці. Для збудника якого захворювання характерні такі ознаки?

Кандидозу

Актиномікозу

Дифтерії

Туберкульозу

Сифілісу

3824 / 4299
Чоловік 56-ти років з вадою серця скаржиться на набряки нижніх кінцівок, що з'явилися останнім часом. Місцевим патогенетичним фактором набряку є:

Зниження проникливості стінки судини

Підвищення онкотичного тиску крові

Підвищення тканинного тиску

Підвищення гідродинамічного тиску крові

Зниження гідродинамічного тиску крові

3825 / 4299
Через декілька годин після травми зуба у пульпі спостерігається гіперемія судин, виражений набряк тканини з поодинокими нейтрофілами, лімфоцитами, у нервових волокнах - незначні дистрофічні зміни. Який найбільш імовірний діагноз?

Гангренозний пульпіт

Гнійний пульпіт

Серозний пульпіт

Фіброзний пульпіт

3826 / 4299
У пацієнтки під час обстеження виявлено потовщення шиї, екзофтальм, пульс - 110 уд/хв. Які додаткові дослідження необхідно провести цій пацієнтці для встановлення діагнозу?

Ультразвукове дослідження яєчників

Дослідження вмісту Т3 і Т4

Дослідження вмісту катехоламінів крові

Тест навантаження глюкозою

3827 / 4299
Хірург-стоматолог у жінки 24-х років діагностував гнійне запалення клиноподібної пазухи. Зроблено все можливе, щоб запобігти втягненню в процес стінки артерії, яка лежить в печеристій пазусі, та тим самим уникнути смертельної кровотечі. Яка це артерія?

A) carotis externa

A) carotis interna

A) infraraorbitalis

A) ophthalmica

3828 / 4299
Молодий чоловік віком 18 років звернувся до лікаря зі скаргами на деформацію обличчя. Під час обстеження виявлено: пухлиноподібне утворення в нижній щелепі. Під час його мікроскопічного дослідження встановлено: потовщена ділянка утворення складається з однорідних великих клітин типу гістіоцитів та великої кількості еозинофілів, а також спостерігається горизонтальна резорбція міжзубних перетинок. Для якого пухлиноподібного захворювання характерна така гістологічна картина?

Фіброматозний епуліс

Гігантоклітинний епуліс

Еозинофільна гранульома

Фіброзна дисплазія

Херувізм

3829 / 4299
У хворого діагностований запальний процес у ділянці вивідної протоки піднижньощелепної слинної залози. Куди відкривається ця протока?

Foramen caecum linguae

Vestibulum oris

Linea terminalis

Caruncula sublingualis

3830 / 4299
Про ураження яких клітин мозочка свідчать порушення координації рухів та тонусу м'язів під час алкогольної інтоксикації організму?

Кошикових клітин зернистого шару

Зірчастих клітин молекулярного шару

Грушеподібних нейронів шару Пуркін'є

Клітин Гольджі зернистого шару

3831 / 4299
У чоловіка виявлено твердий шанкр на статевих органах. Попередній діагноз --- сифіліс. Яка форма інфекції у цього пацієнта?

Змішана інфекція

Рецидив

Вторинна інфекція

Первинна інфекція

3832 / 4299
Пацієнту, який хворіє на тромбофлебіт, призначено антикоагулянт непрямої дії - синкумар. Укажіть, через який проміжок часу треба чекати розвитку максимального антикоагулянтного ефекту.

6 - 12 год

12 - 24 год

3 - 6 год

24 - 72 год

3833 / 4299
Пацієнту проведено глюкозотолерантний тест, який підтвердив відсутність у нього цукрового діабету. За який час після цукрового навантаження у здорової людини спостерігається максимальне підвищення рівня глюкози?

120 хв.

90 хв.

30 - 60 хв.

10 - 20 хв.

150 хв.

3834 / 4299
Хворого доставили до клініки у коматозному стані. В анамнезі цукровий діабет ІІ типу впродовж 5-ти років. Об'єктивно: дихання шумне, глибоке, у видихуваному повітрі чути запах ацетону. Вміст глюкози у крові 15,2 ммоль/л, кетонових тіл - 100 мкмоль/л. Для якого ускладнення цукрового діабету характерні такі клінічні прояви?

Кетоацидотична кома

Печінкова кома

Гіпоглікемічна кома

Гіперосмолярна кома

3835 / 4299
Для лікування туберкульозу призначений антибіотик, який забарвлює сечу в червоний колір. Вкажіть цей антибіотик:

Еритроміцин

Амоксицилін

Нітроксолін

Рифампіцин

Цефотаксим

3836 / 4299
Хворий 5 років скаржиться на сильний головний біль, блювання. Об'єктивно встановлено: ригідність м'язів потилиці, блювання без нудоти, герпетичні висипи на обличчі, лихоманка. На підставі бактеріологічних досліджень якого патологічного матеріалу можливе підтвердження попереднього діагнозу ''цереброспінальний менінгіт''?

Виділення бактерій N. Meningitidis зі слизової оболонки сечостатевої системи

Виділення уринокультур N. Meningitidis

Дослідження блювоти

Пункція спинномозкової рідини

3837 / 4299
У хворого на хронічний гепатит в аналізі крові на білкові фракції виявили зниження загальної кількості білка. Це означає, що у клітинах печінки порушена функція таких органел:

Мітохондрії

Комплекс Гольджі

Лізосоми

Цитоскелет

Гранулярна ендоплазматична сітка

3838 / 4299
Пацієнт помер у реанімаційному відділенні внаслідок синдрому поліорганної дисфункції. Із анамнезу відомо, що його оперували через гострий гнійний періостит. Під час гістологічного дослідження некроптатів виявлено: гіперплазію лімфоїдної тканини мигдаликів, дифузну інфільтрацію нейтрофілами некротично зміненого альвеолярного відростка щелепи, регіонарний гнійний лімфаденіт, флегмону м'яких тканин шиї, двобічну полісегментарну гнійну пневмонію, спленомегалію, незворотні зміни у кардіоміоцитах та епітелії ниркових канальців. Під час бактеріологічного дослідження у крові померлого виявлено золотистий стафілокок. Який вид сепсису, імовірно, спостерігається у пацієнта?

Криптогенний

Тонзилогенний

Терапевтичний

Одонтогенний

3839 / 4299
У хворого на туберкульоз, що перебуває на лікуванні, погіршився слух. Який препарат викликав таке ускладнення?

Етіонамід

Рифампіцин

Стрептоміцин

Ізоніазид

3840 / 4299
Для полоскання ротової порожнини пацієнту призначили лікарський засіб із групи нітрофуранів у таблетках для зовнішнього застосування. Який із нижченаведених препаратів був призначений хворому?

Фурагін

Фурацилін

Фуросемід

Фуразолідон

Фурадонін

3841 / 4299
При розтині тіла 7-річної дитини, що померла внаслідок декомпенсації вродженої вади серця, виявлено збільшення маси та об'єму тимусу. Мікроскопічним дослідженням виявлено нормальну будову тимусу. Який патологічний процес має місце в тимусі?

Вроджена тимомегалія

Агенезія тимусу

Тимома

Акцидентальна інволюція

3842 / 4299
До вірусологічної лабораторії інфекційної лікарні звернувся хворий з метою пройти обстеження на ВІЛ-інфекцію. Які з методів лабораторної діагностики ВІЛ-інфекції та СНІДу використовуються наразі в Україні?

Бактеріологічний

Серологічний

Біологічний

Алергічний

Вірусологічний

3843 / 4299
У хворого, який страждає протягом багатьох років на остеомієліт нижньої щелепи, з'явилися набряки, у сечі виявлено масивну протеїнурію, у крові -- гіперліпідемію. Що з наведеного, найімовірніше, можна запідозрити у хворого?

Нефротичний синдром

Нефрит

Пієлонефрит

Хронічна ниркова недостатність

3844 / 4299
У пацієнта з механічною жовтяницею та яскравим проявом токсико-алергічних реакцій під час додаткового обстеження у фекаліях виявлено дрібні яйця товщиною 26-30 мкм, асиметричні, мають кришечку та невеликий горбок на протилежних кінцях. Із анамнезу життя відомо, що пацієнт протягом 20 років часто вживає недостатньо термічно оброблену рибу. Якому захворюванню відповідають такі патологічні зміни?

Дікроцеліозу

Опісторхозу

Нанофієтозу

Парагонімозу

3845 / 4299
Під час дорожньо-транспортної пригоди водій отримав численні пошкодження бічної поверхні голови, серед яких був перелом виличної дуги. Функція якого м'яза страждатиме?

M. orbicularis oris

M. buccinator

M. procerus

M. masseter

3846 / 4299
Клітини чутливих спинномозкових нервових вузлів входять до складу рефлекторних дуг. До якого типу нейронів вони відносяться?

Біполярні

Псевдоуніполярні

Уніполярні

3847 / 4299
Хворому треба призначити хіміотерапевтичний засіб широкого спектра дії з групи фторхінолонів. Укажіть цей препарат:

Амоксицилін

Азлоцилін

Хіноксидин

Карбеніцилін

Ципрофлоксацин

3848 / 4299
У новонародженої дитини щелепи добре розвинені та в кожній є зачатки молочних і постійних зубів. Скільки зачатків зубів у новонародженого в одній щелепі?

20 молочних

10 молочних та 16 постійних

10 молочних та 8 постійних

20 молочних та 10 постійних

3849 / 4299
На препараті мазку червоного кісткового мозку людини серед клітин мієлоїдного ряду та адипоцитів зустрічаються клітини зірчастої форми з оксифільною цитоплазмою, які контактують своїми відростками. Які це клітини?

Ретикулярні

Макрофаги

Остеоцити

Дендритні клітини

3850 / 4299
У людини розвинувся гігантизм. Функція якої залози внутрішньої секреції порушена?

Гіпофізу

Вилочкової

Епіфізу

Щитовидної

Надниркової

3851 / 4299
Для покращення мінералізації зубів лікарі-стоматологи призначають препарати Ca^2+. На які процеси він textbfНЕ ВПЛИВАЄ в організмі?

Розвиток деполяризації в міокарді

М'язове скорочення

Створення онкотичного тиску

Проведення збудження через синапс

Гемостаз

3852 / 4299
Пацієнту віком 65 років оперативно видалено ділянку слизової оболонки нижньої поверхні язика з великою бляшкою сіро-білого кольору з чіткими контурами, із шорсткою поверхнею, яка не знімалася при зішкрібанні. Із анамнезу відомо, що пацієнт заядлий курець. Мікроскопічно спостерігається: гіперплазія, гіперкератоз, паракератоз, акантоз багатошарового епітелію, лімфо-плазмоцитарна інфільтрація, фіброз власної пластинки слизової оболонки. Укажіть патологію язика.

Хронічний кандидоз

Червоний плоский лишай

Лейкоплакія

Червоний вовчак

Кератоакантома

3853 / 4299
В сечі хворого знайдені кетонові тіла. При якому захворюванні в сечі з'являються кетонові тіла?

Сечокам'яна хвороба

Туберкульоз нирки

Гострий гломерулонефрит

Цукровий діабет

Інфаркт нирки

3854 / 4299
У хворого під час обстеження встановлено подовження I-го тону серця. Цей тон виникає внаслідок:

Відкриття трьохстулкового клапана

Відкриття мітрального клапана

Закриття аортального клапана

Закриття пульмонального клапана

Закриття атріовентрикулярних клапанів

3855 / 4299
У хворого виявлено порушення прохідності дихальних шляхів на рівні дрібних і середніх бронхів. Які зміни кислотно-основної рівноваги можуть розвинутись у пацієнта?

Респіраторний алкалоз

Метаболічний ацидоз

КОР не зміниться

Метаболічний алкалоз

Респіраторний ацидоз

3856 / 4299
Важливим ферментом слини є лужна фосфатаза. До якого класу ферментів вона належить?

Трансфераз

Ліаз

Оксидоредуктаз

Гідролаз

Лігаз

3857 / 4299
5-місячній дитині була призначена антибактеріальна терапія з приводу бронхопневмонії. Який засіб негативно впливає на розвиток зубів?

Пеніцилін

Бісептол

Доксициклін

Нітроксолін

Левоміцетин

3858 / 4299
При вивченні гістологічного препарату слизової оболонки ротової порожнини було виявлено, що багатошаровий плоский незроговілий епітелій інфільтрований лімфоцитами. Слизова оболонка якої ділянки ротової порожнини найбільш імовірно представлена на препараті?

Ясна

Тверде піднебіння

Мигдалик

Щока

Губа

3859 / 4299
Для патогенетичного лікування невралгій, серед інших і альвеолярних нервів, використовують ненаркотичні анальгетики. Визначте найбільш імовірний механізм їх дії.

---

Взаємодія з опіатними рецепторами

Інактивація циклооксигенази та пригнічення синтезу простагландинів

Інактивація ліпоксигенази та пригнічення синтезу лейкотрієнів

Антибрадикінінова активність

3860 / 4299
Чоловіку видалили зуб. Коронка зуба долотоподібна, широка, край вузький. Корінь конусоподібний, стиснутий з боків. Який зуб видалили?

Нижній різець

Верхній різець

Нижнє ікло

Нижній премоляр

Верхній премоляр

3861 / 4299
Під час експерименту повністю інгібовано розвиток клітин мезенхіми. Порушення розвитку якої м'язової тканини спостерігатиметься?

Скелетної м'язової

М'язової епідермального походження

М'язової нейрального походження

Серцевої м'язової

Гладкої м'язової

3862 / 4299
У хворого 49-ти років виявлено: набряк обличчя, значна протеїнурія, гіпопротеїнемія, диспротеїнемія, гіперліпідемія. Який попередній діагноз?

Простатит

Сечокам'яна хвороба

Пієлонефрит

Нефротичний синдром

Цистит

3863 / 4299
При огляді 1-місячної дитини лікар виявив не закрите заднє тім'ячко. В якому віці воно закривається при нормальному розвитку дитини?

На 6-му місяці життя

На 4-му місяці життя

На 2-3-му місяці життя

На 5-му місяці життя

На 2-му році життя

3864 / 4299
Детоксикація білірубіну відбувається в мембранах ендоплазматичного ретикулума гепатоцитів. Основна частина білірубіну секретується гепатоцитами в жовч у формі:

Непрямого

Вільного

Моноглюкуроніда

Некон'югованого

Диглюкуроніда

3865 / 4299
У хворої дитини гінгівіт, спричинений анаеробною інфекцією. Яку групу протимікробних засобів потрібно призначити для лікування?

Нітрофурани

Поліміксини

Аміноглікозиди

Сульфаніламіди

Нітроімідазоли

3866 / 4299
Під час планового огляду дворічної дитини лікар виявив незакрите переднє тім'ячко. У який період життя дитини відбувається його закриття?

В 1-2 місяці

У 6-9 місяців

На першому році

На другому році

У 3 місяці

3867 / 4299
У жінки 60-ти років з цирозом печінки виник геморагічний синдром. Який механізм це зумовив?

Зменшення синтезу протромбіну і фібриногену

Зниження онкотичного тиску в крові

Зменшення запасів глікогену в печінці

Поява у крові нейротоксичних речовин

Підвищення тиску в системі ворітної вени

3868 / 4299
Чоловік віком 35 років скаржиться на біль у ділянці печінки. З'ясовано, що хворий захоплюється риболовлею і часто вживає недосмажену на вогнищі рибу. У фекаліях виявлені дуже маленькі яйця гельмінту, темного кольору, з кришечкою овальної форми. Який вид гельмінтозу найімовірнішийу цьому разі?

Парагонімоз

Опісторхоз

Дикроцеліоз

Фасціольоз

Шистосомоз

3869 / 4299
До лікарні звернулися пацієнти зі скаргами на слабкість, біль у кишечнику, розлади травлення. Після дослідження фекалій виявлено цисти з чотирма ядрами. Для якого найпростішого характерні такі цисти?

Лямблія

Амеба дизентерійна

Балантидій

Амеба кишкова

Амеба ротова

3870 / 4299
У пацієнта із порушенням слуху відбулося скостеніння стремінцевої зв'язки. Укажіть вид такого з'єднання.

Геміартроз

Синдесмоз

Вклинення

Синхондроз

Синостоз

3871 / 4299
Чоловік 42-х років звернувся до медпункту з приводу різаної рани нижньої частини передньої поверхні плеча. Об'єктивно: утруднене згинання передпліччя. Які з наведених м'язів ймовірно ушкоджені у хворого?

M. brachialis, m. biceps brachii

M. biceps brachii, m. anconeus

M. coracobrachialis, m. supraspinatus

M. deltoideus, m. infraspinatus

M. deltoideus, m. biceps brachii

3872 / 4299
Під час профілактичного огляду в одного працівника їдальні у фекаліях виявлено восьмиядерні цисти. Кому з найпростіших вони належать?

Лямблії

Балантидію

Амебі кишковій

Токсоплазмі

Трихомонаді кишковій

3873 / 4299
Після тотальної резекції шлунка у хворого розвинулася тяжка B_12-дефіцитна анемія з порушенням кровотворення і появою в крові змінених еритроцитів. Свідченням її була наявність в крові:

Овалоцитів

Нормоцитів

Мікроцитів

Анулоцитів

Мегалоцитів

3874 / 4299
Пацієнту віком 50 років встановлено діагноз: мікседема. Порушення утворення яких гормонів спричиняє розвиток цієї патології?

Тироксину і трийодтироніну

АКТГ і СТГ

Кортизолу і альдостерону

Окситоцину і вазопресину

Інсуліну і глюкагону

3875 / 4299
До якої групи молекулярних хвороб обміну речовин належить фенілкетонурія?

Спадкові хвороби обміну сполучної тканини

Порушення метаболізму вуглеводів

Обміну мінеральних речовин

Порушення метаболізму амінокислот

Спадкові хвороби обміну ліпідів

3876 / 4299
Пацієнту, який скаржиться на сильний зубний біль, призначено ненаркотичний анальгетик (похідне аніліну), у якого слабкий протизапальний ефект, виражені анальгезуюча та жарознижувальна дії. Укажіть цей препарат.

Ацетилсаліцилова кислота

Бутадіон

Анальгін

Ібупрофен

Парацетамол

3877 / 4299
У дівчинки 6-ти років виражені ознаки гемолітичної анемії. При біохімічному аналізі еритроцитів встановлено дефіцит ферменту глюкозо-6-фосфатдегідрогенази. Порушення якого метаболічного процесу відіграє головну роль у розвитку цієї патології?

Пентозофосфатний шлях

Глюконеогенез

Окисне фосфорилювання

Тканинне дихання

Анаеробний гліколіз

3878 / 4299
Реакція Вассермана у хворого 30-ти років різко позитивна (++++). Для діагностики якого інфекційного захворювання використовується реакція Вассермана?

Туберкульоз

Бруцельоз

Грип

Поліомієліт

Сифіліс

3879 / 4299
У пацієнта віком 50 років раптово з’явилися скарги на головний біль, запаморочення голови та нудоту. АТ 220/110 мм рт.ст. Під час введення 0,1% розчину гігронію в/в крапельно самопочуття пацієнта покращилося. Який механізм дії препарата?

Блокада beta_1-адренорецепторів

Активація alpha_2-адренорецепторів

Блокада АПФ

Блокада N-холінорeцепторів гангліїв

Блокада каналів Са^++

3880 / 4299
Під час мікроскопічного дослідження біопсії нирки виявлено вогнища, у центрі яких знаходяться зернисті еозинофільні маси, оточені інфільтратом із лімфоцитів, епітеліоїдних клітин і поодиноких клітин Пирогова-Лангханса. Укажіть патологічний процес, що найбільше відповідає вищенаведеним змінам.

Проліферація та диференціювання макрофагів

Альтеративне запалення

Казеозний некроз

Коагуляційний некроз

Гранулематозне запалення

3881 / 4299
При обстеженні у хворої виявлений заглотковий абсцес. В який простір шиї повинен проникнути хірург при розтині абсцесу?

Переддрабинчастий

Передвісцеральний

Надгруднинний

Міждрабинчастий

Позавісцеральний

3882 / 4299
Укажіть паразита, проміжним хазяїном якого є молюск.

Печінковий сисун

Трихінела

Ехінокок

Стьожак широкий

Лямблія

3883 / 4299
У підтриманні певної пози задіяні повільні скелетні м'язи, які мають значні резерви кисню. Яка речовина із нижченаведених бере участь в запасанні кисню у цих м'язах?

Міоглобін

Гемоглобін

Цитохром

Креатинфосфат

Кальмодулін

3884 / 4299
Укажіть препарат, що належить до групи beta-лактамних антибіотиків.

Бісептол

Еритроміцин

Тетрациклін

Бензилпеніцилін

Офлоксацин

3885 / 4299
Тривале вживання великих доз аспірину викликає пригнічення синтезу простагландинів через зниження активності ферменту:

Фосфодіестерази

5-ліпоксигенази

Циклооксигенази

Фосфоліпази А2

Пероксидази

3886 / 4299
Експериментальне дослідження мембранних іонних струмів у динаміці розвитку потенціалу дії показало, що лавиноподібний вхід іонів натрію до клітини спостерігається у фазі:

Реверсполяризації

Реполяризації

Деполяризації

Деполяризаційного слідового потенціалу

Гіперполяризаційного слідового потенціалу

3887 / 4299
У складі кісткової тканини виявлено великі багатоядерні клітини з відростками, які містять численні лізосоми. Яку назву мають ці клітини?

Напівстовбурові остеогенні клітини

Мезенхімальні клітини

Хондроцити

Остеокласти

Хондробласти

3888 / 4299
На розтині тіла померлого чоловіка 52-х років виявлені зміни в легенях у вигляді сегментарної ділянки казеозного некрозу, які зливаються та займають верхню частину правої легені. Легеня збільшена, щільна, на розрізі - жовтуватого кольору, на плеврі - фібринозні плівки. Назвіть форму туберкульозу:

Туберкульома

Казеозна пневмонія

Циротичний туберкульоз

Інфільтративний туберкульоз

Гострий кавернозний туберкульоз

3889 / 4299
При аускультації серця було встановлено роздвоєння І тону на основі мечоподібного відростка грудини. Патологію якого клапана вислухав лікар?

Клапана легеневого стовбура

Клапана верхньої порожнистої вени

Двостулкового

Тристулкового

Клапана аорти

3890 / 4299
На електронній мікрофотографії зображено фібробласт, що продукує компоненти міжклітинної речовини. Укажіть органели, які беруть участь у цьому процесі.

Комплекс Гольджі та мітохондрії

Комплекс Гольджі та лізосоми

Гранулярна ендоплазматична сітка та комплекс Гольджі

Агранулярна ендоплазматична сітка та комплекс Гольджі

Гранулярна та агранулярна ендоплазматичні сітки

3891 / 4299
Чоловік чекає своєї черги на прийом до лікаря-стоматолога. Під час очікування від хвилювання він відчув прискорення серцебиття. Вкажіть частоту серцевих скорочень у здорової дорослої людини за нормальних умов:

40-60/хв.

110-120/хв.

60-80/хв.

150-160/хв.

90-110/хв.

3892 / 4299
Для підвищення мінералізації зубів в процесі лікування карієсу застосовуються речовини, що є джерелом надходження мінералів у тверді тканини зубів. Які це речовини?

Гліцерофосфат кальцію

Сульфат міді

Сульфат магнію

Сульфат калію

Хлорид натрію

3893 / 4299
У гістологічному препараті декальцінованої нижньої щелепи, навколо кореня зуба визначаються пучки товстих колагенових волокон, між якими знаходиться пухка волокниста сполучна тканина із кровоносними судинами. Назвіть цю структуру.

Періодонт

Клітинний цемент

Ясна

Дентин

Зубна альвеола

3894 / 4299
Під час огляду плода виявлено розщеплення твердого піднебіння. Встановіть імовірний діагноз вродженої вади обличчя.

Гіпертелоризм

Мікрогнатія

Палатосхізис

Макростомія

Хейлосхізис

3895 / 4299
У психіатричній клиніці використовують нейролептик трифтазин. Який із нижченаведених механизмів антипсихотичної дії властивий цьому засобу?

Стимуляція ГАМК – енергічних рецепторів головного мозку

Модуляція центральних бензодіазепінових рецепторів

Гальмування пуринових рецепторів

Пригнічення дофамінових D2- рецепторів

Збудження опіатних рцепторів

3896 / 4299
На шостому місяці вагітності в жінки з'явилася виражена залізодефіцитна анемія. Її діагностичною ознакою була поява в крові:

Нормоцитів

Анулоцитів

Ретикулоцитів

Пойкілоцитів

Макроцитів

3897 / 4299
У більшості клітин епітелію слизової оболонки ротової порожнини чоловіка виявлено одне тільце статевого Х-хроматину. Це характерно для синдрому:

Трипло-Х

Клайнфельтера

Дауна

Трипло-Y

Шерешевського-Тернера

3898 / 4299
До стоматолога звернувся пацієнт зі скаргами на біль, почервоніння, припухлість ясен. Попередній діагноз: герпетичний гінгівостоматит. Який вірус може викликати це захворювання?

Вірус цитомегалії

Вірус простого герпесу, тип 1

Вірус простого герпесу, тип 2

Вірус оперізувального лишаю

Вірус Епштейна-Барр

3899 / 4299
У хворого 35-ти років, який тривалий час страждав на бронхіальну астму, розвинувся астматичний статус з летальним наслідком. При дослідженні секційного матеріалу в легенях виявлено спазм бронхіол, в їх стінках - виражену клітинну інфільтрацію з переважанням еозинофільних лейкоцитів та лімфоцитів, лаброцити з явищами дегрануляції. Який механізм гіперчутливості обумовив зазначені зміни?

Імунокомплексна

Реагінова реакція

Антитілозалежна

Клітинна цитотоксичність

3900 / 4299
Під час очікування екстракції зуба у пацієнта розвинувся напад бронхіальної астми. Препарат якої фармакологічної групи йому треба призначити для усунення бронхоспазму?

Анальгетики

beta_2-адреноміметики

Психотоніки

Аналептики

М-холіноміметики

3901 / 4299
В експерименті на тварині після перерізки блукаючих нервів спостерігають постійну тахікардію. Який вплив парасимпатичної нервової системи на роботу серця демонструє цей експеримент?

Парадоксальний

Гальмівний

Збуджувальний

Змішаний вплив

Сумація збуджень

3902 / 4299
У людини виявлена пухлина одного з відділів головного мозку, внаслідок чого в неї порушена здатність підтримувати нормальну температуру тіла. Яка структура головного мозку пошкоджена?

Гіпоталамус

Чорна субстанція

Таламус

Мозочок

Стріатум

3903 / 4299
Жінка скаржиться на біль у лівій половині нижньої щелепи та нижніх зубів. Ураження яких нервів є найімовірнішою причиною таких симптомів?

VІІ пари черепних нервів

Другої гілки V пари черепних нервів

Рухових волокон третьої гілки V пари черепних нервів

Чутливих волокон третьої гілки V пари черепних нервів

Першої гілки V пари черепних нервів

3904 / 4299
Під час стану емоційного напруження у людини з'явились поперечні зморшки на лобі посередині. Скорочення якого м'яза відбулося?

M. temporoparietalis

M. corrugator supercilii

M. auricularis anterior

M. procerus

M. occipitofrontalis

3905 / 4299
Під час проведеня апендектомії, пацієнту перев'язали a. appendicularis. Гілкою якої артерії є ця судина?

A) mesenterica inferior

A) colica dextra

A) colica media

A) ileocolica

A) sigmoidea

3906 / 4299
У хворого 14-ти років спостерігається порушення сутінкового зору. Якого вітаміну недостатньо в його організмі?

B_12

B_6

C

A

B_1

3907 / 4299
В клініку потрапив хворий з ознаками спазматичної кишкової непрохідності. При наданні медичної допомоги з кишечника хворого виділено гельмінтів, які належать до класу круглих червів, розмірами 25-40 см. Визначте вид гельмінту:

Аскарида людська

Вугриця кишкова

Волосоголовець людський

Гострик

Кривоголовка дванадцятипала

3908 / 4299
Пацієнт віком 42 роки захворів за тиждень після того, як обробляв шкуру лисиці. Захворювання проявилося нервовим збудженням, гідрофобією, судомами. Під час аутопсії тіла чоловіка виявлено: енцефаліт із ураженням стовбурової частини головного мозку, стінок ІІІ шлуночка та гіпокампа. Проявами енцефаліту були скупчення лімфоцитів та мікрогліальних клітин довкола загиблих нейронів і судин. У нервових клітинах гіпокампа виявлено еозинофільні включення - тільця Бабеша-Негрі. Яке захворювання діагностовано в померлого?

Туляремію

Сибірку

Бруцельоз

Чуму

Сказ

3909 / 4299
Під час судинно-тромбоцитарного гемостазу із тромбоцитів, що зруйнувалися, вивільняється пластинчатий фактор (ПФ-8) тромбостенін. Яку функцію він виконує?

Гемоліз еритроцитів

Аглютинацію еритроцитів

Адгезію тромбоцитів

Ретракцію тромбу

Агрегацію тромбоцитів

3910 / 4299
У людини зуби закріплені в особливих комірках верхньої та нижньої щелеп. До якої системи відносять зуби людини?

Гетеродонтної

Гомодонтної

Текодонтної

Плевродонтної

Акродонтної

3911 / 4299
В експерименті при вивченні процесів всмоктування продуктів гідролізу їжі і води було встановлено, що основним відділом шлунково-кишкового тракту, де відбуваються ці процеси, є:

Шлунок

Ротова порожнина

Пряма кишка

Тонка кишка

Товста кишка

3912 / 4299
Під час експерименту досліджується поріг сили подразнення клітин різних тканин. Де він виявиться найменшим?

У залозистих клітинах

У міоцитах скелетних м'язів

У мотонейронах спинного мозку

У кардіоміоцитах

У міоцитах гладких м'язів

3913 / 4299
У жінки зі зростом 1,70 м і вагою тіла 94 кг виявлено зменшення толерантності до вуглеводів. Нестача якого гормону може бути причиною цього стану?

Кортизолу

Соматотропіну

Адреналіну

Інсуліну

Глюкагону

3914 / 4299
Після прибуття до Заполяр'я дослідники з Австралії протягом 6-ти місяців висловлювали скарги на неврози, втрату апетиту, загострення хронічних захворювань. Який процес порушився в екстремальних умовах?

Тахіфілаксія

Адаптація

Толерантність

Стрес

Репарація

3915 / 4299
Стан зубів залежить від надходження в організм фтору, зокрема з водою. Яка гігієнічна норма вмісту фтору в 1 л питної води?

9,0 мг

3,0 мг

6,0 мг

12,0 мг

1,5 мг

3916 / 4299
Пацієнтці хірургічним шляхом видалено пухлину нижньої щелепи, яка мала вигляд відокремленого щільного вузла. На розрізі виявлено: пухлина червоного кольору, з білими плямами та дрібними кістами. Під час її гістологічного дослідження встановлено: пухлина складається з велетенських багатоядерних та дрібних одноядерних клітин, між якими утворюються кісткові балочки. Для якої пухлини характерні такі клініко-лабораторні показники?

Остеома

Остеобластокластома

Остеосаркома

Амелобластома

Остеоїд-остеома

3917 / 4299
Синтез глюкози з невуглеводних компонентів є важливим біохімічним процесом. Глюконеогенез з амінокислот найбільш активно відбувається за умов білкового харчування. Яка амінокислота з наведених є найбільш глюкогенною?

Лейцин

Валін

Ізолейцин

Лізин

Аланін

3918 / 4299
Тирозин використовується в якості субстрату в процесі синтезу тироксину. Вкажіть хімічний елемент, який бере участь в цьому процесі:

Мідь

Кальцій

Йод

Залізо

Цинк

3919 / 4299
Пацієнтку шпиталізовано зі скаргами на сухість у роті, спрагу та зниження маси тіла. Під час обстеження виявлено глюкозурію. Рівень глюкози у крові - 8,7 ммоль/л. Для якого патологічного стану характерні такі симптоми?

Аліментарної глюкозурії

Цукрового діабету

Стероїдного діабету

Нецукрового діабету

Ниркового діабету

3920 / 4299
Пацієнта віком 38 років госпіталізовано з ознаками гіпоксії, що розвилася після отруєння чадним газом. Стан середньої тяжкості, тахікардія, задишка, АТ --- 160/100 мм рт.ст. Який механізм токсичної дії окису вуглецю на організм людини?

Блокада кальцієвих каналів еритроцитів

Порушення дисоціації оксигемоглобіну

Утворення метгемоглобіну

Утворення карбгемоглобіну

Утворення карбоксигемоглобіну

3921 / 4299
Досліджуючи біоптат щитовидної залози виявлено: інфільтрація лімфоцитами тканини щитовидної залози, зруйнування паренхіматозних елементів. У стромі спостерігається дифузна лімфоцитарна інфильтрація з наявністю лімфоїдних фолікулів. Для якого захворювання характерні такі патологічні зміни?

Недиференційованого раку щитовидної залози

Солідної аденоми щитовидної залози

Аутоімунного тиреоїдиту (Гашімото)

Базедової хвороби

Папілярного раку щитовидної залози

3922 / 4299
Лікар-стоматолог виконує анестезію у ділянці другого верхнього моляра. Які нерви знеболює лікар у цьому разі?

Rr.alveolares inferiores posteriores

Rr.alveolares inferiores anteriores

Rr.alveolares superiores medii

Rr.alveolares superiores posteriores

Rr.alveolares superiores anteriores

3923 / 4299
З метою схуднення жінка обмежувала кількість продуктів у харчовому раціоні. За 3 місяці в неї з'явилися набряки, збільшився діурез. Дефіцит яких компонентів їжі є причиною цього?

Вітамінів

Вуглеводів

Жирів

Мінеральних речовин

Білків

3924 / 4299
У хворого на гострий панкреатит у сечі суттєво підвищено вміст діастази. Який засіб з групи інгібіторів протеолізу необхідно включити до складу комплексної терапії цього хворого?

Фестал

Контрикал

Дигестал

Мезим форте

Панкреатин

3925 / 4299
Скорочення поперечно-посмугованих м'язів неможливе без кальцію. Із якими рецепторами з'єднується цей іон в утворенні актиноміозинових містків?

Адренорецепторами

Холінорецептором

Серотоніновими рецепторами

Тропоніном

Гістаміновими рецепторами

3926 / 4299
До лікарні надійшла дитина з діагнозом дифтерія. Якими препаратами для специфічної терапії Ви скористуєтесь?

Дифтерійні вакцини: АКДП, АДП, АД

Вакцина ''Кодивак'', сульфаніламіди

Протидифтерійна антитоксична сироватка, антибіотики

Дифтерійний бактеріофаг

Дифтерійний анатоксин, антибіотики

3927 / 4299
У пацієнта віком 65 років на тлі портальної гіпертензії крізь портокавальні анастомози у загальний кровотік надходять речовини, що знешкоджуються в печінці. Який різновид печінкової коми розвинеться у цьому разі?

Паренхіматозна

Кетоацидотична

Змішана

Печінково-клітинна

Шунтова

3928 / 4299
Після проникнення в організм бактерії фагоцитуються макрофагами. Яку роль відіграють макрофаги в кооперації імунокомпетентних клітин на першому етапі формування імунної відповіді?

Забезпечують процесинг і презентацію антигена Т-хелперам

Активують Т-кілери

Забезпечують процесинг і презентацію антигена Т-кілерам

Продукують імуноглобуліни

Активують NК-клітини

3929 / 4299
Чоловік 43-х років звернувся до лікаря зі скаргами на раптове почервоніння, набряк шкіри з появою пухирів, свербіж, які з'явилися після вживання креветок. Який тип гіперчутливості характеризується такими місцевими проявами?

Місцевий прояв гіперчутливості I типу

Реакція гіперчутливості III типу

Місцевий прояв гіперчутливості II типу

Місцевий прояв гіперчутливості IV типу

3930 / 4299
У хворого, який помер за 3 тижні після початку пневмонії, нижня частка правої легені різко збільшена, щільна, безповітряна, сіра, з відкладеннями фібрину на плеврі. Під час мікроскопії у всіх альвеолах цієї частки виявлено фібрин та сегментоядерні лейкоцити. Поставте діагноз:

Грипозна пневмонія

Вогнищева бронхопневмонія

Фібринозний плеврит

Інтерстиціальна пневмонія

Крупозна пневмонія

3931 / 4299
Під час обстеження ротової порожнини встановлено пошкодження емалі у вигляді поодиноких та множинних ерозій різної форми від жовто-коричневого до чорного кольору; зуби крихкі, частина з них зруйнована. Якому захворюванню відповідають такі патологічні зміни?

Флюорозу

Ерозії зубів

Атрофії емалі

Середньому карієсу

Глибокому карієсу

3932 / 4299
У обстежуваного реєструється дельта та тета-ритм на ЕЕГ. Це свідчить про те, що він перебуває у стані:

Швидкохвильового сну

Спокою з розплющеними очима

Активного неспання

Повільнохвильового сну

Спокою з заплющеними очима

3933 / 4299
Генетичні дефекти окремих ферментів біосинтезу сечовини призводять до накопичення у крові та тканинах вільного аміаку. Який орган є найчутливішим до гіперамоніємії?

Нирки

Кишківник

Серце

Печінка

Головний мозок

3934 / 4299
Під час гістологічного дослідження внутрішніх органів померлої жінки, якій за життя встановили діагноз: системний колагеноз, було виявлено поширене ураження судин у вигляді мукоїдного і фібриноїдного набрякання, фібриноїдного некрозу стінок артеріол, периваскулярних лімфо-плазмоцитарних інфільтратів. Для якого виду запалення характерні такі симптоми?

Інтерстиціального дифузного запалення

Хронічного імунного запалення

Гранулематозного запалення

---

Гострого імунного запалення

3935 / 4299
В одній з оболонок порожнистого органа визначаються ядровмісні анастомозуючі волокна. Волокна складаються з клітин, які у ділянці контактів утворюють вставочні диски. Яка тканина утворює дану оболонку?

Поперечно-смугаста скелетна

Гладенька м'язова

Щільна неоформлена сполучна

Поперечно-смугаста серцева

Пухка волокниста сполучна

3936 / 4299
У хворого зі скаргами на полідипсію, поліфагію та поліурію визначили наявність глюкози в сечі. Яке захворювання можна припустити?

Хвороба Аддісона

Інсулінома

Нецукровий діабет

Акромегалія

Цукровий діабет

3937 / 4299
Під час мікроскопічного дослідження видаленого зуба виявляється зменшення кількості й розмірів одонтобластів та інших клітин пульпи зуба зі своєрідним склерозом сполучнотканинної основи пульпи. Який загальнопатологічний процес можна припустити в пульпі зуба?

Амілоїдоз

Сітчаста атрофія пульпи

Гіперплазія пульпи

Жирова дистрофія

Гіаліноз

3938 / 4299
Пацієнт віком 59 років мав діагноз трансмуральний інфаркт міокарда лівого шлуночка. Помер від істинного розриву серця --- тампонади серця. Який процес у зоні інфаркту міг сприяти розриву серця?

---

Заміщення сполучною тканиною зони інфаркту (організація)

Аутолітичні процеси розплавлення тканини міокарда (міомаляція)

Формування рубця зі стоншенням стінки лівого шлуночка

Підвищення тиску в малому колі кровообігу

3939 / 4299
При електронній мікроскопії нирки виявлені канальці, які вистелені кубічним епітелієм. В епітелії розрізняють світлі та темні клітини. В світлих клітинах мало органел. Цитоплазма утворює складки. Ці клітини забезпечують реабсорбцію води з первинної сечі у кров. Темні клітини за будовою і функцією нагадують парієтальні клітини шлунка. Які канальці представлені у мікроскопічному препараті?

Висхідні канальці петлі Генле

Збірні ниркові трубочки

Проксимальні канальці

Дистальні канальці

Низхідні канальці петлі Генле

3940 / 4299
Хворий 38-ми років з хронічним алкоголізмом помер від прогресуючої серцевої недостатності. На розтині: лобарна плевропневмонія нижньої часточки правої легені. При гістологічному дослідженні в альвеолах фібринозний ексудат та сегментоядерні лейкоцити. Визначте стадію крупозної пневмонії:

Червоної гепатизації

Сірої гепатизації

Завершення

Припливу

3941 / 4299
У чоловіка 25-ти років на прийомі у стоматолога через кілька хвилин після промивання рота розчином фурациліну виник значний набряк на губах. Укажіть, який тип алергічної реакції спостерігався у цьому випадку:

Цитолітичний

Гіперчутливість сповільненого типу

Стимулюючий

Анафілактичний

Імунокомплексний

3942 / 4299
Послідовність триплетів у ДНК визначає порядок розташування амінокислот у молекулі білка. Укажіть назву цієї властивості генетичного коду.

Триплетність

Колінеарність

Універсальність

Виродженість

Неперекривність

3943 / 4299
Відомо, що в метаболізмі катехоламінових медіаторів особлива роль належить ферменту моноаміноксидазі (МАО). Яким шляхом цей фермент інактивує медіатори (норадреналін, адреналін, дофамін)?

Карбоксилювання

Видалення метильної групи

Гідроліз

Окисне дезамінування

Приєднання аміногрупи

3944 / 4299
У чоловіка 25-ти років після ушкодження периферичних нервів втратились всі види чутливості. Як називається цей вид порушення?

Атаксія

Гіперстезія

Анестезія

Гіпостезія

3945 / 4299
У пацієнта віком 68 років, який переніс інфаркт міокарда, під час ЕКГ-обстеження відмічається прогресуюче збільшення інтервалу PQ з випадінням комплексу QRS, після чого інтервал PQ відновлюється. Із порушенням якої функції серця пов'язане це порушення серцевого ритму?

Проводимості

Автоматизму

Скоротливості

---

Збудливості

3946 / 4299
У гістологічному препараті кіркової речовини нирок можна бачити ниркові тільця та канальці нефронів. Відомо, що в канальцях нефрону відбувається реабсорбція речовин. Яка тканина нефрону бере участь у цьому процесі?

Хрящова

Ретикулярна

Слизова

Власне сполучна

Епітеліальна

3947 / 4299
На мiкропрепаратi зародка людини, взятого з мимовiльного викидня, виявлено зародковий щиток, в якому розпiзнаються два шари клiтин: енто- i ектодерма. На якому етапi ембрiонального розвитку був ембрiон?

Гаструляцiї

Нейруляції

Гістогенезу

Прогенезу

Органогенезу

3948 / 4299
В умовному експерименті дія токсичної речовини порушує механізм передавання нервового імпульсу. Яка структура забезпечує виконання цієї функції?

Мітохондрія

Нейрофібрили

Синапс

Нейролема

Субстанція Ніссля

3949 / 4299
На рибосомі відбувається трансляція. Рибосома дійшла до кодону УАА. Цей кодон у процесі біосинтезу поліпептиду не розпізнається жодною т-РНК, тому синтез поліпептидного ланцюга закінчився. Сигналом якого процесу є кодон УАА?

Ініціації

Термінації

Посттрансляційної модифікації

3950 / 4299
Пацієнту, який хворіє на лейкоз, призначено 5-фторурацил. На який процес впливає цей препарат?

Прискорює реплікацію

Інгібує трансляцію

Інгібує синтез ДНК

Інгібує транскрипцію

3951 / 4299
Під час огляду порожнини рота лікар-стоматолог виявив наявність запалення тканин, які оточують зуб. Запалення якого анатомічного утворення виявив лікар?

Gingiva

Paradontium

Cementum

Alveola dentalis

3952 / 4299
Пацієнту віком 25 років для уточнення діагнозу треба зробити сіалографію привушної залози. Де розташований отвір, через який буде введена рентгеноконтрастна маса?

На plica sublingualis

На щоці в ділянці навпроти 2-го нижнього великого кутнього зуба

На щоці в ділянці навпроти 2-го верхнього великого кутнього зуба

На щоці в ділянці навпроти 2-го верхнього малого кутнього зуба

3953 / 4299
Вкажіть, який серцевий глікозид швидкої дії застосовують під час гострої серцевої недостатності.

Валідол

Кордіамін

Аміодарон

Строфантин

3954 / 4299
На розтині в печінці померлого чоловіка 62-х років виявлено вогнище розпаду тканини діаметром 4 см, заповнене рідиною жовтувато-зеленого кольору. Який діагноз найімовірніший?

Абсцес

Флегмона

Емпієма

Карбункул

3955 / 4299
Пацієнт звернувся до лікаря зі скаргами на значну спрагу (полідипсію) та на часте сечовипускання із великою кількістю сечі (поліурію). Із анамнезу відомо, що 4 тижні тому внаслідок черепно-мозкової травми у пацієнта виявлено некроз задньої долі гіпофіза. Для якого захворювання характерні такі симтоми?

Цукрового діабету

Хвороби Іценко-Кушинга

Акромегалії

Нецукрового діабету

3956 / 4299
У хворого на цукровий діабет після застосування цукрознижувальної терапії виник гіпоглікемічний стан. Надмірне вживання якого гормону могло призвести до цього стану?

Глюкагону

Інсуліну

Тироксину

Кортизолу

3957 / 4299
У процесі біосинтезу білка, що відбувається в еукаріотичній клітині, одним із етапів є перетворення про-іРНК в іРНК. Унаслідок цього іРНК <<дозріває>>. Як називається цей процес?

Трансдукція

Реплікація

Процесинг

Транскрипція

Репарація

3958 / 4299
Після крововиливу в мозок у пацієнта виникло значне погіршення смакової чутливості. Яка структура мозку, ймовірно, ушкоджена?

Гіпокамп

Гіпоталамус

Чорна субстанція

Мигдалеподібне тіло

Постцентральна звивина

3959 / 4299
У пацієнта через 30 хвилин після лікування у стоматолога з'явилися червоні плями на шкірі обличчя і слизовій рота, що сверблять. Був встановлений діагноз: кропивниця. Яке з біологічно активних речовин, що викликають розширення судин, появу свербежу, виділяється при цьому типі алергічної реакції?

Інтерлейкін-1

Брадикінін

Лейкотрієн В4

Гістамін

Простагландин Е2

3960 / 4299
У пацієнта запалення слизової порожнини рота. Яке з нижченаведених речовин із місцевою антиексудативною дією необхідно застосувати у цьому разі?

Натрія гідрокарбонат

Ксероформ

Галуни

Дерматол

Вісмута субнітрат

3961 / 4299
Пацієнт скаржиться на виділення сечі під час статевого акту. Який орган уражений?

Яєчко

Сім'яні міхурці

Передміхурова залоза

Сечовий міхур

3962 / 4299
У хворої 34 років виразка шлунка. Для того щоб точніше описати розташування виразки, лікар повинен знати, з яких частин складається шлунок. Назвіть їх:

Дно, склепіння шлунка, пілорична частина, воротарна частина, кардіальна частина

Дно шлунка, велика та мала кривизна, кардіальна частина

Тіло, дно шлунка, пілорична та кардіальна частини

Тіло, дно, велика та мала кривизна шлунка

3963 / 4299
У хворого після перенесеної психічної травми постійно проявляється тривожність, занепокоєння, страх. Який препарат із групи похідних бензодіазепіну призначили хворому?

Діазепам

Гепарин

Бісакодил

Метоклопрамід

3964 / 4299
Які нерви треба знеболити пацієнту, щоб видалити верхній третій моляр?

Коміркові верхні задні нерви

Задні верхні носові нерви

Коміркові верхні передні нерви

Великий піднебінний нерв

3965 / 4299
Хворому на миготливу аритмію, в анамнезі у якого бронхіальна астма, треба призначити протиаритмічний засіб. Який препарат з цієї групи textbfПРОТИПОКАЗАНИЙ хворому?

Ніфедипін

Новокаїнамід

Анаприлін

Верапаміл

3966 / 4299
Людям, що мешкають в умовах арктичного клiмату, незалежно вiд їхньої раси, притаманний ряд пристосувань до умов iснування. Для представників арктичного адаптивного типу порiвняно з корiнними жителями Центральної Африки характерним є:

Підвищений шар пiдшкiрного жиру

Підвищення потовидiлення

Худорляве тiло

Бiльша довжина нiг, нiж рук

3967 / 4299
У спортсмена після перевантаження під час тренування виникла м'язова контрактура. При цьому м'яз втрачає гнучкість та поступово стає твердим, бо не має можливості розслабитися. Вкажіть імовірну причину контрактури:

Недостатність АТФ

Зниження Ca^++ у крові

Підвищення молочної кислоти у крові

Збільшення K^+ у крові

3968 / 4299
На розтині померлого після абдомінальної операції у венах малого тазу були знайдені численні тромби. Клінічно був зафіксований тромбоемболічний синдром. Де слід шукати ембол?

Головний мозок

Портальна вена

Лівий шлуночок серця

Легеневі артерії

Вени нижніх кінцівок

3969 / 4299
При ідентифікації чистої культури мікроорганізмів найважливішою є серологічна ідентифікація і для цього використовують реакцію аглютинації. Виберіть, які компоненти необхідні для постановки цієї реакції:

Термоекстракт, специфічна сироватка

Специфічний антиген, сироватка хворого

Невідома культура бактерій, специфічні антитіла

Специфічний антиген, відоме антитіло, бактерії

3970 / 4299
Під час голодування зменшується маса органів і тканин. Укажіть орган, який у першому періоді голодування найбільше втрачає в масі.

Нирки

Серце

М’язи

Печінка

3971 / 4299
Який гормон утворюється в мозковому шарі надниркових залоз?

Глюкагон

Тироксин

Гістамін

Адреналін

3972 / 4299
Лікар виявив у пацієнта запалення слизової оболонки ротової порожнини, яке супроводжувалося нестерпним болем. З ураженням якого нерва це пов'язано?

Лицьового

Трійчастого

Барабанної струни

Блукаючого

3973 / 4299
В препараті видно овоцит в момент запліднення його сперматозоїдом. Де за нормальних умов відбувається цей процес?

В ампульній частині маткової труби

В матці

На поверхні яєчника

В черевній порожнині

3974 / 4299
У хворого на гострий апендицит під час операційного втручання відбулося припинення серцевої діяльності. Назвіть ознаки, характерні для клінічної смерті:

Відсутність дихання, відсутність серцевої діяльності

Апнейстичне дихання, відсутність серцевої діяльності

Часте дихання, послаблення серцевих тонів

Відсутність дихання, ниткоподібний пульс

3975 / 4299
Істотним недоліком мікроскопічного методу діагностики інфекцій є його недостатня інформативність у зв'язку з морфологічною подібністю багатьох видів мікроорганізмів. Яка імунологічна реакція дозволяє значно підвищити інформативність цього методу?

Реакція імунофлюоресценції

Радіоімунний аналіз

Реакція опсонізації

Імуноферментний аналіз

Реакція Кумбса

3976 / 4299
Пацієнтка віком 65 років із патологічними переломами нижньої щелепи упродовж 15-ти років хворіла на хронічний остеомієліт. На фоні погіршення загального стану виявлено: в аналізі крові - гіпопротеїнемія та диспротеїнемія; в аналізі сечі - протеїнурія та білкові циліндри. Смерть настала від хронічної ниркової недостатності. Який найімовірніший патологічний процес спостерігатиметься в нирках під час аутопсії?

Пієлонефрит

Первинний амілоїдоз нирок

Вторинний амілоїдоз нирок

Гідронефроз

3977 / 4299
Після хірургічної операції у тварини розвинулася тетанія як наслідок низького рівня кальцію в плазмі крові. Яка залоза внутрішньої секреції була видалена?

Кора надниркових залоз

Тимус

Щитоподібна залоза

Паращитоподібні залози

3978 / 4299
Однією з екологічних проблем, що викликає тривогу людства, є глобальне потепління. Одною з причин зміни клімату вказують парниковий ефект, який пов'язують із:

Зменшенням вмісту вуглекислого газу в атмосфері

Появою озонових ''дір''

Збільшенням вмісту вуглекислого газу в атмосфері

Збільшенням вмісту оксидів сірки в атмосфері

3979 / 4299
Під час загострення ревматоїдного артриту хворому, в анамнезі якого зазначено супутній хронічний гастрит, призначено целекоксиб. Чим зумовлено зменшення побічної дії препарату на травний тракт?

Переважаючою стимуляцією аденілатциклази

Пригніченням фосфоліпази А2

Переважаючим пригніченням циклооксигенази-2

Пригніченням фосфодіестерази

3980 / 4299
Під час сальпінгоотита спостерігається втягнення барабанної перетинки. Рукоятка однієї зі слухових кісточок, яка пов'язана з membrana tympani, набуває більш горизонтального положення. Положення якої кісточки визначає лікар під час обстеження?

Squama os temporale

Incus

Malleus

Stapes

3981 / 4299
У ротову порожнину з водою та харчовими продуктами можуть потрапляти численні небезпечні для організму речовини. Які компоненти слини та гінгівальної рідини забезпечують захист від цих сполук?

Лужна, кисла фосфатаза

Лізоцим, імуноглобуліни, лейкоцити

Молочна кислота, сечовина, аміак

Лактатдегідрогеназа, глюкуронідаза

3982 / 4299
У пацієнта з діагнозом: ниркова недостатність, виявлено ознаки ниркової остеодистрофії, що супроводжується резорбцією кісткової тканини пародонта. Причиною цього є порушення утворення:

D_3

24, 25(ОН)_2 D_3

1,25(ОН)_2 D_3

25(ОН) D_3

3983 / 4299
У хворого 58 років із гострою серцевою недостатністю спостерігалось зменшення добової кількості сечі -- олігурія. Який механізм цього явища?

Зниження клубочкової фільтрації

Зниження онкотичного тиску крові

Зниження проникності клубочкової мембрани

Зниження кількості клубочків, що функціонують

3984 / 4299
У шестимісячної дитини на шкірі виявлено щільний вузол червоного кольору, який під час натиснення стає блідим. Для якого захворювання характерні такі паталогічні зміни?

Пігментного невусу

Гемангіоми

Лімфангіоми

Лейоміоми

Меланоми

3985 / 4299
У пацієнтки віком 18 років фолікулярна ангіна супроводжується болем у горлі, ознаками інтоксикації, слабкістю, гіпертермією. Який жарознижувальний препарат варто призначити у цьому разі?

Аскорбінова кислота

Парацетамол

Лоратадин

Окситоцин

3986 / 4299
Який матеріал найчастіше використовують для дослідження каріотипу методом вивчення статевого хроматину?

Статеві клітини

Епідерміс шкіри

Епітелій ротової порожнини

Еритроцити

3987 / 4299
В клініку звернувся чоловік 45-ти років із скаргами на втрату чутливості в ділянці задньої 1/3 язика. Функція якої пари черепно-мозкових нервів порушена?

ІХ

XII

V

Х

VIII

3988 / 4299
У хворого менінгіт. Показана пункція підпавутинного простору. Визначте, між якими структурами він розташований:

Павутинною та м'якою оболонками

Окістям та твердою мозковою оболонкою

Твердою та павутинною оболонками

3989 / 4299
У хворого з пораненням кисті почав утворюватися набряк. У якій стадії порушення місцевого кровообігу це відбувається?

Престаз

Артеріальна гіперемія

Венозна гіперемія

Стаз

Спазм артеріол

3990 / 4299
Хвора 40-ка років знаходиться на лікуванні в терапевтичному відділенні. В температурному листі хворої відмічаються цикли підвищення температури, які чергуються з періодами її нормалізації, що тривають кілька діб. До якого із типів температурних кривих відносяться дані показники?

Febris intermittent

Febris recurrens

Febris continua

Febris remittens

3991 / 4299
У хворого спостерігається параліч жувальних м'язів зліва. Вкажіть нерв, гілки якого іннервують ці м'язи:

Надочноямковий та підблоковий нерви

Носовійковий нерв

Нижньощелепний нерв

Верхньощелепний нерв

3992 / 4299
При гістологічному дослідженні кістки нижньої щелепи виявлено пухлину, що складається з фіброзної тканини серед якої розташовані базофільні цементоподібні маси у вигляді вогнищ різної величини. Діагностуйте пухлину:

Гігантська цементома

Цементобластома

Одонтогенна фіброма

Цементома

Цементуюча фіброма

3993 / 4299
Однією з реакцій матричного синтезу є реплікація. Яка нова молекула утворюється внаслідок цього з молекули ДНК?

тРНК

Про-іРНК

ДНК

рРНК

3994 / 4299
Травма мозку викликала підвищене утворення аміаку. Яка амінокислота бере участь у знешкодженні аміаку з мозкової тканини?

Лізин

Валін

Глутамінова

Триптофан

Тирозин

3995 / 4299
У пацієнта під час аускультації вислуховується шум у ділянці проєкції верхівки серця. Ваду якого клапана серця можна припустити?

Мітрального

---

Клапана легеневого стовбура

Аортального

3996 / 4299
У відповідь на застосування знеболюючого засобу при екстракції зуба у хворого з'явились: виражений набряк м'яких тканин нижньої та верхньої щелеп, висип на шкірі обличчя, почервоніння, свербіж. Який з патологічних процесів лежить в основі такої реакції на анестетик?

Алергія

Токсична дія препарату

Порушення лімфовідтоку

Недостатність кровообігу

3997 / 4299
Яка мікрофлора переважає на початку утворення зубної бляшки на поверхні зуба?

Лептотрихії

Фузобактерії

Бактероїди, кандіди

Стрептококи, вейлонели

Облігатні анаероби

3998 / 4299
В експерименті вивчалися головні показники гемодинаміки. Який з нижче перерахованих показників гемодинаміки є однаковим для великого й малого кіл кровообігу?

Лінійна швидкість кровотоку

Об'ємна швидкість кровотоку

Діастолічний артеріальний тиск

Опір кровотоку

3999 / 4299
Електрик випадково торкнувся обома руками оголеного електропроводу і загинув. Які порушення серцевого ритму спричинили смерть чоловіка?

Зменшення скоротливої здатності міокарда

Повної атріовентрикулярної блокади

Порушення вагусної регуляції серця

Фібриляції передсердь і шлуночків

Пригнічення автоматизму синоатріального вузла

4000 / 4299
Лікар-стоматолог-хірург видалив у пацієнта кісту нижньої щелепи в ділянці другого премоляра. Кіста була сформована в альвеолярному краю щелепи, стінка її тонка, у порожнині розташований рудиментарний зуб. Мікроскопічно порожнину кісти вистилає плоский багатошаровий епітелій. Яке захворювання розвинулося у пацієнта?

Радикулярна кіста

Парадентальна кіста

Фолікулярна кіста

Резидуальна кіста

Кістогранульома

4001 / 4299
В експерименті збудливу клітину помістили в сольовий розчин, який не містить іонів натрію. Як це позначиться на розвитку процесу збудження?

Тривалість потенціалу дії зменшується

Амплітуда потенціалу дії збільшується

Амплітуда потенціалу дії зменшується

Тривалість потенціалу дії збільшується

Потенціал дії не виникає

4002 / 4299
Під час спалаху внутрішньолікарняної інфекції з носоглоток медичного персоналу та виділень ран у хворих хірургічного відділення висіяно чисті культури S. aureus. Яке дослідження потрібно провести для встановлення ймовірного джерела інфекції?

Фаготипування виділених культур

Сероідентифікація

Установлення біохімічних властивостей

Проведення повторних посівів

Установлення антибіотикочутливості

4003 / 4299
У хворого 28-ми років відзначалися підвищення артеріального тиску, гематурія і набряки на обличчі. Незважаючи на лікування, почали наростати явища ниркової недостатності. Через 6 місяців хворий помер від уремії. Мікроскопічно при дослідженні нирок в клубочках виявлена проліферація нефротелію капсули, подоцитів з утворенням ''напівмісяців'', склероз і гіаліноз клубочків. Який найбільш імовірний діагноз?

Гострий пієлонефрит

Підгострий гломерулонефрит

Гострий гломерулонефрит

Нефротичний синдром

Хронічний гломерулонефрит

4004 / 4299
В експерименті при електричному подразненні блукаючого нерва збільшується вихід в синаптичну щілину ацетилхоліну, що зменшує ЧСС через наступний механізм:

Деполяризація мембрани кардіоміоцитів

Збільшення швидкості проведення збудження в АВ-вузлі

Гіперполяризація мембрани кардіоміоцитів

Зменшення тривалості потенціалу дії

Збільшення тривалості потенціалу дії

4005 / 4299
При досліджені хворого лікар визначив, що його язик не рухається вперед (не висувається з рота). Який м'яз пошкоджений?

Поздовжній м'яз язика

Поперечний м'яз язика

Під'язиково-язиковий

Підборідно-язиковий

Шило-під'язиковий

4006 / 4299
У пацієнта аспермія. Який орган не працює?

Придаток яєчка

Передміхурова залоза

Простата

Сім'яні міхурці

Яєчко

4007 / 4299
У хворої 43-х років на тлі септичного шоку відзначаються тромбоцитопенія, зменшення фібриногену, поява в крові продуктів деградації фібрину, поява петехіальних крововиливів. Вкажіть причину виникнення цих змін:

Геморагічний діатез

Аутоіммунна тромбоцитопенія

ДВЗ-синдром

Екзогенна інтоксикація

Порушення продукування тромбоцитів

4008 / 4299
У бактеріологічній лабораторії проводиться дослідження якості питної води. Її мікробне число виявилося близько 100. Які мікроорганізми враховувалися при цьому?

Всі бактерії, що виросли на живильному середовищі

Бактерії групи кишкової палички

Бактерії, патогенні для людей та тварин

Умовно-патогенні мікроорганізми

Ентеропатогенні бактерії та віруси

4009 / 4299
Пацієнт 45-ти років з гіпертонічною хворобою, що 4 дні лікується антигіпертензивним препаратом, відзначає нормалізацію артеріального тиску, але скаржиться на сонливість і загальмованість. Який препарат приймає хворий?

Празозин

Еналаприл

Каптоприл

Апресин

Клофелін

4010 / 4299
У стінці кровоносної судини визначається велика кількість еластичних волокон в усіх оболонках, вікончасті еластичні мембрани в середній оболонці. Які фактори зумовлюють ці особливості будови стінки судин?

Мала швидкість руху крові

Малий тиск крові

Осмотичний тиск

Великий тиск крові

Велика швидкість руху крові

4011 / 4299
Під час абсолютного голодування єдиним джерелом води для організму є процес окислення органічних сполук. Яка з нижченаведених речовин за цих умов є основним джерелом води?

Білки

Ліпопротеїни

Вуглеводи

Глікопротеїни

Жири

4012 / 4299
Розчин хлораміну Б широко застосовують у медичній практиці. Укажіть, яка фармакологічна дія відсутня у цього засобу.

Протиалергічна

Знебарвлювальна

Дезодоруюча

Антисептична

Сперматоцидна

4013 / 4299
Який з наведених протимікробних препаратів textbfНЕ НАЛЕЖИТЬ до антибіотиків групи цефалоспоринів?

Ципрофлоксацин

Цефтріаксон

Цефазолін

Цефепім

Цефалексин

4014 / 4299
У юнака віком 16 років, який не звернувся за допомогою, коли порізав ногу, розвинувся правець. Який протисудомний засіб необхідно призначити пацієнту?

Тубокураріна хлорид

Фенобарбітал

Этосуксемід

Карбамазепін

Циклодол

4015 / 4299
У пацієнта спостерігається порушення засвоєння жирів. Лікар призначив йому препарат жовчі для покращення травлення жирної їжі. Які компоненти жовчі беруть участь у цьому процесі?

Холестерин і його ефіри

Солі жовчних кислот

Насичені жирні кислоти

Білірубінглюкуроніди

Дигліцериди

4016 / 4299
Пацієнт віком 61 рік помер у реанімаційному відділенні внаслідок синдрому поліорганної дисфункції. Відомо, що раніше пацієнту проводили оперативне втручання із приводу гострого гнійного періоститу. Під час гістологічного дослідження некроптатів виявлено гіперплазію лімфоїдної тканини мигдаликів, дифузну інфільтрацію нейтрофілами некротично зміненого альвеолярного відростка щелепи, реґіонарний гнійний лімфаденіт, флегмону м'яких тканин шиї, двобічну полісегментарну гнійну пневмонію, спленомегалію, незворотні зміни у кардіоміоцитах та епітелії ниркових канальців. Під час бактеріологічного дослідження у крові виявлено золотистий стафілокок. Який вид сепсису спричинив такі паталогічні зміни?

Хірургічний

Одонтогенний

Тонзилогенний

Терапевтичний

Криптогенний

4017 / 4299
Яка речовина в складі слини синтезується в слинних залозах та поза ними і надає слині густий слизовий характер?

Фосфати

Мальтаза

Сульфати

Амілаза

Муцин

4018 / 4299
На одязі людини, яку звинувачують у вбивстві, виявлено плями крові. Яка реак-ція зможе довести, що ці плями крові належать саме людині?

Реакція аглютинації

Реакція преципітації

Реакція зв’язування комплементу

Реакція імунофлюоресценції

Реакція нейтралізації

4019 / 4299
Під час огляду порожнини рота у жінки віком 52 роки спостерігаються гіперемія, набряк та кровоточивість ясен нижньої щелепи. Різці нижньої щелепи мають рухомість ІІ ступеня, наявні зубо-ясенні кишені, під час натискування з-під слизової ясен виділяється жовто-зелений ексудат. Для якого захворювання характерні ці симптоми?

Стоматит

Остеомієліт

Періостит

Періодонтит

Пародонтит

4020 / 4299
У пацієнта виявлено високий рівень альдостерону в крові. Яка з фізіологічно активних речовин, найімовірніше, сприяла цьому?

цГМФ

цАМФ

Ангіотензин ІІ

Натрійуретичний фактор

Простагландин Е2

4021 / 4299
Під час операції на привушній залозі хірург перев'язав вену, що проходить крізь товщу цієї залози. Назвіть цю вену.

Vv. pharyngeae

V. lingualis

V. facialis

Vv. thyroideae superiores

V. retromandibularis

4022 / 4299
Важливим топографічним орієнтиром тіла людини є реберна дуга. Хрящами яких ребер вона утворена?

З 1 по 12

З 11 по 12

З 1 по 7

З 7 по 10

Тільки 12

4023 / 4299
Хворому з невралгією трійчастого нерва був введений парентерально ненаркотичний анальгетик, який має швидку, але короткочасну дію, випускається в таблетках і ампулах. Вкажіть, який препарат був введений хворому:

Піроксикам

Ібупрофен

Анальгін

Індометацин

Кислота мефенамова

4024 / 4299
У хворого, який тривалий час не звертався до лікаря-стоматолога для лікування хронічного карієсу в 36 зубі, з'явився різкий біль в нижній щелепі, припухлість щоки, температура тіла піднялася до 38^oC) Яких змін в аналізі крові слід очікувати у хворого?

Моноцитоз

Нейтрофільоз

Анемія

Лейкопенія

Еозинофілія

4025 / 4299
У дитини 2-х років, що перенесла ГРВІ і померла при явищах легенево-серцевої недостатності, права легеня гіперемована, у 2, 6, 10 сегментах на поверхні і на розрізі виявляються безповітряні вогнища неправильної форми, жовтуватого кольору, розмірами від декількох міліметрів до 1 см. Мікроскопічно у даних ділянках легеневої тканини в альвеолах, бронхіолах і дрібних бронхах - ексудат з переважанням нейтрофілів. Який найбільш імовірний діагноз?

Крупозна пневмонія

Гострий бронхіт

Інтерстиційна пневмонія

Вогнищева пневмонія

Абсцес легень

4026 / 4299
До лікаря-стоматолога звернувся хворий із лихоманкою і характерними дрібними везикулами на слизовій щік, твердого піднебіння і язика. Лікар припускає герпетичний стоматит. Які дослідження слід провести додатково для підтвердження діагнозу?

Посів на середовище Ігла

Посів на середовище 199 з додаванням бичачої сироватки

Посів на середовище Раппопорта

Постановка реакції преципітації

Зараження курячого ембріона в хоріоналантоїс, уведення матеріалу в мозок білим мишам

4027 / 4299
При огляді ротової порожнини чоловіка 50-ти років, який тривалий час палить, на слизовій оболонці язика виявлено неправильної форми бляшку білого кольору. Гістологічно виявлено потовщення багатошарового плоского епітелію, паракератоз, гіперкератоз та акантоз. Вкажіть вид патологічного процесу:

Авітаміноз A

Гіпертрофічний глосит

Хронічний стоматит

Кератоакантома

Лейкоплакія

4028 / 4299
На мікропрепараті великих півкуль кори головного мозку виявлено великі клітини пірамідної форми. Який учений уперше описав ці клітини?

Леношек

Бец

Ніссль

Гольджі

Кахаль

4029 / 4299
Під час ультразвукового дослідження серця у дитини 1,5 років знайдено незарощення овального отвору, що підтверджено клінічно. У якому відділі серця розташований даний дефект?

Міжшлуночкова перегородка

Верхівка серця

Лівий предсердно-шлуночковий клапан

Міжпередсердна перегородка

Правий предсердно-шлуночковий клапан

4030 / 4299
У хворого кривошия. Який м'яз шиї уражений?

M. Omohyoideus

M. Platisma

M. Sternocleidomastoideus

M. Mylohyoideus

M. Sternohyoideus

4031 / 4299
У пацієнта, який упродовж тривалого часу вживає препарати, що блокують вироблення ангіотензину ІІ, виникли брадикардія та порушення серцевого ритму. Яка клінічна проблема є причиною цих розладів?

Гіперкальціємія

Гіперкаліємія

Гіпернатріємія

Гіпокаліємія

Гіпокальціємія

4032 / 4299
Після введення в експлуатацію системи подачі води у новий мікрорайон міста працівники санепідслужби визначили загальне мікробне число води. Яке значення цього показника є гранично допустимим для питної води?

1000

500

100

400

10

4033 / 4299
Чоловік віком 70 років скаржиться на біль у дрібних суглобах рук і ніг. Суглоби деформовані та болючі. Під час лабораторного дослідження виявлено підвищений рівень солей сечової кислоти в крові та сечі. Порушення обміну яких речовин характеризують ці показники?

Нуклеопротеїдів

Ліпопротеїдів

Кальцію

Калію

Хромопротеідів

4034 / 4299
Хворий скаржиться на болі під час жування, особливо при висуненні нижньої щелепи вперед і зміщенні її вбік. Функція яких м'язів порушена?

Медіальні крилоподібні

Латеральні крилоподібні

Щелепно-під'язикові

Жувальні

Скроневі

4035 / 4299
Пацієнтка, яка хворіє на бронхіальну астму, для лікування захворювання щоденно 3 рази на день приймала таблетки під язик. Через деякий час вона поскаржилася на тахікардію, біль у ділянці серця, запаморочення, зниження артеріального тиску, швидку втомлюваність. Який лікарський засіб приймала жінка?

Анаприлін

---

Адреналін

Ізадрин

Дигітоксин

4036 / 4299
Хвору прооперовано з приводу стегнової кили. Де проектується вихідний отвір цієї кили?

Сіднична ділянка

Лобкова ділянка

Пахвинна ділянка

Стегновий трикутник

4037 / 4299
В якому органі відбувається біотрансформація (метаболічні перетворення) більшості лікарських речовин при їх надходженні в організм?

Шкіра

Кишківник

Нирки

Печінка

Легені

4038 / 4299
До лікаря-офтальмолога звернувся підліток з порушенням гостроти зору. Лікар пояснив це спазмом акомодації. Який з названих компонентів очного яблука входить до акомодаційного апарату ока?

Скловидне тіло

Сітківка

Рогівка

Циліарний м'яз

Склера

4039 / 4299
У пацієнта спостерігається пошкодження ціліарного тіла. Порушення роботи якого апарату ока буде спостерігатися в цьому разі?

Світлопровідного апарату

Акомодацiйного апарату

Захисного апарату

Світлочутливого апарату

Трофічного апарату

4040 / 4299
До гастроентеролога звернулась мама з дитиною 12-ти років зі скаргами на зниження апетиту в дитини, метеоризм. При ендоскопічному обстеженні діагностовано дискінезію жовчних проток, а у дуоденальному вмісті виявлено найпростіші грушоподібної форми з двома ядрами та багатьма джгутиками. Яке захворювання найбільш імовірно у дитини?

Амебіаз

Балантидіаз

Токсоплазмоз

Трихомоноз

Лямбліоз

4041 / 4299
У вагітної пацієнтки віком 20 років на вестибулярній поверхні ясен різця виявлено кругле формування червонуватого кольору з виразкуванням на поверхні. Під час мікроскопічного дослідження спостерігається схожість із капілярною гемангіомою. Яке утворення виникло у пацієнтки на яснах?

Папілома

Фіброзний епуліс

Фіброма

Гігантоклітинний епуліс

Ангіоматозний епуліс

4042 / 4299
Важливою складовою частиною ниркового фільтраційного бар'єру є тришарова базальна мембрана, яка має спеціальну сітчасту будову її середнього електроннощільного шару. Ця мембрана міститься в:

Капілярах перитубулярної капілярної сітки

Проксимальному канальці

Нирковому тільці

Дистальному прямому канальці

Тонкому канальці

4043 / 4299
У хворого на малярію під час розтину спостерігається виражена жовтушність шкіри, склер та слизових оболонок. Селезінка збільшена в розмірі, аспідно-сірого кольору. Таке забарвлення селезінки обумовлене наявністю:

Ліпофусцину

Гемосидерину

Гемомеланіну

Меланіну

Гематопорфірину

4044 / 4299
У пацієнта виміряли артеріальний тиск методом вислуховування судинних тонів. Укажіть прізвище дослідника, який запропонував цей метод вимірювання АТ?

Гольц

Коротков

Людвіг

Сєченов

Ріва-Роччі

4045 / 4299
В експерименті у зародка кролика зруйновано міотом. Порушення розвитку якої структури викличе ця маніпуляція?

Скелетна мускулатура

Серозні оболонки

Гладка мускулатура

Сполучна тканина шкіри

Осьовий скелет

4046 / 4299
Під впливом радіації постраждали клітини базального шару епідермісу. Яка функція останнього послабиться або загальмується перш за все?

Регенеративна

Діелектрична

Всмоктувальна

Бар'єрна

Захисна

4047 / 4299
Дитина 3 років із симптомами стоматиту, гінгівіту та дерматиту відкритих ділянок шкіри була госпіталізована. Під час обстеження встановлено спадкове порушення транспорту нейтральних амінокислот у кишечнику. Нестачею якого вітаміну будуть зумовлені ці симптоми?

Біотину

Пантотенової кислоти

Ніацину

Кобаламіну

Вітаміну A

4048 / 4299
На поверхні коронки другого великого кутнього зуба, яка контактує з щокою, лікар виявив каріозну порожнину. Як називається вражена поверхня коронки?

Facies mesialis

Facies distalis

Facies vestibularis

Facies occlusalis

Facies lingualis

4049 / 4299
Під час дослідження клітин було встановлено в їх цитоплазмі високий вміст ферменту аміноацил-тРНК-синтетаза. Він забезпечує в клітині такий процес:

Реплікація

Репарація

Активація амінокислот

Транскрипція

Елонгація

4050 / 4299
Пацієнт 27-ми років з пораненням шиї втратив близько 30% об'єму крові. Стан хворого важкий: артеріальний тиск - 60/40 мм рт.ст., частота серцевих скорочень - 140/хв., частота дихання - 30/хв., свідомість не втрачена. Охарактеризуйте стан пацієнта:

Колапс

Кома

Артеріальна гіпертензія

Кардіогенний шок

Гіповолемічний шок

4051 / 4299
Ріст у людини контролюють кілька неалельних домінантних генів. Встановлено, що при збільшенні кількості цих генів ріст збільшується. Який тип взаємодії між цими генами?

Комплементарність

Епістаз

Плейотропія

Полімерія

Кодомінування

4052 / 4299
У хворого, який користується протезами, лікар-стоматолог помітив у ротовій порожнині ушкоджені ділянки слизової оболонки, укриті білим нальотом. Під час мікроскопічного дослідження нальоту виявлено великі овальні грам-позитивні клітини. Які мікроорганізми стали причиною стоматиту у хворого?

Стрептококи

Дріжджоподібні гриби роду Candida

Актиноміцети

Оральні спірохети

Оральні трихомонади

4053 / 4299
Який лікарський засіб належить до фармакотерапевтичної групи інгібіторів ангіотензинперетворювального ферменту?

Анаприлін

Пентамін

Еналаприл

Резерпін

Верапаміл

4054 / 4299
Які типи вивідних проток розрізняють у великих слинних залозах?

Вставні, посмуговані протоки та загальна протока

Внутрішньочасточкові та позазалозисті протоки

Внутрішньочасточкові, міжчасточкові протоки та протока залози

Внутрішньочасточкові та міжчасточ-кові протоки

Внутрішньочасточкові, посмуговані протоки та загальна протока

4055 / 4299
Хворий 67-ми років був доставлений в кардіологічне відділення зі скаргами на періодичні болі у серці, задишку при незначному фізичному навантаженні, ціаноз та набряки. При ЕКГ-обстеженні виявлені позачергові збудження шлуночків серця. Як називається таке порушення ритму?

Екстрасистолія

Тахікардія

Тріпотіння

Брадикардія

Фібриляція

4056 / 4299
В бактеріологічну лабораторію направлено харкотиння хворого на туберкульоз. Для бактеріоскопічного дослідження препаратів-мазків і виявлення туберкульозної палички потрібно використати один із вказаних методів фарбування:

Ціля-Нільсена

Здродовського

Грама

Буррі-Гінса

Романовського

4057 / 4299
При екстракції зуба у хворого було використано місцевий анестетик лідокаїн. В чому полягає механізм анестезуючої дії цього препарату?

Стимуляція М-холінорецепторів

Блокада Н1-гістамінорецепторів

Блокада beta_2-адренорецепторів

Стимуляція ГАМК-рецепторів

Блокада натрієвих каналів

4058 / 4299
Під час видалення каріозного зуба хірург виявив у ділянці кореня зуба вузлик діаметром 1,3 см, сірувато-рожевого кольору, м'якоеластичної консистенції. Мікроскопічно вузлик представлений грануляційною тканиною з наявністю в ній лімфоцитів, плазматичних і опасистих клітин, макрофагів, ксантомних клітин, фібробластів. Установіть діагноз:

Еозинофільна гранульома

Кістогранульома

Проста гранульома

Епітеліальна гранульома

Гранулюючий періодонтит

4059 / 4299
Припинення кровотечі після пологів пов'язано з дією гормонів на структури матки. Який шар органу найбільше брав у цьому участь?

Периметрій

Ендометрій

Внутрішній шар міометрію

Серединний шар міометрію

Поверхневий шар міометрію

4060 / 4299
У гістологічному препараті шліфа коронки зуба в міжклітинній речовині дентину визначається невелика кількість колагенових волокон (волокон Корфа), що йдуть у радіальному напрямку. Назвіть цей шар дентину:

Предентин

Плащовий дентин

Інтерглобулярний дентин

Зернистий шар

Навколопульпарний дентин

4061 / 4299
Жодна азотиста основа одного кодону ДНК не входить до складу іншого кодону. Укажіть назву цієї властивості генетичного коду.

Неперекривність

Специфічність

Триплетність

Колінеарність

Універсальність

4062 / 4299
У дівчинки віком 12 років на дні порожнини рота спостерігається виразка діаметром 5 мм, оточена тканиною яскраво-червоного кольору, яка під час натискання блідне. Виконана біопсія виразки. Під час мікроскопічного дослідження виявлено пухлину, побудовану з безлічі порожнин, заповнених кров'ю. Порожнини вистелені одним шаром ендотеліальних клітин, між ними спостерігається строма, представлена пухкою сполучною тканиною. Для якого виду пухлини характерні такі клініко-лабораторні показники?

Рабдоміосаркома з вторинними змінами

Остеобластокластома

Звиразкована кавернозна гемангіома

Звиразкована меланобластома

Плоскоклітинний незроговілий рак

4063 / 4299
У чоловіка 23-х років виникла перфорація твердого піднебіння, в області якого виявлене щільне утворення із чіткими межами. Після операції при мікроскопічному дослідженні цього утворення було виявлено: значне вогнище казеозного некрозу, оточене грануляційною тканиною з ендоваскулітом, клітинним інфільтратом, що складається з лімфоцитів, епітеліоїдних клітин, з переважанням плазмоцитів. Яке найбільш імовірне захворювання у хворого?

Лепра

Сифіліс

Саркома

Склерома

Туберкульоз

4064 / 4299
Регенерація епідерміса в ділянках травматичного пошкодження відбувається за рахунок росткової зони (мальпігієвий шар). Які шари епідермісу входять до складу цієї зони?

Базальний і остистий

Базальний і роговий

Зернистий і блискучий

Блискучий і роговий

Остистий і зернистий

4065 / 4299
У людини діагностована спадкова моногенна хвороба. Це є:

Гіпертонія

Гіменолепідоз

Гемофілія

Виразкова хвороба шлунка

Поліомієліт

4066 / 4299
Укажіть правильне твердження про позачергове збудження, яке виникло в міокарді шлуночків.

Знижує швидкість проведення збудження по робочих кардіоміоцитах

Знижує автоматизм синусно-передсердного вузла

Підвищує швидкість проведення збудження по робочих кардіоміоцитах

Підвищує автоматизм синусно-передсердного вузла

Відсутній вплив на автоматизм синусно-передсердного вузла

4067 / 4299
На мікропрепараті яєчника представлено округле утворення, залозисті клітини якого містять ліпідні краплі. Визначте цю структуру:

Атретичне тіло

Зрілий фолікул

Первинний фолікул

Примордіальний фолікул

Жовте тіло

4068 / 4299
У хворого, що страждає впродовж тижня на пневмонію, при мікроскопуванні харкотиння виявлено личинки гельмінта. В крові - еозинофілія. Про який діагноз можна думати в цьому випадку?

Аскаридоз

Фасціольоз

Парагонімоз

Теніоз

Ехінококоз

4069 / 4299
Під час обстеження пацієнта виявлено аномалію розвитку емалі зуба. Який структурний елемент емалевого органа пошкоджено?

Зовнішній епітелій

Шийка

Проміжний шар

Внутрішній епітелій

Пульпа

4070 / 4299
Хворому після радіоактивного опромінення лікар рекомендував збільшити в раціоні вміст рослинних олій - джерела полієнових жирних кислот. Назвіть кислоту, що містить три подвійних зв'язки:

Олеїнова

Стеаринова

Арахідонова

Ліноленова

Пальмітинова

4071 / 4299
При пародонтозі відбувається деструкція білкових та полісахаридних компонентів сполучної тканини. Який з наведених білків входить до складу сполучної тканини?

Альбумін

Трансферин

Антитрипсин

Колаген

Церулоплазмін

4072 / 4299
У гістопрепараті головного кінця зародка 5 тижнів розвитку визначаються зяброві дуги. Укажіть, що розвивається з першої пари цих утворень:

Нижньощелепні та верхньощелепні відростки

Верхньощелепні відростки

Нижньощелепні відростки

Зовнішній слуховий прохід

Щитоподібний хрящ

4073 / 4299
Під час огляду семирічної дитини, яка проживає в районі із вмістом фтору в питній воді понад 2 мг на літр води, лікар-стоматолог виявив на ділянці різців верхньої та нижньої щелеп поодинокі пігментовані, фарфороподібні та крейдоподібні плями і смужки, що вкривають близько половини поверхні коронок зубів. Ураження локалізоване лише в емалі і не зачіпає дентин. Яке захворювання розвинулося у дитини?

Поверхневий карієс

Флюороз

Рахіт

Місцева гіпоплазія

4074 / 4299
Під час розтину тіла чоловіка віком 76 років, який помер від серцево-судинної недостатності, у інтимі аорти виявлені об'ємні біло-жовті бляшки з дрібнозернистими аморфними масами в центрі, які заглиблюються в товщу стінки. Для якої стадії атеросклерозу характерні такі патологічні зміни стінки аорти?

Ліпосклерозу

Звиразкування

Атероматозу

Атерокальцинозу

Ліпоїдозу

4075 / 4299
У піддослідної тварини досліджували види скорочення м'язів травного тракту та виявили різну їх функціональну спрямованість. Було встановлено, що лише один тип рухової активності здійснюється циркуляторним та повздовжніми м'язами. Назвіть його:

Жування

Перистальтика

Ритмічна сегментація

Маятникоподібне скорочення

Тонічне скорочення сфінктерів

4076 / 4299
Під час обстеження порожнини рота у пацієнтки спостерігається гіперемія слизової оболонки щік, дна порожнини рота, м'якого та твердого піднебіння. Слизова оболонка тьмяна, з великою кількістю дрібних виразок, вкрита непрозорим слизом. Який імовірний діагноз?

Гангренозний стоматит

Хронічний рецидивуючий афтозний стоматит

Виразковий стоматит

Гнійний стоматит

Катаральний стоматит

4077 / 4299
При огляді ротової порожнини стоматолог виявив появу у дитини перших великих верхніх кутніх зубів. Який вік дитини?

10-11 років

8-9 років

6-7 років

12-13 років

4-5 років

4078 / 4299
Методом непрямої калориметрії встановлено, що основний обмін досліджуваного на 40% нижче належного. Порушення діяльності якої ендокринної залози є причиною цього?

Підшлункова залоза

Епіфіз

Тимус

Надниркові залози

Щитоподібна залоза

4079 / 4299
У стоматологічну поліклініку доставлено хворого, який замість розчину хлоргексидину прополоскав рот розчином оцтової есенції. Скаржиться на відчуття печіння, біль під час вживання їжі. Під час огляду на слизовій рота виявлено щільну плівку білувато-сірого кольору. Виберіть із призначеного стоматологом лікування кератопластичний препарат:

Вінілін

Діазолін

Анестезин

Натрію гідрокарбонат

Магнію окис

4080 / 4299
Хворий звернувся до лікаря зі скаргами на загальну слабкість, порушення сну. Об'єктивно: шкіра має жовтий колір. У крові: збільшена кількість прямого білірубіну, жовчні кислоти. Кал ахолічний. Для якого стану характерні ці зміни?

Хронічний холецистит

Гемолітична жовтяниця

Механічна жовтяниця

Паренхіматозна жовтяниця

Синдром Жильбера

4081 / 4299
Юнаку 20-ти років з метою профілактики було введено анатоксин. Проти якого з вказаних захворювань проводилося щеплення?

Коклюш

Скарлатина

Дифтерія

Менінгіт

Туберкульоз

4082 / 4299
У хворого аритмія. Який лікарський засіб слід призначити хворому?

Імізин

Нітрогліцерин

Аміодарон

Еуфілін

Кавінтон

4083 / 4299
Під час фізичного й емоційного навантаження людина є менш чутливою до болю. Причиною цього є активація:

Парасимпатичної системи

Функції надниркових залоз

Функції щитоподібної залози

Антиноцицептивної системи

Ноцицептивної системи

4084 / 4299
До кардіологічного відділення госпіталізована жінка, якій діагностовано ішемічну хворобу серця. Який лікарський засіб треба ввести хворій для припинення нападу миготливої аритмії?

Дигоксин

Левоміцетин

Адреналіну гідрохлорид

Біцилін-5

Кислота аскорбінова

4085 / 4299
В лікарню надійшов пацієнт з травмою голови. Діагностовано перелом клиноподібної кістки у ділянці основи крилоподібного відростка. Вміст якого з названих каналів може бути пошкоджено?

Лицевий

Крилоподібний

Барабанний

М'язовотрубний

Сонний

4086 / 4299
Хвора 13-ти років знаходиться на стаціонарному лікуванні у гематологічному відділенні обласної дитячої лікарні з діагнозом: залізодефіцитна анемія. Який тип гіпоксії наявний у даної хворої?

Змішана

Дихальна

Гемічна

Тканинна

Циркуляторна

4087 / 4299
У померлої жінки 86-ти років, яка страждала на атеросклероз судин головного мозку, на розтині виявлена атрофія кори головного мозку. Як називається ця атрофія відносно причини?

Від недостатнього кровопостачання

Нейротична

Від тиску

Від дії фізичних та хімічних факторів

Дисфункціональна

4088 / 4299
При обстеженні хворого виявлено зниження секреторної функції залоз носової порожнини. Який нерв забезпечує парасимпатичну іннервацію даних залоз?

N. chorda tympani

N. petrosus major

N. petrosus profundus

N. maxillaris

N. petrosus minor

4089 / 4299
Через 8 днів після хірургічної операції у пацієнта розвинувся правець. Лікар припустив, що причиною став контамінований збудником правця шовний матеріал, який був доставлений до бактеріологічної лабораторії. Яке живильне середовище необхідно використовувати для первинного посіву шовного матеріалу?

Ендо

Сабуро

Кітта-Тароцці

ЖСА

Гіса

4090 / 4299
Під час огляду пацієнта виявлено перелом латеральної кістки передпліччя в середній третині. Частина якої кістки передпліччя травмована?

Епіфіз ліктьової кістки

Епіфіз променевої кістки

Діафіз променевої кістки

Метафіз ліктьової кістки

Діафіз ліктьової кістки

4091 / 4299
Сироватка крові хворого має молочний вигляд. При біохімічному дослідженні виявлено високий рівень триацилгліцеролів і хіломікронів. Спадковий дефект якого ферменту викликає цей стан?

Ліпопротеїнліпаза

Тканинна гормон-чутлива ліпаза

Фосфодіестераза

Панкреатична ліпаза

Фосфоліпаза

4092 / 4299
Кінцеві відділи апокринових потових залоз містять міоепітеліальні клітини. Укажіть, яку функцію здійснюють ці клітини?

Секреторну

Скоротливу

Регенераторну

Захисну

Підтримуючу

4093 / 4299
У хворого спостерігається остеопороз кісток, в крові - гіперкальціємія, гіпофосфатемія. Яка причина такого стану?

Пригнічення секреції паратгормону

Посилена секреція тироксину

Пригнічення секреції кортикостероїдів

Посилена секреція кортикостероїдів

Посилена секреція паратгормону

4094 / 4299
Лікар-стоматолог призначив пацієнту з артритом щелепно-лицевого суглоба диклофенак-натрій. Який механізм дії цього препарату?

Пригнічення каталази

Активація опіатних рецепторів

Блокада опіатних рецепторів

Активація фосфодіестерази

Пригнічення циклооксигенази-2

4095 / 4299
У хворої набряки. У сечі велика кількість білку. Про порушення функції якого відділу нефрону це свідчить?

Дистальний звивистий каналець

Низхідна частина петлі Генле

Висхідна частина петлі Генле

Ниркове тільце

Проксимальний звивистий каналець

4096 / 4299
Пацієнту встановлено діагноз: гострий риногенний гайморит (запалення верхньощелепної пазухи). Як інфекція могла проникнути в гайморову пазуху?

Крізь передню черепну ямку

Крізь середню черепну ямку

Крізь нижній носовий хід

Крізь верхній носовий хід

Крізь середній носовий хід

4097 / 4299
Яка амінокислота із нижченаведених кодується одним триплетом?

Лейцин

Серин

Аланін

Лізин

Метіонін

4098 / 4299
Дитині 13-ти років видалили другий молочний великий кутній зуб. Який постійний зуб виросте на його місці?

Третій великий кутній зуб

Перший малий кутній зуб

Перший великий кутній зуб

Другий малий кутній зуб

Другий великий кутній зуб

4099 / 4299
Вивчаючи жувальні м'язи, студент зрозумів, що не піднімає нижню щелепу тільки:

Медіальний крилоподібний м'яз

Передні пучки скроневого м'яза

Латеральний крилоподібний м'яз

Середні пучки скроневого м'яза

4100 / 4299
Хворому на виразкову хворобу шлунка призначено препарат в основі дії якого є блокада H_2-гістамінових рецепторів. Який це препарат?

Атропіну сульфат

Омепразол

Фамотидин

Дитилін

4101 / 4299
Під час обстеження пацієнта виявлено підвищення вмісту в сироватці крові ліпопротеїнів низької щільності. Яке захворювання у цього пацієнта?

Запалення легень

Атеросклероз

Анемія

Панкреатит

4102 / 4299
Для формування кісткової тканини зуба потрібен кальцій, в обміні якого велику роль відіграє активна форма вітаміну D, що утворюється в:

Печінці та м'язах

Нирках та серці

Кишечнику та печінці

Нирках та печінці

4103 / 4299
Найчастішою причиною неповного перетравлення жирів у травному каналі та збільшення кількості нейтрального жиру в калі є дефіцит певного ферменту. Назвіть цей фермент.

Ентерокіназа

Кишкова ліпаза

Панкреатична ліпаза

Фосфоліпаза

4104 / 4299
Який метод мікробіологічної діагностики треба використати, щоб підтвердити або спростувати діагноз: холера?

Бактеріоскопічний

Бактеріологічний

Вірусологічний

Біологічний

4105 / 4299
Хворому 60-ти років, який страждає на цукровий діабет, був призначений інсулін. Про який вид медикаментозної терапії йде мова?

Замісна

Профілактична

Симптоматична

Етіотропна

4106 / 4299
Хвора протягом двох тижнів приймала призначену невропатологом мікстуру з приводу неврастенії. Самопочуття хворої дещо покращилось, однак незабаром з'явились скарги на нежить, кон'юнктивіт, шкірні висипи, в'ялість та послаблення пам'яті. Препарат якої групи міг спричинити подібну побічну дію?

Солі брому

Препарати хмелю

Препарати валеріани

Препарати кропиви собачої

4107 / 4299
При розтині тіла померлого, який хворів на туберкульоз, у верхній частці правої легені знайдено порожнину розмірами 3х2 см, яка сполучається з бронхом. Стінка порожнини щільна, має три шари: внутрішній - піогенний, середній - шар туберкульозної грануляційної тканини, зовнішній - сполучнотканинний. Який з перелічених діагнозів найбільш імовірний?

Туберкульома

Фіброзно-осередковий туберкульоз

Фіброзно-кавернозний туберкульоз

Гострий осередковий туберкульоз

4108 / 4299
У дівчинки 8-ми років на тлі ймовірної вірусної інфекції підвищилась температура тіла до 39^oС. Який лікарський засіб слід призначити їй для зниження температури?

Парацетамол

Нікотинамід

Дифенін

Пентазоцин

4109 / 4299
Чоловік, що тривало хворів на хронічний остеомієліт нижньої щелепи, помер від ниркової недостатності. На розтині виявлені збільшені у розмірі нирки, дуже щільні, жовтувато-білі, з воскоподібним блиском. При світломікроскопічному дослідженні виявлено відкладення гомогенних рожевих мас у капілярних петлях клубочків, у стінках артеріол, артерій, у базальній мембрані канальців і у стромі, що дають цегляно-червоне забарвлення за Конго-рот. Який процес розвинувся у нирках?

Фібриноїдний некроз

Первинний амілоїдоз

Загальний гіаліноз

Вторинний амілоїдоз

4110 / 4299
У хворого виявлено злоякісне новоутворення язика. Які особливості цієї пухлини дають можливість віднести її до злоякісних?

Збільшення кількості мітотичних клітин

Експансивний характер росту

Анаплазія

Інфільтративний характер росту

4111 / 4299
У хворого діагностовано тяжку B_12-дефіцитну анемію з порушенням кровотворення. В анамнезі - тотальна резекція шлунка. Наявність яких клітин у периферичній крові дозволяє підтвердити діагноз?

Мікроцити

Мегалоцити

Нормоцити

Овалоцити

Анулоцити

4112 / 4299
Фермент, що з'єднується з субстратом, взаємодіє з ним тільки частиною молекули. Назвіть її:

Кофактор

Активний центр

Кофермент

Ділянка поліпептидного ланцюга

4113 / 4299
Вибрати концентрацію етилового спирту, що має найактивнішу протимікробну дію за наявності білка в середовищі:

15%

40%

70%

60%

4114 / 4299
У пацієнта з різаною раною в ділянці середньої частини груднинно-ключично-соскоподібного м'яза порушена шкірна чутливість у передній ділянці шиї. Який нерв ушкоджено?

N. occipitalis minor

N. auricularis magnus

N. transversus colli

N. phrenicus

4115 / 4299
Чоловік 36-ти років перебуває в інфекційній лікарні з профузною діареєю, ознаками ексикозу, падінням температури тіла. Помер від уремії. Під час розтину знайдено: у просвіті тонкої кишки безколірна рідина у вигляді рисового відвару; слизова оболонка набрякла. При мікроскопічному дослідженні тонкої кишки: повнокрів'я судин, осередкові крововиливи, десквамація ентероцитів, гіперсекреція келихоподібних клітин та лімфо-лейкоцитарна інфільтрація строми слизової оболонки. Поставте діагноз:

Дизентерія

Холера

Черевний тиф

Хвороба Крона

4116 / 4299
Під час аускультації серця було встановлено роздвоєння І тону в V міжреберному просторі по середньоключичній лінії. Патологію якого клапана аускультував лікар?

Клапана верхньої порожнистої вени

Тристулкового

Клапана аорти

Двостулкового

Клапана легеневого стовбура

4117 / 4299
Пацієнтка, якій діагностовано ішемічну хворобу серця, отримувала антиангінальний препарат. Він має такі властивості: розширює вінцеві артерії та периферичні судини (артеріальні й венозні), зменшує потребу міокарда в кисні, поліпшує ендокардіальний кровообіг. Укажіть цей лікарський засіб.

Валідол

Еуфілін

Дибазол

Нітрогліцерин

4118 / 4299
В експериментальній моделі на щурах викликано морфологічне порушення клітин епітелію дистальних відділів нефрону. Які функціональні процеси в нирках при цьому послаблюються?

Реабсорбція натрію та глюкози

Фільтрація

Реабсорбція глюкози

Реабсорбція електролітів та води

4119 / 4299
У пацієнта, якому встановлено діагноз: карієс, узято мазок із ротоглотки. Під час мікроскопічного дослідження, виявлено: одноклітинні організми з широкими псевдоніжками, цитоплазма клітини чітко розділена на два шари, ядро ледь помітне. Якого представника найпростіших виявлено у мазку?

Entamoeba coli

Lamblia intestinalis

Entamoeba histolytica

Entamoeba gingivalis

4120 / 4299
У хворої діагностовано крововилив в задні роги спинного мозку. Якими вони є за функцією?

Чутливими

Парасимпатичними

Руховими

4121 / 4299
Укажіть неколагенові білки органічної фази кісткової тканини пародонта.

Колаген, еластин

Остеокальцин, остеонектин

Енамелін, амелогенін

Альбуміни, глобуліни

4122 / 4299
Чоловік звернувся до хірурга зі скаргами на біль внизу живота справа. Під час глибокої пальпації лікар виявив болісність у правій пахвинній ділянці. У якому відділі кишечника можливий патологічний процес?

Поперечна ободова кишка

Пряма кишка

Сліпа кишка

Нисхідна ободова кишка

4123 / 4299
В чоловіка 33-х років в нижній щелепі виявлене кістозне утворення, що зв'язане з 2-м моляром. В порожнині кісти розташований рудиментарний зуб. Мікроскопічно: внутрішня поверхня кісти вкрита багатошаровим плоским епітелієм, зустрічаються групи муцинопродукуючих клітин. Який найбільш імовірний діагноз?

Пародонтит

Примордіальна кіста

Фолікулярна кіста

Радикулярна кіста

4124 / 4299
Після припинення вживання морфіну за умов його тривалого застосування виникають тяжкі психічні, неврологічні і соматичні порушення. Як називається цей стан?

Абстиненція

Кумуляція

Сенсибілізація

Толерантність

4125 / 4299
У постраждалого з ножовим пораненням шиї кровотеча. При первинній обробці рани встановлено, що пошкоджена судина, яка розташована вздовж латерального краю груднино-ключично-соскоподібного м'яза. Визначте цю судину:

A. carotis interna

A. carotis externa

V. jugularis externa

V. jugularis anterior

V. jugularis interna

4126 / 4299
Результати обстеження хворого з порушенням слиновиділення привушною залозою вказують на можливість ушкодження вушного вузла. Який нерв утворює цей вузол?

N. petrosus minor

N. auricularis magnus

N. petrosus major

N. hypoglossus

4127 / 4299
У хворого віком 30 років, який потрапив до клініки з діагнозом ''гострий гломерулонефрит'', спостерігається протеїнурія. Яке порушення спричинило це явище?

Зниження кількості функціонуючих нефронів

Підвищення гідростатичного тиску на стінку капілярів

Підвищення проникності клубочкової мембрани

Затримка виведення продуктів азотистого обміну

4128 / 4299
Рентгенологічно встановлено скупчення гною у клиноподібній пазусі. В який носовий хід виділяється гній?

Лівий нижній

Правий та лівий верхні

Правий середній

Правий нижній

4129 / 4299
Відомо, що слина у своєму складі має тромбопластини. Яка їх роль у порожнині рота?

Посилюють ферментативні властивості слини

Підвищують імунні властивості слини

Посилюють коагуляційні властивості слини

Посилюють фібринолітичні властивості слини

4130 / 4299
Під час рентгенологічного дослідження грудної клітки в пацієнта діагностовано діафрагмову килу, що розташована в задньому середостінні. За рахунок якого слабкого місця діафрагми утворилася ця кила?

Отвору нижньої порожнистої вени

Попереково-ребрового трикутника

Присередньої та латеральної дугоподібної зв'язок

Сухожилкового центру

4131 / 4299
Під час розвитку зуба першою тканиною закладається дентин. Який емалевий орган є джерелом його розвитку?

Внутрішні клітини

Зубний сосочок

Зубний мішечок

Зубна пластинка

4132 / 4299
У потерпілого травмована верхня щелепа --- вибитий перший малий корінний зуб. Який відросток верхньої щелепи пошкоджено?

Альвеолярний

Лобовий

Виличний

Піднебінний

4133 / 4299
Гостра крововтрата викликала падіння системного артеріального тиску. Посилення секреції якого гормону може стабілізувати цю ситуацію?

Реніну

Інсуліну

Гастрину

Глюкагону

4134 / 4299
Пацієнту після екстракції зуба для зменшення болю призначено ібупрофен. Який із нижченаведених ферментів він пригнічує?

Циклооксигеназу

Фосфоліпазу С

Фосфоліпазу А2

Фосфодіестеразу

4135 / 4299
При гострому запаленні привушної слинної залози спостерігається пошкодження клітин секреторних відділів. Які клітини страждають при цьому?

Серозні, міоепітеліальні

Серозні, клітини з базальною посмугованістю, зірчасті

Білкові, серозні, слизові

Білково-слизові

4136 / 4299
Пацієнтці встановлено діагноз: гнійний стоматит. Який показник загального аналізу крові є характерним для цього захворювання?

Анемія

Лейкоцитоз

Моноцитоз

Тромбоцитоз

4137 / 4299
Під час патологоанатомічного дослідження тіла померлої 9-ти місячної дитини встановлено, що причиною смерті є набряк головного мозку. Який вид порушеного водно-електролітного обміну найбільш імовірно призведе до такого наслідку?

Гіперосмолярна дегідратація

Гіперосмолярна гіпергідратація

Гіпоосмолярна гіпергідратація

Ізоосмолярна дегідратація

4138 / 4299
Під час бактеріоскопічного дослідження мазків спинномозкової рідини, забарвлених за Романовським-Гімзою, виявлено найпростіших у формі півмісяця з загостреними кінцями, блакитною цитоплазмою та рубіново-червоним ядром. Для збудника якого захворювання характерні ці ознаки?

Амебіаз

Малярія

Лейшманіоз

Трипаносомоз

Токсоплазмоз

4139 / 4299
Пацієнта шпиталізовано із травмою потиличної ділянки голови. Під час обстеження виявлено крововилив у ділянці острогової борозни. Укажіть аналізатор, кірковий центр якого, імовірно, пошкодиться.

Вестибулярний

Смаковий

Зоровий

Нюховий

4140 / 4299
У постраждалого із травмою основи черепа встановлено пошкодження гачкоподібного відростка присередньої пластинки крилоподібного відростка клиноподібної кістки. Функція якого м'яза м'якого піднебіння порушиться у цьому разі?

Напрягача піднебінної завіски

М'яза язичка

Піднебінно-глоткового

Підіймача піднебінної завіски

4141 / 4299
Для прискорення заживлення рани слизової оболонки в ротовій порожнині хворому призначено препарат, який являє собою термостабільний білок, що міститься у людини в сльозах, слині, грудному молоці матері, а також його можна виявити в свіжознесеному курячому яйці. Відомо, що він являє собою фактор природної резистентності організму і має назву:

Лізоцим

Іманін

Комплемент

Інтерферон

Інтерлейкін

4142 / 4299
Під час розтину хворого, який помер від серцевої недостатності, в інтимі аорти і коронарних судин виявлено жовтого кольору плями і смуги, сірувато-жовтуваті бляшки, що вибухають над поверхнею інтими, а також осередкове звиразкування цих бляшок із наявністю крововиливів, тромбів і вогнищ звапнення. Для якого захворювання характерні зазначені зміни судин?

Гіпертонічної хвороби

Сифілітичного мезаортиту

Вузликового періартеріїту

--

Атеросклерозу

4143 / 4299
Які органели забезпечують процес перетравлення та видалення рештків?

Клітинний центр

Рибосоми

Мітохондрії

Лізосоми

4144 / 4299
Дитину шпиталізовано з діагнозом: стафілококовий сепсис. На яке живильне середовище потрібно посіяти кров пацієнта з метою виділення збудника?

Середовище Плоскірьова

Цукрово-пептонний бульйон

Жовчно-сольовий агар

Середовище Бучіна

4145 / 4299
В приймальне відділення доставили хворого з тепловим ударом. Які з наведених захисно-компенсаторних реакцій розвиваються при цьому?

Звуження периферійних судин

Стійка гіперглікемія

Підвищення ЧСС

Розширення периферійних судин

4146 / 4299
Під час огляду зуба у його коронці виявлена велика порожнина, дном якої є вузький шар розм'якшеного дентину, що відділяє цю порожнину від пульпи. Який найбільш імовірний діагноз?

Глибокий карієс

Поверхневий карієс

Пульпіт

Середній карієс

4147 / 4299
Для визначення функціонального стану печінки у хворого досліджували екскрецію тваринного індикану у сечі, який утворюється при детоксикації продуктів гниття амінокислоти в товстій кишці. Назвіть цю амінокислоту:

Триптофан

Гліцин

Цистеїн

Валін

4148 / 4299
Стоматолог при огляді хворих зазначив, що у багатьох з них зубна емаль без блиску, з фарфороподібними та пігментованими плямами. У окремих хворих є поодинокі та множинні дефекти емалі у вигляді безбарвних або пігментованих ерозій. Надмірне надходження в організм якої речовини призвело до розвитку таких змін в зубах?

Натрію

Магнію

Кальцію

Фтору

4149 / 4299
При захворюванні нирок у крові зростає вміст залишкового азоту. За рахунок якої фракції зростає вміст залишкового азоту у хворих з патологією нирок?

Амонію

Сечової кислоти

Сечовини

Прокінази

4150 / 4299
Хворий 67-ми років страждає на атеросклероз судин серця і головного мозку. При обстеженні виявлена гіперліпідемія. Який клас ліпопротеїдів плазми крові має найбільше значення в патогенезі атеросклерозу?

Ліпопротеїди низької щільності

Альфа-ліпопротеїди

Ліпопротеїди високої щільності

Хіломікрони

4151 / 4299
До токсикологічного відділення шпиталізовано пацієнта віком 40 років, який отруївся інсектицидом --- хлорофосом. Який із наведених препаратів, що блокує периферичні М-холінорецептори, найефективніший для лікування отруєння?

Амізил

Бензогексоній

Скополамін

Платифілін

Атропіну сульфат

4152 / 4299
Пацієнту з діагнозом: ішемічна хвороба серця, для профілактики тромбоутворення був призначений лікарський засіб із фармакотерапевтичної групи нестероїдних протизапальних препаратів. Вкажіть цей лікарський засіб.

Кислота мефенамова

Піроксикам

Індометацин

Ібупрофен

Кислота ацетилсаліцилова

4153 / 4299
Деякі триплети іРНК (УАА, УАГ, УГА) не кодують амінокислоти, а є термінаторами в процесі зчитування інформації, тобто здатні припинити транскрипцію. Ці триплети мають назву:

Екзони

Стоп-кодони

Інтрони

Оператори

Антикодони

4154 / 4299
Аміак є токсичною речовиною, знешкодження якої відбувається переважно у клітинах печінки в певному циклі. Укажіть цей цикл.

Орнітиновий

Глікогеноліз

Гліколіз

Цитратний

Кнопа-Лінена

4155 / 4299
У потерпілого внаслідок ДТП лікар констатував відсутність дихання і серцевої діяльності протягом 1 хвилини. Якому термінальному стану відповідає ця картина?

Травматичний шок, торпідна фаза

Агонія

Преагонія

Клінічна смерть

Травматичний шок, еректильна фаза

4156 / 4299
Людина отруїлася ціаністим калієм. Укажіть сполуку, поєднання цианіду з якою спричинило смерть цієї людини?

АТФ

Цитохром

Рибофлавін

ДНК

т-РНК

4157 / 4299
У потерпілого від електротравми в ділянці шиї сформувався патологічний фіксований нахил голови у бік пошкодження у поєднанні з поворотом обличчя в протилежний бік. Який м'яз шиї піддався рубцевим змінам?

Під'язиково-лопатковий

Передній драбинчастий

Трапецієподібний

Двочеревцевий

Груднино-ключично-соскоподібний

4158 / 4299
У хворого, померлого від хронічної ниркової недостатності, листки перикарду тьмяні з тонкими ниткоподібними нашаруваннями сірого кольору. Який патологічний процес на перикарді?

Серозне запалення

Фібринозне запалення

Гнійне запалення

Катаральне запалення

Проліферативне запалення

4159 / 4299
Після механічної травми хворому наклали джгут на руку, щоб зупинити кровотечу. Нижче джгута рука зблідла, з'явилося відчуття оніміння. Цей стан є наслідком:

Обтураційної ішемії

Венозного застою

Ангіоспастичної ішемії

Тромбозу

Компресійної ішемії

4160 / 4299
У чоловіка 50-ти років, який декілька років страждав на хронічну печінкову недостатність, виник асцит. Який механізм є головним у виникненні цього порушення?

Поява в крові нейротоксичних речовин

Підвищення тиску в системі воротної вени

Збільшення вмісту в крові ЛПНЩ та ЛПДНЩ

Підвищення онкотичного тиску в крові

Зменшення синтезу альбумінів і глобулінів в печінці

4161 / 4299
Який гормон навколовушних слинних залоз підсилює мінералізацію зубів шляхом стимуляції надходження кальцію у звапнені тканини?

Паратирин

Кортизол

Паротин

Кальцитонін

Глюкагон

4162 / 4299
Для якого гельмінтозу характерна природна осередковість, пов'язана із вживанням населенням річкової риби:

Дікроцеліоз

Фасціольоз

Опісторхоз

Ехінококоз

Теніоз

4163 / 4299
Обстежено 42-річного чоловіка з дещо фемінізованим складом тіла, атрофією сім'яників, слабким ростом волосся на обличчі і грудях. У нейтрофільних лейкоцитах виявлена барабанна паличка. Який діагноз можна поставити?

Фенілкетонурія

Синдром Патау

Синдром Дауна

Синдром Клайнфельтера

Трисомія Х

4164 / 4299
Послідовність триплетів у ДНК визначає порядок розташування амінокислот у молекулі білка. Як називається така властивість генетичного коду?

Колінеарність

Виродженість

Універсальність

Триплетність

Неперекривність

4165 / 4299
У хворого на цукровий діабет після ін'єкції інсуліну настала втрата свідомості, судоми. Який результат може дати біохімічний аналіз крові на вміст цукру?

8,0 ммоль/л

5,5 ммоль/л

1,5 ммоль/л

10,0 ммоль/л

3,3 ммоль/л

4166 / 4299
При якому типі успадкування на гемофілію хворіють чоловіки, а носіями хвороби є жінки?

Х-зчеплене домінантне

Х-зчеплене рецесивне

Аутосомно-рецесивне

Аутосомно-домінантне

Голандричне

4167 / 4299
Під час досліждення кровотворного органу виявлено часточки, утворені лімфоїдною тканиною, строма якої складається з епітеліоретикулоцитів. Який орган досліджується?

Лімфатичний вузол

Селезінка

Червоний кістковий мозок

Тимус

Піднебінний мигдалик

4168 / 4299
Пацієнту встановлено діагноз: аспермія. Функцію якого органа порушено у цьому разі?

Передміхурової залози

Залози Купера

Придатку яєчка

Яєчка

Сім'яних міхурців

4169 / 4299
В епітелії повітроносних шляхів є клітини з куполоподібною апікальною частиною, на поверхні якої розміщуються мікроворсинки. У клітині виявляється добре розвинений синтетичний апарат, а в апікальній частині - секреторні гранули. Укажіть ці клітини.

Камбіальні

Келихоподібні

Клітини Клара

Ендокринні

Клітини без облямівки

4170 / 4299
У пацієнта з діагнозом ''хронічний гінгівіт'' при черговому огляді у лікаря-стоматолога textbfНЕ ВИЯВЛЕНО запальних змін слизової оболонки ясен. Як можна охарактеризувати такий стан пацієнта?

Патологічний процес

Патологічна реакція

Рецидив

Ремісія

Ускладнення

4171 / 4299
Мисливець напився сирої води із ставка. Яким трематодозом він міг заразитися?

Клонорхоз

Дикроцеліоз

Фасціольоз

Опісторхоз

Парагонімоз

4172 / 4299
За допомогою світлової мікроскопії вивчено морфологію Y-хромосоми людини. Центромера хромосоми розташована дуже близько до одного з її кінців. Назвіть тип хромосоми:

Телоцентрична

Субметацентрична

Метацентрична

Політенна

Акроцентрична

4173 / 4299
До лікарні надійшла дитина зі стороннім тілом у дихальних шляхах. У який бронх найбільш імовірно потрапляння стороннього тіла, якщо його діаметр приблизно 1,5 см?

Правий головний

Частковий

Правий сегментарний

Лівий сегментарний

Лівий головний

4174 / 4299
У хворого діагноз -- гайморит. В яке анатомічне утворення відбувається відтік гною із запаленої додаткової пазухи носа?

Нижній носовий хід

Середній носовий хід

--

Загальний носовий хід

Верхній носовий хід

4175 / 4299
Співвідношення між нервовим волокном та всіма м'язовими волокнами, які воно іннервує, називається нейромоторною одиницею. Вкажіть, де найменша кількість м'язових волокон іннервується одним нервовим волокном:

Око

Долоня

Спина

Гомілка

Плече

4176 / 4299
Спеціальний режим харчування призвів до зменшення іонів Са^2+ у крові. До збільшення секреції якого гормону це призведе?

Паратгормону

Вазопресину

Соматотропіну

Тироксину

Тирокальцитоніну

4177 / 4299
Хворому з діагнозом кандидоз порожнини рота було призначено протигрибковий препарат. Який засіб було обрано?

Еритроміцин

Флуконазол

Ампіцилін

Левоміцетин

Бісептол

4178 / 4299
У гістологічному препараті розвитку зуба в одній із тканин, пронизаній канальцями, в міжклітинній речовині видно колагенові волокна, які мають радіальний і тангенціальний напрямок. Визначте, для гістогенезу якої тканини це характерно.

Емалі

Дентину

Пульпи

Щільної оформленої сполучної тканини

Цементу

4179 / 4299
Хворий 23-х років поступив у лікарню із черепно-мозковою травмою у важкому стані. Дихання характеризується судомним посиленим вдихом, який не припиняється, що зрідка переривається видихом. Для якого типу дихання це характерно?

Чейна-Стокса

Апнейстичного

Гаспінг-дихання

Куссмауля

Біота

4180 / 4299
Перед операцією з приводу співставлення уламків верхньої щелепи для нейролептанальгезії у комбінації з анальгетиком фентанілом застосували нейролептик дроперидол. Який вид взаємодії лікарських засобів був використаний?

Синергоантагонізм

Потенційований синергізм

Антагонізм неконкурентний

Антагонізм конкурентний

Адитивний синергізм

4181 / 4299
Фізіологи встановили, що кількість еритроцитів у крові залежить від функціонального стану червоного кісткового мозку й тривалості життя еритроцита. Який термін ''життя'' еритроцита в периферичній крові в середньому?

70 діб

220 діб

50 діб

120 діб

150 діб

4182 / 4299
Хворому зі злоякісною пухлиною для усунення нестерпного болю призначили наркотичний анальгетик. Який механізм розвитку протибольової дії цього засобу?

Гальмування холінергічних рецепторів

Активація опіатних рецепторів

Активація Д2-дофамінових рецепторів

Гальмування серотонінергічних рецепторів

Гальмування гістамінергічних рецепторів

4183 / 4299
У разі зниження pH ротової рідини менше ніж 6,5 порушується:

Інтенсивність метаболічних процесів у пульпі

Надходження мінеральних речовин до твердих тканин зуба

Утворення дентину

Кровопостачання зубів

4184 / 4299
У ВІЛ-інфікованого хворого спостерігається пригнічення активності імунної системи. Ураження яких клітин найбільшою мірою обумовлює стан імунодефіциту у цього хворого?

Плазмоцитів

В-лімфоцитів

Т-хелперів

Т-супресорів

Т-кілерів

4185 / 4299
Під час морфологічного дослідження скелетних м'язів мишей, які тривалий час плавали у басейні, виявлено збільшення кількості мітохондрій із багатьма кристами і просвітленим матриксом. Яка функція клітини перебуває у надзвичайно напруженому стані?

Захисна

Транспортна

Секреторна

Синтетична

Енергетична

4186 / 4299
У препараті сполучної тканини, забарвленому гематоксиліном-еозином, спостерігаються ізогенні групи клітин, оточені базофільною міжклітинною речовиною. Волокнисті структури не виявлені. Яка це сполучна тканина?

Еластична хрящова

Щільна волокниста

Пухка волокниста

Пластинчаста кісткова

Гіалінова хрящова

4187 / 4299
Чоловік 65-ти років захворів на гострий остеомієліт нижньої щелепи. Через 3 дні з'явилися різко виражений набряк шкіри та підщелепних м'яких тканин шиї. В них мікроскопічно виявляється дифузна інфільтрація нейтрофілами. Яке ускладнення основного захворювання розвинулось у шкірі хворого?

Абсцес

Карбункул

Актиномікоз

Фурункул

Флегмона

4188 / 4299
При обстеженні пацієнта встановлено збільшення основного обміну на 50%. Збільшення секреції якого гормону спричинило цю зміну?

Пролактину

Соматотропного

Інсуліну

Тироксину

Паратгормону

4189 / 4299
У пацієнта діагностовано патологію, яка супроводжується зменшенням у внутрішньому середовищі організму летких метаболітів. Через які органи вони виводяться?

Нирки

Легені

Потові залози

Сальні залози

4190 / 4299
Вивчення вiдбиткiв виступiв епiдермiсу пальцiв рук (дактилоскопiя) використовується кримiналiстaми для iдентифiкацiї особи, а також для дiагностики генетичних аномалiй, зокрема хвороби Дауна. Який шар шкiри визначає iндивiдуальнiсть вiдбиткiв?

Сосочковий

Блискучий

Сiтчастий

Роговий

Базальний

4191 / 4299
Під час аутопсії хворого, померлого від серцево-судинної недостатності, виявлено стенозуючий атеросклероз коронарної артерії, ускладнений тромбозом. Гістологічно тромб складається з тромбоцитів, лейкоцитів і фібрину. Який вид тромбу має місце в цьому випадку?

Білий тромб

Червоний тромб

--

Змішаний тромб

Гіаліновий тромб

4192 / 4299
У хворого на бронхіальну астму виникла гостра недостатність дихання. Який тип недостатності дихання виникає в цьому випадку?

Рестриктивне порушення альвеолярної вентиляції

Перфузійний

Дисрегуляторне порушення альвеолярної вентиляції

Дифузійний

Обструктивне порушення альвеолярної вентиляції

4193 / 4299
В зв'язку з наявністю злоякісної пухлини на язику хворому необхідно його видалити. В якому місці легко знайти язичну артерію та перев'язати її?

В лопаточно-підключичному трикутнику

В лопаточно-трапецієвидному трикутнику

В трикутнику Пирогова

В лопаточно-трахейному трикутнику

В сонному трикутнику

4194 / 4299
Клітина є елементарною живою системою, яка забезпечує будову, розвиток, функціонування, пристосування, розмноження та відновлення цілого організму. Вкажіть, з яких основних трьох структурних компонентів складається клітина:

Глікокалікс, ядро, органели

Клітинна оболонка (плазмолема), включення, органели

Цитоплазма, органели, ядро

Гіалоплазма, плазмолема, ядро

Клітинна оболонка (плазмолема), цитоплазма, ядро

4195 / 4299
У потерпілого глибока різана рана у задній ділянці шиї. Який м'яз пошкоджено?

M. trapezius

M. levator scapulae

M. mylohyoideus

M. digastricus

M. sternocleidomastoideus

4196 / 4299
Тривалий вплив на організм людини токсичних речовин призвів до руйнування органел, які відповідають за синтез білків у гепатоцитах печінки. Які органели здійснюють синтез білків у гепатоцитах?

Пероксисоми

Лізосоми

Рибосоми

--

Мітохондрії

4197 / 4299
У хворого спостерігається втрата загальної чутливості на окремих ділянках тіла справа. Яка зі звивин великих півкуль головного мозку уражена?

Зацентральна

Верхня скронева

Середня скронева

Нижня скронева

Передцентральна

4198 / 4299
У препараті паренхіматозного органу розрізняють нечітко відмежовані часточки шестигранної форми, в центрі яких розташована вена, а в міжчасточковій сполучній тканині проходять тріади (артерія, вена і вивідна протока). Який це орган?

Тимус

Селезінка

Щитоподібна залоза

Печінка

Підшлункова залоза

4199 / 4299
Адреналін використовується для подовження дії новокаїну під час інфільтраційної анестезії. З якою дією адреналіну пов'язаний цей ефект?

Розширення судин

Пригнічення тканинних естераз

Звуження судин

Пригнічення функцій нервових закінчень і провідників

Потенціювання дії новокаїну на рівні ЦНС

4200 / 4299
Під час розтину трупа чоловіка віком 46 років, який тривалий час хворів на хронічний бронхіт, виявлено, що легені збільшені в об'ємі, не спадаються, підвищеної повітряності, бліді, тістоподібної консистенції, ріжуться з хрускотом. Під час мікроскопічного дослідження встановлено, що на всіх ділянках легень проксимальні відділи ацинусів різко розширені, стінки альвеол стоншені, капіляри в них недокрівні. Якій клініко-морфологічній формі емфіземи відповідає така морфологічна картина?

Вікарній

Хронічній вогнищевій

Старечій

Ідіопатичній

Хронічній дифузній обструктивній

4201 / 4299
Під час роботи лікарю-стоматологу доводиться довго стояти на ногах, що може призвести до застою крові у венах нижніх кінцівок і до їх варикозного розширення. Провідним механізмом формування застою у даному випадку є зменшення:

Залишкової рушійної сила серця

Присмоктувально-насосного ефекту діафрагми на органи черевної порожнини

Градієнта тиску крові у венозних судинах

Присмоктувального ефекту грудної клітки

Скорочення скелетних м'язів нижніх кінцівок

4202 / 4299
Під час перебування на прийомі у лікаря-стоматолога у пацієнта стався напад стенокардії. Який препарат доцільно використати у цьому разі?

Нітросорбід

Верапаміл

Нітрогліцерин

Но-шпа

Пропранолол

4203 / 4299
З центрифугату порції сечі, одержаної від хворого з підозрою на туберкульоз нирок, приготували препарат для мікроскопії. Який метод фарбування препарату використовують для виявлення збудника?

За Буррі

За Леффлером

За Цілем-Нільсеном

За Грамом

За Ожешко

4204 / 4299
Пацієнта шпиталізовано з пролапсом прямої кишки. Під час огляду прямої кишки виявлено маленькі гельмінти, які нагадують невеличкі батоги з нерівномірним діаметром товщини тіла, прикріплені до слизової. Під час аналізу фекалій виявлено яйця у формі бочки з біполярними пробками. Укажіть найвірогіднішу причину захворювання.

Trichuris trichiura

Lamblia intestinalis

Ascaris lumbricoides

Enterobius vermicularis

Entamoeba histolytica

4205 / 4299
У пацієнта об'єктивно спостерігається: права носогубна складка розгладжена, права очноямкова щілина розширена (її не вдається закрити під час примружування, тому що повіки не змикаються), виникають утруднення під час розмови та вживання їжі (їжа застрягає між щокою і зубами). Який нерв уражено?

N. vagus dexter

N. trigeminus dexter

N. facialis dexter

N. glossopharyngeus sinister

N. abducens dexter

4206 / 4299
У хворого - глибока рвана рана із нерівними краями, вкрита гноєм. У крайових відділах - соковита грануляційна тканина, яка не здіймається над рівнем рани. Назвіть вид загоєння рани:

Загоювання під струпом

Безпосереднє закриття дефекту епітеліальної тканини

Загоювання вторинним натягом

Загоювання первинним натягом

Організація рани

4207 / 4299
У пацієнта віком 46 років під час обстеження виявлені процеси деструкції хрящової тканини колінних суглобів. Які специфічні ознаки характерні для хрящової тканини зазначеної локалізації?

Наявність остеобластів

Наявність охрястя

Відсутність охрястя

Відсутність ізогенних груп хондроцитів

Наявність ізогенних груп остеоцитів

4208 / 4299
Наявність білка у розчині можна виявити за допомогою кольорових реакцій. Яка з нижченаведених реакцій дасть негативний результат при повному гідролізі білка?

Сакагучі

Фоля

Ксантопротеїнова

Нінгідринова

Біуретова

4209 / 4299
У хворого травма склепіння черепа. Які синуси можуть бути пошкоджені?

Сигмоподібний

Нижній сагітальний

Верхній сагітальний

Нижній кам'янистий

Верхній кам'янистий

4210 / 4299
У жінки 38-ми років розвинувся напад бронхіальної астми. Який з перерахованих бронхолітиків, ефективний для надання невідкладної допомоги, належить до групи beta_2-адреноміметиків?

Сальбутамол

Адреналін

Іпратропію бромід

Платифілін

Атропін

4211 / 4299
У хворого при огляді порожнини рота встановлено різке почервоніння слизової оболонки кореня язика. Визначте, яке утворення залучене в запальний процес:

Глотковий мигдалик

Піднебінна завіска

Піднебінний мигдалик

Язиковий мигдалик

Трубний мигдалик

4212 / 4299
Структурною особливістю регуляторних ферментів є наявність алостеричного центру. Укажіть його роль.

Зв'язує кофермент

Змінює структуру субстрату

Зв'язує регуляторний ефектор

Зв'язує субстрат

Сприяє дисоціації коферменту

4213 / 4299
Хворому 55-ти років поставлений діагноз: гострий гломерулонефрит. Вкажіть основний механізм розвитку анемії:

Зменшення продукції еритропоетину

Зменшення синтезу ниркових простагландинів

Зменшення канальцевої реабсорбції

Зменшення клубочкової фільтрації

Ниркова азотемія

4214 / 4299
На гістологічному препараті слизової оболонки органу визначається багатошаровий епітелій, що складається з 20-25 клітинних шарів, поверхневі клітини мають плоску форму. Який це орган?

Дванадцятипала кишка

Товста кишка

Дно шлунка

Стравохід

Тонка кишка

4215 / 4299
Пацієнт 55-ти років із променевою хворобою доставлений у лікарню з ознаками геморагічного синдрому. Які зміни в крові вирішальні в патогенезі цього синдрому?

Імунна толерантність

Тромбоцитопенія

Нейтропенія

Еозинопенія

Лімфопенія

4216 / 4299
Хворому для покращення перетравлення жирної їжі призначено препарат жовчі. Які компоненти даного препарату беруть участь в емульгуванні жирів?

Холестерин і його ефіри

Жовчні кислоти

Вищі жирні кислоти

Білірубінглюкуроніди

Дигліцериди

4217 / 4299
У хворої дитини спостерігаються ознаки ахондроплазії (карликовості). Відомо, що це моногенне захворювання, і ген, який відповідає за розвиток такої аномалії є домінантним. У рідного брата цієї дитини розвиток нормальний. За генотипом здорова дитина буде:

Аа

аа

ААВВ

АаВb

АА

4218 / 4299
Педіатр, оглядаючи однорічну дитину, виявив в порожнині рота 4 зуба. Вкажіть, яку кількість молочних зубів повинна мати дитина цього віку:

14

20

8

12

10

4219 / 4299
Під час надання стоматологічної допомоги лікар одержав травму вказівного пальця з порушенням цілості шкіри і ймовірним забрудненням кров'ю пацієнта. У таких випадках інструкція передбачає обстеження пацієнта щодо ВІЛ-інфекції та вірусних гепатитів. Які дослідження слід провести у пацієнта?

Дослідити в крові рівень Т-хелперів

Виділити збудників шляхом зараження культури клітин

Дослідити кров на наявність маркерів гепатитів і антитіл проти ВІЛ

Зробити посів крові на цукровий бульйон

Виявити у крові специфічні антитіла

4220 / 4299
Підвищення рівня ЛПВЩ веде до зниження ризику захворювання на атеросклероз. Який механізм антиатерогенної дії ЛПВЩ?

Активують перетворення холестерину в жовчні кислоти

Сприяють всмоктуванню холестерину в кишечнику

Беруть участь у розпаді холестерину

Постачають тканини холестерином

Вилучають холестерин із тканин

4221 / 4299
Під час аутопсії тіла чоловіка 43 років, який помер від серцево-легеневої недостатності, знайдено у нижній частці правої легені порожнину діаметром 3 см, заповнену тягучим зелено-сірим вмістом. Під час гістологічного дослідження стінка описаного утвору представлена сполучною та молодою грануляційною тканиною, у просвіті -- нейтрофільні лейкоцити та продукти їх розпаду. Діагностуйте вид запалення:

Емпієма

Хронічний абсцес

Карбункул

Фурункул

Гострий абсцес

4222 / 4299
Після проникнення в організм бактерії фагоцитуються макрофагами. Яку роль відіграють макрофаги в кооперації імунокомпетентних клітин на першому етапі формування імунної відповіді?

Активують NK-клітини

Забезпечують процесинг і презентацію антигену Т-хелперам

Активують Т-кілери

Забезпечують процесинг і презентацію антигену Т-кілерам

Продукують імуноглобуліни

4223 / 4299
В експерименті досліджували поріг подразнення тактильних рецепторів різними подразниками. Для якого з наведених подразників поріг буде найменшим?

Світловий

Холодовий

Тепловий

Хімічний

Механічний

4224 / 4299
Під час дослідження гістопрепаратів сполучної тканини були виявлені нейтрофіли. Яку функцію виконують ці клітини, коли мігрують із крові в тканини?

Розширюють кровоносні судини

Опорну функцію

Регулюють скорочення гладеньких міоцитів

Трофічну функцію

Фагоцитоз мікроорганізмів

4225 / 4299
Під час мітозу в клітинах спостерігається порушення розходження хромосом до полюсів унаслідок розпаду мікротрубочок, які містяться у складі центріолей. З якого білка побудовані мікротрубочки центріолей?

Тубулін

Віментин

Міозин

Динеїн

Актин

4226 / 4299
Пацієнтка віком 48 років, що тривало та безконтрольно приймала натрію бромід, скаржиться на загальмованість, порушення пам'яті, нежить і кашель. Яка причина цього стану у пацієнтки?

Кумуляція

Лікарська залежність

Ідіосинкразія

Тахіфілаксія

Звикання

4227 / 4299
В ході розтину тіла 4-річної дівчинки, яка тривалий час страждала і померла від зливної пневмонії, виявлено, що маса тимуса становить 2 грами. Під час гістологічного дослідження тимуса виявлено різкий спад лімфоцитів, колапс строми залози і нечисленні обвапнені кістоподібно розширені тільця Гассаля. Який патологічний процес розвинувся в тимусі?

Дисплазія тимуса

Гіперплазія тимуса

Тимомегалія

Атрофія тимуса

4228 / 4299
Хворому на гінгівіт призначили зрошування ротової порожнини розчином перекису водню. До якої групи антисептиків належить цей препарат?

Барвники

Окисники

Детергенти

Спирти

Нітрофурани

4229 / 4299
У пацієнта за два роки після видалення зуба спостерігається зменшення в обсязі зубної лунки. Про який патологічний процес це свідчить?

Нейротична атрофія

Атрофія від дії фізичних факторів

Дисфункціональна атрофія

Атрофія від тиску

Атрофія, викликана недостатністю кровообігу

4230 / 4299
Деякі захворювання тонкої кишки пов’язані з порушенням функції екзокриноцитів з ацидофільними гранулами (клітини Панета). Де розташовані ці клітини?

На дні кишкових крипт

На апікальній частині кишкових ворсинок

У місці переходу ворсинок в крипти

На бокових поверхнях кишкових ворсинок

У верхній частині кишкових крипт

4231 / 4299
У жінки виникла загроза передчасного переривання вагітності. З недостатністю якого гормону це пов'язано?

Альдостерону

Тестостерону

Окситоцину

Естрадіолу

Прогестерону

4232 / 4299
В умовах експерименту у тварини вимірювали залежність артеріального тиску від величини судинного опору. Укажіть судини, в яких цей опір найбільший:

Капіляри

Артерії

Вени

Артеріоли

Аорта

4233 / 4299
Пацієнт із діагнозом гострий пульпіт, скаржиться на болючість зуба і набряк нижньої половини обличчя на стороні ураженого зуба. Який механізм розвитку набряку є провідним під час цього захворювання?

Порушення трофічної функції нервової системи

Порушення нервової регуляції водного обміну

Порушення мікроциркуляції в осередку ураження

Підвищення продукції альдостерону

Гіпопротеїнемія

4234 / 4299
У пульпі зуба внаслідок дегенеративних змін утворилися мінералізовані осередки, деякі з них містять канальці. Назвіть ці утворення:

Фіброзні тільця

Цемент

Ділянки осифікації

Кісткова тканина

Дентиклі

4235 / 4299
Обстеження головного мозку за допомогою ЯМР показало, що гематома у хворого знаходиться в коліні внутрішньої капсули. Який провідний шлях пошкоджено у хворого?

Тr. cortico-thalamicus

Tr. thalamo-corticalis

Tr. cortico-spinalis

Tr. cortico-fronto-pontinus

Tr. cortico-nuclearis

4236 / 4299
Після видалення премоляра на верхній щелепі у пацієнта спостерігає-ться кровотеча з альвеолярної лунки. Який засіб доцільно застосувати для зупинки кровотечі?

Гепарин

Амінокапронова кислота

Тромбін місцево

Вікасол

Неодикумарин

4237 / 4299
Під час огляду порожнини рота на губній і язиковій поверхні зубів виявлені темно-жовті і коричневі плями і смуги, що займають більше половини поверхні зубів; емаль і дентин зруйновані. Який найбільш імовірний діагноз?

Дистрофічне звапнування

Флюороз

Клиноподібні дефекти зубів

Зубний камінь

Метастатичне звапнування

4238 / 4299
Внаслідок хондродисплазії (аномалія розвитку хряща) пошкоджено волокнистий хрящ. Де можливо спостерігати патологічні зміни?

У вушній мушлі

У гортані

У бронхах

У міжхребцевих дисках

У трахеї

4239 / 4299
У пацієнта спостерігається порушення кровопостачання медіальної поверхні правої півкулі головного мозку. Яка артерія пошкоджена?

A. cerebri posterior

A. chorioidea

A. cerebri media

A. cerebri anterior

A. communicans posterior

4240 / 4299
У препараті представлена кровоносна судина. Внутрішня оболонка представлена ендотелієм і субендотелієм, середня -- пучками гладеньких міоцитів, прошарками пухкої волокнистої сполучної тканини. Зовнішня оболонка сильно розвинена, утворена пухкою сполучною тканиною і окремими гладенькими міоцитами. Яка судина має таку морфологічну характеристику?

Артерія еластичного типу

Артерія змішаного типу

Артерія м'язового типу

Вена безм'язового типу

Вена м'язового типу

4241 / 4299
Під час обстеження дитини виявлено щільні безболісні вузлики у шкірі потиличної області голови розмірами 5 - 7 мм. Аналогічні утворення виявлено навколо колінних суглобів та вздовж сухожиль нижніх кінцівок. Патологогістологічне заключення біоптату: макрофагальна гранульома. Клінічний діагноз - ревматизм. Укажіть клініко-морфологічну форму ревматизму.

Церебральна

Поліартритична

Кардіоваскулярна

Нодозна еритема

М'язовий ревматизм

4242 / 4299
Пацієнту віком 35 років, який скаржиться на різкий біль в епігастрії натщесерце та печію, лікар призначив препарат групи Н_2-гістаміноблокаторів. Який це препарат?

Алмагель

Метацин

Атропін

Вікалін

Ранітидин

4243 / 4299
У хворого на цукровий діабет з'явився біль у правій нозі, тканини першого пальця стали набряклими, чорного кольору, епідерміс відшарувався, з'явилися виділення з неприємним запахом. Визначте патологічний процес:

Секвестр

Інфаркт

Суха гангрена

Коагуляційний некроз

Волога гангрена

4244 / 4299
Під час аутопсії тіла чоловіка у верхній долі правої легені виявлено крупний клиноподібний осередок темно-червоної щільної тканини. Під час гістологічного дослідження виявлено некроз стінок альвеол, просвіт альвеол щільно заповнений еритроцитами. Який процес розвинувся в легенях?

Ателектаз легень

Карніфікація легень

Гангрена легень

Крововилив у легені

Геморагічний інфаркт легень

4245 / 4299
Відбулося пошкодження структурного гена - ділянки молекули ДНК. Але це не призвело до заміни амінокислот у білку, тому що через деякий час пошкодження було ліквідовано. Це прояв такої властивості ДНК, як здатність до:

Мутації

Репарації

Зворотної транскрипції

Транскрипції

Реплікації

4246 / 4299
У обстежуваного в ІІ міжребер'ї по парастернальній лінії справа при аускультації краще прослуховується ІІ тон, ніж І. Закриттям якого клапану зумовлено формування ІІ тону?

Двостулковий та трьохстулковий клапани

Правий трьохстулковий клапан

Півмісяцевий клапан аорти

Півмісяцевий клапан легеневого стовбура

Лівий двостулковий клапан

4247 / 4299
Жінка з групою крові АВ(0) Rh(-) вагітна вдруге резус-позитивним плодом. Після перших пологів резус-позитивним плодом імунопрофілактика не проводилася. Яке ускладнення може виникнути під час другої вагітності?

Маткова кровотеча

Внутрішньосудинний гемоліз еритроцитів плода

Внутрішньосудинний гемоліз еритроцитів матері

Передчасні пологи

4248 / 4299
Біохімічний аналіз амінокислотного складу щойно синтезованих поліпептидів показав, що в процесі трансляції перша амінокислота в кожному білку одна і та ж. Назвіть її:

Метіонін

Ізолейцин

Серин

Фенілаланін

Гістидин

4249 / 4299
Хворий звернувся до лікаря зі скаргами на біль у зубі. Під час обстеження виявлена каріозна порожнина, яка досягає пульпи. Яка стадія карієсу розвинулася у хворого?

Середній карієс

Поверхневий карієс

Глибокий карієс

Крейдяна пляма

4250 / 4299
У хворого на 2-гу добу після розвитку інфаркту міокарда відбулося різке падіння систолічного АТ до 60 мм рт.ст. з тахікардією 140/хв., задишкою, непритомністю. Який механізм має вирішальне значення у патогенезі шоку, що розвинувся?

Зменшення хвилинного об'єму крові

Підвищення збудливості міокарда продуктами некротичного розпаду

Зниження об'єму циркулюючої крові

Розвиток пароксизмальної тахікардії

4251 / 4299
У пацієнта з відкритою раною обличчя з підритими краями спостерігається некроз тканин із поступовим частковим гангренозним процесом, що майже доходить до кісткової тканини. У рані під час детального обстеження виявлено живі личинки. Пацієнту виставлено діагноз - тканинний міаз. Личинки яких двокрилих викликали це захворювання?

Wohlfahrtia magnifica

Glossina palpalis

Stomoxys calcitrans

Musca domestica

4252 / 4299
Під час проведення бактеріоскопії мазка з уретри у пацієнта виявлено гонорею. Враховуючи, що препаратами вибору для лікування гонореї є фторхінолони, який лікарський засіб із нижченаведених треба призначити пацієнту?

Ципрофлоксацин

Фторурацил

Уросульфан

Фуразолідон

4253 / 4299
Формування великої кількості імуноглобулінів з різною антигенною специфічністю з невеликої кількості генів відбувається внаслідок:

Делеції

Рекомбінації генів

Транскрипції

Реплікації

4254 / 4299
На місці рани, яка загоїлася, утворився рубець зі сполучної тканини. Яка речовина є основним компонентом цього різновиду сполучної тканини?

Кератансульфат

Колаген

Гіалуронова кислота

Еластин

4255 / 4299
Пацієнт із сильним зубним болем впродовж декількох днів не звертався до лікаря і займався самолікуванням. В результаті цього виникла необхідність екстракції зуба. Застосування якого анальгетика збільшує імовірність розвитку кровотечі після екстракції зуба?

Кодеїну фосфату

Ацетилсаліцилової кислоти

Анальгіну

Димедролу

Парацетамолу

4256 / 4299
Під час експерименту вимірювалася величина кровотоку (мл/хв) у різних органах та тканинах. Який із нижченаведених органів має найбільшу величину кровотоку на 100 г маси?

Гладенькі м'язи

Скелетні м'язи

Шлунок

Щитоподібна залоза

4257 / 4299
При аналізі крові лікар-лаборант зробив додатковий висновок, що кров належить людині жіночої статі. Особливості будови якого форменого елементу дають можливість зробити цей висновок?

Нейтрофіли

Еритроцити

Базофіли

Лімфоцити

4258 / 4299
Під час вивчення клітин підшлункової залози на субклітинному рівні виявлено порушення функцій концентрації, зневоднення та ущільнення продуктів внутрішньоклітинної секреції. Яка органела відповідальна за вищенаведені процеси?

Лізосома

Комплекс Гольджі

Ендоплазматичний ретикулум

Мітохондрія

4259 / 4299
До основних способів підвищення резистентності емалі відноситься фторування. З якими процесами пов'язаний механізм протикарієсної дії фтору?

Синтезом фторапатиту

Синтезом хлорапатиту

Демінералізацією зуба

Синтезом органічного матриксу зуба

4260 / 4299
У крові пацієнта за 10 тижнів після жовтяниці виявлено HBsAg. Для якого виду вірусного гепатиту це характерно?

D

А

В

Е

4261 / 4299
На рентгенограмі діагностовано перелом кістки черепа, лінія перелому проходить через надочноямковий край. Яка кістка пошкоджена?

Тім'яна

Лобова

Верхня щелепа

Скронева

4262 / 4299
У дитини травмована нижня губа. Який м'яз при цьому ушкоджений?

M. orbicularis oris

M. risorius

M. buccinator

M. levator anguli oris

4263 / 4299
Після травми обличчя у хворого гематома щічної ділянки. Відтік з якої слинної залози блоковано гематомою?

Під'язикова

Піднижньощелепна

Привушна

Щічна

4264 / 4299
У пацієнта 25-ти років спостерігається виражена м'язова слабкість. Вміст яких електролітів в плазмі крові доцільно визначити в першу чергу?

Іонів калію

Іонів магнію

Іонів кальцію

Іонів натрію

4265 / 4299
У пацієнта з хронічним мієлолейкозом, виникли ознаки виразково-некротичного стоматиту. Під час біопсії слизової оболонки виявлено лейкозні клітини. Із якою ланкою патогенезу пухлини пов'язане ураження ротової порожнини?

Ініціація

Промоція

Епігеномний механізм трансформації

Пухлинна прогресія

Мутаційний механізм трансформації

4266 / 4299
У деяких клітинах дорослої людини протягом життя не спостерігається мітоз і кількісний вміст ДНК залишається постійним. Укажіть ці клітини.

Епідерміс

Ендотелій

Нейрони

М'язові (гладкі)

Кровотворні

4267 / 4299
У дитини діагностовано глистну інвазію. Яких змін лейкоцитарної формули слід при цьому очікувати?

Збiльшення кiлькостi моноцитiв

Збiльшення кiлькостi лiмфоцитiв

Збільшення кількості еозинофілів

Збільшення кількості еритроцитів

4268 / 4299
Скільки сегментів у шийному відділі спинного мозку?

2

8

12

7

5

4269 / 4299
У пацієнта пухлина лівої половини довгастого мозку. Під час обстеженя виявлено, що м'яке піднебіння на стороні ураження провисає, глотковий рефлекс знижений, язичок відхиляється у здоровий бік під час вимови звука 'а', також спостерігається осиплість голосу. Функція яких нервів імовірно порушиться через пухлину?

Додаткового та під'язикового

Блукаючого та додаткового

Язикоглоткового та додаткового

Язикоглоткового та блукаючого

4270 / 4299
До лікаря-невролога звернулася пацієнтка віком 52 роки зі скаргами на втрату чутливості шкіри правої половини обличчя в ділянці нижньої повіки, спинки носа та верхньої губи. Гілка якого нерва ушкоджена у цієї пацієнтки?

Великий кам’янистий нерв лицевого нерва

Нижньощелепний нерв трійчастого нерва

Барабанна струна лицевого нерва

Верхньощелепний нерв трійчастого нерва

4271 / 4299
В хірургічний кабінет звернулась людина, яку покусав невідомий собака. Широкі рвані рани локалізовані на обличчі. Яку лікувально-профілактичну допомогу потрібно надати для профілактики сказу?

Розпочати імунізацію антирабічною вакциною

Терміново ввести вакцину АКДП

Госпіталізувати хворого і тримати під наглядом лікаря

Призначити комбіновану антибіотикотерапію

4272 / 4299
У дитини вроджений імунодефіцит. Уражений клітинний імунітет, що обумовлює часті вірусні інфекції. Порушеннями в якому органі це, найімовірніше, викликано?

Селезінці

Тимусі

Червоному кістковому мозку

Піднебінних мигдаликах

4273 / 4299
На основі лабораторного аналізу у хворого підтверджено діагноз подагра. Який аналіз був проведений для постановки діагнозу?

Визначення креатиніну в сечі

Визначення залишкового азоту в крові

Визначення сечової кислоти в крові та сечі

Визначення аміаку в сечі

4274 / 4299
В плазмі крові здорової людини містяться декілька десятків білків. При захворюванні організму з'являються нові білки, зокрема ''білок гострої фази''. Таким білком є:

Імуноглобулін G

Імуноглобулін А

С-реактивний білок

Фібриноген

4275 / 4299
У соматичних клітинах абортивного плода людини виявлено делецію короткого плеча 5-ї хромосоми. Вкажіть число аутосом у каріотипі цього організму:

46

45

44

47

4276 / 4299
Назвіть гормон, що має виражений протизапальний, антиалергічний та імунодепресивний ефект:

Адреналін

Соматотропін

Гідрокортизон

Тироксин

Альдостерон

4277 / 4299
Хворому перед операцією на щелепно-лицевій ділянці ввели препарат, що є природним протизгортальним фактором, впливає безпосередньо на фактори згортання крові, у разі внутрішньовенного введення дає швидкий ефект. У стоматологічній практиці використовується для профілактики тромбоемболічних ускладнень під час великих операцій на щелепно-лицевій ділянці. Визначте цей препарат:

Кислота амінокапронова

Гепарин

Фенілін

Контрикал

Неодикумарин

4278 / 4299
У приймальне відділення надійшов хворий з ознаками гострої серцевої недостатності: блідістю, акроціанозом, частим поверхневим диханням. Який з перерахованих засобів показаний в цьому випадку?

Адреналіну гідрохлорид

Корглікон

Нітрогліцерин

Дигітоксин

Кордіамін

4279 / 4299
У препараті щитоподібної залози при обробці солями срібла видно великі аргірофільні клітини, які розташовані в стінці фолікулів. Який гормон синтезується даними клітинам?

Паратирин

Тироксин

Адреналін

Кальцитонін

4280 / 4299
У дитини спостерігається гепатомегалія, гіпоглікемія, судоми, особливо натщесерце та при стресових ситуаціях. Діагноз: хвороба Гірке. Генетичний дефект якого фермента має місце при даній хворобі?

Фосфоглюкомутаза

Глікогенфосфорилаза

Глюкозо-6-фосфатаза

Глюкокіназа

4281 / 4299
Під час дослідження функціонального стану нирок застосовують навантажувальну пробу з парааміногіпуровою кислотою (ПАГ). Який механізм сечоутворення досліджують таким чином?

Систему фільтрації

Систему концентрації

Систему реабсорбції

Систему секреції

Поворотно-противопотокову систему

4282 / 4299
У пацієнта віком 59 років діагностована хорея, яка проявляється мимовільними швидкими рухами, що супроводжуються гримасами. Із ушкодженням якої структури мозку пов’язують виникнення хореї?

Смугастого тіла (стріатум)

Таламуса

Мигдалеподібного тіла (мигдалина)

Ядра Даркшевича

Огорожі

4283 / 4299
В клітинах сполучної тканини утворюються ферменти та інші активні речовини, які регулюють її щільність і проникність. Який ферментний препарат використовується з метою розпушення і підвищення проникності сполучнотканинних утворень?

Лідаза

Ліпаза

Холінестераза

Кокарбоксілаза

4284 / 4299
На рентгенограмі діагностовано перелом кісток черепа. Лінія перелому проходить через верхню каркову лінію. Яка кістка пошкоджена?

Скронева

Тім'яна

Піднебінна

Потилична

4285 / 4299
Для моделювання виразки шлунка тварині введено в гастральні артерії атофан, який спричинює їх склерозування. Який механізм пошкодження слизової оболонки шлунка є провідним у цьому експерименті?

Гіпоксичний

Нейродистрофічний

Дисрегуляторний

Нейрогуморальний

4286 / 4299
У трирічної дівчинки краснуха. Її десятирічна сестричка не заразилась цією інфекційною хворобою, хоча весь час контактувала з нею. Педіатр з'ясував, що вона хворіла на краснуху п'ять років тому. Який вид імунітету лежить в основі захисту старшої сестри?

Штучний активний

Природний активний

Природний пасивний

Видовий

Штучний пасивний

4287 / 4299
У хворого з крововиливом у передній гіпоталамус виникла поліурія. Недостатність якого гормону вплинула на зменшення реабсорбції води в канальцях нирок?

Вазопресину

Кальцитоніну

Альдостерону

Окситоцину

4288 / 4299
У хворої діагностували виразку шлунка. З анамнезу відомо, що вона тривалий час хворіє на ревматоїдний артрит. Прийом яких препаратів найбільш імовірно призвів до розвитку даного захворювання?

Глюкокортикоїди

Антибіотики

Антигістамінні препарати

Антигіпертензивні препарати

4289 / 4299
У чоловіка 50 років, який хворіє на фіброз легень, спостерігається недостатність дихання. Який механізм є основною причиною цього явища?

Порушення функції дихального центру

Порушення дифузії газів у легенях

Обструктивна недостатність

Зменшення загального току крові в легенях

Рестриктивна недостатність

4290 / 4299
Хворий не може підняти опущену нижню щелепу. Які м'язи голови textbfНЕ МОЖУТЬ виконати свою функцію?

Великі виличні

Малі виличні

Жувальні

Щічні

4291 / 4299
Під час розтину у верхній долі правої легені виявлено великий клиноподібний осередок темно-червоної щільної тканини. Під час гістологічного дослідження в ній виявлено некроз стінок альвеол, просвіт альвеол щільно заповнений еритроцитами. Який процес розвинувся в легенях?

Крововилив

Геморагічний інфаркт

Ателектаз

Карніфікація

4292 / 4299
Кофакторами ферментів є різні похідні водорозчинних вітамінів. Який із них входить до складу амінотрансфераз?

В1

РР

В3

В6

В2

4293 / 4299
До клініки госпіталізовано пацієнта, який має сильний головний біль, ригідність м'язів потилиці, повторне блювання, біль під час перкусії черепа, підвищену чутливість до світлових подразників. Установлено попередній діагноз: менінгіт. Для підтвердження діагнозу призначено проведення спинномозкової пункції. Визначте анатомічне місце її проведення.

Між 12 грудним і 1 поперековим хребцями

Між 5 поперековим хребцем і основою крижів

Між 3 і 4 поперековими хребцями

Між 11 і 12 грудними хребцями

4294 / 4299
У хворого після черепно-мозкової травми, під час якої була ушкоджена мозочкова ділянка, розвинулись порушення часової і просторової координації рухів. Яка патологія розвинулась у хворого?

Парез

Абазія

Атаксія

Астазія

4295 / 4299
До відділення реанімації поступив хворий з гострим отруєнням невідомим лікарським засобом. Для швидкого виведення отрути з організму провели форсований діурез. Який з перерахованих засобів використали для цієї процедури?

Омепразол

Спіронолактон

Дитилін

Гідрохлортіазид

Фуросемід

4296 / 4299
Хворий 45-ти років, що переніс лівосторонню крупозну пневмонію, загинув від множинних травм в результаті автотранспортної катастрофи. На розтині нижня частка лівої легені в області задньо-бокової стінки зрощена із грудною стінкою фіброзними спайками. Об'єм частки зменшений, вона щільна, на розрізі м'ясистого вигляду, сірувато-рожевого кольору, її шматочки тонуть у воді. При гістологічному дослідженні в цих ділянках відмічається дифузне розростання волокнистої сполучної тканини. Ускладнення крупозної пневмонії:

Карніфікація

Ателектаз

Емфізема

Гангрена

4297 / 4299
Під час обстеження у пацієнта вияв-лено каріозну порожнину на жувальній поверхні першого моляра верхньої щелепи зліва. Порожнина має форму конуса, який верхівкою повернений до кореня зуба. Між порожниною та пульпою є зони розм'якшеного, прозорого та замісного дентину. Яка стадія карієсу виявлена у пацієнта?

Початковий карієс

Хронічний поверхневий карієс

Гострий середній карієс

Гострий глибокий карієс

Гострий поверхневий карієс

4298 / 4299
У людини спостерігається дефіцит вітаміну D, що призводить до недостатнього всмоктування:

Хлору

Кальцію

Заліза

Натрію

4299 / 4299
У недоношеної дитини виявлено злипання стінок альвеол через відсутність сурфактанту. Укажіть, порушення функції яких клітин стінки альвеоли обумовлює такий стан.

Секреторних клітин Клара

Фібробластів

Альвеолоцитів ІІ типу

Альвеолярних макрофагів